Sei sulla pagina 1di 767

2012 PREP SA ON CD-ROM

Question 1
You are caring for a 2-month-old infant who has hypoplastic left heart syndrome. He weighed
2,800 g at term birth. The baby has undergone the first stage of the multistage corrective surgical
procedure. His medications include digoxin and furosemide to ameliorate congestive heart failure. He
visited his cardiologist last week, where his weight was 3,250 g, and no changes in medication dosages
were recommended. At the time of his office visit today, his parents report that the baby is taking 60 mL of
a 24 kcal/30 mL cow milk protein-based formula every 4 hours, but he seems to tire easily. He spits up
formula once or twice after each feeding. His weight today is 3,300 g.
Of the following, the MOST appropriate recommendation for feeding this infant is
A. an every-2-hours feeding schedule
B. an amino acid-based formula
C. formula thickened with rice cereal
D. medium-chain triglyceride supplements
E. nasogastric feedings

Copyright 2012 American Academy of Pediatrics


2012 PREP SA ON CD-ROM

Critique 1 Preferred Response: E


Disturbances in growth and weight gain are common in infants who have congenital heart
disease and congestive heart failure (CHF). Clearly, the most effective preventive measure for such
nutritional complications is total surgical correction of the cardiac lesion. When that option is not feasible,
adjunctive therapy must be directed at providing sufficient energy intake to assure a relatively normal
growth rate, including incremental calorie supplementation to respond to increased metabolic demand
and to achieve catch-up growth, if necessary. Studies have indicated that infants who have CHF and
growth failure require 140 to 150 kcal/kg per day to achieve these objectives.
The weight gain for the infant described in the vignette has been poor (about 7 g/day in the past
week), and his total daily energy intake is approximately 250 kcal/day or less than 80 kcal/kg per day. The
most effective method of dietary management to achieve the targeted intake needed is nasogastric
feedings. Although various formulas (including amino acid-based preparations) and energy
supplementation (rice cereal, medium-chain triglycerides, glucose polymers) may be used, evidence
suggests that the necessary energy intake goal cannot be achieved with oral feedings, either alone or in
combination with nighttime enteral nutrition. More frequent feedings will not aid in achieving the goal.
Energy requirements are highly variable in healthy children and are influenced by basal
metabolism, growth rate, physical activity, sex, body size, and developmental stage. Numerous
methodologies have been proposed to determine total energy requirements for infants, children, and
adolescents. Several recently developed equations for estimating energy needs from birth through
adolescence are offered by Glassman and Kleinman (see references). However, these guidelines do not
account for special situations in which energy needs may increase because of a chronic illness, such as
cystic fibrosis, or decrease because of reduced activity, such as for the child who has cerebral palsy and
severe motor impairment. As demonstrated by this infant, energy requirements may vary widely from
established norms, not only because of increased energy expenditure related to CHF, but also because
of the need to achieve catch-up growth in infants who have long-standing undernutrition for the
malnourished child. Estimates of energy intake necessary to achieve catch-up growth alone may be
calculated as follows, using the National Center for Health Statistics weight for height data:

kcal/kg/day = RDA for weight age (kcal/kg) x target weight for height
actual weight

where the weight age represents the age at which the childs current weight would be at the 50th
percentile and the target weight for height is the median weight for the patients height. In many cases,
catch-up growth alone demands an energy intake totaling 120% to 125% of the age-related
recommended daily allowance (RDA). Increased needs related to clinical disease states must be added
to this estimate.
Enteral nutrition may be defined as the provision of liquid nutrition that involves complex, partially
hydrolyzed, or elemental diets, generally via a nasally or percutaneously placed feeding tube. Enteral
feedings are an essential component of care for patients who are unable to satisfy their nutritional
requirements through regular oral feedings and may be warranted under the following clinical conditions:
Increased energy expenditure (hypermetabolism)
Oral-motor dysfunction
Esophageal and gastric dysmotility
Compromised intestinal function (maldigestion, malabsorption)
Neurologic impairment
As shown in Item C1, enteral feedings are employed in a wide variety of clinical disease states to
achieve targeted nutrient intake. For conditions in which oral feedings cannot maintain nutritional
adequacy, enteral alimentation should be considered as either supportive or primary therapy for patients
retaining either partial or complete gastrointestinal function.

American Board of Pediatrics Content Specification(s):


Know the caloric requirements for infants, children, and adolescents
Understand the indications for providing enteral nutritional support

Copyright 2012 American Academy of Pediatrics


2012 PREP SA ON CD-ROM

Suggested Reading:
Glassman MS, Woolf PK, Schwarz SM. Nutritional considerations in children with congenital heart
disease. In: Baker SB, Baker RD, Davis A, eds. Pediatric Enteral Nutrition. New York, NY: Chapman &
Hall; 1994: 340-350

Kleinman RE. Cardiac disease. In: Pediatric Nutrition Handbook. 6th ed. Elk Grove Village, IL: American
Academy of Pediatrics; 2009:981-1000

Leitch CA. Growth, nutrition and energy expenditure in pediatric heart failure. Progr Pediatr Cardiol.
2000;11:195-202. DOI: 10.1016/S1058-9813(00)00050-3. Abstract available at:
http://www.ncbi.nlm.nih.gov/pubmed/10978712

Schwarz SM, Gewitz MH, See CC, et al. Enteral nutrition in infants with congenital heart disease and
growth failure. Pediatrics. 1990;86:368-373. Available at:
http://pediatrics.aappublications.org/cgi/reprint/86/3/368

Schwarz SM. Feeding disorders in children with developmental disabilities. Infants & Young Children.
2003;16:317-330. Abstract available at:
http://journals.lww.com/iycjournal/Abstract/2003/10000/Feeding_Disorders_in_Children_With_Developme
ntal.5.aspx

Serrano M-S, Mannick EM. Consultation with the specialist: enteral nutrition. Pedatr Rev. 2003;24:417-
423. DOI: 10.1542/pir.24-12-417. Available at:
http://pedsinreview.aappublications.org/cgi/content/full/24/12/417

Copyright 2012 American Academy of Pediatrics


2012 PREP SA ON CD-ROM

Critique 1

Item C1. Conditions in Which Enteral Feedings May Be Indicated


! Prematurity ! Protracted diarrhea of infancy
! Congestive heart failure ! Burn injury
! Chronic pulmonary disease ! Severe head trauma
! Cystic fibrosis ! Cancer
! Short bowel syndrome ! Cerebral palsy
! Inflammatory bowel disease ! Renal disease
! Dysphagia ! Chronic liver disease
! Gastroesophageal reflux ! Inadequate spontaneous oral intake
Adapted from Kleinman RE, ed. Pediatric Nutrition Handbook. 6th ed. Elk Grove
Village, IL: American Academy of Pediatrics; 2009

Copyright 2011 American Academy of Pediatrics


2012 PREP SA ON CD-ROM

Question 2
You are examining a term infant on the second day after birth in the well-baby nursery. Her
mother is 29 years old and has a remote history of heroin abuse. She had been in a methadone
maintenance program throughout the pregnancy, taking 50 mg daily. On physical examination, the infant
has mild tremors when disturbed, a normal Moro reflex, and normal muscle tone. Her heart rate is 130
beats/min, respiratory rate is 60 breaths/min, and temperature is 37.4C. The nurse describes the infant
as being slightly irritable, with poor bottle feeding and several intervals of sneezing overnight but no
vomiting or loose stools. Neonatal abstinence scores using the Finnegan scoring system ranged from 5 to
7 over the past 24 hours.
Of the following, the MOST appropriate treatment at this time for this infant is
A. diazepam
B. morphine
C. paregoric
D. phenobarbital
E. swaddling

Copyright 2012 American Academy of Pediatrics


2012 PREP SA ON CD-ROM

Critique 2 Preferred Response: E


The infant described in the vignette is exhibiting mild signs of withdrawal from methadone and is
best managed using comfort measures such as swaddling. Methadone is the standard treatment for
opiate addiction in pregnancy. It supports a steady-state serum concentration in the mother, which
prevents the fetus from experiencing repeated episodes of withdrawal in utero. However, 48% to 94% of
infants exposed to opiates, including methadone, develop some degree of withdrawal after birth.
Neonatal abstinence syndrome (NAS) is the collection of behavioral and physiologic findings
associated with withdrawal in the newborn. Although opiates are the most common cause of NAS, it may
also be caused by maternal use of stimulants, depressants, and selective serotonin reuptake inhibitors.
Symptoms include tremors, irritability, poor feeding, vomiting, loose stools, sweating, fever, sneezing, and
tachypnea. Left untreated, these clinical findings may escalate in severity and result in generalized
seizures. The onset of symptoms varies with the drug used in pregnancy, with withdrawal from
methadone generally beginning between 2 and 7 days after birth.
The management of at-risk infants is based upon abstinence scoring systems, which include the
Finnegan scoring system. This system uses 20 common signs in neurologic, behaviorial, respiratory, and
gastrointestinal categories. Exposed infants are assessed on a scheduled basis after birth, with elevated
scores triggering treatment interventions.
The infant in the vignette had scores that ranged from 5 to 7. Nonpharmacologic treatment
measures, including swaddling, gentle handling, demand feeding, and a quiet and dark environment
should be initiated when an infant scores 7 or less. If the scores are 8 or greater, pharmacologic
intervention is often indicated. Oral pharmacologic therapies for NAS due to opiates include morphine
sulfate, methadone, and buprenorphine. Phenobarbital and diazepam may be used in cases of maternal
polysubstance abuse or nonopiate-related NAS. Paregoric is not recommended due to its high alcohol
content.

American Board of Pediatrics Content Specification(s):


Know the management of neonatal abstinence syndrome

Suggested Reading:
American Academy of Pediatrics Committee on Drugs. Neonatal drug withdrawal. Pediatrics.
1998;101:1079-1088. Available at: http://pediatrics.aappublications.org/cgi/content/full/101/6/1079

Burgos AE, Burke BL Jr. Neonatal abstinence syndrome. NeoReviews. 2009;10:e222-e229. DOI:
10.1542/neo.10-5-e222. Available at: http://neoreviews.aappublications.org/cgi/content/full/10/5/e222

Osborn DA, Jeffrey HE, Cole MJ. Opiate treatment for opiate withdrawal in newborn infants. Cochrane
Database Syst Rev. 2010;10:CD002059. DOI: 10.1002/14651858.CD002059.pub3. Available at:
http://onlinelibrary.wiley.com/o/cochrane/clsysrev/articles/CD002059/frame.html

Seligman NS, Almario CV, Hayes EJ, Dysart KC, Berghella V, Baxter JK. Relationship between maternal
methadone dose at delivery and neonatal abstinence syndrome. J Pediatr. 2010;157:428-433. DOI:
10.1016/j.jpeds.2010.03.033. Available at: http://www.ncbi.nlm.nih.gov/pubmed/20472252

Copyright 2012 American Academy of Pediatrics


2012 PREP SA ON CD-ROM

Question 3
A mother brings her 7-year-old son to your office because he is having pain from a sunburn. As
part of his care, you discuss sun protection with the boy and his mother.
Of the following, the MOST important information to provide is that
A. sun exposure in childhood sufficient to cause sunburn increases the risk of developing
melanoma
B. sunscreen is less important on cloudy than on sunny days
C. sunscreen should not be used on babies younger than 6 months of age
D. sunscreen with a minimum SPF rating of 45 is recommended for children
E. ultraviolet B (UVB) protection is the only value to consider when choosing sunscreen

Copyright 2012 American Academy of Pediatrics


2012 PREP SA ON CD-ROM

Critique 3 Preferred Response: A


Skin cancers are among the most common neoplastic conditions in developed countries, and the
incidence in both adults and children is increasing. Among adults in the United States, approximately 1
million cases of skin cancer occur per year, of which 80% are basal cell, 16% squamous cell, and 4%
melanoma. Between 1992 and 2006, a 76.9% increase in non-melanoma skin cancers was reported
among Medicare beneficiaries, and melanoma rates per 100,000 American adults more than doubled
between 1973 and 2000. Melanoma incidence among United States children has also been increasing,
although it remains a rare disease.
Risk factors for adult skin cancers include family history of skin cancer; age; propensity to freckle;
and light skin, hair, and eye color. Most affected persons are white, although more than 15% of young
children may be of another race or ethnicity. The most recognized preventable environmental risk factor is
excessive ultraviolet light exposure. Because up to 80% of lifetime sun exposure is estimated to occur
before age 18 years, efforts to prevent skin cancer should begin during childhood with prevention of
excessive sunlight exposure. Melanoma among children, particularly those younger than 10 years of age,
appears to be biologically distinct from adult melanoma, and ultraviolet exposure may be less of a
stimulus than in adults. However, nearly 75% of pediatric melanomas occur in teens 15 to 19 years old,
and it is unclear what role sun exposure plays in this older age group.
Sunlight is composed of ultraviolet B (UVB) (290 to 320 nm wavelength) and UVA (320 to 400 nm
wavelength) radiation as well as UVC. UVA causes darkening of existing skin melanin, resulting in an
immediate tan. It is also absorbed in the dermis, resulting in long-term damage to blood vessels and deep
structures, and it can ultimately lead to skin aging and loss of elasticity. UVB affects the epidermis,
causes development of increased melanin, and is primarily responsible for causing sunburn. Its effects
are seen 6 to 12 hours after exposure, peaking at 24 hours. UVB exposure is most related to
development of skin cancer, with even one blistering sunburn during childhood increasing a persons risk
of developing melanoma as an adult. Accordingly, the mother in the vignette should be told that sun
exposure sufficient to cause sunburn increases the risk of developing melanoma in her son.
Sun protection begins with avoidance of prolonged unprotected exposure, particularly during the
middle of the day, when UV radiation is highest (10 AM to 4 PM). Cloud cover does not provide protection
against UV exposure. If infants and young children are going to be outside for a period of time, protective
clothing and hats are important for sunburn prevention. For older children and adolescents, chemical
sunscreens are typically the first line of defense. They should be applied frequently (every 2 hours) during
times of exposure and should have an SPF (Sun Protective Factor) level of at least 15 (SPF = minimal
dose of sunlight causing cutaneous erythema with sunscreen use/minimal dose of sunlight causing
cutaneous erythema without sunscreen use). SPF is a measure of UVB protection, and the higher the
number, the greater the defense. To prevent UVA damage, parents should look for broad-spectrum
sunscreens that include both UVA and UVB protection. The UVA star rating ranges from 1 star (lowest
protection) to 4 stars (highest).The most commonly used chemical sunscreens are benzophenones,
cinnamates, sulisobenzone, salicylates, and avobenzone. Sunscreen can be used on infants, even those
younger than 6 months of age, but usually is limited to a small amount on exposed surfaces such as
hands, face, and neck. For children who are sensitive to chemical sunscreens or who will have prolonged
exposure, physical sunscreens (zinc oxide or titanium dioxide) should be used, particularly on areas such
as the nose and ears.

American Board of Pediatrics Content Specification(s):


Know that sun damage to the skin is additive and leads to aging of the skin as well as an increased
incidence of skin cancers
Counsel parents regarding sunscreens and exposure to sun

Suggested Reading:
American Academy of Pediatrics. Sun safety. Healthy Children. 2010. Available at:
http://www.healthychildren.org/English/safety-prevention/at-play/Pages/Sun-Safety.aspx

Copyright 2012 American Academy of Pediatrics


2012 PREP SA ON CD-ROM

Cercato MC, Nagore E, Ramazzotti V, et al. Self and parent-assessed skin cancer risk factors in school-
age children. Prev Med. 2008;47:133-135. DOI: 10.1016/j.ypmed.2008.03.004. Abstract available at:
http://www.ncbi.nlm.nih.gov/pubmed/18420261

Ferrari A, Bono A, Baldi M, et al. Does melanoma behave differently in younger children than in adults? A
retrospective study of 33 cases of childhood melanoma from a single institution. Pediatrics.
2005;115:649-654. DOI: 10.1542/peds.2004-0471. Available at:
http://pediatrics.aappublications.org/cgi/content/full/115/3/649

Lange JR, Palis BE, Chang CD, Soong SJ, Balch CM. Melanoma in children and teenagers: an analysis
of patients from the National Cancer Data Base. J Clin Oncol. 2007;25:1363-1368. DOI:
10.1200/JCO.2006.08.8310. Available at: http://jco.ascopubs.org/content/25/11/1363.long

Morelli JG. The skin: photosensitivity. In: Kliegman RM, Stanton BF, St. Geme JW III, Schor NF, and
Behrman RE, eds. Nelson Textbook of Pediatrics. 19th ed. Philadelphia, PA: Saunders Elsevier;
2011:2254-2259

Rogers HW, Weinstock MA, Harris AR, et al. Incidence estimate of nonmelanoma skin cancer in the
United States, 2006.Arch Dermatol. 2010;146:283-287. Abstract available at:
http://www.ncbi.nlm.nih.gov/pubmed/20231499

Tung R, Vidimos A. Melanoma. In: Carey WD, ed. Cleveland Clinic: Current Clinical Medicine. 2nd ed.
Philadelphia, PA: Saunders Elsevier; 2010:250-258

Copyright 2012 American Academy of Pediatrics


2012 PREP SA ON CD-ROM

Question 4
The intern rotating through the newborn nursery reports on rounds that she ordered a hematocrit
measurement for a term newborn who appeared plethoric on physical examination at 6 hours of age. She
suspects the infant has polycythemia, and the hematocrit is 65% (0.65). When you examine the infant, he
appears to be a bit ruddy. Other findings on the examination are normal, including his vital signs;
feeding and elimination patterns are also normal. You ask the intern what method she used to obtain the
specimen, and she reports that a heelstick specimen was obtained.
Of the following, the MOST appropriate method for confirming the hematocrit in this infant is to
obtain a sample via
A. brachial artery
B. peripheral venipuncture
C. repeat heelstick
D. umbilical artery
E. umbilical vein

Copyright 2012 American Academy of Pediatrics


2012 PREP SA ON CD-ROM

Critique 4 Preferred Response: B


Plethora and polycythemia are common clinical problems in the newborn period. Polycythemia is
defined as a hemoglobin or hematocrit value greater than 2 standard deviations above the mean in a
venous blood sample. For a term infant, this represents a hematocrit greater than 65% (0.65) and
hemoglobin of greater than 22 g/dL (220 g/L). The incidence of polycythemia is estimated to be 1% to 5%
in healthy term newborns. Neonatal polycythemia may be due to twin-twin transfusion or maternal-fetal
hemorrhage and is more common in infants of diabetic mothers or infants who have intrauterine growth
restriction or chromosomal anomalies. The maximum hematocrit is found at 2 hours of age, after a period
of equilibration following delivery.
In the newborn period, hemoglobin and hematocrit measurements usually are drawn by capillary
venous sampling from a heelstick. However, hemoglobin and hematocrit values from capillary samples
may be as much as 15% higher than those from venous samples, particularly if the peripheral blood flow
is diminished due to prematurity, sepsis, congenital heart disease, or other conditions. Therefore, it is
important to obtain a venous sample, either from a peripheral vein or the umbilical vein to confirm the
elevated hematocrit of the infant described in the vignette. Since the infant is not exhibiting clinical
manifestations of polycythemia (eg, poor feeding, respiratory distress, poor perfusion), there is no
indication for cannulation of the umbilical vessels or brachial artery. A repeat heelstick most likely would
yield the same results as the first test.

American Board of Pediatrics Content Specification(s):


Know that there may be a difference between a centrally and a peripherally drawn hematocrit

Suggested Reading:
Brandow AM, Camitta BM. Polycythemia (erythrocytosis). In: Kliegman RM, Stanton BF, St. Geme JW III,
Schor NF, and Behrman RE, eds. Nelson Textbook of Pediatrics. 19th ed. Philadelphia, PA: Saunders
Elsevier; 2011:1683

Kates EM, Kates JS. In brief; anemia and polycythemia in the newborn, Pediatr Rev. 2007;28:33-34. DOI:
10.1542/pir.28-1-33. Available at: http://pedsinreview.aappublications.org/cgi/content/full/28/1/33

Linderkamp O. Blood viscosity of the neonate. NeoReviews. 2004;5:e406-e416. DOI: 10.1542/neo.5-10-


e406. Available at: http://neoreviews.aappublications.org/cgi/content/full/5/10/e406

Copyright 2012 American Academy of Pediatrics


2012 PREP SA ON CD-ROM

Question 5
You are evaluating a 5-year-old boy who is hospitalized in the pediatric intensive care unit with
findings of poor perfusion, renal failure, and respiratory compromise requiring intubation. You note an
abnormality on the cardiac monitor (Item Q5).
Of the following, the MOST likely cause of this patients electrocardiographic findings is
A. hyperkalemia
B. hypernatremia
C. hypocalcemia
D. hypokalemia
E. hyponatremia

Copyright 2012 American Academy of Pediatrics


2012 PREP SA ON CD-ROM

Question 5

(Courtesy of A Friedman)
Electrocardiographic tracing, as described for the boy in the vignette.

Copyright 2012 American Academy of Pediatrics


2012 PREP SA ON CD-ROM

Critique 5 Preferred Response: A


Peaked T waves on rhythm strip or electrocardiography (ECG), as shown for the boy in the
vignette, are indicators of hyperkalemia. Other electrocardiographic findings might include atrioventricular
block, widening of the QRS complex, and degeneration of the ECG complex into a sinusoidal shape.
Elevated potassium concentrations appear to have a direct effect on the potassium channels, increasing
their activity and speeding membrane repolarization. Hyperkalemia causes an overall membrane
depolarization that inactivates many sodium channels. The faster repolarization of the cardiac action
potential causes the tenting of the T waves, and the inactivation of sodium channels causes a sluggish
conduction of the electrical wave around the heart, which leads to widening of the QRS complex. The
serum potassium concentration at which electrocardiographic changes develop is somewhat variable.
Hyperkalemia has many causes (Item C5).
Hypernatremia and hyponatremia are not typically associated with rhythm irregularities or
electrocardiographic disturbances. Hypocalcemia can result in prolongation of the QT interval. This type
of electrical instability puts the patient at high risk for torsades de pointes, a specific type of ventricular
fibrillation. Hypokalemia can cause flattened or inverted T waves, ST depression, and a prolongation of
the QT interval.

American Board of Pediatrics Content Specification(s):


Know the signs of hyperkalemia

Suggested Reading:
Wrenn KD, Slovis CM, Slovis BS. The ability of physicians to predict hyperkalemia from the ECG. Ann
Emerg Med.1991;20:12291232. Available at: http://www.ncbi.nlm.nih.gov/pubmed/1952310

Copyright 2012 American Academy of Pediatrics


2012 PREP SA ON CD-ROM

Critique 5

Item C5. Causes of Hyperkalemia


Ineffective Elimination
! Renal insufficiency
! Medications that interfere with urinary excretion
o Angiotensin-converting enzyme inhibitors and angiotensin receptor blockers
o Potassium-sparing diuretics
o Nonsteroidal anti-inflammatory drugs
o Calcineurin inhibitor immunosuppressants
o Trimethoprim
o Pentamidine
! Mineralocorticoid deficiency or resistance
o Addison disease
o Aldosterone deficiency
o Congenital adrenal hyperplasia
o Type IV renal tubular acidosis
Excessive Release From Cells
! Rhabdomyolysis, burns, or any cause of rapid tissue necrosis, including tumor lysis
syndrome
! Blood transfusion or hemolysis
! Shifts/transport out of cells caused by acidosis, low insulin concentrations, beta-
blocker therapy, digoxin overdose, or the paralyzing agent succinylcholine
Excessive Potassium Administration
! Medication errors
! Toxic ingestion

Copyright 2011 American Academy of Pediatrics


2012 PREP SA ON CD-ROM

Question 6
A term 3,500-g male infant is the admitted to the neonatal intensive care unit after a precipitous
delivery in the emergency department to a gravida 34, para 4, 24-year-old woman who did not seek
prenatal care. On physical examination, the infant has normal vital signs, a head circumference of 35.5
cm, and normal general examination findings. Twelve hours after birth, the nurse notes brief jerking of
one of the infants arms. Thirty minutes later, the other arm jerks and the nurse places a hand on the arm,
noting that the jerking is not suppressible.
Of the following, the MOST likely cause of the jerking is
A. benign neonatal myoclonus
B. jitteriness due to drug withdrawal
C. seizure due to cytomegalovirus infection
D. seizure due to hypocalcemia
E. seizure due to hypoxic-ischemic injury

Copyright 2012 American Academy of Pediatrics


2012 PREP SA ON CD-ROM

Critique 6 Preferred Response: E


The neonate described in the vignette has multiple possible reasons to be twitchy. Quick muscle
jerks have a broad differential diagnosis, but the first and most important treatable condition to identify in
this clinical setting is seizures. Seizures can be reliably diagnosed in childhood by history. In the older
affected child, muscle jerking (the clonic in tonic-clonic) follows a cerebral pattern: synchronous bilateral,
synchronous unilateral, or less commonly, synchronous single-limb jerking that is rhythmic and
nonsuppressible. However, seizures cannot be reliably diagnosed by history in neonates because the
features of seizures can be very subtle. Due to the immaturity of connections within the brain and from
brain to muscle, neonatal seizures do not have the same appearance as seizures in older children. In
neonates who have seizures, clonic movements may be nonsustained, multifocal, and migrating.
A critical feature is whether the clonic jerking can be suppressed by touch. If jerking cannot be
suppressed, there is a high likelihood that the cause is a seizure. Rhythmicity also suggests seizure as
the cause. The repeated episodes during the first 24 hours after birth that involve nonsuppressible motor
jerking reported for the infant in the vignette are most likely seizures. Hypoxic-ischemic injury to the brain
is the most common cause for seizures in the first 24 postnatal hours.
Hypocalcemia can occur in newborns, and a newborn presenting with seizures should undergo
routine laboratory evaluation for metabolic derangements. However, hypocalcemia is less prevalent than
hypoxic-ischemic injury to brain. Jitteriness due to drug withdrawal is an important diagnostic
consideration, but this reaction should be more consistent throughout the day, accompanied by some
behavioral irritability, and suppressible. Neonatal seizures can occur due to withdrawal of the infant from
maternal narcotics, so a toxicology screen should be part of a diagnostic evaluation. Benign neonatal
myoclonus generally occurs during sleep, is suppressible by touch or awakening the infant, and is not
epileptic. These are not considered by neurologists to be a form of epilepsy, and electroencephalography
during these episodes does not show abnormal cortical discharges. Congenital infection with
cytomegalovirus causes cerebral injury and can be associated with seizures, intellectual disability, and
microcephaly. This is much less common than hypoxic-ischemic encephalopathy. In addition, this infants
normal head circumference of 35.5 cm makes this condition less likely.

American Board of Pediatrics Content Specification(s):


Recognize that neonatal seizures secondary to hypoxic-ischemic encephalopathy characteristically
occur within 24 hours of birth

Suggested Reading:
de Vries LS, Jongmans MJ. Long-term outcome after neonatal hypoxic-ischaemic encephalopathy. Arch
Dis Child Fetal Neonatal Ed. 2010;95:F220-F224. Available at:
http://www.ncbi.nlm.nih.gov/pubmed/20444814

Hill A. Neurological problems of the newborn. In: Bradley WG, Daroff RB, Fenichel GM, Jankovic J, eds.
Neurology in Clinical Practice. 5th ed. Philadelphia, PA: Butterworth Heinemann Elsevier, 2008:chapter
84

Rennie J, Boylan G. Treatment of neonatal seizures. Arch Dis Child Fetal Neonatal Ed. 2007;92:F148-
F150. DOI: 10.1136/adc.2004.068551. Available at:
http://www.ncbi.nlm.nih.gov/pmc/articles/PMC2675465/?tool=pubmed

Silverstein FS, Jensen FE. Neonatal seizures. Ann Neurol. 2007;62:112-120. DOI: 10.1002/ana.21167.
Abstract available at: http://www.ncbi.nlm.nih.gov/pubmed/17683087

Zupanc ML. Neonatal seizures. Pediatr Clin North Am. 2004;51:961-978. DOI; 10.1016/j.pcl.2004.03.002.
Abstract available at: http://www.ncbi.nlm.nih.gov/pubmed/15275983

Copyright 2012 American Academy of Pediatrics


2012 PREP SA ON CD-ROM

Question 7
Your local newborn screening laboratory notifies your office that a two-week old infant in your
practice has an elevated phenylalanine level. A review of the infants medical record reveals that the
infant was seen in your office last week for a health supervision visit. The mother stated that the infant
was breastfeeding well. The infants physical examination was normal.
Your next step in managing this patient would be to
A. begin the infant on a low phenylalanine formula
B. order an assay for dihydropteridine reductase
C. order an assay for urine pterins
D. order plasma amino acids
E. switch the infant to soy formula

Copyright 2012 American Academy of Pediatrics


2012 PREP SA ON CD-ROM

Critique 7 Preferred Response: D


Clinicians should be aware of some potential pitfalls and incorrect assumptions involved with
newborn screening. Because miscommunication can and often does occur, results of newborn screening
tests never should be assumed to be normal based on a lack of notification by the state screening
program. Also, because these are only screening tests and false-negative results may occur, a negative
result does not guarantee definitive exclusion of any given condition. For example, a blood sample
obtained from an infant younger than 24 hours of age has a higher false-negative rate when screening for
phenylketonuria (PKU) compared with samples obtained later, especially among children who have less
severe enzyme deficiencies. For this reason, some authorities suggest sending a repeat sample to the
state laboratory for infants who had relatively early (before 24 hours of age) screens. In any case, a
negative or normal screening test result should not dissuade a physician from ordering a diagnostic test
for a specific disorder if there are clinical indicators of a condition, even if that condition is included on
newborn screening in the state. Conversely, clinicians should never assume that an abnormal newborn
screening test result is false-positive just because the newborn is clinically well or not exhibiting signs of
the condition. Abnormal test results may (and ideally should) come back before the onset of symptoms.
Before instituting dietary restriction for the infant described in the vignette, who has a positive
PKU test result on newborn screening assay, it is essential to confirm the diagnosis by immediately
assessing plasma amino acids. In many cases, referral to the local metabolic treatment center allows
rapid testing with appropriate and timely follow-up. Starting dietary treatment with a low-phenylalanine
formula before diagnostic confirmation may confound subsequent diagnostic testing and be possibly
harmful to an infant who does not have PKU. Ideally, treatment should be instituted before 20 days of
age, and the diet should be calculated by a specially trained registered dietitian to ensure that the correct
amount of essential amino acids are provided by combining a special metabolic formula with human milk
or infant formula. Soy formula, which is required for infants who have galactosemia, is not beneficial for
infants who have PKU.
Although classic PKU is caused by deficiency of the enzyme phenylalanine hydroxylase (PAH),
much less common forms of hyperphenylalaninemia are caused by defects in the biosynthesis of
tetrahydrobiopterin, which is a cofactor needed for PAH to function properly. Such variants of
hyperphenylalaninemia should be assessed once the diagnosis of PKU is confirmed because infants who
have these defects require supplemental biopterin and have a less favorable long-term prognosis. Urinary
pterin assays are helpful in identifying and characterizing the biopterin defect in these rare cases.
Dihydropteridine reductase is one of a number of enzymes responsible for tetrahydrobiopterin deficiency,
and an enzyme assay can be performed on blood from the newborn screening filter paper if this form of
hyperphenylalaninemia is suspected.

American Board of Pediatrics Content Specification(s):


Know the utility and limitations of PKU screening

Suggested Reading:
Committee on Genetics. Maternal phenylketonuria. Pediatrics. 2008;122:445-449. DOI:
10.1542/peds.2008-1485. Available at: http://pediatrics.aappublications.org/cgi/content/full/122/2/445

Kaye CI and the Committee on Genetics. Newborn screening fact sheets. Pediatrics. 2006;118:e934-
e963. DOI: 10.1542/peds.2006-1783. Available at:
http://pediatrics.aappublications.org/cgi/content/full/118/3/e934

National Center for Medical Home Implementation. Newborn Screening Overview. Elk Grove Village, IL:
American Academy of Pediatrics. Available at:
http://www.medicalhomeinfo.org/how/clinical_care/newborn_screening.aspx

Copyright 2012 American Academy of Pediatrics


2012 PREP SA ON CD-ROM

Question 8
The mother of one of your adolescent patients asks you about human papillomavirus (HPV)
vaccination. She explains that she has just learned that HPV is the most common sexually-transmitted
infectious agent and that there are vaccines available to prevent infection. In discussing this virus, you
inform her that there are more than 100 strains of the virus but that the vaccines protect against the most
important strains.
Of the following, the MOST accurate specific information about HPV strains is that
A. type 43 causes most recurrent respiratory papillomatosis
B. types 11 and 35 account for most cervical cancer
C. types 16 and 18 account for most cervical cancer
D. types 6 and 54 account for most anogenital warts
E. types 43 and 44 account for most anogenital warts

Copyright 2012 American Academy of Pediatrics


2012 PREP SA ON CD-ROM

Critique 8 Preferred Response: C


Of the more than 100 different types of the human papillomavirus (HPV), about 30 to 40 are
responsible for anogenital infections and 15 to 20 of these are oncogenic (high risk). Persistent infection
with these high-risk types is responsible for almost all cervical precancers and cancers as well as a high
percentage of precancers and cancers at other anogenital sites in both sexes. Of the oncogenic strains,
types 16 and 18 are responsible for two thirds of all cervical cancers worldwide. Each year in the United
States, approximately 12,000 women develop cervical cancer and nearly 4,000 die of it.
Of the nononcogenic (low-risk) HPV types, types 6 and 11 together are responsible for greater
than 90% of anogenital warts and almost 100% of recurrent respiratory papillomatosis, a rare
nonmalignant lesion of the larynx and trachea that has a peak incidence between the ages of 2 and 4
years.
The quadrivalent vaccine contains types 6, 11, 16, and 18. The bivalent vaccine contains types
16 and 18. Types 43, 44, and 54 are low-risk types, and 33 and 35 are high-risk types, but they are not
currently included in the vaccines because they account for few infections.

American Board of Pediatrics Content Specification(s):


Recognize that specific human papillomavirus strains are associated with cervical cancer and others
with genital warts

Suggested Reading:
ACOG Committee on Practice BulletingsGynecology. ACOG practice bulletin. number 109: cervical
cytology screening. Obstet Gynecol. 2009;114:1409-1420. DOI: 10.1097/AOG.0b013e3181c6f8a4

Brigham KS, Goldstein MA. Adolescent immunizations. Pediatr Rev. 2009;30:47-56. DOI: 10.1542/pir.30-
2-47. Available at: http://pedsinreview.aappublications.org/cgi/content/full/30/2/47

Centers for Disease Control and Prevention (CDC). FDA licensure of quadrivalent human papillomavirus
vaccine (HPV4, Gardasil) for use in males and guidance from the Advisory Committee on Immunization
Practices (ACIP). MMWR Morb Mortal Wkly Rep. 2010;59:630-632. Available at:
http://www.cdc.gov/mmwr/preview/mmwrhtml/mm5920a5.htm

Committee on Infectious Diseases. Prevention of human papillomavirus infection: provisional


recommendations for immunization of girls and women with quadrivalent human papillomavirus vaccine.
Pediatrics. 2007;120:666-668. DOI: 10.1542/peds.2007-1735. Available at:
http://pediatrics.aappublications.org/cgi/content/full/120/3/666

Dempsey AF, Gebremariam A, Koutsky L, Manhart L. Behavior in early adolescence and risk of human
papillomavirus infection as a young adult: results from a population-based study. Pediatrics. 2008;122:1-
7. DOI: 10.1542/peds.2007-2515. Available at:
http://pediatrics.aappublications.org/cgi/content/full/122/1/1

Fairley CK, Hocking JS, Gurrin LC, Chen MY, Donovan B, Bradshaw CS. Rapid decline in presentations
of genital warts after the implementation of a national quadrivalent human papillomavirus vaccination
programme for young women. Sex Transm Infect. 2009;85:499-502. DOI: 10.1136/sti.2009.037788.
Available at: http://sti.bmj.com/content/85/7/499.long

Forhan SE, Gottlieb SL, Sternberg MR, et al. Prevalence of sexually transmitted infections among female
adolescents aged 14 to 19 in the United States. Pediatrics. 2009;124:1505-1512. DOI:
10.1542/peds.2009-0674. Available at: http://pediatrics.aappublications.org/cgi/content/full/124/6/1505

Copyright 2012 American Academy of Pediatrics


2012 PREP SA ON CD-ROM

Question 9
You are evaluating an 8-month-old previously healthy infant who has a 3-day history of nonbilious
vomiting, watery diarrhea, and decreased oral intake. He is sitting in his mothers lap and responds
appropriately when you examine him. His heart rate is 120 beats/min, respiratory rate is 30 breaths/min,
and blood pressure is 85/50 mm Hg, and he has palpable peripheral pulses. A capillary blood gas on
room air reveals a pH of 7.22, PaCO2 of 25 mm Hg, and bicarbonate (HCO3) of 10 mEq/L (10 mmol/L).
Initial electrolyte values are:
Sodium, 141 mEq/L (141 mmol/L)
Potassium, 4.0 mEq/L (4.0 mmol/L)
Chloride, 120 mEq/L (120 mmol/L)
Bicarbonate, 11 mEq/L (11 mmol/L)
Glucose, 100 mg/dL (5.6 mmol/L)
Of the following, the MOST likely cause of the infants metabolic acidosis is
A. gastrointestinal loss of bicarbonate
B. hypoaldosteronism
C. new-onset diabetes mellitus
D. renal failure
E. septic shock

Copyright 2012 American Academy of Pediatrics


2012 PREP SA ON CD-ROM

Critique 9 Preferred Response: A


Acid-base disturbances can result from various disease processes and are frequently
encountered in the care of hospitalized pediatric patients. Metabolic acidosis (defined as pH of <7.35)
results from either a net reduction in bicarbonate (HCO3) or an increase in acids related to increased
production, decreased excretion, or increased intake. Diarrhea is the most frequent cause of metabolic
acidosis, as described for the child in the vignette, resulting in gastrointestinal loss of HCO3.
Conditions that cause metabolic acidosis can be divided into those that maintain a normal anion
gap (e.g., loss of bicarbonate hypoaldosteronism, posthypocapnea, hyperalimentation, certain drugs) and
those that cause an increased anion gap (e.g., ketoacidosis, renal failure, lactic acidosis, and certain
toxins). The anion gap is the difference between routinely measured cations and anions in the blood and
+ - -
is estimated by using the following equation: sodium [Na ]-chloride [Cl ]-[HCO3 ]. A normal anion gap is 8
to 16 and reflects unmeasured anions such as proteins, sulfates, phosphates, and organic acids. The
anion gap for the child described in the vignette is 141-120-11=10 and, therefore, is consistent with the
underlying diarrheal illness as the cause of this childs metabolic acidosis. Although hypoaldosteronism
also causes normal anion gap metabolic acidosis, this diagnosis is unlikely with this infants clinical
history and absence of hyponatremia and hyperkalemia.
A metabolic acidosis with an increased anion gap occurs when HCO3 is lost and replaced by
unmeasured anions such as ketones, lactate, phosphate, or urate. Common causes of increased anion
gap acidosis in children include ketoacidosis, as seen in diabetes mellitus or starvation; lactic acidosis, as
seen in septic shock or with inborn errors of metabolism; and renal failure. The history, clinical
presentation, and presence of a normal anion gap described for the infant in the vignette are inconsistent
with diabetes mellitus, renal failure, or septic shock.

American Board of Pediatrics Content Specification(s):


Formulate a differential diagnosis of acidosis with a normal anion gap

Suggested Reading:
Greenbaum LA. Electrolyte and acid-base disorders. In: Kliegman RM, Stanton BF, St. Geme JW III,
Schor NF, and Behrman RE, eds. Nelson Textbook of Pediatrics. 19th ed. Philadelphia, PA: Saunders
Elsevier; 2011:212-242

Schwaderer AL, Schwartz GJ. Back to basics: acidosis and alkalosis. Pediatr Rev. 2004;25:350-357.
DOI: 10.1542/pir.25-10-350. Available at:
http://pedsinreview.aappublications.org/cgi/content/full/25/10/350

Copyright 2012 American Academy of Pediatrics


2012 PREP SA ON CD-ROM

Question 10
A 14-year-old boy suffers a nondisplaced fracture of his left radius and ulna while playing soccer.
He had a similar injury to his right radius and ulna 9 months ago. Physical examination reveals Sexual
Maturity Rating 2 pubic hair and testicular volume of 6 mL. A thorough review of his dietary habits
suggests that his daily intake of calcium and phosphorus are 800 mg each. He takes a 400 IU of vitamin
D supplement daily. Serum calcium measures 7.9 mg/dL (1.97 mmol/L), serum phosphorus measures 2.7
mg/dL (0.87 mmol/L), and 25-hydroxyvitamin D (25-OHD) measures 55 pg/mL (normal, 30 to 80 pg/mL).
Of the following, the MOST appropriate recommendation for this boy is to increase his
A. calcium and phosphorus intake to 1,300 mg/day
B. calcium and phosphorus intake to 2,000 mg/day
C. calcium intake to 1,000 mg/day
D. phosphorus intake to 1,000 mg/day
E. vitamin D supplementation to 2,000 IU/day

Copyright 2012 American Academy of Pediatrics


2012 PREP SA ON CD-ROM

Critique 10 Preferred Response: A


The boy described in the vignette reportedly takes 800 mg of calcium, 800 mg of phosphorus,
and 400 IU of vitamin D each day. The American Academy of Pediatrics recommends that
preadolescents and adolescents (9 to 18 years of age) consume 1,300 mg of both calcium and
phosphorus daily. Increasing the boys calcium and phosphorus intake to 2,000 mg/day is not appropriate
because excess supplementation increases the risk of hypercalcemia, hypercalciuria, hyperphosphaturia,
and renal stones.
Because this boy has a normal vitamin D concentration, he should continue to supplement his
diet with the current recommended amount of 400 IU/day. Increasing supplementation to 2,000 IU/day is
not currently indicated, although this dose is unlikely to cause hypervitaminosis. Fortunately, vitamin D
has a very wide therapeutic window, and many physicians use high-dose (500,000 IU) or stoss therapy
(derived from the German to push), followed by maintenance with the recommended dietary allowance
(RDA) when treating vitamin D deficiency. Recent evidence of the high prevalence of vitamin D deficiency
in United States children supports the proposal for a higher vitamin D RDA.
Most children in the United States do not achieve the recommended daily allowance of calcium.
Therefore, a childs intake should be estimated at each health supervision visit. As much as 40% of total
lifetime bone mineral content is accrued during adolescence, with peak calcium accretion rates in the
bones occurring at an average of 12.5 years for girls and 14 years for boys. Thus, optimizing calcium
intake is particularly important during adolescence, and those who experience delayed puberty have an
increased risk for osteoporosis and fracture. Pediatricians should actively promote bone heath by
ensuring that their patients consume adequate daily calcium, phosphorus, and vitamin D and encouraging
them to participate in physical activity and weight-bearing exercises.

American Board of Pediatrics Content Specification(s):


Understand the necessity of adequate calcium and phosphorous intake in children and adolescents
Understand the necessity of adequate vitamin D intake in children and adolescents

Suggested Reading:
Cashman KD, Flynn A. Optimal nutrition: calcium, magnesium and phosphorus. Proc Nutr Soc.
1999;58:477-487. DOI: 10.1017/S0029665199000622. Available at:
http://journals.cambridge.org/action/displayFulltext?type=6&fid=795956&jid=PNS&volumeId=58&issueId=
02&aid=795952&bodyId=&membershipNumber=&societyETOCSession=&fulltextType=MR&fileId=S0029
665199000622

Greer FR, Krebs NF, Committee on Nutrition. Optimizing bone health and calcium intakes of infants,
children, and adolescents. Pediatrics. 2006;117:578-585. DOI: 10.1542/peds.2005-2822. Available at:
www.pediatrics.org/cgi/doi/10.1542/peds.2005-2822

Wagner CL, Freer FR; Section on Breastfeeding and Committee on Nutrition. Prevention of rickets and
vitamin D deficiency in infants, children, and adolescents. Pediatrics. 2008;122:1142-1152. DOI:
10.1542/peds.2008-1862. Available at: www.pediatrics.org/cgi/doi/10.1542/peds.2008-1862

Copyright 2012 American Academy of Pediatrics


2012 PREP SA ON CD-ROM

Question 11
A father brings in his daughter for a routine health supervision visit. The father quietly holds her in
his lap while you call to her. She immediately turns toward you and begins to make babbling sounds. You
respond by saying dada while pointing to the father. The girl begins to imitate the dada sounds.
Of the following, these developmental milestones are MOST typical for a child whose age is
A. 2 months
B. 4 months
C. 6 months
D. 9 months
E. 12 months

Copyright 2012 American Academy of Pediatrics


2012 PREP SA ON CD-ROM

Critique 11 Preferred Response: C


The child described in the vignette exhibits the normal cognitive/behavioral developmental
milestones for 6 months of age: turns directly to sound and voice, babbles consonant sounds, and
imitates speech sounds. A 6-month-old child responds to social overtures by displaying an emotional
response of pleasure or a negative affect. In addition, the child reaches for familiar persons and displays
stranger anxiety with unfamiliar people.
Developmental surveillance should be part of every health supervision visit and any concerns
should be promptly addressed. A 6-month-old child should turn to a sound or voice, and a child of this
age who does not respond should be referred for audiologic testing to rule out a hearing deficit. Further
neurodevelopmental testing and referral for early intervention also should be considered.
The normal cognitive/behavioral developmental milestones for a 2-month-old infant are to smile
on social contact, listen to voice, and coo. The infant may respond to an adult voice and smile. A 4 month
old laughs out loud, squeals, initiates social interactions, and shows interest in his or her own image in a
mirror. A 9 month old imitates sounds, babbles, says mama nonspecifically, follows a pointed finger, and
recognizes familiar people. At 12 months of age, a child shows objects to his or her parents to share
interest and points for requests. The child uses several gestures while vocalizing.

American Board of Pediatrics Content Specification(s):


Recognize the normal cognitive/behavioral developmental milestones for 6 months of age (eg, turns
directly to sound and voice; babbles consonant sounds; imitates speech sounds)
Know that failure to turn to sound or voice by 6 months of age is abnormal

Suggested Reading:
Coplan J. Child development in the first 21 months. In: Parker S, Zuckerman B, Augustyn M, eds.
Developmental and Behavioral Pediatrics: A Handbook for Primary Care. 2nd ed. Philadelphia, PA:
Lippincott Williams & Wilkins, a Wolters Kluwer business; 2005:445-446

Council on Children with Disabilities, Section on Developmental Behavioral Pediatrics, Bright Futures
Steering Committee, Medical Home Initiatives for Children With Special Needs Project Advisory
Committee. Identifying infants and young children with developmental disorders in the medical home: an
algorithm for developmental surveillance and screening. Pediatrics. 2006;118:405-420. DOI:
10.1542/peds.2006-1231. Available at: http://pediatrics.aappublications.org/cgi/content/full/118/1/405

Feigelman S. The first year. In: Kliegman RM, Stanton BF, St. Geme JW III, Schor NF, and Behrman RE,
eds. Nelson Textbook of Pediatrics. 19th ed. Philadelphia, PA: Saunders Elsevier; 2011:26-31

Gerber RJ, Wilks T, Erdie-Lalena C. Developmental milestones: motor development. Pediatr Rev.
2010;31:267-277. DOI: 10.1542/pir.31-7-267. Available at:
http://pedsinreview.aappublications.org/cgi/content/full/31/7/267

Joint Committee on Infant Hearing. Year 2007 position statement: principles and guideline for early
hearing detection and intervention programs. Pediatrics. 2007;120: 898-921. DOI: 10.1542/peds.2007-
2333. Available at: http://pediatrics.aappublications.org/cgi/content/full/120/4/898

OConner Leppert ML. Neurodevelopmental assessment and medical evaluation. In: Voight RG, Macias
MM. Myers SM, eds. American Academy of Pediatrics Developmental and Behavioral Pediatrics. Elk
Grove Village, IL: American Academy of Pediatrics; 2011:93-120

Copyright 2012 American Academy of Pediatrics


2012 PREP SA ON CD-ROM

Question 12
You are precepting a group of medical students and leading a discussion on vaccine-preventable
diseases. One of the students asks you to describe how to differentiate varicella from smallpox in a child
presenting with a vesicular rash.
Of the following, the clinical feature MOST suggestive of smallpox is
A. abrupt onset of rash in a previously well child
B. centripetal spread of the skin lesions
C. involvement of the palms and soles with rash
D. lesions in multiple stages on the same part of the body
E. superficial nature of the skin vesicles

Copyright 2012 American Academy of Pediatrics


2012 PREP SA ON CD-ROM

Critique 12 Preferred Response: C


Although smallpox has been eradicated since 1977 through successful implementation of
worldwide control and vaccine programs, concern has been raised for the virus being a potential
bioterrorism agent. Clinical features of smallpox and comparison to varicella are summarized in (Item
C12A).
In contrast to varicella, the rash in smallpox involves the palms and soles. Smallpox infections
typically are characterized by a 4- to 5-day prodrome of fever and malaise, and the infected individual
appears very ill at the time of rash presentation. The lesions of smallpox spread centrifugally. Although
lesions in varicella appear in multiple stages of development on the same part of the body (Item C12B),
smallpox lesions are homogenous (Item C12C). Finally, smallpox lesions are deeper into the skin
compared to the superficial dew drop on a rose petal appearance in varicella.

American Board of Pediatrics Content Specification(s):


Differentiate the skin lesions of varicella from those of small pox

Suggested Reading:
American Academy of Pediatrics. Smallpox (variola). In: Pickering LK, Baker CJ, Kimberlin DW, Long SS,
eds. Red Book: 2009 Report of the Committee on Infectious Diseases. 28th ed. Elk Grove Village, IL:
American Academy of Pediatrics; 2009:596-598

Breman JG, Henderson DA. Diagnosis and management of smallpox. N Engl J Med. 2002;346:1300
1308. Available at: http://www.nejm.org/doi/full/10.1056/NEJMra020025

Centers for Disease Control and Prevention. Emergency Preparedness & Response. Evaluating a Rash
Illness Suspicious for Smallpox. 2007. Available at:
http://www.bt.cdc.gov/agent/smallpox/diagnosis/riskalgorithm/

World Health Organization. Media Centre. Smallpox Fact Sheet. 2007. Available at:
http://www.who.int/mediacentre/factsheets/smallpox/en/index.html

Copyright 2012 American Academy of Pediatrics


2012 PREP SA ON CD-ROM

Critique 12

Item C12A. Features of Varicella and Smallpox


Varicella Smallpox
No or mild prodrome 1- to 4-day febrile prodrome involving
prostration, chills, headache, backache,
vomiting, or severe abdominal pain
Patient rarely toxic or moribund Toxic or moribund at presentation
Superficial vesicles (dew drops on a rose petal) Deep-seated, firm lesions (item C12C);
(Item C12B) may become umbilicated or confluent
Different stages on any one part of the body Lesions in SAME stage of development
Centripetal (outward in) distribution Centrifugal (inward out) distribution
Palms or soles rarely involved Lesions on palms and soles

Copyright 2011 American Academy of Pediatrics


2012 PREP SA ON CD-ROM

Critique 12

(Courtesy of D Krowchuk)
Varicella is characterized by lesions in varying stages of development. In this patient there are
erythematous macules, papules, and vesicles. The typical appearance of a vesicle is shown (arrow), a
dew drop on a rose petal.

Copyright 2011 American Academy of Pediatrics


2012 PREP SA ON CD-ROM

Critique 12

(Courtesy of the Red Book Online)


In smallpox, all lesions are in the same stage of development, in this case, firm pustules.

Copyright 2011 American Academy of Pediatrics


2012 PREP SA ON CD-ROM

Question 13
You are seeing an infant in the newborn nursery who was born this morning to a woman who has
hepatitis C virus (HCV) infection. The infant is healthy. The mother wants to know if she should
breastfeed her infant.
Of the following, the MOST appropriate information to share with this mother is that
A. breastfeeding is contraindicated if the mother has evidence of liver dysfunction
B. breastfeeding is contraindicated when a mother has HCV infection
C. breastfeeding is recommended if the mother does not also have hepatitis B virus infection
D. breastfeeding is recommended if the mother does not also have human immunodeficiency virus
infection
E. breastfeeding is recommended only if viral cultures of human milk show no evidence of HCV

Copyright 2012 American Academy of Pediatrics


2012 PREP SA ON CD-ROM

Critique 13 Preferred Response: D


Hepatitis C virus (HCV), a small, single-stranded RNA virus, is a member of the Flavivirus family,
and its transmission from mother to infant via human milk never has been documented. Therefore, both
the American Academy of Pediatrics and the Centers for Disease Control and Prevention advise that
maternal HCV infection is not a contraindication to breastfeeding. However, HCV-positive mothers who
have bleeding or cracked nipples should consider deferring breastfeeding until the skin is healed. In
resource-rich countries, such as the United States, where infant formulas are affordable and readily
available, it is recommended that mothers who are infected with human immunodeficiency virus (HIV),
regardless of their HCV infection status, not breastfeed. The rate of transmission of HIV by breastfeeding
increases in mothers who acquire HIV infection late in pregnancy or during the postpartum period (rate of
approximately 0.7% per month of breastfeeding). Therefore, the woman in the vignette should be advised
that breastfeeding her infant is recommended if she does not have HIV infection.
Most patients infected with HCV are asymptomatic. The signs and symptoms of acute infection
usually are indistinguishable from those in persons who have hepatitis A or hepatitis B virus (HBV)
infection. HBV infection also is not a contraindication to breastfeeding. Abnormalities in liver function
studies with HCV infection usually are less pronounced than those that occur with HBV infection, and the
degree of maternal liver dysfunction has no bearing on a womans ability to breastfeed. Chronic hepatitis
develops in 70% to 80% of HCV-infected adults, and HCV infection is the leading indication for liver
transplantation in adults in the United States.
HCV primarily is spread through parenteral blood exposure. Therefore, obtaining viral cultures of
expressed human milk is not recommended. Although both HCV RNA and anti-HCV immunoglobulin G
(IgG) have been found in colostrum, the risk of HCV infection in breastfed and formula-fed infants is the
same. The leading risk factors for HCV infection are injection drug use, receiving blood products before
1992, and having multiple sexual partners.
It is estimated that 1% to 2% of pregnant women in the United States are seropositive for HCV.
Only 5% of infected women will transmit HCV infection to their infants, and perinatal transmission of HCV
is possible only in women who are HCV RNA-positive at delivery. However, maternal coinfection with HIV
increases the risk of transmission.
Maternal anti-HCV antibody (IgG) is placentally transferred and interferes with the ability to detect
antibody produced by the newborn. It may persist for up to 18 months. Assays for detecting HCV IgM are
not available. Nucleic acid amplification tests for detecting HCV RNA in serum or plasma within 1 to 2
weeks of exposure are available and licensed by the United States Food and Drug Administration.
However, because false-positive and false-negative results can occur, a single negative test does not
definitively exclude disease.

American Board of Pediatrics Content Specification(s):


Know for which maternal chronic viral infections breastfeeding is not recommended

Suggested Reading:
American Academy of Pediatrics. Hepatitis C. In: Pickering LK, Baker CJ, Kimberlin DW, Long SS, eds.
Red Book: 2009 Report of the Committee on Infectious Diseases. 28th ed. Elk Grove Village, IL:
American Academy of Pediatrics; 2009:357-360

American Academy of Pediatrics. Human immunodeficiency virus infection. In: Pickering LK, Baker CJ,
Kimberlin DW, Long SS, eds. Red Book: 2009 Report of the Committee on Infectious Diseases. 28th ed.
Elk Grove Village, IL: American Academy of Pediatrics; 2009:380-400

Lawrence RM, Lawrence RA. Breastfeeding: more than just good nutrition. Pediatr Rev. 2011;32:267-
302. DOI: 10.1542/pir.32-7-267. Available at:
http://pedsinreview.aappublications.org/content/32/7/267.full?sid=a39fcde1-f165-40eb-87b6-
6ac38127e27e

Copyright 2012 American Academy of Pediatrics


2012 PREP SA ON CD-ROM

Question 14
A 1-day-old male infant fails to pass urine in the nursery. He was born at 37 weeks gestation to a
30-year-old woman who received prenatal care. On physical examination, the infant exhibits mild
tachypnea, wrinkling of the skin on the anterior abdominal wall (Item Q14), and undescended testicles.
Of the following, the MOST useful test to diagnose the cause of anuria in this infant is
A. mercaptoacetyltriglycine (MAG-3) furosemide renal scan
B. renal/bladder ultrasonography
C. serum creatinine measurement
D. urinalysis obtained by urethral catheterization
E. voiding cystourethrography

Copyright 2012 American Academy of Pediatrics


2012 PREP SA ON CD-ROM

Question 14

(Courtesy of M Rimsza)
Wrinkling of the skin on the anterior abdominal wall, as described for the infant in the vignette.

Copyright 2012 American Academy of Pediatrics


2012 PREP SA ON CD-ROM

Critique 14 Preferred Response: B


The infant described in the vignette has prune-belly syndrome, which is characterized by the triad
of bilateral hydroureteral nephrosis, undescended testicles, and diminished/absent anterior abdominal
wall musculature, resulting in a wrinkled anterior abdominal wall. In addition, he is exhibiting anuria.
Assessment of anuric or oliguric renal failure begins with determination of whether it is prerenal,
intrinsic renal, or postrenal renal failure. The differential diagnosis of anuria in the neonatal period
includes prerenal causes (sepsis, congestive heart failure), intrinsic renal disease of the parenchyma
(acute tubular necrosis, cortical necrosis, or autosomal recessive polycystic kidney disease) or
vasculature (renal vein thrombosis), or postrenal failure (posterior urethral valves, prune-belly syndrome,
neurogenic bladder, obstruction of a single kidney). The clinician should be familiar with this differential
diagnosis to optimize evaluation and treatment of infants who have decreased urine output.
Prune-belly syndrome is a form of obstructive uropathy with associated renal parenchymal
damage from the high back pressure of the obstructed urinary system. Accordingly, the clinician must
assess the status of the urinary tract, including the kidneys and bladder, which is accomplished best with
renal/bladder ultrasonography. This evaluation allows the clinician to determine the most probable cause
of renal failure as well as the status of the urinary tract. For example, the infant in the vignette would be
expected to have hydronephrosis with associated hydroureter. A distended urinary bladder (containing
urine) would further support the likelihood of bladder outlet obstruction.
Another test used to evaluate the urinary tract is voiding cystourethrography, which involves
instillation of radiocontrast into the urinary bladder via a bladder catheter. While under fluoroscopic
surveillance, the infant voids, which allows the clinician to assess bladder volume, bladder anatomy
(including the absence of a ureterocele), and the presence or absence of vesicoureteral reflux (VUR).
Although this is a useful test in the evaluation of an infant who has hydronephrosis to rule out VUR and in
the male infant who has bilateral hydronephrosis to rule out posterior urethral valves, this would not be
the appropriate initial test for the infant in the vignette.
Measurement of serum creatinine can assess renal function, but it will not determine the cause of
anuria. In the immediate newborn period (first 24 to 48 hours after birth), the creatinine concentration is
influenced by maternal creatinine values. However, creatinine values that exceed maternal norms warrant
further assessment of neonatal renal function. Urinalysis is of limited diagnostic value for this infant, and
catheterization at this age can be difficult, possibly requiring the expertise of a pediatric urologist for
infants who have obstructive uropathy. This point should be conveyed to nursing staff in the neonatal
setting to avoid potential complications of catheterization. A mercaptoacetyltriglycine (MAG-3) furosemide
renal scan is a dynamic nuclear medicine test that is best used in the evaluation of suspected
ureteropelvic junction obstruction.

American Board of Pediatrics Content Specification(s):


Plan the evaluation of an anuric infant

Suggested Reading:
Chua AN, Sarwal MM. Acute renal failure management in the neonate. NeoReviews. 2005;6:e369-e376.
DOI: 10.1542/neo.6-8-e369. Available at: http://neoreviews.aappublications.org/cgi/content/full/6/8/e369

Subramanian S, Agarwal R, Deorari AK, Paul VK, Bagga A. Acute renal failure in neonates. Indian J
Pediatr. 2008;75:385-391. DOI: 10.1007/s12098-008-0043-4. Available at:
http://www.ncbi.nlm.nih.gov/pubmed/18536895

Copyright 2012 American Academy of Pediatrics


2012 PREP SA ON CD-ROM

Question 15
A mother brings in her 12-month-old infant for evaluation. One month ago she transitioned him
from human milk to a cow milk formula but noted that each time her son drank the formula, he developed
a rash around his mouth and scattered hives on his trunk. After speaking with her pediatrician she
switched to rice milk, which the infant has tolerated well. She reports that last week when her son
accidently drank cow milk, he developed a perioral rash and vomited once.
Of the following, you are MOST likely to counsel the mother that
A. bloody stools is a common presenting symptom
B. he is unlikely to tolerate a soy-based formula
C. he may tolerate foods containing cow milk if they are extensively heated
D. he will most likely develop other food allergies
E. negative allergy skin testing will exclude the diagnosis

Copyright 2012 American Academy of Pediatrics


2012 PREP SA ON CD-ROM

Critique 15 Preferred Response: C


When evaluating a patient for an adverse food reaction, a careful history is important to
distinguish whether a reaction is immunoglobulin (Ig)E-mediated or non-IgE-mediated. IgE-mediated food
hypersensitivity, as described for the infant in the vignette, typically involves flushing, urticaria, rash, or
angioedema. Other IgE-mediated symptoms may include vomiting, bronchospasm, abdominal cramping,
and anaphylactic shock. Most patients are educated to avoid particular foods that cause adverse
reactions. However, recent studies have demonstrated that certain foods that contain heat-labile proteins
(eg, cow milk and eggs) are tolerated in most children if they are extensively heated. In a recent study of
100 children ages 2 to 17 years, who were allergic to milk, 75% tolerated extensively heated (EH) muffins
and waffles that contained 1.3 g of milk protein. All children who tolerated EH food challenges continued
to eat one to three servings per day without reaction. Three months after beginning the study, the children
who tolerated the challenges demonstrated decreased skin test wheal size and increased serum IgG4,
both of which are encouraging immunologic trends toward possible clinical resolution of their allergic
reactions to unheated food.
Fortunately, although cow milk allergy is one of the most common IgE-mediated food allergies in
children, more than 90% of infants older than 6 months of age can safely be switched to a soy formula.
The risk for developing other food allergies is increased in children who have milk allergy, but only about
one third of patients develop other food allergies. After a thorough history and physical examination, IgE
skin prick testing or specific serum IgE testing should be performed. Interpretation of test results by an
allergist is recommended because the predictive value for each test result is different for different foods,
although a negative skin prick test result has a negative predictive value of 95% or greater for the most
common foods allergens (eg, milk, egg, wheat, soy, peanut, fish). Children at low risk for an IgE-mediated
food allergy, based on an unconvincing clinical history or a negative skin prick test, should undergo a
physician-supervised food challenge.
Non-IgE-mediated cow milk reactions include hematochezia (food protein colitis), abdominal pain,
and failure to thrive. Food protein colitis typically presents in the first 6 months after birth with blood-
streaked stools. Unlike IgE-mediated cow milk allergy, many infants who have food protein colitis or
enterocolitis continue to have symptoms with soy formula and typically require a change to a
hypoallergenic or amino-acid based formula.

American Board of Pediatrics Content Specification(s):


Recognize the signs and symptoms of milk protein allergy

Suggested Reading:
NAID-sponsored Expert Panel. Guidelines for the diagnosis and management of food allergy in the united
states: report of the NIAID-Sponsored Expert Panel. J Allergy Clin Immunol. 2010;126(suppl):S1-S58.
DOI: 10.1016/j.jaci.2010.10.007. Available at: http://www.jacionline.org/article/S0091-6749(10)01566-
6/fulltext

Nowak-Wegrzyn A, Bloom KA, Sicherer SH, et al. Tolerance to extensively heated milk in children with
cow's milk allergy. J Allergy Clin Immunol. 2008;122:342-347. DOI: 10.1016/j.jaci.2008.05.043. Available
at: http://www.jacionline.org/article/S0091-6749(08)01111-1/fulltext

Copyright 2012 American Academy of Pediatrics


2012 PREP SA ON CD-ROM

Question 16
An 18-month-old boy is brought to the office because he has been difficult to arouse for the past
hour. His mother reports that earlier in the day she found him in his grandmothers room playing with her
medicine bottles, but none of the bottles were opened. The mother explains that the grandmother takes
"pills for her heart. The child is somnolent and responsive only to pain. His temperature is 38.6C, heart
rate is 130 beats/min, respiratory rate is 56 breaths/min, and blood pressure is 90/60 mm Hg, and his
pupils are midsized and reactive. The remainder of his physical examination findings are normal.
Of the following, the MOST likely explanation for this childs symptoms is
A. aspirin ingestion
B. intracranial hemorrhage
C. lisinopril ingestion
D. metoprolol ingestion
E. sepsis

Copyright 2012 American Academy of Pediatrics


2012 PREP SA ON CD-ROM

Critique 16 Preferred Response: A


The child described in the vignette is exhibiting signs and symptoms consistent with salicylate
poisoning. Although salicylate poisoning is not seen as commonly as before the introduction of childproof
packaging and other nonsalicylate antipyretics/analgesics, intoxication with aspirin and other salicylates is
responsible for more than 20,000 reported cases to United States poison control centers annually. The
recommended use of low-dose aspirin as primary prevention for cardiovascular disease in adults means
that aspirin is readily available and potentially accessible to children in many households.
The clinical findings of acute salicylism can be explained by the many cellular and systemic
effects of the drugs (Item C16) and are related to ingested dose. Mild symptoms are typically seen in
ingestions of 300 mg/kg or less; those of greater than 300 mg/kg lead to moderate toxicity and greater
than 500 mg/kg lead to death. Early/mild signs and symptoms include fever/hyperpyrexia, nausea,
vomiting, diarrhea, and tinnitus. Tachypnea/hyperpnea, tachycardia, hypoglycemia, metabolic acidosis
with an elevated anion gap, altered mental status (lethargy to coma), and seizures are commonly seen in
more severe poisonings.
Such clinical findings can overlap with a number of equally serious conditions, including diabetic
ketoacidosis, acute iron poisoning, and ethylene glycol toxicity. The patient who has intracranial
hemorrhage is likely to exhibit bradycardia, hypertension, and focal neurologic signs. In lisinopril and
metoprolol ingestions, hypotension predominates, with bradycardia seen additionally with a beta-blocker
overdose. Finally, an obtunded patient who has sepsis is likely to exhibit hypotension and other findings
consistent with septic shock or meningitis.
The management phases of acute salicylate poisoning encompass stabilization, decontamination,
and elimination, with all aspects of therapy directed at maintaining serum alkalemia. Because acidosis
promotes diffusion of salicylic acid into the central nervous system, life-threatening neurologic symptoms
can be avoided with aggressive alkalization. Initial stabilization should focus on airway maintenance,
rehydration, and correction of electrolyte abnormalities. Oxygen should be provided as needed, but
intubation and mechanical ventilation are reserved for those patients who have respiratory failure. The
increases in minute ventilation seen in salicylate-poisoned patients contribute significantly to maintenance
of alkalemia, and attempts to normalize ventilation by eliminating the patients own hyperpnea have
resulted in death from severe acidosis. Hypovolemia and potassium deficits should be corrected.
Intravenous fluids should contain both potassium and sodium bicarbonate to replete potassium stores
and maintain alkalemia. Serum pH measurements should be maintained between 7.5 and 7.6 during
therapy.
Decontamination is accomplished using multiple doses of activated charcoal, which should be
administered every 4 hours until the patient is asymptomatic and the salicylate concentration is less than
30 mg/dL. Salicylic acid is eliminated in the urine and such elimination is maximized if the urine is
alkaline. In severe poisoning associated with significant neurologic signs, pulmonary edema,
unresponsive metabolic acidosis, renal failure, or plasma salicylate concentrations greater than 100
mg/dL, hemodialysis is indicated.

American Board of Pediatrics Content Specification(s):


Recognize the signs and symptoms of salicylate toxicity
Understand the management of salicylate toxicity

Suggested Reading:
Barnett AK, Boyer EW, Traub SJ. Salicylate poisoning in children and adolescents. UpToDate Online
18.3. 2010. Available at: http://www.uptodate.com/online/content/topic.do?topicKey=ped_tox/8460

Chyka PA, Erdman AR, Christianson G, et al. Salicylate poisoning: an evidence-based consensus
guideline for out-of-hospital management. Clin Toxicol (Phila). 2007;45:95-131. Available at:
http://www.guideline.gov/content.aspx?id=9905

ODonnell KA, Burns Ewald M. Pediatric drug therapy: poisonings. In: Kliegman RM, Stanton BF, St.
Geme JW III, Schor NF, and Behrman RE, eds. Nelson Textbook of Pediatrics. 19th ed. Philadelphia, PA:
Saunders Elsevier; 2011:250-270

Copyright 2012 American Academy of Pediatrics


2012 PREP SA ON CD-ROM

Stolbach AI, Hoffman RS, Nelson LS. Mechanical ventilation was associated with acidemia in a case
series of salicylate-poisoned patients. Acad Emerg Med 2008;15:866-869. DOI: 10.1111/j.1553-
2712.2008.00205.x. Available at: http://onlinelibrary.wiley.com/doi/10.1111/j.1553-2712.2008.00205.x/full

US Preventive Services Task Force. Aspirin for the prevention of cardiovascular disease: U.S. Preventive
Services Task Force recommendation statement. Ann Intern Med. 2009;150:396404. Available at:
http://www.annals.org/content/150/6/396.long

Copyright 2012 American Academy of Pediatrics


2012 PREP SA ON CD-ROM

Critique 16

Item C16. Signs and Symptoms of Acute Salicylate Poisoning


Clinical and Laboratory
Pathophysiology
Findings
Tachypnea/hypernea Direct stimulation of the medullary respiratory center. Leads to respiratory
alkalosis and hypovolemia from increased insensible fluid losses
Metabolic acidosis with Uncoupling of mitochondrial oxidative phosphorylation, with subsequent
elevated anion gap impairment of aerobic metabolism and accumulation of organic acids
Respiratory alkalosis Due to tachypnea
Tachycardia Due to hypovolemia and hyperpyrexia
Hyperpyrexia Due to impairment in cellular metabolism
Hypoglycemia Due to impairment in cellular metabolism
Hypovolemia Increased insensible losses from tachypnea and vomiting
Hypokalemia Renal potassium loss as bicarbonate is preferentially excreted to
compensate for respiratory alkalosis
Tinnitus Direct central nervous system effect
Coagulation defects Inhibition of prostaglandin, prostacyclin, thromboxane synthesis
Nausea/vomiting Direct irritant effects on the gastric mucosa and stimulation of the
diarrhea/hematemesis chemoreceptor trigger zone
Altered mental status: Direct central nervous system effects
lethargy, agitation, coma
Seizures Direct central nervous system effect, hypoxia
Noncardiac pulmonary Unknown
edema

Copyright 2011 American Academy of Pediatrics


2012 PREP SA ON CD-ROM

Question 17
You are caring for a 7-year-old boy who developed a mid-gut volvulus at age 3, necessitating
resection of most of his small bowel. Only 10 cm of ileum and a few centimeters of duodenum remain,
and the child is dependent on parenteral nutrition for most of his protein-energy intake. He recently
developed a cough and dyspnea on exertion. Physical examination demonstrates a gallop cardiac rhythm
and the presence of rales at the lung bases. Chest radiography shows cardiomegaly.
Of the following, the nutritional deficiency that is MOST likely responsible for this boys symptoms
and signs is
A. chromium
B. copper
C. manganese
D. selenium
E. zinc

Copyright 2012 American Academy of Pediatrics


2012 PREP SA ON CD-ROM

Critique 17 Preferred Response: D


The child described in the vignette suffered a catastrophic intestinal event that left him with little
functioning bowel and dependent on parenteral nutrition for survival. The subsequent development of
cardiorespiratory dysfunction should alert the clinician to possible complications of central line placement,
including both infection and catheter dislodgement. However, particularly after long-term parenteral
nutrition support (4 years for this boy), factors related to composition of the nutrient infusate must also be
considered. The nutritional cause of what appears to be an emerging cardiomyopathy is most likely a
deficiency of the trace mineral selenium.
Selenium has only recently been identified as an essential nutrient, and deficiency in children has
been limited to those receiving long-term parenteral nutrition without selenium supplementation. This
trace mineral appears to be involved in the bodys antioxidant defense system, and it is incorporated into
cellular, serum, and membrane-bound glutathione peroxidases. Selenium deficiency, where described,
has been associated with skin and hair pigment loss, macrocytosis, and in severe cases,
cardiomyopathy.
Trace minerals constitute less than 0.01% of total body weight in healthy individuals, but these
moieties are integral to numerous enzyme systems, functioning either as activating cofactors or as key
components of metalloenzymes. Symptomatic trace mineral deficiencies, although uncommon, occur
most frequently during periods of rapid growth (eg, infancy, particularly in preterm infants), in chronic
malnourished states, and as a consequence of intestinal maldigestion/malabsorption. In order of their
importance in childhood, the trace elements include iron, zinc, copper, fluoride, iodine, selenium,
manganese, cobalt, molybdenum, nickel, silicon, and vanadium. Calcium, phosphorus, and magnesium
together comprise 98% of the total body mineral content by weight (with the highest concentrations in
bone) and, therefore, are not considered trace elements. Recommended daily allowances (RDAs) for
zinc, copper, manganese, chromium, iodine, molybdenum, and selenium have been established by the
Food and Nutrition Board of the Institute of Medicine.
As noted, trace mineral deficiencies are uncommon in infants and children, occurring most
frequently in the setting of severe malnutrition, often associated with chronic diarrheal states (prominent
in the developing world) or as a consequence of disorders manifested by intestinal
malabsorption/maldigestion (eg, cystic fibrosis, short bowel syndrome, inflammatory bowel disease). For
patients requiring long-term parenteral nutrition support, current formulations of nutrient solutions are
supplemented with trace minerals, making deficiency states much less prevalent than in the past.
Zinc deficiency is the most often reported trace mineral deficiency after iron. Features of
abnormal zinc nutriture include acro-orificial skin lesions, diarrhea, increased susceptibility to infection,
and growth retardation. These clinical findings characterize acrodermatitis enteropathica, an autosomal
recessive disorder of zinc metabolism resulting from a mutation in the gene for intestinal brush-border
membrane ZIP4, a transporter that regulates enterocyte zinc uptake. Affected patients manifest severe
deficiency. Less marked zinc deficiency states may be difficult to recognize but should be suspected in at-
risk individuals who demonstrate slow growth, compromised immunity with frequent infections, and
anorexia due to loss of taste sensation (hypogeusia).
The greatest risk for copper deficiency is seen in preterm infants in whom hepatic copper stores
are low because this trace mineral accumulates in the liver during the third trimester of pregnancy.
However, in this and in other at-risk clinical situations (rapid growth rate, malabsorption syndromes,
severe malnutrition), the deficiency state is generally not seen unless both endogenous copper stores
and dietary copper intake are low. Signs of copper deficiency include neutropenia, hypochromic anemia
unresponsive to iron administration, bone abnormalities, and hair and skin depigmentation. Menkes
syndrome is an X-linked recessive disorder of copper metabolism caused by a defect in the protein
ATP7A, resulting in an inability to export intracellular copper. This syndrome is characterized by anemia,
steely hair, and progressive cerebral degeneration and has a poor prognosis despite aggressive copper
supplementation.
Other trace mineral deficiencies have generally been reported only as a consequence of severe
malnutrition. Chromium functions as a cofactor for insulin, and the deficiency state, although rare, may be
characterized by impaired glucose, fat, and protein metabolism and by growth retardation. Manganese is
a required cofactor for some enzymes, including arginase and pyruvate carboxylase. Its nutrient status is
difficult to assess because manganese is present in very low concentrations in the bloodstream. The

Copyright 2012 American Academy of Pediatrics


2012 PREP SA ON CD-ROM

essential nature of this trace mineral in the human diet has not been demonstrated, and the dietary
requirement for manganese is very small. To date only one possible case of manganese deficiency has
been described.

American Board of Pediatrics Content Specification(s):


Know the diseases that are associated with trace mineral deficiency (zinc, copper, magnesium,
chromium)

Suggested Reading:
Kleinman RE. Trace elements. In: Pediatric Nutrition Handbook. 6th ed. Elk Grove Village, IL: American
Academy of Pediatrics; 2009:423-452

Krebs NF, Westcott JE, Arnold TD, et al. Abnormalities in zinc homeostasis in young infants with cystic
fibrosis. Pediatr Res. 2000;48:256-261. Available at: http://www.ncbi.nlm.nih.gov/pubmed/10926304

Litov RE, Combs GF Jr. Selenium in pediatric nutrition. Pediatrics. 1991;87:339-351. Available at:
http://pediatrics.aappublications.org/cgi/reprint/87/3/339

Lockitch G, Taylor GP, Wong LT, et al. Cardiomyopathy associated with nonendemic selenium deficiency
in a Caucasian adolescent. Am J Clin Nutr. 1990;52:572-577. Available at:
http://www.ajcn.org/content/52/3/572.long

Mertz W. Chromium in human nutrition: a review. J Nutr. 1993;123:626-633. Available at:


http://jn.nutrition.org/content/123/4/626.long

Olivares M, Araya M, Uauy R. Copper homeostasis in infant nutrition: deficit and excess. J Pediatr
Gastroenterol Nutr 2000;31:102-111. Available at:
http://journals.lww.com/jpgn/Fulltext/2000/08000/Copper Homeostasis_in infant_Nutrition__Deficit.4.aspx

Copyright 2012 American Academy of Pediatrics


2012 PREP SA ON CD-ROM

Question 18
You are seeing a 36-year-old primigravida in your office for a pediatric prenatal visit. She is
presently at 32 weeks gestation, and her pregnancy is complicated by chronic hypertension and
smoking. She relates that her obstetrician has told her that she likely will need to be delivered by 36
weeks gestation because my baby is not growing well. She has read in the newspaper that late preterm
infants may have more medical problems than infants born at term and questions why her infant needs to
be delivered so early.
Of the following, you are MOST likely to advise this expectant mother that delivery at 36 weeks
gestation may decrease her babys risk of
A. asphyxia
B. hyperbilirubinemia
C. hypoglycemia
D. hypothermia
E. polycythemia

Copyright 2012 American Academy of Pediatrics


2012 PREP SA ON CD-ROM

Critique 18 Preferred Response: A


The fetus described in the vignette has intrauterine growth restriction (IUGR) and is at increased
risk of asphyxia and intrauterine fetal demise. Close monitoring with serial Doppler velocimetry
measurements alone or in combination with biophysical profile testing or nonstress testing is needed to
optimize antepartum care, which should include carefully timed early delivery to decrease this risk.
IUGR describes a reduction in the expected fetal growth pattern, regardless of whether the infant
is small for gestational age (SGA) at the time of birth. Fetal growth depends upon contributions from the
mother, the placenta, and the fetus. Conditions leading to uterine ischemia, hypoxia, and nutritional
deficiency have a significant negative effect on growth. For example, maternal conditions such as chronic
hypertension and diabetes mellitus as well as maternal habits such as smoking may contribute to
decreased fetal growth.
SGA infants have a higher neonatal mortality rate than appropriate-for-gestational age infants.
IUGR and SGA infants require close medical attention after delivery. They are prone to fasting
hypoglycemia due to decreased glycogen stores and impaired gluconeogenesis resulting from decreased
nutrition in utero. As a response to hypoxia, compensatory polycythemia may develop. These infants are
susceptible to temperature instability due to both decreased brown fat and small size. The risk of these
conditions developing is not decreased by early delivery. In addition, any late preterm infant (34 0/7 to 36
6/7 weeks) is at higher risk of hyperbilirubinemia than an infant delivered at term.

American Board of Pediatrics Content Specification(s):


Recognize that small-for-gestational-age infants have a higher neonatal mortality rate than
appropriate-for-gestational age infants
Know that small-for-gestational-age infants are prone to fasting hypoglycemia, polycythemia, and
temperature instability, and manage appropriately
Know that perinatal asphpyxia is a frequent complicaton of intrauterine growth restriction

Suggested Reading:
Oelberg DG. Consultation with the specialist: prenatal growth: the sum of maternal, placental, and fetal
contributions. Pediatr Rev. 2006;27:224-229. DOI: 10.1542/pir.27-6-224. Available at:
http://pedsinreview.aappublications.org/cgi/content/full/27/6/224

Rogers EE, Piecuch RE. Neurodevelopmental outcomes of infants who experience intrauterine growth
restriction. NeoReviews. 2009;10 e100-e112. DOI: 10.1542/neo.10-3-e100. Available at:
http://neoreviews.aappublications.org/cgi/content/full/10/3/e100

Thureen PJ, Anderson MS, Hay WW Jr. The small-for-gestational age infant. NeoReviews. 2001;2:e139-
149. DOI: 10.1542/neo.2-6-e139. Available at:
http://neoreviews.aappublications.org/cgi/content/full/2/6/e139

Copyright 2012 American Academy of Pediatrics


2012 PREP SA ON CD-ROM

Question 19
A 3-year-old child presents with the sudden onset of labored breathing and a barking cough. His
father explains that he was fine yesterday and had no respiratory symptoms. He has had two similar
episodes in the past 18 months. Physical examination reveals a temperature of 37.2C, respiratory rate of
28 breaths/min, and heart rate of 132 breaths/min. He has stridor with agitation, occasional suprasternal
retractions, and clear lungs.
Of the following, the MOST likely additional finding to expect for this boy is
A. a 5-mm hemangioma on his leg
B. episodes of wheezing associated with past respiratory illnesses
C. exposure to family members who have strep throat
D. four past episodes of otitis media
E. receipt of only two doses of Haemophilus influenzae type b vaccine

Copyright 2012 American Academy of Pediatrics


2012 PREP SA ON CD-ROM

Critique 19 Preferred Response: B


The boy described in the vignette has croup, a term often used to describe a clinical picture
resulting from several different underlying conditions. Patients present with inspiratory stridor, barking
cough, hoarseness, and varying degrees of respiratory distress. Affected children generally are between
6 and 36 months of age, with most presenting in the second year after birth. The occurrence rate for boys
is 1.5 times that of girls. Typically, croup is considered a single event resulting from a viral infection, most
often parainfluenza 1, although several other viruses also have been implicated. The illness is preceded
or accompanied by other viral-induced symptoms, including coryza and fever. This viral illness might be
more properly called laryngotracheitis or if it also involves the lower respiratory tract,
laryngotracheobronchitis.
Some patients, such as the boy in the vignette, have repeated bouts of stridor, barking cough,
and hoarseness and are described as having frequently recurrent croup, allergic croup, or spasmodic
croup. Spasmodic croup most often is used to describe the sudden onset of respiratory symptoms at
night that persist for only a short duration; it is associated with, at most, mild upper respiratory tract
symptoms. Affected patients may demonstrate a viral pathogen similar to patients who have
laryngotracheitis and may have repeated attacks during the course of an illness, but they often do not
have the continuous symptoms over days seen with patients who have laryngotracheitis. Frequently, they
have a history of atopy, including positive skin tests to environmental allergens, a higher incidence of
asthma as they grow older, and increased serum immunoglobulin E concentrations. Many have been
treated for reactive airway disease or have had additional episodes of wheezing with viral illnesses.
In addition to hyperactive airways, other conditions may predispose children to recurrent croup. A
foreign body in the larynx produces stridor and hoarseness, and a foreign body in the esophagus can
distort the adjacent trachea, causing a barking cough and stridor. Anatomic abnormalities, including
laryngeal webs, vocal cord paralysis, vascular rings, subglottic stenosis, cysts, hemangiomas, and
laryngeal papillomas, lead to airway narrowing that, particularly in the face of an upper respiratory tract
viral infection, can result in varying degrees of airway obstruction. A recent study of bronchoscopy
performed in 30 children who had recurrent croup found that one third had an underlying airway disorder,
most commonly subglottic stenosis unrelated to previous intubation. Other studies have shown a high
incidence of gastroesophageal reflux disease by endoscopic examination and resolution of croup
symptoms with treatment for reflux.
Although bacterial tracheitis sometimes may be labeled bacterial croup, it is distinguished by an
older age at onset (4 to 8 years), thick mucopurulent exudates in the trachea, and a greater degree of
toxicity. Group A beta-hemolytic Streptococcus is one of the possible bacterial causes of this condition,
but exposure to adults who have streptococcal pharyngitis is not a typical risk factor for bacterial
tracheitis, viral croup, or recurrent croup. Epiglottitis due to Haemophilus influenzae type b is another
cause for the acute onset of upper airway obstruction, but the disease is distinct etiologically,
anatomically, and in severity from croup. Incomplete immunization for H influenzae type b, therefore,
would not be expected to be associated with recurrent croup. The incidence of both otitis media and
recurrent croup may be increased in children who have atopic disease, but there is no direct relationship
between the two conditions. Airway hemangiomas can cause recurrent bouts of stridor, but a single small
hemangioma on an extremity is rarely associated with airway hemangiomas.

American Board of Pediatrics Content Specification(s):


Distinguish between viral and noninfectious croup

Suggested Reading:
Castro-Rodriguez JA, Holberg CJ, Morgan WJ, et al. Relation of two different subtypes of croup before
age three to wheezing, atopy, and pulmonary function during childhood: a prospective study. Pediatrics.
2001;107:512-518. Available at: http://pediatrics.aappublications.org/cgi/content/full/107/3/512

Cherry JD. Clinical practice: croup. N Engl J Med. 2008;358:384-391. Extract Available at:
http://www.nejm.org/doi/full/10.1056/NEJMcp072022

Copyright 2012 American Academy of Pediatrics


2012 PREP SA ON CD-ROM

Cherry JD. State of the evidence for standard-of-care treatments for croup: are we where we need to be?
Pediatr Infect Dis J. 2005;24(11 suppl):S198-S202. Available at:
http://journals.lww.com/pidj/Fulltext/2005/11001/State_of_the_Evidence_for_Standard_of_Care.8.aspx

Chun R, Preciado DA, Zalzal GH, Shah RK. Utility of bronchoscopy for recurrent croup. Ann Otol Rhinol
Laryngol. 2009;118:495-499. Abstract available at: http://www.ncbi.nlm.nih.gov/pubmed/19708488

Kwong K, Hoa M, Coticchia JM. Recurrent croup presentation, diagnosis, and management. Am J
Otolaryngol. 2007;28:401-407. DOI: 10.1016/j.amjoto.2006.11.013. Abstract available at:
http://www.ncbi.nlm.nih.gov/pubmed/17980773

Woods CR. Clinical features, evaluation, and diagnosis of croup. UpToDate Online. 2010. Available for
subscription at: http://www.uptodate.com/online/content/topic.do?topicKey=pedi_id/22768

Copyright 2012 American Academy of Pediatrics


2012 PREP SA ON CD-ROM

Question 20
A game warden accompanies his wife, new baby, and 13-year-old stepson to the infants 2-week
health supervision visit. He explains that he must store the gun he is required to carry for his job at home,
but he is concerned about the risks of having a gun in the home.
Of the following, the BEST advice to give this father is to
A. enroll his stepson in a formal firearm safety course
B. show the stepson how to handle the gun appropriately
C. store the gun locked and loaded in a high, secret cabinet
D. store the gun locked and unloaded with ammunition locked and stored separately
E. store the gun unloaded in a locked gun safe with the ammunition stored adjacently

Copyright 2012 American Academy of Pediatrics


2012 PREP SA ON CD-ROM

Critique 20 Preferred Response: D


Firearm injuries are common in the United States, which has the highest rate of gun injuries
among developed nations. In 2004, the Centers for Disease Control and Prevention recorded 2,852
firearm-related deaths in children as well as 13,846 nonfatal gun-related injuries. In 2007, there were 12.5
firearm deaths per 100,000 children in the United States. Males 15 to 19 years of age are eight times
more likely to die of firearm-related injuries than females and African American male youth sustained the
highest rates of firearm related deaths (combined homicide, suicide, and accidental deaths) at 70 per
100,000 adolescents in 2007.
A 2005 study in the Journal of the American Medical Association documented that safe storage of
both long guns and handguns reduced the risk of suicide and accidental injury due to firearms. Thus,
parents who own guns should be advised of the need to safely store guns. Unfortunately, some parents
may not wish to discuss their gun ownership with their childs physician and since most firearms are
owned and stored by men, mothers may not know if there is a gun in the home and how it is stored.
Children also may be at increased risk for firearm injury if there are accessible guns in the homes
of their playmates or child care provider. Therefore, parents should also be advised to ask child care
providers and others who may care for their child about accessible guns in their homes and it may be
advisable to discuss firearm injury prevention with all families regardless of gun ownership.
Ideally, safe storage of firearms involves placement of unloaded and locked firearms and
ammunition in separate storage areas with separate locks since a loaded firearm or easy availability of
ammunition which is stored near the firearm increases the risk that an unsupervised child will be injured.
There is no evidence that firearm education is an effective way to prevent firearm injury.

American Board of Pediatrics Content Specification(s):


Counsel parents regarding the risks of having firearms in the home
Know the epidemiology of firearms in US households
Identify components of an injury prevention plan for firearms in a household (locked, unloaded,
discarded)
Know that firearms are a leading cause of death in adolescents

Suggested Reading:
Committee on Injury and Poison Prevention. American Academy of Pediatrics. Firearm-related injuries
affecting the pediatric population. Pediatrics. 2000;105:888-895. Available at:
http://pediatrics.aappublications.org/cgi/content/full/105/4/888

Coyne-Beasley T, Baccaglini L, Johnson RM, Webster B, Wiebe DJ. Do partners with children know
about firearms in their home? Evidence of a gender gap and implications for practitioners Pediatrics.
2005;115:e662-e667. DOI: 10.1542/peds.2004-2259. Available at:
http://pediatrics.aappublications.org/cgi/content/full/115/6/e662

Grossman DC, Mueller BA, Riedy C, et al. Gun storage practices and risk of youth suicide and
unintentional firearm injuries JAMA. 2005;293:707-714. DOI: 10.1001/jama.293.6.707. Available at:
http://jama.ama-assn.org/content/293/6/707.full.pdf+html

Guralnick S, Serwint JR. In brief: firearms. Pediatr Rev. 2007;28:396-397. DOI: 10.1542/pir.28-10-396.
Available at: http://pedsinreview.aappublications.org/cgi/content/full/28/10/396

Okoro CA, Nelson DE, Mercy JA, Balluz LS, Crosby AE, Mokdad AH. Prevalence of household firearms
and firearm-storage practices in the 50 states and the District of Columbia: FINDINGS FROM the
Behavioral Risk Factor Surveillance System, 2002. Pediatrics. 2005;116:e370-e376. DOI:
10.1542/peds.2005-0300. Available at: http://pediatrics.aappublications.org/cgi/content/full/116/3/e370

Teen homicide, suicide, and firearm deaths. Child Trends Data Bank. 2010. Available at:
http://www.childtrendsdatabank.org/?q=node/124

Copyright 2012 American Academy of Pediatrics


2012 PREP SA ON CD-ROM

Question 21
The mother of a 9-year-old boy in your practice requests a refill of his medications. He has a past
history of disruptive behavior, irritability, and impulsivity for which a psychiatrist prescribed
methylphenidate 10 mg three times a day and risperidone 0.5 mg daily. Both medications were initiated
about 18 months ago. His mother says he is now relatively asymptomatic and has been doing OK in
school for the past year The mother states that the psychiatrist did not share with her any specific
diagnoses for the boy and he is no longer available for consultation. You note that the boys weight has
increased from 25 kg (25th percentile) to 40 kg (95th percentile) over the past year, with his height
remaining at the 25th percentile (now 132 cm). His body mass index is now 23 (>95th percentile). You
counsel the family on nutritional management.
Of the following, the approach that is MOST likely to assist in his weight management is to
A. discontinue the risperidone
B. increase his methylphenidate dosage to a total of 40 mg daily
C. initiate metformin at 500 mg twice a day
D. initiate topiramate at 50 mg every night at bedtime
E. stop the risperidone and start olanzapine 2.5 mg daily

Copyright 2012 American Academy of Pediatrics


2012 PREP SA ON CD-ROM

Critique 21 Preferred Response: A


The weight gain experienced by the boy described in the vignette is most likely due to his use of
risperidone, which, like all atypical antipsychotics, commonly increases appetite in children. Significant
weight gain, as described for this boy, is a major complication of atypical antipsychotic use and
necessitates corrective action.
The most appropriate step is to stop the risperidone because its clinical benefits for him are
unknown and removing the drug will likely lead to normalization of his appetite with subsequent weight
loss. For the next month after discontinuing the risperidone he should be monitored for any re-emergence
of any emotional or behavioral symptoms, which would indicate that the risperidone had, in fact, been
producing clinical benefits. If he was taking a dosage of risperidone of around 1 mg daily or higher, then
first decreasing the medication dose by for 2-4 weeks before a planned discontinuation would generally
be advised. The higher the daily dosage of an antipsychotic, the more gradual the discontinuation taper
should be to give the process of discontinuation the greatest chance for success. Because antipsychotics
are associated with cumulative medical risks, such as tardive dyskinesia and the metabolic syndrome,
they generally should be used only as long as clinically necessary. Appetite stimulation from atypical
antipsychotics is generally not dose-related, so simply decreasing the risperidone dosage is unlikely to
cause a change in appetite.
Some patients have experienced weight loss with topiramate, but this has not been shown in
children, and the medication may cause adverse cognitive effects such as word-finding difficulties.
Metformin has been reported to reduce some weight gain from atypical antipsychotics, but its use creates
new medical risks, such as hypoglycemia and metabolic acidosis.
Use of stimulant drugs such as methylphenidate has been related to decreased appetite, but
several problems are associated with increasing this boys stimulant dose as a strategy to treat
antipsychotic-induced weight gain. Data about the utility of this approach are lacking, but more
importantly, increasing stimulant doses to higher than the maximum effective dose for treatment of
attention-deficit/hyperactivity disorder is very likely to cause other adverse effects, such as insomnia. In
addition, higher methylphenidate doses cannot be guaranteed to generate further appetite suppression
than might already be occurring at the 30-mg dose.
All atypical antipsychotics studied in children are associated with weight gain and olanzapine is
associated with significantly higher weight gain than the other atypical antipsychotics in children. For
those who require antipsychotic medication, switching to another agent of this class might be tried to
manage weight gain, but it is unlikely to be effective.

AAP Mental Health Competency:


Recognize there is no predicted weight neutral atypical antipsychotic for children.

Suggested Reading:
American Academy of Pediatrics Task Force on Mental Health. Addressing Mental Health Concerns in
Primary Care: A Clinician's Toolkit [CD-ROM]. Elk Grove Village, IL: American Academy of Pediatrics;
2010

Correll CU, Manu P, Olshanskiy V, Napolitano B, Kane JM, Malhotra AK. Cardiometabolic risk of second-
generation antipsychotic medications during first-time use in children and adolescents. JAMA.
2009;302:1765-1773. DOI: 10.1001/jama.2009.1549. Available at:
http://www.ncbi.nlm.nih.gov/pmc/articles/PMC3055794/?tool=pubmed

Maayan L, Correll CU. Management of antipsychotic-related weight gain. Expert Rev Neurother.
2010;10:1175-1200. DOI:10.1586/ern.10.85. Abstract available at:
http://www.ncbi.nlm.nih.gov/pubmed/20586697

Copyright 2012 American Academy of Pediatrics


2012 PREP SA ON CD-ROM

Question 22
A 3-year-old girl presents to the emergency department with irritability and weakness that was
followed by the development of nausea and vomiting and finally a seizure. Her mother reports that earlier
in the day she found the girl playing in the medicine cabinet but did not see her take any pills. Physical
examination reveals a toxic-appearing, febrile child who has hypertension, tachycardia, dilated pupils,
hyperreflexia, reduced muscle strength, and abdominal tenderness. You order a toxicology panel.
Of the following, the MOST likely cause of the childs clinical findings is an overdose of
A. acetaminophen
B. amphetamines
C. barbiturates
D. digoxin
E. narcotics

Copyright 2012 American Academy of Pediatrics


2012 PREP SA ON CD-ROM

Critique 22 Preferred Response: B


The girl described in the vignette is hypertensive and exhibits other typical signs and symptoms of
an amphetamine overdose. Amphetamines are powerful central nervous system stimulants that also have
peripheral adrenergic actions. The acute toxic effects due to overdose include hyperactive reflexes,
dilated pupils, talkativeness, irritability, weakness, and fever. In addition, the patient may experience
palpitations, tachycardia, hypertension, nausea, vomiting, diarrhea, and abdominal cramps. Severe
overdose is associated with seizure, coma, and stroke. A wide variety of prescription drugs (eg,
corticosteroids, combined oral contraceptives), over-the-counter drugs (eg, cough and cold medicines,
nonsteroidal anti-inflammatory drugs) and supplements (eg, caffeine-containing products, ginseng) can
elevate blood pressure.
Hepatotoxicity is the primary and most dangerous adverse effect of acetaminophen overdose.
Barbiturates are central nervous system sedative-hypnotic agents that primarily depress the level of
consciousness. They may cause normal or small pupils and uncommonly result in hypotension. Digoxin
has a narrow therapeutic index, and overdose is not uncommon at the higher end of the prescribed dose
range. Toxic effects may include palpitations due to atrial and ventricular arrhythmias, nausea and
vomiting, and visual disturbances that include blurred vision and xanthopsia (a disturbance of color vision
involving yellow and green). Electrocardiographic findings may include bradycardia, atrioventricular block,
and a highly reproducible change that results in slurring of the upstroke of the PR interval. Narcotic
ingestion causes sedation, analgesia, respiratory depression, pinpoint pupils, hypotension, nausea, and
vomiting.

American Board of Pediatrics Content Specification(s):


Recognize prescription, over-the-counter, and illicit drugs likely to elevate the blood pressure

Suggested Reading:
Coupey SM. Specific drugs. In: Schydlower M, ed. Substance Abuse. A Guide for Health Professionals.
2nd ed. Elk Grove Village, IL: American Academy of Pediatrics; 2002:191-276

Osterhoudt KC, Burns Ewald M, Shannon M, Henretig FM. Toxicologic emergencies. In: Fleisher GR,
Ludwig S, eds. Textbook of Pediatric Emergency Medicine. 6th ed. Philadelphia, PA, Lippincott Williams
& Wilkins, a Wolters Kluwer business; 2010:1171-1223

Copyright 2012 American Academy of Pediatrics


2012 PREP SA ON CD-ROM

Question 23
A 9-year-old boy who has acquired short stature presents to the emergency department with a
headache and vomiting. He had been one of the tallest boys in his class in the first grade but now is one
of the shortest. His clothing size has not changed in 18 months. Review of systems is positive for frequent
nocturnal urination and negative for fever, stomach pain, and diarrhea. On physical examination, the boy
appears well after vomiting, and general examination findings are within normal parameters. On
neurologic examination, he converses appropriately. Visual field testing shows an apparent inability to
count fingers in either lateral visual field. Extraocular movements are full, facial movements are
symmetric, and tongue and palate movements are normal. Strength and reflexes are normal in all limbs.
His gait is normal and not broad-based. Brain magnetic resonance imaging shows a midline mass with
both cystic and enhancing solid components (Item Q23).
Of the following, the MOST likely explanation of these findings is
A. brain abscess
B. craniopharyngioma
C. meningioma
D. primitive neuroectodermal tumor
E. subependymal giant cell astrocytoma

Copyright 2012 American Academy of Pediatrics


2012 PREP SA ON CD-ROM

Question 23

(Courtesy of M Sutton)
T1-weighted sagittal MRI, as described for the boy in the vignette. There is a suprasellar mass (defined
by yellow arrows). The inferior portion is solid and partially calcified (red arrow) and the superior portion
is cystic and fluid-filled (blue arrow).

Copyright 2012 American Academy of Pediatrics


2012 PREP SA ON CD-ROM

Critique 23 Preferred Response: B


The child described in the vignette presents with vomiting and headache but no other signs of
acute illness. This raises a concern for elevated intracranial pressure, and careful neurologic examination
is warranted. A key finding on his examination is a bitemporal hemianopsia or loss of vision in lateral
fields on both sides. Taken together with the acquired short stature (possibly due to growth hormone
deficiency or central hypothyroidism) and frequent urination (possibly due to deficient antidiuretic
hormone), this suggests the presence of a slow-growing, suprasellar lesion pressing on the optic chiasm
(causing the visual field loss) and involving the hypothalamus or pituitary. The mass seen on magnetic
resonance imaging (MRI), which has both solid (inferior) and cystic (superior) components, is consistent
with a craniopharyngioma. Clarification may be seen on computed tomography (CT) scan.
Craniopharyngiomas are slow-growing tumors that often present in the first decade. The cell source is
believed to be embryonic epithelial cells in the region of Rathkes cleft. The typical presentation of such
tumors involves headaches, visual findings, and endocrinopathies. A critical point is that growth failure
may be one of the first signs of this tumor. If the condition is identified early, treatment is more successful,
with less long-term visual impairment, endocrine dysfunction, or cognitive disability. Treatment is surgery
with or without radiation.
Brain abscess can present somewhat indolently with fever and encephalopathy; it should not
present with growth failure. MRI or CT scan with contrast should demonstrate ring enhancement (Item
C23A). Meningiomas are slow-growing tumors, but they do not have a cystic component and are much
less common than craniopharyngiomas in childhood.
Primitive neuroectodermal tumors are the most common malignant childhood brain tumors. Of
note, 85% occur in the cerebellar vermis, where they are designated medulloblastomas (Item C23B).
Therefore, they tend to present with ataxia as well as signs of elevated intracranial pressure due to
obstructive hydrocephalus. They are solid, without cystic components. Findings on the examination in this
case do not support the presence of a cerebellar lesion.
Subependymal giant cell astrocytomas are found along the walls of the lateral ventricles (Item
C23C); they are not suprasellar. They occur in patients who have tuberous sclerosis complex. They can
cause hydrocephalus and headache but not the constellation of visual and growth symptoms described in
the vignette. Treatment with mammalian target of rapamycin (mTOR) inhibitors shrinks these tumors,
reducing complications such as hydrocephalus.

American Board of Pediatrics Content Specification(s):


Recognize the signs and symptoms of craniopharyngioma

Suggested Reading:
Garr ML, Cama A. Craniopharyngioma: modern concepts in pathogenesis and treatment. Curr Opin
Pediatr. 2007;19:471-479. Abstract available at: http://www.ncbi.nlm.nih.gov/pubmed/17630614

Krueger DA, Care MM, Holland K, et al. Everolimus for subependymal giant-cell astrocytomas in tuberous
sclerosis. N Engl J Med. 2010;363:1801-1811. Available at:
http://www.ncbi.nlm.nih.gov/pubmed/21047224

Kuttesch J Jr, Zieber Rush S, Ater JL. Brain tumors in childhood. In: Kliegman RM, Stanton BF, St. Geme
JW III, Schor NF, and Behrman RE, eds. Nelson Textbook of Pediatrics. 19th ed. Philadelphia, PA:
Saunders Elsevier; 2011:1746-1753

Lin LL, El Naqa I, Leonard JR, et al. Long-term outcome in children treated for craniopharyngioma with
and without radiotherapy. J Neurosurg Pediatr. 2008;1:126-130. DOI: 10.3171/PED/2008/1/2/126.
Available at: http://thejns.org/doi/full/10.3171/2010.1.FOCUS09297

Maity A, Pruitt AA, Judy KD, Phillips PC, Lustig R. Cancer of the central nervous system. In: Abeloff MD,
Armitage JO, Niederhuber JE, Kastan MB, McKenna WG, eds. Abeloff's Clinical Oncology. 4th ed.
Philadelphia, PA: Churchill Livingstone, an Imprint of Elsevier, 2008:chapter 70

Copyright 2012 American Academy of Pediatrics


2012 PREP SA ON CD-ROM

Critique 23

(Courtesy of D Mulvihill)
Sagittal T1-weighted MRI demonstrates a low-attenuation lesion in the parietal lobe, with a contrast-
enhancing rim and surrounding edema.

Copyright 2011 American Academy of Pediatrics


2012 PREP SA ON CD-ROM

Critique 23

(Courtesy of D Krueger)
Sagittal T1weighted MRI following contrast shows a heterogeneous, enhancing solid lesion in the
midline cerebellar vermis (arrows) consistent with a medulloblastoma.

Copyright 2011 American Academy of Pediatrics


2012 PREP SA ON CD-ROM

Critique 23

(Courtesy of D Krueger)
Coronal MRI demonstrating a subependymal giant cell astrocytoma. These lesions typically appear in
persons who have tuberous sclerosis complex and arise at the foramina of Monro where they may cause
obstructive hydrocephalus.

Copyright 2011 American Academy of Pediatrics


2012 PREP SA ON CD-ROM

Question 24
During the health supervision visit of a 5-year-old girl, her 37year-old mother reports that she
and her husband have decided to have another child. She states that since the birth of her daughter, she
has been diagnosed with type 2 diabetes and requires insulin twice a day to control her blood glucose
concentrations. She is monitoring her blood glucose carefully, and her hemoglobin A1C is less than 7.5%.
She asks if her diabetes poses any additional risks for her pregnancy.
Of the following, the MOST helpful piece of information that she should have before becoming
pregnant is that
A. excellent maternal blood glucose control will prevent neonatal hypoglycemia and macrosomia
B. her greatest risk of having a child with a birth defect is her age
C. her risk of having a child with a birth defect is similar to a woman who has gestational diabetes
D. second-trimester level II ultrasonography is recommended
E. the most common birth defect occurs in the kidney

Copyright 2012 American Academy of Pediatrics


2012 PREP SA ON CD-ROM

Critique 24 Preferred Response: D


Preexisting diabetes currently affects approximately 1% of pregnancies and includes both type 1
and type 2 diabetes mellitus. Despite relatively good glycemic control, women who have preexisting
diabetes, such as the woman described in the vignette, still face increased risks for congenital
malformations and should be offered second-trimester ultrasonography to look for serious malformations
in the developing fetus. The most common congenital anomalies occurring in infants of diabetic mothers
(IDMs) involve the heart and central nervous system. Although many types of heart defects are seen in
IDMs, there is overrepresentation of certain types of cardiac malformations, including transposition of the
great vessels, truncus arteriosus, tricuspid atresia, and hypoplastic left heart, which may be identified in
the second trimester via fetal echocardiography. Therefore, fetal echocardiography as well as level II
ultrasonography in the second trimester of pregnancy may be useful in identifying significant fetal
anomalies.
Neural tube defects also occur much more frequently than in the general population, and
gastrointestinal, genitourinary, and skeletal anomalies also occur frequently. Macrosomia is seen in 15%
to 45% of IDMs, which represents an approximate threefold increase over the nondiabetic population.
The risk for neonatal hypoglycemia in IDMs is five times that of the general population, and there is a
twofold increase in risks for neonatal jaundice. Fetal lung maturity lags in IDMs compared with
nondiabetic pregnancies, with more than 99% of infants lung maturation completed by 37 weeks
gestation compared to 38.5 weeks gestation for IDMs. To date, even excellent diabetic control cannot
completely eliminate such increased risks for the IDM.
The risk of chromosomal abnormalities also is increased in IDMs. For example, the risk for having
a child affected by a chromosomal abnormality such as Down syndrome or any chromosomal trisomy if a
mother is age 37 at delivery is 1 in 227 but increases to 1 in 127 if the mother also has diabetes. For a
woman who is 38 years of age at delivery, the risk is 1 in 175 but increases to 1 in 102 if she has
diabetes. Risks for birth defects (which include heart, brain, spine, kidney, gastrointestinal, and limb
defects) in the general nondiabetic population are approximately 1.5%; risks for birth defects in the IDM
had been as high as 18% in previous decades, dropping to between 5.1% and 9.8% with more attention
devoted to preconceptional intervention and glucose control in recent years. However, the risk for birth
defects is still at least three to six times greater than for the nondiabetic population. To put the problem of
maternal diabetes in perspective, of all infants born with a single congenital anomaly, 1 in 50 is born to a
woman who has diabetes and 1 in 20 infants who have multiple congenital anomalies is born to a woman
who has diabetes. Although there are recognized risks for certain problems associated with gestational
diabetes, such as macrosomia, respiratory distress, and even subtle neurobehavioral difficulties in
attention span as well as fine and gross motor abilities, the increased risks for birth defects lie primarily
with those women who have preexisting diabetes in the first trimester, which leads to altered
organogenesis and secondary birth defects.
Maternal diabetes is associated with increased risks for stillbirth as well as neonatal mortality and
morbidity, with poorly controlled diabetes conferring the greatest risks for adverse outcome. Therefore,
excellent preconceptional glucose control can improve neonatal outcome but cannot yet completely
mitigate the effects of maternal diabetes. The rate of miscarriage among women who have diabetes
continues to be high at 9% to 14%; women who have a history of diabetes for more than 10 years and
hemoglobin A1C values greater than 11% have a miscarriage risk as high as 44%.

American Board of Pediatrics Content Specification(s):


Recognize that congenital anomalies are more frequent among infants of diabetic mothers than
among normal control infants

Suggested Reading:
Boinpally T, Jovanovic L. Management of type 2 diabetes and gestational diabetes in pregnancy. Mt Sinai
J Med. 2009;76:269-280. DOI: 10.1002/msj.20115. Available at:
http://onlinelibrary.wiley.com/doi/10.1002/msj.20115/pdf

de Valk HW, van Nieuwaal NH, Visser GH. Pregnancy outcome in type 2 diabetes mellitus: a
retrospective analysis from the Netherlands. Rev Diabet Stud. 2006;3:134-142. DOI:

Copyright 2012 American Academy of Pediatrics


2012 PREP SA ON CD-ROM

10.1900/RDS.2006.3.134. Available at:


http://www.ncbi.nlm.nih.gov/pmc/articles/PMC1783588/?tool=pubmed

Dunne F, Brydon P, Smith K, Gee H. Pregnancy in women with type 2 diabetes: 12 years outcome data
1990-2002. Diabet Med. 2003;20:734-738. DOI: 10.1046/j.1464-5491.2003.01017.x. Abstract available
at: http://www.ncbi.nlm.nih.gov/pubmed/12925053

Copyright 2012 American Academy of Pediatrics


2012 PREP SA ON CD-ROM

Question 25
A sexually active girl presents with a slightly itchy, yellowish discharge and occasional
dyspareunia. She says she is in a monogamous relationship, so they do not use condoms. Her boyfriend
has no symptoms. Results of her external genital examination are normal. On speculum examination, the
vaginal walls appear erythematous and a frothy discharge is present. The pH is 4.7, and you perform wet
mount microscopy (Item Q25).
Of the following, the MOST likely cause of this girls vaginal discharge is
A. bacterial vaginosis
B. Candida vaginalis
C. Chlamydia trachomatis
D. nonspecific vaginitis
E. trichomoniasis

Copyright 2012 American Academy of Pediatrics


2012 PREP SA ON CD-ROM

Critique 25 Preferred Response: E


Vaginal discharge may be the presenting symptom of vaginitis or cervicitis; it may or may not be
the result of an infection with a sexually transmitted agent. The findings described for the patient in the
vignette suggest a Trichomonas vaginalis infection. Although this infection may be asymptomatic (more
often in males), it usually presents with itching, dysuria, dyspareunia, lower abdominal pain, and
postcoital bleeding. The incubation period is 4 to 28 days. The onset of symptoms may be with
menstruation or soon thereafter, perhaps related to a change in vaginal acidity. On speculum
examination, the discharge, which may be white, yellowish, or greenish, appears frothy, can be very
copious, and has an elevated pH. In addition, vaginal wall inflammation and, occasionally, punctuate
hemorrhages are present on the cervix. The wet mount shows the organism T vaginalis (Item C25A). In
severe infections, there may be edema and excoriation of the external genitalia and occasionally
bartholinitis.
Bacterial vaginosis, which may not be sexually transmitted, results in a malodorous, grayish
discharge. Because it is noninflammatory, it is referred to as vaginosis and not vaginitis. It is not
associated with pain or vulvar or vaginal wall redness. Complaints of an itch are often seen with a
Candida infection, but the discharge is usually white rather than yellow and can be milky or thick and
clumpy. When infection is severe, there may be complaints of vaginal soreness, pain with intercourse,
and external dysuria. Speculum examination may show vulvar redness, edema, excoriations, and satellite
lesions, and a wet mount shows pseudohyphae or spores (Item C25B). Chlamydia trachomatis causes
cervicitis rather than vaginitis and may be asymptomatic. There are no vulvar or vaginal findings. The
cervix may be inflamed, with a purulent discharge from the os, and friable. Nonspecific vaginitis results
from contact of the vaginal mucosa with certain chemicals that cause symptoms of vaginal burning or
swelling. Although there might be slight redness of the vaginal mucosa, the discharge is normal on
microscopy.

American Board of Pediatrics Content Specification(s):


Recognize the clinical manifestations of Trichomonas vaginalis infection

Suggested Reading:
Centers for Disease control and Prevention. Sexually Transmitted Diseases (STDs): Trichomoniasis.
2010. Available at: http://www.cdc.gov/std/trichomonas/default.htm

Hollman D, Coupey SM, Fox AS, Herold BC. Screening for Trichomonas vaginalis in high-risk adolescent
females with a new transcription-mediated nucleic acid amplification test (NAAT): associations with
ethnicity, symptoms, and prior and current STIs. J Pediatr Adolesc Gynecol. 2010;23:312-316. DOI:
10.1016/j.jpag.2010.03.004. Available at: http://www.ncbi.nlm.nih.gov/pubmed/20493735

Hwang LY, Shafer M-AB. Vaginitis and vaginosis. In: Neinstein LS, Gordon CM, Katzman DK, Rosen DS,
Woods ER, eds. Adolescent Health Care: A Practical Guide. 5th ed. Philadelphia, PA: Lippincott Williams
& Wilkins, a Wolters Kluwer business; 2008:729-731

Kaufman M and the Committee on Adolescence. Care of the adolescent sexual assault victim. Pediatrics.
2008;122:462-470. DOI: 10.1542/peds.2008-1581. Available at:
http://pediatrics.aappublications.org/cgi/content/full/122/2/462

Kokotos F, Adam HM. In brief: vulvovaginitis. Pediatr Rev. 2006;27:116117. DOI: 10.1542/pir.27-3-116.
Available at: http://pedsinreview.aappublications.org/cgi/content/full/27/3/116

Woods ER, Emans SJ. Vulvovaginal complaints in the adolescent. In: Emans SJH, Laufer MR, Goldstein
DP, eds. Pediatric and Adolescent Gynecology. 5th ed. Philadelphia, PA: Lippincott Williams & Wilkins, a
Wolters Kluwer business; 2005:525-564

Copyright 2012 American Academy of Pediatrics


2012 PREP SA ON CD-ROM

Critique 25

(Courtesy of D Krowchuk)
Normal saline wet preparation in vaginal candidiasis showing pseudohyphae (red arrow) and spores
(yellow arrows).

Copyright 2011 American Academy of Pediatrics


2012 PREP SA ON CD-ROM

Question 26
You are called to evaluate the abnormal laboratory findings for a 16-year-old boy who you
admitted to the hospital yesterday for a presumed community-acquired pneumonia. The boy had been
reported to be improving on the antibiotics you ordered, but his serum electrolyte results were abnormal
this morning, so the resident on call obtained additional laboratory studies. On physical examination, the
young man is alert and complains of no distress. His temperature is 37.0C, heart rate is 80 beats/min,
respiratory rate is 15 breaths/min, and blood pressure is 120/80 mm Hg. He reports that he has not
urinated since yesterday afternoon. His skin turgor is normal, and you do not notice jugular venous
distension or signs of edema. Results of his laboratory studies include:
Serum sodium, 125 mEg/L (125 mmol/L)
Serum potassium, 4 mEq/L (4 mmol/L)
Serum chloride, 95 mEq/L (95 mmol/L)
Serum bicarbonate, 25 mEq/L (25 mmol/L)
Blood urea nitrogen, 10 mg/dL (3.6 mmol/L)
Serum creatinine, 1.0 mg/dL (88.4 mcmol/L)
Serum glucose, 100 mg/dL (5.6 mmol/L)
Serum osmolality, 260 mOsm/kg
Urine osmolality, 500 mOsm/kg
Urine sodium, 30 mEq/L (30 mmol/L)
Of the following, the MOST likely cause of the patients hyponatremia is
A. acute renal failure
B. congestive heart failure
C. diabetes insipidus
D. mineralocorticoid deficiency
E. syndrome of inappropriate antidiuretic hormone

Copyright 2012 American Academy of Pediatrics


2012 PREP SA ON CD-ROM

Critique 26 Preferred Response: E


Disorders of sodium homeostasis are frequent in the hospitalized pediatric patient. Hypernatremia
(serum sodium >145 mEq/L [145 mmol/L]) and hyponatremia (serum sodium <135 mEq/L [135 mmol/L])
can result from a variety of disease processes and are often complicated by the administration of
intravenous fluids. An important cause of hyponatremia is the syndrome of inappropriate antidiuretic
hormone secretion (SIADH), which is characterized by euvolemic hyponatremia, decreased urine output,
decreased serum osmolality, and increased urine osmolality, as described for the boy in the vignette.
Common causes of SIADH are shown in (Item C26).
Clinical symptoms of SIADH include lethargy, changes in mental status, and seizures. Treatment
consists of fluid restriction and close monitoring of fluid volume and electrolytes. Patients who have acute
neurologic changes, such as seizures or coma that are due to severe hyponatremia (<120 mEq/L [120
mmol/L]) should be treated with 3% hypertonic saline to raise their serum sodium concentrations
approximately 6 to 10 mEq/L (6 to 10 mmol/L).
Hypervolemic hyponatremia is characterized by a relative greater increase in total body water
content compared with total sodium body content and is manifested by clinical signs such as jugular
venous distension and generalized edema. Common causes of hypervolemic hyponatremia, such as
renal failure and congestive heart failure, are unlikely in this patient. Hypovolemic dehydration results
from a loss of both sodium and water, although the sodium loss is relatively greater. The resultant
dehydration is characterized by clinical signs such as decreased perfusion and tachycardia. Common
causes include both extrarenal losses (diarrhea, vomiting, and ileostomy) and renal losses (thiazide
diuretic use, cerebral salt wasting, and mineralocorticoid deficiency). Patients who have mineralocorticoid
deficiency usually present with signs of cardiovascular insufficiency, including shock. Diabetes insipidus
presents with hypernatremic hypovolemia, not hyponatremia.

American Board of Pediatrics Content Specification(s):


Recognize disease conditions and medications associated with SIADH

Suggested Reading:
Hauser GJ, Kulick AF. Electrolyte disorders in the pediatric intensive care unit. In: Wheeler DS, Wong HR,
Shanley TP, eds. Pediatric Critical Care Medicine: Basic Science and Clinical Evidence. New York, NY:
Springer-Verlag; 2007:1156-1175

Hoorn EJ, Geary D, Robb M, Halperin ML, Bohn D. Acute hyponatremia related to intravenous fluid
administration in hospitalized children: an observational study. Pediatrics. 2004;113;1279-1284. Available
at: http://pediatrics.aappublications.org/cgi/content/full/113/5/1279

Montaana PA, Modesto I Alapont V, Ocn AP, Lpez PO, Lpez Prats JL, Toledo Parreo JD. The use
of isotonic fluid as maintenance therapy prevents iatrogenic hyponatremia in pediatrics: a randomized,
controlled open study. Pediatr Crit Care Med. 2008;9;589-597. DOI: 10.1097/PCC.0b013e31818d3192.
Available at: http://www.ncbi.nlm.nih.gov/pubmed/18838929

Moritz ML, Ayus JC. Disorders of water metabolism in children: hyponatremia and hypernatremia. Pediatr
Rev. 2002;23;371-380. DOI: 10.1542/10.1542/pir.23-11-371. Available at:
http://pedsinreview.aappublications.org/cgi/content/full/23/11/371

Copyright 2012 American Academy of Pediatrics


2012 PREP SA ON CD-ROM

Critique 26

Item C26. Causes of SIADH


Central Nervous System Disorders
! Infection: meningitis, encephalitis
! Neoplasms
! Vascular abnormalities
! Psychosis
! Hydrocephalus
! Postpituitary surgery
Pulmonary Disorders
! Pneumonia
! Tuberculosis
! Asthma
! Positive pressure ventilation
! Pneumothorax
Carcinomas
! Bronchogenic carcinomas
! Oat cell of the lung
! Duodenal
! Pancreatic
! Neuroblastoma
Medications
! Vincristine
! Intravenous cyclophosphamide
! Carbamazepine
! Serotonin reuptake inhibitors
Reprinted with permission from Moritz ML, Ayus JC. Disorders of water
metabolism in children: hyponatremia and hypernatremia. Pediatr Rev.
2002;23:371-380

Copyright 2011 American Academy of Pediatrics


2012 PREP SA ON CD-ROM

Question 27
A 15-year-old girl presents to your clinic for evaluation of primary amenorrhea. She has a history
of mild seasonal allergies and a right radius fracture at age 9. Her mothers height is 5 ft 9 in and fathers
height is 6 ft 2 in. The mother reports that her menarche occurred at 11 years, and the father recalls
shaving at 14 years. On physical examination, the girl has Sexual Maturity Rating (SMR) 3 pubic hair and
SMR 1 breast development. Bone age radiography shows a skeletal maturity of 13 years. No other
abnormalities are noted. Her growth curve is shown (Item Q27).
Of the following, the MOST appropriate next step in this girls evaluation and treatment is to
A. initiate oral conjugated estrogen therapy
B. measure serum estradiol
C. measure serum thyroid-stimulating hormone
D. obtain a karyotype
E. perform a bimanual examination

Copyright 2012 American Academy of Pediatrics


2012 PREP SA ON CD-ROM

Question 27

(Courtesy of M Haller)

Copyright 2012 American Academy of Pediatrics


2012 PREP SA ON CD-ROM

Critique 27 Preferred Response: D


The differential diagnosis of primary amenorrhea includes constitutional delay, eating disorders,
chronic disease (typically causing low weight for height), severe androgen resistance, Rokitansky
syndrome (absence of uterus), imperforate hymen, gonadotropin deficiency, and primary ovarian failure.
Based on the short stature, growth pattern, and absence of breast development in the presence of pubic
hair described for the girl in the vignette, gonadal failure due to Turner syndrome should be considered
and a karyotype obtained. Indeed, the most common cause of primary ovarian failure in otherwise healthy
girls is Turner syndrome. Initiating estrogen therapy in a patient without establishing a clear diagnosis is
unwise. Measurement of serum estradiol is unnecessary because the finding of breast Sexual Maturity
Rating 1 indicates absence of pubertal estradiol concentrations. Although thyroid disease can cause
menstrual irregularities, other symptoms and signs would be expected, making the measurement of
thyroid-stimulating hormone unnecessary. A bimanual examination is not required in the first stages of an
amenorrhea evaluation when Turner syndrome is suspected and could be distressing for this girl.
Classic Turner syndrome occurs in girls who have a complete absence of one X chromosome
(karyotype 45XO). However, a large percentage of affected girls have mosaic presentations, meaning
they have a mix of 45XO and 46XX chromosomes when multiple cells are examined. Such girls often
have less of the characteristic stigmata of Turner syndrome (eg, shield chest, webbed neck, low posterior
hair line, short stature). Girls who have mosaic Turner syndrome are more likely to present with primary
amenorrhea if they are not diagnosed on the basis of the more classic physical features of the condition.
Of note for the girl in the vignette, her parents are both tall, providing a mid-parental height of 5 ft
9 in, which is approximately the 95th percentile for adult women, but the patient is growing at only the 5th
percentile on a standard growth curve. It is important to consider parental height even when children
maintain normal growth velocity. The 95th percentile for height on a Turner syndrome growth curve nearly
overlaps with the 5th percentile for height in unaffected girls. Accordingly, girls who have Turner
syndrome rarely reach the 5th percentile without intervention such as growth hormone or oxandrolone
therapy. Growth hormone therapy is approved for short stature in girls who have Turner syndrome, even
though they do not have growth hormone deficiency. In this case, the girls above-average mid-parental
height contributed to her ability to reach a height within the reference range for unaffected girls.

American Board of Pediatrics Content Specification(s):


Know that gonadal dysgenesis is uniformly present in Turner syndrome

Suggested Reading:
Bondy CA; Turner Syndrome Study Group. Care of girls and women with Turner syndrome: a guideline of
the Turner Syndrome Study Group. J Clin Endocrinol Metab. 2007;92:10-25. Abstract available at:
http://www.ncbi.nlm.nih.gov/pubmed/17047017

Fras JL, Davenport ML, Committee on Genetics, Section on Endocrinology. Health supervision for
children with Turner syndrome. Pediatrics. 2003;111:692-702. Available at:
http://pediatrics.aappublications.org/cgi/content/full/111/3/692

Copyright 2012 American Academy of Pediatrics


2012 PREP SA ON CD-ROM

Question 28
During a health supervision visit, a boy calls to his father by saying dada. His mother enters the
room holding a snack. The child reaches out to her and cries loudly mama. When the boy notices his
mother is holding a banana, he smiles and says nana. His mother picks him up and offers him the
banana. You inquire if he is saying any other words. His mother replies not yet.
Of the following, these findings are MOST expected for a typically developing child who is
A. 9 months old
B. 12 months old
C. 15 months old
D. 18 months old
E. 24 months old

Copyright 2012 American Academy of Pediatrics


2012 PREP SA ON CD-ROM

Critique 28 Preferred Response: B


The child described in the vignette exhibits the normal cognitive/behavioral developmental
milestones for 12 months of age: says mama and dada with meaning and says at least one additional
specific word. A 9-month-old child imitates speech sounds, babbles, says mama nonspecifically, follows
a pointed finger, and recognizes familiar people. At 15 months of age, a child uses three to five words,
has mature jargoning, greets people, and shows empathy. An 18 month old uses 10 to 25 words, names
a picture on demand, and engages in pretend play with others. A 24-month-old child has a vocabulary of
more than 50 words, speaks in two-word sentences, and engages in parallel play. Early identification of
developmental disorders is crucial to a childs well-being. In addition to routine developmental
surveillance, a developmental screening tool should be administered regularly at the 9-, 18-, and either
the 24- or 30-month health supervision visits.

American Board of Pediatrics Content Specification(s):


Recognize the normal cognitive/behavioral developmental milestones for 12 months of age (eg, says
"mama" and "dada" with specific meaning; says at least one specific word in addition to "mama" and
"dada")

Suggested Reading:
Council on Children with Disabilities, Section on Developmental Behavioral Pediatrics, Bright Futures
Steering Committee, Medical Home Initiatives for Children With Special Needs Project Advisory
Committee. Identifying infants and young children with developmental disorders in the medical home: an
algorithm for developmental surveillance and screening. Pediatrics. 2006;118:405-420. Available at:
http://pediatrics.aappublications.org/content/118/1/405

Feigelman S. The first year. In: Kliegman RM, Stanton BF, St. Geme JW III, Schor NF, and Behrman RE,
eds. Nelson Textbook of Pediatrics. 19th ed. Philadelphia, PA: Saunders Elsevier; 2011:26-31

First Signs web site. Available at: http://www.firstsigns.org

OConner Leppert ML. Neurodevelopmental assessment and medical evaluation. In: Voight RG, Macias
MM. Myers SM, eds. American Academy of Pediatrics Developmental and Behavioral Pediatrics. Elk
Grove Village, IL: American Academy of Pediatrics; 2011: 93-120

Copyright 2012 American Academy of Pediatrics


2012 PREP SA ON CD-ROM

Question 29
An 8-year-old boy has been hospitalized for 2 days with Neisseria meningitidis meningitis that has
been confirmed by culture. He is responding well to intravenous antibiotic therapy. He has a healthy 8-
year-old twin brother.
Of the following, the BEST choice for prophylaxis for his twin brother is
A. azithromycin
B. ciprofloxacin
C. penicillin VK
D. rifampin
E. trimethoprim-sulfamethoxazole

Copyright 2012 American Academy of Pediatrics


2012 PREP SA ON CD-ROM

Critique 29 Preferred Response: D


Prophylaxis is recommended for close contacts of a child who has invasive meningococcal
infection. Rifampin administered orally in four doses over 2 days is an appropriate prophylactic regimen
for the twin brother described in the vignette. Other prophylactic regimens include a single intramuscular
dose of ceftriaxone or a single oral dose of ciprofloxacin, although fluoroquinolones are not routinely
indicated for children if safer alternatives exist.
Intravenous penicillin G is effective for treatment of invasive meningococcal infections, but oral
penicillin VK has not been shown to be effective for prophylaxis of this infection. Azithromycin and
trimethoprim-sulfamethoxazole have not demonstrated effectiveness as prophylactic agents for exposure
to meningococcal infections.
Rifampin is also indicated for prophylaxis after exposure to invasive Haemophilus influenzae type
b infection if there is a susceptible child (eg, inadequately immunized or immunocompromised) younger
than 4 years of age in the household but with a different dosing schedule (once daily for 4 days).
Other indications for rifampin in pediatrics include use:
1. as a part of standard tuberculosis treatment regimens may be used as an alternative regimen for
preventive therapy when isoniazid is not an option.
2. in combination with vancomycin or a beta-lactam antibiotic in certain staphylococcal infections (eg,
ventriculoperitoneal shunt infections, osteomyelitis, endocarditis).
3. in combination with a beta-lactam antibiotic in an attempt to clear persistent group A streptococcal
pharyngitis.
4. in select instances in an attempt to eradicate methicillin-resistant Staphylococcus aureus carriage.

American Board of Pediatrics Content Specification(s):


Know the appropriate use of rifampin

Suggested Reading:
American Academy of Pediatrics. Haemophilus influenzae infections. In: Pickering LK, Baker CJ,
Kimberlin DW, Long SS, eds. Red Book: 2009 Report of the Committee on Infectious Diseases. 28th ed.
Elk Grove Village, IL: American Academy of Pediatrics; 2009: 314-321

American Academy of Pediatrics. Meningococcal infections. In: Pickering LK, Baker CJ, Kimberlin DW,
Long SS, eds. Red Book: 2009 Report of the Committee on Infectious Diseases. 28th ed. Elk Grove
Village, IL: American Academy of Pediatrics; 2009:455-463

American Academy of Pediatrics. Staphylococcal infections. In: Pickering LK, Baker CJ, Kimberlin DW,
Long SS, eds. Red Book: 2009 Report of the Committee on Infectious Diseases. 28th ed. Elk Grove
Village, IL: American Academy of Pediatrics; 2009:601-615

American Academy of Pediatrics. Tuberculosis. In: Pickering LK, Baker CJ, Kimberlin DW, Long SS, eds.
Red Book: 2009 Report of the Committee on Infectious Diseases. 28th ed. Elk Grove Village, IL:
American Academy of Pediatrics; 2009: 680-701

Drew RH. Rifampin and other rifamycins. UpToDate Online 18.3. 2010. Available for subscription at:
http://www.uptodate.com/online/content/topic.do?topicKey=antibiot/8824

Copyright 2012 American Academy of Pediatrics


2012 PREP SA ON CD-ROM

Question 30
A 15-year-old girl presents with a 2-day history of abdominal pain, fever, and worsening emesis.
On physical examination, her temperature is 40.0C, heart rate is 115 beats/min, and respiratory rate is
25 breaths/min. Abdominal examination reveals diffuse rebound tenderness greatest in the lower
quadrants and decreased bowel sounds. On gynecologic examination, you note purulent endocervical
3
discharge and acute cervical motion and adnexal tenderness. The white blood cell count is 25.0x10 /mcL
9
(25.0x10 /L), with 75% polymorphonuclear leukocytes, 20% lymphocytes, and 5% monocytes. The
patient has a history of hives and shortness of breath after receiving ceftriaxone for a sexually transmitted
infection.
Of the following, the MOST appropriate initial antimicrobial therapy for this girl is
A. ampicillin-sulbactam plus doxycycline
B. cefotetan plus doxycycline
C. clindamycin plus gentamicin
D. levofloxacin plus metronidazole
E. trimethoprim-sulfamethoxazole plus metronidazole

Copyright 2012 American Academy of Pediatrics


2012 PREP SA ON CD-ROM

Critique 30 Preferred Response: C


The girl described in the vignette has pelvic inflammatory disease (PID), characterized by
purulent endocervical discharge and cervical and adnexal motion tenderness. Antimicrobial therapy for
PID should be initiated promptly in all suspected cases to minimize inflammatory sequelae in the upper
reproductive tract. Empirical treatment should include broad-spectrum antibiotics that cover Neisseria
gonorrhoeae and Chlamydia trachomatis, regardless of endocervical screening results, as well as
anaerobes. Clindamycin and gentamicin are recommended for the treatment of PID in patients who
require inpatient management and cannot tolerate cephalosporin therapy, such as the girl in the vignette,
who reported hives and respiratory distress following receipt of ceftriaxone. This treatment regimen
requires that the patient receive 14 days of oral doxycycline following the intravenous treatment course.
Clindamycin is a lincosamide antibiotic that reversibly binds to the bacterial 50S ribosomal
subunit and inhibits peptide bond formation and subsequent bacterial protein synthesis. It is bacteriostatic
or bactericidal, depending on infection site, pathogen, and concentration of drug. It is active against
anaerobic organisms, such as Bacteroides, and is useful for the treatment of genitourinary and intra-
abdominal infections. It also has activity against most staphylococci and streptococci (except enterococci)
and, therefore, is useful for the treatment of respiratory tract infections, skin and soft-tissue infections, and
sepsis.
Patients who have significant hypersensitivity reactions to beta-lactam drugs should be treated
with nonbeta-lactam antibiotics. Therefore, the patient in the vignette should not receive ampicillin-
sulbactam or cefotetan, although either of these drugs in combination with doxycycline could be used for
the treatment of PID in those who have no significant allergy to beta-lactam agents. Levofloxacin (for N
gonorrhoeae and C trachomatis) in combination with metronidazole (for anaerobes) is acceptable
treatment for PID. However, due to the increasing prevalence of quinolone-resistant N gonorrhoeae in
many parts of the world, including areas of the United States, quinolones should be reserved for cases in
which culture and susceptibility testing of clinical specimens has been performed. Trimethoprim-
sulfamethoxazole is inadequate for the treatment of PID.
The Centers for Disease Control and Prevention recommend prompt empiric treatment of PID in
sexually active women if they have pelvic or lower abdominal pain, no other explanation for their
symptoms, and one or more of the following minimum criteria on physical examination:
Cervical motion tenderness
Uterine tenderness
Adnexal tenderness
The presence of fever (temperature >38.3C), elevated inflammatory markers (erythrocyte
sedementation rate or C-reactive protein), numerous white blood cells on microscopy of vaginal
secretions, purulent cervical discharge, or laboratory confirmation of N gonorrhoeae or C trachomatis
cervical infection can further support the diagnosis of PID. Endometrial biopsy, transvaginal
ultrasonography, or magnetic resonance imaging and laparoscopy can provide the most specific criteria
for diagnosing PID but only are warranted in select cases. Hospitalization of patients who have PID
should be considered for pregnant women, those unable to tolerate oral antibiotics (such as the patient in
the vignette who has worsening emesis), those who have high fever, and those experiencing severe
illness, including tubo-ovarian abscess.

American Board of Pediatrics Content Specification(s):


Know the appropriate use of clindamycin

Suggested Reading:
American Academy of Pediatrics. Chlamydia trachomatis. In: Pickering LK, Baker CJ, Kimberlin DW,
Long SS, eds. Red Book: 2009 Report of the Committee on Infectious Diseases. 28th ed. Elk Grove
Village, IL: American Academy of Pediatrics; 2009:255-259

American Academy of Pediatrics. Gonococcal infections. In: Pickering LK, Baker CJ, Kimberlin DW, Long
SS, eds. Red Book: 2009 Report of the Committee on Infectious Diseases. 28th ed. Elk Grove Village, IL:
American Academy of Pediatrics; 2009:305-313

Copyright 2012 American Academy of Pediatrics


2012 PREP SA ON CD-ROM

Bradley JS, Sauberan J. Antimicrobial agents. In: Long SS, Pickering LK, Prober CG, eds. Principles and
Practice of Pediatric Infectious Diseases. 3rd ed. Philadelphia, PA: Churchill Livingstone Elsevier;
2008:1420-1452

Centers for Disease Control and Prevention. Sexually transmitted diseases treatment guidelines, 2006.
MMWR Morb Mortal Wkly Rep. 2006;55(RR-11):1-94. Available at:
http://www.cdc.gov/std/treatment/2006/rr5511.pdf

Centers for Disease Control and Prevention. Updated Recommended Treatment Regimens for
Gonococcal Infections and Associated Conditions - United States, April 2007. Available at:
http://www.cdc.gov/std/treatment/2006/updated-regimens.htm

Copyright 2012 American Academy of Pediatrics


2012 PREP SA ON CD-ROM

Question 31
A grandmother discovers her 18-month-old grandson in the garage near an opened container of
greenish fluid that she suspects to be antifreeze. She brings the child to the urgent care center for
evaluation. On physical examination, the boy appears sleepy and somewhat ataxic. Initial laboratory
evaluation reveals:
Sodium, 140 mEq/L (140 mmol/L)
Potassium, 4.1 mEq/L (4.1 mmol/L)
Chloride, 105 mEq/L (105 mmol/L)
Bicarbonate, 16 mEq/L (16 mmol/L)
Calcium, 9.0 mg/dL (2.25 mmol/L)
Magnesium, 2.0 mEq/L (1.0 mmol/L)
Phosphorus, 5.5 mg/dL (1.8 mmol/L)
Glucose, 90 mg/dL (5.0 mmol/L)
Blood urea nitrogen, 14 mg/dL (5.0 mmol/L)
Creatinine, 0.4 mg/dL (35.4 mcmol/L)
Albumin, 4.0 g/dL (40 g/L)
Serum osmolality, 310 mOsm/kg
Of the following, the osmolar gap in this child, who has a possible ingestion, is CLOSEST to
A. 8
B. 12
C. 16
D. 20
E. 24

Copyright 2012 American Academy of Pediatrics


2012 PREP SA ON CD-ROM

Critique 31 Preferred Response: D


Antifreeze can contain up to 95% ethylene glycol, and clinical suspicion that the child described in
the vignette ingested antifreeze is supported by the history and physical examination findings. Laboratory
evaluation reveals an elevated anion gap metabolic acidosis. The anion gap is calculated by subtracting
the sum of the commonly measured anions (chloride and bicarbonate) from the most commonly
measured cation (sodium):
+ - -
Na [Cl + HCO3 ].

A normal gap is 12+4; this child has an anion gap of 19. Furthermore, he has an elevated serum
osmolality at 310 mOsm/kg (normal, 275 to 290 mOsmol/kg). For a patient in whom an ingestion is
suspected, serum osmolality should be measured in pursuit of a possible increased osmolar gap,
especially in the setting of an increased anion gap metabolic acidosis. An osmolar gap is calculated by
subtracting measured plasma osmolality from a calculated plasma osmolality, with a normal value being
10.
Osmolality is measured in the plasma by freezing point depression technique. The osmolality can
be calculated using the following equation:
+
Calculated Posm = 2 x plasma [Na ] + [glucose]/18 + BUN/2.8

For the patient in the vignette, the calculated osmolality is:

Posm = 2 x [140] + [90]/18 + [14]/2.8 = 280 + 5 + 5 = 290 mOsmol/kg

Using this figure and the measured plasma osmolality, the osmolar gap is calculated with the
equation:

Osmolar gap = Measured Posm Calculated Posm

In this patient: Osmolar gap = 310 mOsmol/kg 290 mOsmol/kg = 20 mOsmol/kg


The rationale behind the osmolar gap is that alcohols such as methanol, ethanol, and ethylene
glycol are small in size, with molecular weights ranging from 32 to 62 mg/mmol. This means that relatively
small amounts of these substances can result in relatively large changes in osmolality. The relationship of
molecular weight to osmolality explains why most drugs (eg, salicylates), which have much large
molecular weights, have minimal effects on osmolality.
The value of calculating both the anion gap and osmolar gap in a patient who has a suspected
ingestion of ethylene glycol or methanol is earlier recognition and treatment. Metabolism of these alcohols
by alcohol dehydrogenase can result in production of toxic metabolites, which increase the risks of
morbidity and mortality. Ethylene glycol is metabolized to glycolic acid and oxalic acid, which are
associated with neurologic, cardiopulmonary, and renal toxicity. Methanol (a component of deicing
solutions and varnish) is metabolized to formaldehyde and formic acid, which are associated with
blindness, coma, and death.
Treatment for both ethylene glycol and methanol toxicity includes ethanol or fomepizole (to block
alcohol dehydrogenase metabolism of these alcohols to toxic metabolites) and possibly hemodialysis.
The use of fomepizole has obviated the need for ethanol and hemodialysis in select pediatric cases of
ethylene glycol toxicity. Nonetheless, the practitioner should strongly consider consultation with a medical
toxicologist and possibly a nephrologist to determine if hemodialysis may be needed.

American Board of Pediatrics Content Specification(s):


Know that the plasma osmolality can be estimated using serum electrolytes, blood urea nitrogen, and
blood glucose concentrations

Suggested Reading:

Copyright 2012 American Academy of Pediatrics


2012 PREP SA ON CD-ROM

Boyer EW, Mejia M, Woolf A, Shannon M. Severe ethylene glycol ingestion treated without hemodialysis.
Pediatrics. 2001;107:172-173. Available at: http://pediatrics.aappublications.org/cgi/content/full/107/1/172

Kraut JA, Kurtz I. Toxic alcohol ingestions: clinical features, diagnosis, and management. Clin J Am Soc
Nephrol. 2008;3:208-225. DOI: 10.2215/CJN.03220807. Available at:
http://cjasn.asnjournals.org/content/3/1/208.long

McQuillen KK, Anderson AC. Osmol gaps in the pediatric population. Acad Emerg Med. 1999;6:27-30.
DOI: 10.1111/j.1553-2712.1999.tb00090.x. Available at: http://onlinelibrary.wiley.com/doi/10.1111/j.1553-
2712.1999.tb00090.x/abstract

Rose BD, Post TW. The total body water and the plasma sodium concentration. In: Clinical Physiology of
Acid-base and Electrolyte Disorders. 5th ed. New York, NY: McGraw-Hill Medical Publishing Division;
2001:241-257

Schwaderer AL, Schwartz GJ. Back to basics: acidosis and alkalosis. Pediatr Rev. 2004;25:350-357.
DOI: 10.1542/pir.25-10-350. Available at:
http://pedsinreview.aappublications.org/cgi/content/full/25/10/350

Copyright 2012 American Academy of Pediatrics


2012 PREP SA ON CD-ROM

Question 32
A 12-year-old girl presents to your office for a follow-up visit. She has had several episodes of
abdominal pain over the past 3 months but has had no associated vomiting, weight loss, fever,
gastrointestinal bleeding, or pain that awakens her at night. Laboratory testing for anemia, Helicobacter
pylori infection, and celiac disease has yielded no findings of note. Dietary adjustments (fiber
supplementation and lactose-free diet trial) have resulted in little relief. On physical examination, she has
no abdominal tenderness or mass, and she has no history of constipation. The girl has missed numerous
days of school due to the pain.
Of the following, the MOST appropriate next step is to
A. administer a screening tool for anxiety
B. prescribe a selective serotonin reuptake inhibitor
C. prescribe a tricyclic antidepressant
D. refer her for cognitive behavioral therapy
E. refer her to a gastroenterologist

Copyright 2012 American Academy of Pediatrics


2012 PREP SA ON CD-ROM

Critique 32 Preferred Response: A


Pediatric recurrent abdominal pain (RAP) has been defined as at least three episodes of
abdominal pain that occur over a period of at least 3 months and are severe enough to affect typical
functioning of the child. Approximately 80% of children who present with functional RAP have an anxiety
disorder. Therefore, mental health screening (eg, Screen for Child Anxiety Related Disorders [SCARED]
or the Spence Anxiety Scale for Children) and assessment should be considered early in the medical
evaluation of children such as the girl described in the vignette.
If the girls screening test indicates anxiety, the most appropriate next step is to educate the
family about the high comorbidity of anxiety disorders and RAP. A specific treatment for anxiety (either a
medication or psychotherapy) can be initiated after confirming the presence of an anxiety problem and
helping the patient/family decide that a problem must be addressed. Also, prescribing a medication to
treat suspected anxiety for this girl is premature; other approaches may be more effective. For example, if
the child has a particular stressor triggering the RAP, working on modifying or addressing that stressor is
more appropriate than prescribing a medication or embarking on a course of cognitive behavioral therapy.
Results of the patients history and medical evaluation to date do not indicate any findings of
concern that warrant referral to a gastroenterologist. Such findings would include weight loss,
gastrointestinal bleeding, systemic symptoms (eg, fever), laboratory evidence of anemia or inflammation,
persistent vomiting, or pain that frequently awakens the child at night.

AAP Mental Health Competency:


Know how to counsel a patient/family when a child has somatic complaints that could be due to
anxiety

Suggested Reading:
American Academy of Pediatrics. Addressing Mental Health Concerns in Primary Care: A Clinician's
Toolkit [CD-ROM]. Elk Grove Village, IL: American Academy of Pediatrics; 2010

American Academy of Pediatrics. Evidence-based child and adolescent psychosocial interventions.


Available at:
http://www.aap.org/commpeds/dochs/mentalhealth/docs/CR%20Psychosocial%20Interventions.F.0503.p
df

Campo JV, Bridge J, Ehmann M, et al. Recurrent abdominal pain, anxiety, and depression in primary
care. Pediatrics. 2004;113;817-824. Available at:
http://pediatrics.aappublications.org/cgi/content/full/113/4/817

Connolly SD, Bernstein GA; Work Group on Quality Issues. Practice parameter for the assessment and
treatment of children and adolescents with anxiety disorders. J Am Acad Child Adolesc Psychiatry.
2007;46:267-283. Abstract available at: http://www.ncbi.nlm.nih.gov/pubmed/17242630

Sakolsky D, Birmaher B. Pediatric anxiety disorders: management in primary care. Curr Opin Pediatr.
2008;20:538-543. DOI: 10.1097/MOP.0b013e32830fe3fa. Available at:
http://www.ncbi.nlm.nih.gov/pubmed/18781116

Copyright 2012 American Academy of Pediatrics


2012 PREP SA ON CD-ROM

Question 33
A 13-year-old girl who has moderate persistent asthma complains of daily ocular and nasal
allergy symptoms. She states that her symptoms are worse at home and bother her, despite a regimen of
an oral antihistamine and a nasal corticosteroid. On further questioning, she states that she has five dogs
and two cats, sleeps in a finished room in the basement, and enjoys collecting stuffed animals. She asks
how she can reduce her symptoms at home after learning that her serum immunoglobulin (Ig) E tests
were positive to cat and dust mite.
Of the following, you are MOST likely to advise her that she should
A. decrease indoor humidity to less than 50%
B. install a high-efficiency particulate air (HEPA) filter
C. use an impermeable mattress cover
D. vacuum carpets twice a week
E. wash the cats once a month

Copyright 2012 American Academy of Pediatrics


2012 PREP SA ON CD-ROM

Critique 33 Preferred Response: A


Identification and avoidance of indoor allergens is an important step in the management of
immunoglobulin (Ig)E-mediated diseases such as allergic rhinitis and allergic asthma. Dust mite is one of
the most common indoor allergens, except in locations where humidity is consistently less than 50%.
When dust mite allergen is significantly reduced, patients have demonstrated improvement in allergic
rhinitis symptoms and bronchial hyperresponsiveness.
Other allergens that have been identified as potential causes of IgE-mediated symptoms include
animal dander (eg, cat, dog, rodent), cockroach, and mold. Some of the recommended avoidance
measures for the bedroom include covering the mattress, box spring, and pillows with impermeable
covers; washing bedding regularly at 130.0F; removing stuffed animals and carpeting; and vacuuming
regularly. A recent Cochrane review, however, demonstrated that no single intervention alone, such as
impermeable covers, is likely to have a significant overall effect. Vacuums or high-efficiency particulate air
(HEPA) filters can collect dust mite allergen, but most allergen exposure located in the bedding is not
affected by these interventions.
Studies over the past 20 years have demonstrated interesting findings about cat allergen. Fel d 1,
the major allergen, initially was believed to be produced in saliva, but studies in which cats were
prevented from licking themselves demonstrated that this allergen is made in the skin. Because washing
the animals reduces allergen found in the skin and fur, studies have been performed to evaluate regular
washing as a possible intervention because most pet owners are unwilling to remove their pets from the
home. Two studies demonstrated that immersion up to the neck for 3 minutes followed by towel drying
does significantly reduce allergen, but the reduction is short-lived; concentrations of the allergen return to
baseline values by the next day. Patients who remove the cat(s) from the home find that Fel d 1
concentrations do decrease, although significant reduction may take 2 to 3 months.
Avoidance continues to be the recommendation for patients who are allergic to cats, but they
often describe a rapid onset of ocular and nasal symptoms when entering a home that contains a cat. In
addition, concentrations of cat dander that can result in IgE-mediated sensitization can be found in
buildings in which a cat never has been, illustrating the ease of transporting cat dander inadvertently by
people who own cats. More recently, studies have shown that children exposed to pet dander since birth
(ie, the pet is already in the home) have a lower incidence of rhinitis and asthma to these allergens
compared with children exposed later in life.

American Board of Pediatrics Content Specification(s):


Know that indoor pets may contribute to allergic disease

Suggested Reading:
Nageotte C, Park M, Havstad S, Zoratti E, Ownby D. Duration of airborne Fel d 1 reduction after cat
washing. J Allergy Clin Immunol. 2006;118:521-522. DOI: 10.1016/j.jaci.2006.04.049. Available at:
http://www.jacionline.org/article/S0091-6749(06)00944-4/fulltext

Sheikh A, Hurwitz B, Nurmatov U, van Schayck CP. House dust mite avoidance measures for perennial
allergic rhinitis. Cochrane Database Syst Rev. 2010;7:CD001563. DOI:
10.1002/14651858.CD001563.pub3. Available at:
http://onlinelibrary.wiley.com/o/cochrane/clsysrev/articles/CD001563/frame.html

Sublett J, Seltzer J, Burkhead R, Williams PB, Wedner HJ, Phipatanakul W; American Academy of
Allergy, Asthma, and Immunology Indoor Allergen Committee. Air filters and air cleaners: rostrum by the
American Academy of Allergy, Asthma, and Immunology Indoor Allergen Committee. J Allergy Clin
Immunol. 2010;125:32-38. DOI: 10.1016/j.jaci.2009.08.036. Available at:
http://www.jacionline.org/article/S0091-6749(09)01317-7/fulltext

Copyright 2012 American Academy of Pediatrics


2012 PREP SA ON CD-ROM

Question 34
A father brings his 2-year-old son to the emergency department after they had spent several
hours in the garage while the father worked on the car. The father reports that approximately 30 minutes
ago he heard the child coughing and found him with an open bottle of charcoal lighter fluid in his hands.
On physical examination, the awake and alert childs temperature is 37.0C, heart rate is 120 beats/min,
respiratory rate is 24 breaths/min, blood pressure is 90/60 mm Hg, and oxygen saturation is 98%. His
shirt is saturated with lighter fluid. You remove the boys shirt and decontaminate his skin.
Of the following, the MOST appropriate next step is to
A. obtain a STAT chest radiograph
B. obtain a urine toxicology screen
C. perform gastric lavage
D. place the child under observation
E. reassure the father and discharge the patient

Copyright 2012 American Academy of Pediatrics


2012 PREP SA ON CD-ROM

Critique 34 Preferred Response: D


The boy described in the vignette has ingested a hydrocarbon-containing substance. In contrast
to most other toxic ingestions, management does not typically involve decontamination or elimination but
focuses on respiratory stabilization and observation. The asymptomatic patient who has had a
hydrocarbon exposure should be observed for the development of signs or symptoms for a minimum of 6
hours. A chest radiograph should be obtained 4 to 6 hours after exposure. If the radiograph is normal and
the patient does not develop any symptoms during the observation period, he or she may be safely
discharged.
Hydrocarbons are a large family of compounds that most commonly cause toxic effects in the
pulmonary and central nervous systems when ingested. Hydrocarbon ingestion does not often lead to
systemic toxicity unless other toxic substances such as camphor or pesticides are admixed with the
hydrocarbon. Rather, aspiration during the swallowing event deposits the hydrocarbon in the pulmonary
tree, where it directly injures the respiratory mucosa. The resulting damage leads to chemical
pneumonitis. Aspiration risk is highest with low-viscosity, high-volatility hydrocarbons such as kerosene,
furniture polish, mineral spirits, and gasoline. Highly volatile hydrocarbons may diffuse rapidly into the
central nervous system, leading to ataxia, somnolence, stupor, or coma.
Clinical suspicion should be high for an aspiration event in any child who presents following a
hydrocarbon exposure with a history of coughing/gagging or respiratory signs and symptoms such as
tachypnea, wheezing, or hypoxemia. Evaluation of symptomatic patients should include assessment of
oxygen saturation or arterial blood gasses and chest radiography. Urine toxicology screening is not
routinely helpful unless an illicit coingestant is suspected. A chest radiograph (Item C34) should be
obtained acutely in a symptomatic patient, but because radiographic findings often lag behind clinical
findings, the radiograph should be repeated in 4 to 6 hours if it appears initially normal. Supportive care
should be provided, including oxygen therapy, beta-agonist treatment for wheezing, and
intubation/mechanical ventilation for respiratory failure. Antibiotics and corticosteroids are not indicated.
Gastric lavage is contraindicated because it has the potential to cause further aspiration. Patients who
have any respiratory signs or symptoms on presentation should be observed for 24 to 48 hours for
disease progression.

American Board of Pediatrics Content Specification(s):


Know how to manage a child who has ingested a substance containing a hydrocarbon

Suggested Reading:
Levine MD, Greshem C III. Toxicity, hydrocarbons. eMedicine Specialties, Emergency Medicine,
Toxicology. 2009. Available at: emedicine.medscape.com/article/821143-overview

Lewander WJ, Aleguas A Jr. Hydrocarbon poisoning. UpToDate Online 18.1. 2010. Available at:
http://www.uptodate.com/online/content/topic.do?topicKey=ped_tox/11453

ODonnell KA, Burns Ewald M. Pediatric drug therapy: poisonings. In: Kleigman RM, Stanton BF, St.
Geme JW III, Schor NF, and Behrman RE, eds. Nelson Textbook of Pediatrics. 19th ed. Philadelphia, PA:
Saunders Elsevier; 2011:250-270

Copyright 2012 American Academy of Pediatrics


2012 PREP SA ON CD-ROM

Item 34

(Courtesy of D Krowchuk)
Chest radiograph in a 13-month-old child who ingested lamp oil shows bibasilar and paramediastinal
opacities.

Copyright 2011 American Academy of Pediatrics


2012 PREP SA ON CD-ROM

Question 35
A 5-month-old male infant presents to your office because of pallor and irritability. He was born at
term following an uncomplicated pregnancy and delivery and had a birthweight of 3,150 g. Because of
newborn screening results, a sweat chloride examination was performed at 1 month of age that confirmed
a diagnosis of cystic fibrosis. Since birth, the baby has been exclusively breastfed. At the time of
diagnosis, pancreatic enzyme supplementation was begun and now includes 8,000 units of lipase per
nursing session. Approximately 1 week ago, the mother noted that the baby was breathing fast and
appeared very pale. On physical examination today, the well-developed infant has a weight of 6.0 kg,
heart rate of 160 beats/min, and respiratory rate of 40 breaths/min. You also note conjunctival and
mucous membrane pallor and a liver edge palpable 2 cm below the right costal margin. Laboratory results
include:
Hemoglobin, 9.0 g/dL (90 g/L)
3 9
White blood cell count, 9.0x10 /mcL (9.0x10 /L)
Albumin, 3.8 g/dL (38 g/L)
Reticulocyte count, 12.5% (0.125)
A blood smear demonstrates polychromasia and numerous schistocytes (Item Q35).
Of the following, the MOST appropriate treatment for this infant is
A. alpha-tocopherol
B. ascorbic acid
C. cyanocobalamin
D. folic acid
E. thiamine

Copyright 2012 American Academy of Pediatrics


2012 PREP SA ON CD-ROM

Question 35

(Courtesy of D Krowchuk)
Schistocytes (arrows), as described for the infant in the vignette.

Copyright 2012 American Academy of Pediatrics


2012 PREP SA ON CD-ROM

Critique 35 Preferred Response: A


As per current recommendations of the Cystic Fibrosis (CF) Foundation, the infant described in
the vignette is breastfed and supplemented with exogenous pancreatic enzymes. The most common
nutritional problems in children who have CF are the consequences of fat and fat-soluble vitamin
malabsorption due to pancreatic insufficiency and often cholestasis. The infant described in the vignette
has developed a hemolytic anemia, which is most consistent with a deficiency in alpha-tocopherol
(vitamin E). Deficiencies of the water-soluble vitamins (including ascorbic acid, folic acid,
cyanocobalamin, and thiamine) are uncommon and would be the result of severe malnutrition in
association with reduced dietary intake of these vitamins.
Achievement of normal growth is a critically important objective in the care of patients who have
CF. This goal requires appropriate energy and nutrient intake that must account for disease-related
disturbances in gastrointestinal and pulmonary function. Studies have shown that poor clinical outcomes
are associated with undernutrition in patients who have CF. As stated in the 2002 Consensus Report,
The CF care team should monitor growth, provide anticipatory counseling, and plan intervention
strategies when patients are at risk for undernutrition or are diagnosed with nutritional failure. A registered
dietitian must be a part of the team to provide the discipline-specific expertise needed for optimal
nutritional management and may, along with other team members such as the physician, nurse, social
worker or a psychologist, provide expertise concerning developmental and behavioral aspects of eating.
Prevention and early intervention are most successful in combating nutritional failure.
Specific nutritional deficits in CF fall into two broad categories: overall protein-energy malnutrition
due to insufficient macronutrient intake and specific nutritional deficiencies related to pancreatic
insufficiency and impaired digestion/absorption. Guidelines have been established for pancreatic enzyme
supplementation for infants (Item C35A) and young children using both standard and enteric-coated
preparations. Similar guidelines for enzyme dosing have been established for older children and adults.
The goal is to achieve normal growth and weight gain by appropriate dietary guidance but also to limit
enzyme supplements to less than 2,500 U lipase/kg per meal to avoid fibrosing colonopathy, a
complication of high-dose enzyme supplementation. With the possible exception of patients who have
severe lung disease, the use of oral protein-energy supplements is controversial. A recent systematic
study suggests that dietary advice alone is a satisfactory approach for patients who have CF and
moderate malnutrition and that specialized supplemental formulas are of little value.
Careful monitoring of patients who have CF is required not only to assure sufficient energy intake
and optimize pancreatic enzyme dosing, but also to assure adequate supplementation with the fat-soluble
vitamins A, D, E, and K. At diagnosis, most infants require 125% of the recommended dietary allowance
for energy until catch-up growth is achieved and a long-term nutritional plan is implemented.
Malabsorption of fat-soluble vitamins is a particular concern in those who have pancreatic insufficiency; in
one study, biochemical evidence of fat-soluble vitamin deficiency was found in almost 50% of patients
studied. Accordingly, the required intake of these vitamins for those who have CF may range from 2 to 20
times the daily recommended intake (with the exception of vitamin K, where required intake values are
unknown), as shown in (Item C35B). These vitamins should be provided as their water-soluble analogs to
assure maximum absorption. A number of formulations specifically developed for CF patients are
available. Compliance with vitamin therapy is an important objective in overall CF care, and
concentrations of vitamins A, E, and D should be measured annually. In CF patients, adverse effects of
fat-soluble vitamin deficiencies are not uncommon, and they include:
Vitamin A: xerophthalmia
Vitamin E: neuropathy, retinopathy, and hemolytic anemia
Vitamin D: osteopenia, fractures
Vitamin K: clotting abnormalities
The risk for these deficiency states is heightened in patients who manifest cholestasis in addition
to pancreatic insufficiency.

American Board of Pediatrics Content Specification(s):


Recognize the specific nutritional problems in children with cystic fibrosis

Suggested Reading:

Copyright 2012 American Academy of Pediatrics


2012 PREP SA ON CD-ROM

Borowitz D, Baker RD, Stallings V. Consensus report on nutrition for pediatric patients with cystic fibrosis.
J Pediatr Gastroenterol Nutr 2002;35:246-259. Available at:
http://journals.lww.com/jpgn/Fulltext/2002/09000/Consensus_Report_on_Nutrition_for_Pediatric.4.aspx

Feranchak AP, Sontag MK, Wagener JS, Hammond KB, Accurso FJ, Sokol RJ. Prospective, long-term
study of fat-soluble vitamin status in children with cystic fibrosis identified by newborn screen. J Pediatr.
1999;135:601-610. Available at: http://www.ncbi.nlm.nih.gov/pubmed/10547249

Kleinman RE. Nutrition in cystic fibrosis. In: Pediatric Nutrition Handbook. 6th ed. Elk Grove Village, IL:
American Academy of Pediatrics; 2009:1001-1020

Rayner RJ. Fat-soluble vitamins in cystic fibrosis. Proceedings of the Nutrition Society. 1992;51:245-250.
DOI: 10.1079/PNS19920035. Available at:
http://journals.cambridge.org/action/displayFulltext?type=1&fid=752416&jid=PNS&volumeId=51&issueId=
02&aid=752408&bodyId=&membershipNumber=&societyETOCSession=

Smyth RL, Walters S. Oral calorie supplements for cystic fibrosis. Cochrane Database Syst Rev. 2007;1:
CD000406. DOI: 10.1002/14651858.CD000406.pub2. Available at:
http://onlinelibrary.wiley.com/o/cochrane/clsysrev/articles/CD000406/frame.html

Stallings VA, Stark LJ, Robinson KA, Feranchak AP, Quinton H; Clinical Practice Guidelines on Growth
and Nutrition Subcommittee, Ad Hoc Working Group. Evidence-based practice recommendations for
nutrition-related management of children and adults with cystic fibrosis and pancreatic insufficiency:
results of a systematic review. J Am Diet Assoc. 2008;108:832-839. DOI: 10.1016/j.jada.2008.02.020.
Abstract available at: http://www.ncbi.nlm.nih.gov/pubmed/18442507

Copyright 2012 American Academy of Pediatrics


2012 PREP SA ON CD-ROM

Critique 35

Item C35A. Suggested Pancreatic Enzyme Dosing for Infants Who Have Cystic Fibrosis
Age Nonenteric-Coated Products Enteric-coated Products
Birth to 1 year 8 to 16,000 U lipase/120 mL formula 8,000 U lipase/240-mL formula
8 to 16,000 U lipase/nursing session 4 to 8,000 U lipase/nursing session
8,000 U lipase/60 mL pureed solids 8,000 U lipase/120 mL pureed solids
Adapted from Kleinman RE, ed. Pediatric Nutrition Handbook. 6th ed. Elk Grove Village, IL:
American Academy of Pediatrics; 2009

Copyright 2011 American Academy of Pediatrics


2012 PREP SA ON CD-ROM

Critique 35

Item C35B. Daily Recommended Fat-soluble Vitamin Intake In Cystic Fibrosis

Vitamin 0 to 2 years 2 to 8 years 8 years to adult


A 1,500 IU 5,000 IU 5,000 to 10,000 IU
0 to 6 months 6 to 12 months 1 to 4 years 4 to 10 years >10 years
D 400 IU 400 IU 600 IU 600 IU 600 IU

E 25 IU 50 IU 100 IU 100 to 200 IU 200 to 400 IU

Adapted from Kleinman RE, ed. Pediatric Nutrition Handbook. 6th ed. Elk Grove Village, IL:
American Academy of Pediatrics; 2009

Copyright 2011 American Academy of Pediatrics


2012 PREP SA ON CD-ROM

Question 36
You attend the vaginal delivery of a 42-weeks gestation infant that is complicated by moderate
meconium-stained fluid. The prenatal course was unremarkable except for group B Streptococcus
colonization that was treated adequately during labor. The infant requires endotracheal suctioning in the
delivery room, which produces scant meconium-stained fluid from below the vocal cords. He is taken to
the special care nursery receiving blow-by oxygen, but worsening respiratory distress leads to intubation
and initiation of assisted ventilation requiring an FiO2 of 70% to maintain an oxygen saturation of 95%. As
you are calling the tertiary center to arrange transport, his saturations acutely deteriorate, requiring 100%
oxygen to maintain his oxygen saturation at 85%. You obtain a chest radiograph (Item Q36).
Of the following, the intervention MOST likely to improve this infants respiratory status is
A. adjustment of the endotracheal tube
B. administration of exogenous surfactant
C. infusion of prostaglandin E1
D. initiation of inhaled nitric oxide
E. insertion of a chest tube

Copyright 2012 American Academy of Pediatrics


2012 PREP SA ON CD-ROM

Question 36

(Courtesy of S Izatt)
Chest radiograph, as described for the infant in the vignette.

Copyright 2012 American Academy of Pediatrics


2012 PREP SA ON CD-ROM

Critique 36 Preferred Response: E


The infant described in the vignette has a left-sided tension pneumothorax that is compromising
his respiratory status, and prompt evacuation of the pleural air by insertion of a chest tube is likely to lead
to improved ventilation and oxygenation. A pneumothorax may be associated with decreased lung
volume, mediastinal shift, compression of the intrathoracic veins, and increased pulmonary vascular
resistance, which affect both ventilation and cardiac output. Clinical examination findings in an affected
infant may include tachypnea, grunting, flaring, retractions, and cyanosis, with auscultation demonstrating
diminished or absent breath sounds on the affected side. The examiner may note that the chest wall is
asymmetric, with prominence of the affected side and a shift of the point of maximum cardiac impulse.
Spontaneous pneumothorax occurs in up to 2% of healthy infants, with the initial high opening
pressures generated by crying and uneven ventilation due to retained lung fluid leading to rupture of
overdistended alveoli. Most spontaneous pneumothoraces are asymptomatic and resolve without
intervention. Air leaks are more common in conditions that lead to alveolar distension. Atelectasis seen in
respiratory distress syndrome and plugging of the small airways in meconium aspiration syndrome may
lead to unequal ventilation, with overdistension and subsequent alveolar rupture. Continuous positive
airway pressure and positive-pressure mechanical ventilation further increase the risk of pneumothorax.
Strategies such as surfactant administration, synchronized ventilation, and jet ventilation have been found
to decrease the incidence of air leak in at-risk populations. However, the administration of surfactant does
not aid in treating a pneumothorax once it has developed.
According to the chest radiograph of the infant in the vignette, the endotracheal tube is placed
appropriately above the carina. Further assessment of the infant may be needed after decompression of
the pneumothorax. Pulse oximetry should be used to evaluate preductal and postductal saturations, with
the finding of preductal saturations more than 10% greater than postductal saturations suggesting
pulmonary hypertension that might benefit from initiation of inhaled nitric oxide. Cardiac echocardiography
may demonstrate pulmonary hypertension or structural heart disease, and the finding of ductal-dependent
congenital heart disease supports the initiation of prostaglandin E1 therapy to maintain ductal patency.

American Board of Pediatrics Content Specification(s):


Recognize that pulmonary air leaks are common in newborn infants who are treated with assisted
ventilation
Recognize the characterisitc clinical and radiographic appearance of a pneumonthorax in a newborn
infant

Suggested Reading:
Donn SM, Sinha SK. Complications of assisted ventilation. In: Martin RJ, Fanaroff AA, Walsh MC, eds.
th
Fanaroff and Martins Neonatal-Perinatal Medicine. 9 ed. St. Louis, MO: Elsevier Mosby; 2011: 1134-
1136.

Litmanovitz I, Carlo WA. Expectant management of pneumothorax in ventilated neonates. Pediatrics.


2008;122:e975-e979. DOI: 10.1542/peds.2008-1269. Available at:
http://pediatrics.aappublications.org/cgi/content/full/122/5/e975

Venkatesh MP. Pulmonary air leak. In: Cloherty JP, Eichenwald EC, Stark AR, eds. Manual of Neonatal
Care. 6th ed. Philadelphia, PA: Lippincott Williams & Wilkins, a Wolters Kluwer business; 2008:352-356

Copyright 2012 American Academy of Pediatrics


2012 PREP SA ON CD-ROM

Question 37
A 12-year-old girl who has a history of chronic rhinitis, recurrent sinusitis, and multiple
pneumonias has had a productive cough for 2 months. She has had no fever or other systemic symptoms
except fatigue. Bronchodilators provide limited symptomatic relief. Two previous courses of antibiotics
have produced transient but limited improvement. On physical examination, you note a slender child
without clubbing. Her respiratory rate is 28 breaths/min, and faint crackles and wheezes are audible
throughout her chest. She has purulent rhinitis with maxillary sinus tenderness. A chest radiograph shows
areas of linear atelectasis with thickened airways (Item Q37). Results of sweat chloride testing and direct
mutation analysis for cystic fibrosis are negative.
Of the following, the MOST appropriate next test in her evaluation is
A. allergy skin testing
B. echocardiography
C. high-resolution chest computed tomography scan
D. nasopharyngeal culture for virus
E. sinus radiographs

Copyright 2012 American Academy of Pediatrics


2012 PREP SA ON CD-ROM

Item 37

(Courtesy of B Wood)
Chest radiograph, as described for the girl in the vignette.

Copyright 2012 American Academy of Pediatrics


2012 PREP SA ON CD-ROM

Critique 37 Preferred Response: C


The girl described in the vignette demonstrates clinical and radiologic signs suggestive of
bronchiectasis. Bronchiectasis is a chronic disease of the conducting airways characterized by
irreversible dilation of the bronchial tree and manifested clinically by chronic cough with production of
thick, often purulent, sputum. Hemoptysis may occur, and fever may be present during infectious
exacerbations. Children frequently have anorexia and demonstrate poor weight gain. Clubbing occurs,
but it is not universal and is reported in only 2% of affected adults. With very severe disease, the child
may develop dyspnea and hypoxemia. Pulmonary function tests can show an obstructive, restrictive, or
mixed picture.
Plain radiographs are not specific for bronchiectasis but may demonstrate a wide variety of
abnormalities, including increased size and loss of definition of the bronchovascular markings,
honeycomb appearance, loss of lung volume of an affected lobe, linear atelectasis (Item C37A), and
irregular opacities representing mucus plugs. The currently accepted gold standard test for bronchiectasis
is high-resolution computed tomography (HRCT) scan of the chest. Tram (parallel) lines or ring shadows
(cross-section or end-on bronchial walls) may be visible on both plain radiography and HRCT. Further,
HRCT can show three patterns of bronchiectasis:
The cylindrical pattern shows diffuse but regular dilation of the bronchi with abruptly ending bronchial
lumen (Item C37B)
The varicose pattern also shows dilation of the bronchi, but local constriction results in an irregular
outline
The saccular or cystic variation appears as progressive dilation and ballooning of the bronchi, ending
in fluid/mucus-filled sacs
Because of concern for the amount of radiation exposure with CT scans, recent studies using
magnetic resonance imaging instead of HRCT have shown promising results for ongoing monitoring of
disease progression.
Bronchiectasis is not a disease in itself but rather the end result of several different processes
that can be classified as obstructive, infectious, defective host defense/immunologic, and
congenital/genetic. In the developed world, the most common cause for bronchiectasis in children is
cystic fibrosis. However, in the developing world and among indigenous populations, infectious causes, in
particular pertussis, mycobacteria, measles, and pneumococcus, are common. Allergic
bronchopulmonary aspergillosis also may lead to bronchiectasis. Airway obstruction, such as from foreign
body aspiration or extraluminal compression by vascular rings, has been associated with bronchiectasis.
Host defense conditions that have been associated include ciliary dyskinesia (a likely cause for the
patient in the vignette), hypogammaglobulinemia, immunoglobulin G subclass deficiency, and impaired
cough from neuromuscular disease. Congenital/genetic conditions include entities affecting airway
development (Williams-Campbell syndrome [absence of annular cartilage] and Mounier-Kuhn syndrome
[tracheobronchomegaly]) and alpha-1-antitrypsin deficiency.
Although patients who have bronchiectasis may have asthma or allergies, particularly those who
have allergic bronchopulmonary aspergillosis, allergy testing does not usually contribute to the diagnosis
or management. Echocardiography may be helpful if there are concerns that airway obstruction from
vascular anomalies underlies bronchiectasis, but HRCT still is necessary to confirm the presence of
bronchiectasis. Nasopharyngeal cultures for virus would not help explain the chronic course of this
patients disease. Although sinus disease with abnormal-appearing sinus radiographs can be seen with
cystic fibrosis, ciliary dyskinesia, and other pathogenic conditions, its presence does not explain or define
the underlying pulmonary disease in this patient.

American Board of Pediatrics Content Specification(s):


Know the differential diagnosis of bronchiectasis
Know that high-resolution CT of the chest is useful to diagnose bronchiectasis in a child

Suggested Reading:
Lakser O. Disorders of the respiratory tract: bronchiectasis. In: Kliegman RM, Stanton BF, St. Geme JW
III, Schor NF, and Behrman RE, eds. Nelson Textbook of Pediatrics. 19th ed. Philadelphia, PA: Saunders
Elsevier; 2011:1479

Copyright 2012 American Academy of Pediatrics


2012 PREP SA ON CD-ROM

Montella S, Santamaria F, Salvatore M, et al. Assessment of chest high-field magnetic resonance


imaging in children and young adults with noncystic fibrosis chronic lung disease: comparison to high-
resolution computed tomography and correlation with pulmonary function. Invest Radiol. 2009;44:532-
538. Abstract available at: http://www.ncbi.nlm.nih.gov/pubmed/19652613

Piedra PA, Stark AR. Bronchiolitis in infants and children: clinical features and diagnosis. UpToDate
Online 18.3. 2010. Available for subscription at:
http://www.uptodate.com/online/content/topic.do?topicKey=pedi_id/28044

Redding GJ. Bronchiectasis in children. Pediatr Clin North Am. 2009;56:157-171. DOI:
10.1016/j.pcl.2008.10.014. Abstract available at: http://www.ncbi.nlm.nih.gov/pubmed/19135586

Copyright 2012 American Academy of Pediatrics


2012 PREP SA ON CD-ROM

Critique 37

(Courtesy of B Wood)
Lateral chest radiograph in bronchiectasis demonstrating linear atelectasis (arrows).

Copyright 2011 American Academy of Pediatrics


2012 PREP SA ON CD-ROM

Critique 37

(Courtesy of E Anthony)
Chest computed tomography scan in bronchiectasis demonstrating cylindrical dilation of an airway
(arrows).

Copyright 2011 American Academy of Pediatrics


2012 PREP SA ON CD-ROM

Question 38
A 14-year-old girl presents for an office visit because of fatigue and increased sleepiness.
Menarche occurred when she was 11 years old, but her menses have always been irregular, and she has
noted some spotting at times between menses. She also reports a normal diet with occasional
constipation. Physical examination reveals slightly pale conjunctivae and oral mucosa. Her heart rate is
110 beats/min, she has a grade I/VI systolic ejection murmur heard best at the left sternal border, and her
blood pressure measures 110/70 mm Hg supine and 105/60 mm Hg sitting. Abdominal examination is
negative for tenderness or mass. There is a small amount of blood on a sanitary pad on external
genitourinary examination
Laboratory studies reveal:
3 9
White blood cell count, 14.0x10 /mcL (14.0x10 /L)
Hemoglobin, 9.9 mg/dL (99 g/L)
Hematocrit, 29% (0.29)
Mean corpuscular volume, 65 fL
Reticulocyte count, 0.2% (0.002)
Of the following, the BEST treatment for this patient is
A. daily multivitamin with iron
B. dietary counseling for iron-rich foods
C. intramuscular iron administration
D. oral supplementation with folic acid
E. prescription for oral ferrous sulfate three times daily

Copyright 2012 American Academy of Pediatrics


2012 PREP SA ON CD-ROM

Critique 38 Preferred Response: E


The girl described in the vignette has both historical and physical examination findings consistent
with a diagnosis of iron deficiency anemia. Typical laboratory findings in patients who have iron deficiency
include a low mean corpuscular volume and mean corpuscular hemoglobin; a hypochromic, microcytic
peripheral blood smear; and a normal or low reticulocyte count. Iron deficiency is common in adolescent
girls and most typically results from menstrual blood loss and inadequate dietary iron intake. However,
the clinician must be vigilant for more insidious causes, including gastrointestinal blood loss due to
inflammatory bowel disease, eating disorders, autoimmune diseases such as lupus erythematosus, and
abdominal trauma.
The most likely cause of the anemia for the girl described in the vignette is menstrual blood loss
or dietary iron insufficiency. Appropriate initial management includes evaluation and management of
menorrhagia, perhaps including oral hormone therapy to diminish dysfunctional uterine bleeding, and oral
iron supplementation with ferrous sulfate three times daily. The amount of iron in a multivitamin is
insufficient to treat iron deficiency; it is intended only to prevent iron deficiency.
The typical American teenagers diet is low in iron-rich foods, and recent reports suggest that
overweight children may be at special risk for iron deficiency. However, counseling about iron-rich foods
is not the best treatment for the adolescent described in the vignette because oral ferrous sulfate is
needed to reverse the anemia rapidly.
Folic acid supplementation is unlikely to reverse the anemia because the laboratory findings are
not suggestive of an anemia due to folic acid deficiency (macrocytosis). Intramuscular iron administration
is not indicated for this adolescent because these injections are painful, may lead to permanent skin
discoloration, and have been associated with anaphylactic reactions. In addition, intramuscular iron is
neither more efficacious nor rapidly acting than oral iron.

2American Board of Pediatrics Content Specification(s):


Understand the need to look for bleeding as a cause of iron deficiency in the child with a normal diet
Know how to diagnose iron deficiency anemia
Know that intramuscular iron injections or erythrocyte infusions are not appropriate for the child with
routine nutritional iron deficiency
Know that iron deficiency anemia is the major nutritional deficiency of American youths and identify
the signs and symptoms of the disorder

Suggested Reading:
Baker RD, Greer FR, The Committee on Nutrition. Diagnosis and prevention of iron deficiency and iron-
deficiency anemia in infants and young children (0-3 years of age). Pediatrics. 2010;126:1040-1050. DOI:
10.1542/peds.2010-2576. Available at: http://pediatrics.aappublications.org/cgi/content/full/126/5/1040

Lerner NB, Sills R. Anemias of inadequate production: iron deficiency anemia. In: Kliegman RM, Stanton
BF, St. Geme JW III, Schor NF, and Behrman RE, eds. Nelson Textbook of Pediatrics. 19th ed.
Philadelphia, PA: Saunders Elsevier; 2011:1655-1658

Janus J, Moerschel SK. Evaluation of anemia in children. Am Fam Physician. 2010;81:1462-1471.


Abstract available at: http://www.ncbi.nlm.nih.gov/pubmed/20540485

Killip S, Bennett JM, Chambers MD. Iron deficiency anemia. Am Fam Physician. 2007;75:671-678.
Available at: http://www.aafp.org/afp/2007/0301/p671.html

Nead KG, Halterman JS, Kaczorowski JM, Auinger P, Weitzman M. Overweight children and adolescents:
a risk group for iron deficiency. Pediatrics. 2004;114:104-108. Available at:
http://pediatrics.aappublications.org/cgi/content/full/114/1/104

Copyright 2012 American Academy of Pediatrics


2012 PREP SA ON CD-ROM

Question 39
You have been following a healthy 5-year-old boy since birth. You noted a heart murmur at his 6-
month health supervision visit, and you have heard it intermittently at subsequent evaluations. You are
seeing the boy today because of a viral upper respiratory tract infection associated with 2 days of fever.
On physical examination, the murmur sounds particularly prominent.
Of the following, the finding that MOST strongly suggests that this murmur is benign is
A. a decrease in murmur intensity when moving from supine to upright position
B. a higher blood pressure in the arms than legs
C. its continuous machinery quality in the left infraclavicular position
D. its diastolic nature
E. the harsh, high-pitched auscultatory features heard best on the lateral neck

Copyright 2012 American Academy of Pediatrics


2012 PREP SA ON CD-ROM

Critique 39 Preferred Response: A


Cardiac murmurs are the result of turbulent flow through the cardiac valves, chambers, and
arteries. With auscultation of any cardiac murmur, the clinician can identify and characterize the following
qualities of the sound:
Loudness, graded on a scale of 1 through 6
Timing within the cardiac cycle: systolic, diastolic, continuous
Location (or its greatest intensity) on the chest wall
Radiation, which often indicates the direction of the turbulent blood flow
Pitch, which results from the degree of pressure difference between the flow before and after the
source of turbulence
Such qualities and characteristics of the murmur help the examiner to determine whether the
turbulent blood flow results from intrinsic cardiovascular pathology or normal flow through a normal
cardiovascular system. The latter condition is considered an innocent murmur, also referred to as a
functional, benign, or normal murmur, and may be transient during childhood. Such murmurs may be
accentuated during high-output states (eg, fever, infection, anemia).
Physical examination begins with the patient in the supine position, using both the bell and the
diaphragm components of the stethoscope. The entire precordium, axillae, abdomen, anterior fontanelle,
and neck are evaluated. The examination is repeated with the patient in a sitting position. Finally, the
patients back is auscultated.
Innocent murmurs are typically relatively quiet (grades 1 or 2), systolic, located over the left
sternal border, possibly radiating to the upper left or right sternal border, low-pitched, vibratory, and
typically decreasing in intensity when the patient transitions from a supine to a sitting position.
The history of onset in infancy, intermittent nature, and lack of other signs and symptoms of
congenital heart disease for the child described in the vignette are suggestive of a benign murmur. The
decrease in murmur intensity when he is repositioned from a lying to a sitting position is also consistent
with a functional (benign) murmur. The murmur is more prominent at his current visit for a respiratory
illness due to the increased cardiac output associated with infection and fever. Because the history and
examination findings are consistent with a benign flow murmur, no additional testing or follow-up
evaluation is required.
A higher blood pressure in the arms than in the legs suggests the possibility of coarctation of the
aorta, which is not a benign condition. A continuous machinery murmur at the infraclavicular region is
suggestive of a patent ductus arteriosus. Diastolic murmurs are rarely benign and may result from aortic
or pulmonary insufficiency or inflow obstruction across the tricuspid or mitral valves. A high-pitched
systolic murmur heard over the neck is typical of aortic stenosis. It is often accompanied by a systolic
click and may be accentuated when the patient moves from a sitting to a lying position.

American Board of Pediatrics Content Specification(s):


Recognize the qualities of innocent heart murmurs and provide appropriate counseling
Recognize that a child with an innocent murmur requires no further evaluation

Suggested Reading:
Menashe V. Heart murmurs. Pediatr Rev. 2007;28:e19-e22. DOI: 10.1542/pir.28-4-e19. Available at:
http://pedsinreview.aappublications.org/cgi/content/full/28/4/e19

Moller JH. Clinical history and physical examination. In: Moller JH, Hoffman JIE, eds. Pediatric
Cardiovascular Medicine. Philadelphia, PA: Churchill Livingstone; 2000:97-110

Copyright 2012 American Academy of Pediatrics


2012 PREP SA ON CD-ROM

Question 40
A 12-year-old 40-kg girl presents for a health supervision visit. Physical examination, including
vital signs, yields normal results. The mother notes that the girl had to come home from school three
times last semester due to headaches. Her typical headaches are bifrontal, with sensitivity to light and
sound and often nausea. They last 1 to 6 hours. During the headache, she feels and looks sick and
prefers to lie in a dark room. The mother requests an acute treatment plan for her daughter. You provide
education about migraine headaches and discuss lifestyle issues, including good sleep hygiene, exercise,
diet, hydration, and stress management. For abortive headache treatment, you explain that it is ideal to
treat within 30 minutes, even at school.
Of the following, the PREFERRED abortive treatment for this girl is
A. butalbital (50 mg), acetaminophen (325 mg), caffeine (40 mg) orally
B. ibuprofen (400 mg) orally
C. promethazine (12.5 mg) rectally
D. sumatriptan (5 mg) intranasally
E. topiramate (25 mg) orally

Copyright 2012 American Academy of Pediatrics


2012 PREP SA ON CD-ROM

Critique 40 Preferred Response: B


The girl described in the vignette has an idiopathic, nonprogressive headache disorder consistent
with migraine. She does not describe any aura, but the headaches are severe enough for her to miss
school. Although painful and lasting for hours, they are relatively infrequent. In addition to lifestyle and
healthy habits education, it is important to have a plan for abortive treatment. The preferred abortive
treatment for migraine is ibuprofen because it is readily available and inexpensive, does not require a
prescription, and can be administered at home or school. Studies in adolescents suggest that ibuprofen
and acetaminophen are as effective as the triptans, with fewer adverse effects.
Combination products such as butalbital plus acetaminophen plus caffeine are not considered
first-line treatments. Sumatriptan and other triptans act through 5HT (serotonin) 1B 1D receptor
antagonism. Although shown to be effective in some placebo-controlled studies, they have a higher
adverse effect risk and can, therefore, be deferred until after implementation of healthy habits and
appropriate use of ibuprofen or acetaminophen.
Dopamine receptor-blocking agents such as metoclopramide and promethazine may be
considered, but they are not used as first-line therapy because of possible adverse effects such as
akathisia and acute dystonic reactions. Rectal administration is advantageous in the presence of nausea
but has other disadvantages in the adolescent population.
Topiramate is not used for acute treatment of migraine, although studies support its use at doses
of 25 to 75 mg twice a day as a daily medication for migraine prevention. Daily preventive medications
are generally considered when disabling headaches are more frequent (at least weekly) than described
for the girl in the vignette.

American Board of Pediatrics Content Specification(s):


Plan the abortive treatment of an acute migraine

Suggested Reading:
Damen L, Bruijn JK, Verhagen AP, Berger MY, Passchier J, Koes BW. Symptomatic treatment of
migraine in children: a systematic review of medication trials. Pediatrics. 2005;116:e295-e302. DOI:
10.1542/peds.2004-2742. Available at: http://pediatrics.aappublications.org/cgi/content/full/116/2/e295

Gunner KB, Smith HD. Practice guideline for diagnosis and management of migraine headaches in
children and adolescents: part one. J Pediatr Health Care. 2007;21:327-332. DOI: 1
0.1016/j.pedhc.2007.06.004

Hershey AD. Current approaches to the diagnosis and management of paediatric migraine. Lancet
Neurol. 2010;9:190-204. DOI: 10.1016/S1474-4422(09)70303-5. Abstract available at:
http://www.ncbi.nlm.nih.gov/pubmed/20129168

Lewis DW. Pediatric migraine. Neurol Clin. 2009;27:481-501. DOI: 10.1016/j.ncl.2008.11.003. Abstract
available at: http://www.ncbi.nlm.nih.gov/pubmed/19289227

Lewis DW. Pediatric migraine. Pediatr Rev. 2007;28:43-53. DOI: 10.1542/pir.28-2-43. Available at:
http://pedsinreview.aappublications.org/cgi/content/full/28/2/43

Medina LS, Kuntz KM, Pomeroy S. Children with headache suspected of having a brain tumor: a cost-
effectiveness analysis of diagnostic strategies. Pediatrics. 2001;108:255-263. Available at:
http://pediatrics.aappublications.org/cgi/content/full/108/2/255

Wang S-J, Juang K-D, Fuh J-L, Lu S-R. Psychiatric comorbidity and suicide risk in adolescents with
chronic daily headache. Neurology. 2007;68:1468-1473. Abstract available at:
http://www.ncbi.nlm.nih.gov/pubmed/17470748

Copyright 2012 American Academy of Pediatrics


2012 PREP SA ON CD-ROM

Question 41
A 16-year-old boy complains of pain in his feet after running long distances as well as intermittent
numbness and tingling in his fingers and toes for the past 4 months. On physical examination, the only
finding of note is somewhat high arches (pes cavus). Consultation with an orthopedist results in referral
for electromyography and nerve conduction studies, and the finding of low-normal nerve conduction
velocities is suggestive of Charcot-Marie-Tooth Disease type 2 (CMT2). Review of the family history
reveals that neither of the boys parents nor his younger sister have any signs or symptoms of hereditary
motor and sensory neuropathy. His maternal grandfather may have had had high foot arches and later
in life had an awkward gait believed to be a result of poor balance. This grandfather also had a brother
who had foot drop, and their mother had to retire from working on an assembly line in a factory in her
forties due to hand weakness, but she never had any recognizable problems with her feet.
Of the following, the MOST likely cause for CMT2 in this patient is
A. a new (de novo) autosomal dominant mutation in one of the CMT2 genes
B. an unbalanced chromosomal translocation that disrupts one of this boys CMT2 genes
C. incomplete penetrance of this autosomal dominant gene passed down through his mother
D. preferential gender expression of this autosomal dominant gene, with males more likely to be
symptomatic
E. variable expression of a mitochondrial gene passed to him from his mother and to his other
affected relatives from their mother

Copyright 2012 American Academy of Pediatrics


2012 PREP SA ON CD-ROM

Critique 41 Preferred Response: C


The teenage boy described in the vignette has symptoms of a peripheral sensory and motor
neuropathy that is confirmed on electrophysiologic testing and believed to be Charcot-Marie-Tooth
Disease type 2 (CMT2). As with many autosomal dominant conditions, there is great variability in the
expression of this condition among family members, and this boy is exhibiting more clinical symptoms at
a younger age than other possibly affected maternal family members (Item C41). The complete lack of
symptoms in the boys mother (which occurs in up to 25% of CMT2 carriers) could be attributed to
incomplete penetrance, which is the complete absence of symptoms despite carrying an autosomal
dominant condition or trait. However, there is still some potential for her to develop symptoms later in life
like her father, uncle, and paternal grandmother. If she ultimately develops symptoms, the CMT2 would
be considered variable expression rather than incomplete penetrance.

Although de novo mutations occur in a number of patients who have CMT, the presence of symptoms
such as pes cavus and gait difficulties in the boys maternal grandfather, foot drop in his maternal great-
uncle, and peripheral neuropathy in his maternal great-grandmother are much more suggestive of
autosomal dominant transmission. An unbalanced translocation disrupting a CMT gene in this young man
would be expected to cause additional problems such as intellectual disabilities or congenital anomalies;
even a balanced translocation would be a rare cause of CMT and might be suspected if there was a
history of recurrent pregnancy losses. Although preferential sex expression or increased severity of
expression in one sex over another is described in a few autosomal dominant conditions (such as Alport
syndrome), there does not appear to be any such bias in expression of symptoms in CMT. The variability
in clinical features described in this scenario is probably simply chance and not sex-specific.
Mitochondrial transmission has not been described in CMT and could not explain this family history
because the boys maternal grandfather could not pass a mitochondrial trait to any of his children; the
mitochondrial genome is transmitted exclusively from mothers to their offspring in the cytoplasm of
oocytes at conception.
CMT is a group of hereditary sensory and motor neuropathies associated with variable symptoms
such as pes cavus, distal hand or foot muscle weakness and atrophy, mild-to-moderate sensory loss, and
sometimes depressed deep-tendon reflexes. Diagnosis may be made using electromyelography and
nerve conduction velocity testing, with molecular studies sometimes helpful in confirming a suspected
diagnosis or used in family planning. Although autosomal recessive and X-linked CMT exist, they
represent the minority of cases. Patients who have CMT1 typically develop symptoms between the ages
of 5 and 25 years, whereas those who have CMT2 may have a slightly less severe course and less
significant changes in their nerve conduction velocities. There are 15 molecular subtypes of CMT2,
making molecular diagnostic testing very expensive and difficult to obtain. Although few patients who
have CMT1 or CMT2 become wheelchair-bound, they do experience significant disability and pain. Care
requires a multidisciplinary team of specialists that may include neurologists, physiatrists, orthopedists,
orthotists, and occupational and physical therapists. Both nonsteroidal anti-inflammatory agents and
tricyclic antidepressants or GABA analogs such as gabapentin may be used to treat pain.

American Board of Pediatrics Content Specification(s):


Recognize the inheritance pattern associated with an autosomal dominant disorder with incomplete
penetrance

Suggested Reading:
Bird TD. Charcot-Marie-Tooth neuropathy type 2. GeneReviews. 2010. Available at:
http://www.ncbi.nlm.nih.gov/bookshelf/br.fcgi?book=gene&part=cmt2

Lupski JR, Garcia CA. Charcot-Marie-Tooth peripheral neuropathies and related disorders. In: Valle D,
Beaudet AL, Vogelstein B, et al, eds. The Online Metabolic and Molecular Bases of Inherited Disease
(OMMBID). New York, NY: McGraw-Hill; Chap 227. Abstract available at:
http://www.ommbid.com/OMMBID/the_online_metabolic_and_molecular_bases_of_inherited_disease/b/a
bstract/part28/ch227

Copyright 2012 American Academy of Pediatrics


2012 PREP SA ON CD-ROM

Padua L, Aprile I, Cavallaro T, et al. Variables influencing quality of life and disability in Charcot-Marie-
Tooth (CMT) patients: Italian multicentre study. Neurol Sci. 2006;27:417-423. DOI: 10.1007/s10072-006-
0722-8. Available at: http://www.springerlink.com/content/c77162446714g472/

Copyright 2012 American Academy of Pediatrics


2012 PREP SA ON CD-ROM

Critique 41

(Courtesy of A Johnson)
Pedigree, as described for the boy in the vignette.

Copyright 2011 American Academy of Pediatrics


2012 PREP SA ON CD-ROM

Question 42
A 13-year-old boy presents for a routine health supervision visit. He is asymptomatic today, has
no findings of significance on past medical history, and currently is not taking any medications. He
appears well and has normal vital signs and a body mass index of 20.6. He is at Sexual Maturity Rating 2.
You find a slightly tender, rubbery mass under his right areola that is approximately 2 cm in diameter. The
remainder of his examination results, including a testicular evaluation, are normal. He tells you that one of
his grandmothers has breast cancer. He was embarrassed to tell you about the lump, and he is
concerned that he might have cancer.
Of the following, the MOST appropriate statement to make to the boy is that
A. his risk of breast cancer is high because of his family history
B. the breast mass should regress spontaneously
C. weight loss is necessary to resolve the mass
D. you will refer him to a plastic surgeon
E. you will refer him to an endocrinologist

Copyright 2012 American Academy of Pediatrics


2012 PREP SA ON CD-ROM

Critique 42 Preferred Response: B


The presence of palpable fibroglandular breast tissue under the nipple-areolar complex in a male
that measures at least 0.5 cm in diameter, as described for the boy in the vignette, is referred to as
gynecomastia. In the neonatal period and early adolescence, this is most often a physiologic
phenomenon that regresses spontaneously. Breast tissue has both estrogen (stimulatory) and androgen
(inhibitory) receptors. In males, during early puberty, the ratio of estrogen to testosterone is increased,
and up to 70% of males in Sexual Maturity Rating (SMR) 2 stage of pubertal development (testicular
volume of 5 to 10 mL) have some breast enlargement on examination that may be tender as a result of
edema and inflammation. Initially unilateral, the process becomes bilateral in up to 75% of cases. In most
males, such breast tissue no longer is palpable in 18 months (1 to 3 years), as puberty progresses and
androgen concentrations increase.
In false or pseudogynecomastia, as seen in obese males, adipose tissue does not feel like a
distinct, firm, rubbery mass. The doughnut sign is occasionally elicited when lying supine, in which the
areola forms a depression within the surrounding adipose tissue.
Pathologic conditions involving the male breast can be local masses or result from hormonal
aberrations of estrogen excess or androgen insufficiency. Rapid enlargement or a size greater than 4 cm
is usually pathologic. Local masses, such as hemangiomas, lymphangiomas, lipomas, and
neurofibromas, are usually unilateral, not circular, and not directly beneath the areola. Breast cancer in a
male adolescent is extremely rare. Such a mass is hard and fixed rather than rubbery and movable over
the chest wall. Estrogen excess may result from exogenous administration (eg, various teas and creams),
testicular and adrenal tumors, and chronic kidney and liver disease. Some causes of androgen
insufficiency are genetic conditions (eg, Klinefelter syndrome and androgen insensitivity, an X-linked
disorder) and testicular failure (eg, anorchia and primary hypogonadism). A large number of medications
and street drugs have been implicated in male breast enlargement and work by either estrogenic,
antiandrogenic, or other mechanisms. Some examples are: neurologic/psychiatric medications (eg,
diazepam and risperidone), antiandrogens (eg, ketoconazole and spironolactone), cardiovascular
medications (eg, calcium channel blockers), antiulcer drugs (eg, omeprazole), antineoplastic medications
(eg, methotrexate), antimicrobials (eg, isoniazid), and a host of other exogenous hormones and products
(eg, vaginal creams, licorice, and teas). Among the illicit drugs that have been implicated are anabolic
steroids, marijuana, amphetamines, and alcohol.
The description of the mass in the vignette is consistent with physiologic gynecomastia and does
not require a referral to endocrinology. If the mass does not regress and especially if it is causing
emotional concerns, referral to a plastic surgeon may be considered at that time. Weight loss is useful
only in pseudogynecomastia.

American Board of Pediatrics Content Specification(s):


Know the pathophysiology and differentiating features of normal vs abnormal gynecomastia in males

Suggested Reading:
Cakan N, Kamat D. Gynecomastia: evaluation and treatment recommendations for primary care
providers. Clin Pediatr (Phila). 2007;46:487-490. Abstract available at:
http://www.ncbi.nlm.nih.gov/pubmed/17579100

Diamantopoulos S, Bao Y. Gynecomastia and premature thelarche: a guide for practitioners. Pediatr Rev.
2007;28:e57-e68. DOI: 10.1542/pir.28-9-e57. Available at:
http://pedsinreview.aappublications.org/cgi/content/full/28/9/e57

Joffe A. Gynecomastia. In: Neinstein L, Gordon CM, Katzman DK, Rosen DS, Woods ER, eds.
Adolescent Health Care: A Practical Guide. 5th ed. Philadelphia, PA: Lippincott Williams & Wilkins, a
Wolters Kluwer business; 2008:180-184

Ma NS, Geffner ME. Gynecomastia in prepubertal and pubertal men. Curr Opin Pediatr. 2008;20:465-
470. DOI: 10.1097/MOP.0b013e328305e415. Abstract available at:
http://www.ncbi.nlm.nih.gov/pubmed/18622206

Copyright 2012 American Academy of Pediatrics


2012 PREP SA ON CD-ROM

Reddy SRV,Singh HR. Chest pain in children and adolescents. Pediatr Rev. 2010;31:e1-e9. DOI:
10.1542/pir.31-1-e1. Available at: http://pedsinreview.aappublications.org/cgi/content/full/31/1/e1

Copyright 2012 American Academy of Pediatrics


2012 PREP SA ON CD-ROM

Question 43
The mother of a 14-year-old girl brings her in for an appointment. Her daughter was sexually
assaulted by two older boys 2 months ago. The girl immediately told her mother, who brought her for a
medical examination and notified the police. Charges were filed against the boys, and the case is moving
through the court system. Physically, the girl has no residual problems, but her mother has been
concerned that she is not handling this experience well. She is increasingly clingy and tearful, will not go
to parties with friends if there might be new people there, will not talk about what happened, is not
sleeping well, and is having nightmares. She will not answer the door or the phone because she is afraid
it might be an unfamiliar man. The mother understands that these symptoms are likely related to the
stressful event, but the girls most recent report card indicates that she is failing two classes for the first
time ever, and her teachers note that she appears to be having problems concentrating. The mother asks
what she should do to help her daughter get over this event and get her life back on track, noting that
she was totally fine before they did this to her. In your office, the girl is very quiet, has downcast eyes,
and says she has nothing further to add to moms recounting of her story. Findings on her physical
examination are normal.
Of the following, the MOST appropriate next step is to
A. initiate sertraline therapy
B. initiate trazodone therapy
C. reassure the mother that her daughter will get past this soon
D. recommend trauma-focused cognitive behavioral therapy
E. suggest that the girl take time off from school to relax

Copyright 2012 American Academy of Pediatrics


2012 PREP SA ON CD-ROM

Critique 43 Preferred Response: D


The adolescent described in the vignette is experiencing symptoms consistent with a diagnosis
of posttraumatic stress disorder (PTSD). Her symptoms include: not answering the door in case
strangers are there, suggesting she experiences distress at cues that resemble the event (re-
experiencing); nightmares that are likely related to the event (re-experiencing); refusing to talk about what
happened or see friends (avoidance); and having difficulty falling asleep and concentrating (increased
arousal). The development of the symptoms 2 months after the event is consistent with the diagnosis of
PTSD. The severity of the symptoms and the disruption they have caused in her developmental trajectory
(peer interactions, school success) warrant intervention.
Despite a relative paucity of empirically based data to inform the treatment of PTSD in the
pediatric population, trauma-focused cognitive behavioral therapy has been shown to be helpful. Core
elements of this therapy include psychoeducation, stress management techniques, and some variant of
exposure using an interactive cognitive behavioral approach. However, which element(s) are most critical
to achieve symptom reduction or resolution is unknown.
The findings from one randomized study of sertraline in children who had PTSD indicated no
benefit over placebo. Thus, without data to support its use in PTSD and without evidence suggesting pre-
existing mood or anxiety problems for this girl, sertraline is not recommended at this time. Trazodone can
be used to facilitate sleep, but it does not address the symptom clusters of avoidance and re-
experiencing. Reassurance is an insufficient response to this presentation both because the girls
functioning is declining, which can lead to additional problems with loss of social and academic growth,
and because symptom clusters do not spontaneously resolve in a subgroup of children. Taking time off
from school is not helpful because it fuels the avoidance strategies that she has developed, thereby
limiting her optimal development.

AAP Mental Health Competency:


Recognize that there is no primary medication recommendation for treating PTSD in children.

Suggested Reading:
American Academy of Pediatrics. Evidence-based child and adolescent psychosocial interventions.
Available at:
http://www.aap.org/commpeds/dochs/mentalhealth/docs/CR%20Psychosocial%20Interventions.F.0503.p
df

Cohen JA, Bukstein O, Walter H, et al; AACAP Work Group on Quality Issues. Practice parameter for the
assessment and treatment of children and adolescents with posttraumatic stress disorder. J Am Acad
Child Adolesc Psychiatry. 2010;49:414-430. Abstract available at:
http://www.ncbi.nlm.nih.gov/pubmed/20410735

Donnelly CL, March JS, Amaya-Jackson L. Posttraumatic stress disorder. In: Dulcan MK, Wiener JM,
eds. Essentials of Child and Adolescent Psychiatry. Arlington, VA: American Psychiatric Publishing, Inc;
2006:479-504

Copyright 2012 American Academy of Pediatrics


2012 PREP SA ON CD-ROM

Question 44
You are evaluating an 8-year-old boy who developed the acute onset of coughing of bright red,
frothy blood. He has had no previous illnesses but has experienced fevers, cough, and fatigue over the
past week or two. On physical examination, the boy is in mild respiratory distress. He has a temperature
of 38.0C, heart rate of 110 beats/min, respiratory rate of 25 breaths/min, blood pressure of 110/70 mm
Hg, and oxygen saturation of 88% in room air. His lung sounds are coarse bilaterally. Physical
examination of his naso- and oropharynx reveal no evidence of trauma.
Of the following, the BEST initial test to aid in the diagnosis of the patients cause of hemoptysis
is
A. bronchoscopy
B. chest radiography
C. echocardiography
D. open lung biopsy
E. pulmonary function testing

Copyright 2012 American Academy of Pediatrics


2012 PREP SA ON CD-ROM

Critique 44 Preferred Response: B


Hemoptysis, defined as the expectoration of blood or blood-tinged sputum from the lower
respiratory tract, is a rare occurrence in children compared to adults. Diagnosing pediatric hemoptysis
can be challenging for various reasons, including the limitations in obtaining a pediatric medical history,
the tendency for children to swallow their sputum, and the difficulty in distinguishing hemoptysis from
hematemesis. Hemoptysis tends to produce bright, frothy blood, as described for the boy in the vignette,
in contrast to the dark red blood seen during hematemesis. In addition, hemoptysis is alkaline and
hematemesis is acidic.
The most common causes of hemoptysis are infection, tracheostomy-related trauma, foreign
bodies, cystic fibrosis, congenital heart disease, arteriovenous malformations, and hemosiderosis. Initial
medical management should include assurance of a stable airway as well as adequate ventilation,
oxygenation, and hemodynamics. Diagnostic screening studies are a complete blood count, coagulation
studies, and chest radiography. Chest radiography may be useful in revealing underlying cardiac or
pulmonary disease such as a pulmonary hemorrhage, which is characterized by either unilateral or
bilateral diffuse infiltrates (Item C44A).
Further imaging studies such as chest computed tomography scan (Item C44B) are indicated
when significant findings are seen on the initial chest radiograph. Evidence of congestive heart failure on
physical examination or an enlarged heart on chest radiography should prompt echocardiographic
evaluation. Bronchoscopy may be useful in establishing a diagnosis and in management (eg, removal of
foreign body, control of bleeding through application of topical vasoconstrictors) but should follow chest
radiography. Pulmonary function testing may be indicated to establish a baseline pulmonary status or in
following the status of a known patient. Open lung biopsy might be undertaken to establish the presence
of an underlying autoimmune disease or malignancy.

American Board of Pediatrics Content Specification(s):


Know the differential diagnosis of hemoptysis in children
Know the initial management of hemoptysis in children and adolescents
Know that hemosiderosis is associated with hemoptysis

Suggested Reading:
Nevin MA. Pulmonary embolism, infarction, and hemorrhage. In: Kliegman RM, Stanton BF, St. Geme JW
III, Schor NF, and Behrman RE, eds. Nelson Textbook of Pediatrics. 19th ed. Philadelphia, PA: Saunders
Elsevier; 2011:1500-1502

Nevin MA. Pulmonary hemosiderosis. In: Kliegman RM, Stanton BF, St. Geme JW III, Schor NF, and
Behrman RE, eds. Nelson Textbook of Pediatrics. 19th ed. Philadelphia, PA: Saunders Elsevier;
2011:1498-1500

Quintero DR. Hemoptysis in children. UpToDate Online 18.3. 2010. Available for subscription at:
http://www.utdol.com/online/content/topic.do?topicKey=pulm_dxs/4387

Copyright 2012 American Academy of Pediatrics


2012 PREP SA ON CD-ROM

Critique 44

(Courtesy of B Poss)
Chest radiograph of the patient described in the vignette, which shows diffuse bilateral infiltrates
consistent with pulmonary hemorrhage (in this case caused by systemic lupus erythematosus).

Copyright 2011 American Academy of Pediatrics


2012 PREP SA ON CD-ROM

Critique 44

(Courtesy of B Poss)
Computed tomography scan of the patient whose chest radiograph was presented as (Item C41A)
demonstrates diffuse bilateral posterior infiltrates with a fine reticulonodular pattern consistent with
pulmonary hemorrhage.

Copyright 2011 American Academy of Pediatrics


2012 PREP SA ON CD-ROM

Question 45
A 13-year-old boy presents with concerns about his pubertal progression. He is otherwise healthy
and takes no medications. His mother reports that she had menarche at 14 years of age, and his father
reports shaving for the first time in the 12th grade. Physical examination of the boy reveals Sexual
Maturity Rating 2 pubic hair and testicular volumes of 2 mL (testicular lengths of 2 cm) bilaterally. His
growth curve is shown in image (Item Q45).
Of the following, the MOST appropriate next step in the evaluation of this boy is to
A. determine bone age
B. measure follicle-stimulating and luteinizing hormones
C. measure testosterone
D. measure thyroid-stimulating hormone
E. obtain a karyotype

Copyright 2012 American Academy of Pediatrics


2012 PREP SA ON CD-ROM

Question 45

(Courtesy of M Haller)

Copyright 2012 American Academy of Pediatrics


2012 PREP SA ON CD-ROM

Critique 45 Preferred Response: A


The most likely diagnosis for the boy described in the vignette is constitutional delay of pubertal
development. His mother and father both reported that their pubertal development was somewhat later
than average, a historical factor that helps narrow the differential diagnosis because delayed puberty is
common in children whose parents had similar experiences. Further, the boys growth curve is reassuring
and demonstrates that he has maintained a normal prepubertal growth velocity of 4 to 6 cm/yr tracking
along the 10th percentile. A bone age would provide the most useful information because the boys bone
age would be expected to be delayed to coincide with his current level of pubertal progression. Based on
his growth curve, his height age (the age at which his current height is at the 50th percentile) is 10.25
years. A bone age between 9.75 and 10.75 years would strongly suggest that the child has constitutional
delay.
The measurement of gonadotropin-releasing hormones (GnRHs), which are produced by the
hypothalamus and stimulate the pituitary production of luteinizing hormone (LH) and follicle-stimulating
hormone (FSH), is not helpful when constitutional delay is the most likely cause of pubertal delay.
Measurement of LH and FSH is also unlikely to be helpful in the evaluation of the boy who has pubertal
delay, except in the uncommon situation where primary testicular failure is suspected. LH and FSH
values would be elevated in the boy who has primary testicular failure if the testicular hypothalamic
pituitary feedback loop is intact and prepubertal in the boy who has constitutional delay.
Because the physical examination of the boy reveals that he has testicular volumes (<4 mL) and
length (<2.5 cm) that are prepubertal and no other signs of virilization, there is no need to measure serum
testosterone, which would be low. Severe thyroid dysfunction that can cause abnormalities of pubertal
development would cause more signs or symptoms. In addition, a karyotype is an unnecessary and
expensive test in an otherwise healthy boy who has mild short stature and normal growth rate.
Treatment of constitutional delay typically involves only reassurance and observation. However,
in select cases, a 3- to 4-month course of testosterone cypionate or oxandrolone can be used to initiate
central puberty (using the positive feedback of androgen on LH/FSH secretion in normal early puberty).
Once a presumed diagnosis of constitutional delay is made, boys should be re-evaluated every 4 to 6
months to ensure that progression toward puberty ensues. The first evidence of true puberty in boys is
testicular volume of at least 4 mL (testicular length >2.5 cm). Pediatricians should keep in mind that boys
do not reach their peak pubertal growth velocities (8 to 12 cm/yr) until they reach Sexual Maturity Rating
stage 3 to 4; girls typically reach peak growth velocities between stages 2 and 3.

American Board of Pediatrics Content Specification(s):


Know how to use laboratory tests effectively to distinguish between constitutional growth delay and
other conditions

Suggested Reading:
Rosen D. Physiologic growth and development during adolescence. Pediatr Rev. 2004;25:194-200. DOI:
10.1542/pir.25-6-194. Available at: http://pedsinreview.aappublications.org/cgi/content/full/25/6/194

Sun SS, Schubert CM, Chumlea WC, et al. National estimates of the timing of sexual maturation and
racial differences among US children. Pediatrics. 2002;110:911-919. Available at:
http://pediatrics.aappublications.org/cgi/content/full/110/5/911

Copyright 2012 American Academy of Pediatrics


2012 PREP SA ON CD-ROM

Question 46
When a mother and her child enter the examination room for a health supervision visit, the child
quickly states, I want book. Her mother takes out her picture book, and you ask the girl to find the baby
in the story book. She looks through the book and points to a picture of a baby. She then turns and points
to her doll and says mine. Her mother proudly states that her daughter has just begun to combine
words.
Of the following, these findings are MOST expected for a typically developing child who is
A. 12 months
B. 18 months
C. 24 months
D. 30 months
E. 36 months

Copyright 2012 American Academy of Pediatrics


2012 PREP SA ON CD-ROM

Critique 46 Preferred Response: C


The child described in the vignette exhibits the normal cognitive/behavioral developmental
milestones for a 24-month-old child: combines words into two- or three-word phrases; points to pictures
named; uses vocabulary of 50+ words; and uses I, me, and mine in speech. At 12 months of age, a
child says mama and dada with meaning and at least one additional specific word. At 18 months of
age, a child has a vocabulary of 7 to 10 words. A child of 30 months of age is able to use pronouns
appropriately. A child of 36 months of age has a vocabulary of 250 words. Early identification of
developmental disorders is crucial to a childs well-being. Routine developmental surveillance should be
undertaken at each health supervision visit. A developmental screening tool should be administered at
the 9-, 18-, and either the 24- or 30-month health supervision visits. An autism spectrum disorder-specific
screening tool also should be administered at ages 18 and 24 months of age.
Indications for evaluation of language delay are: no babbling, pointing, or gesturing by 12 months;
no single words by 16 months of age; no two-word spontaneous phrases by 24 months of age; and any
regression in language or social skills at any age. A child whose results on screening tools are concerning
should be referred for developmental and medical evaluation as well as early developmental
intervention/early childhood services.

American Board of Pediatrics Content Specification(s):


Recognize the normal cognitive/behavioral developmental milestones for 24 months of age (eg,
combines words into two-or three-word phrases; points to pictures named; uses vocabulary of 50+
words; uses "I," "me," and "mine" in speech)
Know that failure to use single words by 24 months of age is abnormal

Suggested Reading:
Agin MC. The late talker-when silence isnt golden. Contemp Pediatr. 2004;21(Nov):22-34. Available at:
http://www.modernmedicine.com/modernmedicine/article/articleDetail.jsp?id=132720&sk=&date=&pageID
=2

Council on Children with Disabilities, Section on Developmental Behavioral Pediatrics, Bright Futures
Steering Committee, Medical Home Initiatives for Children With Special Needs Project Advisory
Committee. Identifying infants and young children with developmental disorders in the medical home: an
algorithm for developmental surveillance and screening. Pediatrics. 2006;118:405-420. Available at: DOI:
http://pediatrics.aappublications.org/content/118/1/405

Johnson CP, Myers SM. Overview of the AAP autism spectrum disorders toolkit and guidelines. Contemp
Pediatr. 2008;25(Oct):43-67. Available at:
http://www.modernmedicine.com/modernmedicine/Neurology/Overview-of-the-AAP-autism-spectrum-
disorders-tool/ArticleStandard/Article/detail/558616

Leppert MO. Neurodevelopmental Assessment and Medical Evaluation. In: Voight RG, Macias MM,
Myers SM, eds. American Academy of Pediatrics Developmental and Behavioral Pediatrics. Elk Grove
Village, IL: American Academy of Pediatrics; 2011:100-102

Copyright 2012 American Academy of Pediatrics


2012 PREP SA ON CD-ROM

Question 47
An 18-month-old girl in your practice has been evaluated for recurrent episodes of hemolytic
anemia. Her hematologist has diagnosed hereditary spherocytosis and recommended an elective
splenectomy.
Of the following, the MOST appropriate treatment before splenectomy for this child is
A. hepatitis A vaccine
B. measles, mumps, rubella vaccine
C. meningococcal conjugate vaccine
D. penicillin prophylaxis for 2 months
E. pneumococcal vaccine

Copyright 2012 American Academy of Pediatrics


2012 PREP SA ON CD-ROM

Critique 47 Preferred Response: E


Splenectomy (functional, anatomic, or surgical) creates an increased risk for severe disseminated
infection with encapsulated bacteria, including Streptococcus pneumoniae, Haemophilus influenzae type
b, and Neisseria meningitidis. Therefore, the girl described in the vignette, if not previously or completely
immunized against S pneumoniae, should receive pneumococcal conjugate vaccine. The 13-valent
conjugate pneumococcal vaccine is routinely recommended for all infants and young children 2 months to
5 years of age. In addition, previously unimmunized children ages 5 to 9 years who have undergone or
will undergo a splenectomy should receive this vaccine followed by the 23-valent pneumococcal
polysaccharide vaccine. The polysaccharide vaccine covers more serotypes but is less immunogenic and
should only be given to children 2 years of age and older.
Meningococcal conjugate vaccine also is indicated for this child, but at present, the available
meningococcal vaccines are not approved in children younger than 2 years of age. Splenectomized hosts
are not at increased risk from viral infections, including measles, mumps, rubella, or hepatitis A, although
they should receive routine recommended vaccines. There is no increased risk from live viral vaccines in
asplenic hosts.
Even with receipt of appropriate vaccines, asplenic hosts are still advised to receive penicillin
prophylaxis to protect against primarily overwhelming pneumococcal sepsis, but it is administered
lifelong, not just for 2 months.

American Board of Pediatrics Content Specification(s):


Know that a child with hereditary spherocytosis or other erythrocyte membrane disorders should
receive pneumococcal, meningococcal, and Hemophilus influenzae vaccines before splenectomy and
prophylactic penicillin after splenectomy
Know that children with asplenia are susceptible to increased morbidity and mortality from infection
with encapsulated organisms

Suggested Reading:
American Academy of Pediatrics. Immunocompromised children. In: Pickering LK, Baker CJ, Kimberlin
DW, Long SS, eds. Red Book: 2009 Report of the Committee on Infectious Diseases. 28th ed. Elk Grove
Village, IL: American Academy of Pediatrics; 2009:72-86

American Academy of Pediatrics. Pneumococcal infections. In: Pickering LK, Baker CJ, Kimberlin DW,
Long SS, eds. Red Book: 2009 Report of the Committee on Infectious Diseases. 28th ed. Elk Grove
Village, IL: American Academy of Pediatrics; 2009: 524535

Committee on Infectious Diseases. Policy statement: recommendations for the prevention of


Streptococcus pneumoniae infections in infants and children: use of the 13-valent pneumococcal
conjugate vaccine (PCV13) and pneumococcal polysaccharide vaccine (PPSV23). Pediatrics.
2010;126:186-190. DOI: 10.1542/peds.2010-1280. Available at:
http://pediatrics.aappublications.org/cgi/content/full/126/1/186

Pasternack MS. Prevention of sepsis in the asplenic patient. UpToDate Online 18.3. 2010. Available for
subscription at: www.uptodate.com/online/content/topic.do?topicKey=immuninf/9667

Copyright 2012 American Academy of Pediatrics


2012 PREP SA ON CD-ROM

Question 48
A 14-year-old previously well boy presents to your office with a 10-day history of cough and
posttussive emesis. His grandmother also has a coughing illness. His 3-week-old sibling, with whom he
lives, is well. On physical examination, the boy appears generally well, with a temperature of 37.0C,
heart rate of 80 beats/min, and respiratory rate of 16 breaths/min. He has mild nasal congestion, and his
lungs are clear to auscultation. When you use a tongue depressor to examine his posterior oropharynx,
he begins coughing and cannot stop. This cough paroxysm lasts about 30 seconds and is followed by
gasping and vomiting.
Of the following, the MOST appropriate intervention for the patients 3-week-old healthy sibling is
A. azithromycin for 5 days
B. clarithromycin for 7 days
C. erythromycin for 14 days
D. penicillin for 10 days
E. trimethoprim-sulfamethoxazole for 14 days

Copyright 2012 American Academy of Pediatrics


2012 PREP SA ON CD-ROM

Critique 48 Preferred Response: A


The boy described in the vignette has a paroxysmal cough, gasping, and posttussive emesis that
strongly suggest infection caused by the gram-negative bacterium Bordetella pertussis. Pertussis is a
highly contagious respiratory disease, with an attack rate of approximately 90% for household contacts.
Unvaccinated infants younger than 1 year of age, such as the 3-week-old in the vignette, have the highest
risk of severe and life-threatening complications.
Postexposure chemoprophylaxis is recommended for all household contacts, regardless of their
age or immunization status. If administered promptly (within 21 days of onset of cough in index case),
chemoprophylaxis may prevent secondary transmission of pertussis. For infants younger than 1 month of
age, azithromycin is the preferred drug for chemoprophylaxis and therapy.
Azithromycin is a macrolide antibiotic that inhibits protein synthesis by binding to the 50S subunit
of the bacterial ribosome. Macrolide antibiotics (erythromycin, clarithromycin, azithromycin, and
telithromycin) are inhibitory to bacteria. They are most active against gram-positive cocci and bacilli but
also are active against gram-negative bacilli. Macrolide antibiotics, especially azithromycin, achieve high
intracellular concentrations and, therefore, are active against susceptible intracellular pathogens.
Because azithromycin provides prolonged tissue concentrations of drug at the site of infection, it can be
administered for a short period of time.
Azithromycin is the drug of choice for pertussis treatment and chemoprophylaxis for infants
younger than 1 month of age because of the association between orally administered erythromycin and
the development of infantile hypertrophic pyloric stenosis. It also is considered appropriate therapy for
treatment or chemoprophylaxis in older children and adults. It is much better tolerated than erythromycin
and can be dosed once daily.
Azithromycin is approved for the treatment of community-acquired pneumonia, sinusitis, and
acute otitis media in children, although increasingly widespread resistance of Streptococcus pneumoniae
to the macrolides may limit its use. Children who have severe penicillin allergy may receive azithromycin
for the treatment of streptococcal pharyngitis. Azithromycin is active against organisms that cause
atypical pneumonia, such as Chlamydia, Mycoplasma, and Legionella. It also can be used to treat
sexually transmitted infections such as those caused by Chlamydia trachomatis and Neisseria
gonorrhoeae. Azithromycin is moderately active against Haemophilus influenzae and Moraxella
catarrhalis. Compared to the other macrolides, azithromycin has better activity against gastrointestinal
pathogens such as Salmonella, Shigella, Campylobacter, and Escherichia coli. It is the only antibiotic that
has been prospectively evaluated for the treatment of lymphadenitis caused by Bartonella henselae.
Clarithromycin, like azithromycin, is better tolerated than erythromycin. It can be used to treat
pertussis but is not recommended for use in children younger than 1 month of age. In addition to the
previously cited indications for azithromycin, clarithromycin has an important role in the prevention and
treatment of disseminated Mycobacterium avium-intracellulare complex (MAC) infections in patients who
have human immunodeficiency virus infection. It also can be used for the treatment of pneumonia and
cervical adenitis due to MAC. In addition, clarithromycin, in combination with amoxicillin and a proton
pump inhibitor, is used for the treatment of Helicobacter pylori infections. Although clarithromycin is
approved for the treatment of skin and skin structure infections, other drugs that are more active against
Staphylococcus aureus are available.
Trimethoprim-sulfamethoxazole also is an appropriate drug for pertussis treatment and
chemoprophylaxis but is contraindicated in children younger than 2 months of age. Penicillin has no
activity against B pertussis.

American Board of Pediatrics Content Specification(s):


Know the appropriate use of clarithromycin and azithromycin

Suggested Reading:
American Academy of Pediatrics. Pertussis (whooping cough). In: Pickering LK, Baker CJ, Kimberlin DW,
Long SS, eds. Red Book: 2009 Report of the Committee on Infectious Diseases. 28th ed. Elk Grove
Village, IL: American Academy of Pediatrics; 2009:504-519

Copyright 2012 American Academy of Pediatrics


2012 PREP SA ON CD-ROM

Bradley JS, Sauberan J. Antimicrobial agents. In: Long SS, Pickering LK, Prober CG, eds. Principles and
Practice of Pediatric Infectious Diseases. 3rd ed. Philadelphia, PA: Churchill Livingstone Elsevier;
2008:1420-1452

Centers for Disease Control and Prevention. Pertussis (Whooping Cough). 2010. Available at:
http://www.cdc.gov/pertussis/index.html

Copyright 2012 American Academy of Pediatrics


2012 PREP SA ON CD-ROM

Question 49
A 4-year-old boy presents with a 3-day history of vomiting followed by diarrhea. The vomiting
resolved after 24 hours, and for the past 2 days, he has been keeping down clear liquids. On physical
examination, the afebrile boy has a heart rate of 90 beats/min, respiratory rate of 18 breaths/min, and
blood pressure of 106/58 mm Hg. He has tacky mucous membranes and a capillary refill time of 3
seconds. Other findings are within normal parameters. Laboratory evaluation reveals:
Sodium, 130 mEq/L (130 mmol/L)
Potassium, 3.7 mEq/L (3.7 mmol/L)
Chloride, 102 mEq/L (102 mmol/L)
Bicarbonate, 16 mEq/L (16 mmol/L)
Glucose, 100 mg/dL (5.6 mmol/L)
Blood urea nitrogen, 34 mg/dL (12.1 mmol/L)
Creatinine, 0.6 mg/dL (53 mcmol/L)
Urinalysis reveals a specific gravity of 1.030, pH of 5.5, 1+ ketones, and otherwise negative
findings.
Of the following, the MOST likely urine sodium concentration and urine osmolality for this patient
are
Urine Sodium Urine Osmolality
A. 6 mEq/L 1,100 mOsm/kg
B. 28 mEq/L 300 mOsm/kg
C. 60 mEq/L 450 mOsm/kg
D. 90 mEq/L 900 mOsm/kg
E. 130 mEq/L 1,100 mOsm/kg

Copyright 2012 American Academy of Pediatrics


2012 PREP SA ON CD-ROM

Critique 49 Preferred Response: A


The boy described in the vignette presents with signs and symptoms consistent with acute
gastroenteritis. During his recovery period, he is rehydrating with oral fluids. His physical examination is
noteworthy for features of reduced effective circulating blood volume (tachycardia, absence of moist
mucous membranes, and delayed capillary refill time), and the laboratory evaluation documents
hyponatremia, a normal anion gap metabolic acidosis, and azotemia (elevation of blood urea nitrogen and
creatinine). The urinalysis is essentially normal, except for a high urine specific gravity and ketonuria.
The challenge to clinicians evaluating children who have hyponatremia is determining the
underlying cause (Item C49) because the appropriate treatment is related to the cause. Most pediatric
disorders of serum sodium concentration (hyponatremia or hypernatremia) are caused by derangements
of water balance: excess water to sodium in cases of hyponatremia and insufficient water to sodium for
hypernatremia. Because water balance is critical to maintaining a normal serum sodium concentration,
antidiuretic hormone (ADH) and the renal response to this hormone are of paramount importance. For
water excretion to occur via the kidneys, the patient must have normal renal perfusion, normal renal
function, and normal ability to inhibit the response to ADH.
Another step in the evaluation is to classify patients as hypovolemic, euvolemic, or hypervolemic,
based on the history and physical examination findings. Hypovolemic disorders are characterized by
losses of sodium and water. In some cases, sodium and fluid losses through the kidneys or skin result in
hypovolemia. The hyponatremia can be aggravated by fluid replacement of hypotonic fluids, which are
avidly retained by the body in the setting of hypovolemia due to appropriate ADH secretion, sometimes
known as syndrome of appropriate ADH secretion (SAADH). Euvolemic or borderline hypervolemic states
associated with hyponatremia include hypothyroidism, glucocorticoid deficiency, water intoxication, and
syndrome of inappropriate ADH secretion (SIADH). Hypervolemic states are characterized by volume
overload (eg, edema) and hyponatremia.
One test that is particularly useful in the evaluation of hyponatremia is measurement of the urine
sodium concentration. Patients who have hyponatremia and hypovolemia of extrarenal origin (eg, acute
gastroenteritis) can be expected to have an activated renin-angiotensin-aldosterone axis and increased
ADH secretion. These compensatory mechanisms cause avid sodium and water retention in an effort to
restore the intravascular volume to normal. As a result, the urine sodium is low (<20 mEq/L) and the urine
osmolality is elevated (>600 mOsm/kg). In contrast, conditions of hyponatremia and hypovolemia
associated with renal dysplasia or a tubulopathy feature elevated urine sodium (>40 mEq/L) with an
inability to concentrate the urine (<300 mOsm/kg). Hypervolemic states associated with decreased
effective circulating blood volume (eg, congestive heart failure, cirrhosis, or nephrotic syndrome)
generally behave like hypovolemia from a renal perspective and are characterized by a low urine sodium
and high urine osmolality. Patients who have SIADH often have elevated urine sodium concentrations
(>40 mEq/L) and a high urine osmolality (often >600 mOsm/kg).
Because the boy in the vignette has hyponatremia and hypovolemia associated with decreased
effective circulating blood volume and gastroenteritis, his urine sodium should be low (6 mEq/L) and urine
osmolality should be high (1,100 mOsm/kg). His clinical condition and high urine osmolality should not be
mistaken for SIADH, which would lead to the erroneous treatment approach of fluid/free water restriction.
Rather, his ADH secretion is appropriate (SAADH), and once this diagnosis is established, he should
receive fluid resuscitation with isotonic fluids to minimize the addition of free water, which could result in
worsening hyponatremia.

American Board of Pediatrics Content Specification(s):


Recognize the importance of urinary sodium concentration and urinary osmolality in the differential
diagnosis of hyponatremia

Suggested Reading:
Farrell C, Del Rio M. In brief: hyponatremia. Pediatr Rev. 2007;28:426-428. DOI: 10.1542/pir.28-11-426.
Available at: http://pedsinreview.aappublications.org/cgi/content/full/28/11/426

Copyright 2012 American Academy of Pediatrics


2012 PREP SA ON CD-ROM

Quan A, Quigley R, Satlin LM, Baum M. Water and electrolyte handling by the kidney. In: Kher KK,
Schnaper HW, Makker SP, eds. Clinical Pediatric Nephrology. 2nd ed. London, England: Informa Healthcare;
2007:15-35

Rivkees SA. Differentiating appropriate antidiuretic hormone secretion, inappropriate antidiuretic hormone
secretion and cerebral salt wasting: the common, uncommon, and misnamed. Curr Opin Pediatr.
2008;20:448-452

Rose BD, Post TW. Hypoosmolal states-hyponatremia. In: Clinical Physiology of Acid-base and
Electrolyte Disorders. 5th ed. New York, NY: McGraw-Hill Medical Publishing Division; 2001:696-745

Watkins S, Okamura D, Rodriguez Soriano J. Hyponatremia. In: Zelikovic I, Eisenstein I, eds. Practical
Algorithms in Pediatric Nephrology. Basel, Switzerland: Karger; 2008:72-73

Copyright 2012 American Academy of Pediatrics


2012 PREP SA ON CD-ROM

Critique 49

Item C49. Causes of Hyponatremia

Hypovolemic States: Reduced total body sodium with reduced total body water
(water in excess of sodium)
! Gastrointestinal losses
- Vomiting
- Diarrhea
! Renal losses
- Renal dysplasia (renal salt wasting)
- Tubulopathies (Bartter syndrome or Gitelman syndrome)
- Diuretics (usually thiazides)
- Congenital adrenal hyperplasia
- Cerebral salt wasting
! Skin losses
- Cystic fibrosis
- Burns
Euvolemic States: Normal sodium with slight excess water (water > sodium)
! Syndrome of inappropriate antidiuretic hormone secretion
! Hypothyroidism
! Adrenal insufficiency
! Water intoxication (psychogenic polydipsia)
Hypervolemic States: Excess total body sodium and water (water > sodium)
! Congestive heart failure
! Nephrotic syndrome
! Cirrhosis
! Renal failure

Copyright 2011 American Academy of Pediatrics


2012 PREP SA ON CD-ROM

Question 50
You are evaluating a 6-month-old girl who has mild eczema that improves with regular use of a
topical moisturizer with a group of pediatric residents. After evaluating the infant, you decide to discuss
atopy and its components with them.
Of the following, the MOST accurate statement regarding components of atopy is that
A. allergic rhinitis is not a risk factor for the development of asthma
B. children who wheeze before 1 year of age are at higher risk for persistent asthma compared with
children who begin wheezing after 6 years of age
C. early child care exposure reduces the risk for the development of asthma
D. exclusive breastfeeding longer than 6 months significantly reduces asthma incidence beyond 6
years of age
E. maternal dietary restriction during breastfeeding prevents the development of atopy

Copyright 2012 American Academy of Pediatrics


2012 PREP SA ON CD-ROM

Critique 50 Preferred Response: C


Atopy represents the genetic predisposition to develop allergic diseases such as atopic
dermatitis, allergic rhinitis, food allergy, and asthma. Not only has the risk of atopy increased over the
past 30 years, but having one component of atopy portends a threefold risk for developing another atopic
condition. Many variables are related to the risk for development or prevention of atopic conditions, and a
number have been shown clearly to influence the development of asthma. For example, early child care
exposure (ie, <6 months old) or having multiple siblings in the household increases the risk for lower
respiratory tract infections but is protective against the development of atopy. The association of viral
illnesses and decreased asthma risk supports the hygiene hypothesis that states that infants are born
with a predominantly Th2based immune system, but through natural infection, the response shifts from a
Th2 to a Th1 immune response. Without these infections, the bodys immune system is more likely to
remain a Th2-based immune response that produces cytokines that favor immunoglobulin (Ig)E-mediated
responses.
Longitudinal studies from the Tucson Childrens Respiratory Study have identified three asthma
phenotypes: transient wheezers, nonatopic wheezers, and atopic wheezers. More than 80% of transient
wheezers who begin wheezing before 1 year of age have the condition resolve by age 3 years. This
group does have smaller airways compared with other groups but is not at increased risk to develop later
asthma. In contrast, children who begin wheezing after age 3 (late wheezers) are at increased risk for
IgE-mediated aeroallergen sensitization and the development of allergic asthma.
Although it is difficult to compare studies of the duration of breastfeeding and atopy risk,
breastfeeding for at least 3 months reduces the risk for eczema. The protective benefit of breastfeeding
for longer than 6 months becomes more complex when controlling for maternal atopy. One large
longitudinal study found that infants who were breastfed exclusively by mothers who had high IgE
concentrations were actually at increased risk for asthma at age 6 years.
Currently, no evidence supports maternal dietary restriction to reduce the risk of atopy in children.
Some investigators have examined administration of probiotics (eg, Lactobacillus rhamnosus) to the
mother 2 to 4 weeks before delivery and subsequently to the infant for 6 months. One study
demonstrated a reduction in eczema at 2 years but no reduction in asthma, IgE concentrations, or
allergen sensitization.

American Board of Pediatrics Content Specification(s):


Know that children with one component of atopy syndrome (allergic rhinitis, asthma, eczema) have a
threefold greater risk of developing a second component

Suggested Reading:
Greer FR, Sicherer SH, Burks AW; Committee on Nutrition and Section on Allergy and Immunology.
Effects of early nutritional interventions on the development of atopic disease in infants and children: the
role of maternal dietary restriction, breastfeeding, timing of introduction of complementary foods, and
hydrolyzed formulas. Pediatrics. 2008;121:183-191. DOI: 10.1542/peds.2007-3022. Available at:
http://pediatrics.aappublications.org/cgi/content/full/121/1/183

Mihrshahi S, Ampon R, Webb K, et al; CAPS Team. The association between infant feeding practices
and subsequent atopy among children with a family history of asthma. Clin Exp Allergy. 2007;37:671-679.
DOI: 10.1111/j.1365-2222.2007.02696.x. Abstract available at:
http://www.ncbi.nlm.nih.gov/pubmed/17456214

Snijders BE, Thijs C, Dagnelie PCet al. Breast-feeding duration and infant atopic manifestations, by
maternal allergic status, in the first 2 years of life (KOALA Study). J Pediatr. 2007;151:347-351. DOI:
10.1016/j.jpeds.2007.03.022. Available at: http://www.jpeds.com/article/S0022-3476(07)00262-4/fulltext

Copyright 2012 American Academy of Pediatrics


2012 PREP SA ON CD-ROM

Question 51
A 14-year-old boy is brought to the emergency department after being struck by a car. On arrival,
he is unresponsive and hypotensive. You intubate him endotracheally, place two large-bore intravenous
lines, and infuse 3 L of 0.9% saline. Following these measures, his heart rate is 100 beats/min and blood
pressure is 100/60 mm Hg. On secondary survey, you find a large swelling on the back of his head, a
distended abdomen, blood at the urethral meatus, guaiac-positive stool, and a right femur fracture.
Of the following, the procedure that is CONTRAINDICATED in this patient is
A. diagnostic peritoneal lavage
B. femoral traction splint placement
C. orogastric tube placement
D. retrograde urethrography
E. urethral catheter placement

Copyright 2012 American Academy of Pediatrics


2012 PREP SA ON CD-ROM

Critique 51 Preferred Response: E


The boy described in the vignette has sustained multiple significant injuries and requires
evaluation and stabilization. All of the procedures listed are potentially indicated, with the exception of
urethral catheter placement. The presence of blood at the boys urethral meatus raises the clinical
concern for urethral trauma, and the continuity of the urethra must be ascertained before any
instrumentation is undertaken.
Urethral trauma in children is not common. When it does occur, it is seen in the setting of severe
pelvic or straddle injuries. Urethral injury should be suspected in patients who have known or suspected
pelvic injury, lower abdominal trauma, direct perineal injury, scrotal hematoma, blood at the meatus, or
inability to void. Before urethral catheterization, patients should undergo retrograde urethrography, a
procedure in which a small catheter is placed in the urethral meatus, the balloon is inflated to occlude the
urethral opening, and 10 mL of radiographic contrast is instilled. A plain radiograph is obtained or
fluoroscopy is performed. Any evidence of contrast extravasation is an indication of urethral injury and
should prompt consultation with a surgeon or urologist.

American Board of Pediatrics Content Specification(s):


Understand that gross urethral bleeding is a contraindication to catheterization following acute trauma

Suggested Reading:
American College of Surgeons. Advanced Trauma Life Support. 8th ed. Chicago, IL: American College of
Surgeons; 2008

Avarello JT, Cantor RM. Pediatric major trauma: an approach to evaluation and management. Emerg
Med Clin North Am. 2007;25:803836. DOI: 10.1016/j.emc.2007.06.013. Abstract available at:
http://www.ncbi.nlm.nih.gov/pubmed/17826219

Shlamovits GZ, Mower WR, Bergman J, et al. Lack of evidence to support routine digital rectal
examination in pediatric trauma patients. Pediatr Emerg Care. 2007;23:537-543. DOI:
10.1097/PEC.0b013e318128f836. Abstract available at: http://www.ncbi.nlm.nih.gov/pubmed/17726412

Copyright 2012 American Academy of Pediatrics


2012 PREP SA ON CD-ROM

Question 52
You are called to the emergency department to see a 5-day-old infant, who has presented
because of bile-stained emesis. When the infant was born at term following an uncomplicated pregnancy
and delivery, he weighed 3,300 g. He was sent home at 36 hours of age. Since discharge, he has been
nursing every 2 to 3 hours at home, but he recently developed apparent discomfort postprandially. His
mother has observed no stools since discharge. This morning, he began to spit up after feeding, and the
last emesis was bile-stained. Upon arrival at the emergency department, the infant appeared alert but
irritable and had moderate, generalized abdominal distension. The emergency department physician
obtained an upright abdominal radiograph (Item Q52).
Of the following, the MOST appropriate next step is
A. abdominal computed tomography scan
B. contrast enema
C. rectal suction biopsy
D. surgical decompression
E. upper gastrointestinal tract radiographic series

Copyright 2012 American Academy of Pediatrics


2012 PREP SA ON CD-ROM

Question 52

(Courtesy of S Schwarz)

Copyright 2012 American Academy of Pediatrics


2012 PREP SA ON CD-ROM

Critique 52 Preferred Response: B


The newborn described in the vignette presents with overt signs of intestinal obstruction,
manifested by marked abdominal distension and bile-stained emesis. An abdominal film confirms this
clinical impression, demonstrating diffuse bowel loop distension. Importantly, no signs of perforation (eg,
free air in the peritoneal cavity) are apparent. In addition, no stool passage has been observed in this 5-
day-old infant since discharge. These clinical and radiologic findings strongly suggest distal bowel
obstruction, with the most likely diagnosis being either Hirschsprung disease or meconium ileus (MI).
Accordingly, the most appropriate next step in management is a contrast enema with water-soluble
contrast material. This procedure has the potential to both identify the level of obstruction and relieve the
obstruction. Surgical decompression may be indicated if this is unsuccessful. Whenever a distal bowel
obstruction is suspected, oral contrast media (eg, computed tomography scan with contrast, upper
gastrointestinal tract radiographic series) should be avoided. The rectal suction biopsy may be indicated
in young infants who have suspected Hirschsprung disease and do not present with clinical signs of
obstruction.
Although MI has been reported in infants who do not have an underlying medical disorder, more
than 95% of cases are associated with cystic fibrosis (CF), with an incidence as high as 20% in some CF
series. The greatest risk factor for MI and the most complex cases are associated with CF patients who
are homozygous for the F508 gene mutation. MI is caused by abnormally sticky and viscid meconium,
which is the result of both intestinal and pancreatic dysfunction. Albumin is the major protein present in
the meconium of infants who have MI, with a concentration 5 to 10 times higher than
normaljavascript:showcontent('active','references');. This increased albumin load produces a viscid
meconium that, along with increased intraluminal concentrations of -glutamyltranspeptidase (GGTP) and
5'-nucleotidase in the meconium of CF patients, produces MI. Because the addition of pancreatic
proteases liquefies the viscid meconium, pancreatic insufficiency has been considered an important
etiologic factor in MI, but the disorder has been reported in patients who do not have pancreatic secretory
failure. Abnormal intestinal motility, with prolonged transit times, may also contribute to MI development.
The most common presentation of MI is abdominal distension at birth, failure to pass meconium
(thus, the consideration of Hirschsprung disease in the differential diagnosis), and bilious vomiting.
Physical examination often suggests dilated loops of bowel, and rectal examination demonstrates a
narrowed vault, again suggesting Hirschsprung disease as a potential diagnosis. In all uncomplicated
cases in which bowel perforation is not suspected, the diagnostic and therapeutic intervention of choice is
a nonoperative water-soluble contrast enema. Surgical intervention, most frequently enterotomy and
bowel irrigation, should be reserved for enema failures.
The early reported success rate of water-soluble contrast enemas for patients who had
uncomplicated MI was 63% to 83%, but a recent analysis suggests a much lower current success rate in
the range of 35%. Although the reasons for the reduced rate of nonoperative success are unclear, several
contributing factors, including delay in diagnostic/therapeutic intervention, experience of the radiologist
performing the examination, and reluctance to perform repeated enemas, may be important. The use of
water-soluble contrast enemas to relieve MI was first described in 1969, and the initial procedural criteria
are still followed, including:
The initial contrast enema must exclude other causes of neonatal distal intestinal obstruction.
No clinical or radiologic signs of complication should be evident (eg, perforation, peritonitis, sepsis,
volvulus).
Adequate fluid and electrolyte replacement should be achieved before and maintained throughout the
study.
The enema must be performed under fluoroscopic control.
Intravenous antibiotics should be administered before and after the procedure.
Close surgical supervision must be provided throughout the hospital course.
Because MI is virtually diagnostic of CF, infants such as the one in the vignette should be treated
as having CF until confirmatory tests (sweat test or genotyping) are obtained. In most patients, the serum
immunoreactive trypsinogen newborn screen is positive for CF, although definitive diagnosis requires
additional testing.

American Board of Pediatrics Content Specification(s):

Copyright 2012 American Academy of Pediatrics


2012 PREP SA ON CD-ROM

Know the etiology, diagnostic evaluation, and treatment of meconium ileus

Suggested Reading:
Blackman SM, Deering-Brose R, McWilliams R et al. Relative contribution of genetic and nongenetic
modifiers to intestinal obstruction in cystic fibrosis Gastroenterology. 2006;131:1030-1039. DOI:
10.1053/j.gastro.2006.07.016. Available at:
http://www.ncbi.nlm.nih.gov/pmc/articles/PMC1764617/?tool=pubmed

Burke MS, Ragi JM, Karamanoukian HL, et al. New strategies in nonoperative management of meconium
ileus. J Pediatr Surg. 2002;37:760-764. DOI: 10.1053/jpsu.2002.32272. Abstract available at:
http://www.ncbi.nlm.nih.gov/pubmed/11987095

Chaudry G, Navarro OM, Levine DS, Oudjhane K. Abdominal manifestations of cystic fibrosis in children.
Pediatr Radiol. 2006;36:233-240. DOI: 10.1007/s00247-005-0049-2. Available at:
http://www.springerlink.com/content/yr2622r276511686/

Copeland DR, St Peter SD, Sharp SW, et al. Diminishing role of contrast enema in simple meconium
ileus. J Pediatr Surg. 2009;44:2130-2132. DOI: 10.1016/j.jpedsurg.2009.06.005. Abstract available at:
http://www.ncbi.nlm.nih.gov/pubmed/19944221

Noblett HR. Treatment of uncomplicated meconium ileus by Gastrografin enema: a preliminary report. J
Pediatr Surg. 1969;4:190-197. Abstract available at: http://www.jpedsurg.org/article/0022-3468(69)90390-
X/abstract

Ziegler MM. Meconium ileus. Curr Probl Surg. 1994;9:731-777. Abstract available at:
http://www.ncbi.nlm.nih.gov/pubmed/8062591

Copyright 2012 American Academy of Pediatrics


2012 PREP SA ON CD-ROM

Question 53
You are called to attend an urgent cesarean section delivery at 36 weeks' gestation necessitated
by maternal fever, fetal tachycardia, and a nonreassuring fetal heart tracing. The prenatal course has
been uneventful, but the group B Streptococcus status is unknown. Six hours before delivery, the mother
began to develop fever, nausea, and vomiting, followed by the onset of contractions. Due to a history of
anaphylaxis to penicillin, she received one dose of vancomycin during labor. Artificial rupture of the
membranes occurs at the time of the cesarean section and reveals light meconium-stained fluid. The
infant emerges vigorous, and physical examination reveals a faint erythematous rash, tachypnea, and
grunting (Item Q53).
Of the following, the MOST appropriate therapy to initiate for this infant is
A. acyclovir
B. amphotericin B
C. ampicillin
D. ceftriaxone
E. vancomycin

Copyright 2012 American Academy of Pediatrics


2012 PREP SA ON CD-ROM

Question 53

(Reprinted with permission from Heras PC, Garcia-Patos V, Palacio L, et al. Actas Dermosifiliogr.
2006;97:59-61)
Rash, as described for the infant in the vignette.

Copyright 2012 American Academy of Pediatrics


2012 PREP SA ON CD-ROM

Critique 53 Preferred Response: C


The infant described in the vignette is at risk for early-onset neonatal sepsis, likely caused by
Listeria monocytogenes, and antimicrobial coverage with ampicillin and an aminoglycoside should be
initiated promptly. The maternal presentation of preterm labor accompanied by fever and gastrointestinal
symptoms combined with meconium-stained amniotic fluid is suggestive of maternal listeriosis. The faint
erythematous rash with small, pale nodules, also known as granulomatosis infantisepticum, may be
seen in affected infants.
Early-onset neonatal sepsis develops before 7 days of age and often is a result of vertical
transmission of the organism from the mothers genital tract. The leading bacterial cause of early-onset
sepsis in the United States remains group B Streptococcus (GBS), despite an 80% decline after the
initiation of universal GBS screening. Escherichia coli is the second most common bacterial pathogen,
and other causative organisms include Haemophilus influenzae, Enterococcus, Staphylococcus aureus,
and Streptococcus viridians. L monocytogenes has become relatively uncommon, although it must be
considered when the mother presents in preterm labor with an influenzalike illness, fever, gastrointestinal
symptoms, and meconium-stained fluid. The leading cause of early-onset sepsis in developing countries
is gram-negative organisms.
Empiric antibiotic treatment is designed to address the most commonly found pathogens for
early-onset neonatal sepsis. GBS, Listeria, and Enterococcus are susceptible to ampicillin, although
increasing resistance has been shown by E coli. An aminoglycoside, most commonly gentamicin, is used
for coverage of gram-negative enteric bacilli. Many centers change to or add a third-generation
cephalosporin for improved central nervous system penetration if meningitis is suspected. Third-
generation cephalosporins are less effective against GBS and are ineffective against Listeria and
Enterococcus.
Although ceftriaxone is a third-generation cephalosporin, it is rarely indicated in the first weeks
after birth because of the risk of neonatal death resulting from precipitation when infused with intravenous
calcium as well as the increased risk of kernicterus due to competitive binding with albumin. Amphotericin
B is used for treatment of fungal sepsis, which is seldom seen at birth, except in the extremely low-
birthweight population. Vancomycin is used primarily for nosocomial infection with Staphylococcus
epidermidis. Neonatal herpes simplex virus (HSV) infection must be considered for any sick infant
because the diagnosis may be difficult to determine, and more than 75% of affected infants are born to
mothers who have no history of active HSV. It would not be unreasonable to add acyclovir if this infants
clinical status continued to deteriorate.

American Board of Pediatrics Content Specification(s):


Know the appropriate antibiotic treatment for suspected sepsis in the immediate newborn period

Suggested Reading:
American Academy of Pediatrics. Listeria monocytogenes infections. In: Pickering LK, Baker CJ,
Kimberlin DW, Long SS, eds. Red Book: 2009 Report of the Committee on Infectious Diseases. 28th ed.
Elk Grove Village, IL: American Academy of Pediatrics; 2009:428-430

Benitz WE. Neonatal sepsis. In: Polin RA, Yoder MC, eds. Workbook in Practical Neonatology. 4th ed.
Philadelphia, PA: Saunders Elsevier; 2007:221-247

Centers for Disease Control and Prevention. Trends in perinatal group B streptococcal disease-United
States, 2000-2006.MMWR Morb Mortal Wkly Rep. 2009;58:109-112. Available at:
http://www.cdc.gov/mmwr/preview/mmwrhtml/mm5805a2.htm

Centers for Disease Control and Prevention. Prevention of perinatal group B streptococcal disease.
Revised guidelines from CDC, 2010. MMWR Recomm Rep. 2010;59(RR10):1-32. Available at:
http://www.cdc.gov/mmwr/preview/mmwrhtml/rr5910a1.htm?s_cid=rr5910a1_w

Schrag SJ, Stoll BJ. Early-onset neonatal sepsis in the era of widespread intrapartum chemoprophylaxis.
Pediatr Infect Dis J. 2006;25: 939-940. DOI: 10.1097/01.inf.0000239267.42561.06

Copyright 2012 American Academy of Pediatrics


2012 PREP SA ON CD-ROM

Question 54
You have been following a 16-year-old girl for symptoms of irritability, loss of interest in activities,
and decreased appetite. She has experienced these symptoms every day for several weeks. She has
stopped attending band practice, which she previously enjoyed. You offer treatment with a selective
serotonin reuptake inhibitor (SSRI) and cognitive behavioral therapy. Her mother asks you how long her
daughter will need to take the SSRI before determining if it is effective.
Of the following, the MOST appropriate timeframe to assess medication efficacy is
A. 1 to 2 days
B. 1 to 2 weeks
C. 4 to 6 weeks
D. 8 to 10 weeks
E. 12-14 weeks

Copyright 2012 American Academy of Pediatrics


2012 PREP SA ON CD-ROM

Critique 54 Preferred Response: C


Therapy involving an selective serotonin reuptake inhibitor (SSRI) for depression in an
adolescent typically is initiated at a low dose (eg, 10 mg daily for fluoxetine) and gradually increased as
often as every 2 weeks, depending on depressive symptoms and adverse effects. Effects on depressive
symptoms may not be fully apparent until 4 to 6 weeks of therapy have been completed. Counseling the
patient and family that 4 to 6 weeks of therapy at a given dose is required for full effects is important to
assure compliance. It is unlikely to see improvements in 1 to 2 days, other than placebo effects.
Patients should be reevaluated within 1 to 2 weeks of treatment initiation for adverse effects and
increased depressive symptoms and to improve adherence to therapy. Adverse effects of SSRIs may
include sleep disturbance, gastrointestinal upset, headaches, agitation, and suicidal ideation.
The patient should be assessed at 4 to 6 weeks after initiating an SSRI to determine if the
medication is providing adequate benefit. If symptoms have improved and adverse effects are tolerable,
the patient can continue with the same dose to see if improvements continue over time. If only partial
benefit is apparent when a full dose of the SSRI has been reached, the medication could be changed or
consideration given to adding another antidepressant medication that has a different mechanism of
action to the regimen. If there is no improvement, consideration should be given to changing the
medication to another SSRI. Failure of two different SSRIs to improve symptoms indicates a more difficult
problem, and if not done so before, the clinician should consider consulting a child psychiatrist.

AAP Mental Health Competency:


Know the duration of time to wait before dose increases of SSRI medications

Suggested Reading:
Cheung AH, Zuckerbrot RA, Jensen PS, Ghalib K, Laraque D, Stein REK, GLAD-PC Steering Group.
Guidelines for adolescent depression in primary care (GLAD-PC): II. treatment and ongoing
management. Pediatrics. 2007(5);120:e1313-e1326

Jensen PS, Cheung AH, Zuckerbrot R,Ghalib K, Levitt A. Guidelines for Adolescent Depression in
Primary Care (GLAD-PC) Tool Kit. 2007. Available at:
http://www.thereachinstitute.org/files/documents/GLAD-PCToolkit.pdf.

Prager LM. Depression and suicide in children and adolescents. Pediatr Rev. 2009;30:199-205. DOI:
10.1542/pir.30-6-199. Available at: http://pedsinreview.aappublications.org/cgi/content/full/30/6/199

Zuckerbrot RA, Cheung AH, Jensen PS, Stein REK, Laraque D, and the GLAD-PC Steering Group.
Guidelines for adolescent depression in primary care (GLAD-PC): I. identification, assessment, and initial
management. Pediatrics. 2007;120(5):e1299-e1312. Available at:
http://pediatrics.aappublications.org/content/120/5/e1299

Copyright 2012 American Academy of Pediatrics


2012 PREP SA ON CD-ROM

Question 55
A 15-year-old girl is brought to your office after passing out while she was participating in a
band program outside on an 80.0F day. She recalls feeling lightheaded, then awakening surrounded by
her bandmates. The reported duration of the episode was 1 minute. She has had one similar episode in
the past. She has no underlying medical problems, and there is no family history of seizures or heart
disease. Currently, her temperature is 37.3C, heart rate is 84 beats/min, respiratory rate is 18
breaths/min, and blood pressure is 98/64 mm Hg. The remainder of her findings, including those of
cardiovascular and neurologic examinations, are normal.
Of the following, the MOST appropriate next step in her evaluation is
A. cardiac event monitoring
B. computed tomography scan of the brain
C. electrocardiography
D. electroencephalography
E. tilt table testing

Copyright 2012 American Academy of Pediatrics


2012 PREP SA ON CD-ROM

Critique 55 Preferred Response: C


Syncope, as experienced by the girl in the vignette, is defined as the transient loss of
consciousness and postural tone associated with an acute disruption in cerebral perfusion. It is a
common pediatric problem, occurring at least once in 20% or more of all persons before age 20,
according to one survey. Because it has a wide variety of underlying causes, some of which are life-
threatening, a number of tests may be considered to evaluate this phenomenon. In most patients, the
cause is benign, and the evaluation should be based primarily on history and physical examination
findings. Despite its frequency, there is not yet a standardized approach to the evaluation of children and
adolescents who experience syncope. Although the data are limited, most authors agree that
electrocardiography (ECG) may be the only test required for patients who have a typical history and
normal physical examination findings, such as the girl described in the vignette.
The underlying cause for syncope can be cardiac, neurologic, psychological, or metabolic, but in
most studies, about 75% of cases are diagnosed as neurocardiogenic (also known as simple faint or
vasovagal or neurally mediated syncope). Neurally mediated syncope (NMS) can be further divided into
central (eg, pain- or emotion-induced), postural (associated with prolonged standing or sitting), and
situational (eg, micturition, coughing, hair grooming-induced). The exact mechanisms causing NMS are
unclear but involve complex interactions of parasympathetic, sympathetic, and cardiac reflexes that can
result in hypotension or bradycardia. An interesting recent study suggests a higher prevalence of iron
deficiency with or without anemia in patients who have NMS, so testing for this possibility (eg, with a
ferritin measurement) might be considered in some patients.
Typically, the patient has been standing, usually for at least several minutes to sometimes hours,
often in a warm environment. There is a distinct prodromal phase of seconds to a few minutes of nausea,
clamminess, pallor, dizziness, visual and hearing changes, and weakness. This is followed by loss of
consciousness and collapse. Recovery is usually quick, in 1 to 2 minutes, with no subsequent symptoms
besides fatigue or a feeling of shakiness. Alternatively, the patient or observers may describe inciting
events such as blood draws, injections, or hair grooming.
Perhaps the most concerning potential etiologic category for syncope is cardiac, with conditions
ranging from arrhythmias (especially prolonged QT syndrome) to pulmonary hypertension to
cardiomyopathy. Studies indicate that a cardiac diagnosis can be found in 2% to 3% of children who have
syncope. The history in such patients differs from that of children who have NMS and may direct the
clinician to the cause. The child may have a past history of congenital heart disease; syncope may occur
during exercise or suddenly without prodrome; there may be a family history of cardiomyopathy, sudden
death, or deafness; or the child may experience atypical precordial pain or a sensation of palpitations. A
personal or family history of recurrent syncope also may suggest an underlying cardiac cause. Although a
normal ECG in the presence of the usual history of NMS syncope reassures the examiner of a low
likelihood of an arrhythmia, if the history suggests a cardiac diagnosis, further testing may be warranted.
In that case, echocardiography may detect structural abnormalities, and a cardiac event monitor or tilt
table test may reveal an arrhythmia not present on ECG. However, in one study, echocardiography
accounted for 14% of the costs of testing children who had syncope and did not reveal a single
abnormality not already detected by history, physical examination, and ECG. Despite its utility in adults
who have syncope, cardiac event monitoring has not yielded substantial benefit in children, possibly
because of the lower incidence of heart disease. Tilt table testing has shown a high yield of diagnoses,
but there is concern about low specificity of the test, and it is not necessary with a typical history and
physical examination findings associated with NMS.
Other causes of transient loss of consciousness in one study of children included seizures
(1.9%), migraines (0.2%), conversion reactions (1.4%), hypoglycemia (0.4%), severe anemia (0.2%), and
hyperventilation (0.2%). In this study, cranial imaging was not found to be helpful in diagnosing a single
case. Electroencephalography was diagnostic in 39% of patients, but all of the children had history or
examination findings indicative of underlying neurologic disease. History and physical examination
findings also typically led to the appropriate diagnosis of hypoglycemia and anemia.

American Board of Pediatrics Content Specification(s):


Understand that the description of a syncopal episode usually directs the evaluation

Copyright 2012 American Academy of Pediatrics


2012 PREP SA ON CD-ROM

Suggested Reading:
Jarjour IT, Jarjour LK. Low iron storage in children and adolescents with neutrally mediated syncope. J
Pediatr. 2008;153:40-44. DOI: 10.1016/j.jpeds.2008.01.034. Abstract available at:
http://www.ncbi.nlm.nih.gov/pubmed/18571533

Ramaswamy P. Syncope. In: McInerny TK, Adam HM, Campbell DE, Kamat DM, Kelleher KJ, Hockelman
RA, eds. American Academy of Pediatrics Textbook of Pediatric Care. Elk Grove Village, IL: American
Academy of Pediatrics; 2009:1753-1757

Steinberg LA, Knilans TK. Syncope in children: diagnostic tests have a high cost and low yield. J Pediatr.
2005;146:355-358. DOI: 10.1016/j.jpeds.2004.10.039. Abstract available at:
http://www.ncbi.nlm.nih.gov/pubmed/15756219

Strieper MJ. Distinguishing benign syncope from life-threatening cardiac causes of syncope. Semin
Pediatr Neurol. 2005;12:32-38. Abstract available at: http://www.ncbi.nlm.nih.gov/pubmed/15929463

Zhang Q, Du J, Wang C, Du Z, Wang L, Tang C. The diagnostic protocol in children and adolescents with
syncope: a multi-centre prospective study. Acta Paediatr. 2009;98:879-884. DOI: 10.1111/j.1651-
2227.2008.01195.x. Abstract available at: http://www.ncbi.nlm.nih.gov/pubmed/19183119

Copyright 2012 American Academy of Pediatrics


2012 PREP SA ON CD-ROM

Question 56
The father of a 3-week-old baby calls for an urgent appointment because he has noted blood in
her last two diapers. When he arrives, he shows you two diapers, each of which contains a dime-sized
spot of blood. The baby is thriving on human milk. She is active and has gained 500 g over her
birthweight. The father states that the blood seems to be coming from her vagina. He also is concerned
about asymmetric swelling of the infants breast, which is not accompanied by redness.
Of the following, this constellation of signs and symptoms is BEST characterized by exposure to
maternal
A. cortisol
B. estrogen
C. oxytocin
D. progesterone
E. prolactin

Copyright 2012 American Academy of Pediatrics


2012 PREP SA ON CD-ROM

Critique 56 Preferred Response: B


The infant described in the vignette is displaying the effects of withdrawal of maternal estrogens:
physiologic pseudomenses and benign neonatal gynecomastia. Such breast development is often
asymmetric and not associated with signs of infection such as erythema, fever, or purulent discharge,
which would be observed in neonatal mastitis. The vaginal bleeding is usually short-lived, lasting only a
few days. If the bleeding persists, other conditions such as gynecologic tumors (eg, sarcoma botryoides)
or an underlying bleeding disorder should be considered. This infants father can be reassured that
neonatal vaginal bleeding and gynecomastia are benign conditions that do not require intervention. There
are no known syndromes associated with withdrawal of maternal cortisol, oxytocin, progesterone, or
prolactin. Other associations with hormonal changes in the newborn may include neonatal acne.

American Board of Pediatrics Content Specification(s):


Recognize the influence of maternal estrogens on an infant during the first three weeks of life

Suggested Reading:
Diamantopoulos S, Bao Y. Gynecomastia and premature thelarche: a guide for practitioners. Pediatr Rev.
2007;28:e57-e68. DOI:10.1542/pir.28-9-e57. Available at:
http://pedsinreview.aappublications.org/cgi/content/full/28/9/e57

Sugar NF, Graham EA. Common gynecologic problems in prepubertal girls. Pediatr Rev. 2006;27:213-
223. DOI: 10.1542/pir.27-6-213. Available at:
http://pedsinreview.aappublications.org/cgi/content/full/27/6/213

Copyright 2012 American Academy of Pediatrics


2012 PREP SA ON CD-ROM

Question 57
A 6-year-old previously healthy boy presents with the recent development of nocturnal dyspnea.
On questioning of his parents, you discover that the child has experienced exercise intolerance, two
episodes of syncope while running, poor appetite, and a cough without congestion over the past year. His
physical examination reveals a heart rate of 120 beats/min, respiratory rate of 26 breaths/min, gallop
rhythm, III/VI high-pitched blowing systolic murmur at the apex, hepatomegaly, and diminished pulses.
Chest radiography documents an enlarged cardiac silhouette with pulmonary vascular congestion (Item
Q57), and echocardiography demonstrates a regurgitant mitral valve with a dilated left ventricle and
markedly reduced systolic contractility.
Of the following, the MOST likely cause for this childs dilated cardiomyopathy is
A. a congenital mitral valve abnormality
B. Duchenne muscular dystrophy
C. Friedreich ataxia
D. rheumatic heart disease
E. sickle cell disease

Copyright 2012 American Academy of Pediatrics


2012 PREP SA ON CD-ROM

Question 57

(Courtesy of M Lewin)
Chest radiograph, as described for the boy in the vignette.

Copyright 2012 American Academy of Pediatrics


2012 PREP SA ON CD-ROM

Critique 57 Preferred Response: A


The causes of a dilated cardiomyopathy are diverse and include viral myocarditis, arrhythmias,
metabolic conditions, muscle disorders, drug toxicities, and congenital cardiac lesions. Among the mitral
valve disorders are abnormalities of the valve leaflets or chordal apparatus, which result in either acute
neonatal symptoms or chronic, indolent findings that eventually degenerate. Mitral valve disorders can
occur in isolation or as a part of a complex of left heart pathology that may include abnormalities of the
aortic valve (eg, bicuspid aortic valve with stenosis or regurgitation), aortic arch (coarctation of the aorta),
and left ventricle (hypoplastic left heart syndrome). Mitral valve disease can result in either stenosis or
regurgitation. Both can be tolerated by the child for long periods of time, but eventually they worsen and
result in left atrial dilation, left ventricular dilation, and elevated pressure back into the lungs (pulmonary
hypertension). As the left ventricle continues to dilate, efficiency of contractility is reduced because fiber
cross-linking exceeds the most optimal portion of the Starling curve, and eventually a dilated
cardiomyopathy results. Accordingly, the boy in the vignette most likely has a congenital mitral valve
abnormality and congestive heart failure (CHF).
Symptoms of CHF in older children include exercise intolerance, fatigue, dizziness or syncope,
shortness of breath, palpitations, diaphoresis, abdominal discomfort, and anorexia. Among the signs are
tachycardia, rales, poor tissue perfusion, hepatomegaly, and a gallop rhythm. Auscultation may identify a
blowing systolic ejection murmur at the apex with radiation to the back from mitral regurgitation. Even in
the absence of an intrinsic mitral valve disorder, mitral regurgitation can develop when left ventricular
dilation causes a change in geometry that results in poor leaflet coaptation. Echocardiography is the gold
standard for diagnosing both mitral valve disorders and a cardiomyopathic process. In children, the mitral
valve can be seen in exquisite detail, allowing identification of pathologic features, as described for this
boy. Color Doppler interrogation assists with determination of mitral regurgitation or stenosis. The left
ventricular cavity can be assessed for chamber dilation, and quantitative determination of contractility is
the routine part of echocardiographic assessment.
Duchenne muscular dystrophy often leads to the development of a dilated cardiomyopathy due to
the adverse effect of the dystrophin mutation on the cardiomyocyte. However, the child in the vignette is
younger than the typical child who develops left ventricular dysfunction (unusual before 10 years of age).
In addition, most boys who have Duchenne muscular dystrophy and develop a dilated cardiomyopathy
already manifest skeletal muscle involvement. Friedreich ataxia is associated with the development of a
cardiomyopathy, but it is a hypertrophic rather than a dilated cardiomyopathy. In addition, a child who has
Friedreich ataxia and advanced cardiac findings invariably exhibits ataxia. Mitral valve (as well as aortic
valve) thickening and dysfunction are found in rheumatic heart disease. However, this child has none of
the other cardinal features of acute rheumatic fever (dermatologic, infectious, neurologic, or joint). Sickle
cell disease can result in high-output heart failure due to anemia, but this child does not have the features
of a chronic disease state, which very likely would have developed by the age of 6 years.

American Board of Pediatrics Content Specification(s):


Identify early fatigue, exercise intolerance, anorexia, and cough as symptoms of congestive heart
failure in older children

Suggested Reading:
Macicek SM, Macias CG, Jefferies JL, Kim JJ, Price JF. Acute heart failure syndromes in the pediatric
emergency department. Pediatrics. 2009;124:e898-e904. DOI: 10.1542/peds.2008-2198. Available at:
http://pediatrics.aappublications.org/cgi/content/full/124/5/e898

Madriago E, Silberbach M. Heart failure in infants and children. Pediatr Rev. 2010;31:4-12. DOI:
10.1542/pir.31-1-4. Available at: http://pedsinreview.aappublications.org/cgi/content/full/31/1/4

Copyright 2012 American Academy of Pediatrics


2012 PREP SA ON CD-ROM

Question 58
A 17-year-old boy presents with the complaint of an intermittent spinning sensation with nausea.
He first noted the symptoms this morning when he rolled out of bed. He is a competitive hockey player
and had received a hard blow to the side of his head during a game last night. He is not experiencing
dizziness in your office. Physical examination findings are normal. Neurologic examination shows normal
mental status, with normal visual fields and equally reactive and symmetric pupils. Extraocular
movements are full, with normal horizontal and vertical tracking (visual pursuits), and there is no
nystagmus or reported double vision. Facial sensation and movements are symmetric. Hearing is intact to
finger rub bilaterally. Tongue and palate movements are normal.
Of the following, these findings are MOST consistent with
A. aneurysm of the posterior communicating artery
B. benign paroxysmal positional vertigo
C. complex migraine
D. orthostatic hypotension
E. schwannoma of the vestibular nerve

Copyright 2012 American Academy of Pediatrics


2012 PREP SA ON CD-ROM

Critique 58 Preferred Response: B


Dizziness is a common complaint of patients presenting to physicians offices and emergency
departments. Potential causes are numerous, and some are very serious. Most individuals complaining of
dizziness are light-headed and have benign problems such as presyncope. However, when the patient
describes a spinning sensation, the term vertigo is more appropriate, and the diagnostic investigations
should focus on the pathology, which may be peripheral (vestibular nerve, inner ear) or central
(brainstem) (Item C58).
The boy described in the vignette is experiencing brief bouts of dizziness with a subjective
spinning sensation and nausea. In between bouts, he is asymptomatic, and findings on his examination
are normal. The dizziness occurring when he got out of bed is consistent with benign paroxysmal
positional vertigo (BPPV), a diagnosis that is supported by the report of recent head trauma. Trauma can
dislodge calcium carbonate debris within the inner ears semicircular canal, causing BPPV.
Aneurysms in cerebral blood vessels typically present with a subarachnoid hemorrhage (SAH).
Trauma can be a precipitant for bleeding. The patient in the vignette does not have symptoms of SAH,
such as extremely severe headache and nuchal rigidity. A large posterior communicating artery aneurysm
could cause third nerve compression and palsy, presenting with dilated pupil and everted eye, but would
not cause intermittent bouts of vertigo.
Migraine is a great mimicker of neurologic diseases. It can present in some cases with confusion
or strokelike symptoms, and dizziness is a common accompaniment. A very brief episode of dizziness
when getting out of bed is not consistent with migraine.
Orthostatic hypotension produces light-headedness and feelings of nearly passing out, not
vertigo. Lesions of a vestibular nerve, such as a schwannoma or a viral vestibular neuritis, typically
produce persistent (not episodic) symptoms and examination findings consistent with unilateral pathology.
Examination should show unidirectional horizontal nystagmus caused by the imbalance in peripheral
vestibular nerve input (the eyes drift toward the affected side, then rapidly beat back toward the normally
functioning side).

American Board of Pediatrics Content Specification(s):


Know the common causes of vertigo

Suggested Reading:
Kerber KA. Vertigo and dizziness in the emergency department. Emerg Med Clin North Am. 2009;27:39-
50. DOI: 10.1016/j.emc.2008.09.002. Available at:
http://www.ncbi.nlm.nih.gov/pmc/articles/PMC2676794/?tool=pubmed

Labuguen RH. Initial evaluation of vertigo. Am Fam Physician. 2006;73:244-251. Available at:
http://www.aafp.org/afp/2006/0115/p244.html

Vanderhoff BT, Carroll WE. Neurology. In: Rakel RE, ed. Textbook of Family Medicine. 7th ed.
Philadelpia, PA: Saunders Elsevier; 2007:chapter 54

Copyright 2012 American Academy of Pediatrics


2012 PREP SA ON CD-ROM

Critique 58

Item C58. Causes of Vertigo


Type Conditions
Peripheral Benign paroxysmal vertigo
Vestibular neuronitis
Mnire disease
Endolymphatic hydrops
Benign paroxysmal positional vertigo
Labyrinthitis
Central Migraine-associated dizziness
Demyelinating diseases
Postconcussive
Other Psychogenic

Copyright 2011 American Academy of Pediatrics


2012 PREP SA ON CD-ROM

Question 59
A couple who is planning to bring their child to you for care, present for a prenatal consultation at
28 weeks gestation. After reviewing basic information about your practice, the wife explains that she is
one of six children, four of whom are currently alive. She had a sister and a brother who both died
between the ages of 3 and 4 years. Although normal at birth, both children had progressive loss of
milestones beginning at about 1 year of age, with hypotonia and later severe muscle weakness, difficult-
to-control seizures, visual loss, and eventual death. No other individuals in the extended family had such
problems. The woman is concerned about this family history and wishes to know possible risks for her
children as well as advice about testing for her newborn.
Of the following, you are MOST likely to tell her that her affected siblings probably had
A. an autosomal dominant condition with incomplete penetrance, and the risk to her unborn child
may be as high as 50%
B. an autosomal recessive condition, and the risk to her unborn child is probably low
C. a mitochondrial condition, and the risk to her unborn child may be as high as 100%
D. an X-linked dominant condition, and the risk to her unborn child may be as high as 50%
E. an X-linked recessive condition, and the risk to her unborn child may be as high as 50% if she
gives birth to a son

Copyright 2012 American Academy of Pediatrics


2012 PREP SA ON CD-ROM

Critique 59 Preferred Response: B


The family history described in the vignette is most suggestive of a progressive, neurologically
devastating autosomal recessive condition, such as Tay Sachs disease or metachromatic
leukodystrophy, which can occur in both male and female siblings (Item C59). No other family members
are affected in other generations, and it is not known if the parents of the affected children are blood
relatives (consanguineous). In the absence of consanguinity in this pregnancy, risks for the woman
described in the vignette to have an affected child likely are small, based on this rare autosomal recessive
condition in her siblings. There would be more of a concern for this pregnancy if her siblings had a
relatively common recessive condition such as cystic fibrosis (CF) or sickle cell anemia because the
general population carrier frequency for these conditions is relatively high. Her own risks of being a carrier
of this autosomal recessive condition is 2 in 3, so risks to her pregnancy would rely on the estimated risks
that her partner is also a carrier. If this patient is from an ethnic group that has a higher risk for autosomal
recessive disorders such as Tay-Sachs disease in individuals of Ashkenazi Jewish descent (and her
siblings had a disorder associated with this particular ethnic group), then the risks to her child may be
greater because of the carrier frequency for this condition in this particular population.
An affected male and female in a single sibship is not suggestive of an autosomal dominant, X-
linked dominant, or X-linked recessive condition because if the condition was autosomal or X-linked
dominant, one of the parents should be affected and if the condition was X-linked recessive only boys
should be affected. With mitochondrial inheritance, all children born to a mother who harbors the
mitochondrial mutations in a certain proportion of her mitochondrial genomes would be expected to be
affected as well as potentially the mother herself. Although the onset of symptoms and severity of disease
can vary and may be modified by the relative percentages of normal and abnormal mitochondrial genes
within a given individual, the history provided by the expectant mother is far more suggestive of a life-
limiting neurodegenerative autosomal recessive condition.
For some of the more common autosomal recessive conditions, carrier screening may be
available. Such screening should be considered before conception to allow couples to have the
opportunity for preconception counseling.

American Board of Pediatrics Content Specification(s):


Recognize the clinical and laboratory features associated with an autosomal recessive disorder

Suggested Reading:
Crumrine PK. Degenerative disorders of the central nervous system. Pediatr Rev. 2001;22:370-379. DOI:
10.1542/pir.22-11-370. Available at: http://pedsinreview.aappublications.org/cgi/content/full/22/11/370

Cunniff C and the Committee on Genetics. Prenatal screening and diagnosis for pediatricians. Pediatrics.
2004;114:889-894. DOI: 10.1542/peds.2004-1368. Available at:
http://pediatrics.aappublications.org/cgi/content/full/114/3/889

Fluharty AL. Arylsulfatase A deficiency. GeneReviews. 2008. Available at:


http://www.ncbi.nlm.nih.gov/bookshelf/br.fcgi?book=gene&part=mld

Kaback MM. Hexosaminidase A deficiency. GeneReviews. 2006. Available at:


http://www.ncbi.nlm.nih.gov/bookshelf/br.fcgi?book=gene&part=tay-sachs

Kaback M, Lim-Steele J, Dabholkar D, Brown D, Levy N, Zeiger K. Tay-Sachs disease--carrier screening,


prenatal diagnosis and the molecular era. An international perspective, 1970 to 1993. The International
TSD Data Collection Network. JAMA. 1993;270:2307-2315. Abstract available at:
http://www.ncbi.nlm.nih.gov/pubmed/8230592

Copyright 2012 American Academy of Pediatrics


2012 PREP SA ON CD-ROM

Critique 59

(Courtesy of A Johnson)
Pedigree, as described for the family in the vignette.

Copyright 2011 American Academy of Pediatrics


2012 PREP SA ON CD-ROM

Question 60
A 17-year-old sexually active girl presents for a follow-up evaluation after her third episode of a
urinary tract infection. She is currently asymptomatic. The results of renal ultrasonography and voiding
cystourethrography are negative. She asks you how to prevent further episodes.
Of the following, you are MOST likely to advise her to
A. drink cranberry juice frequently
B. increase her daily water intake
C. make sure to void after intercourse
D. self-medicate with antibiotics for 3 days when symptomatic
E. use single-dose postcoital antibiotic prophylaxis

Copyright 2012 American Academy of Pediatrics


2012 PREP SA ON CD-ROM

Critique 60 Preferred Response: E


Acute cystitis, or an inflamed bladder mucosa, is much more common in females, who are also
more likely to have recurrences. The occurrence of three infections with positive cultures in the preceding
year, as described for the girl in the vignette, meets the criteria for recurrent lower urinary tract infection
(UTI) or cystitis.
Currently, the best evidence supports the use of antibiotics as a preventive measure. Those who
associate their symptoms with sexual intercourse should be offered postcoital antibiotic prophylaxis
(taken within 2 hours of intercourse). This involves a single dose of an effective antimicrobial (eg,
nitrofurantoin 50 mg, trimethoprim-sulfamethoxazole [TMP-SMX] 40/200 mg, or cephalexin 500 mg).
Continuous antimicrobial prophylaxis may be required in women who fail a postcoital regimen, do not
associate frequent UTIs with a modifiable cause, or are at risk for recurrent complicated UTIs. Regimens
include trimethoprim (100 mg daily at bedtime or 3 times per week), TMP-SMX (40/200 mg daily at
bedtime or 3 times per week), nitrofurantoin (50 to 100 mg daily at bedtime or 3 times per week),
norfloxacin (200 mg daily at bedtime or 3 times per week), and cephalexin (250 mg daily). These
regimens have been shown to be safe and effective, even after 5 years of use. Because one third of
women experience a prolonged UTI-free period, a trial without the medication can be attempted after 6 to
12 months of continuous therapy, with prophylaxis reinstituted if the patient develops recurrent UTIs. Self-
medication when symptomatic with a 3-day regimen is another option, although the evidence for this
therapy is less clear than for the previous ones. Studies show that women who have a past history of at
least two UTIs are capable of self-diagnosis and treatment. This regimen is best for those who have less
frequent episodes (ie, one to two per year).
An important risk factor for recurrent UTIs or cystitis is sexual activity, with an odds ratio of 12.4
(95% confidence interval, 3 - 59) for symptoms developing after intercourse. Other coital behaviors that
may be risk factors include coital frequency more than five times a week and use of a diaphragm and
spermicides. Consideration should be given to changing the method of contraception in such patients.
The infections are often closely spaced and usually have no anatomic or functional abnormalities to
account for the recurrences (eg, calculus, abscess, cystic disease). Therefore, if the patient has had no
history of infections before the onset of sexual activity, radiologic evaluation with ultrasonography and
voiding cystourethrography is not indicated. Obtaining cultures with each episode is important to diagnose
cystitis. Prompt resolution of symptoms with antibiotics differentiates it from other conditions that mimic
cystitis.
Behavior modifications that are often recommended but have not been well studied include use of
a proper wiping technique from front to back, avoidance of tight pants, urge-initiated voiding, postcoital
voiding, increased fluid intake, and the daily consumption of cranberry juice. Although evidence supports
the use of cranberry juice and other products as beneficial, there are no clear directions for the amount or
concentration of juice to be consumed or the duration of a regimen.

American Board of Pediatrics Content Specification(s):


Plan the management of recurrent cystitis

Suggested Reading:
Azzarone G, Liewehr S, O'Connor K, Adam HM. In brief: cystitis. Pediatr Rev. 2007;28:474-476. DOI:
10.1542/pir.28-12-474. Available at: http://pedsinreview.aappublications.org/cgi/content/full/28/12/474

Craig JC, Simpson JM, Williams GJ, et al; Prevention of Recurrent Urinary Tract Infection in Children with
Vesicoureteric Reflux and Normal Renal Tracts (PRIVENT) Investigators. Antibiotic prophylaxis and
recurrent urinary tract infection in children. N Engl J Med. 2009;361:1748-1759. Available at:
http://www.nejm.org/doi/full/10.1056/NEJMoa0902295#t=articleBackground

DAngelo LJ, Neinstein LS. Genitourinary tract disorders. In: Neinstein LS, Gordon CM, Katzman DK,
Rosen DS, Woods ER, eds. Adolescent Health Care: A Practical Guide. 5th ed. Philadelphia, PA:
Lippincott Williams & Wilkins, a Wolters Kluwer business; 2008:379-385

Copyright 2012 American Academy of Pediatrics


2012 PREP SA ON CD-ROM

Rudaitis S, Pundziene B, Jievaltas M, Uktveris R, Kevelaitis E. Recurrent urinary tract infection in girls: do
urodynamic, behavioral and functional abnormalities play a role? J Nephrol. 2009;22:766-773. Abstract
available at: http://www.ncbi.nlm.nih.gov/pubmed/19967656

Sen A. Recurrent cystitis in non-pregnant women. Clin Evid (Online). 2008;2008:0801. Available at:
http://www.ncbi.nlm.nih.gov/pmc/articles/PMC2907990/?tool=pubmed

Copyright 2012 American Academy of Pediatrics


2012 PREP SA ON CD-ROM

Question 61
During morning teaching rounds, while discussing oxygenation and ventilation, a medical student
asks you if capillary blood gas measurement is an acceptable substitute for arterial blood gas
measurement. You inform her that with one exception, properly performed capillary blood gas testing has
been shown to correlate well with results of arterial blood gas testing in several recent studies.
Of the following, the correlation between capillary and arterial blood gas readings is MOST likely
to be decreased in patients who have
A. bradycardia
B. hyperthermia
C. hypotension
D. hypothermia
E. tachypnea

Copyright 2012 American Academy of Pediatrics


2012 PREP SA ON CD-ROM

Critique 61 Preferred Response: C


Arterial blood gas measurements (ABGs) are frequently used to assess the status of
oxygenation, ventilation, and overall acid-base balance in severely ill patients. Although noninvasive
assessments (eg, pulse oximetry, transcutaneous monitoring, and end-tidal CO2) are extremely useful in
monitoring oxygenation and ventilation, they do not provide information on the acid-base status. Serial
ABGs or placement of arterial catheters can be technically difficult to perform in small patients and are
associated with a variety of complications, including ischemia, infection, thrombosis, and development of
vascular aneurysms. For these reasons, capillary blood gas measurements (CBGs) have been used as a
substitute for ABGs for nearly 40 years, although their correlation with the gold-standard of ABG
measurements is often debated.
Several studies performed in pediatric intensive care units have demonstrated a good correlation
between ABGs and CBGs with respect to pH, PCO2, bicarbonate, and base excess in a wide variety of
circumstances, including hypothermia, hyperthermia, and prolonged capillary refill time. However, the
correlation between ABGs and CBGs is significantly diminished in the presence of hypotension. ABGs
also should be used in patients in whom close monitoring of actual PaO2 is needed. Bradycardia should
not affect the results of the CBG unless there is accompanying hypotension. Tachypnea would affect the
PCO2 but would not affect the correlation between CBG and ABG.

American Board of Pediatrics Content Specification(s):


Recognize the limitations of capillary blood gas testing

Suggested Reading:
Escalante-Kanashiro R, Tantalen-Da-Fieno J. Capillary blood gases in a pediatric intensive care unit.
Crit Care Med. 2000;28:224-226. Abstract available at: http://www.ncbi.nlm.nih.gov/pubmed/10667527

Pope J, McBride J. Consultation with the specialist: respiratory failure in children. Pediatr Rev.
2004;25:160-167. DOI: 10.1542/10.1542/pir.25-5-160. Available at:
http://pedsinreview.aappublications.org/cgi/content/full/25/5/160

Yildizda D, Yapiciolu H, Yilmaz H, Sertdemir Y. Correlation of simultaneously obtained capillary,


venous, and arterial blood gases of patients in a paediatric intensive care unit. Arch Dis Child.
2004;89:176-180. DOI: 10.1136/adc.2002.016261. Available at:
http://www.ncbi.nlm.nih.gov/pmc/articles/PMC1719810/pdf/v089p00176.pdf

Copyright 2012 American Academy of Pediatrics


2012 PREP SA ON CD-ROM

Question 62
During a 14-year-old girls annual health supervision visit, you learn that she has not yet had
menarche. During most of the visit, the girls mother speaks for her and states that her daughter is a
straight A student and has many friends. The mother is not worried about the delay in her daughters
menarche but does raise concerns that she does not exercise on a regular basis. Mid-parental height is at
the 50th percentile. Physical examination of the well but thin-appearing girl reveals a weight of 35 kg (3rd
percentile); height of 160 cm (50th percentile); and normal findings on eye, ear, nose, heart, lung,
abdomen, and extremities evaluation. She has a Sexual Maturity Rating of 3 for pubic hair and 2 for
breast development.
Of the following, the MOST appropriate next step in the evaluation of this patient is to
A. measure luteinizing and follicle-stimulating hormones
B. obtain a bone age radiograph
C. obtain a diet history
D. perform a karyotype
E. schedule a follow-up appointment in 2 months

Copyright 2012 American Academy of Pediatrics


2012 PREP SA ON CD-ROM

Critique 62 Preferred Response: C


The average age of menarche in is 12 years. Although many girls begin menses sooner (and
some later), breast development is the first sign of pubertal progression in girls and should be noted by 14
years of age. The 14-year-old girl described in the vignette does have evidence of breast development
(albeit somewhat delayed for her chronologic age). More importantly, she has a very low weight for
height, with a body mass index of 13.67. A common cause of delayed puberty is inadequate weight for
height, which could be caused by eating disorders. Although this girl does not apparently exercise
excessively and, therefore, is not likely to have the athletic triad (disordered eating, osteoporosis, and
amenorrhea), she may have an isolated eating disorder. High-achieving (straight As) girls who have
controlling parents may be at high risk for developing eating disorders such as anorexia and bulimia.
A careful diet history is indicated for this girl before any additional studies are warranted. In any
situation involving low weight for height, luteinizing hormone and follicle-stimulating hormone values are
low, and, therefore, their measurement is not helpful diagnostically. With a Sexual Maturity Rating of only
2 for breast development, this girls bone age should be considerably delayed. Although a delayed bone
age may be informative, the girls diet is a more pressing issue. A karyotype to screen for Turner
syndrome is an important part of the evaluation of the girl who has primary amenorrhea and short stature,
but this diagnosis is unlikely for this girl, who has some pubertal progression and a height consistent with
her mid-parental target. Finally, the pressing concerns about her weight for height require more urgent
evaluation than a follow-up appointment in 2 months.

American Board of Pediatrics Content Specification(s):


Recognize the signs and symptoms of delayed puberty
Know the causes of delayed puberty

Suggested Reading:
Emmans SJ. Delayed puberty. In: Emans SJ, Laufer MR, Goldstein DP, eds. Pediatric and Adolescent
Gynecology. 5th ed. Philadelphia, PA: Lippincott Williams & Wilkins, a Wolters Kluwer business;
2005:181-213

Prabhakaran R, Misra M, Miller KK, et al. Determinants of height in adolescent girls with anorexia
nervosa. Pediatrics. 2008;121:e1517-e1523. DOI: 10.1542/peds.2007-2820. Available at:
http://pediatrics.aappublications.org/cgi/content/full/121/6/e1517

Roz C, Doyen C, Le Heuzey MF, Armoogum P, Mouren MC, Lger J. Predictors of late menarche and
adult height in children with anorexia nervosa. Clin Endocrinol. 2007;67:462467.DOI: 10.1111/j.1365-
2265.2007.02912.x. Abstract available at: http://www.ncbi.nlm.nih.gov/pubmed/17561975

Copyright 2012 American Academy of Pediatrics


2012 PREP SA ON CD-ROM

Question 63
A child walks into the examination room, accompanied by his parents. He points to a box of
crayons that he notices on a nearby shelf. You offer him a paper and the crayons. You then draw a
vertical line and a cross, which he successfully copies. You ask him to draw a person. He draws two
circles, one for the head and one for the body. He then stops drawing and says, Look at me while he
balances on his foot for 3 seconds.
Of the following, these developmental milestones are MOST typical for a child whose age is
A. 24 months
B. 30 months
C. 36 months
D. 48 months
E. 60 months

Copyright 2012 American Academy of Pediatrics


2012 PREP SA ON CD-ROM

Critique 63 Preferred Response: D


The child described in the vignette exhibits the normal developmental milestones for 4 years of
age. He can copy a cross, draw a simple figure of a person (head plus one other body part), and hop on
one foot two to three times. If asked, it is likely that he also could copy a square. A child of 24 months is
able to walk down stars holding a rail, throw a ball overhand, and imitate drawing a horizontal line. A 30-
month-old child can alternate feet while walking up stairs with a railing, jump in place, and make an eight-
cube tower. A 36 month old can balance on one foot for 3 seconds, pedal a tricycle, and copy a circle. A
60 month old can walk down stairs holding a railing while alternating feet, hop on one foot 15 times, skip,
copy a triangle, and write his or her name.
Motor developmental milestones are obtained via parental history and from observation during
neurodevelopmental examination. To ascertain if a child has delays in motor milestones, the clinician
must consider the history, standard neurodevelopmental milestones, and the presence or absence of
primitive reflexes and postural responses. A motor quotient (MQ) can be determined by the following:
MQ=motor age/chronological age x 100. An MQ of greater than 70 is normal, between 50 and 70 is
suspicious and requires monitoring, and less than 50 is abnormal and requires referral to a subspecialist.

American Board of Pediatrics Content Specification(s):


Recognize the normal motor developmental milestones for 4 years of age (eg, copies cross; draws
simple figure of a person (head plus one other body part); balances on one foot for 3 seconds)

Suggested Reading:
Gerber RJ, Wilks T, Erdie-Lalena C. Developmental milestones: motor development. Pediatr Rev.
2010;31:267-277. DOI: 10.1542/pir.31-7-267. Available at:
http://pedsinreview.aappublications.org/cgi/content/full/31/7/267

Lingam R, Missiuna C, Forsyth K, Emond L. Developmental coordination disorder: a physicians guide to


assessment and diagnosis. AAP Section on Developmental and Behavioral Pediatrics Newsletter.
2009;Fall:4-6. Available at:
http://www.aap.org/sections/dbpeds/pdf/PDFNewContent/ScreeningAssess/Developmental%20Coordinat
ion%20Disorder.pdf

Copyright 2012 American Academy of Pediatrics


2012 PREP SA ON CD-ROM

Question 64
You diagnose measles in a 3-year-old child who recently returned from a trip to Switzerland. The
diagnosis is confirmed by immunoglobulin M serology. The girls mother states that a 7-month-old cousin
was visiting with her daughter the day before the rash developed. She asks if the other child needs to
receive treatment.
Of the following, the BEST measure to protect the exposed child is
A. individual component measles vaccine
B. intramuscular immune globulin
C. intravenous acyclovir
D. measles, mumps, rubella vaccine
E. oral acyclovir

Copyright 2012 American Academy of Pediatrics


2012 PREP SA ON CD-ROM

Critique 64 Preferred Response: B


Measles transmission occurs through respiratory droplets. The virus is infectious from 3 to 5 days
before the onset of the morbilliform rash until 4 days after the development of the exanthem. Prevention
or modification of measles infection in susceptible unimmunized exposed children is accomplished by
administration of intramuscular immune globulin (0.25 mL/kg to a maximum of 15 mL) within 6 days of
exposure. Accordingly, the 7-month-old contact described in the vignette is a candidate for immune
globulin administration. Immune globulin prophylaxis is also indicated for immunocompromised
individuals regardless of vaccine status, but the recommended dose is higher (0.5 mL/kg) to a maximum
of 15 mL.
Immunization after exposure has not demonstrated efficacy in preventing measles in a child this
young. However, measles vaccine administered within 72 hours of exposure may provide protection in
some cases and is recommended for use in control of outbreaks in schools and child-care centers. Pre-
exposure administration of measles vaccine for a 6- to 12-month-old child traveling to a high-risk area is
also advisable, although such a child still requires two doses of measles, mumps, and rubella (MMR)
vaccine after 12 months of age for full immunization. Individual component measles vaccine is no longer
manufactured, so in settings where vaccine is indicated, MMR is the only available option.
Acyclovir does not have clinical activity against measles virus. Ribavirin has in vitro activity
against measles virus and has been tried in severe cases and immunocompromised patients, although no
controlled trials have been undertaken.
Although measles is no longer endemic in the United States, cases continue to occur as the
result of international importation. Most of these cases occur in people who were unimmunized or of
unknown immunization status. Once a case is suspected or confirmed, that individual should be placed
under airborne transmission precautions as well as standard isolation for 4 days after the onset of the
rash. Immunocompromised individuals who have measles are isolated for the entire duration of their
illness.

American Board of Pediatrics Content Specification(s):


Know that intramuscular immune globulin should be given to immunocompromised patients and
infants who are closely exposed to measles but have not been immunized
Know the route and ease of transmission of measles
Know the control measures for measles: isolation (airborne precautions), care of exposed personnel,
immunization

Suggested Reading:
American Academy of Pediatrics. Measles. In: Pickering LK, Baker CJ, Kimberlin DW, Long SS, eds. Red
Book: 2009 Report of the Committee on Infectious Diseases. 28th ed. Elk Grove Village, IL: American
Academy of Pediatrics; 2009:444-455

Centers for Disease Control and Prevention. Update: measles United States. January July 2008.
MMWR Morb Mortal Wkly Rep. 2008;57:893896. Available at:
http://www.cdc.gov/mmwr/preview/mmwrhtml/mm5733a1.htm

Parker AA, Staggs W, Dayan GH, et al. Implications of a 2005 measles outbreak in Indiana for sustained
elimination of measles in the United States. N Engl J Med. 2006;355:44 455. Available at:
http://www.nejm.org/doi/full/10.1056/NEJMoa060775#t=article

Copyright 2012 American Academy of Pediatrics


2012 PREP SA ON CD-ROM

Question 65
You are seeing a 12-year-old boy for follow-up of problematic handwashing and bathing routines.
He has been receiving cognitive behavior therapy for 12 weeks and has had minimal improvement. He
has always been somewhat of a perfectionist and rigid, but recently has begun to have persistent fears of
germs. The routines are time-consuming and embarrassing, sometimes causing him to be late for school.
He is otherwise healthy, although he is beginning to have significant skin irritation on his hands.
Of the following, the BEST next course of action is to initiate
A. alprazolam treatment
B. clomipramine treatment
C. propranolol treatment
D. risperidone treatment
E. sertraline treatment

Copyright 2012 American Academy of Pediatrics


2012 PREP SA ON CD-ROM

Critique 65 Preferred Response: E


The boy described in the vignette has evidence of obsessive-compulsive disorder (OCD).
Although he has not shown the expected response, cognitive behavior therapy (CBT) is considered a
first-line treatment. Use of a selective serotonin reuptake inhibitor (SSRI), such as fluoxetine,
fluvoxamine, or sertraline, along with CBT has been associated with a superior response to either
modality alone. He should continue CBT rather than searching for other therapeutic modalities.
Clomipramine (a tricyclic antidepressant) has good efficacy for OCD but is usually not a first-line
treatment because of less tolerance and a greater need for medical monitoring. There is no evidence that
benzodiazepines such as alprazolam are effective in the treatment of pediatric OCD. In addition,
significant risks are associated with the use of benzodiazepines, including disinhibition and agitation.
Neuroleptics, such as risperidone, may be used as adjuncts to SSRIs for those who are refractory to
attempts at treatment, but they also have worrisome adverse effects (metabolic and extrapyramidal) that
require clinical and laboratory monitoring. Propranolol is sometimes used as a treatment for performance
anxiety, but it is not a recommended treatment for OCD.

AAP Mental Health Competency:


Know how to treat obsessive-compulsive disorder (OCD) in children

Suggested Reading:
American Academy of Pediatrics. Evidence-based child and adolescent psychosocial interventions.
Available at:
http://www.aap.org/commpeds/dochs/mentalhealth/docs/CR%20Psychosocial%20Interventions.F.0503.p
df

King RA, Leonard H, March J and the Work Group on Quality Issues. Practice parameters for the
assessment and treatment of children and adolescents with obsessive-compulsive disorder. J Am Acad
Child Adolesc Psychiatry. 1998;37(suppl):27S-45S. Abstract available at:
http://www.ncbi.nlm.nih.gov/pubmed/9785727

Rosenberg DR, Vandana P, Chiriboga JA. Anxiety disorders. In: Kliegman RM, Stanton BF, St. Geme JW
III, Schor NF, and Behrman RE, eds. Nelson Textbook of Pediatrics. 19th ed. Philadelphia, PA: Saunders
Elsevier; 2011:77-82

Copyright 2012 American Academy of Pediatrics


2012 PREP SA ON CD-ROM

Question 66
A 5-year-old boy who is otherwise well presents with an area of hair loss; there is some scale
and several pustules (Item Q66). The area does not fluoresce on exposure to a Woods lamp. The
remainder of his physical examination findings are normal.
Of the following, the MOST appropriate therapy is
A. oral clindamycin
B. oral griseofulvin
C. oral ivermectin
D. topical ketoconazole
E. topical selenium sulfide

Copyright 2012 American Academy of Pediatrics


2012 PREP SA ON CD-ROM

Question 66

(Courtesy of D Krowchuk)

Copyright 2012 American Academy of Pediatrics


2012 PREP SA ON CD-ROM

Critique 66 Preferred Response: B


The boy described in the vignette has tinea capitis (ringworm of the scalp) caused by
dermatophytic fungi. Trichophyton tonsurans is the most common species causing tinea capitis in the
United States and does not fluoresce under Woods lamp. Griseofulvin (20 to 25 mg/kg per day orally) is
the standard first-line therapy for the treatment of tinea capitis in children older than 2 years of age
because of its favorable safety profile. Both tablet and suspension formulations of the drug are available.
The medication should be taken with food, especially high-fat meals, to increase drug concentrations.
No laboratory assessment of hepatic enzymes is required during the 8-week course of therapy in
children who have no history or clinical examination findings that are concerning for liver disease. For
children who have severe or refractory infection in whom longer courses of therapy are required, serum
hepatic enzyme values should be measured every 8 weeks because prolonged therapy has been
associated with an increased risk of hepatotoxicity. In general, most adverse effects of griseofulvin
therapy are mild and consist of gastrointestinal upset or rash. Hypersensitivity cross-reactions between
penicillin and griseofulvin can occur in those who have penicillin allergy. Griseofulvin can decrease
concentrations of warfarin, cyclosporine, and oral contraceptives, and phenobarbital can decrease the
antifungal effect of griseofulvin. Concomitant use of griseofulvin and alcohol can result in a disulfiram-like
reaction (tachycardia, flushing, headache, nausea).
Oral itraconazole can be used to treat tinea capitis in adults but is not approved for use in
children. Oral fluconazole is an option for treatment in all age groups, but efficacy varies. A 6-week
course of oral terbinafine is as effective as griseofulvin for the treatment of tinea capitis in children age 4
years and older, but baseline assessment of serum hepatic enzymes is recommended.
Topical antifungal therapy, such as topical ketoconazole, is ineffective in the treatment of tinea
capitis because topical agents do not penetrate sufficiently into the hair shaft. Topical selenium sulfide
used twice weekly may decrease fungal shedding and help prevent spread of infection, but it is not
curative. Oral ivermectin is used to treat strongyloidiasis and onchoceriasis. Oral clindamycin is useful for
the treatment of bacterial infections of the skin caused by susceptible organisms such as Streptococcus
pyogenes and Staphylococcus aureus.

American Board of Pediatrics Content Specification(s):


Know the appropriate use of griseofulvin
Recognize the adverse effects of griseofulvin therapy and when monitoring of liver function is
indicated

Suggested Reading:
American Academy of Pediatrics. Tinea capitis (ringworm of the scalp). In: Pickering LK, Baker CJ,
Kimberlin DW, Long SS, eds. Red Book: 2009 Report of the Committee on Infectious Diseases. 28th ed.
Elk Grove Village, IL: American Academy of Pediatrics; 2009:661-662

Shy R. Tinea corporis and tinea capitis. Pediatr Rev. 2007;28:164-174. DOI: 10.1542/pir.28-5-164.
Available at: http://pedsinreview.aappublications.org/cgi/content/full/28/5/164

Steinbach WJ, Dvorak CC. Antifungal agents. In: Long SS, Pickering LK, Prober CG, eds. Principles and
Practice of Pediatric Infectious Diseases. 3rd ed. Philadelphia, PA: Churchill Livingstone Elsevier;
2008:1453-1460

Copyright 2012 American Academy of Pediatrics


2012 PREP SA ON CD-ROM

Question 67
A 3-month-old male infant presents with poor feeding and occasional vomiting. He has no history
of fever, cough, irritability, constipation, or diarrhea. He has approximately 12 wet diapers per day. On
physical examination, the afebrile infant has a heart rate of 130 beats/min, respiratory rate of 28
breaths/min, and blood pressure of 94/50 mm Hg. He has tacky mucous membranes and a capillary refill
time of 2 seconds; all other findings are normal. Laboratory evaluation reveals:
Sodium, 160 mEq/L (160 mmol/L)
Potassium, 3.7 mEq/L (3.7 mmol/L)
Chloride, 122 mEq/L (122 mmol/L)
Bicarbonate, 16 mEq/L (16 mmol/L)
Glucose,100 mg/dL (5.6 mmol/L)
Osmolality, 345 mOsm/kg
Blood urea nitrogen, 34 mg/dL (12.1 mmol/L)
Creatinine, 0.6 mg/dL (53.0 mcmol/L)
Of the following, the MOST likely urine specific gravity and urine osmolality for this patient are

Urine Specific Gravity Urine Osmolality


A. 1.001 80 mOsm/kg
B. 1.010 300 mOsm/kg
C. 1.015 400 mOsm/kg
D. 1.020 600 mOsm/kg
E. 1.030 1,200 mOsm/kg

Copyright 2012 American Academy of Pediatrics


2012 PREP SA ON CD-ROM

Critique 67 Preferred Response: A


The infant described in the vignette exhibits evidence of hypernatremia (sodium of 160 mEq/L
[160 mmol/L]), a mildly increased anion gap acidosis, and azotemia (elevated blood urea nitrogen and
creatinine). The presence of azotemia and mild acidosis in conjunction with hypernatremia raises the
question of hypernatremic dehydration.
Hypernatremia is defined as a serum sodium concentration greater than 150 mEq/L (150
mmol/L). It is caused most commonly by a net water deficit but in rare instances can be due to a net
sodium gain. Water loss can occur as a consequence of gastrointestinal or renal losses of hypotonic fluid.
Osmotic diarrhea, as may develop with lactulose ingestion or malabsorption, also can result in
hypernatremia. Renal losses of hypotonic urine (water and electrolytes) can occur with diuretics (loop or
osmotic) and renal dysplasia (especially obstructive uropathy); renal losses of water in isolation occur
classically with diabetes insipidus (DI). There are two forms of DI: central DI due to insufficient antidiuretic
hormone (ADH) release and nephrogenic DI due to an inadequate renal response to ADH. Both forms of
DI have a number of underlying causes (Item C67).
Differentiating the causes of hypernatremia begins with a history and physical examination, which
should provide an indication of the most likely cause. Laboratory evaluation should focus on the
concentrating ability of the kidneys by assessing the urine osmolality compared with plasma osmolality. In
the setting of volume depletion and hypernatremia, the kidneys should maximally concentrate the urine to
retain water, thereby correcting both the hypernatremia and volume depletion. Accordingly, a high urine
osmolality and high urine specific gravity would be expected if normal ADH secretion and normal ADH
responsiveness are present.
For the infant described in the vignette, the normal or expected physiologic response to the
hypernatremia and hyperosmolar state is high ADH secretion with maximally concentrated urine.
However, his clinical state of severe hypernatremia in combination with 12 wet diapers per day suggests
a problem with physiologic compensation. This raises the possibility of a defect in concentrating his urine,
with resultant hypernatremic dehydration, such as seen with X-linked nephrogenic DI. Accordingly, the
most likely urine specific gravity would be 1.001 and urine osmolality would be 80 mOsm/kg. A specific
gravity of 1.010 and urine osmolality of 300 mOsm/kg would be indicative of isosthenuria. which can be
seen in sickle hemoglobinopathies (disease or trait). A patient who has a specific gravity of 1.015 and a
urine osmolality of 400 mOsm/kg demonstrates some urinary concentrating ability but less than that
expected in this clinical setting and suggests partial DI. A patient who has hypernatremic dehydration
from extrarenal losses of hypotonic fluids would be expected to have a high urine specific gravity (>1.020)
and high urine osmolality (>600 mOsm/kg).

American Board of Pediatrics Content Specification(s):


Understand how to differentiate (nephrogenic) diabetes insipidus from hypernatremic dehydration (ie,
urine specific gravity, urine and serum osmolalities)
Recognize the clinical manifestations and laboratory findings associated with (nephrogenic) diabetes
insipidus

Suggested Reading:
Quan A, Quigley R, Satlin LM, Baum M. Water and electrolyte handling by the kidney. In: Kher KK,
Schnaper HW, Makker SP, eds. Clinical Pediatric Nephrology. 2nd ed. London, England: Informa Healthcare;
2007:15-35

Rose BD, Post TW. Hyperosmolal states-hypernatremia. In: Clinical Physiology of Acid-base and
Electrolyte Disorders. 5th ed. New York, NY: McGraw-Hill Medical Publishing Division; 2001:746-793

Saborio P, Tipton GA, Chan JCM. Diabetes insipidus. Pediatr Rev. 2000;21;122-129. DOI:
10.1542/pir.21-4-122. Available at: http://pedsinreview.aappublications.org/cgi/content/full/21/4/122

Watkins S, Okamura D, Rodriguez Soriano J. Hypernatremia. In: Zelikovic I, Eisenstein I, eds. Practical
Algorithms in Pediatric Nephrology. Basel, Switzerland: Karger; 2008:74-75

Copyright 2012 American Academy of Pediatrics


2012 PREP SA ON CD-ROM

Critique 67

Item C67. Causes of Diabetes Insipidus

Central Diabetes Insipidus


! Head trauma
! Brain tumors (craniopharyngioma and pinealoma)
! Histiocytosis X
! Encephalitis or meningitis
Nephrogenic Diabetes Insipidus
! X-linked nephrogenic diabetes insipidus (mutation of antidiuretic hormone receptor-V2R)
! Autosomal recessive diabetes insipidus (mutation of aquaporin 2 water channel)
! Obstructive uropathy
! Hypercalcemia
! Hypokalemia
! Sickle cell disease

Copyright 2011 American Academy of Pediatrics


2012 PREP SA ON CD-ROM

Question 68
A 12-month-old boy presents with a 7-month history of a worsening skin rash. The rash is pruritic
and involves his neck, anterior and posterior trunk, antecubital and popliteal fossae, and hands and feet.
Use of a moisturizer and topical corticosteroid has resulted in some improvement. The remainder of his
past medical history is unremarkable. On physical examination, you observe multiple erythematous,
lichenified patches and diagnose severe atopic dermatitis.
Of the following, the MOST helpful next step is to
A. eliminate milk, eggs, soy, and wheat from the diet
B. measure serum immunoglobulins (IgG, IgA, and IgM)
C. perform aeroallergen allergy testing
D. perform food allergy testing
E. start oral corticosteroids

Copyright 2012 American Academy of Pediatrics


2012 PREP SA ON CD-ROM

Critique 68 Preferred Response: D


Approximately 30% to 40% of infants who have moderate-to-severe atopic dermatitis (AD), such
as described for the infant in the vignette, may have an underlying immunoglobulin (Ig)E-mediated food
allergy exacerbating the AD and should undergo food allergy testing. In some infants, food ingestion may
result in immediate worsening of AD severity, although most infants do not demonstrate this immediate
reaction. Parents may be concerned about multiple causative foods, but five (ie, milk, eggs, soy, wheat,
and peanut) account for 90% of the causative allergens in the United States.
Both allergy skin testing and serum IgE measurements to these foods can be helpful in identifying
and eliminating likely triggers although the positive predictive value for a skin or blood test may only be as
high as 50%. The negative predictive value of food allergy testing also is helpful. For example, a negative
blood IgE result (<0.35 kU/L, Pharmacia CAP system) or negative allergy skin test result provides a high
negative predictive value.
The most common implicated foods can be eliminated from the diet, but this will not improve
symptoms in most (60% to 70%) children who do not have IgE-mediated AD. Because the unnecessary
elimination of multiple foods affects a childs nutrition, food avoidance should be guided by the dietary
history, eczema severity, and results of skin or blood testing.
Parents often request to have their children who have eczema tested for allergies. House dust
mite has been implicated in studies, although it is less likely a cause for moderate-to-severe AD than food
allergies. Climate changes such as cold, dry air or hot, humid weather can worsen AD, but specific
aeroallergens such as mold, trees, and weeds are not associated with eczema in infants, and testing for
aeroallergens within the evaluation for moderate-to-severe AD is not recommended.
Measuring serum immunoglobulins (eg, IgA, IgM, and IgG) should be considered for infants and
children who experience recurrent infections, but it is not recommended for patients who present with only
eczema. Wiskott-Aldrich syndrome can present with severe eczema, thrombocytopenia, and low IgM
concentrations, and this diagnosis should be considered in infants who have severe eczema, bleeding,
and recurrent infections.
The treatment of eczema involves identification of possible triggers, family education, and a
consistent regimen of topical moisturizers and corticosteroids. Topical corticosteroids, not oral
corticosteroids, are the cornerstone for reducing inflammation and improving eczema. Oral corticosteroids
can be used for severe flares, although they generally are not indicated because of their significant
systemic adverse effects and potential for rebound flaring after discontinuation.

American Board of Pediatrics Content Specification(s):


Know that some patients with moderate or severe eczema have positive skin tests to food, may or
may not have acute symptoms on ingesting these foods, and experience improvement in their
eczema after eliminating these foods

Suggested Reading:
Cartwright RC, Dolen WK. Consultation with the specialist: who needs allergy testing and how to get it
done. Pediatr Rev. 2006;27:140-146. DOI: 10.1542/pir.27-4-140. Available at:
http://pedsinreview.aappublications.org/cgi/content/full/27/4/140

Epps RE. Atopic dermatitis and ichthyosis. Pediatr Rev. 2010;31:278-286. DOI: 10.1542/pir.31-7-278.
Available at: http://pedsinreview.aappublications.org/cgi/content/full/31/7/278

Copyright 2012 American Academy of Pediatrics


2012 PREP SA ON CD-ROM

Question 69
A 13-year-old girl comes to the office with a 4-day history of a brownish, foul-smelling vaginal
discharge. She reports that the discharge has been associated with perineal itching and burning with
urination. She has had no fever, abdominal pain, or other constitutional symptoms. She denies sexual
activity and reports that her last menstrual period was 1 week ago. Her physical examination findings are
normal, except for mild labial erythema and the presence of a scant, watery, malodorous discharge at the
vaginal opening.
Of the following, the MOST appropriate next step is to
A. obtain bacterial cultures of the discharge
B. obtain urine for nucleic acid amplification tests for Chlamydia trachomatis and Neisseria
gonorrhoeae
C. perform a pelvic examination
D. prescribe clotrimazole cream
E. recommend sitz baths

Copyright 2012 American Academy of Pediatrics


2012 PREP SA ON CD-ROM

Critique 69 Preferred Response: C


The presence of a malodorous vaginal discharge and perineal pain described for the girl in the
vignette suggests the presence of a vaginal infection or foreign body. Differentiating between the two
requires an appropriate physical examination that includes perineal inspection and, in some cases, a
speculum examination. Although cultures and other diagnostic tests may be obtained, treatment should
be based on the physical examination findings. It is likely that the girl has a retained vaginal foreign body
(eg, tampon), and appropriate treatment is removal rather than sitz baths or clotrimazole cream.
Foreign body insertion into the genitourinary tract of boys or girls is a well-recognized cause of
genital pain, foul-smelling urethral/vaginal discharge, dysuria, or hematuria. Most often the result of
curiosity, in some cases, insertion is related to psychological disorders, autoerotic behavior, or a history of
sexual abuse. This diagnosis should be considered in any child or adolescent who has persistent genital
symptoms despite seemingly appropriate or repeated treatment of the symptoms. The patient often does
not disclose the history of insertion, and delayed medical presentation is common, so the practitioner
must have a high index of suspicion for this condition. Depending on the location and size of the foreign
body, the physical examination may not be revealing. If clinical suspicion for a vaginal foreign body is
high, examining the patient in knee-chest position or performing digital rectal examination may be helpful.
In some cases, sedation may be necessary. An internal vaginal examination is indicated for an
adolescent girl. In boys, a foreign body below the external urinary sphincter is usually palpable, but in
both boys and girls, if the condition is considered and the physical examination is unrevealing, other
diagnostic testing, including ultrasonography, computed tomography scan, or cystography, may be
necessary. Removal typically cures the patients symptoms and may require the involvement of a
urologist or gynecologist. In addition, psychological evaluation or sexual abuse evaluation should be
considered, depending on the circumstances.

American Board of Pediatrics Content Specification(s):


Consider instrumentation of the penis or insertion of a foreign body into the vagina as a cause of
genital pain and malodorous discharge

Suggested Reading:
Davis AJ, Katz VL. Pediatric and adolescent gynecology: gynecologic examination, infections, trauma,
pelvic mass, precocious puberty In: Katz VL, Lentz GM, Lobo RA, Gershenson DM, eds. Comprehensive
Gynecology. 5th ed. Philadelphia PA: Mosby Elsevier; 2007:257-274

Jamieson MA. A photo album of pediatric and adolescent gynecology. Obstet Gynecol Clin North Am.
2009;36:1-24. DOI: 10.1016/j.ogc.2009.01.004. Abstract available at:
http://www.ncbi.nlm.nih.gov/pubmed/19344845

Rahman N, Featherstone NC, DeCaluwe D. Spider-man, magnets, and urethral-cutaneous fistula.


Urology. 2010;76:162-163. Available at: http://www.ncbi.nlm.nih.gov/pubmed/20350757

Parks LA, Merritt DF. Gynecologic problems of childhood: bleeding. In: Kliegman RM, Stanton BF, St.
Geme JW III, Schor NF, and Behrman RE, eds. Nelson Textbook of Pediatrics. 19th ed. Philadelphia, PA:
Saunders Elsevier; 2011:1869-1870

Copyright 2012 American Academy of Pediatrics


2012 PREP SA ON CD-ROM

Question 70
You are seeing a 15 month-old boy who has been placed in foster care. He was born to a 22-
year-old human immunodeficiency virus (HIV)-positive woman who had a history of intravenous drug use.
His birthweight was 2,850 g. The child experienced no neonatal complications and, at 13 months of age,
was found to be HIV-negative and hepatitis C antibody-positive. At the time of his office visit today, the
boy appears well and demonstrates no abnormal physical findings. Based upon his perinatal exposure,
you obtain the following laboratory studies:
Hemoglobin, 12.5 g/dL (125 g/L)
3 9
White blood cell count, 6.5x10 /mcL (6.5x10 /L) (40% neutrophils, 56% lymphocytes, 4% eosinophils)
Aspartate aminotransferase, 30 units/L (normal, 5 to 30 units/L)
Alanine aminotransferase, 35 units/L (normal, 10 to 30 units/L)
4
Hepatitis C RNA (PCR), 1x10 copies/mL
Of the following, the MOST appropriate next management step includes
A. follow-up in 6 months
B. interferon therapy
C. lamivudine therapy
D. liver biopsy
E. pegylated interferon and ribavirin therapy

Copyright 2012 American Academy of Pediatrics


2012 PREP SA ON CD-ROM

Critique 70 Preferred Response: A


As demonstrated by the boy described in the vignette, maternal coinfection with human
immunodeficiency virus (HIV) is a major risk factor for vertical transmission of the hepatitis C virus (HCV).
Therapy for HCV is generally not recommended for children younger than 2 years of age in the absence
of signs of hepatic decompensation, which is extremely rare during childhood. Furthermore, even in the
absence of detectable HCV RNA, an infant may demonstrate HCV antibody in the serum until 18 months
of age as a consequence of passively transferred maternal antibody. For children who have active
infection, hepatitis C tends to follow an indolent course, which makes the timing of therapy controversial
and dependent, in part, upon evidence of liver damage on biopsy as well as viral genotype. In chronic
hepatitis C infection, serum transaminase values alone are poor predictors of disease severity and (in
contrast to hepatitis B infection) should not be used to predict the clinical course or response to treatment.
A prudent approach for the asymptomatic toddler in the vignette, who has HCV viremia and minimal
elevation in liver enzymes, is to monitor both liver functions and hepatitis C profile (including quantitative
assessment of viral load) in 6 months to confirm active infection and plan further follow-up. Other
diagnostic studies or therapeutic interventions are not indicated.
HCV is an enveloped, single-stranded RNA virus, first identified in 1989. Recently, significant new
information has been gathered about the molecular biology, pathology, and treatment of HCV liver
disease. Based on sequence analysis, six major HCV genotypes are recognized, with genotype I being
the most prevalent in the United States (74%). The primary mode of acquisition during infancy and
childhood is via vertical transmission. However, unlike the high transmission rate seen for hepatitis B
virus, vertical transmission of HCV occurs in only 5% to 10% of deliveries. Factors associated with higher
6
HCV vertical transmission rates include a maternal serum viral load of greater than 10 copies/mL,
coinfection with HIV, prolonged or difficult delivery, and the use of internal fetal monitoring during delivery.
Typically, HCV RNA reaches detectable concentrations several weeks after birth, but a positive antibody
titer is not sufficient to indicate active infection in children younger than 18 months of age. Chronic
infection is defined as the persistence of HCV RNA for at least 6 months after detection.
Treatment guidelines have undergone significant modifications in recent years, owing to the
discovery of therapeutic options that offer improved rates of viral eradication. The American Association
for the Study of Liver Diseases currently recommends treatment with a combination of pegylated
interferon plus ribavirin, and recent evidence in pediatric patients demonstrates the therapeutic efficacy of
this therapy. In adult studies, a sustained virologic response (SVR) previously has been related directly to
HCV genotype, with most data indicating that a durable response is achieved in only 40% to 50% of
patients who have viral genotype I. However, one recent report describes a mean SVR greater than 80%
in these patients. Individuals exhibiting viral genotypes II and III (common in Australia and Asia)
demonstrate an SVR of approximately 80%, in response to pegylated interferon/ribavirin treatment.
Recent adult studies have also shown that the addition of the protease inhibitor telaprevir to this regimen
is effective in treating all three major genotypes and further may permit reducing the duration of therapy
from 48 to 24 weeks. This treatment also appears to offer a greater likelihood of achieving an SVR in
patients who have failed prior therapy without telaprevir.
Because HCV infection often exhibits a benign, nonprogressive course during childhood, clear
pediatric management criteria have not yet been established. However, most centers recommend
periodic monitoring of viral load and liver function profile before considering antiviral therapy. Children
who have newly diagnosed HCV infection should undergo a thorough medical evaluation. Although
uncommon, progressive liver disease, including hepatocellular carcinoma and cirrhosis necessitating liver
transplantation, has been reported during childhood. Accordingly, periodic monitoring of all children who
manifest chronic HCV infection should include ultrasonographic evaluation and measurement of serum
alpha-fetoprotein. The requirement for liver biopsy in children is controversial, particularly for patients who
have genotypes II and III, in whom earlier treatment (even in the absence of overt histologic evidence of
liver disease) should be considered because of the high likelihood of achieving an SVR. However,
particularly in patients who have HCV genotype I, which is the most likely genotype for the infant in the
vignette, a liver biopsy may be useful in guiding therapy by evaluating the degree of liver injury in the
setting of normal liver function test results.
When combination therapy with pegylated interferon and ribavirin is used, current data indicate
improved SVR rates in older patients, particularly in those who have HCV genotype II and III. Importantly,

Copyright 2012 American Academy of Pediatrics


2012 PREP SA ON CD-ROM

virologic response rates are independent of pretreatment serum transaminase values. Despite these
advances in HCV therapy, the decision to treat the asymptomatic child who has little or no evidence of
liver disease remains a difficult one.

American Board of Pediatrics Content Specification(s):


Know the long-term outcome of hepatitis C infection: chronic carriers, chronic hepatitis, cirrhosis,
hepatocellular carcinoma
Recognize that children with chronic hepatitis C infection should undergo periodic screening tests for
hepatic complications, and that treatment regiments are available

Suggested Reading:
Ghany MG, Strader DB, Thomas DL, Seeff LB; American Association for the Study of Liver Diseases.
Diagnosis, management, and treatment of hepatitis C: an update. Hepatology. 2009;49:1335-1374. DOI:
10.1002/hep.22759. Available at: http://onlinelibrary.wiley.com/doi/10.1002/hep.22759/full

Hzode C, Forestier N, Dusheiko G, et al; PROVE2 Study Team. Telaprevir and peginterferon with or
without ribavirin for chronic HCV infection. N Engl J Med. 2009;360:1839-1850. Available at:
http://www.nejm.org/doi/full/10.1056/NEJMoa0807650#t=article

Karnsakul W, Alford MK, Schwarz KB. Managing pediatric hepatitis C: current and emerging treatment
options. Ther Clin Risk Manag. 2009;5:651-660. Available at:
http://www.ncbi.nlm.nih.gov/pmc/articles/PMC2731021/?tool=pubmed

Marcellin P, forns X, Goeser T, et al. Telaprevir Is Effective Given Every 8 or 12 Hours With Ribavirin and
Peginterferon Alfa-2a or -2b to Patients With Chronic Hepatitis C. Gastroenterol 2011;140:459-468.
Available at: http://www.gastrojournal.org/article/S0016-5085(10)01584-
2/fulltexthttp://www.gastrojournal.org/article/S0016-5085(10)01584-2/fulltext - article-footnote-
1#article-footnote-1http://www.gastrojournal.org/article/S0016-5085(10)01584-2/fulltext -
article-footnote-2#article-footnote-2http://www.gastrojournal.org/article/S0016-5085(10)01584-
2/fulltext - article-footnote-3#article-footnote-3
McHutchison JG, Everson GT, Gordon SC, et al; PROVE1 Study Team. Telaprevir with peginterferonand
ribavirin for chronic HCV genotype 1 infection. N Engl J Med. 2009;360:1827-1838. Available at:
http://www.nejm.org/doi/full/10.1056/NEJMoa0806104#t=article

Mohan N, Gonzalez-Peralta RP, Fujisawa T, et al. Chronic hepatitis C virus infection in children. J Pediatr
Gastroenterol Nutr. 2010;50:123-131. DOI: 10.1097/MPG.0b013e3181c61995. Available at:
http://journals.lww.com/jpgn/Fulltext/2010/02000/Chronic_Hepatitis_C_VirusInfection_in_Children.5.aspx
#

Schwarz KB, Gonzalez-Peralta RP, Murray KF, et al. The combination of ribavirin and peginterferon is
superior to peginterferon and placebo for children and adolescents with chronic hepatitis C.
Gastroenterology. 2011;140:450-458. Available at: http://www.gastrojournal.org/article/S0016-
5085(10)01585-4/fulltexthttp://www.gastrojournal.org/article/S0016-5085(10)01585-4/fulltext -
article-footnote-1#article-footnote-1http://www.gastrojournal.org/article/S0016-5085(10)01585-
4/fulltext - article-footnote-2#article-footnote-2

Copyright 2012 American Academy of Pediatrics


2012 PREP SA ON CD-ROM

Question 71
A term infant is delivered by repeat elective cesarean section complicated by oligohydramnios.
Mild bilateral pelviectasis was noted on prenatal ultrasonography. Artificial rupture of the membranes at
delivery reveals scant meconium-stained fluid. The infant initially has a strong cry, but she develops
escalating respiratory distress and cyanosis requiring endotracheal intubation in the delivery room. Upon
arrival in the nursery, she is placed on the ventilator, with the following settings: peak inspiratory pressure
of 26 mm Hg, positive end-expiratory pressure of 6 mm Hg, rate of 40 breaths/min, and FiO2 of 100%.
Physical examination reveals central cyanosis, a full anterior fontanelle, II/VI systolic murmur at the left
lower sternal border, and slightly diminished breath sounds bilaterally. A right radial arterial blood gas
reveals:
pH, 7.10
PCO2, 70 mm Hg
PCO2, 26 mm Hg
Base excess, -7 mmol/L
Bicarbonate, 22 mmol/L
Of the following, the MOST appropriate next study is
A. chest radiography
B. echocardiography
C. head ultrasonography
D. renal ultrasonography
E. tracheal aspirate

Copyright 2012 American Academy of Pediatrics


2012 PREP SA ON CD-ROM

Critique 71 Preferred Response: A


A term infant who experiences respiratory failure at birth requires immediate stabilization that
begins with the establishment of an airway, effective ventilation, and achievement of adequate circulation.
If the infant fails to ventilate or oxygenate, such as the baby described in the vignette, the most useful
initial study for evaluation is the standard chest radiograph. A chest radiograph can rapidly exclude
mechanical issues such as incorrect endotracheal tube placement, pneumothorax, or pleural effusion,
while allowing assessment of situs, cardiac size and shape, pulmonary vascularity, and parenchymal
disease.
If the chest radiograph does not reveal a mechanical issue contributing to ineffective ventilation or
oxygenation, both pulmonary and nonpulmonary causes must be pursued. A complete blood count with
differential count, blood culture, serum glucose, serum lactate, and tracheal aspirate may be useful in the
evaluation of infectious or metabolic causes. Simultaneous preductal and postductal oximetry can allow
the assessment of persistent pulmonary hypertension, with a preductal saturation of at least 10% greater
than the postductal saturation suggesting right-to-left shunting of cardiac blood flow through the patent
foramen ovale and patent ductus arteriosus. Conversely, a postductal saturation at least 10% greater
than the preductal saturation can suggest cyanotic heart disease such as transposition of the great
vessels. Echocardiography is the definitive study to evaluate for structural heart disease or pulmonary
hypertension at the bedside, but it may not be immediately available.
Pneumonia, amniotic fluid aspiration, meconium aspiration, and respiratory distress syndrome
often appear on the chest radiograph as patchy infiltrates, atelectasis, or almost complete white-out.
Term infants whose chest radiographs are consistent with respiratory distress syndrome and who fail to
improve despite mechanical ventilation, oxygen, and repeated surfactant administration should have
bronchoalveolar lavage sent for assessment of surfactant protein B deficiency. Computed tomography
scan may be helpful if developmental lung abnormalities, such as congenital cystic adenomatoid
malformation, are suggested by radiographic findings. A relatively clear chest radiograph may be seen
with pulmonary hypertension, pulmonary hypoplasia, and congenital lymphangiectasia. Oligohydramnios
and suspected pulmonary hypoplasia may be associated with renal dysplasia, prompting evaluation with
renal ultrasonography.
If the infant fails to respond to aggressive respiratory management, extracorporeal membrane
oxygenation (ECMO) may be considered. Head ultrasonography to evaluate for intracranial hemorrhage
should be performed before cannulization because the heparinization on ECMO may lead to extension of
the hemorrhage. Lung biopsy may be considered to rule out congenital lymphangiectasia and capillary
alveolar dysplasia if the infant fails to improve on ECMO.

American Board of Pediatrics Content Specification(s):


Plan the evaluation of a full-term infant who has severe respiratory failure at birth that does not
respond to intubation and assisted ventilation

Suggested Reading:
Ballard RA, Hansen TN, Corbet A. Respiratory failure in the term infant. In: Taeusch HW, Ballard RA,
Gleason CA, eds. Averys Diseases of the Newborn. 8th ed. Philadelphia, PA: Elsevier Saunders;
2005:705-722

Flidel-Rimon O, Shinwell ES. Respiratory distress in the term and near-term infant. NeoReviews.
2005;6:e289-e297. DOI: 10.1542/neo.6-6-e289. Available at:
http://neoreviews.aappublications.org/cgi/content/full/6/6/e289

Copyright 2012 American Academy of Pediatrics


2012 PREP SA ON CD-ROM

Question 72
A previously well 22-month-old child has had the onset of pallor, jaundice, and dark urine over the
past 2 days. His parents report no fever, vomiting, or diarrhea. He continues to take fluids well, but he has
been very fatigued. Physical examination of the pale child reveals marked scleral icterus, temperature of
38.2C, heart rate of 132 beats/min, respiratory rate of 20 breaths/min, and blood pressure of 92/44 mm
Hg. He has a nonradiating II/VI systolic murmur audible at the left upper sternal border, clear lungs, and
splenomegaly 3 cm below the left costal margin. He has 4+ hemoglobinuria. Other laboratory test results
include:
Hemoglobin, 5.8 g/dL (58 g/L)
3 9
White blood cell count, 14.8x10 /mcL (14.8x10 /L)
3 9
Platelet count, 240x10 /mcL (240x10 /L)
Spherocytes, nucleated red blood cells, and polychromasia on peripheral blood smear
Reticulocyte count, 15% (0.15)
Serum creatinine, 0.6 mg/dL (53.0 mcmol/L)
Of the following, the additional laboratory test that is MOST likely to show an abnormal result is
A. antistreptolysin O titer
B. direct antiglobulin (Coombs) test
C. fecal occult blood
D. hepatitis B surface antigen
E. urine culture

Copyright 2012 American Academy of Pediatrics


2012 PREP SA ON CD-ROM

Critique 72 Preferred Response: B


The child described in the vignette presents with symptoms and signs that suggest hemolytic
anemia, including jaundice, dark urine from hemoglobinuria, pyrexia, a flow murmur, and fatigue. The
laboratory studies confirm the presence of anemia, and the elevated reticulocyte count and nucleated red
blood cells (RBCs) on peripheral smear indicate that this is a problem with RBC loss or destruction rather
than production. The presence of jaundice suggests RBC destruction. The direct antiglobulin (Coombs)
test is the most appropriate next step in the evaluation of hemolytic anemia.
The presence of jaundice in the child in the vignette could suggest the possibility of liver
dysfunction, including that resulting from hepatitis B, but the presence of profound anemia and
hemoglobinuria are more indicative of a hemolytic process. Dark, cola-colored urine is a typical finding
in poststreptococcal glomerulonephritis, which is evaluated appropriately with an antistreptolysin O titer,
but the presence of severe anemia with an elevated reticulocyte count, splenomegaly, and a normal
serum creatinine make glomerulonephritis a less likely possibility. Although a urinary tract infection may
be associated with hyperbilirubinemia in newborns, the physical findings and laboratory studies for this
patient are not typical of such an infection. Blood loss detected by fecal occult blood testing is an
important cause of anemia and may be associated with elevated reticulocyte counts, but such a diagnosis
does not explain this patients jaundice, hemoglobinuria, or splenomegaly.
Hemolytic anemia can be due to RBC trauma, infection, membrane disorders, enzymatic
abnormalities, hemoglobinopathies, or immune-mediated mechanisms. Microangiopathic trauma causing
hemolytic anemia is seen in disseminated intravascular coagulation, hemolytic-uremic syndrome,
thrombocytopenic purpura, preeclampsia, eclampsia, and malignant hypertension. Infectious agents can
induce many of the other mechanisms leading to hemolysis, but some microbes (Plasmodium, Babesia,
Bartonella bacilliformis) invade RBCs directly, leading to extravascular hemolysis; Clostridium perfringens
releases a toxin that disrupts the RBC membrane. Erythrocytes with either enzyme defects
(spherocytosis, elliptocytosis) or hemoglobinopathies (sickle cell disease, thalassemias) are subject to
extravascular hemolysis in the spleen. Family and personal history might suggest the presence of either
of these types of genetic conditions.
Immune-mediated hemolysis occurs when the patient develops antibodies against antigens on
the RBC surface and is characterized by the presence of microspherocytes on the peripheral blood smear
and a positive direct antibody (Coombs) test. Alloimmune hemolysis is seen with mismatched blood
transfusions and can be severe and acute (eg, ABO incompatibility) or delayed, usually the result of minor
RBC antigens. Autoimmune hemolytic anemia, as exhibited by the boy in the vignette, has an estimated
incidence of about 1 to 3 cases per 100,000 population (adult and child) per year, with children affected
less frequently than adults. Although most cases are considered idiopathic, children often have a
preceding infection, and the condition is well documented after infectious mononucleosis and
Mycoplasma pneumoniae infection. Many commonly used medications, including the penicillins, can
produce drug-related immune-mediated hemolysis, but this is rare in children. In addition,
lymphoproliferative diseases and autoimmune conditions (systemic lupus erythematosus) can present as
autoimmune hemolytic anemia.

2American Board of Pediatrics Content Specification(s):


Know that jaundice, dark urine, and a sudden change in exercise tolerance may indicate a hemolytic
anemia

Suggested Reading:

Dhaliwal G, Cornett PA, Tierney LM Jr. Hemolytic anemia. Am Fam Physician. 2004;69:2599-2606.
Available at: http://www.aafp.org/afp/2004/0601/p2599.html

Vaglio S, Arista MC, Perrone MP, et al. Autoimmune hemolytic anemia in childhood: serologic features in
100 cases. Transfusion. 2007;47:50-54. DOI: 10.1111/j.1537-2995.2007.01062.x. Abstract available at:
http://www.ncbi.nlm.nih.gov/pubmed/17207229

Copyright 2012 American Academy of Pediatrics


2012 PREP SA ON CD-ROM

Zuckerman KS. Approach to the anemias. In: Goldman L, Ausiello D, eds. Cecil Medicine. 23rd ed.
Philadelphia, PA: Saunders Elsevier; 2008:chapter 162

Copyright 2012 American Academy of Pediatrics


2012 PREP SA ON CD-ROM

Question 73
A 2-month-old boy presents for a health supervision visit. Physical examination reveals a
normally growing and thriving infant whose left testis is easily palpable but whose right testis cannot be
palpated. The remainder of his physical examination results are normal.
Of the following, the MOST appropriate approach is to
A. order chromosomal studies to confirm genetic sex
B. order pelvic ultrasonography
C. plan for surgical exploration at 6 months of age
D. plan for surgical exploration at 2 years of age
E. refer the child immediately for urgent surgical consultation

Copyright 2012 American Academy of Pediatrics


2012 PREP SA ON CD-ROM

Critique 73 Preferred Response: C


The infant described in the vignette has a testis that cannot be palpated in the scrotum. True
undescended testis is the most common congenital anomaly, occurring in 3% to 5% of newborn males.
Most undescended testes subsequently descend during early infancy, with only 1% of boys having an
undescended testis at age 1 year. Therefore, urgent surgical consultation is not necessary for this child.
Bilateral undescended testes may be associated with more severe urogenital disorders and are generally
evaluated completely before hospital discharge of the newborn.
The concern for the testis that is not located in the scrotum is the risk for diminished
spermatozoal production and motility, leading to a reduction in fertility and malignant transformation. In
such infants, abdominal testicular malignancy would be difficult to detect and, therefore, treat before
metastasis occurs. Accordingly, early orchiopexy of the undescended testis is important.
Controversy surrounds optimum timing of surgical exploration and orchiopexy, the utility of
ultrasonography, and the use of hormone therapy to induce descent. Recent evidence supports the
optimum timing of surgical intervention at no later than 6 months. Biopsy results show that testes that
have remained out of the scrotum beyond even 1 year may be smaller and less likely to grow compared
with those for which surgery was performed earlier. There is no role for ultrasonography in the
management of unilateral undescended testis because surgical exploration is indicated regardless of
outcome to avoid malignant degeneration of a testis that is very small or technically difficult to see.
Adjunctive use of hormones to encourage descent remains controversial, although it may occasionally be
beneficial and is used more commonly in Europe.
There is no need to perform genetic studies for a boy who has a normal penis and one
descended testis. In contrast, bilaterally undescended testes require genetic and endocrinologic
evaluation in the newborn hospital period, as noted previously.
The differential diagnosis of an undescended testis includes retractile testis, but this is uncommon in the
infant and usually does not appear until after age 1. With retractile testis, a brisk cremasteric reflex
causes the testicle to ascend during palpation of the perineum or medial thigh.
Increasingly, the phenomenon of acquired or ascending undescended testis has been seen in
older boys, typically between the ages of 4 and 10 years. The testis is descended on examinations in
infancy and early childhood, but it does not remain in the scrotum. Such an ascending undescended
testis may result in the previously noted complications. Accordingly, clinicians should conduct testicular
examinations in all male patients, regardless of age.

American Board of Pediatrics Content Specification(s):


Distinguish between undescended testes and retractile testes
Plan the appropriate management of a patient with undescended testes

Suggested Reading:
Kokorowski PJ, Routh JC, Graham DA, Nelson CP. Variations in timing of surgery among boys who
underwent orchidopexy for cryptorchidism. Pediatrics. 2010;126:e576-e582. DOI: 10.1542/peds.2010-
0747. Available at: http://pediatrics.aappublications.org/cgi/content/full/126/3/e576

La Scala GC, Ein SH. Retractile testes: an outcome analysis on 150 patients. J Pediatr Surg.
2004;39:1014-1017. DOI: 10.1016/j.jpedsurg.2004.03.057. Abstract available at:
http://www.ncbi.nlm.nih.gov/pubmed/15213889

Ritzn EM. Undescended testes: a consensus on management Eur J Endocrinol. 2008;159(suppl 1):S87-
S90. DOI: 10.1530/EJE-08-0181. Available at: http://eje-online.org/cgi/content/full/159/suppl_1/S87

Copyright 2012 American Academy of Pediatrics


2012 PREP SA ON CD-ROM

Question 74
You are seeing a 4-week-old previously healthy infant in your office because of concern about
poor feeding. On questioning, the parents report that the child has developed grunting respirations
associated with feedings, diaphoresis, pallor, and prolonged periods of sleep. On physical examination,
the boys heart rate is 160 beats/min, respiratory rate is 55 breaths/min, blood pressure in the right arm is
75/48 mm Hg, blood pressure in the left leg is 88/55 mm Hg, and oxygen saturation is 95%. He exhibits
tachypnea, rales and retractions, a II/VI low-pitched holosystolic murmur across the precordium (Item
Q74), and a palpable liver 2 cm below the right costal margin.
Of the following, the MOST likely explanation for the childs findings of congestive heart failure is
A. aortic valve stenosis
B. coarctation of the aorta
C. tetralogy of Fallot
D. transposition of the great arteries
E. ventricular septal defect

Copyright 2012 American Academy of Pediatrics


2012 PREP SA ON CD-ROM

Critique 74 Preferred Response: E


The natural history of a ventricular septal defect (VSD) depends on the magnitude of the left-to-
right shunt. A moderate-to-large VSD is associated with a lower-grade murmur, as described for the infant
in the vignette, due to the lack of a pressure gradient between the two ventricles. In this case, there is
systemic pressure in the right ventricle, and assuming that there is no obstruction to pulmonary outflow
(eg, pulmonary stenosis or infundibular muscular obstruction of the right ventricular outflow tract, as is
present in tetralogy of Fallot), the pulmonary artery pressure also is at systemic levels. Congestive heart
failure (CHF) evolves over the first 1 to 2 postnatal months as the pulmonary vascular resistance naturally
falls. In contrast, smaller defects are restrictive, resulting in a louder and higher-pitched murmur due to
the pressure gradient between the normal systemic pressure of the left ventricle and the much lower
pulmonary artery pressure of the right ventricle. In these patients, the magnitude of the left-to-right shunt
is minimal and, therefore, pulmonary overcirculation and associated CHF do not occur. Other causes of
neonatal and infant CHF include any congenital heart lesion that results in either a large left-to-right shunt
(eg, atrioventricular canal defect or patent ductus arteriosus) or pulmonary hypertension (eg, mitral valve
disorders), myocarditis, severe anemia, drug toxicity, or metabolic/mitochondrial disorders.
Symptoms of CHF include tachypnea, poor feeding, diaphoresis, and poor energy. Affected
children grow poorly due to reduced caloric intake from their lack of energy and resultant poor intake and
an unmet increase in metabolic requirements. A combination of pharmacologic and nutritional
interventions is often required. The standard pharmacologic regimen includes diuretics, digoxin, and
afterload-reducing agents such as captopril. Diuretics reduce the volume of shunting to the lung bed,
thereby reducing dilation of the left atrium and left ventricle. Such left heart dilation is a result of the
inefficient left-to-right shunt in which previously oxygenated blood from the left heart is again shunted
through the VSD back out to the pulmonary circulation. Many children receive human milk or formula
fortified to achieve a caloric density between 22 and 30 kcal/oz, depending upon caloric needs.
Medication and calories are titrated to achieve a weight gain of 15 to 30 g/day.
Obstructive left heart lesions (such as coarctation of the aorta and aortic stenosis) can result in
CHF if the obstructive process is severe, but neither would result in the murmur described in the vignette.
Tetralogy of Fallot more commonly produces cyanosis due to reduced pulmonary circulation rather than
CHF due to pulmonary overcirculation. Similarly, transposition of the great arteries is associated with
neonatal cyanosis and does not cause the signs and symptoms of CHF.

American Board of Pediatrics Content Specification(s):


Recognize the common causes of congestive heart failure in infants and children

Suggested Reading:
Macicek SM, Macias CG, Jefferies JL, Kim JJ, Price JF. Acute heart failure syndromes in the pediatric
emergency department. Pediatrics. 2009;124:e898-e904. DOI: 10.1542/peds.2008-2198. Available at:
http://pediatrics.aappublications.org/cgi/content/full/124/5/e898

Madriago E, Silberbach M. Heart failure in infants and children. Pediatr Rev. 2010;31:4-12. DOI:
10.1542/pir.31-1-4. Available at: http://pedsinreview.aappublications.org/cgi/content/full/31/1/4

Minette MS, Sahn DJ. Ventricular septal defects. Circulation. 2006;114:2190-2197. DOI:
10.1161/CIRCULATIONAHA.106.618124. Available at:
http://circ.ahajournals.org/cgi/content/full/114/20/2190

Copyright 2012 American Academy of Pediatrics


2012 PREP SA ON CD-ROM

Question 75
The parents of an 11-year-old girl have become concerned about their daughters waning
strength. She has always been a strong competitive swimmer, but for the past 2 years, her race times
have not improved. They state that she seems to have progressive difficulty pulling her body out of the
pool, and at this summers first swim competition, her younger sisters freestyle time was faster than hers.
There is no family history of diseases causing progressive weakness. On physical examination, the girl
has normal mental status, cranial nerves, and muscle bulk and tone. Reflexes are 1+. She has 4/5
strength in neck flexors, deltoids, and hip flexors and extensors and 5/5 strength in distal muscles.
Of the following, the MOST useful initial diagnostic test for this girl is
A. brain magnetic resonance imaging
B. cervical spine magnetic resonance imaging
C. electromyography
D. nerve conduction velocities
E. serum creatine kinase

Copyright 2012 American Academy of Pediatrics


2012 PREP SA ON CD-ROM

Critique 75 Preferred Response: E


The girl described in the vignette has chronic progressive weakness that requires a thoughtful
and detailed diagnostic evaluation to identify possible causes (Item C75). The constellation of gradually
progressive proximal weakness, with proximal weakness on examination and normal reflexes and
sensation, suggests that the disease involves muscle pathology, which is most appropriately assessed by
measuring serum creatine kinase.
Children who have acute/subacute (progressing over hours to several days) generalized
weakness are in imminent danger of respiratory failure and require evaluation of forced vital capacity or
negative inspiratory force. Because the child in the vignette has a prolonged course, the evaluation can
proceed systematically, guided primarily by the:
1) Distribution of weakness identified by history
2) Time course: acute/subacute/chronic, static/progressive/waxing and waning, sudden exacerbations
followed by recovery
3) Family history: autosomal dominant, autosomal recessive, X-linked, or mitochondrial inheritance
patterns
4) Neurologic examination, whose aim is to localize the source of the weakness on the axis from motor
cortex to muscle (brain, brainstem, spinal cord, anterior horn cell, root, nerve, neuromuscular junction,
muscle)
Applying these four points to the girl in the vignette:
1) The childs parents note difficulty with swimming and exiting the pool, which suggests bilateral upper
body proximal weakness. The physician should ask further about (distal) hand tasks such as opening
objects or writing and leg tasks such as walking up and down stairs or running.
2) The time course is chronic and progressive.
3) The family history is unrevealing, which supports autosomal recessive, autosomal dominant with
reduced penetrance or due to spontaneous mutation, or possibly other inheritance patterns.
4) The examination findings of diffuse proximal weakness with normal mentation, sensation, and reflexes
are consistent with muscle pathology as the source of the weakness.
Serum creatine kinase is an easy, relatively inexpensive, useful test because elevated serum
values generally indicate muscle disease to be the likely source of weakness. Brain magnetic resonance
imaging may eventually be part of the diagnostic evaluation, but diffuse bilateral weakness in the absence
of mentation changes greatly reduces the likelihood of a primary brain source. Similarly, absence of any
cranial nerve deficits and the normal sensory examination findings support an intact brainstem and spinal
cord. Electromyography obtains data about the electrical firing within muscle. This test can be very
informative in the setting of certain types of muscle or nerve disease, but it involves needle insertion, is
uncomfortable and expensive, and is not generally obtained as an initial diagnostic test. Nerve conduction
studies are used to measure the velocity and strength (amplitude) of electrical signaling through the
nerves, clarifying whether a demyelinating (slowed velocity) or axonal (reduced amplitude) neuropathy
may be involved in the disease. The intact sensory examination and preserved reflexes for this girl do not
support the presence of a neuropathy.

American Board of Pediatrics Content Specification(s):


Distinguish among acute and chronic causes of weakness
Know the benefits and limitations of ancillary neurodiagnostic tests in the evaluation of weakness (eg,
serum creatine kinase activity, electromyography

Suggested Reading:
Greenberg SA. Inflammatory myopathies: evaluation and management. Semin Neurol. 2008;28:241-249.
DOI: 10.1055/s-2008-1062267. Abstract available at: http://www.ncbi.nlm.nih.gov/pubmed/18351525

McNally EM, Pytel P. Muscle diseases: the muscular dystrophies. Annu Rev Pathol. 2007;2:87-109.
Abstract available at: http://www.ncbi.nlm.nih.gov/pubmed/18039094

Copyright 2012 American Academy of Pediatrics


2012 PREP SA ON CD-ROM

Sarnat HB. Neuromuscular disorders: evaluation and investigation. In: Kliegman RM, Stanton BF, St.
Geme JW III, Schor NF, and Behrman RE, eds. Nelson Textbook of Pediatrics. 19th ed. Philadelphia, PA:
Saunders Elsevier, 2011:2109-2112

Copyright 2012 American Academy of Pediatrics


2012 PREP SA ON CD-ROM

Critique 75

Item C75. Common Causes of Acute and Chronic Weakness and Their Neuroanatomic Localization
Location Key Features Examples
Brain/Cerebrum If unilateral: Face and arm or face, Vascular insults, stroke, cerebral palsy,
arm, and leg on same side genetic/metabolic disorders
If right body: aphasia
Visual field cut possible
Brainstem Eye, face, palate/tongue, crossed Brainstem tumors, demyelination
findings
Spinal cord Sensory level, bowel/bladder Stroke, transverse myelitis, spastic
involvement paraplegias
Root Back pain, hyporeflexia Guillain Barr syndrome
Nerve Hyporeflexia, Multifocal or mononeuropathies, hereditary
root/nerve/dermatomal patterns of sensorimotor neuropathies
sensory loss

Junction Fluctuation, fatigability, cranial Myasthenia gravis, botulism


nerve involvement
Muscle Proximal weakness, pain Myositis, myopathies, dystrophies
Paroxysmal Triggered Channelopathies/periodic paralyses

Copyright 2011 American Academy of Pediatrics


2012 PREP SA ON CD-ROM

Question 76
A 18-year-old college student who has a history of attention-deficit/hyperactivity disorder (ADHD)
presents to your office as a new patient, requesting a renewal prescription for methylphenidate. You note
that her current supply, according to the empty bottle that she hands you, should be refilled in 2 weeks.
She fills out a release of information form for her prior physician, but that person cannot be reached
today. When you ask why she takes this medication, she replies, It helps me pay attention. When you
ask why the bottle is empty, she states, The pills spilled in the sink.
Of the following, the MOST appropriate next step is to
A. ask for a urine sample to perform a urine drug screen
B. ask her to return in 1 to 2 weeks to give you adequate time to contact the previous prescribing
physician
C. complete a brief medical history and write a prescription for a 1-month supply
D. complete an inventory to assess risk for alcohol and recreational drug use and write the
requested prescription for 1 month
E. have her complete a Vanderbilt Diagnostic Rating Scale form

Copyright 2012 American Academy of Pediatrics


2012 PREP SA ON CD-ROM

Critique 76 Preferred Response: B


The girl described in the vignette has a stated history of ADHD. Her answers to the questions are
limited, vague, and therefore suspicious, so further specific information is needed to verify appropriate
use of stimulants. Furthermore, requests to refill a stimulant prescription long before it is due for a refill
are suggestive of substance diversion. The best next step in this situation is to delay refilling the
prescription until more information becomes available.
The Vanderbilt Diagnostic Rating Scale is meant to be completed by parents and teachers; it is
not a patient self-report form. Validated report rating scales such as the Connors Adult ADHD Rating
Scale (CAARS) or the Adult ADHD Self Report Rating Scale (ASRS-v1.1) are appropriate self-
administered forms that could be used for this patient. Obtaining a urine drug screen is not indicated at
this time because it does not confirm or refute a diagnosis of ADHD. In addition, it does not confirm or
refute a diagnosis of substance use disorder because it can only identify recent drug use. Although
obtaining a medical history and assessing the risk for alcohol and recreational drug use is appropriate,
these do not provide sufficient information to support prescribing methylphenidate at this time.
Nonmedical use of stimulant and nonstimulant medication used to treat ADHD has been reported
to be highest in the 18- to 25-year-old group (4.3%). Individuals who have prescriptions for ADHD were
the leading source of medication diversion. Most individuals using the medication for nonmedical reasons
reported obtaining it from friends or family. Productivity was the most commonly cited reason for
nonmedical use of ADHD prescription medications, which suggests possible self-medication for
nonmedical use or underdosing or noncompliance by those who have the prescriptions. Concomitant use
of alcohol and other drugs has been reported to be as high as 68% among individuals who self-report
nonmedical use.
Opportunities to discourage diversions are limited because most medications used to treat ADHD
are covered under Schedule II of the Controlled Substance Act. Physicians should be educated about
potential drug diversion and need to establish individual office policies and procedures that address such
concerns, such as providing a replacement for a lost prescription no more than once.

AAP Mental Health Competency:


Know how to approach diversion of stimulant medications

Suggested Reading
American Academy of Pediatrics Task Force on Mental Health. Child and Adolescent Substance Use
Fact Sheet. Addressing Mental Health Concerns in Primary Care: A Clinician's Toolkit [CD-ROM]. Elk
Grove Village, IL: American Academy of Pediatrics; 2010

American Academy of Pediatrics Task Force on Mental Health. Addressing Mental Health Concerns in
Primary Care: A Clinician's Toolkit [CD-ROM]. Elk Grove Village, IL: American Academy of Pediatrics;
2010

McCabe SE, Boyd CJ. Sources of prescription drugs for illicit use. Addict Behav. 2005;30:1342-1350.
DOI: 10.1016/j.addbeh.2005.01.012. Available at:
http://www.ncbi.nlm.nih.gov/pmc/articles/PMC1706073/?tool=pubmed

Novak SP, Kroutil LA, Williams RL, van Brunt D. The nonmedical use of prescription ADHD medications:
results from a national Internet panel. Subst Abuse Treat Prev Policy. 2007;2:32. DOI: 10.1186/1747-
597X-2-32. Available at: http://www.ncbi.nlm.nih.gov/pmc/articles/PMC2211747/

Office of Applied Studies. 2005 National Survey on Drug Use & Health: Detailed Tables. Rockville, MD:
Substance Abuse and Mental Health Services Administration; 2008. Available at:
http://oas.samhsa.gov/NSDUH/2k5NSDUH/tabs/TOC.htm#TopOfPage

Copyright 2012 American Academy of Pediatrics


2012 PREP SA ON CD-ROM

Question 77
During a health supervision visit for a 5-year-old girl, her mother reports that she herself is
currently 16 weeks pregnant. She expresses concern about this pregnancy because her brother and two
of her maternal uncles died from complications of hemophilia A. She was told at the time of her most
recent ultrasonographic examination that she is carrying a male fetus. She asks if you can tell her the
chances that her son will be affected with hemophilia A.
Of the following, the chance that her son will be affected is CLOSEST to
A. 12.5%
B. 25%
C. 50%
D. 75%
E. 90%

Copyright 2012 American Academy of Pediatrics


2012 PREP SA ON CD-ROM

Critique 77 Preferred Response: B


The pregnant mother described in the vignette has a 50% chance of being a carrier for
hemophilia A based on the fact that her mother is an obligate carrier because she (the pregnant womans
mother) had affected brothers and an affected son (Item C77). This is the typical pattern of transmission
seen with X-linked recessive inheritance, but without specific testing, only probability can be used to
assess the patients mothers risk. With a 50% a priori risk of the mother carrying hemophilia A, the
chances that her male fetus will be affected is half of that (25%). Because the pregnant woman has two X
chromosomes, if she is a carrier, there is a 50:50 chance of passing on the one with the X-linked
recessive gene mutation. Another approach to calculate her risk is times (risk for her to have
received the hemophilia A gene mutation from her mother times the risk for her to pass on that X-
chromosome) = or 25%. The risk of 12.5% (half of 25%) would apply to this womans daughters
offspring if the baby boy/her brother turns out to be unaffected and no molecular or diagnostic testing is
undertaken to determine actual carrier status. The 50% risk applies to a known carrier female, with 75%
and 90% being much greater than expected for an X-linked or even autosomal dominant condition.
Hemophilia A is a serious condition, with risks for bleeding in the newborn period (eg,
circumcision) and with minor falls and minor procedures such as dental extractions. In light of this, if a
male infant is at-risk for a bleeding disorder based on the family history, circumcision or other non-urgent
invasive procedures should be deferred until he has been evaluated.
Molecular testing for hemophilia A is available through commercial laboratories and is extremely
valuable in clarifying a womans carrier status in the context of a positive family history. The best initial
person to test is an affected boy because almost 50% of boys who have severe hemophilia A have a
specific intron 22 inversion. In cases of severe hemophilia without this inversion, as well as milder forms
of factor VIII deficiency, sequencing of the gene may be necessary to identify other pathogenic point
mutations. Molecular testing ideally should be performed before a pregnancy because testing results are
available only after several weeks and more than one person in the family (eg, affected male and possible
female carrier) needs to be tested.

American Board of Pediatrics Content Specification(s):


Recognize the clinical features associated with an X-linked recessive disorder
Recognize the inheritance patterns of X-linked recessive disorders

Suggested Reading:
Brower C, Thompson AR. Hemophilia A. GeneReviews. 2008. Available at:
http://www.ncbi.nlm.nih.gov/bookshelf/br.fcgi?book=gene&part=hemo-a

Sharathkumar AA and Pipe SW. Bleeding Disorders. Pediatr. Rev., Apr 2008; 29:121-130. Available at:
http://pedsinreview.aappublications.org/content/29/4/121

Copyright 2012 American Academy of Pediatrics


2012 PREP SA ON CD-ROM

Critique 77

(Courtesy of A Johnson)
Pedigree, as described for the family in the vignette.

Copyright 2011 American Academy of Pediatrics


2012 PREP SA ON CD-ROM

Question 78
A 16-year-old girl presents to the emergency department with right upper quadrant abdominal
pain. She has no fever or other systemic symptoms. The pain began a few days after her last menstrual
period, which was heavier than usual. The pain is not related to meals and not accompanied by any
scapular or shoulder pain. She does complain of intermittent dysuria. On physical examination, the girl is
in mild distress from the pain. Her vital signs are normal, and her body mass index is 21.2. You hear
normal bowel sounds. Palpation of her abdomen reveals diffuse tenderness that is accentuated in the
right upper quadrant, with no rebound. A pregnancy test is negative. Urinalysis and urine culture results
are pending.
Of the following, the MOST appropriate next step is
A. abdominal ultrasonography
B. abdominal upright radiography
C. complete blood count
D. emergency surgical consultation
E. pelvic examination

Copyright 2012 American Academy of Pediatrics


2012 PREP SA ON CD-ROM

Critique 78 Preferred Response: E


A sexual history should be obtained for an adolescent female who presents with abdominal pain,
such as the girl described in the vignette, and the possibility of a sexually transmitted infection should be
considered. The presence of right upper quadrant pain raises the possibility of perihepatitis or Fitz-Hugh-
Curtis syndrome (FHCS) that results from pelvic inflammatory disease (PID) when the infection tracks
along the peritoneal gutter to the right upper quadrant. A pelvic examination is the most appropriate next
step for this girl to rule out PID.
Although typically associated with salpingitis, FHCS may present without other signs of PID and,
thus, may mimic other abdominal emergencies. The girls history of a recent heavier menstrual period is
an important clue for a pelvic infection.
Intermittent dysuria, although suggestive of a urinary tract infection, also may indicate urethritis
from a sexually transmitted infection. Urinalysis and culture should be obtained after the pelvic
examination. The lack of relationship of pain to meals and lack of radiation to the shoulder in this
nonobese girl makes gallbladder disease less likely. Therefore, ultrasonography and a surgical
consultation should be postponed until the pelvic examination has been completed. The history and
physical examination findings do not suggest a stone, obstruction, or perforation, making abdominal
radiography of little value. Results of a complete blood count may be helpful but are not diagnostic.

American Board of Pediatrics Content Specification(s):


Recognize that pelvic inflammatory disease may be a cause of right upper quadrant pain in an
adolescent girl (Fitz-Hugh-Curtis syndrome)

Suggested Reading:
Chandran L, Boykan R. Chlamydial infections in children and adolescents. Pediatr Rev. 2009;30:243-250.
DOI: 10.1542/pir.30-7-243. Available at: http://pedsinreview.aappublications.org/cgi/content/full/30/7/243

Holder NA. Gonococcal infections. Pediatr Rev. 2008;29:228-234. DOI: 10.1542/pir.29-7-228. Available
at: http://pedsinreview.aappublications.org/cgi/content/full/29/7/228

Miller CA, Shafer M-AB. Chlamydia trachomatis. In: Neinstein LS, Gordon CM, Katzman DK, Rosen DS,
Woods ER, eds. Adolescent Health Care: A Practical Guide. 5th ed. Philadelphia PA: Lippincott Williams
& Wilkins, a Wolters Kluwer business; 2008:805-818

Ross A, LeLeiko NS. Acute abdominal pain. Pediatr Rev. 2010;31:135-144. DOI: 10.1542/pir.31-4-135.
Available at: http://pedsinreview.aappublications.org/cgi/content/full/31/4/135

Shrier LA. Bacterial sexually transmitted infections: gonorrhea, chlamydia, pelvic inflammatory disease,
and syphilis. In: Emans SJH, Laufer MR, Goldstein DP, eds. Pediatric and Adolescent Gynecology. 5th
ed. Philadelphia, PA: Lippincott Williams & Wilkins, a Wolters Kluwer business; 2005:565-614

Copyright 2012 American Academy of Pediatrics


2012 PREP SA ON CD-ROM

Question 79
You are evaluating a 2-month-old infant in the emergency department whose parents state that
she has had trouble breathing for the past week. The infant was born at term via vaginal delivery and had
no prenatal or neonatal complications. The parents explain that for the past couple of days, she appears
to be breathing fast and seems to suck her chest in when she breathes. Physical examination reveals a
thin infant in moderate respiratory distress whose temperature is 37.0C, heart rate is 150 beats/min,
respiratory rate is 50 breaths/min, blood pressure is 74/48 mm Hg, and oxygen saturation is 94% in room
air. Her lungs are clear to auscultation, but she has suprasternal and subcostal retractions. She also
bobs her head with inspiration. The nurse was able to suction the nasopharynx by passing a suction
catheter through each nostril, but there was no improvement in the infants respiratory status. You order
chest radiography (Item Q79).
Of the following, the MOST likely cause of this infants symptoms is
A. bronchomalacia
B. choanal atresia
C. mediastinal tumor
D. pneumonia
E. pneumothorax

Copyright 2012 American Academy of Pediatrics


2012 PREP SA ON CD-ROM

Question 79

(Courtesy of B Poss)
Chest radiograph of the infant described in the vignette.

Copyright 2012 American Academy of Pediatrics


2012 PREP SA ON CD-ROM

Critique 79 Preferred Response: C


Airway obstruction, as described for the infant in the vignette, can be a life-threatening problem
that can have a variety of causes, including anatomic abnormalities, trauma, malignancies, infection, and
foreign bodies. Croup remains the leading cause of upper airway obstruction in children, followed by
foreign bodies. Among the signs of airway obstruction are dyspnea, stridor, cough, gagging, retractions,
or respiratory failure. The first priority in evaluating patients who have signs of airway obstruction is to
determine the degree of obstruction by observing for audible sounds, effective air movement, and
adequate respiratory effort. The infant in the vignette has clear evidence of respiratory distress, as
evidenced by the presence of tachypnea and retractions. Chest radiography (Item C79A) demonstrates a
widened mediastinum, marked shift of the trachea to the right, and normal lung fields, findings that are
consistent with an intrathoracic mass. Follow-up computed tomography scan of the chest demonstrates a
mediastinal mass causing severe narrowing of the trachea (Item C79B) and (Item C79C).
Acute respiratory failure due to a new-onset chest mass is uncommon but must be recognized
quickly and managed appropriately to prevent fatal consequences. Clinical presentations can vary,
depending on the degree of vascular and airway compression by the mass, but 60% of children present
with respiratory symptoms. Superior vena cava compression results in head, neck, and upper extremity
edema. Initial management is independent of the cause of the mass and consists of prompt attention to
the potential for airway compromise, which can occur in up to 20% of patients.
Choanal atresia is the most common congenital anomaly of the nose and can present with a
spectrum of breathing patterns ranging from noisy breathing to respiratory distress, but the ability to pass
a suction catheter easily through each nostril excludes this diagnosis in this infant. Bronchomalacia, loss
of airway patency due to insufficient cartilage or compression, typically presents in infancy but is
characterized by expiratory wheezing. Pneumonia and pneumothorax can both present with acute
respiratory failure, but the presence of normal breath sounds on physical examination and results of chest
radiography make these diagnoses unlikely.

American Board of Pediatrics Content Specification(s):


Recognize the signs associated with severe airway obstruction
Recognize intrathoracid airway obstruction by x-ray study of the chest

Suggested Reading:
Gangadharan SP. Evaluation of mediastinal masses. UpToDate Online 18.3. 2010. Available for
subscription at: http://www.utdol.com/online/content/topic.do?topicKey=pulm_dxs/4387

Loftis LL. Emergent evaluation of acute upper airway obstruction in children. UpToDate Online 18.3.
2009. Available for subscription at: http://www.utdol.com/online/content/topic.do? ped_symp/2976

Pope J, McBride J. Consultation with the specialist: respiratory failure in children. Pediatr Rev.
2004;25:160-167. DOI: 10.1542/10.1542/pir.25-5-160. Available at:
http://www.uptodate.com/contents/emergent-evaluation-of-acute-upper-airway-obstruction-in-
children?source=search_result&search=emergent+evaluation+of+acute+upper+airway+obstruction+in+c
hildren&selectedTitle=1%7E150

Copyright 2012 American Academy of Pediatrics


2012 PREP SA ON CD-ROM

Critique 79

(Courtesy of B Poss)
Chest radiograph of the infant described in the vignette demonstrates a widened mediastinum, marked
shift of the trachea to the right (arrows), and normal lung fields, findings that are consistent with an
intrathoracic tumor.

Copyright 2011 American Academy of Pediatrics


2012 PREP SA ON CD-ROM

Item 79

(Courtesy of B Poss)
Computed tomography scan of the chest of the infant described in the vignette demonstrating a 4x4-cm
mediastinal mass with significant deviation of the trachea to the right.

Copyright 2011 American Academy of Pediatrics


2012 PREP SA ON CD-ROM

Critique 79

(Courtesy of B Poss)
Computed tomography scan of the chest with three-dimensional reconstruction demonstrating marked
narrowing of the trachea (arrow).

Copyright 2011 American Academy of Pediatrics


2012 PREP SA ON CD-ROM

Question 80
An 8-year-old girl presents for evaluation of neck swelling. Her mother reports that the swelling
began approximately 1 year ago and has progressed with time. The swelling is not causing any pain. On
physical examination, the girls thyroid gland is diffusely enlarged, has a cobblestone texture, but has no
discrete nodules (Item Q80). Findings for all other systems are within normal parameters. Free thyroxine
and thyroid-stimulating hormone measurements are normal.
Of the following, the BEST next step in the evaluation of this patient is to
A. assess thyroglobulin
B. assess thyroid peroxidase antibodies
C. assess thyroid-stimulating immunoglobulins
D. order neck ultrasonography
E. perform a thyroid biopsy

Copyright 2012 American Academy of Pediatrics


2012 PREP SA ON CD-ROM

Question 80

(Courtesy of M Rimsza)
Thyroid enlargement, as described for the girl in the vignette.

Copyright 2012 American Academy of Pediatrics


2012 PREP SA ON CD-ROM

Critique 80 Preferred Response: B


The girl described in the vignette has classic signs and symptoms of chronic lymphocytic
thyroiditis (Hashimoto thyroiditis). Patients who have chronic lymphocytic thyroiditis are commonly
euthyroid (have normal free thyroxine and thyroid-stimulating hormone values) for many years despite
having other physical (goiter) or laboratory (autoantibody) signs of autoimmune thyroid disease. Although
this girl has thyromegaly, as many as two thirds of patients who have Hashimoto thyroiditis have atrophic
disease that results in small or even nonpalpable glands. The most sensitive laboratory test to confirm the
presence of autoimmune thyroid disease is measurement of autoantibodies specific to thyroid antigens. In
the case of Hashimoto thyroiditis, antibodies to thyroid peroxidase alone have greater than 85%
sensitivity. Sensitivity increases to more than 90% if thyroglobulin autoantibodies are also measured.
Thyroglobulin concentrations (not to be confused with thyroglobulin antibody titers) simply reflect
the total volume of thyroid tissue and are expected to be elevated in patients who have thyromegaly.
Thus, measuring thyroglobulin is not helpful in determining the cause of thyromegaly. Thyroid-stimulating
immunoglobulins are an excellent tool for assessing risk for Graves disease (which should result in a
large, smooth, and firm gland) but are not specific for Hashimoto disease. On rare occasions, patients
who have negative antibodies can still be proven to have Hashimoto thyroiditis by virtue of a biopsy and
histologic review of thyroid tissue. However, unless the patient has a discrete nodule that is palpable on
physical examination, neither ultrasonography nor thyroid biopsy is indicated during the initial evaluation.
Of note, patients who have any type of autoimmune thyroid disease are at increased lifetime risk
for thyroid nodules and thyroid cancers compared with the general population. Pain on palpation of the
thyroid could be indicative of acute or subacute thyroiditis but is rarely noted in those who have
Hashimoto thyroiditis.

American Board of Pediatrics Content Specification(s):


Recognize the signs and symptoms of Hashimoto thyroiditis

Suggested Reading:
Fisher DA, Grueters A. Thyroid disorders in childhood and adolescence. In: Sperling MA, ed. Pediatric
Endocrinology. 3rd ed. Philadelphia, PA: Saunders; 2008:227-253

Foley TP Jr. Hypothyroidism. Pediatr Rev. 2004;25:94-100. DOI: 10.1542/pir.25-3-94. Available at:
http://pedsinreview.aappublications.org/cgi/content/full/25/3/94

Polak M, Van Vliet G. Disorders of the thyroid gland. In: Sarafoglou K, Hoffmann G, Roth K, eds. Pediatric
Endocrinology and Inborn Errors of Metabolism. New York, NY: McGraw-Hill Professional; 2009:355-382

Copyright 2012 American Academy of Pediatrics


2012 PREP SA ON CD-ROM

Question 81
A 7-year-old child who has quadriplegia uses a wheelchair for mobility. He requires assistance for
eating and dressing due to his delay in motor skills. He has difficulty articulating his words. His parents
are concerned that he is becoming increasingly frustrated due to his difficulty expressing his needs. They
ask your guidance in helping him to become a more effective communicator.
Of the following, the MOST appropriate intervention is to
A. evaluate him for an amplification system
B. evaluate him for an augmented communication device
C. focus on improving his fine motor skills
D. refer him for behavioral therapy
E. teach him American sign language

Copyright 2012 American Academy of Pediatrics


2012 PREP SA ON CD-ROM

Critique 81 Preferred Response: B


The child described in the vignette has cerebral palsy (CP), which hinders his ability to
communicate. CP is a varied group of neuromotor conditions that affect movement or posture and create
weakness due to a static brain lesion, injury, or malformation that occurs in utero or during the first 2
postnatal years. The use of an augmented communication device should help to alleviate this boys
frustration. These devices provide individuals who have limited communication with the means to
augment their verbal and written expression. The devices range from picture communication boards to
computers that have synthesized speech output.
An amplification system is used to help individuals who have auditory processing difficulties and
would not specifically target expressive language. Individuals who have CP have motor planning
difficulties that would make learning sign language challenging. Although behavior therapy could aid the
boy in handling frustration, it would not deal with the underlying language issues. Fine motor skill
development also would not address his difficulties with language.
Children who have CP also may have significant feeding and mobility issues due to their motor
difficulties. Feeding therapy helps to promote appropriate food intake. Physical therapy focuses on gross
motor skills and functional mobility. Physical assistive devices for individuals who have CP include
walkers, crutches, orthoses, and wheelchairs. Educational services provide individualized special
educational programming to promote functional skills and independence.

American Board of Pediatrics Content Specification(s):


Know the principles of management for children with cerebral palsy (eg, feeding, spasticity, mobility,
activities of daily living, education)

Suggested Reading:
Burstein JR, Wright-Drechsel ML, Wood A. Assistive technology. In: Dormans JP, Pellegrino L, eds.
Caring for Children with Cerebral Palsy: A Team Approach. Baltimore, MD: Paul H. Brookes Publishing
Co; 1998:371-389

Cooley WC and the Committee on Children with Disabilities. Providing a primary care medical home for
children and youth with cerebral palsy. Pediatrics. 2004;114:1106-1113. DOI: 10.1542/peds.2004-1409.
Available at: http://pediatrics.aappublications.org/cgi/content/full/114/4/1106

Michaud LJ and the Committee on Children With Disabilities. Prescribing therapy services for children
with motor disabilities. Pediatrics. 2004;113:1836-1838. Available at:
http://pediatrics.aappublications.org/cgi/content/full/113/6/1836

Schultz MB, Blasco PA. Motor development. In: Voight RG, Macias MM, Myers SM, eds. American
Academy of Pediatrics Developmental and Behavioral Pediatrics. Elk Grove Village, IL: American
Academy of Pediatrics; 2011:147-170

Solot CB. Promoting function: communication. In: Dormans JP, Pellegrino L, eds. Caring for Children with
Cerebral Palsy: A Team Approach. Baltimore, MD: Paul H. Brookes Publishing Co; 1998:347-369

Copyright 2012 American Academy of Pediatrics


2012 PREP SA ON CD-ROM

Question 82
An 11-month-old boy presents to your office with a 5-day history of fever, nasal congestion,
conjunctivitis, and the development of a rash over the past 24 hours. The rash began on his head and
neck and spread to his trunk (Item Q82) and extremities. The family recently returned from a trip to
Ireland. His past medical history is unremarkable, and his immunizations are up to date.
Of the following, the BEST test for diagnosing this childs condition is
A. measles immunoglobulin (Ig) M serology
B. nasal aspirate for viral culture
C. rubella IgM serology
D. skin biopsy
E. throat culture for group A Streptococcus

Copyright 2012 American Academy of Pediatrics


2012 PREP SA ON CD-ROM

Question 82

(Courtesy of M Rimsza)
Rash, as described for the boy in the vignette.

Copyright 2012 American Academy of Pediatrics


2012 PREP SA ON CD-ROM

Critique 82 Preferred Response: A


The 5-day prodrome of respiratory symptoms, conjunctivitis, and fever followed by development
of a rash beginning on the head and neck and spreading to the trunk and extremities reported for the boy
in the vignette is suggestive of measles. Malaise and lesions on the buccal mucosa (Koplik spots) (Item
C82) just before rash development on the head and neck are other findings suggestive of clinical
measles. The Centers for Disease Control and Prevention clinical definition for measles is an illness
characterized by generalized rash lasting 3 days with temperature of 38.3C, cough, coryza, or
conjunctivitis and epidemiologic linkage to a confirmed case of measles. The rash of measles, which is
due to immune complex deposition, may not occur in immunosuppressed hosts.
Most measles cases in the United States arise from importation from countries where the disease
persists. Many European countries, including Ireland, remain endemic areas for measles. In 2010, Ireland
had the second highest incidence rate for measles in Europe. Although immunizations for the boy in the
vignette are described as up to date, an 11-month-old child would not have routinely received a measles
vaccine unless it was administered as part of the preparation for travel.
Measurement of specific measles immunoglobulin (Ig)M is the most reliable test for diagnosing
measles and is available through local health departments with a rapid turnaround time. As a
paramyxovirus, measles can be cultivated in tissue culture, but this is not routine in most clinical
laboratories.
Rubella IgM serology can be used to diagnosis rubella, but this illness is generally milder than
measles and does not have the extent of prodromal symptoms. A skin biopsy can detect IgG complexes
of measles antigen but would not be a routine test used for diagnosing typical measles. The rash of
scarlet fever may have morbilliform features, but the pattern of downward spread from the head and neck
and the prodromal symptoms described are more consistent with measles than group A streptococcal
infection.
Management of measles infection is supportive. Complications include otitis media,
laryngotracheobronchitis (croup), bacterial tracheitis, bronchopneumonia, and diarrhea. In
immunosuppressed children, severe bronchopneumonia may develop. Neurologic involvement may
include acute encephalitis (~1 in 1,000 cases) or subacute sclerosing panencephalitis, a rare
degenerative disease characterized by behavioral and intellectual deterioration and seizures that occurs 7
to 10 years after acute measles infection. Overall mortality from measles is 1 to 3 in 1,000 cases in the
United States, with higher death rates in immunocompromised individuals.
Individuals who received a killed measles vaccine in the 1960s have developed atypical measles,
characterized by distal extremity rash, nodular pneumonia, and neurologic symptoms, after exposure to
wild-type virus.

American Board of Pediatrics Content Specification(s):


Identify the complications of measles
Recognize the clinical manifestations of measles

Suggested Reading:
American Academy of Pediatrics. Measles. In: Pickering LK, Baker CJ, Kimberlin DW, Long SS, eds. Red
Book: 2009 Report of the Committee on Infectious Diseases. 28th ed. Elk Grove Village, IL: American
Academy of Pediatrics; 2009:444-455

Centers for Disease Control and Prevention. Complications of Measles. 2009. Available at:
http://www.cdc.gov/measles/about/complications.html

Centers for Disease Control and Prevention. Measles (Rubeola): 2010 Case Definition. 2010. Available
at: www.cdc.gov/ncphi/disss/nndss/casedef/measles_2010.htm

Copyright 2012 American Academy of Pediatrics


2012 PREP SA ON CD-ROM

Critique 82

(Courtesy of Walter W Tunnessen, Jr)


Koplik spots are tiny gray-to-white papules located on the buccal mucosae, often adjacent to the molars.

Copyright 2011 American Academy of Pediatrics


2012 PREP SA ON CD-ROM

Question 83
You are examining a 6-day-old infant who was born at 36-1/7 weeks gestation to a 19-year-old
primigravida by cesarean section due to preterm labor. The mother had no prenatal care. Apgar scores
were 7 and 8 at 5 and 10 minutes, respectively. The infant was stable in room air until today, when she
developed apnea requiring intubation, bradycardia, and profound hypotension. On physical examination,
the intubated infant appears pale. Auscultation of the lungs reveals diffuse rhonchi. The liver is palpable 4
cm below the costal margin. Laboratory findings include:
3 9
White blood cell count, 5.6x10 /mcL (5.6x10 /L) with 20% neutrophils, 68% lymphocytes, and 12%
monocytes
Hemoglobin, 10 g/dL (100 g/L)
3 9
Platelet count, 60x10 /mcL (60x10 /L)
Aspartate aminotransferase, 455 units/L
Alanine aminotransferase, 538 units/L
Cerebrospinal fluid examination reveals:
3
White blood cells, 10/mm with 80% lymphocytes and 20% monocytes
3
Red blood cells, 300/mm
Protein, 89 mg/dL (0.89 g/L)
Glucose, 57 mg/dL (3.2 mmol/L)
A chest radiograph demonstrates diffuse pneumonitis.
Of the following, the test MOST likely to yield the patients diagnosis is
A. blood culture for virus
B. cerebrospinal fluid Gram stain
C. human immunodeficiency virus Western blot
D. Mycoplasma polymerase chain reaction
E. rubella serology

Copyright 2012 American Academy of Pediatrics


2012 PREP SA ON CD-ROM

Critique 83 Preferred Response: A


The infant described in the vignette presents in the first postnatal week with clinical sepsis
associated with pneumonitis, respiratory failure, hypotension, hepatitis, and leukopenia (for a newborn)
accompanied by a lymphocyte predominance. The most likely cause of her illness is viral infection caused
by herpes simplex virus (HSV), although adenovirus and enterovirus infections also can occur. A viral
culture may grow HSV in 48 to 72 hours in patients with overwhelming sepsis. In addition, serum and
cerebrospinal fluid HSV polymerase chain reaction tests should be performed.
Most neonatal HSV disease is acquired in the peripartum period (85%), although postnatal (10%)
and congenital (5%) infection also can occur. Peripartum and postnatal disease (together referred to as
perinatal) are classified as follows:
1) Disseminated disease, which can involve multiple organ systems such as liver, lung, brain, skin, eye,
and adrenal glands
2) Central nervous system (CNS) disease
3) Skin, eye, and mouth (SEM) disease
Disseminated disease accounts for approximately 25% of perinatal HSV infection and occurs, on
average, at postnatal days 10 through 12. Patients usually present with viral sepsis characterized by
respiratory failure, pneumonitis, hepatitis (often with liver failure), and disseminated intravascular
coagulation. CNS involvement is common, occurring in nearly 75% of patients. However, 20% of
neonates who have disseminated disease do not have cutaneous vesicles at any point during their
illness. Therefore, a high index of suspicion is required to make the diagnosis when vesicles are absent.
With the use of intravenous acyclovir, the 12-month mortality for disseminated HSV disease has
decreased from 85% to 30%. Patients who have lethargy and severe hepatitis are at highest risk of death.
Among those who receive antiviral therapy for disseminated HSV disease, nearly 85% have normal
neurologic development.
CNS disease accounts for approximately 30% of perinatal HSV infection and usually presents in
the third postnatal week (days 16 through 19). Common clinical manifestations of CNS disease include
focal or generalized seizures, bulging fontanelle, lethargy, irritability, poor feeding, and temperature
instability. Associated cutaneous vesicles occur in 60% to 70% of patients at some point during the
course of illness. Mortality is associated with prematurity and seizures and usually results from severe
cerebral destruction, which can involve any and multiple parts of the brain. With antiviral therapy, the
mortality rate for CNS HSV disease is 4%, but only 30% of patients have normal neurologic development.
Infants who have SEM HSV disease account for 45% of perinatal HSV infection, and 80% to 85%
of affected patients have cutaneous vesicles or lesions (Item C83). By definition, SEM disease is limited
to the skin, eyes, or mouth. Mortality associated with SEM disease is negligible, and more than 98% of
infants treated with antiviral therapy have normal neurologic development.
Neonatal enteroviral sepsis syndrome is uncommon and usually presents in the first 14 days after
birth. Infants may have fever, lethargy, irritability, hepatomegaly, seizures, and bulging fontanelle in
addition to other findings. Chest radiography can reveal pulmonary infiltrates. Transaminitis is common,
and severe hepatitis can occur. Additional manifestations of severe neonatal enteroviral sepsis can
include meningoencephalitis, myocarditis, pancreatitis, and adrenalitis. The mortality rate for infants who
have sepsis syndrome involving the liver is greater than 80%.
Neonatal adenoviral sepsis is rare. It usually involves the lung, liver, and brain. Most cases are
fatal.
Human immunodeficiency virus, Mycoplasma, and rubella virus do not produce sepsis syndromes
in neonates, obviating the need for associated diagnostic tests in this infant. Cerebrospinal fluid Gram
stain is useful for the detection of bacteria but is not helpful for diagnosing viral infection.

American Board of Pediatrics Content Specification(s):


Recognize the clinical manifestations of herpes simplex virus infection in the neonatal period, and that
skin lesions are not always present

Suggested Reading:

Copyright 2012 American Academy of Pediatrics


2012 PREP SA ON CD-ROM

American Academy of Pediatrics. Adenovirus infections. In: Pickering LK, Baker CJ, Kimberlin DW, Long
SS, eds. Red Book: 2009 Report of the Committee on Infectious Diseases. 28th ed. Elk Grove Village, IL:
American Academy of Pediatrics; 2009:204-206

American Academy of Pediatrics. Enterovirus (nonpoliovirus) infections. In: Pickering LK, Baker CJ,
Kimberlin DW, Long SS, eds. Red Book: 2009 Report of the Committee on Infectious Diseases. 28th ed.
Elk Grove Village, IL: American Academy of Pediatrics; 2009:287-288

American Academy of Pediatrics. Herpes simplex. In: Pickering LK, Baker CJ, Kimberlin DW, Long SS,
eds. Red Book: 2009 Report of the Committee on Infectious Diseases. 28th ed. Elk Grove Village, IL:
American Academy of Pediatrics; 2009:363-373

American Academy of Pediatrics. Rubella. In: Pickering LK, Baker CJ, Kimberlin DW, Long SS, eds. Red
Book: 2009 Report of the Committee on Infectious Diseases. 28th ed. Elk Grove Village, IL: American
Academy of Pediatrics; 2009:579-584

Kimberlin DW, Palazzi DL, Whitley RJ. Therapy for perinatal and neonatal infections. In: Rudolph CD,
Rudolph AM, Lister GE, First L, Gershon AA, eds. Rudolphs Pediatrics. 22nd ed. New York, NY:
McGraw-Hill Professional, 2011:Chapter 230

Copyright 2012 American Academy of Pediatrics


2012 PREP SA ON CD-ROM

Critique 83

(Courtesy of D Palazzi)
Vesicles of neonatal herpes simplex virus infection.

Copyright 2011 American Academy of Pediatrics


2012 PREP SA ON CD-ROM

Question 84
A 5-year-old girl presents with mild flank pain. She has no history of fever, trauma, gross
hematuria, frequency, urgency, or dysuria. Physical examination of the afebrile child reveals a heart rate
of 90 beats/min, respiratory rate of 20 breaths/min, blood pressure of 106/62 mm Hg, and normal growth
parameters. There are no other findings of note. A dipstick urinalysis reveals a specific gravity of 1.020;
pH of 8.5; 2+ protein; and negative for blood, leukocyte esterase, and nitrite. A urine protein-to-creatinine
ratio performed on this specimen is subsequently reported as 0.02.
Of the following, the MOST likely explanation for this girls urinary findings is
A. alkaline urine
B. minimal change disease
C. orthostatic proteinuria
D. urinary tract infection
E. urolithiasis

Copyright 2012 American Academy of Pediatrics


2012 PREP SA ON CD-ROM

Critique 84 Preferred Response: A


Proteinuria is typically diagnosed on dipstick testing of the urine as part of the urinalysis. The
dipstick is very sensitive to the detection of albumin down to 15 mg/dL (trace), and the test is a
semiquantitative evaluation for proteinuria. The test strip incorporates tetrabromophenol blue as an
indicator that changes color when in contact with protein. False-positive results can be seen in the setting
of gross hematuria, chlorhexidine contamination, pH greater than 8.0, and phenazopyridine treatment.
False-negative dipstick tests for proteinuria can occur with dilute urine samples (urine specific gravity
<1.015). Nonalbumin proteinuria, such as beta-2 microglobulin, is not detected by urine dipstick testing.
A more quantitative test for assessing proteinuria in the pediatric setting is a urine protein-to-
creatinine ratio on a random sample. The accuracy and convenience of this test has obviated the need for
24-hour urine collections for proteinuria. The quantitative nature of the ratio allows more accurate
categorization of the proteinuria as physiologically appropriate or pathologic. The normal protein excretion
in adults is 150 mg/day. The normal values for the urine protein-to-creatinine (mg/mg) ratio is less than
0.2 for children 2 years of age and older. Children younger than 2 years of age can have slightly greater
protein excretion, with a normal protein-to-creatinine ratio of less than 0.5. By accounting for the urine
creatinine, this test can detect proteinuria even in the setting of a dilute sample.
Urinalysis for the girl in the vignette, who has mild flank pain, reveals 2+ protein on dipstick
testing. However, quantitative assessment from the same sample reveals a normal urine protein-to-
creatinine ratio. This finding can only occur if the dipstick test reading is a false-positive. Closer inspection
of the full dipstick reading shows a high urine pH. Thus, the most likely explanation for the patients 2+
protein on dipstick reading is a false-positive reading from alkaline urine.
Although orthostatic proteinuria is more common and the most common cause of nonpathologic
proteinuria in adolescents, this girls urine protein-to-creatinine ratio reveals the absence of proteinuria.
Other causes of nonpathologic proteinuria include fever and exercise. The absence of lower urinary tract
symptoms and either pyuria or bacteruria exclude urinary tract infection for this child. Urolithiasis, if
present, may result in hematuria, but isolated proteinuria should not occur. Finally, minimal change
disease should present with the clinical features of the nephrotic syndrome accompanied by 3 to 4+
proteinuria and a urine protein-to-creatinine ratio greater than 2.

American Board of Pediatrics Content Specification(s):


Recognize that a dipstick examination of an alkaline urine might yield a false-positive result for
proteinuria

Suggested Reading:
Abitbol C, Zilleruelo G, Freundlich M, Strauss J. Quantitation of proteinuria with urinary protein/creatinine
ratios and random testing with dipsticks in nephrotic children. J Pediatr. 1990;116:243-247. Abstract
available at: http://www.ncbi.nlm.nih.gov/pubmed/2299494

Bergstein JM. A practical approach to proteinuria. Pediatr Nephrol. 1999;13:697-700. Abstract available
at: http://www.ncbi.nlm.nih.gov/pubmed/10502130

Hogg RJ, Portman RJ, Milliner D, Lemley KV, Eddy A, Ingelfinger J. Evaluation and management of
proteinuria and nephrotic syndrome in children: recommendations from a pediatric nephrology panel
established at the National Kidney Foundation Conference on Proteinuria, Albuminuria, Risk,
Assessment, Detection, and Elimination (PARADE). Pediatrics. 2000;105:1242-1249. Available at:
http://pediatrics.aappublications.org/cgi/content/full/105/6/1242

Moxey-Mims M. Hematuria and proteinuria. In: Kher KK, Schnaper HW, Makker SP, eds. Clinical Pediatric
Nephrology. 2nd ed. London, England: Informa Healthcare; 2007:129-141

Copyright 2012 American Academy of Pediatrics


2012 PREP SA ON CD-ROM

Question 85
An 18-year-old boy requires a computed tomography scan with intravenous contrast for an
upcoming surgery. During his preoperative physical examination, he states that he experienced diffuse
urticaria and tongue angioedema when he underwent a previous imaging study that required intravenous
contrast.
Of the following, the BEST method to prevent future contrast reactions is to
A. administer 1 L intravenous normal saline before the procedure
B. perform desensitization to the contrast agent
C. pretreat with oral antihistamines and corticosteroids
D. use a contrast agent that has a low iodine content
E. use a high-osmolar contrast agent

Copyright 2012 American Academy of Pediatrics


2012 PREP SA ON CD-ROM

Critique 85 Preferred Response: C


Reactions to radiocontrast media (RCM) are estimated to occur in 1% to 12% of all individuals
receiving standard RCM. The mechanism for these reactions may be either immunoglobulin (Ig)E-
mediated or non-IgE-mediated, but almost all such reactions are non-IgE-mediated. Clinical symptoms
are indistinguishable because both result in mast cell degranulation and histamine release. Current
recommendations suggest pretreating individuals who have a history of RCM reactions with
corticosteroids and a combination of histamine-1 and histamine-2 antihistamines, which should reduce
the reaction risk to less than 1% (Item C85).
Initially, RCM reactions were believed to be due to the iodine content of the contrast, but studies
have demonstrated that the osmolarity of the solution is the primary cause. Compared with low-osmolar
RCM, high-osmolar RCM is associated with higher rates of adverse reactions during both initial and
subsequent exposures. Desensitization is a procedure used to allow safe administration of medications
that cause IgE-mediated reactions. It currently does not have a role in the treatment of RCM reactions
because most of these reactions are non-IgE-mediated. Appropriate hydration is important before any
procedure using contrast to limit the risk of nephrotoxicity, but there is no specific recommendation
regarding administration of intravenous fluid to decrease the risk for a RCM reaction.

American Board of Pediatrics Content Specification(s):


Know that reactions to contrast media are not IgE mediated and can be prevented by pre-treatment
with corticosteroids and antihistamines

Suggested Reading:
Hunt CH, Hartman RP, Hesley GK. Frequency and severity of adverse effects of iodinated and
gadolinium contrast materials: retrospective review of 456,930 doses. AJR Am J Roentgenol.
2009;193:1124-1127. DOI: 10.2214/AJR.09.2520. Available at:
http://www.ajronline.org/cgi/content/full/193/4/1124

Trcka J, Schmidt C, Seitz CS, Brcker E, Gross GE, Trautmann A. Anaphylaxis to iodinated contrast
material: nonallergic hypersensitivity or IgE-mediated allergy? AJR Am J Roentgenol. 2008;190:666-670.
DOI: 10.2214/AJR.07.2872. Available at: http://www.ajronline.org/cgi/content/full/190/3/666

Copyright 2012 American Academy of Pediatrics


2012 PREP SA ON CD-ROM

Critique 85

Item C85. Two Regimens Recommended by the American Academy of Roentgenology to Prevent
Radiocontrast Reactions
Regimen 1 Regimen 2
Prednisone 50 mg orally at 13 hours, 7 hours, Methylprednisolone 32 mg orally 12 hours and 2
and1 hour before injection hours before injection
Diphenhydramine 50 mg intravenously or orally Diphenhydramine can be added if desired
1 hour before injection

Copyright 2011 American Academy of Pediatrics


2012 PREP SA ON CD-ROM

Question 86
An 18-month-old boy is brought to the emergency department after being found in his
grandfathers room with several open pill bottles. The family reports that they removed two unidentifiable
tablets from his mouth and found 23 more scattered on the floor. The medications include terazosin,
simvastatin, aspirin, acetaminophen, and allopurinol. The sleepy but arousable child has a temperature of
37.0C, heart rate of 160 beats/min, respiratory rate of 24 breaths/min, and blood pressure of 66/34 mm
Hg. The remainder of his physical examination findings are normal.
Of the following, the medication that is MOST likely to be the cause of this childs clinical findings
is
A. acetaminophen
B. allopurinol
C. aspirin
D. simvastatin
E. terazosin

Copyright 2012 American Academy of Pediatrics


2012 PREP SA ON CD-ROM

Critique 86 Preferred Response: E


The boy described in the vignette is lethargic and hypotensive, presumably due to a toxic
ingestion. Of the medications listed, only terazosin, an alpha-blocker used to treat symptoms associated
with prostatic hypertrophy, causes hypotension. Many medications commonly used to treat various
conditions in adults, from hypertension to insomnia, may be found in a childs environment and ingested
accidentally. Hypotension following an ingestion can be an important clue to the identity of the ingested
agent (Item C86).
When evaluating a child following an unknown ingestion, it is important to collect information that
will help in identifying agents that may cause life-threatening symptoms or complications and have
specific treatments or antidotes. Assessment of vital signs, pupil size, skin, and neurologic status can
provide useful clues. More importantly, however, is initial stabilization of the patient and correction of any
vital sign derangements. The adage treat the patient, not the poison reminds the clinician that the
treatment of patients who have ingested toxic substances is primarily supportive, reactive, and often not
specific to the actual agent.
A patient who is hypotensive following an unknown ingestion should be treated initially with fluid
resuscitation. Fluid boluses of 20 mL/kg 0.9% saline should be administered rapidly, with careful
reassessment following each bolus. If the patients blood pressure does not normalize after three boluses,
vasopressor agents such as dopamine or dobutamine should be considered. Electrocardiography and
bedside glucose measurement also should be obtained to determine if the hypotension is related to a
cardiac dysrhythmia or hypoglycemia.

American Board of Pediatrics Content Specification(s):


Recognize the signs and symptoms of ingestion of medications that produce hypotension

Suggested Reading:
ODonnell KA, Burns Ewald M. Pediatric drug therapy: poisonings. In: Kleigman RM, Stanton BF, St.
Geme JW III, Schor NF, and Behrman RE, eds. Nelson Textbook of Pediatrics. 19th ed. Philadelphia, PA:
Saunders Elsevier; 2011:250-270

Velez LI, Shepherd JG, Goto CS. Approach to the child with occult toxic exposure. UpToDate Online
18.3. 2010. Available at: http://www.uptodate.com/online/content/topic.do?topicKey=ped_tox/3023

Copyright 2012 American Academy of Pediatrics


2012 PREP SA ON CD-ROM

Critique 86

Item C86. Toxin-related Symptom Complexes (Toxidromes)


Toxidrome Vital Signs Pupils Mental Status Skin Findings
STIMULANT Hyperthermia, Mydriasis Agitation, Diaphoresis
(eg, cocaine, amphetamines, tachycardia, hallucinations,
pseudoephedrine, theophylline, hypertension, tachypnea paranoia, seizures
caffeine)

ANTICHOLINERGIC Hyperthermia, Mydriasis Agitation, Dry, flushed


(eg, antihistamines, tricyclic tachycardia, hallucinations, delirium
antidepressants, antispasmodics, hypertension, tachypnea
phenothiazines, atropine,
belladonna alkaloids such as
Jimson weed)

HALLUCINOGENIC Hyperthermia, Mydriasis, Hallucinations, Diaphoresis


(eg, lysorgic acid diethylamide tachycardia, nystagmus agitation
(LSD), phencyclidine, psilocybin, hypertension, tachypnea
3,4 methylenedioxymeth-
amphetamine (MDMA, ecstacy)
3,4 methylenedioxy-N-
ethylamphetamine (MDEA)

OPIOID Hypothermia, Miosis Sedation, coma Dry, pale


(eg, morphine, heroin, oxycodone, bradycardia,
hydromorphone, diphenoxylate) hypotension, bradypnea

SEDATIVE-HYPNOTIC Hypothermia, Miosis or variable Sedation, stupor, Dry, pale


(eg, benzodiazepines, barbiturates, bradycardia, coma
alcohols) hypotension, bradypnea

CHOLINERGIC Bradycardia, Miosis Confusion, delirium, Diaphoresis (Remember


(eg, organophosphate and hypertension or coma, seizures SLUDGE: salivation,
carbamate insecticides, nerve hypotension, tachypnea lacrimation, urination,
agents, nicotine) or bradypnea defecation, gastroenteritis)

SEROTONIN SYNDROME Hyperthermia, Mydriasis Confusion, agitation, Diaphoresis, flushed


(eg, monoamine oxidase inhibitors, tachycardia, coma, rigidity
SSRIs) hypertension, tachypnea

Copyright 2011 American Academy of Pediatrics


2012 PREP SA ON CD-ROM

Question 87
A 16-year-old boy presents to your office with a history of increasing irritability, low mood, social
withdrawal, a decline in school performance, and decreased energy despite a marked increase in sleep.
These symptoms began approximately 1 year ago, when his father lost his job and the family had to
move. He has no history of substance use. The boy began counseling 3 months ago but has not
improved. Findings on physical examination are unremarkable. You administer a Patient Health
Questionnaire-9 (PHQ-9), which supports the diagnosis of depression, and you decide to suggest
treatment with a medication.
Of the following, the BEST choice for management is
A. bupropion
B. clomipramine
C. fluoxetine
D. paroxetine
E. trazodone

Copyright 2012 American Academy of Pediatrics


2012 PREP SA ON CD-ROM

Critique 87 Preferred Response: C


The boy described in the vignette has typical findings for depression. Although the condition
initially may have been related to environmental stressors, symptoms have persisted despite counseling
and are affecting his performance and quality of life. The degree and duration of symptoms warrant
consideration of pharmacotherapy. The drug of choice is a selective serotonin reuptake inhibitor (SSRI),
and fluoxetine has been shown to be efficacious in evidence-based studies. In addition, it has been
approved by the United States Food and Drug Administration (FDA) for treatment of depression in this
age group. Other SSRIs may be effective, but there is less supporting evidence for their use in children.
Of note, use of paroxetine should be avoided in children and adolescents because it is less well tolerated,
is more commonly associated with withdrawal symptoms, and may be teratogenic (category D risk).
Combined serotonin-norepinephrine reuptake inhibitors (eg, venlafaxine and duloxetine) and tricyclic
antidepressants (eg, clomipramine, imipramine, desipramine) are not first-line medications for depression
in this age group. Clomipramine is useful for obsessive-compulsive disorder (not depression), and
trazodone may be helpful for both initiation and maintenance of sleep but has not been shown to improve
depression in children. There is less evidence for the use of bupropion than fluoxetine for adolescent
depression.
Because depression is a risk factor for suicidality, this boy should be screened and monitored for
this risk. Although the SSRI medications carry a black box warning for suicidal ideation, the rate of
completed suicides is significantly higher in patients who have depression and receive no treatment.
It is likely that counseling will be more beneficial once the patient is receiving medication.
However, the clinician should clarify that the counseling he is receiving is a therapeutic modality that is
evidence-based (eg, cognitive behavior therapy, interpersonal psychotherapy) and that the counselor has
appropriate education and experience with the modality. A bidirectional release of information can help
identify the type of therapy and will facilitate the sharing of data and further treatment planning.
Proper monitoring of treatment response can be assisted by baseline and periodic administration
of standardized depression symptom rating scales, such as the PHQ-9 and Kutcher Adolescent
Depression Scale-11 Item (KADS-11).

AAP Mental Health Competency:


Identify the evidence based selective serotonin reuptake inhibitor (SSRI) choices for child
depression

Suggested Reading:
American Academy of Child and Adolescent Psychiatry. Practice parameter for the assessment and
treatment of children and adolescents with suicidal behavior. J Am Acad Child Adolesc Psychiatry.
2001;40(7 suppl):24S-51S. Abstract available at: http://www.ncbi.nlm.nih.gov/pubmed/11434483

American Academy of Pediatrics. Depression. In: Addressing Mental Health Concerns in Primary Care: A
Clinicians Toolkit. Elk Grove Village, IL: American Academy of Pediatrics; 2010

Birmaher B, Brent D; AACAP Work Group on Quality Issues, Bernet W, et al. Practice parameter for the
assessment and treatment of children and adolescents with depressive disorders. J Am Acad Child
Adolesc Psychiatry. 2007;46:1503-1526. Abstract available at:
http://www.ncbi.nlm.nih.gov/pubmed/18049300

Cheung AH, Zuckerbrot RA, Jensen PS, Ghalib K, Laraque D, Stein REK, GLAD-PC Steering Group.
Guidelines for adolescent depression in primary care (GLAD-PC): II. treatment and ongoing
management. Pediatrics. 2007(5);120:e1313-e1326. Available at:
http://pediatrics.aappublications.org/content/120/5/e1313.full

Jensen PS, Cheung AH, Zuckerbrot R,Ghalib K, Levitt A. Guidelines for Adolescent Depression in
Primary Care (GLAD-PC) Tool Kit. 2007. Available at:
http://www.thereachinstitute.org/files/documents/GLAD-PCToolkit.pdf.

Copyright 2012 American Academy of Pediatrics


2012 PREP SA ON CD-ROM

US Preventive Services Task Force. Screening and treatment for major depressive disorder in children
and adolescents: US Preventive Services Task Force recommendation statement. Pediatrics.
2009;123:1223-1228. DOI: 10.1542/peds.2008-2381. Available at:
http://pediatrics.aappublications.org/cgi/content/full/123/4/1223

Williams SB, OConnor EA, Eder M, Whitlock EP. Screening for child and adolescent depression in
primary care settings: a systemic evidence review for the US Preventive Services Task Force. Pediatrics.
2009;123:e716-e735. DOI: 10.1542/peds.2008-2415. Available at:
http://pediatrics.aappublications.org/cgi/content/full/123/4/e716

Zuckerbrot RA, Cheung AH, Jensen PS, Stein REK, Laraque D, and the GLAD-PC Steering Group.
Guidelines for adolescent depression in primary care (GLAD-PC): I. identification, assessment, and initial
management. Pediatrics. 2007;120(5):e1299-e1312. Available at:
http://pediatrics.aappublications.org/content/120/5/e1299.full

Copyright 2012 American Academy of Pediatrics


2012 PREP SA ON CD-ROM

Question 88
You are seeing a 15-year-old girl because of abdominal pain. She was well until 6 months ago,
when she began complaining of epigastric discomfort occurring at any time of the day. She has been
awakened several times at night because of pain. Because of the severity of discomfort, she visited the
local emergency department 3 days ago, and the following study results were obtained:
Hemoglobin, 13.8 g/dL (138 g/L)
3 9
White blood cell count, 8.5x10 /mcL (8.5x10 /L)
Erythrocyte sedimentation rate, 10 mm/hr
Liver function studies, normal
Amylase, 45 units/L (normal, 10 to 50 units/L)
Helicobacter pylori antibody, positive
She denies diarrhea, constipation, fevers, or weight loss. She has experienced occasional
nausea and two or three episodes of vomiting, most recently this morning. Her menses are regular.
Physical examination of the well-developed, well-nourished adolescent reveals moderate, direct
abdominal tenderness in the epigastrium and the left upper quadrant. Rectal examination yields no
findings of note, with an empty ampulla, and the examining glove tests negative for occult blood.
Of the following, the MOST appropriate next step is
A. abdominal ultrasonography
13
B. C-urea breath test
C. lansoprazole, amoxicillin, and clarithromycin therapy
D. stool H pylori antigen testing
E. upper gastrointestinal tract endoscopy

Copyright 2012 American Academy of Pediatrics


2012 PREP SA ON CD-ROM

Critique 88 Preferred Response: E


The girl described in the vignette presents with a history of epigastric discomfort associated with
occasional nausea, vomiting, and nocturnal symptoms, and physical examination confirms tenderness in
the epigastrium. All laboratory studies, with the exception of a positive H pylori antibody titer, yield normal
results. Although the positive H pylori antibody result suggests the possibility of disease associated with
this bacterium, available evidence clearly indicates that serologic tests for H pylori infection are unreliable
markers of disease. Normative values and interpretation of results vary widely from laboratory to
laboratory, and antibody titers have been shown to exhibit poor sensitivity and specificity as predictors of
disease. Thus, an upper gastrointestinal tract endoscopy is the most appropriate next step to establish a
13
definitive diagnosis and plan further therapy. Other diagnostic studies (H pylori stool antigen, C-urea
breath test) show promise as diagnostic tools, but their utility in pediatric patients has not been confirmed.
Endoscopy remains the gold standard for evaluating H pylori disease in children and adolescents.
Radiologic or ultrasonographic evaluation is of little value.
Investigation of the child who has chronic abdominal pain requires careful attention to the clinical
history and physical examination findings before undertaking a diagnostic evaluation. The decision to
proceed with endoscopy for the girl in the vignette is based on a combination of factors, including pain
location (epigastrium), presence of nocturnal symptoms, and associated factors (nausea, vomiting) that
suggest the likelihood of upper gastrointestinal tract inflammatory disease. In this case, endoscopy
demonstrated a nodular gastropathy, prominent in the antrum (Item C88A). Histologic sections of the
endoscopically obtained antral biopsy confirmed a diagnosis of H pylori-associated chronic gastritis.
Indications for treatment of H pylori infection remain controversial. Therapy aimed at eradicating
the organism should clearly proceed under the following conditions:
Endoscopically confirmed gastric or duodenal ulcer
Histologically proven gastric metaplasia
Gastric mucosa-associated lymphoid tissue (MALT) lymphoma
Prior ulcer disease and current active infection
In other clinical situations, such as for this girl, who has nodular gastropathy, the decision to treat
is less clear. Although she does not satisfy any of the previously cited strict criteria, current best evidence
suggests that bacterial eradication offers an excellent prospect for symptomatic relief in the patient who
has confirmed infection, histologic evidence of gastritis, and symptoms that alter the quality of life.
The standard treatment for H pylori infection in children is triple therapy with a proton pump
inhibitor combined with two antibiotics. This regimen has been shown to be very effective in clearing the
organism from the stomach. Current recommendations call for a 14-day course of treatment, and the first-
line therapeutic alternatives (depending on the presence of any antibiotic allergy) are shown in (Item
C88B). Although eradication failures are primarily the consequence of noncompliance, bacterial
resistance to one or more antibiotics (especially metronidazole and clarithromycin) is becoming a more
common problem.

American Board of Pediatrics Content Specification(s):


Plan the treatment of a Helicobacter pylori infection

Suggested Reading:
Bourke B, Ceponis P, Chiba N, et al; Canadian Helicobacter Study Group. Canadian Helicobacter Study
Group consensus conference: update on the approach to Helicobacter pylori infection in children and
adolescents--an evidence-based evaluation. Can J Gastroenterol . 2005;19:399-408. Available at:
http://www.pulsus.com/journals/abstract.jsp?origPg=abstract.jsp&sCurrPg=journal&jnlKy=2&atlKy=1385&
isuKy=258&isArt=t&&HCtype=Physician

Chelimsky G, Czinn S. Peptic ulcer disease in children. Pediatr Rev. 2001;22:349-355. DOI:
10.1542/pir.22-10-349. Available at: http://pedsinreview.aappublications.org/cgi/content/full/22/10/349

Di Lorenze C, Colletti RB, Lehmann HP, et al; AAP Subcommittee and NASPGHAN Committee on
Chronic Abdominal Pain. Chronic abdominal pain in children: a technical report of the American Academy
of Pediatrics and the North American Society for Pediatric Gastroenterology, Hepatology and Nutrition. J

Copyright 2012 American Academy of Pediatrics


2012 PREP SA ON CD-ROM

Pediatr Gastroenterol Nutr. 2005;40:249261. Available at:


http://journals.lww.com/jpgn/Fulltext/2005/03000/Chronic_Abdominal_Pain_In_Children__A_Technical.2.
aspx

Gold BD, Colletti RB, Abbott M, et al; North American Society for Pediatric Gastroenterology and
Nutrition. Helicobacter pylori infection in children: recommendations for diagnosis and treatment. J Pediatr
Gastroenterol Nutr. 2000;31:490-497. Available at:
http://journals.lww.com/jpgn/Fulltext/2000/11000/Helicobacter_pylori_Infection_in_Children_.7.aspx

Guarner J, Kalach N, Elitsur Y, Koletzko S. Helicobacter pylori diagnostic tests in children: review of the
literature from 1999 to 2009. Eur J Pediatr. 2010;169:15-25. Available at:
http://journals.lww.com/jpgn/Fulltext/2000/11000/Helicobacter_pylori_Infection_in_Children_.7.aspx

Kato S, Sherman PM. What is new related to Helicobacter pylori infection in children and teenagers? Arch
Pediatr Adolesc Med. 2005;159:415-421. Available at: http://archpedi.ama-
assn.org/cgi/content/full/159/5/415

Kindermann A, Lopes AI. Helicobacter pylori infection in pediatrics. Helicobacter. 2009;14 (suppl 1):52-
57. DOI: 10.1111/j.1523-5378.2009.00700.x. Abstract available at:
http://www.ncbi.nlm.nih.gov/pubmed/19712169

Copyright 2012 American Academy of Pediatrics


2012 PREP SA ON CD-ROM

Critique 88

(Courtesy of A Bousvaros)
Endoscopic view of the stomach demonstrating nodular inflammation in the gastric antrum, a common
feature of Helicobacter pylori gastritis.

Copyright 2011 American Academy of Pediatrics


2012 PREP SA ON CD-ROM

Critique 88

Item C88B. First-line Eradication Therapies for H pylori Disease in Children


! Amoxicillin ! 50 mg/kg per day up to 1 g BID
! Clarithromycin ! 15 mg/kg per day up to 500 mg BID
! Lansoprazole (or comparable PPI at ! 1 mg/kg per day up to 30 mg BID
acid-inhibitory doses)
! Amoxicillin ! 50 mg/kg per day up to 1 g BID
! Metronidazole ! 20 mg/kg per day up to 500 mg BID
! Lansoprazole (or comparable PPI at ! 1 mg/kg per day up to 30 mg BID
acid-inhibitory doses)
! Clarithromycin ! 15 mg/kg per day up to 500 mg BID
! Metronidazole ! 20 mg/kg per day up to 500 mg BID
! Lansoprazole (or comparable PPI at ! 1 mg/kg per day up to 30 mg BID
acid-inhibitory doses)

PPI=proton pump inhibitor

Copyright 2011 American Academy of Pediatrics


2012 PREP SA ON CD-ROM

Question 89
A 1-month-old infant who was born at 29 weeks gestation develops vomiting. Her hospital course
included respiratory distress syndrome that required 2 days of mechanical ventilation and
hyperbilirubinemia that required phototherapy. She has been taking full nasogastric feedings for 1 week
that consist of preterm cow milk-based formula concentrated to 24 kcal/oz. She has had no previous
feeding issues. The infant has had emesis with the last two feedings that appeared to be partially
digested, slightly green-tinged, and an estimated 20% of the feeding volume. She last passed a stool 24
hours ago. Physical examination reveals an alert infant who has a mildly distended abdomen with
hypoactive bowel sounds that appears slightly tender to gentle palpation. To evaluate her abdomen
further, you order an abdominal radiograph (Item Q89).
Of the following, the MOST likely diagnosis is
A. constipation
B. cow milk protein intolerance
C. ileal stenosis
D. intestinal malrotation
E. necrotizing enterocolitis

Copyright 2012 American Academy of Pediatrics


2012 PREP SA ON CD-ROM

Question 89

(Courtesy of D Mulvihill)
Abdominal radiograph, as described for the infant in the vignette.

Copyright 2012 American Academy of Pediatrics


2012 PREP SA ON CD-ROM

Critique 89 Preferred Response: E


The infant described in the vignette has pneumatosis intestinalis on the abdominal radiograph
(Item C89), which is the hallmark of necrotizing enterocolitis (NEC). NEC is primarily a disease of the
preterm infant, with fewer than 10% of cases occurring in term infants. The development of NEC is
associated with enteral feeding, particularly with nonhuman milk. NEC often presents several weeks after
the initiation of feedings with nonspecific clinical findings. The classic triad includes abdominal distension,
gastric aspirates, and guaic-positive stools, although NEC also must be considered in an infant who
develops apnea, lethargy, and temperature instability with evidence of mild feeding intolerance.
An infant who has clinical findings suggestive of NEC should be evaluated promptly with an
abdominal radiograph and laboratory studies. Nonspecific early radiographic findings are similar to those
of an ileus, including distended loops, air fluid levels, and occasional thickening of the bowel wall. The
diagnostic radiographic signs of NEC are pneumatosis intestinalis (gas in the bowel wall), hepatic portal
venous gas, or pneumoperitoneum. Laboratory studies also yield nonspecific findings, although
worrisome results include leukocytosis with bandemia, neutropenia, thrombocytopenia, hyponatremia,
hyperkalemia, and metabolic acidosis.
If the abdominal radiograph for this infant had not demonstrated findings consisted with NEC, the
clinician may have obtained an upper gastrointestinal imaging study due to the green-tinged emesis that
is also associated with intestinal malrotation. No evidence of obstruction is seen on the abdominal
radiograph, which is not consistent with ileal stenosis. Cow milk protein intolerance can mimic the early
clinical findings of NEC, but it is not associated with pneumatosis intestinalis on imaging. Constipation
may also present with abdominal distension and gastric aspirates, but the abdominal radiograph should
only demonstrate dilated loops or stool.
The Bell staging system for NEC is a combination of clinical, radiographic, and laboratory findings
that guide clinical management. Initial management includes bowel rest following decompression of the
bowel with an orogastric tube, intravenous fluids, broad-spectrum antibiotics, and serial abdominal
radiographs. Consultation with a pediatric surgeon is strongly recommended because up to one third of
affected infants require surgical intervention.
The mortality for infants who have NEC ranges from 15% to 25%, with markedly higher morbidity
and mortality among those who require surgery. Intestinal stricture formation is a late complication of
NEC, independent of whether surgical intervention was needed. If bowel resection was necessary, late
complications include short gut syndrome, failure to thrive, and hyperalimentation hepatitis. Infants who
are managed without surgery have neurodevelopmental outcomes similar to age-matched preterm
infants; those who require surgical intervention have a significantly increased risk of poor
neurodevelopmental outcome.

American Board of Pediatrics Content Specification(s):


Know the usual presentation of necrotizing enterocolitis and plan initial management
Know that the radiographic finding of pneumatosis intestinalis is the hallmark of necrotizing
enterocolitis
Recognize that intestinal stricture formation is a late complication of necrotizing enterocolitis

Suggested Reading:
Berseth CL, Poenaru D. Necrotizing enterocolitis and short bowel syndrome. In: Taeusch HW, Ballard
RA, Gleason CA, eds. Averys Diseases of the Newborn. 8th ed. Philadelphia, PA: Elsevier Saunders;
2005:1123-1133

Epelman M, Daneman A, Navarro OM, et al. Necrotizing enterocolitis: review of state-of-the-art imaging
findings with pathologic correlation. Radiographics. 2007;27:285-305. DOI: 10.1148/rg.272055098.
Available at: http://radiographics.rsnajnls.org/content/27/2/285.long

Henry MCW, Moss RL. Necrotizing enterocolitis. Annu Rev Med. 2009;60:111-124. DOI:
10.1146/annurev.med.60.050207.092824. Abstract available at:
http://www.ncbi.nlm.nih.gov/pubmed/18817461

Copyright 2012 American Academy of Pediatrics


2012 PREP SA ON CD-ROM

Stevenson DK, Blakely ML. Historical perspectives: necrotizing enterocolitis: an inherited or acquired
condition? NeoReviews. 2006;7:e125-e134. DOI: 10.1542/neo.7-3-e125. Available at:
http://neoreviews.aappublications.org/cgi/content/full/7/3/e125

Walsh MC, Kliegman RM, Hack M. Severity of necrotizing enterocolitis: Influence on outcome at 2 years
of age. Pediatrics 1989;84:808-814. Available at:
http://pediatrics.aappublications.org/content/84/5/808.full.pdf+html

Copyright 2012 American Academy of Pediatrics


2012 PREP SA ON CD-ROM

Critique 89

(Courtesy of D Mulvihill)
Abdominal radiograph demonstrating dilated loops of bowel and pneumatosis intestinalis (arrow).

Copyright 2011 American Academy of Pediatrics


2012 PREP SA ON CD-ROM

Question 90
You are performing a follow-up evaluation on a 4-year-old boy in whom anemia was diagnosed

during a Head Start screening. He has been receiving 3 mg/kg per day of elemental iron for the past 6
weeks. His mother reports no changes in activity or appetite. He drinks two to three glasses of milk each
day and eats a relatively diverse diet. His growth parameters are in the 75th percentile, and his physical
examination findings are normal. Results of laboratory tests include:
Hemoglobin. 9.4 g/dL (94 g/L)
Hematocrit, 29% (0.29)
Mean corpuscular volume, 66 fL
Red cell distribution width, 12.2%
6 12
Red blood cell count, 5.8 x 10 /mcL (5.8 x 10 /L)
Reticulocyte count, 1% (0.01)
Ferritin, 54 ng/mL (121.3 pmol/L) (normal, 7 to 140 ng/mL [15.7 to 314.6 pmol/L])
Of the following, the MOST appropriate next step is to
A. obtain direct and indirect antiglobulin (Coombs) tests
B. obtain hemoglobin electrophoresis
C. perform a bone marrow aspirate
D. repeat the course of iron
E. transfuse with packed red blood cells

Copyright 2012 American Academy of Pediatrics


2012 PREP SA ON CD-ROM

Critique 90 Preferred Response: B


Anemia is defined as having hemoglobin and hematocrit values more than 2 standard deviations
below the mean for age, and it can be classified morphologically as microcytic, macrocytic, or normocytic.
The boy described in the vignette has microcytic anemia (defined by a low mean corpuscular volume
[MCV]), which is the most frequent type of anemia in childhood. Although nutritional iron deficiency is the
most common underlying cause for microcytosis, other conditions, particularly beta thalassemia trait, also
can have this presentation. Rarer causes of microcytic anemia include lead poisoning, alpha thalassemia
trait, hemoglobin H disease, chronic inflammation, hemoglobinopathy, and sideroblastic anemia. Failure
to respond to a course of iron supplementation with an increase in reticulocyte count and hemoglobin, as
described for this boy, should lead to consideration of these other conditions. Further, the red cell
distribution width (RDW) for iron deficiency anemia is typically elevated (>14.5%), and the narrow RDW in
this patient raises questions about the diagnosis. Calculation of the Mentzer index (MCV/red blood cell
count) results in a value of 11.4 (66/5.8), and a Mentzer index of less than 13 suggests beta thalassemia
trait. To confirm the diagnosis, hemoglobin electrophoresis should be performed to document an increase
in the percent of hemoglobin A2.
The Coombs test is important in the evaluation of hemolytic anemia, but the boy in the vignette
does not have the signs expected with that condition (eg, jaundice, splenomegaly, pallor), and the
presence of microcytosis is not typical of hemolysis. Bone marrow aspiration is reserved for instances
when there is a concern for neoplastic disease or diagnostic uncertainty in a more severely ill patient.
Transfusion is rarely needed for iron deficiency anemia and is not indicated for thalassemia trait; patients
who have beta thalassemia major depend on chronic red blood cell transfusions but have physical and
laboratory findings markedly more severe than those reported for this boy. Repeating the course of iron
treatment is often necessary for iron deficiency anemia because nonadherence to the treatment regimen
is frequent. However, in the presence of a Mentzer index of less than 13, further diagnostic evaluation
should be undertaken before assuming that nonadherence is the cause of this childs failure to respond to
treatment.

2American Board of Pediatrics Content Specification(s):


Know that iron deficiency and thalassemia minor are the most common causes of a microcytic
anemia

Suggested Reading:
Jain S, Kamat D. Evaluation of microcytic anemia. Clin Pediatr (Phila). 2009;48:7-13. Abstract available
at: http://www.ncbi.nlm.nih.gov/pubmed/18832550

Janus J, Moerschel SK. Evaluation of anemia in children. Am Fam Physician. 2010;81:1462-1471.


Abstract available at: http://www.ncbi.nlm.nih.gov/pubmed/20540485

Richardson M. Microcytic anemia. Pediatr Rev. 2007;28:5-13. DOI: 10.1542/pir.28-1-5. Available at:
http://pedsinreview.aappublications.org/cgi/content/full/28/1/5

Copyright 2012 American Academy of Pediatrics


2012 PREP SA ON CD-ROM

Question 91
A 17-year-old boy presents for a preparticipation sports physical examination for high school
varsity football. He denies sexual activity, but examination of his genital area reveals marks consistent
with excoriation due to scratching. You also note debris on the skin under the pubic hair and attached to
the pubic hair.
Of the following, the MOST appropriate initial treatment for this condition is
A. oral ivermectin
B. oral trimethoprim-sulfamethoxazole
C. topical lindane
D. topical malathion
E. topical permethrin

Copyright 2012 American Academy of Pediatrics


2012 PREP SA ON CD-ROM

Critique 91 Preferred Response: E


The evidence of scratching and debris on the skin and pubic hair of the young man described in
the vignette indicate an infestation of the pubic louse, Phthirus pubis (Item C91), colloquially referred to
as crabs or crab louse. It is almost always transmitted by sexual contact and not by household objects
such as towels. The louse feeds on human blood and itching is a common presentation. Incubation is
usually 6 to 10 days before hatching from eggs. Hatchlings become adults in 2 to 3 weeks.
Treatment includes good hygiene, treatment of contacts to avoid reinfestation, and use of a
pediculicide. Most of the pediculicides are the same as those employed in eradication of head lice.
Permethrin 1% and pyrethrins with piperonyl butoxide are topical pediculicides recommended by the
Centers for Disease Control and Prevention (CDC) for treatment of pediculosis pubis. Both are
inexpensive and available without a prescription. Repeat treatment in 7 to 10 days often is recommended
because neither agent is uniformly ovicidal. Alternative therapies endorsed by the CDC (but not approved
by the United States Food and Drug Administration [FDA] for this purpose) include topical malathion
0.05% (applied for 8 to 12 hours) and oral ivermectin. Lindane is not recommended due to its potential
neurotoxicity, particularly if it is inadvertently ingested or applied excessively. Use of trimethoprim-
sulfamethoxazole may be effective but is not approved by the FDA for this purpose.
Pubic lice also may infest the eyelashes and should be considered evidence for possible sexual
abuse in prepubertal children. Treatment involves the application of an ophthalmic ointment (eg,
erythromycin) to the lid margins twice daily for 10 days. Clearly, patients who have infestations of pubic
lice should be evaluated for other sexually transmitted infections.

American Board of Pediatrics Content Specification(s):


Recognize the signs and symptoms of pediculosis pubis
Know the treatment regimens for pediculosis pubis
Recognize the life cycle of human lice

Suggested Reading:
American Academy of Pediatrics. Pediculosis pubis (pubic lice, crab lice). In: Pickering LK, Baker CJ,
Kimberlin DW, Long SS, eds. Red Book: 2009 Report of the Committee on Infectious Diseases. 28th ed.
Elk Grove Village, IL: American Academy of Pediatrics; 2009:499

Centers for Disease Control and Prevention. Sexually transmitted diseases treatment guidelines, 2010.
MMWR Recomm Rep. 2010;59(No. RR-12):1-110. Available at:
http://www.cdc.gov/std/treatment/2010/default.htm

Frankowski BL, Bocchini JA Jr; Council on School Health and Committee on Infectious Diseases. Head
lice. Pediatrics. 2010;126:392-403. DOI:10.1542/peds.2010-1308. Available at:
http://pediatrics.aappublications.org/cgi/content/full/126/2/392

Copyright 2012 American Academy of Pediatrics


2012 PREP SA ON CD-ROM

Critique 91

(Reprinted with permission. Courtesy of Majid Shahdi, MD, DermAtlas; www.DermAtlas.org)


The pubic louse, Phthirus pubis.

Copyright 2011 American Academy of Pediatrics


2012 PREP SA ON CD-ROM

Question 92
You are called to the emergency department to assess a 7-day-old infant who has presented with
poor perfusion and respiratory collapse. He has been well-appearing but slightly dusky since birth. Pulse
oximetry reveals an oxygen saturation of 90% in the right arm but 55% in the left leg. His blood pressures
are symmetric but diffusely reduced. Although he has no obvious cardiac murmur, he does exhibit a
gallop rhythm, his liver is enlarged, and his extremities are cool, with reduced pulses.
Of the following, the MOST likely diagnosis for this infant is
A. coarctation of the aorta
B. dilated cardiomyopathy due to viral myocarditis
C. hypertrophic cardiomyopathy due to gestational diabetes
D. hypoplastic left heart syndrome
E. transposition of the great arteries

Copyright 2012 American Academy of Pediatrics


2012 PREP SA ON CD-ROM

Critique 92 Preferred Response: D


Hypoplastic left heart syndrome (HLHS) occurs in approximately 2 in 1,000 live births and
represents approximately 3% of congenital heart disease. The spectrum of HLHS ranges from hypoplasia
of the mitral or aortic valves to valvar atresia. In addition, there is virtually always coarctation of the aorta,
and the left ventricle is markedly hypoplastic and dysfunctional. Although HLHS can often be detected
prenatally through fetal echocardiography, most cases are still unsuspected at birth. Affected infants may
initially appear well; the only clinical finding may be cyanosis, which may be overlooked. There may be a
soft murmur or absence of a murmur because the blood flow typically is not turbulent. When the ductus
arteriosus begins to close spontaneously, tachypnea and respiratory distress develop, with mottling,
pallor, hypotension, weak pulses, cool extremities, and eventually acidosis and shock due to systemic
hypoperfusion, as described for the infant in the vignette. Because the ductus arteriosus must supply
circulation to the area distal to the area of aortic arch hypoplasia, the oxygen saturation is reduced in the
lower body due to the obligate ductal right-to-left shunt of desaturated right heart blood. Management of
the neonate suspected of having shock due to cyanotic congenital heart disease consists of the
administration of prostaglandin E1 to maintain patency of the ductus arteriosus. Other congenital heart
lesions that can produce neonatal shock include aortic stenosis and coarctation of the aorta. Neonatal
cardiomyopathy also can manifest as shock, as can sepsis, catastrophic blood loss, and respiratory
failure.
As noted, coarctation of the aorta can result in neonatal shock, but the absence of a blood
pressure discrepancy in the child described in the vignette makes this diagnosis highly unlikely. Similarly,
dilated cardiomyopathy due to viral myocarditis can produce shock. However, this neonate is described
as having been well-appearing since birth and has highly discrepant oxygen saturations between the arm
and leg, findings that point toward a ductal-dependent lesion. Hypertrophic cardiomyopathy can cause
neonatal shock due to the presence of obstructive left ventricular outflow tract obstruction. This condition
is characterized by a high-pitched murmur but no oxygen saturation disparity between arm and leg.
Although transposition of the great vessels is a ductal-dependent lesion, it involves the left heart
supplying well-saturated blood flow to the pulmonary artery, ductus arteriosus, and lower body, while the
desaturated blood from the right heart travels to the aorta and upper body, thus resulting in lower
saturation in the upper extremities compared to the lower extremities.

American Board of Pediatrics Content Specification(s):


Know what important lesions are associated with the shock-like presentation in a newborn infant

Suggested Reading:
Dorfman AT, Marino BS, Wernovsky G, et al. Critical heart disease in the neonate: presentation and
outcome at a tertiary care center. Pediatr Crit Care Med. 2008;9:193-202. DOI:
10.1097/PCC.0b013e318166eda5. Abstract available at: http://www.ncbi.nlm.nih.gov/pubmed/18477933

Silberbach M, Hannon D. Presentation of congenital heart disease in the neonate and young infant.
Pediatr Rev. 2007;28:123-131. DOI: 10.1542/pir.28-4-123. Available at:
http://pedsinreview.aappublications.org/cgi/content/full/28/4/123

Copyright 2012 American Academy of Pediatrics


2012 PREP SA ON CD-ROM

Question 93
You note an upcoming health supervision visit appointment for a 10-year-old child who has
myelomeningocele complex. You are seeing him for the first time after your partner, who previously cared
for him, retired. In preparation for the visit, you review the childs medical history and some background
on this complex condition. The boy had an open spinal dysraphism repaired at birth. He also had
herniation of the cerebellar vermis (a Chiari II malformation) that required surgical decompression and
hydrocephalus that necessitated a ventriculoperitoneal shunt. The boy has been stable for 3 years, but
according to your reading, he is at risk for acute deterioration due to shunt malfunction or new problems
in the brainstem or upper or lower spinal cord.
Of the following, the finding that is MOST indicative of potential neurological deterioration in this
child is
A. double vision
B. facial weakness
C. gait dysfunction
D. impulsive behavior
E. unilateral headache

Copyright 2012 American Academy of Pediatrics


2012 PREP SA ON CD-ROM

Critique 93 Preferred Response: C


It is critical that parents and physicians caring for children who have spinal dysraphism be aware
of possible complications. Two particular complications warrant special vigilance in older children. First,
many children who have spina bifida have hydrocephalus and have ventriculoperitoneal shunts placed in
infancy. Shunt failure can cause encephalopathy, irritability, headache, and vomiting. Second, an
abnormal spinal cord may be tethered to deformed tissue or bone, and as the child grows, the cord may
stretch and become ischemic, leading to worsening distal function. Presenting signs of neurologic
decompensation due to cord tethering include worsening of gait, pain in the back or legs, worsening
bladder or bowel function, increasing spasticity, and increasing scoliosis. Worsening spinal cord
syringomyelia may also present with pain and motor dysfunction.
At the age of the child described in the vignette, complications of cord tethering would be the
most common problem and would be signaled with gait dysfunction. Difficulties with visual tracking can
occur in children who have spina bifida. Double vision could present with an acute decompensation due
to shunt failure, but other signs of elevated intracranial pressure should also be present. Facial weakness
due to frontal lobe, brainstem, or facial nerve palsy is not a complication of spina bifida.
Impulsive behavior and other cognitive or behavioral problems may occur in children who have
hydrocephalus, but such problems are usually chronic and not indicative, in isolation, of acute neurologic
deterioration. Nonetheless, parent reports of significant changes in behavior should be considered
carefully. Behavioral or cognitive changes could indicate subtle increases in intracranial pressure.
Unilateral headache is typical of migraine, not of headache due to elevated intracranial pressure.

American Board of Pediatrics Content Specification(s):


Know the differential diagnosis of acute neurologic deterioration in a child with myelomeningocele

Suggested Reading:
Kinsman SL, Johnston MV. Congenital anomalies of the central nervous system. In: Kliegman RM,
Stanton BF, St. Geme JW III, Schor NF, and Behrman RE, eds. Nelson Textbook of Pediatrics. 19th ed.
Philadelphia, PA: Saunders Elsevier; 2011:1998-2012

Sandler AD. Children with spina bifida: key clinical issues. Pediatr Clin North Am. 2010;57:879-892. DOI:
10.1016/j.pcl.2010.07.009. Available at: http://www.ncbi.nlm.nih.gov/pubmed/20883878

Copyright 2012 American Academy of Pediatrics


2012 PREP SA ON CD-ROM

Question 94
A 2-year-old boy in your practice has clinical signs of rickets; laboratory testing reveals a low
phosphorus, normal calcium, elevated alkaline phosphatase, low-normal 1,25-dihydroxyvitamin D3, and
high normal parathyroid hormone values. You refer him to a pediatric endocrinologist who suspects that
he has X-linked dominant hypophosphatemic rickets.
Of the following, the history that is MOST suggestive of the X-linked dominant form of
hypophosphatemic rickets is that
A. his father has significant short stature, genu varum, and recurrent dental abscesses
B. his mother and maternal aunt have mild short stature and his maternal grandfather has severe
genu varum
C. his parents are second cousins (their grandfathers are brothers)
D. neither parent has signs of rickets, but his mothers brother and one of her maternal uncles were
diagnosed with rickets
E. the number of affected males in the extended family history is approximately twice that of
affected females

Copyright 2012 American Academy of Pediatrics


2012 PREP SA ON CD-ROM

Critique 94 Preferred Response: B


Hypophosphatemic rickets is a relatively common condition that can be transmitted in an X-linked
dominant, an autosomal dominant, or rarely an autosomal recessive pattern. A family history most
suggestive of X-linked dominant inheritance includes mild short stature in the boys mother and aunt, with
more severe symptoms in his maternal grandfather (Item C94). As with a few other X-linked dominant
conditions, X-linked dominant hypophosphatemic rickets (XLDHR) has variable phenotypic features, and
affected men tend to have more severe findings than affected women. An affected father could not pass
on this X-linked condition to his son because he contributes his Y and not his X chromosome to a boy at
conception. An affected father and son would be suggestive of autosomal dominant transmission.
Consanguinity (parental relatedness) would increase concerns about an autosomal recessive form of
hypophosphatemic rickets but would not be seen with XLDHR. Affected males related through clinically
unaffected or normal females are suggestive of X-linked recessive inheritance, which has not been
described for hypophosphatemic rickets. In reviewing extensive or large pedigrees with X-linked dominant
traits, the number of affected females is generally twice that of affected males because mothers can have
both affected sons and daughters, and affected males can only have affected daughters.
XLDHR is typically associated with short stature; frontal bossing; genu varum; recurrent dental
abscesses; and low phosphate (due to renal phosphate wasting), elevated alkaline phosphatase, normal
serum calcium, and normal to low-normal serum 1,25-dihydroxyvitamin D concentrations. It is caused by
defects in the phosphate-regulating endopeptidase homolog, X-linked gene (PHEX), and treatment
consists of oral phosphate supplements and supplemental 1,25 dihydroxy vitamin D. Serum calcium,
phosphate, and alkaline phosphate should be monitored over time because some children receiving 1,25-
dihydroxyvitamin D develop hypercalcemia or nephrocalcinosis. Recent studies have suggested that
prepubertal use of human growth hormone for patients who have XLDHR may improve final adult height,
but concerns remain about disproportionate truncal growth compared with long bone growth. Orthopedic
care is also important to address leg bowing, and aggressive dental surveillance may be helpful in light of
the enamel and dentin defects identified in affected patients.

American Board of Pediatrics Content Specification(s):


Recognize the clinical features associated with an X-linked dominant disorder

Suggested Reading:
Carpenter TO, Mitnick MA, Ellison A, Smith C, Insogna K. Nocturnal hyperparathyroidism: a frequent
feature of X-linked hypophosphatemia. J Clin Endocr Metab. 1994;78:1378-1383. Abstract available at:
http://www.ncbi.nlm.nih.gov/pubmed/8200940

Gaucher C, Walrant-Debray O, Nguyen T-M, Esterle L, Garabdian M, Jehan F. PHEX analysis in 118
pedigrees reveals new genetic clues in hypophosphatemic rickets. Hum Genet. 2009;125:401-411. DOI:
10.1007/s00439-009-0631-z. Abstract available at: http://www.ncbi.nlm.nih.gov/pubmed/19219621

Haffner D, Nissel R, Whl E, Mehls O. Effects of growth hormone treatment on body proportions and final
height among children with X-linked hypophosphatemic rickets. Pediatrics. 2004;113:e593-e596.
Available at: http://pediatrics.aappublications.org/cgi/content/full/113/6/e593

Copyright 2012 American Academy of Pediatrics


2012 PREP SA ON CD-ROM

Critique 94

(Courtesy of A Johnson)
Pedigree of the boy in the vignette.

Copyright 2011 American Academy of Pediatrics


2012 PREP SA ON CD-ROM

Question 95
You are seeing a 17-year-old boy for the first time because of symptoms of burning on urination
and occasional staining of his underwear. On physical examination, you note scant clear fluid at his
urethral meatus but no other genital findings. He has had three sexual partners in the past 6 months and
uses condoms intermittently. You obtain a urine specimen to test for sexually transmitted infections. He
has no allergies to any medications. You are concerned that he may not return for the test results.
Of the following, the BEST choice for treatment for this boy is
A. ampicillin
B. ampicillin and doxycycline
C. benzathine penicillin and doxycycline
D. ceftriaxone and azithromycin
E. levofloxacin

Copyright 2012 American Academy of Pediatrics


2012 PREP SA ON CD-ROM

Critique 95 Preferred Response: D


The symptoms of urethral discharge and dysuria described for the boy in the vignette are the
result of inflammation of the urethral mucosa, which can have infectious or noninfectious causes.
Chlamydia trachomatis and Neisseria gonorrhoeae are the primary bacterial causes. Atypical organisms
include Mycoplasma genitalium and Ureaplasma urealyticum. Among the less common pathogens are
Trichomonas, adenovirus, herpes simplex virus, and human papillomavirus. All these infections can be
asymptomatic. Proper identification and treatment of the patient and partner are important to prevent
complications from further spread of the infection. Currently, the use of urine nucleic acid amplification
tests for C trachomatis and N gonorrhoeae has eliminated the need to collect urethral swabs.
Treatment ideally should await laboratory confirmation of an infectious agent because a specific
diagnosis helps with reporting to the health department and partner notification and improves compliance
with treatment, especially by the partner. However, empiric treatment is recommended in high-risk
patients who are unlikely to return for follow-up evaluations, such as this boy. Such empiric therapy
should cover the common pathogens N gonorrhoeae and C trachomatis and preferably be administered
in a single dose for each to improve compliance. If possible, therapy should be observed. A single dose of
250 mg ceftriaxone intramuscularly is the preferred treatment for N gonorrhoeae because it provides high
blood bactericidal concentrations and is effective for gonorrhea at all anatomic sites. The advantage of a
single 400-mg oral dose of cefixime is offset by its limited efficacy as compared to ceftriaxone. The
increasing prevalence of N gonorrhoeae strains resistant to penicillin and the fluoroquinolones is the
reason that ampicillin, penicillin, and levofloxacin are no longer recommended. A single dose of
azithromycin (1 g) is the recommended treatment for C trachomatis. Although doxycycline is effective
therapy for C trachomatis infections, it involves a 7-day course of therapy. The approach to urethritis in a
female is similar to that for a male.

American Board of Pediatrics Content Specification(s):


Know the treatment of urethritis

Suggested Reading:
Centers for Disease Control and Prevention. Sexually Transmitted Diseases Treatment Guidelines for
2010: Diseases characterized by urethritis and cervicitis. Available at:
http://www.cdc.gov/std/treatment/2010/STD-Treatment-2010-RR5912.pdf

Chandran L, Boykan R. Chlamydial infections in children and adolescents. Pediatr Rev. 2009;30:243-250.
DOI: 10.1542/pir.30-7-243. Available at: http://pedsinreview.aappublications.org/cgi/content/full/30/7/243

DAngelo LJ, Neinstein LS. Genitourinary tract disorders. In: Neinstein LS, Gordon CM, Katzman DK,
Rosen DS, Woods ER, eds. Adolescent Health Care: A Practical Guide. 5th ed. Philadelphia PA:
Lippincott Williams & Wilkins, a Wolters Kluwer business; 2008:379-385

Holder NA. Gonococcal infections. Pediatr Rev. 2008;29:228-234. DOI: 10.1542/pir.29-7-228. Available
at: http://pedsinreview.aappublications.org/cgi/content/full/29/7/228

Wang SA, Harvey AB, Conner SM, et al. Antimicrobial resistance for Neisseria gonorrhoeae in the United
States, 1988 to 2003: the spread of fluoroquinolone resistance. Ann Intern Med. 2007;147:81-88.
Available at: http://www.annals.org/content/147/2/81.full.pdf+html

Copyright 2012 American Academy of Pediatrics


2012 PREP SA ON CD-ROM

Question 96
You are called to the infusion center to evaluate a 15-year-old girl who just received a transfusion
of two units of packed red blood cells over 4 hours for anemia due to her underlying acute lymphocytic
leukemia. Before starting her transfusion, her temperature was 37.0C, heart rate was 80 beats/min,
respiratory rate was 16 breaths/min, blood pressure was 110/70 mm Hg, and oxygen saturation was 97%
in room air. As the transfusion was completed, she began to feel short of breath and developed chills,
headache, nausea, and vomiting. Currently, her temperature is 38.5C, heart rate is 100 beats/min,
respiratory rate is 26 breaths/min, blood pressure is 105/65 mm Hg, and oxygen saturation is 95% in
room air. You discuss the apparent transfusion reaction with the patient and her parents.
Of the following, the information you are MOST likely to share with the family is that
A. fevers are always associated with a hemolytic reaction
B. fevers are the most common reaction seen
C. leukocyte reduction filters can eliminate the risk of a reaction with future transfusions
D. the incidence is approximately 25%
E. washed blood cell products can eliminate the risk of a reaction with future transfusions

Copyright 2012 American Academy of Pediatrics


2012 PREP SA ON CD-ROM

Critique 96 Preferred Response: B


Blood products such as packed red blood cells (pRBCs), fresh frozen plasma, and platelets can
cause various early-onset reactions, including febrile nonhemolytic reactions, hemolytic reactions (with or
without fever), anaphylaxis, urticaria, and transfusion-related acute lung injury. Early-onset reactions are
defined as those seen during the transfusion or within 4 hours of its completion. In addition, blood product
transfusions can produce various late-onset complications, such as hemolytic reactions, infections, graft-
versus-host disease, and even death.
o
Fever (a rise of 1 C unexplained by other patient factors) is the most common transfusion
reaction, with an estimated incidence of 1% for pRBC and 10% for platelet transfusions. In general, the
fever is self-limiting and not of significant consequence unless associated with hemolysis or a bacterial
contamination. Febrile reactions are often accompanied by dyspnea, rigors, tachycardia, headache, and
nausea and vomiting. Febrile nonhemolytic transfusion reactions are generally believed to be due to
either cytokine presence in the blood product or recipient antibody reaction to donor white blood cells
during the transfusion.
All blood product transfusions have a risk of infectious complications, including bacterial
infections. The risk of fatal infection is estimated to be 1 in 40,000 for platelets and 1 in 500,000 for
pRBCs; the higher risk for platelets is due to the higher temperature at which they are stored. Viral
infections such as human immunodeficiency or hepatitis (A, B, or C) have become rare due to routine
testing, although other viruses such as cytomegalovirus or parvovirus remain of concern.
Judicious use of blood products is imperative because complications of transfusion can be
reduced but not eliminated by the use of measures such as irradiation, leukocyte reduction filters, and
component washing. In a landmark study, LaCroix and associates demonstrated a 96% reduction in the
number of patients who had transfusion exposure and a 44% decrease in number of transfusions when
comparing a group of critically ill but stable pediatric patients who were randomized to either a liberal
transfusion (hemoglobin threshold of 9.5 g/dL [95 g/L]) or restrictive transfusion (hemoglobin threshold of
7 g/dL [70 g/L]) strategy.

American Board of Pediatrics Content Specification(s):


Understand the risk of transmitting infectious diseases during blood transfusion(s)
Recognize that erythrocyte transfusions may be associated with hemolytic, febrile, and urticarial
reactions

Suggested Reading:
LaCroix J, Hbert PC, Hutchison JS, et al. Transfusion strategies for patients in the pediatric intensive
care unit. N Engl J Med. 2007;356:1609-1619. Available at:
http://www.nejm.org/doi/full/10.1056/NEJMoa066240#t=article

LaCroix J, Tucci M, Gauvin F, Toledano B, Hume H. Transfusion medicine. In: Wheeler DS, Wong HR,
Shanley TP, eds. Pediatric Critical Care Medicine: Basic Science and Clinical Evidence. New York, NY:
Springer-Verlag; 2007:1263-1280

Strauss RG. Blood component transfusions: risks of blood transfusions. In: Kliegman RM, Stanton BF, St.
Geme JW III, Schor NF, and Behrman RE, eds.Nelson Textbook of Pediatrics. 19th ed. Philadelphia, PA:
Saunders Elsevier; 2011:1693

Copyright 2012 American Academy of Pediatrics


2012 PREP SA ON CD-ROM

Question 97
A 9-year-old girl presents with a 2-month history of diarrhea and weight loss. A thorough review
of systems reveals that she has been having difficulty sleeping at night for the past month. On physical
examination, her heart rate is 95 beats/min, blood pressure is 121/85 mm Hg, weight is 22 kg, and height
is 132 cm. You palpate a firm enlarged thyroid gland without nodules. Laboratory studies reveal a free
thyroxine (FT4) value of 2.4 ng/dL (30.9 pmol/L) (normal, 0.9 to 1.6 ng/dL [11.6 to 20.6 pmol/L]) and
thyroid-stimulating hormone (TSH) value of less than 0.01 mIU/L (normal, 0.5 to 4.0 mIU/L).
Of the following, the next BEST step in the management of this patient is to
A. assess thyroid-stimulating immunoglobulins
B. perform I-123 uptake scan
C. repeat FT4 and TSH measurements in 1 week
D. start methimazole
E. start propylthiouracil

Copyright 2012 American Academy of Pediatrics


2012 PREP SA ON CD-ROM

Critique 97 Preferred Response: D


The girl described in the vignette has classic signs and symptoms of Graves disease
(autoimmune hyperthyroidism). Given her degree of tachycardia, hypertension, and laboratory evidence
of hyperthyroidism, she should be treated with antithyroid medications immediately. Methimazole is the
clear antithyroid medication of choice because the risk of potentially fatal liver toxicity with propylthiouracil
(PTU) has been well documented in recent years. PTU should not be used in the management of Graves
disease unless there is a strong contraindication to other medical or surgical therapy. Both methimazole
and PTU inhibit thyroid hormone biosynthesis by decreasing the oxidation of iodide and iodination of
tyrosine. Although PTU offers the additional theoretical advantage of diminishing peripheral conversion of
thyroxine into triiodothyronine, the risk of liver dysfunction outweighs the potential benefit. Methimazole
also has adverse effects that should be appreciated. Fewer than 10% of patients taking methimazole
develop a drug-induced rash during therapy. More importantly, granulocytopenia can be observed in as
many as 12% of patients, necessitating close and frequent monitoring of the white blood cell count during
therapy. Methimazole should be administered at a dose of 0.5 to 1 mg/kg per day divided twice a day and
can be given in conjunction with a long-acting beta-blocker to alleviate symptoms until the FT4 value is
normal. For patients who cannot tolerate antithyroid medication, alternate options for treating Graves
disease include thyroidectomy or I-131 ablation.
Measurement of thyroid-stimulating immunoglobulins for the girl in the vignette would not be
helpful in the management of her disease. When additional evidence is needed to confirm the diagnosis
(ie, results of physical examination and laboratory tests are not classic), assessment of antibodies or I-
123 scanning may be helpful. If hashitoxicosis (short-term hyperthyroidism sometimes seen in acute
destruction of thyroid tissue due to Hashimoto thyroiditis) is suspected, the presence of thyroid
peroxidase antibodies and absence of thyroid-stimulating immunoglobulins can help make the correct
diagnosis. An I-123 uptake scan could be helpful in differentiating between hashitoxicosis (patchy
uptake), exogenous thyroid intake (low uptake), or Graves disease (high uptake). Additional tests,
including a repeat measurement of her FT4 and TSH 1 week later, are not indicated.
Once therapy is initiated, FT4 should be measured regularly until values are within the reference
range. However, titrating methimazole doses to achieve normal thyroid function is very difficult.
Accordingly, many endocrinologists prefer to use a block and replace strategy when treating Graves
disease, in which methimazole is administered at an adequate dose to induce hypothyroidism (block)
and then thyroid hormone (replace) is provided to achieve euthyroidism. Typically, a 1- to 2-year trial of
methimazole is provided before the medication is weaned to determine if the patient has entered a
remission. Remission rates are reported to be between 20% and 50%. If patients fail withdrawal of
therapy, a second course of methimazole can be initiated or alternative therapies such as I-131 or
thyroidectomy can be considered.

American Board of Pediatrics Content Specification(s):


Recognize the signs and symptoms of hyperthyroidism

Suggested Reading:
Kaguelidou F, Carel JC, Lger J. Graves' disease in childhood: advances in management with antithyroid
drug therapy. Horm Res. 2009;71(6):310-317. DOI: 10.1159/000223414. Available at:
http://content.karger.com/produktedb/produkte.asp?typ=fulltext&file=000223414

Kokotos F, Adam HM. In brief: hyperthyroidism. Pediatr Rev. 2006;27:155-157. DOI: 10.1542/pir.27-4-
155. Available at: http://pedsinreview.aappublications.org/cgi/content/full/27/4/155

Rivkees SA, Mattison DR. Ending propylthiouracil-induced liver failure in children [letter]. N Engl J Med.
2009;360:1574-1575. Available at: http://www.nejm.org/doi/full/10.1056/NEJMc0809750

Copyright 2012 American Academy of Pediatrics


2012 PREP SA ON CD-ROM

Question 98
The mother a 10-year-old girl whom you have been seeing since birth calls you because of
concerns that her daughter might have depression. When they arrive at the office, one of your office
assistants asks both the mother and child to complete a Child Depression Inventory (CDI). The mothers
inventory scores are borderline significant and the childs scores are below the cutoff for depression.
When you interview the girl in her mothers presence, she admits to trouble sleeping, difficulty
concentrating on her schoolwork, and no longer enjoying school. Physical examination of the quiet but
cooperative girl yields normal findings.
Of the following, the next BEST step in care is to
A. administer the Patient Health Questionnaire-9 (PHQ-9)
B. interview the child alone regarding other possible symptoms of depression
C. prescribe a trial of fluoxetine therapy
D. reassure the mother that her child does not have depression based on results of the CDI
E. schedule a follow-up visit in about 1 month for reassessment

Copyright 2012 American Academy of Pediatrics


2012 PREP SA ON CD-ROM

Critique 98 Preferred Response: B


The girl described in the vignette exhibits some symptoms consistent with depression, but results
of her CDI are negative for this diagnosis. In this ambiguous situation, more history is needed to
determine if the child has depression. Mental health rating scales may assist with screening and
monitoring symptoms over time, but they cannot be used to exclude a diagnosis definitively. For example,
the child might want to minimize his or her symptoms or may not understand the questions. A clinical
depression might be missed unless specific symptoms are elicited in an interview. Interviewing the child
alone allows her to share symptoms that she may not want to reveal in her mothers presence.
The Patient Health Questionnaire-9 (PHQ-9) is a rating scale validated for use in adolescents and
adults, and it is not a substitute for a more detailed history from the child. Sending the girl home without a
further interview could miss the diagnosis. Initiating a trial of fluoxetine for this girl is not appropriate
because neither has a diagnosis been established nor would an apparent positive response to medicine
be proof of diagnosis.

AAP Mental Health Competency:


Know the limitations (in terms of sensitivity and specificity) of mental health rating scales

Suggested Readings:
Birmaher B, Brent D; AACAP Work Group on Quality Issues, Bernet W, et al. Practice parameter for the
assessment and treatment of children and adolescents with depressive disorders. J Am Acad Child
Adolesc Psychiatry. 2007: 46:1503-1526. Abstract available at:
http://www.ncbi.nlm.nih.gov/pubmed/18049300

Cheung AH, Zuckerbrot RA, Jensen PS, Ghalib K, Laraque D, Stein RE; the GLAD-PC Steering Group.
Guidelines for Adolescent Depression in Primary Care (GLAD-PC): II. Treatment and ongoing
management. Pediatrics. 2007;120:e1313-e1326. DOI: 10.1542/peds.2006-1395. Available at:
http://pediatrics.aappublications.org/cgi/content/full/120/5/e1313

Stancin T, Aylward GP. Assessment of development and behavior. In: Wolraich ML, Drotar DD, Dworkin
PH, Perrin EC, eds. Developmental-Behavioral Pediatrics: Evidence and Practice. Philadelphia, PA:
Mosby Elsevier; 2008:144-176

Stancin T, Perrin EC. Behavioral screening. In: Augustyn M, Zuckerman B, Caronna EB, eds. The
Zuckerman Parker Handbook of Developmental and Behavioral Pediatrics for Primary Care. 3rd ed.
Philadelphia, PA: Lippincott, Williams & Wilkins, a Wolters Kluwer business; 2011:44-47

Zuckerbrot RA, Cheung AH, Jensen PS, Stein RE, Laraque D; the GLAD-PC Steering Group. Guidelines
for Adolescent Depression in Primary Care (GLAD-PC): I. Identification, assessment, and initial
management. Pediatrics. 2007;120:e1299-e1311. DOI: 10.1542/peds.2007-1144. Available at:
http://pediatrics.aappublications.org/cgi/content/full/120/5/e1299

Copyright 2012 American Academy of Pediatrics


2012 PREP SA ON CD-ROM

Question 99
A young mother brings in her 12-month-old child for a health supervision visit. The child is
breastfeeding well and tolerating solids. She has a vocabulary of 5 words and is able to follow simple
directions. The grandmother, who also is in attendance, is concerned that the child is tongue-tied and
worries that she will not be able to speak correctly. On physical examination, you notice a short lingual
frenulum. The mother asks if her infant is all right.
Of the following, the BEST response is to
A. reassure the family that this is normal
B. recommend that the child begin to use a cup
C. refer the child for a frenulectomy
D. send the child for an otolaryngology consultation
E. send the child for speech evaluation

Copyright 2012 American Academy of Pediatrics


2012 PREP SA ON CD-ROM

Critique 99 Preferred Response: A


The child described in the vignette has a short lingual frenulum. Most children who have this
condition do not require any therapy. In the past, standard practice was to cut the frenulum at the time of
delivery because of the belief that a short lingual frenulum (tongue-tie) was associated with speech
abnormalities such as difficulty pronouncing certain sounds and lisping. The presence of a short lingual
frenulum is considered a partial ankyloglossia compared with fusion, which represents complete
ankyloglossia.
Indications for therapy of ankyloglossia remain controversial due to the lack of controlled trials.
Because the tip of the tongue normally grows until 4 years of age, the initial limited tongue mobility may
improve with age. Therefore, for young infants, such as this child, appropriate management is parental
education and reassurance. The breastfed infant should be monitored to ensure appropriate weight gain.
Because this breastfed child is gaining weight appropriately, there is no reason to recommend that the
mother switch to feeding from a cup. Frenulectomy is not advisable before age 4 years unless there is
complete fusion. There is no indication at this time for the child to be referred either to an otolaryngologist
or for speech evaluation.
Circumstances that may affect typical language development include hearing issues, family
history, and environmental factors. Therefore, any child who exhibits a delay in language or social
development should undergo a formal hearing evaluation. This is especially important for a child who has
a history of otitis media with effusion. Parents of young children should provide personal interaction and
avoid the use of television in the first postnatal years.

American Board of Pediatrics Content Specification(s):


Know which circumstances influence language development in a typically developing child
Know that most children with a short lingual frenulum require no therapy

Suggested Reading:
Agin MC. The late talker-when silence isnt golden. Contemp Pediatr. 2004;21(Nov):22-34. Available at:
http://www.modernmedicine.com/modernmedicine/article/articleDetail.jsp?id=132720&sk=&date=&pageID
=2

Levy P. In brief: tongue-tie, management of short sublingual frenulum. Pediatr Rev. 1995;16:345-346.
Available at: http://pedsinreview.aappublications.org/cgi/reprint/16/9/345

Copyright 2012 American Academy of Pediatrics


2012 PREP SA ON CD-ROM

Question 100
You are evaluating a newborn whose mother has known hepatitis C infection. The mother asks
when the baby should be tested to determine if the infection has been transmitted to her infant.
Of the following, the BEST course of action is
A. nucleic acid amplification testing (NAAT) for hepatitis C genome at 2 months of age
B. NAAT for hepatitis C genome before discharge from the nursery
C. serial liver enzyme testing
D. serologic testing for hepatitis C antibodies at 12 months of age
E. serologic testing for hepatitis C antibodies before discharge from the nursery

Copyright 2012 American Academy of Pediatrics


2012 PREP SA ON CD-ROM

Critique 100 Preferred Response: A


Hepatitis C virus (HCV) can be transmitted vertically from mother to infant, although not
commonly (~5%). Therefore, babies born to mothers who are HCV-positive, such as the infant described
in the vignette, should be tested for HCV infection. Nucleic acid amplification testing (NAAT) to detect
HCV RNA may be performed on a potentially affected infant at 1 to 2 months of age for reliable evidence
of viral transmission to the baby. The genome may not be detectable in the infants blood at birth or may
be falsely positive, reflecting maternal blood cell contamination. Maternal antibodies to HCV may persist
for up to 18 months after birth, making testing for antibodies at birth or even at 12 months of age
unreliable for detecting infection. Serial evaluations of liver function tests are not sufficiently sensitive or
specific for diagnosing HCV infection.
Other indications for HCV testing include individuals who have injected illicit drugs, received
clotting factors before 1987, received blood transfusion or organ transplantation before 1992, ever
received chronic hemodialysis, or have evidence of liver disease. In most of these instances, the
screening test used is enzyme-linked immunosorbent assay (EIA or ELISA) that detects HCV antibodies.
A recombinant immunoblot assay that is more specific (although not more sensitive) is used to confirm
those who have positive results with ELISA. Immunocompromised individuals, including those who have
human immunodeficiency virus infection, may not develop detectable anti-HCV antibody responses with
HCV infection. For these patients, NAAT is indicated to detect HCV RNA if HCV infection is suspected.

American Board of Pediatrics Content Specification(s):


Know the risk factors for acquiring hepatitis C infection: blood transfusion, IV drug abuse, multiple
sexual partners, homosexual activity, infant whose mother has hepatitis C
Know the appropriate diagnostic tests for hepatitis C infection (nucleic acid amplification, serology),
and the appropriate timing of these tests for exposed individuals

Suggested Reading:
Alter MJ, Kuhnert WL, Finelli L. Guidelines for laboratory testing and result reporting of antibody to
hepatitis C virus. Centers for Disease Control and Prevention. MMWR Recomm Rep. 2003;52(RR-3):1
15. Available at: http://www.cdc.gov/mmwr/preview/mmwrhtml/rr5203a1.htm

American Academy of Pediatrics. Hepatitis C. In: Pickering LK, Baker CJ, Kimberlin DW, Long SS, eds.
Red Book: 2009 Report of the Committee on Infectious Diseases. 28th ed. Elk Grove Village, IL:
American Academy of Pediatrics; 2009:357-360

Ghany MG, Strader DB, Thomas DL, Seeff LB; American Association of the Study of Liver Diseases.
Diagnosis, management, and treatment of hepatitis C: an update. Hepatology. 2009;49:1335-1374. DOI:
10.1002/hep.22759. Available at: http://onlinelibrary.wiley.com/doi/10.1002/hep.22759/pdf

Rich JD, Taylor LE. The beginning of a new era in understanding hepatitis C prevention. J Infect Dis.
2010;202:981983. DOI: 10.1086/656213. Available at:
http://jid.oxfordjournals.org/content/202/7/981.long

Copyright 2012 American Academy of Pediatrics


2012 PREP SA ON CD-ROM

Question 101
A pregnant woman has recently been exposed to varicella-zoster virus and is concerned about
the risk to her fetus if she develops varicella-zoster infection.
Of the following, the most accurate information to share with this mother is that
A. her infant is at high risk of death if the infection occurs during active labor at term
B. her infant is at highest risk of damage to the heart if the infection occurs between 13 to 20 weeks
gestation
C. most mothers infected in the first trimester experience fetal loss
D. transplacental transfer of antibody is ineffective in modifying the disease in the term newborn
E. ultrasonography can be used to exclude infection in the fetus after 20 weeks gestation

Copyright 2012 American Academy of Pediatrics


2012 PREP SA ON CD-ROM

Critique 101 Preferred Response: A


The mother described in the vignette has been exposed to varicella-zoster virus (VZV) but does
not yet know if she is infected. Because approximately 90% of women of child-bearing age have antibody
to VZV, varicella infection during pregnancy is uncommon (0.7 per 1,000 pregnancies). The timing of the
onset of varicella disease in the mother and infant determines the severity of illness and outcome for the
infant. Infants are at highest risk of death from neonatal varicella if the disease onset in the mother is from
5 days before to 2 days after delivery, presumably because passive (transplacental) transfer of maternal
antibody, which can modify the disease in the newborn, has not had time to occur. Therefore, the infant
described in the vignette is at high risk of death if the acute VZV infection occurs during active labor.
Congenital varicella syndrome, characterized by cutaneous scarring (often cicatricial), eye
abnormalities (including chorioretinitis), central nervous system (CNS) damage (eg, microcephaly, cortical
atrophy, intellectual disability), and limb hypoplasia, occurs in 1% to 2% of infants born to mothers who
have VZV infection occurring in the first 20 weeks of gestation. The damage is believed to be the result of
viral replication and destruction in the fetal ectodermal tissues. More than 90% of cases occur after
maternal chickenpox, not zoster. Cardiac malformations are not a feature of congenital varicella
syndrome. No apparent anomalies were present in infants born to mothers who had VZV infection during
pregnancy in several large prospective studies.
Only occasionally does fetal infection in the first and early second trimester result in intrauterine
demise. When mothers are infected with VZV in the second half of pregnancy, infants may be unaffected
or may have inapparent VZV infection, with subsequent zoster in early life. Because infants who acquire
VZV infection in utero have variable clinical manifestations, prenatal ultrasonography performed after 20
weeks gestation may not reveal limb or CNS anomalies. Viral culture or polymerase chain reaction assay
for VZV performed on amniotic fluid may establish the diagnosis prenatally. In addition, a fetal blood
specimen obtained via cordocentesis can be tested for anti-VZV immunoglobulin M. However, in most
cases, diagnosis of the syndrome is clinical and based on recognition of the characteristic physical
examination findings in the infant whose mother has a history of VZV infection.

American Board of Pediatrics Content Specification(s):


Know the manifestations of varicella infections acquired in utero

Suggested Reading:
American Academy of Pediatrics. Varicella-zoster infections. In: Pickering LK, Baker CJ, Kimberlin DW,
Long SS, eds. Red Book: 2009 Report of the Committee on Infectious Diseases. 28th ed. Elk Grove
Village, IL: American Academy of Pediatrics; 2009:714-727

Snchez PJ, Demmler-Harrison GJ. Viral infections of the fetus and neonate. In: Feigin RD, Cherry JD,
Demmler-Harrison GJ, Kaplan SL, eds. Feigin & Cherrys Textbook of Pediatric Infectious Diseases. 6th
ed. Philadelphia, PA: Saunders Elsevier; 2009:895-940

Copyright 2012 American Academy of Pediatrics


2012 PREP SA ON CD-ROM

Question 102
You are examining a 1-day-old term male infant during rounds in the newborn nursery and
palpate a right upper quadrant abdominal mass. The remainder of the examination findings are normal.
The infants mother is an otherwise healthy 22-year-old gravida 1, para 1, woman who received no
prenatal care. You note on the chart that the infant voided at 6 hours of age.
Of the following, the MOST likely diagnosis in this infant is
A. posterior urethral valves
B. ureterocele
C. ureteropelvic junction obstruction
D. urolithiasis
E. Wilms tumor

Copyright 2012 American Academy of Pediatrics


2012 PREP SA ON CD-ROM

Critique 102 Preferred Response: C


The differential diagnosis of an infant who has a palpable abdominal mass begins with underlying
renal causes. The most common causes of abdominal masses in the newborn period are multicystic
dysplastic kidney (MCDK) and hydronephrosis due to ureteropelvic junction (UPJ) obstruction, which
account for a combined 40% of all causes. Other, less common renal causes include polycystic kidney
disease, renal vein thrombosis, and renal tumors (such as mesoblastic nephroma). Among the nonrenal
causes of abdominal masses are gastrointestinal anomalies (duplications or obstruction) and ovarian
cysts.
The abdominal mass described for the infant in the vignette most likely is due to UPJ obstruction
(although MCDK would be equally likely). Posterior urethral valves are less common and should have
bilateral involvement. Ureterocele could be unilateral but is less common. Urolithiasis can cause unilateral
hydronephrosis but is extremely unlikely in the newborn. Wilms tumor is rare in newborns; the most
common newborn renal/adrenal tumor is mesoblastic nephroma, followed by neuroblastoma or a
teratoma.
Hydronephrosis often is related to obstruction or vesicoureteral reflux. Causes of obstruction
include UPJ obstruction, ureterovesical junction obstruction, ureterocele, bladder outlet obstruction from
posterior urethral valves, or functional obstruction from a neurogenic bladder. The most common cause of
hydronephrosis is UPJ obstruction, which is usually unilateral and presents as hydronephrosis in utero or
a palpable abdominal mass in the nursery. It is customary to perform voiding cystourethrography (VCUG)
in patients who have unilateral involvement to look for possible reflux into the contralateral (seemingly
uninvolved) side. Prophylactic antibiotics are recommended in cases of severe hydronephrosis because
urinary stasis can predispose to urinary tract infections. A radionuclide test (MAG 3) with furosemide is
recommended to measure the half-life of the radioisotope within the collecting system and determine the
differential contribution to total renal function (eg, 60% contribution from uninvolved side and 40% from
involved side). Delayed clearance from the upper urinary tract is consistent with UPJ obstruction (Item
C102). Referral to a pediatric urologist is recommended to determine the potential need for surgical
intervention. Typically, patients who have mild involvement are managed conservatively, and those
whose involved kidney is contributing less than 35% to total renal function are treated surgically with a
pyeloplasty. Of note, male infants who have bilateral hydronephrosis warrant urgent VCUG to rule out
posterior urethral valves that require more aggressive intervention.
MCDK is a nonfunctioning kidney, which occurs in an estimated 1 in 2,500 births. Bilateral
involvement is incompatible with life. Ultrasonography shows multiple cysts of varying size that do not
communicate and a small amount of abnormal renal parenchyma adjacent to the cysts. Distinguishing
MCDK from severe hydronephrosis can be a challenge. Unlike severe hydronephrosis, the cystic lesions
do not communicate and are not medially located in MCDK. In addition to the absence of renal function
on the involved side of the MCDK, the contralateral kidney may be affected with vesicoureteral reflux in
30% of cases and UPJ obstruction in 15% of cases.
The natural history of MCDK is spontaneous involution of the involved side in approximately 60%
of cases by 6 years of age. There is a very small likelihood of malignant potential in MCDK, including
Wilms tumor in younger children and renal cell carcinoma in adolescents. Due to the high risk of
vesicoureteral reflux, when MCDK is diagnosed in the neonatal period, prophylactic antibiotics should be
started (10 mg/kg per day of amoxicillin) to protect the contralateral, single functional kidney from
pyelonephritis. Due to the malignant potential in the involved kidney, it is standard to obtain
ultrasonography every 3 to 6 months during the first postnatal year, then every 6 to 12 months until age 5.
This strategy allows monitoring of the growth of the normal kidney, which often exhibits compensatory
hypertrophy (because it is performing the work of two kidneys), as well as the MCDK for signs of
involution or unexpected growth. In the event of unexpected growth, referral to a pediatric urologist for
consideration of a nephrectomy of the MCDK is recommended to eliminate concerns of malignant
transformation.

American Board of Pediatrics Content Specification(s):


Know the differential diagnosis of urinary tract obstruction

Suggested Reading:

Copyright 2012 American Academy of Pediatrics


2012 PREP SA ON CD-ROM

Chevalier RL. Obstructive uropathy. In: Kher KK, Schnaper HW, Makker SP, eds. Clinical Pediatric
Nephrology. 2nd ed. London, England: Informa Healthcare; 2007:507-518

Greenbaum LA. Avner ED. Cystic kidney disease. In: Kher KK, Schnaper HW, Makker SP, eds. Clinical
Pediatric Nephrology. 2nd ed. London, England: Informa Healthcare; 2007:261-273

Mesrobian HG. Urologic problems of the neonate: an update. Clin Perinatol. 2007;34:667-679. DOI:
10.1016/j.clp.2007.09.004. Abstract available at: http://www.ncbi.nlm.nih.gov/pubmed/18063112

Schwartz MZ, Shaul DB. Abdominal masses in the newborn. Pediatr Rev. 1989;11:172-179. Abstract
available at: http://pedsinreview.aappublications.org/cgi/content/abstract/11/6/172

Copyright 2012 American Academy of Pediatrics


2012 PREP SA ON CD-ROM

Critique 102

(Courtesy of R Valentini)
99
Tc MAG-3 diuretic renal scan (posterior view) in ureteropelvic junction obstruction. Prediuretic images
(yellow arrow) show symmetric uptake of the radionuclide in both kidneys. In the postdiuretic images (one
shown by red arrow) the radionuclide is clearing from the right kidney and is collecting in the bladder.
However, it is retained in the left kidney as a result of obstruction.

Copyright 2011 American Academy of Pediatrics


2012 PREP SA ON CD-ROM

Question 103
A 14-year-old boy who has seasonal allergic rhinitis and moderate persistent asthma is currently
receiving allergen immunotherapy. Today in the clinic, he received his usual allergen injection, but after
10 minutes, he started coughing and complaining of dyspnea and throat swelling. On physical
examination, he exhibits moderate respiratory distress and has diffuse expiratory wheezing on
auscultation. His oropharynx appears normal and without tongue or uvula edema. Vital signs include a
room air pulse oximetry reading of 97%, blood pressure of 130/70 mm Hg, and heart rate of 90
beats/minute.
Of the following, the MOST appropriate next action is to administer
A. a short-acting beta-2 agonist nebulization
B. an oral antihistamine
C. an oral corticosteroid
D. intramuscular epinephrine
E. supplemental oxygen via face mask

Copyright 2012 American Academy of Pediatrics


2012 PREP SA ON CD-ROM

Critique 103 Preferred Response: D


Allergen immunotherapy (AIT) (allergy shots) is currently used for the treatment of allergic
asthma, allergic rhinitis, allergic conjunctivitis, and stinging insect hypersensitivity. As described for the
boy in the vignette, anaphylaxis is a risk during AIT. A mandatory waiting time of 30 minutes after
administration, prompt recognition of symptoms, and prompt administration of intramuscular epinephrine
are key steps to preventing the rare outcome of fatal anaphylaxis. Other treatment should include
supplemental oxygen, oral or intramuscular antihistamine, oral corticosteroids, and nebulization therapy
with a short-acting beta-2 agonist, although these are secondary steps after epinephrine.
Most patients who have allergic rhinitis can be managed with avoidance and medications, but AIT
has some unique advantages. Once a patient completes the initial build-up phase (eg, once- to twice-
weekly visits for 3 to 6 months), therapy can be spaced to every 2 to 4 weeks. Patients who have
seasonal allergic rhinitis and reach this maintenance phase typically report a marked decrease in
symptoms and medication use. Further, AIT has been demonstrated to decrease new allergen
sensitization and the development of asthma in children. AIT is approved for children, although it
generally is not administered to those younger than 5 years of age because of both adherence issues and
the need to recognize symptoms of anaphylaxis. Patients who experience mild anaphylaxis may continue
to receive AIT, but the subsequent dose is reduced. Fatal anaphylaxis has been associated with patients
who experience more than three anaphylactic reactions to AIT. Other drawbacks to AIT include requiring
patients to receive therapy in a physicians office, remaining in the office for at least 30 minutes,
discomfort from a subcutaneous injection, and local erythema and swelling at the injection site. Local
reactions do not portend more severe reactions on subsequent injections, but they can reduce adherence
with AIT if large or bothersome.

American Board of Pediatrics Content Specification(s):


Know that immunotherapy is most effective in treating allergic rhinitis
Recognize and manage the side effects of immunotherapy injections

Suggested Reading:
Joint Task Force on Practice Parameters; American Academy of Allergy, Asthma and Immunology;
American College of Allergy, Asthma and Immunology; Joint Council of Allergy, Asthma and Immunology.
Allergen immunotherapy: a practice parameter second update. J Allergy Clin Immunol. 2007;120(3
suppl):S25-S85. DOI: 10.1016/j.jaci.2007.06.019. Available at: http://www.jacionline.org/article/S0091-
6749(07)01202-X/fulltext

Wallace DV, Dykewicz MS, Bernstein DI, et al; Joint Task Force on Practice, American Academy of
Allergy, Asthma & Immunology; American College of Allergy, Asthma and Immunology; Joint Council of
Allergy, Asthma and Immunology. The diagnosis and management of rhinitis: an updated practice
parameter. J Allergy Clin Immunol. 2008;122(2 suppl):S1-S84. DOI: 10.1016/j.jaci.2008.06.003. Available
at: http://www.jacionline.org/article/S0091-6749(08)01123-8/fulltext

Copyright 2012 American Academy of Pediatrics


2012 PREP SA ON CD-ROM

Question 104
A 14-year-old boy is brought to the emergency department by emergency medical services after
he fell 30 feet out of a tree. At the scene, he was unconscious. The paramedics immobilized his cervical
spine, endotracheally intubated him, began mechanical ventilation with 100% oxygen, and inserted two
intravenous catheters. On arrival at the emergency department, his temperature is 36.5C, heart rate is
140 beats/min, blood pressure is 80/60 mm Hg, and oxygen saturation is 100%, and he is being
ventilated at 15 breaths/min. Physical examination reveals a Glasgow Coma Scale score of 5 and a right
femur deformity. His abdomen is nondistended and his breath sounds are equal bilaterally. Following
rapid infusion of two 20-mL/kg 0.9% saline fluid boluses, his heart rate is 70 beats/min and his blood
pressure is 76/45 mm Hg.
Of the following, the MOST likely explanation for the boys persistent hypotension is
A. blood loss from the femur fracture
B. epidural hematoma
C. hemothorax
D. liver laceration
E. spinal cord injury

Copyright 2012 American Academy of Pediatrics


2012 PREP SA ON CD-ROM

Critique 104 Preferred Response: E


The most common cause of hypotension in an injured patient is hemorrhagic hypovolemia, as
might be caused by a hemothorax, severe femur fracture, or a liver laceration. The absence of
tachycardia, as described for the boy in the vignette, makes these causes unlikely. Bradycardia could be
due to imminent cerebral herniation from an epidural hematoma, but herniation is more commonly
associated with hypertension than hypotension.
In a pediatric trauma patient, persistent hypotension unresponsive to fluid administration in
association with relative bradycardia should raise concern for spinal cord injury and neurogenic shock.
Neurogenic or spinal shock occurs when autonomic pathways below T6 are disrupted, causing
peripheral vasodilation and inhibition of the appropriate compensatory tachycardic response. Neurogenic
shock should be considered in an injured patient who has hypotension and bradycardia after a careful
examination eliminates blood loss as the cause for hypotension.
Initial evaluation of a patient who has a suspected spinal injury should proceed systematically
with the patient in protective spinal immobilization. Appropriate spinal immobilization involves placing the
patient in a rigid cervical collar supine on a long backboard, with lateral supports on either side of the
head and five-point restraints. The primary (assessment of vital functions) and secondary (head-to-
toe/front-to-back) surveys should be performed to identify any injuries and treat life-threatening conditions
such as airway obstruction, respiratory failure, and hypotension. Patients who have high spinal cord
lesions (above T5) may require intubation and mechanical ventilation to address respiratory dysfunction
from thoracic and diaphragmatic motor weakness. Treatment of hypotension in the spinal cord-injured
patient may require vasopressor infusions in addition to fluid resuscitation. The early use of high-dose
corticosteroids for the prevention of secondary injury effects is controversial.
Spinal cord injuries may be complete or partial. Complete injuries are associated with total loss of
motor and sensory functions below the level of injury. Partial injuries are associated with variable
neurologic findings related to the location of the disruption (Item C104A). A complete neurologic
examination, including motor (Item C104B), reflex, and full sensory testing, is necessary to define the
level of injury fully. Initial radiographic evaluation should be performed with plain films. Lateral,
anteroposterior, and odontoid views of the cervical spine and lateral/anteroposterior views of the thoracic
and lumbosacral spine should be obtained, with computed tomography scan and magnetic resonance
imaging ordered as indicated by physical findings.

American Board of Pediatrics Content Specification(s):


Evaluate and stabilize a patient with a possible spinal cord injury
Know the signs and symptoms of spinal trauma

Suggested Reading:
Avarello JT, Cantor RM. Pediatric major trauma: an approach to evaluation and management. Emerg
Med Clin North Am. 2007;25:803836. DOI: 10.1016/j.emc.2007.06.013. Abstract available at:
http://www.ncbi.nlm.nih.gov/pubmed/17826219

Bracken MB. Steroids for acute spinal cord injury. Cochrane Database Syst Rev. 2002;2:CD001046. DOI:
10.1002/14651858.CD001046. Available at:
http://onlinelibrary.wiley.com/o/cochrane/clsysrev/articles/CD001046/frame.html

Schreiber D. Spinal cord injuries. eMedicine Specialties, Emergency Medicine, Neurology. 2009.
Available at: http://emedicine.medscape.com/article/793582-overview

Wheeless CR III. Management of the spine injured patient. In: Wheeless Textbook of Orthopaedics.
2010. Available at: http://www.wheelessonline.com/ortho/management_of_the_spine_injured_patient

Copyright 2012 American Academy of Pediatrics


2012 PREP SA ON CD-ROM

Critique 104

Item C104A. Partial Spinal Cord Injury Syndromes


Partial Cord Lesion Physical Findings
Anterior cord syndrome Loss of motor function, pinprick, or temperature
sensation, with ipsilateral preservation of proprioception
Central cord syndrome Upper extremity motor weakness, distal > proximal,
variable sensory loss, pain or temperature >
proprioception or vibration
Brown-Sequard Loss of motor function and proprioception, with
syndrome contralateral loss of pain and temperature sensation
Conus medullaris Sacral cord injury characterized by areflexia in the
bladder and bowel, with variable motor and sensory loss
in the lower limbs
Spinal cord concussion Transient motor/sensory deficits localized to the spinal
cord that fully recover without any apparent structural
damage

Copyright 2011 American Academy of Pediatrics


2012 PREP SA ON CD-ROM

Critique 104

Item C104B. Motor Testing in Spinal Cord Injury


Muscle Group Spinal Cord Level
Biceps C5
Wrist extensors C6
Triceps C7
Finger extensors C8
Finger abductors T1
Hip flexors L2
Knee extensors L3
Ankle dorsiflexors L4
Great toe extensor L5
Ankle plantar flexor S1

Copyright 2011 American Academy of Pediatrics


2012 PREP SA ON CD-ROM

Question 105
A 14-year-old girl presents with painless rectal bleeding. The well-appearing childs height and
weight are both between the 10th and 25th percentiles. A digital rectal examination demonstrates normal
anatomy, and stool is positive for occult blood. Upon further questioning, the girls father states that he
also experienced rectal bleeding as a child and had a few polyps removed at age 12 years. At that time,
his parents were told that the polyps were benign and that no further follow-up was required. He has been
asymptomatic since then. The girls initial laboratory study results show a hemoglobin of 12.5 g/dL (125
g/L), an albumin of 3.8 g/dL (38 g/L), and a C-reactive protein of less than 1.0 mg/dL. You refer the child
for colonoscopy. Several pedunculated polyps (Item Q105A) in the descending colon are identified at
colonoscopy and removed by electrocautery. Histologic findings (Item Q105B) are typical for a juvenile
(inflammatory) polyp.
Of the following, the girl MOST likely will require
A. colonoscopy annually
B. colonoscopy every 5 years until age 45
C. perinuclear antineutrophil cytoplasmic antibody (pANCA) assay
D. phosphatase and tensin homolog (PTEN) mutation analysis
E. stool occult blood testing annually

Copyright 2012 American Academy of Pediatrics


2012 PREP SA ON CD-ROM

Question 105

(Courtesy of A Bousvaros)
Polyp, as observed during colonoscopy.

Copyright 2012 American Academy of Pediatrics


2012 PREP SA ON CD-ROM

Question 105

(Courtesy of S Schwarz)
Histologic section of a typical juvenile polyp demonstrating dilated, mucus-filled glands (arrows) and an
intense inflammatory infiltrate. There is no smooth muscle proliferation.

Copyright 2012 American Academy of Pediatrics


2012 PREP SA ON CD-ROM

Critique 105 Preferred Response: A


Rectal bleeding in infants, children, and adolescents has myriad causes, but the correct diagnosis
may often be predicted on the basis of age at presentation, presence of concurrent symptoms, and family
history. The girl described in the vignette most likely has juvenile polyposis syndrome (JPS) and will
require colonoscopy annually.
JPS is an autosomal dominant condition that usually presents before the end of the second
decade with multiple polyps. The malignant potential of juvenile polyps in JPS (estimated at 9% to 50%)
is much below that associated with familial adenomatous polyposis (virtually 100%), but current
recommendations of the American Cancer Society call for yearly colonoscopic examinations. Patients
who have JPS have an increased lifetime risk of other gastrointestinal cancers, including pancreatic
cancer. Evaluation of JPS by phosphatase and tensin homolog (PTEN) mutation analysis or perinuclear
antineutrophil cytoplasmic antibody (pANCA) assay as well as monitoring via stool occult blood testing
are of little diagnostic or predictive value. Diagnostic criteria for JPS are:
Greater than five colonic polyps
Polyps throughout the gastrointestinal tract (juvenile polyps may be found in the stomach and small
bowel)
OR
Any number of polyps with a positive family history
The most common genetic determinants associated with JPS are mutations on chromosome loci
18q21.1 (MADH4) and 10q22.3 (BMPR1A). Juvenile polyps contain more than one cell type and, hence,
are described as hamartomatous; their occurrence has also been described with mutations in the PTEN
hamartoma gene. However, screening studies have been able to identify a specific, known mutation in
only 25% of patients.
This girl has no associated complaints such as abdominal pain, constipation, diarrhea, fever,
signs of other organ involvement, or systemic symptoms. Although colonic polyps may present with one
or more of such clinical signs, most patients are asymptomatic, and blood is generally described as bright
red and passed either mixed with or separate from stool. The typical patient who has a juvenile polyp is 2
to 5 years of age and has a solitary lesion in the rectosigmoid colon (although polyp location may be
anywhere throughout the colon). In the case of the isolated lesion, the malignant potential is extremely
low and routine surveillance is not required.

American Board of Pediatrics Content Specification(s):


Recognize the signs and symptoms of juvenile polyposis

Suggested Reading:
Gallione CJ, Repetto GM, Legius E, et al. A combined syndrome of juvenile polyposis and hereditary
haemorrhagic telangiectasia associated with mutations in MADH4 (SMAD4). Lancet. 2004;363:852859.
DOI:10.1016/S0140-6736(04)15732-2. Abstract available at:
http://www.ncbi.nlm.nih.gov/pubmed/15031030

Howe JR, Mitros FA, Summers RW. The risk of gastrointestinal carcinoma in familial juvenile polyposis.
Ann Surg Oncol. 1998;5:751756. Abstract available at: http://www.ncbi.nlm.nih.gov/pubmed/9869523

Howe JR, Sayed MG, Ahmed AF, et al. The prevalence of MADH4 and BMPR1A mutations in juvenile
polyposis and absence of BMPR2, BMPR1B, and ACVR1 mutations. J Med Genet. 2004;41:484491.
DOI: 10.1136/jmg.2004.018598. Available at:
http://www.ncbi.nlm.nih.gov/pmc/articles/PMC1735829/?tool=pubmed

Qualia CM, Brown MR, Leung AKC, et al. Index of suspicion: case 1. Pediatr Rev. 2007;28:193-198. DOI:
10.1542/pir.28-5-193. Available at: http://pedsinreview.aappublications.org/cgi/content/full/28/5/193

Zbuk KM, Eng C. Hamartomatous polyposis syndromes. Nat Clin Pract Gastroenterol Hepatol.
2007;4:492502. DOI: 10.1038/ncpgasthep0902. Available at:
http://www.nature.com/nrgastro/journal/v4/n9/full/ncpgasthep0902.html

Copyright 2012 American Academy of Pediatrics


2012 PREP SA ON CD-ROM

Question 106
You are called to the newborn nursery to evaluate a term newborn for abdominal distention. The
infant was born 12 hours ago by vaginal delivery. The pregnancy was complicated by gestational
diabetes and polyhydramnios. Spontaneous rupture of the membranes occurred 1 hour before delivery
with copious amniotic fluid that appeared to be lightly meconium-stained. The infant is breastfeeding well
and has passed one stool since birth. Her mother describes the infant as spitting up about a teaspoon of
green stuff twice in the past hour. Physical examination reveals an active, alert infant who has a
moderately distended abdomen that is not tender to palpation. The rectum appears patent. You obtain a
radiograph of the abdomen (Item Q106).
Of the following, the MOST appropriate next step is
A. abdominal computed tomography scan
B. abdominal ultrasonography
C. contrast enema
D. lateral abdominal radiograph with the infant prone
E. upper gastrointestinal radiographic series

Copyright 2012 American Academy of Pediatrics


2012 PREP SA ON CD-ROM

Question 106

(Courtesy of S Izatt)
Abdominal radiograph, as described for the infant in the vignette.

Copyright 2012 American Academy of Pediatrics


2012 PREP SA ON CD-ROM

Critique 106 Preferred Response: E


The most appropriate imaging to assess an infant who has bilious emesis and an abdominal
radiograph suggesting a proximal bowel obstruction, such as the newborn described in the vignette, is an
upper gastrointestinal radiographic series (UGI). The infant in the vignette has a radiograph that
demonstrates limited passage of air into the proximal small bowel (Item C106A). Bowel obstruction in the
newborn is an emergency that requires rapid diagnosis. Antenatal clues may include polyhydramnios and
bilious amniotic fluid. Careful attention to clinical findings can help distinguish between proximal and distal
bowel obstruction, leading to selection of optimal imaging studies and management.
Proximal bowel obstructions include duodenal atresia, jejunoileal atresia, and malrotation with
midgut volvulus. Infants who have duodenal atresia or jejunoileal atresia present with vomiting and
abdominal distension. The vomiting may be bilious if the obstruction is distal to the ampulla of Vater. The
abdominal radiograph of newborns who have duodenal atresia usually demonstrates the classic double-
bubble sign (Item C106B), whereas newborns who have jejunoileal atresia often will have large dilated
loops proximal to the obstruction. The classic presentation of a midgut volvulus is bilious vomiting, with an
abdomen that may initially be soft until blood flow through the superior mesenteric artery is compromised
and ischemia develops. The radiograph of the abdomen is often nonspecific except for dilated bowel
loops. UGI should be obtained when the infants predominant symptom is bilious vomiting because
midgut volvulus may be catastrophic, and early surgical intervention is essential.
Distal bowel obstructions include distal ileal atresia, colonic atresia, meconium ileus, meconium
plug syndrome, Hirshsprung disease, and imperforate anus. Affected infants often present with abdominal
distension and failure to pass meconium within the first 24 hours after birth; bilious vomiting is uncommon
but may be seen. The abdominal radiograph in distal bowel obstruction shows dilated loops of bowel.
Inspissated meconium or calcifications may be seen in meconium ileus. Contrast enema may be both
diagnostic and therapeutic for meconium ileus and meconium plug syndrome. The finding of a transition
zone on contrast enema suggests the diagnosis of Hirshsprung disease and the need for diagnostic rectal
biopsy for ganglion cells. The absence of air in the rectum on abdominal radiograph may suggest an
imperforate anus, for which a follow-up lateral abdominal radiograph with the infant prone permits
assessment of the distal progression of gas and assists in determining the surgical plan.
Abdominal ultrasonography and computed tomography scanning have a very limited role in
assessing congenital bowel obstruction in the neonate. Surgical consultation is often needed for definitive
diagnosis and treatment.

American Board of Pediatrics Content Specification(s):


Recognize the clinical signs and symptoms of congenital bowel obstruction

Suggested Reading:
Bales W, Liacouras CA. Intestinal atresia, stenosis, and malrotation. In: Kliegman RM, Stanton BF, St.
Geme JW III, Schor NF, and Behrman RE, eds. Nelson Textbook of Pediatrics. 19th ed. Philadelphia, PA:
Saunders Elsevier; 2011:1277-1281

Berseth CL, Poenaru D. Structural anomalies of the gastrointestinal tract. In: Taeusch HW, Ballard RA,
Gleason CA, eds. Averys Diseases of the Newborn. 8th ed. Philadelphia, PA: Elsevier Saunders;
2005:1086-1102

Copyright 2012 American Academy of Pediatrics


2012 PREP SA ON CD-ROM

Critique 106

(Courtesy of S Izatt)
Abdominal radiograph of the infant in the vignette showing dilated loops of small bowel with no evidence
of progression of gas through the distal bowel.

Copyright 2011 American Academy of Pediatrics


2012 PREP SA ON CD-ROM

Critique 106

(Courtesy of D Mulvihill)
Abdominal radiograph demonstrating the "double-bubble" sign in duodenal atresia. This appearance
results from a distended, gas-filled stomach and proximal duodenum.

Copyright 2011 American Academy of Pediatrics


2012 PREP SA ON CD-ROM

Question 107
A 6-year-old child has become pale and tired over the past 48 hours. He has had no fever or
respiratory or gastrointestinal symptoms. He has a past history of anemia, as does his father. On physical
examination, he has a heart rate of 128 beats/min, respiratory rate of 20 breaths/min, blood pressure of
92/64 mm Hg, and temperature of 36.8C. He is pale and has scleral icterus. His spleen is palpable 3 cm
below the left costal margin. Other findings on his examination are normal. An initial blood count shows a
hemoglobin of 7.6 g/dL (76 g/L) and a mean corpuscular hemoglobin concentration of 37 g/dL (370 g/L)
(mean for age, 34 g/dL [340 g/L]). The smear shows anisocytosis and numerous spherocytes (Item
Q107).
Of the following, the MOST important study(ies) to guide management of this patients condition
is(are)
A. acute hepatitis panel
B. alanine aminotransferase, aspartate aminotransferase, conjugated bilirubin
C. gallbladder ultrasonography
D. serial complete blood counts and reticulocyte counts
E. viral cultures

Copyright 2012 American Academy of Pediatrics


2012 PREP SA ON CD-ROM

Question 107

(Courtesy of M Wofford)
Spherocytes (arrows), as described for the boy in the vignette.

Copyright 2012 American Academy of Pediatrics


2012 PREP SA ON CD-ROM

Critique 107 Preferred Response: D


The combination of anemia, jaundice, and splenomegaly described for the boy in the vignette is
highly suggestive of a hemolytic anemia, and the presence of spherocytes on peripheral smear with an
elevated mean corpuscular hemoglobin concentration (MCHC) is typical of hereditary spherocytosis (HS).
Although transfusions rarely are needed for patients who have this condition, careful monitoring of
hemoglobin and reticulocyte count is important to detect excessive hemolysis or red cell aplasia that
necessitates more aggressive intervention. HS is the most common inherited cause of hemolytic anemia,
and the fathers history of anemia contributes to the likelihood of this diagnosis.
Although spherocytes are characteristic of HS, they are not diagnostic. They can be seen in many
other conditions, such as autoimmune hemolytic anemia, glucose-6-phosphate dehydrogenase
deficiency, ABO incompatibility, envenomation, burns, and microangiopathic hemolytic anemia. The
standard for confirming HS is the osmotic fragility test. However, this test may yield normal results in 20%
to 30% of patients, especially in the presence of numerous reticulocytes that are more resistant to lysis in
hypotonic solution than are mature spherocytes. Incubating the red blood cells for 24 hours produces a
more reliable osmotic fragility test. The combination of elevated MCHC and red cell distribution width is
nearly 100% specific for HS. Newer tests for HS include the acidified glycerol lysis test, the cryohemolysis
test, and a flow cytometric test. Numerous genotypes are associated with HS, and gene testing may
become available but likely will not affect the immediate clinical approach.
Patients who have HS may experience two types of crises: hemolytic and aplastic. Hemolytic
crises are more common and usually triggered by a viral illness. Like the patient in the vignette, affected
children experience worsening anemia and a transient increase in jaundice, spleen size, and reticulocyte
count. Hyperbilirubinemia is the result of hemolysis and is, therefore, unconjugated. Results of an acute
hepatitis panel and liver function studies are likely to be normal in this setting. Unless the anemia is
severe (hemoglobin <5 to 6 g/dL [50 to 60 g/L]), expectant monitoring is sufficient until recovery. Aplastic
crises occur in the presence of parvovirus B19 infection, and patients may experience profound anemia
and even congestive heart failure. The reticulocyte count is depressed. Such patients may require
transfusion, but appropriate supportive care usually results in recovery in 10 to 14 days. Viral culture for
either of these crises may be interesting but contributes little to patient management. Because the
patients experience chronic hemolysis, cholelithiasis is likely to develop, but such a presentation is
distinct from the symptoms demonstrated by the patient in the vignette.

American Board of Pediatrics Content Specification(s):


Recognize jaundice and splenomegaly as findings of hereditary spherocytosis
Know that increasing pallor in a child with hereditary spherocytosis may be a sign of an aplastic crisis
that warrants monitoring of the hemoglobin concentration and reticulocyte count
Know that a sudden increase in jaundice in a child with hereditary spherocytosis may be a sign of
increasing hemolysis, that warrants monitoring of the hemoglobin concentration

Suggested Reading:
Mentzer WC. Hereditary spherocytosis: clinical features; diagnosis; and treatment. UpToDate Online
18.3. 2010. Available for subscription at:
http://www.uptodate.com/online/content/topic.do?topicKey=red_cell/13139

Perkins SL. Pediatric red cell disorders and pure red cell aplasia. Am J Clin Pathol. 2004;122(suppl):S70-
S86. Available at: http://ajcp.ascpjournals.org/content/supplements/122/Suppl_1/S70.long

Perrotta S, Gallagher PG, Mohandas N. Hereditary spherocytosis. Lancet. 2008;372:1411-1426. DOI:


10.1016/S0140-6736(08)61588-3. Abstract available at: http://www.ncbi.nlm.nih.gov/pubmed/18940465

Shah S, Vega R. Hereditary spherocytosis. Pediatr Rev. 2004;25:168-170. DOI: 10.1542/pir.25-5-168.


Available at: http://pedsinreview.aappublications.org/cgi/content/full/25/5/168

Copyright 2012 American Academy of Pediatrics


2012 PREP SA ON CD-ROM

Question 108
An intern asks you to evaluate a 4-day-old infant who has a rash. On physical examination, the
healthy-appearing infant has a rash on the trunk and extremities that is comprised of erythematous
macules, each of which has a central papule or vesicle (Item Q108). A Gram stain of the lesion shows
white blood cells but no organisms and a Wright stain shows numerous eosinophils.
Of the following, the MOST likely diagnosis is
A. congenital candidiasis
B. erythema toxicum
C. herpes simplex virus infection
D. staphylococcal folliculitis
E. transient neonatal pustular melanosis

Copyright 2012 American Academy of Pediatrics


2012 PREP SA ON CD-ROM

Question 108

(Courtesy of P Sagerman)

Copyright 2012 American Academy of Pediatrics


2012 PREP SA ON CD-ROM

Critique 108 Preferred Response: B


The infant described in the vignette has erythema toxicum, which occurs in about 50% of term
newborns. Although the cause of the rash is unknown, histology reveals eosinophils. Erythema toxicum is
very common and has an evanescent nature, with lesions disappearing from one area and appearing in
another soon thereafter. The peak incidence is on the second postnatal day, but the condition may
persist, waxing and waning, for slightly more than 1 month. It is uncommon in preterm infants.
Transient neonatal pustular melanosis also occurs in the newborn period and is most common in
African American infants. The histology of these lesions is characterized by more leukocytes than
eosinophils. The pustule appears early, followed by a hyperpigmented macule that may persist for up to 3
months (Item C108A). Both erythema toxicum and pustular melanosis are benign conditions that have no
sequelae and require no treatment.
Neonatal infections, including candidiasis, herpes simplex virus (HSV) infection, and
staphylococcal disease, require therapy and, therefore, prompt identification. Cutaneous herpes simplex
infections may appear as vesicles (Item C108B) and ulcerated lesions rather than pustules and involve
weeping or crusting lesions. The clinician must consider this diagnosis even for infants of mothers who
have no history of herpes infection because neonatal herpes infection has a poor prognosis and there
often is no maternal history of HSV infection in affected newborns. Dissemination and systemic illness
usually are characterized by clinical signs of illness, but up to 40% of neonatal herpes infection (skin, eye,
and mucous membrane subtype) may begin with asymptomatic skin lesions. Typically, the lesions are
identified initially at the site of the fetal scalp electrode, the diaper area, or adjacent to suckling blisters
on the hands.
Staphylococcal disease in term asymptomatic newborns often presents as a mixed picture that
includes pustules (Item C108C), vesicles, or bullae of various sizes with desquamation. Isolated, localized
areas of pustules have been described, frequently in the diaper area. Onset is usually later than for
erythema toxicum or pustular melanosis (7 to 12 days of age). Congenital candidiasis, which is cutaneous
and not invasive, generally appears as clusters of pustules on the head and neck but occasionally
diffusely (Item C108D). In the absence of signs of systemic illness, it may be treated with topical
antifungals. Invasive candidiasis is unusual in term infants who have no risk factors.

American Board of Pediatrics Content Specification(s):


Recognize erythema toxicum
Know that the lesions of erythema toxicum are filled with eosinophils

Suggested Reading:
Diana A, Epiney M, Ecoffey M, Pfister RE. White dots on the placenta and red dots on the baby:
Congenital cutaneous candidiasis--a rare disease of the neonate. Acta Paediatr. 2004;93:996-999.
Abstract available at: http://www.ncbi.nlm.nih.gov/pubmed/15303819

Faloyin M, Levitt J, Bercowitz E, Carrasco D, Tan J. All that is vesicular is not herpes: incontinentia
pigmenti masquerading as herpes simplex virus in a newborn. Pediatrics. 2004;114:e270-e272. Available
at: http://pediatrics.aappublications.org/cgi/content/full/114/2/e270

Fortunov RM, Kaplan SL. Methicillin-resistant Staphylococcus aureus in previously healthy neonates.
NeoReviews. 2008;9:e580-e584. DOI: 10.1542/neo.9-12-e580. Available at:
http://neoreviews.aappublications.org/cgi/content/full/9/12/e580

Johr RH, Schachner LA. Neonatal dermatologic challenges. Pediatr Rev. 1997;18:86-94 . DOI:
10.1542/pir.18-3-86. Available at: http://pedsinreview.aappublications.org/cgi/content/full/18/3/86

Morelli JG. The skin: diseases of the neonate. In: Kliegman RM, Stanton BF, St. Geme JW III, Schor NF,
and Behrman RE, eds. Nelson Textbook of Pediatrics. 19th ed. Philadelphia, PA: Saunders Elsevier;
2011:2218-2220

Copyright 2012 American Academy of Pediatrics


2012 PREP SA ON CD-ROM

Critique 108

(Reprinted with permission from Krowchuk DP, Mancini AJ, eds. Pediatric Dermatology. A Quick
Reference Guide. 2nd ed. Elk Grove Village, IL: American Academy of Pediatrics; 2011)
Transient neonatal pustulosis may appear as sterile pustules or hyperpigmented macules, often with a
rim or collarette of scale (arrows), the remnants of the pustule roofs.

Copyright 2011 American Academy of Pediatrics


2012 PREP SA ON CD-ROM

Critique 108

(Courtesy of D Krowchuk)
Herpes simplex virus infection is characterized by grouped vesicles on an erythmatous base.

Copyright 2011 American Academy of Pediatrics


2012 PREP SA ON CD-ROM

Critique 108

(Courtesy of A Mancini)
Staphylococcal folliculitis appears as erythematous papules or pustules with surrounding erythema.

Copyright 2011 American Academy of Pediatrics


2012 PREP SA ON CD-ROM

Critique 108

(Reprinted with permission from Krowchuk DP, Mancini AJ, eds. Pediatric Dermatology. A Quick
Reference Guide. 2nd ed. Elk Grove Village, IL: American Academy of Pediatrics; 2011)
Congenital candidiasis is characterized by erythematous papules, pustules, and scaling.

Copyright 2011 American Academy of Pediatrics


2012 PREP SA ON CD-ROM

Question 109
The mother of an 8-year-old boy brings him in for follow-up care 6 weeks after a motor vehicle
crash in which he sustained minor abrasions and contusions. The mother notes that her son is healing
well, and she is pleased that he does not have any significant scarring. However, she comments that he
has not been himself lately and is driving her nuts. He has been whiny and easily tearful, and all he
wants to do is play with some old toy cars he found in the basement. He refused to go to school the last
two Mondays, stating he had a stomachache and begging to stay home. He has begun to come into his
parents room at night, complaining that he had a scary dream and cant fall asleep. For the past few
weeks, he has not wanted to go to his friends house to play. On physical examination, the boys
abrasions are healed and he has no residual bruising or scarring. His vital signs and other findings on
physical examination, including abdominal evaluation, are within normal limits. He is quiet but
cooperative, he has his fists in his pockets, and he makes adequate eye contact. He shrugs his shoulders
when asked why he doesnt want to visit his friend or why he suddenly doesnt like going to school. His
energy is fine and his appetite is good. He says that he would just rather stay home and leans into his
mother, burying his head in her shoulder and crying silently.
Of the following, the MOST likely diagnosis for this boy is
A. major depressive disorder
B. normal behavior
C. oppositional defiant disorder
D. posttraumatic stress disorder
E. separation anxiety disorder

Copyright 2012 American Academy of Pediatrics


2012 PREP SA ON CD-ROM

Critique 109 Preferred Response: D


Evaluation of emotional and behavioral symptoms in children should always include the
relationship among the presenting concerns, the childs pre-existing baseline functioning, and any
significant stressors in the childs environment. The boy described in the vignette has no history of
problems going to school, visiting friends in their homes, or getting along with his family, suggesting that
he is generally well adjusted. However, his behavior has changed following a significant event: the motor
vehicle crash. Although he was not significantly injured, he easily could have perceived it as a life-
threatening event.
The diagnosis of posttraumatic stress disorder (PTSD) requires first that an event occur that
either is or is perceived to be life-threatening. One in four children experiences a significant traumatic
event, such as child abuse, domestic violence, school violence, community violence, natural disasters, or
motor vehicle crashes. The response to such events is critical to understand in exposed children because
three clusters of symptoms must be present for the diagnosis of PTSD: 1) re-experiencing of the event, 2)
persistent avoidance, and 3) hyperarousal. This boy is having nightmares, frequently refuses to get in the
car for drives, and is having trouble falling asleep 6 weeks after the crash. These new symptoms are
causing him to have problems with his family, school, and social arena and, thus, meet the criteria for
PTSD.
The diagnosis of major depressive disorder requires at least 2 weeks of low or irritable mood plus
at least four additional symptoms from a list that includes poor sleep, poor appetite, low energy, fatigue,
poor concentration, low self-esteem, guilt, worthlessness, hopelessness, helplessness, passive thoughts
of death, and suicidal thinking. Although this boy has some irritability, easy tearfulness, and poor sleep,
he does not exhibit enough other symptoms to consider major depressive disorder as the primary
concern. Of note, some children who have PTSD do develop comorbid mood problems.
Separation anxiety does present with avoidance of school and friends homes, a preference for
staying home, and having trouble sleeping alone. However, such findings would have a pervasive
presentation and not develop after an acute event.
The boys response is not within the normal range of behavior, primarily because it is causing
difficulties for him (missing school, avoiding social events) and for his family (sleeping with parents,
irritating them, power struggles as they try to limit the avoidance). Although many children have some
acute symptoms immediately after a traumatic event, fewer than one third have persistent symptoms and
difficulties beyond 1 month. His presentation is worrisome and should not be considered a normal
response because the event occurred 6 weeks ago.
Oppositional defiant disorder represents a pattern of negativistic, disobedient behavior of at least
6 months duration aimed at authority figures. Although this boy is refusing certain activities, he does not
have the requisite stubbornness, resistance to following directions, or intentional annoyance of others that
oppositional defiant disorder describes.

AAP Mental Health Competency:


Recognize the symptoms of posttraumatic stress disorder (PTSD) in a child

Suggested Reading:
American Academy of Pediatrics Task Force on Mental Health. Anxiety Cluster Guidance. Addressing
Mental Health Concerns in Primary Care: A Clinician's Toolkit [CD-ROM]. Elk Grove Village, IL: American
Academy of Pediatrics; 2010

Cohen JA, Bukstein O, Walter H, et al; AACAP Work Group on Quality Issues. Practice parameter for the
assessment and treatment of children and adolescents with posttraumatic stress disorder. J Am Acad
Child Adolesc Psychiatry. 2010;49:414-430. Abstract available at:
http://www.ncbi.nlm.nih.gov/pubmed/20410735

Donnelly CL, March JS, Amaya-Jackson L. Posttraumatic stress disorder. In: Dulcan MK, Wiener JM,
eds. Essentials of Child and Adolescent Psychiatry. Arlington, VA: American Psychiatric Publishing, Inc;
2006:479-504

Copyright 2012 American Academy of Pediatrics


2012 PREP SA ON CD-ROM

Question 110
You diagnose an interrupted aortic arch in a newborn. You order an infusion of prostaglandin E1
to maintain patency of the ductus arteriosus.
Of the following, the physiologic benefits of this medication MOST likely are due to increased
A. pulmonary blood flow via left-to-right ductal shunting
B. pulmonary blood flow via right-to-left ductal shunting
C. pulmonary vasodilatory effects
D. systemic blood flow via left-to-right ductal shunting
E. systemic blood flow via right-to-left ductal shunting

Copyright 2012 American Academy of Pediatrics


2012 PREP SA ON CD-ROM

Critique 110 Preferred Response: E


The advent of prostaglandin E1 therapy in the 1970s was an important advance in the treatment
of neonates who have ductal-dependent congenital heart disease. Previously, the only option was
emergency surgery, usually in the form of an aortopulmonary communication (most commonly, the
Blalock-Taussig shunt type). The ability to maintain or re-establish patency of the ductus arteriosus with
prostaglandin E1 allows for effective stabilization of the severely cyanotic neonate, which provides a
period of circulatory control while metabolic derangements are corrected and controlled surgical planning
can ensue.
In the setting of an interrupted aortic arch, as described for the newborn in the vignette, the
physiologic benefits of prostaglandin E1 are based upon providing systemic blood flow via right-to-left
shunting at the level of the ductus arteriosus. In the absence of ductal flow, the interruption in arch
circulation results in peripheral hypoperfusion, acidosis, myocardial dysfunction, and shock. Once the
ductus arteriosus flow is re-established, peripheral perfusion should improve and allow for a period of
stability. During this time, surgical decisions can be made. The most common correction for interrupted
aortic arch is single-stage arch reconstruction in which the region of hypoplasia is removed and replaced
with either native tissue or a tube graft and the ventricular septal defect is closed with a patch.
Prostaglandin E1 is also used in hypoplastic left heart syndrome, critical aortic valve stenosis, and
coarctation of the aorta to maintain systemic circulation. In other ductal-dependent lesions, such as
pulmonary valve atresia, severe tetralogy of Fallot, and critical pulmonary valve stenosis, prostaglandin
allows for left-to-right flow to maintain pulmonary artery circulation. In these cases, closure of the ductus
arteriosus severely compromises flow to the pulmonary circulation, causing profound cyanosis.
Transposition of the great arteries is also ductal-dependent. The right heart supplies desaturated
blood to the aorta, which shunts the blood to the lungs via the ductus arteriosus. The highly saturated
blood returning from the lungs to the left heart supplies oxygenated blood through the foramen ovale to
the body via the aorta. In many cases, prostaglandin alone is inadequate to improve oxygen saturation;
until resolution of restriction at the foramen ovale, severe desaturation may persist. Therefore, a balloon
atrial septostomy is required for unrestricted left-to-right atrial flow, which allows for the availability of an
adequate amount of saturated blood to the systemic circulation and resultant improvement in tissue
oxygen delivery.
Although prostaglandin E1 does have pulmonary vasodilatory effects, pulmonary blood flow is
unrestricted in aortic arch interruption. Therefore, this drug effect is of no benefit for this particular cardiac
lesion.

American Board of Pediatrics Content Specification(s):


Understand the role of ductus arteriosus in cyanotic congenital heart disease and the use of
prostaglandin E1 in treatment

Suggested Reading:
Bernstein D. Evaluation of the infant or child with congenital heart disease. In: Kliegman RM, Stanton BF,
St. Geme JW III, Schor NF, and Behrman RE, eds. Nelson Textbook of Pediatrics. 19th ed. Philadelphia,
PA: Saunders Elsevier; 2011:1549-1551

Dorfman AT, Marino BS, Wernovsky G, et al. Critical heart disease in the neonate: presentation and
outcome at a tertiary care center. Pediatr Crit Care Med. 2008;9:193-202. DOI:
10.1097/PCC.0b013e318166eda5. Abstract available at: http://www.ncbi.nlm.nih.gov/pubmed/18477933

Gersony WM, Apfel HD. Patent ductus arteriosus and other aortopulmonary anomalies. In: Moller JH,
Hoffman JIE, eds. Pediatric Cardiovascular Medicine. Philadelphia, PA: Churchill Livingstone; 2003:38

Silberbach M, Hannon D. Presentation of congenital heart disease in the neonate and young infant.
Pediatr Rev. 2007;28:123-131. DOI: 10.1542/pir.28-4-123. Available at:
http://pedsinreview.aappublications.org/cgi/content/full/28/4/123

Copyright 2012 American Academy of Pediatrics


2012 PREP SA ON CD-ROM

Question 111
During the hottest week of the summer, a 6-month-old previously healthy infant presents to the
emergency department via ambulance in tonic-clonic status epilepticus. After two doses (each 0.05
mg/kg) of intravenous lorazepam, the seizure continues. The bedside glucose measurement is normal, as
is a noncontrast head computed tomography scan.
Of the following, the MOST likely cause of the childs prolonged seizure is
A. hyperthyroidism
B. hypocalcemia
C. hypomagnesemia
D. hyponatremia
E. pyridoxine deficiency

Copyright 2012 American Academy of Pediatrics


2012 PREP SA ON CD-ROM

Critique 111 Preferred Response: D


The infant described in the vignette has presented to the emergency department in status
epilepticus, a medical emergency. Treatment is primary, and initial treatment with a benzodiazepine has
been administered. The next step is to administer an intravenous loading dose of 20 mg/kg fosphenytoin.
The possible causes for status epilepticus must be evaluated rapidly and systematically. These
include focal lesions (stroke, trauma, abscess), infections, toxins, and metabolic derangements. In an
infant, the history and physical examination often yield less information than they do in an older child. For
example, it can be more difficult to localize a focal brain process through examination for an infant than
for an older child. A routine evaluation for status epilepticus in an infant generally should include serum
chemistries to evaluate for metabolic derangements and head computed tomography scan to look for
blood or other focal processes.
Hyponatremia can occur in infants, such as the one in the vignette, who receive excessive free
water and can result in status epilepticus that may not respond to initial treatment with benzodiazepines.
Hyperthyroidism is uncommon in infancy and is not likely to present with status epilepticus. Hypocalcemia
may occur in the context of hypoparathyroidism, as in neonates who have DiGeorge syndrome, but that
condition usually presents in the first month after birth. Hypomagnesemia can present with status
epilepticus, but this is rare and usually accompanies other electrolyte abnormalities. It can be iatrogenic
or produced by sustained gastrointestinal or renal losses. Pyridoxine deficiency can cause an epileptic
encephalopathy, but it presents in the neonatal period, not at age 6 months.

2American Board of Pediatrics Content Specification(s):


Recognize the metabolic causes of seizures

Suggested Reading:
Breault DT, Majzoub JA. Other abnormalities of arginine vasopressin metabolism and action. In:
Kliegman RM, Stanton BF, St. Geme JW III, Schor NF, and Behrman RE, eds. Nelson Textbook of
Pediatrics. 19th ed. Philadelphia, PA: Saunders Elsevier; 2011:1884-1886

Mikati MA. Seizures in childhood. In: Kliegman RM, Stanton BF, St. Geme JW III, Schor NF, and
Behrman RE, eds. Nelson Textbook of Pediatrics. 19th ed. Philadelphia, PA: Saunders Elsevier;
2011:2013-2039

Major P, Thiele EA. Seizures in children: laboratory diagnosis and management. Pediatr Rev.
2007;28:405-414. DOI: 10.1542/pir.28-11-405. Available at:
http://pedsinreview.aappublications.org/cgi/content/full/28/11/405

Copyright 2012 American Academy of Pediatrics


2012 PREP SA ON CD-ROM

Question 112
You are examining a 6-year-old girl who is new to your practice. Past medical history reveals that
she had a normal head circumference at birth but experienced head growth deceleration over time. Her
development was relatively normal until she was about 1 year of age but has regressed since then. She
developed a seizure disorder when she was three years old. Her mother reports that her daughter has an
abnormal breathing pattern (intermittent hyperventilation and breath-holding) and a stiff, awkward gait.
Of the following, the MOST likely diagnosis for this girl is
A. Angelman syndrome
B. duplication 15q11
C. phenylketonuria
D. Rett syndrome
E. Smith-Lemli-Opitz syndrome

Copyright 2012 American Academy of Pediatrics


2012 PREP SA ON CD-ROM

Critique 112 Preferred Response: D


The girl described in the vignette exhibits some of the classic features of Rett syndrome, including
relatively normal neurodevelopmental progress for the first 6 to 18 months after birth, followed by a period
of stagnation and subsequent developmental regression. Episodic apnea and hyperpnea are typical
features, as are seizures, ataxia, and acquired microcephaly. Although the classic Rett syndrome
described in the vignette is seen almost exclusively in girls and is associated with stereotypic hand
movements (such as handwringing) and loss of purposeful hand movements, now that the molecular
defect has been characterized, clinicians are beginning to appreciate the much broader clinical spectrum
of the disorder. Many affected girls also have unusual behavioral patterns, with episodes of unprovoked
screaming or crying, bruxism, or panic attacks.
Mutations in the MECP2 gene are not only associated with classic Rett syndrome but are also
found in girls who have atypical features and are often misdiagnosed with Angelman syndrome. Atypical
patients may exhibit intellectual disabilities with hypertonia or tremors, only mild learning disabilities
without developmental regression, or simple autism without other findings. Most males who have the
MECP2 mutation have severe neonatal encephalopathy and die early, but some males who have somatic
mosaicism for a MECP2 mutation or even 47, XXY karyotype may present with similar features to
affected females. In light of these new data on atypical presentations for MECP2 mutations, some
clinicians recommend MECP2 genetic testing for all girls diagnosed with autism. If this approach is
considered to be overly inclusive, it would be especially important to consider such testing in a girl who
has autism, microcephaly, concordant behavioral disturbances, or neurologic findings suggestive of
atypical Rett syndrome.
Children who have Angelman syndrome frequently have acquired microcephaly, ataxia, and
neurodevelopmental disabilities with frequent lack of verbal output, but they do not exhibit normal
development in early life and generally do not experience developmental regression. Children who have a
15q11 duplication, which includes the Prader-Willi and Angelman loci, may present with hypotonia,
developmental delays, and features of autism, but microcephaly or developmental regression are not
characteristic. Children who have phenylketonuria (PKU) generally are identified on newborn screening
and treated appropriately, but if such a case is missed, an untreated child typically presents with vomiting,
irritability, an eczematous rash, and mousy urine body odor, later developing hypertonia, intellectual
disability, and seizures. Although acquired microcephaly is also seen in children who have untreated
PKU, the clinical features are somewhat different than in girls who have classic Rett syndrome. Children
who have Smith-Lemli-Opitz syndrome (SLOS) often have multiple congenital anomalies (polydactyly, 2-
3-toe syndactyly, cleft palate, cardiac defects, renal malformations, or hypospadias in boys), along with
facial dysmorphisms, microcephaly, hypotonia, failure to thrive, and seizures. They do not have
completely normal development in infancy because of neonatal hypotonia and poor feeding. SLOS is
diagnosed by measuring 7-dehydrocholesterol, which is extremely elevated due to a block in the
cholesterol synthesis pathway resulting from delta 7-dehydrocholesterol reductase deficiency.

American Board of Pediatrics Content Specification(s):


Understand that the identification of the genetic defect in Rett syndrome has greatly expanded the
understanding of its clinical spectrum

Suggested Reading:
Christodoulou J, Ho G. MECP2-related disorders. GeneReviews. 2009. Available at:
http://www.ncbi.nlm.nih.gov/bookshelf/br.fcgi?book=gene&part=rett

Johnson CP, Myers SM and the Council on Children with Disabilities. Identification and evaluation of
children with autism spectrum disorders. Pediatrics. 2007;120:1183-1215. DOI: 10.1542/peds.2007-2361.
Available at: http://pediatrics.aappublications.org/cgi/content/full/120/5/1183

Laurvick CL, Msall ME, Silburn S, Bower C, de Klerk N, Leonard H. Physical and mental health of
mothers caring for a child with Rett syndrome. Pediatrics. 2006;118:e1152-e1164. DOI:
10.1542/peds.2006-0439. Available at: http://pediatrics.aappublications.org/cgi/content/full/118/4/e1152

Copyright 2012 American Academy of Pediatrics


2012 PREP SA ON CD-ROM

Question 113
You are evaluating a 13-year-old boy who has had intermittent low back pain and stiffness for the
past 3 months. He also has some ankle pain but no fever or other complaints. He plays soccer but does
not recall any particular injuries. On physical examination, you note some tenderness of his sacroiliac
joints. He has normal lower extremity strength, sensation, and reflexes and negative results on a straight
leg-raising test. There is moderate tenderness to palpation over the insertion of his Achilles tendon. He
has no skin or eye findings and no other joint findings. Spinal radiographs are read as normal.
Of the following, the BEST next step is
A. acetaminophen use as needed
B. corticosteroid injections in the Achilles tendon
C. evaluation for spondyloarthropathies
D. heat to his back and bed rest
E. screening for sexually transmitted infections

Copyright 2012 American Academy of Pediatrics


2012 PREP SA ON CD-ROM

Critique 113 Preferred Response: C


Back pain as a presenting symptom is more common in older adolescents and athletes. In
younger adolescents, clinicians should be careful not to attribute the pain to a simple musculoskeletal
strain without further evaluation. Spondylolysis, a defect of the pars interarticularis, and spondylolisthesis
(the forward slippage of one vertebra over another, commonly of L5 on S1) are the most common causes
of chronic low back pain in adolescents. Some sports or activities, such as ballet dancing, gymnastics, or
wrestling, are more likely to be associated with these conditions because they require hyperextension of
the spine.
The young adolescent described in the vignette is not involved in one of these high-risk activities
and has low back pain and tenderness to palpation of the sacroiliac joints and at the site of insertion of his
Achilles tendon (ie, enthesitis). Together, these findings suggest the presence of a spondyloarthropathy
such as ankylosing spondylitis. Enthesitis is the result of mechanical stress or an inflammatory reaction,
often resulting in fibrosis or calcification at the site. It may be asymptomatic or associated with pain that
can be mild or severe and disabling. Enthesitis is more frequently seen in the lower than the upper limbs;
may involve synovial, cartilaginous, or fibrous (eg, syndesmoses) joints; or could be extra-articular.
Peripheral extra-articular enthesitis (such as that affecting the insertion of the Achilles tendon) is now
recognized as a distinctive pathologic feature of spondyloarthritis and is often the primary lesion, which
suggests the need to evaluate this boy for spondyloarthropathies. Extra-articular enthesitis may be the
only longstanding clinical manifestation of an HLA-B27-associated disease process. Bone changes on
radiography may be a late finding. Although magnetic resonance imaging is the most commonly used
imaging technique, findings are nonspecific. Recently, studies indicated that B-mode ultrasonography
combined with power Doppler has a high sensitivity and specificity for diagnosing spondyloarthritis. A
complete blood count, erythrocyte sedimentation rate, and HLA-B27 assessment should also be
considered along with an ophthalmologic evaluation.
The pain is more likely to respond to nonsteroidal anti-inflammatory drugs than acetaminophen.
Modification of activity rather than bed rest should be recommended because the pain often worsens with
rest and decreases with activity. Corticosteroid injections into the tendon increase the risk of rupture.
Reiter syndrome is a symptom complex that includes sacroiliitis, arthritis, conjunctivitis, urethritis, and a
skin rash (keratoderma blenorrhagica). It is strongly associated with HLA-B27 and sometimes results from
exposure to a sexually transmitted infection such as Chlamydia. However, this boy exhibits no other
features of Reiter syndrome.

American Board of Pediatrics Content Specification(s):


Recognize enthesitis with arthritis as a possible precursor to ankylosing spondylitis
Recognize the clinical and laboratory findings in a patient with ankylosing spondylitis

Suggested Reading:
Brent LH, Kalagate R. Akylosing spondyolitis and undifferentiated spondyloarthropathy.eMedicine
Specialties, Rheumatology, Spondyloarthropaties. 2010. Available at:
http://emedicine.medscape.com/article/332945-overview

D'Agostino MA, Olivieri I. Enthesitis. Best Pract Res Clin Rheumatol. 2006;20:473-486. DOI:
10.1016/j.berh.2006.03.007. Abstract available at: http://www.ncbi.nlm.nih.gov/pubmed/16777577

Flat B, Hoffmann-Vold AM, Reiff A, Frre , Lien G, Vinje O. Long-term outcome and prognostic factors
in enthesitis-related arthritis: a case-control study. Arthritis Rheum. 2006;54:3573-3582. DOI:
10.1002/art.22181. Available at: http://onlinelibrary.wiley.com/doi/10.1002/art.22181/full

Goldmuntz EA, White PH. Juvenile idiopathic arthritis: a review for the pediatrician. Pediatr Rev.
2006;27:e24-e32. DOI: 10.1542/pir.27-4-e24. Available at:
http://pedsinreview.aappublications.org/cgi/content/full/27/4/e24

Tse SML Laxer RM. Approach to acute limb pain in childhood. Pediatr Rev. 2006;27:170-180. DOI:
10.1542/pir.27-5-170. Available at: http://pedsinreview.aappublications.org/cgi/content/full/27/5/170

Copyright 2012 American Academy of Pediatrics


2012 PREP SA ON CD-ROM

Question 114
You are providing moderate sedation to a 9-month-old infant who weighs 10 kg to allow complete
transthoracic echocardiography. The child underwent an uncomplicated repair of a large ventricular septal
defect 1 week ago, has no allergies, and has been appropriately fasting. Your health evaluation before
administering the sedative reveals no abnormal physical findings, except for a well-healed sternotomy
scar. His temperature is 37.0C, heart rate is 120 beats/min, respiratory rate is 25 breaths/min, blood
pressure is 90/55 mm Hg, and oxygen saturation is 98% in room air. You order 0.5 mg (0.05 mg/kg) of
midazolam, which the nurse administers intravenously. Shortly after administration, the child develops
apnea, with a decrease in blood pressure to 75/40 mm Hg and no change in heart rate. As you reposition
the airway and begin bag-valve-mask ventilation, the nurse reports that an error had been made and that
5 mg (0.5 mg/kg) of midazolam was administered inadvertently.
Of the following, the MOST appropriate next step is administration of
A. atropine
B. epinephrine
C. fentanyl
D. flumazenil
E. naloxone

Copyright 2012 American Academy of Pediatrics


2012 PREP SA ON CD-ROM

Critique 114 Preferred Response: D


Children can be anxious and fearful when they are ill or injured and often require the
administration of sedative medications in addition to reassurance and appropriate analgesics. Sedatives
also are provided to relieve anxiety and help minimize patient movement during procedures. The
provision of safe and effective sedation requires appropriate patient selection, evaluation, and
preparation. Recognition of oversedation, as described for the infant in the vignette, is imperative and
requires appropriate monitoring, resuscitation equipment, and trained personnel for a timely and
appropriate response. Specific reversal agents should be administered rapidly, such as flumazenil in the
event of a benzodiazepine overdose, as for this infant. Naloxone is indicated for a narcotic overdose.
Neither atropine nor epinephrine is indicated for this infant because he does not have bradycardia or
significant hypotension. Additional sedatives or narcotics, such as fentanyl, are contraindicated.
Level of consciousness should be followed closely when administering sedative agents, but the
exact level of sedation may be difficult to assess accurately because sedation represents a continuum,
and patients have varying responses to individual medications and dosing. Sedation scales such as
COMFORT (alertness, calmness, muscle tone, physical movement, facial tension, respiratory response,
blood pressure, and heart rate), FLACC (face, legs, activity, cry, consolability), or RASS (Richmond
Agitation Sedation Scale) are being increasingly used. Because sedatives affect respiratory drive and
hemodynamic stability, changes in heart rate, blood pressure, respiratory effort, or oxygen saturation
should prompt intervention with airway management, oxygen administration, and fluid administration, as
indicated.
Cot and associates have studied the issue of pediatric sedation extensively and found that
negative outcomes are associated with the administration of sedative medications without appropriate
monitoring. Adverse sedation events may occur with all classes of medications and routes of
administration. Frequently, such events are associated with drug overdoses and drug interactions,
particularly when three or more drugs are used.

American Board of Pediatrics Content Specification(s):


Recognize side effects and signs and symptoms of an overdosage of commonly prescribed
sedatives, and manage appropriately

Suggested Reading:
American Academy of Pediatrics, American Academy of Pediatric Dentistry, Cot CJ, Wilson S, the Work
Group on Sedation. Guidelines for monitoring and management of pediatric patients during and after
sedation for diagnostic and therapeutic procedures: an update. Pediatrics. 2006;118:2587-2602. DOI:
10.1542/peds.2006-2780. Available at: http://pediatrics.aappublications.org/cgi/content/full/118/6/2587

Cot CJ, Karl HW, Notterman DA, Weinberg JA, McCloskey C. Adverse sedation events in pediatrics:
analysis of medications used for sedation. Pediatrics. 2000;106:633-644. Available at:
http://pediatrics.aappublications.org/cgi/content/full/106/4/633

Cot CJ, Notterman DA, Karl HW, Weinberg JA, McCloskey C. Adverse sedation events in pediatrics: a
critical incident analysis of contributing factors. Pediatrics. 2000;105:805-814. Available at:
http://pediatrics.aappublications.org/cgi/content/full/105/4/805

Copyright 2012 American Academy of Pediatrics


2012 PREP SA ON CD-ROM

Question 115
A 9-year-old boy presents with a 2- to 3-month history of increased urination, intermittent back
pain, and constipation. There are no other findings of note on his history, and the only abnormality noted
on physical examination is mild hyporeflexia. Among the screening laboratory results are total calcium of
12.1 mg/dL (3.0 mmol/L) and phosphorus of 1.7 mg/dL (0.55 mmol/L).
Of the following, the test that is MOST likely to establish the diagnosis is
A. serum alkaline phosphatase
B. serum ionized calcium
C. serum parathyroid hormone
D. serum parathyroid hormone-related protein
E. urinary calcium/creatinine ratio

Copyright 2012 American Academy of Pediatrics


2012 PREP SA ON CD-ROM

Critique 115 Preferred Response: C


The boy described in the vignette has classic signs, symptoms, and laboratory evidence of
hypercalcemia due to primary hyperparathyroidism. The most helpful test to establish the diagnosis is
measurement of parathyroid hormone (PTH). Calcium homeostasis is critical for the normal functioning of
nearly all human systems and is exquisitely controlled by a standard feedback loop that relies on the
calcium-sensing receptors located primarily on the parathyroid glands. When these receptors sense a low
calcium concentration, they stimulate parathyroid cells to produce parathyroid hormone that, in turn, acts
to increase calcium absorption (via activation of vitamin D), increase calcium reabsorption from the bones
(via activation of osteoclasts), and increase calcium retention via the direct effects of PTH in the distal
tubules and the thick ascending limbs of the kidneys. Conversely, when the calcium-sensing receptor
detects a high calcium concentration, the normal feedback loop is designed to reduce PTH production
and enhance calcium secretion.
Alkaline phosphatase may be elevated in the boy in the vignette, but it is a nonspecific finding
that would not help in establishing the diagnosis. Similarly, an ionized calcium value would only provide
additional information if there was concern for an abnormally high albumin value that results in an
elevated total calcium value. Assessment of PTH-related protein might be helpful if there were concerns
that a malignancy was causing the boys hypercalcemia; some tumors can secrete this hormone, with
similar effects on bone, kidney, and vitamin D, but this assessment would provide a relatively low yield as
an initial screen in this patient. Finally, a urinary calcium/creatinine ratio might be helpful in evaluating for
familial hypocalciuric hypercalcemia, which results in an inappropriately low urine ratio for the elevated
serum calcium (due to a mutation in the calcium-sensing receptor). However, it would not be likely to lead
to the diagnosis in cases of the more common, but still relatively rare, pediatric primary
hyperparathyroidism. Multiple endocrine neoplasia types 1 and 2 should be considered once
hyperparathyroidism is diagnosed in a child.

American Board of Pediatrics Content Specification(s):


Understand that hypercalcemia with hypophosphatemia suggests hyperparathyroidism

Suggested Reading:
Corathers SD. Focus on diagnosis: the alkaline phosphatase level: nuances of a familiar test. Pediatr
Rev. 2006;27:382-384. DOI: 10.1542/pir.27-10-382. Available at:
http://pedsinreview.aappublications.org/cgi/content/full/27/10/382

Kollars J, Zarroug AE, van Heerden J, et al. Primary hyperparathyroidism in pediatric patients. Pediatrics.
2005;115:974-980. DOI: 10.1542/peds.2004-0804. Available at:
http://pediatrics.aappublications.org/cgi/content/full/115/4/974

Mastrandrea LD, Albini CH. Bisphosphonate treatment of tumor-induced hypercalcemia in a toddler: case
report and review of related literature. Endocr Pract. 2006;12:670-675. Abstract available at:
http://www.ncbi.nlm.nih.gov/pubmed/17229665

Root AW, Diamond FB Jr. Disorders of mineral homeostasis in the newborn, infant, child, and adolescent.
In: Sperling MA, ed. Pediatric Endocrinology. 3rd ed. Philadelphia, PA: Saunders; 2008:686-769

Copyright 2012 American Academy of Pediatrics


2012 PREP SA ON CD-ROM

Question 116
A 7-year-old girl is having behavioral problems in school. Her academic skills are strong, but she
is impulsive and has difficulty staying on task and remaining quiet while the teacher is talking. When the
students line up, she pushes to be at the head of the line. At home, her parents have problems getting
her to comply with their requests. She needs frequent reminders to sit and do her homework.
Of the following, the MOST appropriate next step is to
A. begin a trial of stimulant medication
B. complete Vanderbilt questionnaires
C. have the parents institute a token economy behavior plan
D. obtain a thyroid function test
E. refer the child for psychoeducational testing

Copyright 2012 American Academy of Pediatrics


2012 PREP SA ON CD-ROM

Critique 116 Preferred Response: B


The child described in the vignette is having behavioral issues both at home and school. Having
the parents and teacher complete rating questionnaires such as the Vanderbilt can aid in delineating the
childs attention and activity levels and the presence of oppositional or other disruptive behaviors.
Instituting a behavioral plan such as a token economy may help to modify her behavior after a full
assessment is completed. Stimulant medication may be beneficial if the child is diagnosed with attention-
deficit disorder. Because the girl is not having any academic issues, a psychoeducational evaluation is
not indicated, and there are no clinical findings to suggest the need for a thyroid function test.
Although rating scales are not diagnostic in isolation, they help obtain standardized information
from both parents and teachers. Behavioral rating forms are used to document core symptoms of specific
behavioral disorders for an individual child compared with his or her peers. The Vanderibilt is an attention-
deficit/hyperactivity disorder (ADHD)-specific rating form for children in the elementary school years that
also provides information about comorbid symptoms such as oppositional defiant disorder. However,
there are no normative data for adolescents. The Conners is another ADHD-specific rating scale that has
normative data for ages 6 to 18 years; a separate version is valid for children 2 to 6 years of age.

American Board of Pediatrics Content Specification(s):


Understand the role of rating scales and questionnaires in the assessment of disruptive behaviors
(eg, Vanderbilt, Conners)

Suggested Reading:
Committee on Quality Improvement, Subcommittee on Attention-Deficit/Hyperactivity Disorder. Clinical
practice guideline: diagnosis and evaluation of the child with attention-deficit/hyperactivity disorder.
Pediatrics. 2000;105:1158-1170. Available at:
http://pediatrics.aappublications.org/cgi/content/full/105/5/1158

Wilms Floet AM. Scheiner, C. Grossman L. Attention-deficit/hyperactivity disorder. Pediatr Rev.


2010;31:56-69 DOI: 10.1542/pir.31-2-56. Available at:
http://pedsinreview.aappublications.org/cgi/content/full/31/2/56

Copyright 2012 American Academy of Pediatrics


2012 PREP SA ON CD-ROM

Question 117
A 3-year-old girl is admitted to the intensive care unit with the acute onset of fever, altered mental
status, and nuchal rigidity. She, her 1-year old brother, and her 4-year-old sister have not received
immunizations because of parental religious objection. Cerebrospinal fluid evaluation reveals a white
3 9
blood cell count of 2.45x10 /mcL (2.45x10 /L) with 98% polymorphonuclear neutrophils, glucose of 3
mg/dL (0.2 mmol/L), and protein of 266 mg/dL (2.7 g/L). Culture results are pending.
Of the following, the culture result that MOST strongly indicates the need for chemoprophylaxis
for this childs siblings is
A. enterovirus
B. Haemophilus influenzae type b
C. nontypeable Haemophilus influenzae
D. Streptococcus pneumoniae
E. West Nile virus

Copyright 2012 American Academy of Pediatrics


2012 PREP SA ON CD-ROM

Critique 117 Preferred Response: B


Antimicrobial chemoprophylaxis with rifampin is indicated for all household contacts in cases of
invasive Haemophilus influenzae type b (Hib) infection if there is at least one contact younger than 4
years of age who is incompletely immunized or unimmunized. In addition, prophylaxis is indicated if there
is a child in the household who is younger than 12 months of age and has not completed the primary Hib
vaccine series or if there is an immunocompromised child (regardless of immunization status).
Accordingly, the children (as well as the parents) described in the vignette require chemoprophylaxis for
Hib infection. Similar recommendations for antibiotic prophylaxis apply to nursery school and child-care
center contacts when two or more cases of invasive Hib disease have occurred within 60 days.
During a recent shortage of Hib vaccine, an outbreak of Hib disease occurred in Minnesota in
which five children younger than 5 years of age developed invasive Hib disease and one child died. Three
of the five children were unvaccinated because of parental refusal and the other two had received only
two doses of vaccine. One of the latter two children was also found to have hypogammaglobulinemia. A
similar outbreak occurred in Pennsylvania during this shortage.
An increased risk for disease or benefit from chemoprophylaxis has not been demonstrated for
Streptococcus pneumoniae or nontypeable H influenzae infections. Chemoprophylaxis does not have any
role in prevention of viral infections such as those caused by enterovirus or West Nile virus.
Antibiotic prophylaxis has also been shown to be of benefit in preventing person-to-person
transmission of Neisseria meningitidis infection, although the rifampin dosing schedule for meningococcal
disease is different from that for Hib infection (two doses per day for 2 days versus one dose per day for 4
days).

American Board of Pediatrics Content Specification(s):


Know the indications for the use of prophylactic antibiotics for control of diseases that spread from
person to person (eg, H. influenzae infections)

Suggested Reading:
American Academy of Pediatrics. Haemophilus influenzae infections. In: Pickering LK, Baker CJ,
Kimberlin DW, Long SS, eds. Red Book: 2009 Report of the Committee on Infectious Diseases. 28th ed.
Elk Grove Village, IL: American Academy of Pediatrics; 2009:314321

Centers for Disease Control and Prevention. Invasive Haemophilus influenzae type B disease in five
young children Minnesota, 2008. MMWR Morbid Mortal Wkly Rep. 2009;58:5860. Available at:
http://www.cdc.gov/mmwr/preview/mmwrhtml/mm5803a4.htm

Copyright 2012 American Academy of Pediatrics


2012 PREP SA ON CD-ROM

Question 118
A healthy term newborn has done well in the nursery and is ready for discharge. While speaking
with the new mother, you learn that her brother has a coughing illness accompanied by fever, weight loss,
and hemoptysis. He often stays at his sisters home. No other family members, including the infants
mother, are ill.
Of the following, the next BEST management step for the newborn is to
A. isolate the infant and the mother
B. obtain a chest radiograph
C. place a tuberculin skin test
D. separate the infant from the uncle
E. start isoniazid therapy

Copyright 2012 American Academy of Pediatrics


2012 PREP SA ON CD-ROM

Critique 118 Preferred Response: D


The infant described in the vignette, whose maternal uncle may have active tuberculosis, should
be separated from the uncle. Cases of suspected or proven tuberculosis require immediate reporting to
the local health department to initiate a contact investigation. If the infants mother is asymptomatic and
tuberculosis is not suspected, separating the infant from the mother is not necessary. An interferon-
gamma release assay (IGRA) or a tuberculin skin test (TST) should be performed on the mother, and if
results are positive, a chest radiograph is necessary. Because the infant has never been in contact with
the uncle, it is unnecessary to place a TST or obtain a chest radiograph for the infant unless evaluation of
the mother raises concerns for tuberculosis. In addition, the infant does not require isoniazid therapy.
If tuberculosis is suspected in the mother or in a household member with whom the infant has
been in contact, the infant should be separated from the mother (or household contact) until the mother
(or household contact) has been evaluated and the mother (contact) and infant are receiving appropriate
antituberculosis therapy. The mother also must comply with proper infection control measures, including
wearing a mask. Infants of mothers who have tuberculosis at the time of delivery require an evaluation for
congenital infection, including a TST, chest radiograph, and lumbar puncture for cerebrospinal fluid
analysis that involves stain and culture for acid-fast bacilli. IGRAs should not be used in infants.
Congenital infection is rare and is unlikely in cases of maternal pulmonary disease but can occur with
maternal bacillary tuberculosis. Because TSTs often are negative in infants who have congenital or
perinatally acquired tuberculosis, antituberculosis therapy (isoniazid, rifampin, pyrazinamide, amikacin)
should be initiated in all infants undergoing evaluation for congenital infection. If congenital infection is
excluded, exposed infants should receive isoniazid until a repeat TST is performed at 3 to 4 months of
age. If the TST is positive, the infant should undergo repeat evaluation for tuberculosis disease. If the
TST is negative and the adult contact who has tuberculosis is culture-negative (ie, not contagious) and
has good antituberculosis medication compliance, isoniazid can be discontinued.

American Board of Pediatrics Content Specification(s):


Plan the management of a child who has an adult household contact with active tuberculosis

Suggested Reading:
American Academy of Pediatrics. Tuberculosis. In: Pickering LK, Baker CJ, Kimberlin DW, Long SS, eds.
Red Book: 2009 Report of the Committee on Infectious Diseases. 28th ed. Elk Grove Village, IL:
American Academy of Pediatrics; 2009:680-701

Cruz AT, Starke JR. Pediatric tuberculosis. Pediatr Rev. 2010;31:13-26. DOI: 10.1542/pir.31-1-13.
Available at: http://pedsinreview.aappublications.org/cgi/content/full/31/1/13

Copyright 2012 American Academy of Pediatrics


2012 PREP SA ON CD-ROM

Question 119
You are examining a newborn who has wrinkling of the abdominal wall skin. His mother recalls
her obstetrician mentioning that her fluid was low. The infant was born at 37 weeks gestation, and his
birthweight was appropriate for gestational age.
Of the following, the MOST likely additional findings expected in this infant are cryptorchidism and
A. bilateral hydronephrosis
B. hypospadias
C. nephrocalcinosis
D. polycystic kidney disease
E. unilateral renal agenesis

Copyright 2012 American Academy of Pediatrics


2012 PREP SA ON CD-ROM

Critique 119 Preferred Response: A


The infant described in the vignette has wrinkling of the anterior abdominal wall. Coupled with the
maternal history of apparent oligohydramnios, this suggests possibility of obstructive uropathy due to
prune-belly syndrome (PBS) (Eagle-Barrett syndrome). PBS is characterized by the triad of bilateral
hydroureteral nephrosis, absence of abdominal wall musculature, and undescended testicles. Although
associated with renal dysplasia, the syndrome does not feature renal agenesis, nephrocalcinosis, cystic
renal disease, or urethral abnormalities such as hypospadias.
PBS has an estimated incidence of 1 in 50,000. It occurs almost exclusively in males, and
although its pathogenesis is unknown, one hypothesis suggests intrauterine bladder outlet obstruction
that results in bilateral hydronephrosis, thereby leading to interference with descent of the testicles from
the abdominal cavity into the scrotum and hindered development of the anterior abdominal wall
musculature. A genetic basis is speculated because the condition has been reported in two monozygotic
twins, but its exact inheritance is unknown. Because of the significant clinical manifestations of this
syndrome, affected children often require a multitude of surgeries to correct the anomalies. From a
urinary tract perspective, patients may require a form of urinary diversion to relieve the bladder outlet
obstruction. Unlike posterior urethral valves, which often are treated successfully with a fulguration of the
valves, PBS usually requires a vesicostomy to facilitate bladder emptying and decompression of the
upper urinary tracts to relieve the hydronephrosis, thereby lowering the high back pressure placed upon
the kidneys and reducing the likelihood of urinary tract infections, which can cause further renal damage.
Overall, the prognosis for this disorder is poor and usually dictated by the renal prognosis.
Approximately 30% of affected patients eventually develop end-stage renal disease. Bilateral renal
involvement, nadir serum creatinine greater than 0.7 mg/dL (61.9 mcmol/L) at 1 year of age, and clinical
pyelonephritis were statistically significantly associated with the development of renal failure in a large
retrospective case series. Renal replacement therapy, including hemodialysis, peritoneal dialysis, and
transplantation, has been used successfully in children who have PBS.
Bilateral orchiopexy should be performed in early childhood. One study found that 93% of
children who had PBS and underwent early childhood orchiopexy were able to achieve sufficient
testicular function to achieve normal pubertal onset in adolescence and normal sexual function in
adulthood. Reconstructive surgery of the anterior abdominal wall (abdominoplasty) is often performed for
both cosmetic and functional purposes. This procedure can result in improved bladder emptying and
bowel function (constipation is another feature of the disease). Overall, mortality rates have improved
from 50% in historical reports to approximately 30% at present.

American Board of Pediatrics Content Specification(s):


Know the urologic findings associated with prune-belly (Eagle-Barrett) syndrome

Suggested Reading:
Chevalier RL. Obstructive uropathy. In: Kher KK, Schnaper HW, Makker SP, eds. Clinical Pediatric
Nephrology. 2nd ed. London, England: Informa Healthcare; 2007:507-518

Noh PH, Cooper CS, Winkler AC, Zderic SA, Snyder HM 3rd, Canning DA. Prognostic factors for long-
term renal function in boys with the prune-belly syndrome. J Urol. 1999;162:1399-1401. Abstract available
at: http://www.ncbi.nlm.nih.gov/pubmed/10492223

Patil KK, Duffy PG, Woodhouse CR, Ransley PG. Long-term outcome of Fowler-Stephens orchiopexy in
boys with prune-belly syndrome. J Urol. 2004;171:1666-1669. DOI: 10.1097/01.ju.0000118139.28229.f5.
Abstract available at: http://www.ncbi.nlm.nih.gov/pubmed/15017263

Wallner M, Kramer R. Prune-belly syndrome. UpToDate Online 18.3. 2010. Available for subscription at:
http://www.uptodate.com/online/content/topic.do?topicKey=neonatol/24647&selectedTitle=1%7E15&sour
ce=search_result#

Copyright 2012 American Academy of Pediatrics


2012 PREP SA ON CD-ROM

Question 120
You are scheduled to see an 11-year-old girl for a health supervision visit. In the past she was
seen by your partner who recently retired. Her medical records reveal that she has been taking
risperidone, clonidine, and methylphenidate for almost 2 years. You notice that her weight has increased
over the past year. Her mother states that her daughter has been in good health and has not yet
achieved menarche. On physical examination, you note Sexual Maturity Rating (SMR) 3 pubic hair and
breast tissue and mild acne. Her body mass index (BMI) is elevated and pulse and blood pressure are
normal. Her review of systems is negative for abnormal involuntary movements, muscle stiffness, fainting
or shortness of breath. There is no family history of cardiac disease or sudden unexplained death.
Of the following, the MOST appropriate tests to obtain for a patient taking these medications are
A. fasting glucose, liver enzymes, and cholesterol/lipid profile
B. fasting glucose, thyroxine, and electrocardiogram
C. prolactin, fasting glucose, and cholesterol/lipid profile
D. risperidone concentration, liver enzymes, and fasting glucose
E. thyroxine, lipid profile, and serum creatinine

Copyright 2012 American Academy of Pediatrics


2012 PREP SA ON CD-ROM

Critique 120 Preferred Response: A


Of the three medications prescribed for the girl described in the vignette, only risperidone requires
routine laboratory evaluation to monitor for potential adverse effects. Risperidone belongs to the atypical
antipsychotic family of medications, which are commonly associated with weight gain in children and, not
infrequently, the development of abnormalities in glucose and lipid metabolism and high blood pressure,
together known as the metabolic syndrome. Guidelines for use of atypical antipsychotics advise
monitoring height, weight, lifestyle behaviors, and sedation at each visit. Fasting glucose and lipids should
be assessed every 6 months. Assessment for parkinsonian adverse effects and akathisia also should be
undertaken annually. Once a year, if significant weight gain has occurred, as in this girl, liver function
should be assessed. Unless there is clinical indication, a complete blood count with differential count and
complete metabolic panel are not necessary.
Thyroxine would be appropriate to monitor along with thyroid-stimulating hormone in patients who
are taking lithium, but such assessment is not recommended for those prescribed atypical antipsychotics.
Also if one suspected hypothyroidism as causing weight gain, a thyroxine level alone is difficult to
interpret without pairing the result to a simultaneous thyroid stimulating hormone level. Risperidone use is
frequently associated with increased prolactin concentrations, but rarely are there clinical manifestations.
Prolactin concentrations need to be evaluated only when symptoms are present. Age, sex, and SMR
stage-appropriate questions about sexual and reproductive function should be part of each medication
management visit. Because this girl has not yet achieved menarche, has normal SMR findings for age,
and has no complaints of breast discharge, her prolactin does not need to be assessed. A serum
creatinine is not indicated since risperidone, clonidine, and methylphenidate are not associated with renal
toxicity. An electrocardiogram would be recommended for a child taking stimulants with either a family
history of silent cardiac disorders like long QT syndrome or symptoms suggestive of a cardiac problem
(unexplained fainting, shortness of breath, abnormal vital signs), but it is not a test recommended for
routine stimulant monitoring.
Although evaluations to monitor risperidone are available, most laboratories do not perform them
locally and must send out specimens for assessment. If a question of compliance cannot be answered
reliably by any other means, measuring risperidone concentrations can be helpful.

AAP Mental Health Competency:


Identify the drug monitoring practices needed if one is the prescriber of atypical antipsychotics

Suggested Readings:
American Academy of Pediatrics Task Force on Mental Health. Anxiety. Addressing Mental Health
Concerns in Primary Care: A Clinician's Toolkit [CD-ROM]. Elk Grove Village, IL: American Academy of
Pediatrics; 2010. Available at: http://pediatrics.aappublications.org/content/122/2/451.full.pdf

Correll, CU. Antipsychotic medications. In: Dulcan MK, ed. Dulcans Textbook of Child and Adolescent
Psychiatry. Arlington, VA: American Psychiatric Publishing, Inc; 2010:743-774

Correll CU. Antipsychotic use in children and adolescents: minimizing adverse effects to maximize
outcomes. J Am Acad Child Adolesc Psychiatry. 2008;47:9-20. DOI: 10.1097/chi.0b013e31815b5cb1.
Abstract available at: http://www.jaacap.com/article/S0890-8567(09)62080-3/abstract

Correll CU, Carlson HE. Endocrine and metabolic adverse effects of psychotropic medications in children
and adolescents. J Am Acad Child Adolesc Psychiatry. 2006;45:771-791. DOI:
10.1097/01.chi.0000220851.94392.30. Abstract available at: http://www.jaacap.com/article/S0890-
8567(09)61524-0/abstract

Perrin JP, Friedman RA, Knilans TK, et al., American Academy of Pediatrics. Policy Statement:
Cardiovascular Monitoring and Stimulant Drugs for Attention-Deficit/Hyperactivity Disorder. Pediatrics.
2008;122:451-453. DOI: 10.1542/peds.2008-1573. Available at:
http://pediatrics.aappublications.org/content/122/2/451

Copyright 2012 American Academy of Pediatrics


2012 PREP SA ON CD-ROM

Question 121
A-14 year-old girl presents with a 4-year history of recurrent infections. Her parents state that it
seems she is on antibiotics almost every other month for the treatment of otitis media, sinusitis, or
pneumonia. During a recent hospitalization for lobar pneumonia, the inpatient team measured serum
immunoglobulins (Igs), which showed:
Low IgG of 54 mg/dL (0.54 g/L) (normal range, 700 to 1,500 mg/dL [7 to 15 g/L])
Absent IgA at <7.5 mg/dL (75 mg/L) (normal range, 15 to 200 mg/dL [150 to 2,000 mg/L])
Low IgM of 10 mg/dL (100 mg/L) (normal range, 50 to 300 mg/dL [500 to 3,000 mg/L])
Despite the recurrent infections, the girl is otherwise growing and developing appropriately and
has no other specific medical concerns.
Of the following, the MOST appropriate next laboratory test is
A. flow cytometry for B lymphocytes, T lymphocytes, and natural killer cells
B. genetic analysis for mutations of the Bruton tyrosine kinase (Btk) gene
C. lymphocyte proliferation assay of peripheral blood mononuclear cells to mitogens
D. measurement of antibody responses to protein and polysaccharide vaccines
E. measurement of IgG subclasses (IgG1, IgG2, IgG3, IgG4)

Copyright 2012 American Academy of Pediatrics


2012 PREP SA ON CD-ROM

Critique 121 Preferred Response: D


An adolescent or young adult who has recurring infections and hypogammaglobulinemia, such as
the girl described in this vignette, should be evaluated for common variable immunodeficiency (CVID).
The diagnosis of CVID requires three criteria: a decrease of more than 2 standard deviations of one
immunoglobulin below the age-adjusted mean (usually IgG, with decreased IgA or IgM), poor antibody
response to protein (eg, diphtheria tetanus toxoid) and polysaccharide vaccines (eg, 23-valent
pneumococcal vaccine), and exclusion of other causes of hypogammaglobulinemia (Item C121). Because
the girl already meets the first criterion, the next step is to measure baseline antibody titers, vaccinate
her, and repeat antibody measurements in 3 to 4 weeks. The recommended appropriate protein vaccine
response in adolescents consists of a fourfold increase in titers. An appropriate pneumococcal
polysaccharide response in patients 5 years and older is a titer of 1.3 g/mL or higher in 70% of
pneumococcal serotypes. Patients who demonstrate both hypogammaglobulinemia and impaired vaccine
response should be referred to an immunologist for additional testing and consideration for either
intravenous or subcutaneous immunoglobulin replacement therapy.
Further testing that is usually performed by the immunologist includes flow cytometry,
consideration of genetic analysis for mutations in the Bruton tyrosine kinase (Btk) gene, lymphocyte
proliferation assay, and assessment of IgG subclasses. Flow cytometry uses technology that can detect
specific cell surface markers of T cells (CD3, CD4, CD8), B cells (CD19), and natural killer (NK) cells
(CD16, CD56). Most patients who have CVID have normal B, T, and NK cell numbers, although up to
10% can have low B cell numbers, and many patients have an inverted CD4/CD8 ratio. Although flow
cytometry is important, the results are not part of the current laboratory criteria for diagnosing CVID.
Patients who have low-to-absent B-cell concentrations or who are younger than 2 years of age should be
evaluated for X-linked recessive (Bruton) agammaglobulinemia. Flow cytometry screening detects most
patients who have complete or partial expression of the Btk protein, but up to 30% of patients have
abnormal function that can only be detected by Btk gene sequencing.
Additional T-cell qualitative analysis can include both anergy testing (eg, delayed hypersensitivity
testing to Candida, tetanus, mumps, Trichophyton) and lymphocyte proliferation assay of peripheral blood
mononuclear cells. Mitogen proliferation testing is expensive, typically yields normal results in patients
who have CVID, and should be performed under the direction of an immunologist.
Many clinicians assess IgG subclasses (ie, IgG1, IgG2, IgG3, and IgG4), but the clinical
significance of a low IgG subclass value is unclear. Although low IgG2 concentrations have been
associated with a poor polysaccharide response and low IgA concentrations in some patients, low values
for one or more IgG subclasses is currently not recognized as a specific primary immunodeficiency, and
the use of immunoglobulin replacement is controversial.

American Board of Pediatrics Content Specification(s):


Plan the laboratory evaluation of antibody function (quantitative immunoglobulin concentrations,
specific antibody to responses to both protein and polysaccharide vaccines)

Suggested Reading:
Fleisher TA. Back to basics: primary immune deficiencies: windows into the immune system. Pediat Rev.
2006;27:363-372. DOI: 10.1542/pir.27-10-363. Available at:
http://pedsinreview.aappublications.org/cgi/content/full/27/10/363

Geha RS, Notarangelo LD, Casanova J, et al; International Union of Immunological Societies Primary
Immunodeficiency Diseases Classification Committee. Primary immunodeficiency diseases: an update
from the International Union of Immunological Societies Primary Immunodeficiency Diseases
Classification Committee. J Allergy Clin Immunol. 2007;120:776794. DOI: 10.1016/j.jaci.2007.08.053.
Available at: http://www.ncbi.nlm.nih.gov/pmc/articles/PMC2601718/?tool=pubmed

Copyright 2012 American Academy of Pediatrics


2012 PREP SA ON CD-ROM

Critique 121

Item C121. Differential Diagnosis of Hypogammaglobulinemia


Medications Glucocorticoids, captopril, antimalarial medication, phenytoin,
sulfasalazine
Genetic disorders CVID, severe combined immunodeficiency, X-linked
agammaglobulinemia, trisomy 21, hyper-IgM syndrome
Infection Human immunodeficiency virus, congenital rubella, Epstein-Barr virus
Malignancy Chronic lymphocytic leukemia, non-Hodgkin lymphoma, B-cell
malignancies
Systemic disorders Malnutrition, hypercatabolism, nephrotic syndrome, severe diarrhea,
severe burns

Copyright 2011 American Academy of Pediatrics


2012 PREP SA ON CD-ROM

Question 122
A 2-year-old boy is brought to the emergency department after his mother found him with an open
bottle of toilet bowl cleaner. She reports that he had spilled some on his shirt and had some on his face,
but she does not know if he drank any of it. The child is awake and alert, and his vital signs are normal.
He is drooling slightly, but examination of his oropharynx reveals no lesions.
Of the following, the MOST appropriate next step is to
A. administer activated charcoal
B. administer syrup of ipecac
C. perform gastric lavage
D. provide no further treatment
E. refer the boy to a gastroenterologist for urgent endoscopy

Copyright 2012 American Academy of Pediatrics


2012 PREP SA ON CD-ROM

Critique 122 Preferred Response: E


Ingestion of a caustic substance causes injury to mucosal and skin surfaces by liquefaction
necrosis in alkali exposures and by protein coagulation in acid exposures. Substances at the extremes of
the pH scale (<2 and >12) are especially damaging. Patients typically present with drooling, dysphagia,
odynophagia, and in many cases, intraoral burns. In addition, they may have vomiting with hematemesis;
respiratory distress with stridor or wheezing; and burns on the face, hands, or chest. Because the primary
mode of injury is direct tissue corrosion and systemic symptoms are rare, decontamination of patients
following caustic ingestions is focused on washing the skin and flushing the eyes, if indicated. Use of
activated charcoal is not indicated because it can make subsequent endoscopic evaluation of the
esophagus difficult. Syrup of ipecac and gastric lavage are contraindicated because of potential aspiration
risk. Further, gastric lavage carries the risk of esophageal perforation.
The major clinical concerns with a caustic ingestion are airway or esophageal injury. Severe
gastritis, perforation, or late stricture formation also may result from significant ingestions. Early airway
visualization and protection are indicated in any patient who presents with stridor or respiratory distress,
and evaluation of the esophagus by upper endoscopy is indicated in patients who have intraoral burns or
other symptoms. In addition, some asymptomatic patients, such as the boy in the vignette, should be
considered endoscopy candidates, based on history or other clinical concerns. As many as 45% of
patients who do not have oral burns and 12% of asymptomatic patients have findings on endoscopy. The
need for endoscopy in the asymptomatic patient in whom a significant caustic ingestion is questionable is
controversial.

American Board of Pediatrics Content Specification(s):


Recognize that gastric lavage is contraindicated in a caustic ingestion
Recognize that corrosive material such as hydrochloric and sulfuric acids can be transported to the
stomach with few or no esophageal burns, causing severe gastritis, perforation, or late stricture
formation

Suggested Reading:
Ferry GD. Caustic esophageal injury in children UpToDate Online 18.3. 2010 Available at:
http://www.uptodate.com/online/content/topic.do?topicKey=pedigast/11441

Kardon EM. Toxicity, caustic ingestions. eMedicine Specialties, Emergency Medicine, Toxicity. 2010.
Available at: http://emedicine.medscape.com/article/813772-overview

ODonnell KA, Burns Ewald M. Pediatric drug therapy: poisonings. In: Kleigman RM, Stanton BF, St.
Geme JW III, Schor NF, and Behrman RE, eds. Nelson Textbook of Pediatrics. 19th ed. Philadelphia, PA:
Saunders Elsevier; 2011:250-270

Copyright 2012 American Academy of Pediatrics


2012 PREP SA ON CD-ROM

Question 123
You are caring for an 18-year-old girl who has a presumptive diagnosis of cyclic vomiting
syndrome. Extensive gastroenterologic and neurologic evaluations have failed to establish an alternative
diagnosis. You began prophylactic treatment with amitriptyline several months ago, and the frequency of
vomiting episodes has decreased from every 4 to 6 weeks to every 8 to 10 weeks. During these
episodes, vomiting usually persists for 48 to 72 hours and has required hospitalization for intravenous
hydration on two occasions in the past year. The girls parents want to know if any treatment is available
to abort these episodes at home.
Of the following, the best available evidence suggests that the MOST appropriate treatment is
A. cyproheptadine
B. erythromycin
C. ondansetron
D. propranolol
E. sumatriptan

Copyright 2012 American Academy of Pediatrics


2012 PREP SA ON CD-ROM

Critique 123 Preferred Response: E


A 5-HT1B/1D receptor agonist such as sumatriptan may be an effective abortive agent for the cyclic
vomiting syndrome (CVS) experienced by the girl described in the vignette. The agents actions occur via
central nervous system inhibition of serotonin release. Open-label reports have shown that when
administered early in the symptomatic course, sumatriptan can abort a CVS episode in both adults and
children. Triptans also are widely used in the management of classic migraine, suggesting that the two
disorders may share pathogenic similarities. Sumatriptan is recommended as an abortive intervention in
CVS for children 12 years of age and older who have infrequent (ie, <1/month) or mild vomiting episodes.
This recommendation follows the consensus statement of the North American Society for Pediatric
Gastroenterology, Hepatology, and Nutrition (NASPGHN), although the drug is not currently approved for
use in children younger than 18 years of age.
In contrast to the triptans, the 5-HT3 receptor antagonists (STRAs), such as ondansetron, are
considered supportive (as opposed to abortive) antiemetic agents. This class of drugs has been used
extensively and with excellent effect for patients undergoing chemotherapy and now is recommended as
first-line supportive therapy during CVS attacks. Ondansetron is well tolerated in doses ranging from 0.3
to 0.4 mg/kg (up to 20 mg/dose). Because they are effective in both oral and intravenous forms, the
STRAs are used extensively in both the ambulatory and inpatient settings. A detailed approach to the
inpatient management of CVS is provided in the NASPGHN consensus statement.
Another drug that has been extensively used in CVS is the antihistamine and STRA
cyproheptadine. This agent has been found to be effective in the baseline (ie, prophylactic)
management of CVS in children younger than 5 years of age. However, it is not generally recommended
for use in school-age children because of its effect on increasing appetite, resulting in unacceptable
weight gain. The beta-blocker propranolol is recommended as second-line therapy (after amitriptyline) for
preventing CVS attacks in children younger than 5 years of age. Effectiveness of the so-called prokinetic
drugs has not been demonstrated in CVS, except in isolated case reports, and current best evidence
does not support their utility. For example, erythromycin, an antibiotic that acts as a motilin agonist when
administered in subtherapeutic doses, has demonstrated efficacy in patients who have delayed gastric
emptying and feeding difficulties, especially in preterm infants. Although erythromycin was reported to be
effective in one patient who had CVS, its routine use in this disorder requires further study.

American Board of Pediatrics Content Specification(s):


Know the role of serotonin receptor antagonists in preventing and treating vomiting

Suggested Reading:
Anderson JM, Sugerman KS, Lockhart JR, Weinberg WA. Effective prophylactic therapy for cyclic
vomiting syndrome in children using amitriptyline or cyproheptadine. Pediatrics. 1997;100:977981.
Available at: http://pediatrics.aappublications.org/cgi/content/full/100/6/977

Benson JM, Zorn SL, Book LS. Sumatriptan in the treatment of cyclic vomiting. Ann Pharmacother.
1995;29:997999. Abstract available at: http://www.ncbi.nlm.nih.gov/pubmed/8845562

Li BU, Balint JP. Cyclic vomiting syndrome: evolution in our understanding of a brain-gut disorder. Adv
Pediatr. 2000;47:117-160. Abstract available at: http://www.ncbi.nlm.nih.gov/pubmed/10959442

Li BU, Lefevre F, Chelimsky GG, et al; North American Society for Pediatric Gastroenterology,
Hepatology, and Nutrition. North American Society for Pediatric Gastroenterology, Hepatology, and
Nutrition consensus statement on the diagnosis and management of cyclic vomiting syndrome. J Pediatr
Gastroenterol Nutr. 2008;47:379-393. DOI: 10.1097/MPG.0b013e318173ed39. Available at:
http://journals.lww.com/jpgn/Fulltext/2008/09000/North_American_Society_for_Pediatric.22.aspx

Li BU, Misiewicz L. Cyclic vomiting syndrome: a brain-gut disorder. Gastroenterol Clin North Am.
2003;32:997-1019. Abstract available at: http://www.ncbi.nlm.nih.gov/pubmed/14562585

Copyright 2012 American Academy of Pediatrics


2012 PREP SA ON CD-ROM

Vanderhoof JA, Young R, Kaufmann SS, Ernst L. Treatment of cyclic vomiting syndrome in childhood
with erythromycin. J Pediatr Gastroenterol Nutr. 1993;17:387391. Abstract available at:
http://www.ncbi.nlm.nih.gov/pubmed/8145093

Copyright 2012 American Academy of Pediatrics


2012 PREP SA ON CD-ROM

Question 124
You are called to attend the delivery of a preterm infant at 32 weeks gestational age. The mother
presented with preterm labor 12 hours ago. Tocolytic therapy and the administration of antenatal
corticosteroids were begun at that time. Due to unknown group B Streptococcus status, the mother also
was begun on ampicillin. Labor has now progressed to imminent vaginal delivery.
Of the following, the tocolytic agent MOST likely to affect the neonatal resuscitation of this infant
is
A. indomethacin
B. magnesium sulfate
C. nifedipine
D. ritodrine
E. terbutaline

Copyright 2012 American Academy of Pediatrics


2012 PREP SA ON CD-ROM

Critique 124 Preferred Response: B


The use of magnesium sulfate in labor can be associated with respiratory depression requiring
resuscitation in the infant. Affected infants manifest apnea at birth that responds to bag-mask ventilation
with prompt return of heart rate and color. However, respiratory effort, reflex irritability, and tone often
remain diminished. Some infants may require endotracheal intubation due to marked delay in
spontaneous respiratory drive.
Historically, magnesium sulfate has been used as a tocolytic agent as well as in the management
of severe preeclampsia because of its role in preventing maternal seizure activity. Antenatal magnesium
sulfate is now believed to have a role in improving neonatal neurodevelopmental outcome when
administered to women at risk for preterm delivery. Maternal serum concentrations greater than 6 mg/dL
are associated with both decreased fetal breathing movements and heart rate reactivity. Although clinical
symptoms correlate poorly with neonatal serum magnesium concentrations, affected infants may
demonstrate lethargy, poor feeding, decreased bowel motility, hypotonia, hypoventilation, and apnea.
Recovery often takes several hours to days and is dependent on urinary clearance of the magnesium.
Beta-adrenergic agents such as terbutaline and ritodrine may be used for short-term tocolysis to
allow completion of antenatal corticosteroid therapy. Transient hypoglycemia may be seen after delivery,
but there is no impact on neonatal resuscitation. Nifedipine, a calcium channel blocker, has been used
both as a short- and long-term tocolytic agent and has no described effects on the newborn.
Prostaglandin inhibitors such as indomethacin have been demonstrated to be effective tocolytic agents,
but they are associated with the risk of premature closure of the ductus arteriosus in utero when used
after 32 weeks of gestation for more than 48 to 72 hours. Premature ductal closure and associated
pulmonary hypertension may develop after lengthy in utero exposure to indomethacin, with the infant
emerging cyanotic and requiring vigorous resuscitation after birth.

American Board of Pediatrics Content Specification(s):


Understand the effects of drugs given to the mother during labor (eg opiates, beta adrenergic tocolytic
agents) on the fetus/neonate

Suggested Reading:
Benitz WE, Druzin ML. Pharmacology review: drugs that effect neonatal resuscitation. NeoReviews.
2005;6:e189-e195. DOI: 10.1542/neo.6-4-e189. Available at:
http://neoreviews.aappublications.org/cgi/content/full/6/4/e189

Doyle LW, Crowther CA, Middleton P, Marret S, Rouse D. Magnesium sulphate for women at risk of
preterm birth for neuroprotection of the fetus. Cochrane Database Syst Rev. 2009;1: CD004661. DOI:
10.1002/14651858.CD004661.pub3. Available at:
http://onlinelibrary.wiley.com/o/cochrane/clsysrev/articles/CD004661/frame.html

Haas DM, Imperiale TF, Kirkpatrick PR, Klein W, Zollinger TW, Golichowski AL. Tocolytic therapy: a
meta-analysis and decision analysis. Obstet Gynecol. 2009;113:585-594. DOI:
10.1097/AOG.0b013e318199924a. Available at:
http://journals.lww.com/greenjournal/Fulltext/2009/03000/Tocolytic_Therapy__A_Meta_Analysis_and_De
cision.4.aspx

Copyright 2012 American Academy of Pediatrics


2012 PREP SA ON CD-ROM

Question 125
The parents of a 22-month-old boy are concerned because he appears pale. Approximately 3
weeks ago he had a viral illness with fever and respiratory symptoms, from which he recovered in 6 days.
Since then, he has had no fever, rashes, or complaints of discomfort. His appetite has been fair and
activity level has decreased minimally. On physical examination, the generally well-appearing but
somewhat pale boy has a temperature of 37.2C, heart rate of 132 beats/min, and respiratory rate of 20
breaths/min. He has no lymphadenopathy, and his liver and spleen are not enlarged. Laboratory findings
are:
Hemoglobin, 6.8 g/dL (68 g/L)
3 9
White blood cell count, 7.2x10 /mcL (7.2x10 /L) with normal differential count
3 9
Platelet count, 402x10 /mcL (402x10 /L)
Mean corpuscular volume, 78 fL
Red cell distribution width, 11.5%
Reticulocyte count, 0.5% (0.005)
Lactate dehydrogenase and red blood cell adenosine deaminase, normal
Direct antiglobulin (Coombs) test, negative
Of the following, the MOST appropriate treatment for this patient at this time is
A. close observation
B. erythropoietin
C. iron
D. prednisone
E. red blood cell transfusion

Copyright 2012 American Academy of Pediatrics


2012 PREP SA ON CD-ROM

Critique 125 Preferred Response: A


The boy described in the vignette has the characteristic findings of transient erythroblastopenia of
childhood (TEC), an acquired, self-limited red blood cell aplasia that usually resolves in 1 to 2 months
without intervention. This condition can affect children from 6 months to 6 years of age, but it usually
occurs between 1 and 3 years. Children generally have no underlying medical conditions or unusual
findings, although about 50% have had a viral illness within the preceding 1 to 2 months. Based on a
number of reports of affected siblings and twins, many authors postulate a genetic susceptibility that is
triggered by a viral infection. The patients hemoglobin decreases, usually to the range of 6 to 8 g/dL (60
to 80 g/L), and, thus, the child may present with the gradual onset of pallor and mild fatigue. Additional
laboratory findings include an inappropriately depressed reticulocyte count, a normal-to-mildly elevated
platelet count, and sometimes mild neutropenia. The mean corpuscular volume (MCV) is typically normal
during the course of the disease but may increase during recovery as reticulocytosis occurs.
In addition to the normal MCV, TEC is distinguished from Diamond-Blackfan anemia (congenital
hypoplastic anemia) by the absence of dysmorphic features; older age of onset; and normal adenosine
deaminase, hemoglobin F, and i antigen values. Acquired aplastic anemia frequently involves other cell
lines; TEC typically is associated with only minimal changes to the platelet and leukocyte counts.
Leukemia can present with severe anemia, but the patient often has other signs of illness,
lymphadenopathy and hepatosplenomegaly, and blast cells on peripheral smear. Iron deficiency anemia
classically affects a similar age group to TEC but is distinguished by low MCV and elevated red cell
distribution width.
As noted previously, most children who have TEC do not require any treatment; 80% recover
within a month of diagnosis. They should be observed closely with serial blood and reticulocyte counts.
An increase in the reticulocyte count is a marker for recovery. Red blood cell transfusion is reserved for
cases of hemodynamic instability, significant exercise intolerance, or altered mental status. Corticosteroid
administration has been tried for this condition but does not speed recovery. Iron therapy is not warranted
because this is an immune-mediated aplasia rather than the result of a nutritional deficiency. Similarly,
TEC is not the result of decreased erythropoietin, so treatment with this agent is not indicated.

American Board of Pediatrics Content Specification(s):


Know the signs, symptoms and laboratory findings of transient erythroblastopenia of childhood
Understand the role of erythrocyte transfusions in transient erythroblastopenia of childhood

Suggested Reading:
Bomgaars L. Anemia in children due to decreased red blood cell production. UpToDate Online 18.3.
2010. Available for subscription at:
http://www.uptodate.com/online/content/topic.do?topicKey=pedi_hem/6539

Lerner N. Diseases of the blood: acquired pure red blood cell anemias. In: Kleigman RM, Stanton BF, St.
Geme JW III, Schor NF, and Behrman RE, eds. Nelson Textbook of Pediatrics. 19th ed. Philadelphia, PA:
Saunders Elsevier; 2011:1652-1653

Shaw J, Meeder R. Transient erythroblastopenia of childhood in siblings: case report and review of the
literature. J Pediatr Hematol Oncol. 2007;29:659-660. DOI: 10.1097/MPH.0b013e31814684e9. Abstract
available at: http://www.ncbi.nlm.nih.gov/pubmed/17805047

Skeppner G, Kreuger A, Elinder G. Transient erythroblastopenia of childhood: prospective study of 10


patients with special reference to viral infections. J Pediatr Hematol Oncol. 2002;24:294-298. Abstract
available at: http://www.ncbi.nlm.nih.gov/pubmed/11972099

Copyright 2012 American Academy of Pediatrics


2012 PREP SA ON CD-ROM

Question 126
A mother brings her 14-year-old daughter to see you because of hair loss of several weeks
duration. On physical examination, you see an area of relative alopecia located at the vertex within which
are hairs of varying lengths (Item Q126). There is no erythema or scaling of the scalp, and no black dot
hairs are apparent. During the evaluation, you note that the child seems very shy and that she bites her
fingernails.
Of the following, the MOST likely diagnosis is
A. alopecia areata
B. nevus sebaceus
C. tinea capitis
D. traction alopecia
E. trichotillomania

Copyright 2012 American Academy of Pediatrics


2012 PREP SA ON CD-ROM

Question 126

(Courtesy of D Krowchuk)

Copyright 2012 American Academy of Pediatrics


2012 PREP SA ON CD-ROM

Critique 126 Preferred Response: E


The girl described in the vignette is suffering from trichotillomania, which is defined as the
compulsive pulling and twisting of hair that results in hair breakage and irregular areas of hair loss. The
hair remaining in the areas of loss are classically of varying lengths. The scalp itself appears normal in
most cases, although excoriation from tearing at hair and mild chronic folliculitis from repeated trauma
may be seen. Treatment is directed at the underlying psychosocial causes, which may be anxiety or
depression.
Alopecia areata constitutes rapid and complete loss of hair from an area of the scalp, leaving a
smooth and normal skin appearance (Item C126A). In the United States, tinea capitis (Item C126B)
usually results from infection with Trichophyton tonsurans and is characterized by black dot hairs with
scaling, erythema, and chronic loss and regrowth of hair; inflammation may occur and kerion formation (a
boggy, inflammatory mass) is not uncommon. Oral systemic antifungal preparations and possibly
corticosteroids are used for treatment.
Nevus sebaceus generally presents early in infancy or childhood as a small yellowish plaque
(Item C126C) without hair in the scalp that may become more nodular in adolescence. Because of a
propensity to undergo malignant transformation to basal cell carcinoma, removal is recommended before
adolescence.
Traction alopecia usually results from hair styling techniques (eg, ponytail holders) that put
significant traction on the hair follicle, resulting in localized hair loss (Item C126D). Mild inflammation and
folliculitis with keratinized debris plugging the follicles and resultant local infection may occur.
Management involves advice to minimize traction and apply hair ornaments to alternate areas of the
scalp. Rarely, hair loss may be permanent due to scarring of follicles.

American Board of Pediatrics Content Specification(s):


Recognize that trichotillomania is characterized by the presence of hair shafts of different lengths in
the area of alopecia

Suggested Reading:
American Academy of Pediatrics. Tinea capitis (ringworm of the scalp). In: Pickering LK, Baker CJ,
Kimberlin DW, Long SS, eds. Red Book: 2009 Report of the Committee on Infectious Diseases. 28th ed.
Elk Grove Village, IL: American Academy of Pediatrics; 2009:661-662. Available at:
http://aapredbook.aappublications.org/cgi/content/full/2009/1/3.133

Bravender T. Index of suspicion: case 3. Pediatr Rev. 2000;21:354-357. DOI: 10.1542/pir.21-10-354.


Available at: http://pedsinreview.aappublications.org/cgi/content/full/21/10/354

Delamere FM, Sladden MJ, Dobbins HM, Leonardi-Bee J. Interventions for alopecia areata. Cochrane
Database Syst Rev. 2008;2:CD004413. DOI: 10.1002/14651858.CD004413.pub2. Available at:
http://onlinelibrary.wiley.com/o/cochrane/clsysrev/articles/CD004413/frame.html

Gonzlez U, Seaton T, Bergus G, Jacobson J, Martnez-Monzn C. Systemic antifungal therapy for tinea
capitis in children. Cochrane Database Syst Rev. 2007;4:CD004685. DOI:
10.1002/14651858.CD004685.pub2. Available at:
http://onlinelibrary.wiley.com/o/cochrane/clsysrev/articles/CD004685/frame.html

Morelli JG. The skin: diseases of the neonate. In: Kliegman RM, Stanton BF, St. Geme JW III, Schor NF,
and Behrman RE, eds. Nelson Textbook of Pediatrics. 19th ed. Philadelphia, PA: Saunders Elsevier;
2011:2289-2293

Tay Y-K, Levy ML, Metry DW. Trichotillomania in childhood: case series and review. Pediatrics.
2004;113:e494-e498. Available at: http://pediatrics.aappublications.org/cgi/content/full/113/5/e494

Copyright 2012 American Academy of Pediatrics


2012 PREP SA ON CD-ROM

Williams JV, Eichenfield LF, Burke BL, Barnes-Eley M, Friedlander SF. Prevalence of scalp scaling in
prepubertal children. Pediatrics. 2005;115:e1-e6. DOI: 10.1542/peds.2004-1616. Available at:
http://pediatrics.aappublications.org/cgi/content/full/115/1/e1

Copyright 2012 American Academy of Pediatrics


2012 PREP SA ON CD-ROM

Critique 126

(Courtesy of D Krowchuk)
Alopecia areata produces one or more patches of hair loss. The scalp appears normal, without scaling,
inflammation, or black dot hairs.

Copyright 2011 American Academy of Pediatrics


2012 PREP SA ON CD-ROM

Critique 126

(Courtesy of D Krowchuk)
Tinea capitis produces one or more patches of alopecia. In the most common form, black dot hairs
(yellow arrow) often are observed; they represent the remnants of broken hairs within follicles. When an
inflammatory response occurs, pustules may be present (red arrows).

Copyright 2011 American Academy of Pediatrics


2012 PREP SA ON CD-ROM

Critique 126

(Courtesy of D Krowchuk)
An orange, yellow, or tan hairless plaque is characteristic of nevus sebaceus.

Copyright 2011 American Academy of Pediatrics


2012 PREP SA ON CD-ROM

Critique 126

(Courtesy of A Nopper. Reproduced with permission from Krowchuk DP, Mancini AJ, eds. Pediatric
Dermatology. A Quick Reference Guide. 2nd ed. Elk Grove Village, IL: American Academy of Pediatrics;
2011)
Partial alopecia involves the hairline in this child, who has traction alopecia. Note the tightly-pulling braids
and multiple hair ornaments.

Copyright 2011 American Academy of Pediatrics


2012 PREP SA ON CD-ROM

Question 127
You are evaluating a 10-year-old boy for a rash and fatigue. He recently returned from visiting
family in South Africa, where he experienced an illness characterized by fever and sore throat 2 weeks
ago. On physical examination, he is afebrile and his heart rate is 100 beats/min, respiratory rate is 28
breaths/min, and blood pressure is 110/65 mm Hg. He has a macular, erythematous rash on his trunk
(Item Q127). In addition, you note a III/VI blowing systolic murmur at the apex as well as a II/VI long
diastolic murmur at the left lower sternal border. Twelve-lead electrocardiography reveals sinus rhythm
with first-degree atrioventricular block. Echocardiography documents valve dysfunction.
Of the following, the MOST likely explanation for this childs diastolic murmur is
A. aortic valve insufficiency
B. aortic valve stenosis
C. mitral valve insufficiency
D. mitral valve stenosis
E. pulmonary valve stenosis

Copyright 2012 American Academy of Pediatrics


2012 PREP SA ON CD-ROM

Question 127

(Courtesy of G Nankervis and the Red Book Online)


Rash, as described for the boy in the vignette.

Copyright 2012 American Academy of Pediatrics


2012 PREP SA ON CD-ROM

Critique 127 Preferred Response: A


The boy described in the vignette has two major (erythema marginatum and carditis) and one
minor (prolongation of the PR interval) criteria for the diagnosis of acute rheumatic fever (ARF) (Item
C127). In ARF, carditis (an inflammatory process that involves the myocardium, pericardium, and valves)
can lead to chronic cardiac conditions of mitral (and occasionally aortic) regurgitation and stenosis.
Carditis generally resolves with therapy, but many individuals who have ARF develop permanent damage
to the aortic and mitral valves. The valves become thickened, which can lead to both reduced valve
excursion (stenosis) and poor leaflet coaptation (regurgitation). The high-pitched diastolic murmur
originating at the left sternal border and radiating to the apex described for the boy in the vignette is
consistent with aortic insufficiency. The insufficient jet occurs during diastole, when the left ventricle is in
its relaxed phase, and has a high pitch because of the high-pressure gradient from the high-pressure
aorta to the low-pressure relaxed left ventricle. This child also has a high-pitched blowing systolic murmur
at the apex that radiates to the back, which is consistent with mitral insufficiency. Mitral insufficiency
occurs at the onset of left ventricular systole. The flow is directed to the posteriorly positioned left atrium
(hence, the radiation of the murmur to the back) and is of high pitch due to the high-pressure gradient
between the high-pressure left ventricle during systole and the low-pressure left atrium.
Aortic valve stenosis leads to a systolic ejection murmur, which is typically best heard at the left
sternal border (directly over the valve), with radiation to the neck and right infraclavicular region (along the
pathway of the ascending aorta). Often, an audible systolic click is associated with the snapping open of
the thickened valve leaflets. Mitral valve stenosis results in a diastolic murmur that is generally of low
pitch, difficult to discern unless the stenosis is extreme, and best heard along the left axillary line. The
murmur of pulmonary valve stenosis is systolic, is best heard along the right sternal border (over the
valve), and radiates to the left infraclavicular region (along the path of the main pulmonary artery). An
early systolic click often is associated with valve opening. Pulmonary valve stenosis is not associated with
rheumatic heart disease.

American Board of Pediatrics Content Specification(s):


Identify the murmurs of mitral insufficiency and aortic insufficiency as the most common murmurs in
rheumatic fever

Suggested Reading:
Cilliers AM. Rheumatic fever and its management. BMJ. 2006;333:1153-1156. DOI:
10.1136/bmj.39031.420637.BE. Available at:
http://www.ncbi.nlm.nih.gov/pmc/articles/PMC1676147/?tool=pubmed

Dajani A, Taubert K, Ferrieri P, Peter G, Shulman S; Committee on Rheumatic Fever, Endocarditis, and
Kawasaki Disease of the Council on Cardiovascular Disease in the Young, the American Heart
Association. Treatment of acute streptococcal pharyngitis and prevention of rheumatic fever: a statement
for health professionals. Pediatrics. 1995;96:758-764. Abstract available at:
http://pediatrics.aappublications.org/cgi/content/abstract/96/4/758

Steer AC, Carapetis JR. Acute rheumatic fever and rheumatic heart disease in indigenous populations.
Pediatr Clin North Am. 2009;56:1401-1419. DOI: 10.1016/j.pcl.2009.09.011. Abstract available at:
http://www.ncbi.nlm.nih.gov/pubmed/19962028

Copyright 2012 American Academy of Pediatrics


2012 PREP SA ON CD-ROM

Critique 127

Item C127. Jones Criteria for Diagnosis of Acute Rheumatic fever

Diagnosis: requires 2 major criteria or 1 major and 2 minor criteria with supporting evidence of
antecedent group A streptococcal infection
Major Criteria Minor Criteria Supporting Evidence
Carditis Clinical findings Positive throat culture or rapid test
Polyarthritis Fever, arthralgia OR
Chorea Laboratory findings: Elevated or rising streptococcal
Erythema marginatum Elevated acute phase reactants; antibody test
Subcutaneous nodules prolonged PR interval

(Reprinted with permission American Academy of Pediatrics. Group A streptococcal infections. In:
Pickering LK, Baker CJ, Kimberlin DW, Long SS, eds. Red Book:2009 Report of the Committee on
Infectious Diseases. 28th ed. Elk Grove Village, IL: American Academy of Pediatrics; 2009:616-628)

Copyright 2011 American Academy of Pediatrics


2012 PREP SA ON CD-ROM

Question 128
A 4-year-old boy has had two fairly similar, brief episodes within the past month consisting of
abrupt arrest of ongoing behavior, glassy-eyed staring, and lip smacking, followed by confusion and
sleepiness for 1 hour. He has had no fevers or other signs of illness at the time of either event. Findings
on his medical and developmental histories are otherwise normal. Physical and neurologic examination
results are normal. Brain magnetic resonance imaging yields normal results, and electroencephalography
shows no abnormalities.
Of the following, the MOST appropriate treatment for this boy is
A. carbamazepine
B. ethosuximide
C. felbamate
D. phenobarbital
E. phenytoin

Copyright 2012 American Academy of Pediatrics


2012 PREP SA ON CD-ROM

Critique 128 Preferred Response: A


The boy described in the vignette has newly diagnosed epilepsy, based on a presentation of two
unprovoked seizures. These are complex partial seizures, involving loss of awareness, automatisms, and
a postictal recovery period with somnolence and confusion. As is often the case in an otherwise healthy
child, the diagnostic evaluation (laboratory tests, electroencephalography [EEG], magnetic resonance
imaging) is unrevealing of the cause of the seizures. An abnormal EEG reading is not required for a
diagnosis of epilepsy.
Selection of seizure medication is based on type of seizure and, if possible, a specific epilepsy
syndrome diagnosis (Item C128). The descriptions of the two seizures experienced by this child are
consistent with a localized onset, possibly in the temporal lobe. Therefore, the clinician should choose a
medication such as carbamazepine that is generally safe and effective in partial epilepsies (epilepsies in
which seizures start at one cortical location and subsequently spread to adjacent areas to produce the
seizure).
Ethosuximide is used almost exclusively to treat primary absence epilepsy. Felbamate use is
typically restricted to intractable epilepsies involving multiple seizures types, including drop seizures
(atonic seizures). Phenytoin is generally not favored as an oral epilepsy medication in children because of
the potential for gum overgrowth (gingival hyperplasia) and fairly erratic enteral absorption. Phenobarbital
is seldom used at this age due to somnolence and adverse behavioral effects.

American Board of Pediatrics Content Specification(s):


Know that drug selection is based on seizure type

Suggested Reading:
Friedman MJ, Sharieff GQ. Seizures in children. Pediatr Clin North Am. 2006;53:257-277. DOI:
10.1016/j.pcl.2005.09.010. Available at: http://www.ncbi.nlm.nih.gov/pubmed/16574525

Glauser TA, Cnaan A, Shinnar S, et al; Childhood Absence Epilepsy Study Group. Ethosuximide, valproic
acid, and lamotrigine in childhood absence epilepsy. N Engl J Med. 2010;362:790-799. DOI:
10.1056/NEJMoa0902014. Available at:
http://www.ncbi.nlm.nih.gov/pmc/articles/PMC2924476/?tool=pubmed

Holland KD, Glauser TA. Response to carbamazepine in children with newly diagnosed partial onset
epilepsy. Neurology. 2007;69:596-599. Abstract available at:
http://www.ncbi.nlm.nih.gov/pubmed/17679679

Mikati MA. Seizures in childhood. In: Kliegman RM, Stanton BF, St. Geme JW III, Schor NF, and
Berhman RE, eds. Nelson Textbook of Pediatrics. 19th ed. Philadelphia, PA: Saunders Elsevier;
2011:2013-2039

Major P, Thiele EA. Seizures in children: laboratory diagnosis and management.Pediatr Rev.
2007;28:405-414. DOI: 10.1542/pir.28-11-405. Available at:
http://pedsinreview.aappublications.org/cgi/content/full/28/11/405

Copyright 2012 American Academy of Pediatrics


2012 PREP SA ON CD-ROM

Critique 128

Item C128. Seizure Type and Drug Selection

Epilepsy/Seizure Type Commonly Used Medication

Absence Ethosuximide
Generalized (not absence) Valproic acid, lamotrigine, topiramate,
levetiracetam
Partial Carbamazepine, oxcarbazepine, and
most anticonvulsants marketed in the
United States for the past 10 years are
indicated for refractory partial epilepsy

Myoclonic Valproic acid, clonazepam


Neonatal Phenobarbital, levetiracetam

Copyright 2011 American Academy of Pediatrics


2012 PREP SA ON CD-ROM

Question 129
You diagnose an atrial septal defect, bilateral hydronephrosis, and clubfeet in a newborn female.
Cytogenetic analysis reveals an unbalanced translocation between the short arm of chromosome 2 and
the long arm of chromosome 8 designated as 46, XX , der (2), t(2;8) (p24; q24), resulting in a loss of
genetic material from the top of chromosome 2 and a duplication of genetic material from the bottom of
chromosome 8. The newborn has three healthy siblings, and the family history is negative for congenital
anomalies or recurrent pregnancy losses. The parents ask about the risks for having another child with
this condition.
Of the following, you are MOST likely to advise them that the risks for recurrence
A. are as high as 50% because most unbalanced translocations are passed down from a balanced
translocation carrier
B. are between 1% and 5% because of risks for gonadal mosaicism in an otherwise normal parent
C. are negligible since the parents and sibs are healthy and there is no history of recurrent
miscarriages
D. are probably between 20% and 30% because of a higher risk for miscarriage in fetuses who have
unbalanced translocations
E. cannot be determined unless chromosome studies are performed on both parents

Copyright 2012 American Academy of Pediatrics


2012 PREP SA ON CD-ROM

Critique 129 Preferred Response: E


Although some unbalanced translocations are passed down from a healthy balanced
translocation carrier parent, most are de novo events, occurring at the time of conception. Therefore, no
reasonable risks for recurrence in future pregnancies can be ascertained without performing
chromosomal studies on both parents to determine with certainty if one of them is a balanced
translocation carrier.
In apparently chromosomally normal parents, gonadal mosaicism is a theoretical cause for
recurrence of an unbalanced translocation and would be exceedingly rare. Unfortunately, the fact that this
couple has three healthy children and no recurrent pregnancy losses does not eliminate the possibility
that one of the parents could still be a balanced translocation carrier. Because some chromosomally
unbalanced embryos may be lost very early after conception, there may be no recognition of a lost
pregnancy or blighted ovum that would raise concerns about a possible balanced translocation carrier. If
one of the parents is, in fact, a balanced translocation carrier, the risk for having an unbalanced liveborn
child is probably less than the 50% that would be predicted based on chromosomal segregation alone
because some unbalanced offspring may be lost before birth. The estimated risks for pregnancy loss
versus live birth depend on the size and content of the chromosomal segments involved in the
translocation. In general, very large deletion of genetic material carries the highest risk for pregnancy
loss; very small duplications carry the lowest risks. Cases of unbalanced translocation generally include
both a duplicated and a deleted segment. Some cytogeneticists may offer estimated risks for live birth
versus miscarriage based on the chromosomes involved and translocation breakpoints, but this is not
very precise and cannot be used exclusively for couples planning a future pregnancy to estimate risks
without also seriously considering prenatal diagnosis by way of chorionic villus sampling or amniocentesis
to confirm the fetal karyotype.
Chromosomal translocations are associated with any combination of multiple congenital
anomalies, intellectual disabilities, and poor growth. Robertsonian translocation involving the acrocentric
chromosomes (chromosomes 13, 14, 15, 21, or 22) can result in the birth of a child who has Down
syndrome (trisomy 21) or trisomy 13. If an infant is born with a trisomy due to a Robertsonian
translocation, parental blood tests should always be performed to determine if one of the parents is a
translocation carrier because this has significant implications for future pregnancies.

American Board of Pediatrics Content Specification(s):


Recognize that the finding of a structural chromosomal abnormality in an infant (eg, unbalanced
translocation) necessitates chromosomal analysis of both parents if future children are planned or
possible

Suggested Reading:
Committee on Genetics. Health supervision for children with Down syndrome. Pediatrics. 2001;107:442-
449. Available at: http://pediatrics.aappublications.org/cgi/content/full/107/2/442

Moreno-Fuenmayor H, Zackai EH, Mellman WJ, Aronson M. Familial partial trisomy of the long arm of
chromosome 10 (q24-26). Pediatrics. 1975;56:756-761. Available at:
http://pediatrics.aappublications.org/cgi/content/abstract/56/5/756

Copyright 2012 American Academy of Pediatrics


2012 PREP SA ON CD-ROM

Question 130
A 14-year-old girl presents to the office for a routine health supervision visit. Her mother, who had
her menarche at age 13 years, asks if she should be concerned that her daughter has not started
menstruating yet. Chart review confirms that the adolescent began breast development at age 10 years.
She has been tracking along the 5th to 10th percentile for height and weight since entering puberty. Her
fathers growth spurt occurred around age 16 years. The girl is at Sexual Maturity Rating (SMR) 4 for
breast development and SMR 5 for pubic hair development and has normal external genitalia. The
remainder of her physical examination findings are normal.
Of the following, the MOST appropriate next step is
A. follow-up evaluations every 6 months for 1 year
B. hand and wrist radiograph for bone age
C. luteinizing hormone and follicle-stimulating hormone assessment
D. pelvic ultrasonography
E. thyroid function testing

Copyright 2012 American Academy of Pediatrics


2012 PREP SA ON CD-ROM

Critique 130 Preferred Response: A


Progression through the development of secondary sexual characteristics has been divided into
stages, referred to as the Sexual Maturity Rating (SMR) scale (previously, Tanner stages). Although the
events are the same, the timing of onset and rate of progression through the stages may differ between
individuals in each sex. Stage 1 is prepubertal. The average length of time from stage 2 to 5 in females is
4 years but can range from 1.5 to 8 years. For males, the duration is 3 years but can range from 2 to 5
years. The average interval between breast development (thelarche, the first clearly visible sign of
pubertal development in most females) and menarche is approximately 2 years, but the range is 0.5 to
5.75 years. Increase in growth velocity is the first sign of puberty. Pubic hair development usually starts
later than breast development but reaches SMR 5 earlier.
Most females menstruate in SMR 4. The girl in the vignette is progressing through pubertal
stages normally. Because she is now 3.5 years from the start of thelarche and younger than age 16
years, observation for further development, with follow-up evaluations every 6 months for 1 year, is a safe
and prudent course. If she remains asymptomatic and has no menstruation by age 16 years, an
endocrine evaluation is appropriate and should include assessment of bone age, measurement of
luteinizing and follicle-stimulating hormones, thyroid testing, and pelvic ultrasonography, among other
tests.

American Board of Pediatrics Content Specification(s):


Know that pubertal progression from sexual maturity rating stage 2 to stage 5 can require 2.5 to five
years to complete

Suggested Reading:
Carswell JM, Stafford DEJ. Normal physical growth and development. In: Neinstein LS, Gordon CM,
Katzman Dk, Rosen DS, Woods EF, eds. Adolescent Health Care: A Practical Guide. 5th ed.
Philadelphia, PA: Lippincott Williams & Wilkins, a Wolters Kluwer business; 2008:3-26

Gordon CM, Laufer MR. The physiology of puberty. In: Emans SJH, Laufer MR, Goldstein DP, eds.
Pediatric and Adolescent Gynecology. 5th ed. Philadelphia, PA: Lippincott Williams & Wilkins, a Wolters
Kluwer business; 2005:120-155

Kaplowitz PB. Delayed puberty. Pediatr Rev. 2010;31:189-195. DOI: 10.1542/pir.31-5-189. Available at:
http://pedsinreview.aappublications.org/cgi/content/full/31/5/189

Copyright 2012 American Academy of Pediatrics


2012 PREP SA ON CD-ROM

Question 131
A 15-year-old girl is recovering in the pediatric ward of a hospital after an intentional overdose of
acetaminophen. She admits she wanted to die because of her shame over her first sexual activity and the
conflict it has caused with her parents. Her father states he is very disappointed in her and plans to have
her work with their youth minister, who made an example of her to the congregation when she
attempted to date in the past. She took extra aspirin a few days before this attempt, although her family
is unaware of this. She timed the current suicide attempt for when she thought no one would be able to
bring her to the hospital. She denies substance use or other psychiatric disturbance. You have decided
that she no longer needs inpatient medical treatment. She tells you she is very embarrassed by the
incident and is asking to go home.
Of the following, the MOST appropriate course of action is to
A. prescribe a selective serotonin reuptake inhibitor and send the girl home
B. pursue acute inpatient psychiatric hospitalization
C. pursue admission to a long-term residential psychiatric facility
D. send the girl home and arrange for counseling with her youth minister
E. send the girl home with plans for office follow-up with her primary care physician in 1 week

Copyright 2012 American Academy of Pediatrics


2012 PREP SA ON CD-ROM

Critique 131 Preferred Response: B


Each year, 15% to 25% of high school students seriously consider suicide. Although ideation
alone may not require hospitalization, patients who have made serious suicide attempts, such as the girl
described in the vignette, often benefit from acute inpatient psychiatric care to ensure safety from
additional self-harm while undergoing further evaluation and treatment planning. Known risks for
completing suicide can be recalled by the SAD PERSONS mnemonic adapted for children and
adolescents by Juhnke:

Sex (male higher risk)


Age (15 to 19 years higher risk)
Depression

Previous suicidal attempts


Excessive alcohol or drug use
Rational thinking loss (like psychosis)
Social supports are lacking
Organized plan or serious past suicide attempt
No social support, negligent parenting
School problems (aggression or experiencing humiliation)

Other known risk factors for suicide include:


Significant lethality (not expecting to be rescued)
Access to lethal means (firearms or drugs)
Concurrent psychiatric illness (especially psychotic disorders, substance abuse, mood/anxiety
disorders, or disorders involving excessive impulsivity)
Persistent wish to die
Having few or inappropriate coping abilities (such as substance abuse, self-injury, or somatization)
Interpersonal conflict (especially if associated with fear of abandonment or public humiliation)
Family dysfunction
This girls serious premeditation, repeated attempts, past social humiliation, and hopelessness
place her at high risk for another suicide attempt.
Sending the girl home without further psychiatric evaluation and treatment is not appropriate. In
addition, although a selective serotonin reuptake inhibitor may be indicated for her, it is not a substitute
for psychiatric hospitalization because it is unlikely to reduce her immediate risk for suicide. Such
medications may not be effective for weeks and would not address her family and social risk factors.
Because it is important for patients to view their individual therapists as their own advocates rather than
agents of their parents, counseling by the youth minister from her parents church is potentially harmful.
Residential long-term psychiatric facilities are not used for acute management of suicidality; they are
reserved for patients who have not responded to shorter term psychiatric hospital care.

AAP Mental Health Competency:


Know when to refer a child with suicidality for psychiatric hospitalization

Suggested Reading
American Academy of Child and Adolescent Psychiatry. Practice parameter for the assessment and
treatment of children and adolescents with suicidal behavior. J Am Acad Child Adolesc Psychiatry.
2001;40(7 suppl);24S-51S. Available at:
http://www.aacap.org/galleries/PracticeParameters/JAACAP%20Suicide%202001.pdf

American Academy of Pediatrics. Depression. In: Addressing Mental Health Concerns in Primary Care: A
Clinicians Toolkit. Elk Grove Village, IL: The American Academy of Pediatrics; 2010

Copyright 2012 American Academy of Pediatrics


2012 PREP SA ON CD-ROM

Birmaher B, Brent D; AACAP Work Group on Quality Issues, Bernet W, et al. Practice parameter for the
assessment and treatment of children and adolescents with depressive disorders. J Am Acad Child
Adolesc Psychiatry. 2007;46:1503-1526. Available at:
http://www.aacap.org/galleries/PracticeParameters/Vol%2046%20Nov%202007.pdf

Cheung AH, Ghalib K, Jensen PS, et al. Guidelines for Adolescent Depression in Primary Care (GLAD-
PC) Tool Kit. 2007. Available at: http://www.thereachinstitute.org/files/documents/GLAD-PCToolkit.pdf.

Cheung AH, Zuckerbrot RA, Jensen PS, Ghalib K, Laraque D, Stein REK, GLAD-PC Steering Group.
Guidelines for adolescent depression in primary care (GLAD-PC): II. treatment and ongoing
management. Pediatrics. 2007(5);120:e1313-e1326. Available at:
http://pediatrics.aappublications.org/content/120/5/e1313.full.pdf+html

Juhnke GE. The adapted SAD PERSONS: assessment scale designed for use with children. Elementary
School Guidance and Counseling. 1996:252-258

US Preventive Services Task Force. Screening and treatment for major depressive disorder in children
and adolescents: US Preventative Services Task Force recommendation statement. Pediatrics.
2009;123:1223-1228. DOI: 10.1542/peds.2008-2381. Available at:
http://pediatrics.aappublications.org/content/123/4/1223.long

Williams SB, OConnor EA, Eder M, Whitlock EP. Screening for child and adolescent depression in
primary care settings: a systemic evidence review for the US Preventive Services Task Force. Pediatrics.
2009;123:e716-e735. DOI: 10.1542/peds.2008-2415. Available at:
http://pediatrics.aappublications.org/content/123/4/e716.full

Zuckerbrot RA, Cheung AH, Jensen PS, Stein REK, Laraque D, and the GLAD-PC Steering Group.
Guidelines for adolescent depression in primary care (GLAD-PC): I. identification, assessment, and initial
management. Pediatrics. 2007;120(5):e1299-e1312. Avaiable at:
http://pediatrics.aappublications.org/content/120/5/e1299

Copyright 2012 American Academy of Pediatrics


2012 PREP SA ON CD-ROM

Question 132
You are teaching a group of medical students in the well baby nursery. One of them asks about
pain perception in newborns and if pain medications are needed before circumcision.
Of the following, the MOST accurate statement regarding pain perception in term newborns is
that
A. neonatal pain fibers are fully myelinated at birth
B. neonates are at increased risk of pain wind-up (hypersensitivity)
C. neonates are incapable of perceiving pain
D. neonates lack opioid receptors at birth
E. sensory nervous system development begins at 12 weeks of gestation

Copyright 2012 American Academy of Pediatrics


2012 PREP SA ON CD-ROM

Critique 132 Preferred Response: B


Pain is classically defined as an unpleasant sensory and emotional experience associated with
an actual or anticipated noxious stimulus. Pain is subjective, and repeated painful experiences can result
in altered pain sensitivity and has been demonstrated to lead to behavioral disturbances in laboratory
animals. Children and adults often express and deal with pain differently and, therefore, undertreatment
of pediatric pain is a concern, especially among neonates.
The human sensory nervous framework begins to develop by the seventh week of gestation and
is essentially complete at term. Although newborns lack complete central nervous system myelination, the
increased density of nociceptive receptors and shorter nerve transmission distances make them highly
sensitive to painful stimuli. In fact, newborns may be at greater risk for pain wind-up, in which repeated
painful stimuli produce central sensitization and a resultant hyperalgesic state. Narcotics remain the
mainstay of management when pain is more than moderate, but it is important to remember that
newborns are at increased risk of respiratory depression due to the relative predominance of mu opioid
receptors.
The goals of pain management are anticipation, treatment, reassessment, and use of
nonpharmacologic measures to the maximum degree possible. The latter include open communication,
reassurance, parental presence, distraction, hypnotherapy, biofeedback, acupuncture, and sucrose, as
indicated by the developmental status and condition of the patient.

American Board of Pediatrics Content Specification(s):


Recognize that dealing with and tolerance to pain vary with a child's developmental stage
Describe different techniques to manage pain/anxiety in children at different developmental stages

Suggested Reading:
American Academy of Pediatrics, Committee on Fetus and Newborn; American Academy of Pediatrics,
Section on Surgery; Section on Anesthesiology and Pain Medicine, Canadian Paediatric Society, Fetus
and Newborn Committee. Prevention and management of pain in the neonate: an update. Pediatrics.
2006;118:22312241. DOI: 10.1542/peds.2006-2277. Available at:
http://pediatrics.aappublications.org/cgi/content/full/118/5/2231

Holsti L, Grunau RE. Considerations for using sucrose to reduce procedural pain in preterm infants.
Pediatrics. 2010;125:10421047. DOI: 10.1542/peds.2009-2445. Available at:
http://pediatrics.aappublications.org/cgi/content/full/125/5/1042

Zeltzer LK, Krane EJ. Pediatric pain management. In: Kliegman RM, Stanton BF, St. Geme JW III, Schor
NF, and Behrman RE, eds. Nelson Textbook of Pediatrics. 19th ed. Philadelphia, PA: Saunders Elsevier;
2011:360-375

Copyright 2012 American Academy of Pediatrics


2012 PREP SA ON CD-ROM

Question 1
A 4-year-old boy who has severe recurrent asthma and an allergy to cockroaches has had poor
disease control despite inhaled fluticasone propionate 220 mcg twice a day for the previous 4 months. He
most recently required 4 weeks of prednisone at 2 mg/kg per day to manage asthma symptoms and
avoid hospitalization. After moving to a new home, his asthma symptoms improved dramatically, and he
discontinued both the fluticasone and the prednisone. When he presents today, his mother states that he
has not suffered any further asthma exacerbations, but he has developed nausea and severe fatigue.
Of the following, the MOST appropriate treatment for this boy is to
2
A. begin hydrocortisone at 10 mg/m per day
2
B. begin hydrocortisone at 50 mg/m per day
C. restart both fluticasone at 220 mcg/day and prednisone at 2 mg/kg per day
D. restart fluticasone at 220 mcg twice a day
E. restart prednisone at 5 mg/kg per day

Copyright 2012 American Academy of Pediatrics


2012 PREP SA ON CD-ROM

Critique 133 Preferred Response: A


Although inhaled corticosteroids are an important part of the standard armamentarium for treating
persistent asthma, it is important to remember that this treatment can have systemic effects. When
patients who have severe asthma require long-term use of either inhaled corticosteroids or the
combination of inhaled and oral corticosteroids, pediatricians must be cognizant of the potential for
adrenal insufficiency to arise when such therapy is discontinued. The boy described in the vignette has
been receiving 2 mg/kg prednisone per day which has a glucocorticoid potency that is five times higher
than hydrocortisone [eg, equivalent to 10 mg/kg hydrocortisone per day (Item C133)] in addition to high-
dose fluticasone. Therefore, it is not surprising that he may have lost his capacity to make endogenous
glucocorticoids and, as a result, has developed nausea and fatigue. Oral hydrocortisone doses of 8 to 12
2
mg/m per day are required to replace endogenous glucocorticoid production and should be weaned over
a period of time equivalent to the duration of high-dose steroid therapy(at least 4 weeks in this case).
Glucocorticoid production is controlled by a standard feedback loop, with higher serum
glucocorticoid concentrations feeding back to the hypothalamus and pituitary, resulting in reduced
adrenocorticotropin hormone concentrations and eventually adrenal atrophy. Because recovery of the
hypothalamic-pituitary-adrenal axis often takes weeks to months, slow weaning of corticosteroids is
required when high doses have been used for more than 2 weeks. Weaning of exogenous steroids
should typically match the same period of time as administration of the initial supraphysiologic
glucocorticoids.
2
A hydrocortisone dose of 50 mg/m per day would be far too great for the boy in the vignette and
would likely result in symptoms of Cushing syndrome. Similarly, a combination of prednisone 2 mg/kg per
day and fluticasone or 5 mg/kg per day of prednisone alone would provide more glucocorticoid than is
physiologically needed and would not allow recovery of endogenous glucocorticoid production. Finally,
fluticasone 220 mcg twice a day alone would not provide adequate systemic glucocorticoids during the
acute period of adrenal insufficiency.

American Board of Pediatrics Content Specification(s):


Know the complications of sudden withdrawal of corticosteroids in pharmacologic doses in patients
with adrenal insufficiency

Suggested Reading:
Allen DB. Effects of inhaled steroids on growth, bone metabolism, and adrenal function. Adv Pediatr.
2006;53:101-110. Abstract available at: http://www.ncbi.nlm.nih.gov/pubmed/17089864

Stewart PM. The adrenal cortex. In: Kronenberg HM, Melmed S, Polonsky KS, Larsen PR, eds.Williams
Textbook of Endocrinology. 11th ed. Philadelphia, PA: Saunders Elsevier; 2008:Chapter 14

Copyright 2012 American Academy of Pediatrics


2012 PREP SA ON CD-ROM

Critique 133

Item C133. Comparative Potency in Milligram Equivalent


Doses of Commonly Used Steroids
Glucocorticoid Mineralocorticoid
Steroid
Potency Potency
Hydrocortisone 1 1
Prednisolone 4 0.8
Prednisone 5 0.8
Methylprednisolone 7.5 0.5
Fludrocortisone 15 200
Dexamethasone 40 0

This table assigns hydrocortisone a relative glucocorticoid


and mineralocorticoid potency of 1 and compares potency
of other steroids if given in milligram equivalent doses.
Thus, prednisone has a glucocorticoid potency that is five
times higher per milligram than hydrocortisone.

Copyright 2011 American Academy of Pediatrics


2012 PREP SA ON CD-ROM

Question 134
A 3-year-old child is showing evidence of significant delay in his expressive and receptive
language; other aspects of his development are normal. His hearing has been tested and is normal. You
review the situation with his mother.
Of the following, the MOST appropriate action is to
A. have the boy evaluated for an augmented communication device
B. have the boy return for a follow-up visit in 6 months
C. have the mother begin to teach him simple signs to minimize his frustration
D. refer the boy for a psychoeducational evaluation
E. refer the boy for speech evaluation and therapy as indicated

Copyright 2012 American Academy of Pediatrics


2012 PREP SA ON CD-ROM

Critique 134 Preferred Response: E


The boy described in the vignette is showing significant expressive and receptive language delay
that should be addressed by referral for a speech therapy evaluation and services, as indicated. A
therapy evaluation should be undertaken at this time; the family should not be advised to return in 6
months, which would delay the needed intervention. The therapist should use evidence-based practice in
developing an intervention plan. The interventions can range from clinician-directed to child-centered or a
hybrid of the two approaches. Clinician-directed therapy is a highly structured approach. The clinician
controls the situation and provides reinforcement and feedback on the performance. In child-directed
interventions, such as indirect language stimulation or developmental/pragmatic approaches, the clinician
sets up a situation offering opportunities for the child to provide responses that are natural in the situation.
This approach is very useful for a child who is either noncompliant or not a very assertive communicator.
In the hybrid approach, the clinician focuses on a few specific language goals, maintaining control but
facilitating spontaneous responses using the language goals that are being targeted.
An augmented communication device would only be indicated if speech therapy services have
limited success in improving the boys ability to communicate expressively. These devices provide
individuals who have limited communication with the means to augment their verbal and written
expression. The devices range from picture communication boards to computers that have synthesized
speech output.
Once speech therapy is initiated, psychoeducational evaluation may be considered to determine
if additional specialized educational services are required. Although the use of signs may aid the childs
ability to communicate, speech therapy evaluation can best delineate the appropriate interventions.
As noted in a Cochrane review, speech and language therapy interventions have a positive effect
for children who have expressive phonologic and expressive vocabulary difficulties. The evidence
supporting their success for expressive syntax (sentence structure) difficulties is more mixed. Further
research is required to investigate interventions for receptive language difficulties.

American Board of Pediatrics Content Specification(s):


Recognize the available evidence-based treatments for speech and language disorders

Suggested Reading:
Cherab Foundation. About Speech/Language Delays and Disorders. Available at:
http://www.cherab.org/information/indexinformation.html

Coplan J. Language delays. In: Parker S, Zuckerman B, Augustyn M, eds. Developmental and Behavioral
Pediatrics: A Handbook for Primary Care. 2nd ed. Philadelphia, PA: Lippincott Williams & Wilkins, a
Wolters Kluwer business; 2005:222-226

Law J, Garrett Z, Nyle C. Speech and language therapy interventions for children with primary speech
and language delay or disorder. Cochrane Database Syst Rev. 2003;3:CD004110. DOI:
10.1002/14651858.CD004110. Available at: http://www2.cochrane.org/reviews/en/ab004110.html

Copyright 2012 American Academy of Pediatrics


2012 PREP SA ON CD-ROM

Question 135
A 4-year-old boy is brought to your office for treatment of a dog bite to his right hand that
occurred several hours ago. A neighbors dog bit the child when he was playing near the dogs food dish.
Physical examination reveals several bite marks on the dorsum of his hand that have broken the skin.
According to your records, the boy developed a diffuse pruritic rash after receiving amoxicillin for a
streptococcal pharyngitis 6 months ago. The records confirm that his tetanus immunization status is up to
date.
Of the following, the MOST appropriate antibiotic coverage for this child is
A. amoxicillin-clavulanate
B. azithromycin and trimethoprim-sulfamethoxazole
C. cefdinir
D. ciprofloxacin
E. clindamycin and trimethoprim-sulfamethoxazole

Copyright 2012 American Academy of Pediatrics


2012 PREP SA ON CD-ROM

Critique 135 Preferred Response: E


Dog bite wounds, such as sustained by the boy in the vignette, are an important source of
infection. Because of the high rate of infection from such injuries (especially when they occur on the face,
hands, or feet), antimicrobial therapy is recommended, especially for moderate or severe bite wounds.
Administration of antibiotics within 8 to 12 hours of injury (for 2 to 3 days duration) may decrease the risk
for infection.
Appropriate regimens need to cover potential pathogens, including Pasteurella, Staphylococcus
aureus, streptococci, anaerobes, and Capnocytophaga. Pasteurella, including P multocida, are small
gram-negative coccobacilli that are normal flora in a number of animals, including cats and dogs. These
organisms are susceptible to penicillin. Among the alternative agents with activity against Pasteurella are
trimethoprim-sulfamethoxazole (TMP/SMX), extended-spectrum cephalosporins, doxycycline, and
fluoroquinolones. Macrolides, first-generation cephalosporins, and clindamycin have poor activity against
Pasteurella.
Both amoxicillin-clavulanate and clindamycin plus TMP/SMX cover the organisms seen with
animal bites, including Pasteurella, and are the recommended regimens for prophylaxis or treatment of
bite wound infections. However, amoxicillin-clavulanate would not be used in a child who has a history of
allergy to amoxicillin because there is an acceptable alternative. Azithromycin and TMP/SMX would not
provide adequate coverage for anaerobes and possibly streptococci. Cefdinir would not provide adequate
gram-positive coverage. Ciprofloxacin is not routinely indicated for pediatric use if there are acceptable
alternatives, as in this case, and may not afford optimal S aureus coverage.

American Board of Pediatrics Content Specification(s):


Know the treatment of Pasteurella multocida infection

Suggested Reading:
American Academy of Pediatrics. Pasteurella infections. In: Pickering LK, Baker CJ, Kimberlin DW, Long
SS, eds. Red Book: 2009 Report of the Committee on Infectious Diseases. 28th ed. Elk Grove Village, IL:
American Academy of Pediatrics; 2009:493-494

Stevens DL, Bisno AL, Chambers HF, et al; Infectious Diseases Society of America. Practice guidelines
for the diagnosis of skin and soft-tissue infections. Clin Infect Dis. 2005;41:13731406. DOI:
10.1086/497143. Available at: http://cid.oxfordjournals.org/content/41/10/1373.long

Copyright 2012 American Academy of Pediatrics


2012 PREP SA ON CD-ROM

Question 136
A 2-year-old girl who has no significant exposure history presents with a 4-week history of
progressive right neck swelling. On physical examination, her temperature is 37.0C and weight is 12 kg
(50th percentile). She has a 3x2-cm area of swelling of the right anterior neck that is mildly fluctuant and
tender to palpation. The overlying skin is purplish and thin. There is no hepatosplenomegaly or diffuse
lymphadenopathy or other findings of note on the remainder of the physical examination. A tuberculin skin
test read at 48 hours measures 5 mm of induration.
Of the following, the MOST appropriate next step is to
A. initiate clindamycin and trimethoprim-sulfamethoxazole therapy
B. initiate isoniazid and rifampin therapy
C. obtain a chest radiograph
D. perform an excisional biopsy of the lymph node
E. perform needle aspiration of the lymph node

Copyright 2012 American Academy of Pediatrics


2012 PREP SA ON CD-ROM

Critique 136 Preferred Response: D


The girl described in the vignette has cervical lymphadenitis caused by nontuberculous
mycobacteria (NTM). NTM can cause numerous diseases in humans, including cutaneous infection,
lymphadenitis, otologic infection, pulmonary infection, catheter-associated bloodstream infection, skeletal
infection, and disseminated disease. In children, lymphadenitis is most common.
Complete surgical resection of the infected lymph node is curative in otherwise healthy children
who have NTM lymphadenitis and provides a definitive diagnosis. Needle aspiration of the lymph node is
contraindicated because of the risk of developing a chronic draining fistula. Antimicrobial therapy with
clarithromycin or azithromycin in combination with ethambutol or rifampin or rifabutin may be beneficial
when complete resection is not possible or in cases of recurrent disease. Isoniazid has no activity against
NTM. Trimethoprim-sulfamethoxazole in combination with agents other than clindamycin may be useful
for treating cutaneous infection caused by certain NTM species, but it is not typically first-line therapy.
Clindamycin may be appropriate for treating acute bacterial lymphadenitis caused by staphylococci and
streptococci, but it has no role in the treatment of subacute or chronic lymphadenitis caused by NTM.
Signs and symptoms associated with cervical lymphadenitis caused by NTM can be
indistinguishable from those associated with Mycobacterium tuberculosis adenitis. However, in general,
NTM adenitis occurs most commonly in children 1 to 5 years of age, and M avium complex is the most
common cause in children in the United States. NTM organisms are ubiquitous in the soil, and younger
children are more likely to ingest them. The subsequent lymph node swelling usually is unilateral,
submandibular, and increasing in size over several weeks. Pain and systemic symptoms are uncommon.
The node may be fluctuant, and the overlying skin color evolves from pink to violaceous (Item C136). The
result of the tuberculin skin test is usually less than 15 mm of induration, although larger reactions can
occur. Chest radiography in patients with NTM adenitis will be normal. Lateral neck radiographs will not
be helpful in the management of this patient who does not have evidence of airway compromise.
Tuberculosis lymphadenitis tends to occur in older children (>5 years) and adults. Systemic
symptoms and a history of an exposure to tuberculosis are common. Bilateral lower cervical
lymphadenitis most frequently occurs. The enlarged lymph nodes usually are rubbery and painless early
in the course of infection. The tuberculin skin test result usually is greater than 15 mm of induration.

American Board of Pediatrics Content Specification(s):


Understand the diagnosis and treatment of cervical adenitis secondary to nontuberculous
mycobacteria

Suggested Reading:
American Academy of Pediatrics. Diseases caused by nontuberculous mycobacteria. In: Pickering LK,
Baker CJ, Kimberlin DW, Long SS, eds. Red Book: 2009 Report of the Committee on Infectious
Diseases. 28th ed. Elk Grove Village, Il: American Academy of Pediatrics; 2009:701-708

Cruz AT, Ong LT, Starke JR. Mycobacterial infections in Texas children: a 5-year case series. Pediatr
Infect Dis J. 2010;29:772-774. DOI: 10.1097/INF.0b013e3181da5795. Abstract available at:
http://www.ncbi.nlm.nih.gov/pubmed/20661106

Thorell EA, Chesney PJ. Cervical lymphadenitis and neck infections. In: Long SS, Pickering LK, Prober
CG, eds. Principles and Practice of Pediatric Infectious Diseases. 3rd ed. Philadelphia, PA: Churchill
Livingstone Elsevier; 2008:143-154

Copyright 2012 American Academy of Pediatrics


2012 PREP SA ON CD-ROM

Critique 136

(Courtesy of C Bower)
Nodes infected with atypical mycobacteria suppurate, and the overlying skin becomes red. The node may
rupture and form a sinus tract to the skin surface, resulting in drainage and crusting.

Copyright 2011 American Academy of Pediatrics


2012 PREP SA ON CD-ROM

Question 137
A 3-year-old boy presents with difficulty voiding. His mother states that he appears fussy and
seems to strain when trying to void. He is otherwise well and has no history of fever, vomiting, difficulty
feeding, swelling, or gross hematuria. He does have a history of acute pyelonephritis at 10 months of age.
Ultrasonography and voiding cystourethrography (VCUG) results were reported as normal. His mother
noted that he had some visible blood in his urine immediately after undergoing his VCUG, which cleared
several hours later. The boy was toilet trained at 2 years of age. He has had no recent urinary tract
infections. Physical examination reveals an uncircumcised male who has easily retractable foreskin and
no sacral dimples or hair tufts on the back. Urinalysis results include a specific gravity of 1.020; pH of 6;
and no blood, protein, leukocyte esterase, or nitrite. Microscopy is negative.
Of the following, the MOST likely explanation for this boys difficulty voiding is
A. bladder stone
B. cystitis
C. neurogenic bladder
D. posterior urethral valves
E. urethral stricture

Copyright 2012 American Academy of Pediatrics


2012 PREP SA ON CD-ROM

Critique 137 Preferred Response: E


Young children who are unable to void may be voluntarily withholding due to pain, as can occur
with cystitis or urethritis. However, if a toilet-trained child is attempting to void but is unsuccessful, as
described for the boy in the vignette, the cause may be obstruction, typically bladder outlet obstruction.
One common explanation in a male is posterior urethral valves (PUV), which has an incidence of 1 in
5,000 male infants. However, this boy has already undergone voiding cystourethrography (VCUG), the
results of which were normal, which rules out a congenital lesion such as PUV. A neurogenic bladder can
occur, but that typically is due to myelomeningocele, a condition that is highly unlikely with normal
findings on back examination and normal VCUG results. The absence of blood on urinalysis makes a
bladder stone highly unlikely, and normal urinalysis findings rule out cystitis. Therefore, the most likely
explanation for this boys difficulty voiding is a urethral stricture, especially in light of the visible blood in
the urine following his VCUG, which suggests traumatic catheterization.
Urethral strictures occur almost exclusively in boys and usually are due to trauma. Although a
rare condition, most cases generally are iatrogenic. Urethral strictures have been reported following
traumatic catheterization, cystourethrography, prolonged indwelling catheters, aggressive fulguration
(resection) of posterior urethral valves in infants, and hypospadias repair. Clinical symptoms include
acute urinary retention, difficulty voiding, and weak urinary stream. This diagnosis should be entertained
in patients who have histories suggestive of underlying bladder or urethral pathology, including
myelomeningocele. Urology consultation should be sought to aid with the diagnosis and treatment.
Diagnosis is made by retrograde urethrography or by direct visualization (cystourethroscopy). Treatment
consists of urethral dilatation, endoscopic urethrotomy, or formal urethroplasty with excision of urethral
scar and anastomosis.

American Board of Pediatrics Content Specification(s):


Know that urethral strictures in boys almost always result from urethral trauma (iatrogenic or
accidental)

Suggested Reading:
Baskin LS, McAninch JW. Childhood urethral injuries: perspectives on outcome and treatment. Br J Urol.
1993;72:241-246. Abstract available at: http://www.ncbi.nlm.nih.gov/pubmed/8402031

Brock WA, Kaplan GW. Abnormalities of the lower urinary tract. In: Edelmann CM, Bernstein J, Meadow
SR, Sptizer A, Travis LB, eds. Pediatric Kidney Disease. 2nd ed. Boston, MA: Little, Brown and Company;
1992:2037-2075

Farhat W, McLorie G. Urethral syndromes in children. Pediatr Rev. 2001;22:17-21. DOI: 10.1542/pir.22-1-
17. Available at: http://pedsinreview.aappublications.org/cgi/content/full/22/1/17

Harshman MW, Cromie WJ, Wein AJ, Duckett JW. Urethral stricture disease in children. J Urol.
1981;126:650-654. Abstract available at: http://www.ncbi.nlm.nih.gov/pubmed/7299929

Kaplan GW, Brock WA. Urethral strictures in children. J Urol. 1983;129:1200-1203. Abstract available at:
http://www.ncbi.nlm.nih.gov/pubmed/6854799

Koraitim MM. Posttraumatic posterior urethral strictures in children: a 20-year experience. J Urol.
1997;157:641645. Abstract available at: http://www.ncbi.nlm.nih.gov/pubmed/8996388

Lindehall B, Abrahamsson K, Hjlms K, Jodal U, Olsson I, Silln U. Complications of clean intermittent


catheterization in boys and young males with neurogenic bladder dysfunction. J Urol. 2004;172:1686-
1688. Abstract available at: http://www.ncbi.nlm.nih.gov/pubmed/15371790

Copyright 2012 American Academy of Pediatrics


2012 PREP SA ON CD-ROM

Question 138
You are evaluating a 3-month-old boy who was born at 38 weeks gestation and has experienced
chronic diarrhea and recurrent pneumonia, although his growth and development have been normal.
During a recent hospitalization for pneumonia, a chest radiograph demonstrated a right lower pneumonia
but absence of a thymic shadow. You suspect severe combined immunodeficiency and discuss the case
with an immunologist.
Of the following, the MOST appropriate initial screening test for this child is
A. antibody response to polysaccharide vaccines
B. antibody response to protein vaccines
C. complete blood count
D. immunoglobulin G subclasses
E. T-cell receptor excision circles

Copyright 2012 American Academy of Pediatrics


2012 PREP SA ON CD-ROM

Critique 138 Preferred Response: C


The infant described in the vignette, who presents with recurring infections and an absent thymic
shadow, should be evaluated aggressively for severe combined immunodeficiency (SCID). Although
some affected infants have normal lymphocyte counts, lymphopenia is a classic laboratory finding, and a
complete blood count (CBC) is the appropriate initial screening test. Recognizing SCID and referring the
patient to an immunology center is critical because early diagnosis, classification, and stem cell
transplantation improve survival. Other common initial signs and symptoms of SCID are oral candidiasis,
recalcitrant diaper dermatitis, failure to thrive, absence of tonsillar tissue or palpable lymph nodes, and
chronic diarrhea. Further testing to evaluate cell-mediated immunity includes T-, B-, and natural killer
(NK)-cell flow cytometry and lymphocyte proliferation studies. Anergy testing for delayed-type
hypersensitivity is a good qualitative screening of the cellular immune system but is limited by higher
false-negative rates in infants compared to older children. Additional lymphocyte proliferation assays may
be required to assess the cellular immune system in patients who have negative results to anergy testing.
Measurement of T-cell receptor excision circles (TRECs) has been evaluated for some time in
Native Americans due to the prevalence of SCID in these populations. The evaluation of TRECs using
neonatal screening filter papers has become more widespread, with some states adopting this screening
test for all newborns. TRECs are formed during the normal maturation of T cells; low-to-absent TRECs
are consistent with T-cell lymphopenia. Measurement of TRECs may be appropriate for newborns, but a
CBC is more appropriate screening for a 3-month-old infant.
Antibody response to protein and polysaccharide vaccines is an important assessment of
humoral immunity, but infants younger than 2 years of age have a poor response to polysaccharide
vaccines, limiting its applicability. Antibody responses that can be measured at an early age are
isohemagglutinins, which are immunoglobulin (Ig)M-derived antibodies against blood group antigens that
can be measured in infants who have blood group A, B, or O.
IgG subclasses encompass IgG1, IgG2, IgG3, and IgG4. Measurement of IgG subclasses is
controversial, and measurement before 4 to 5 months of age is not helpful because the IgG in such
infants is primarily maternally derived. Hypogammaglobulinemia is a laboratory feature of SCID, but
infants may have a transient hypogammaglobulinemia due to a physiologic gap between transplacental
maternal IgG and the infants intrinsic IgG production.

American Board of Pediatrics Content Specification(s):


Plan the laboratory evaluation of cell-mediated immunity (lymphocyte counts and lymphocyte
function)

Suggested Reading:
Baker MW, Laessig RH, Katcher ML, et al. Implementing routine testing for severe combined
immunodeficiency within Wisconsin's newborn screening program. Public Health Rep. 2010;125(suppl
2):88-95. Abstract available at: http://www.ncbi.nlm.nih.gov/pubmed/20518449

Ballow M. Approach to the patient with recurrent infections. In: Atkinson NF Jr, Bochner BS, Busse WW,
Holgate ST, Lemanske RF Jr, Simons FER, eds. Middletons Allergy Principles and Practice. 7th ed.
Philadelphia, PA: Mosby Elsevier; 2009:1405-1421

Bonilla FA, Bernstein IL, Khan DA, et al; American Academy of Allergy, Asthma and Immunology;
American College of Allergy, Asthma and Immunology; Joint Council of Allergy, Asthma and Immunology.
Practice parameter for the diagnosis and management of primary immunodeficiency. Ann Allergy Asthma
Immunol. 2005;94(suppl 1):S1-S63

Copyright 2012 American Academy of Pediatrics


2012 PREP SA ON CD-ROM

Question 139
A 14-year-old softball player comes to the emergency department after being struck in the eye by
a pitch. She is awake and alert, complaining of right eye and face pain. She has obvious swelling and
ecchymosis around her right orbit. Her extraocular movements are normal on the left, but she is unable to
look up with her right eye. Her globe is intact, vision is 20/20 in both eyes, and pupils are equally round
and reactive; no corneal injuries are apparent on fluorescein examination.
Of the following, the injury that BEST explains this girls physical findings is
A. epidural hematoma
B. intracranial contusion
C. LeFort fracture, type 1
D. right orbital floor fracture
E. right temporal skull fracture

Copyright 2012 American Academy of Pediatrics


2012 PREP SA ON CD-ROM

Critique 139 Preferred Response: D


Isolated loss of upward gaze following blunt trauma to the face, as described for the girl in the
vignette, is a typical finding associated with an orbital floor (or blow-out) fracture. The restricted
extraocular movement is caused by herniation and entrapment of the inferior rectus muscle into the
maxillary sinus through the fracture on the affected side. Because epidural hematomas and intracranial
contusions injure the cerebral cortex, patients who have these injuries present with more global changes
in mental status (eg, confusion, lethargy, coma) and, in some cases, hemiparesis or evidence of uncal
herniation. LeFort fractures (Item C139) involve the middle and lower half of the face and are detected
when physical examination demonstrates midface instability. Temporal bone fractures are typically
accompanied by hemotympanum, bruising behind the ear or under the eyes, and cranial nerve palsies.
Orbital fractures are seen most commonly in adolescent and young adult males following a direct
blow to the orbit during sports, assaults, and motor vehicle crashes. In the case of an orbital floor fracture,
the force applied to the orbit may fracture the infraorbital rim directly or blow out the orbital floor through
a sudden explosive increase in intraorbital pressure. Affected patients present with periorbital
ecchymoses and eye/face pain. Other findings may include epistaxis, subcutaneous/orbital emphysema,
limited extraocular movements, anesthesia of the cheek and upper gum on the affected side from
infraorbital nerve injury, and displacement of the globe within the orbit (enophthalmos or exophthalmus).
Evaluation of patients who have suspected orbital floor fractures should include a thorough eye
examination to detect any globe-threatening injuries. Because substantial force is required to cause
orbital fractures, associated globe injuries are reported to be present in 10% to 50% of patients. These
can include globe rupture, hyphema, optic nerve hematoma, and retinal detachment. Evaluation of the
orbit and face for fractures is best accomplished using thin-cut coronal computed tomography scan of the
orbit. Consultation with a surgeon who manages facial trauma is indicated for orbital fractures. In general,
surgery may be indicated in patients who have persistent diplopia or cosmetic deformities after swelling
and edema have resolved. In addition, consultation with an ophthalmologist is recommended for patients
who have associated globe injury.

American Board of Pediatrics Content Specification(s):


Recognize the clinical signs of a blow-out fracture of the orbit

Suggested Reading:
Carroll SC, Ng SGJ. Outcomes of orbital blowout fracture surgery in children and adolescents. Br J
Ophthalmol 2010;94:736-739. Abstract available at: http://www.ncbi.nlm.nih.gov/pubmed/20508048

Kwitko GM. Orbital fracture, floor. eMedicine Specialties, Ophthalmology, Orbit. 2009. Available at:
http://emedicine.medscape.com/article/1218283-overview

Neuman MI, Bachur RG. Orbital fractures. UpToDate Online 18.3. 2010. Available for subscription at:
http://www.uptodate.com/online/content/topic.do?topicKey=ped_trau/2166

Widell T. Orbital fracture. eMedicine Specialties, Emergency Medicine, Trauma and Orthopedics. 2008.
Available at: http://emedicine.medscape.com/article/825772-overview

Copyright 2012 American Academy of Pediatrics


2012 PREP SA ON CD-ROM

Critique 139

Item C139. LeFort Fractures


Type Fracture Sites Clinical Presentation
Type 1: Horizontal Fracture passes through the Swelling of upper lip, malocclusion,
maxillary fracture alveolar ridge, lateral nose, and mobility of of maxillary teeth with
inferior wall of maxillary sinus pressure
Type 2: Pyramidal Fracture passes through Gross edema of soft tissue over the
fracture posterior alveolar ridge, medial- middle third of the face, including
orbital rim, and nasal bones nasal bridge and maxilla, bilateral
circumorbital ecchymosis, epistaxis
Type 3: Craniofacial Fracture passes through the As for Type 2 plus tenderness and
disjunction nasofrontal and maxillofrontal separation at frontozygomatic suture,
suture, orbital wall, and lengthening of face, enophthalmos
zygomatic arch

Copyright 2011 American Academy of Pediatrics


2012 PREP SA ON CD-ROM

Question 140
A 17-year-old boy is referred to you by his school because of obesity. He has been in good
health, except for a rapid increase in weight for the past 2 years. During that time, his parents have noted
a change in his behavior, and they are concerned about his poor school performance. Because of
declining grades and some difficulty in concentrating on his studies, the young man recently was
diagnosed with attention-deficit disorder and has been treated with methylphenidate. Physical
examination demonstrates a well-developed, obese adolescent, whose height is 170 cm and weight is 80
kg. The only other findings of note are a firm liver edge palpated 2 cm below the right costal margin and a
palpable spleen tip. Initial laboratory results include:
Hemoglobin, 13.0 g/dL (130 g/L)
Aspartate aminotransferase, 60 units/L (normal, 10 to 30 units/L)
Alanine aminotransferase, 55 units/L (normal, 10 to 30 units/L)
Alkaline phosphatase, 280 units/L (normal, 10 to 140 units/L)
Bilirubin, 3.5 mg/dL (59.9 mcmol/L)
Erythrocyte sedimentation rate, 30 mm/hr
Because of these results, you obtain the following additional studies:
Hepatitis A antibody, negative
Hepatitis B surface antibody, positive
Hepatitis B surface antigen, negative
Hepatitis C antibody, negative
Alpha-1-antitrypsin, 220 mg/dL (40.5 mcmol/L) (normal, 150 to 350 [27.6 to 64.4 mcmol/L])
Serum ceruloplasmin, 22 mg/dL (220 mg/L) (normal, 20 to 60 [200 to 600 mg/L])
Of the following, the MOST appropriate next study is
A. abdominal ultrasonography
B. Epstein-Barr virus titers
C. liver biopsy
D. serum copper measurement
E. slit lamp examination

Copyright 2012 American Academy of Pediatrics


2012 PREP SA ON CD-ROM

Critique 140 Preferred Response: E


The adolescent boy described in the vignette presents with obesity, behavioral changes, and
elevations in transaminases. Results of laboratory studies are consistent with immunity to hepatitis B and
no evidence of either hepatitis A or C infection. In addition, the alpha-1-antitrypsin value is normal, serum
ceruloplasmin is just above the lower limit of normal, and erythrocyte sedimentation rate (ESR) is 30
mm/hr. Based on these clinical and biochemical data and the knowledge that serum ceruloplasmin is an
acute-phase reactant that may be increased in the presence of an elevated ESR, Wilson disease (WD)
must be ruled out. This may be achieved by performing both a slit lamp examination to determine the
presence of Kayser-Fleischer (K-F) rings (Item C140A), the manifestation of copper deposition in
Descemet membrane of the cornea, and a 24-hour urine copper determination. The serum copper
concentration is not of diagnostic significance for WD in this clinical setting, and the presence of K-F
rings, in conjunction with a 24 hour urine copper value of greater than 40 g is sufficient to diagnose WD.
A liver biopsy (for copper content) may not be required. Abdominal ultrasonography may demonstrate
hepatosplenomegaly in this case but will provide no further diagnostic clues. Assessment for other
possible viral causes, including Epstein-Barr virus infection, is not indicated unless WD is excluded.
WD, also known as hepatolenticular degeneration, was described initially in 1912 by Kinnear
Wilson. If untreated, WD is a fatal disorder characterized by chronic liver disease leading to cirrhosis
along with progressive neurologic deterioration. It is inherited in an autosomal recessive pattern and
results from an abnormal ATP7B gene on chromosome 13. This gene encodes a metal-transporting P-
type adenosine triphosphatase (ATPase), which is expressed primarily in hepatocytes and functions in
the transmembrane transport of copper. Absent or reduced function of ATP7B protein resulting from this
genetic abnormality leads to reduced hepatocellular excretion of copper into bile and the failure to
incorporate copper into the copper-carrying protein, ceruloplasmin. Excessive hepatic copper
accumulation causes hepatocellular injury that leads to inflammatory and, ultimately, cirrhotic changes.
Eventually, copper is released into the bloodstream and deposited in other organs, including the brain,
kidneys, and cornea (hence the appearance of K-F rings).
The clinical presentation of WD may be subtle or overt, depending upon the age at presentation
and extent of copper-induced organ damage. The clinical characteristics of symptomatic WD patients are
listed in (Item C140B). During childhood and early adolescence, other than cases diagnosed on the basis
of family history (16% in one series), the overwhelming majority of patients present with evidence of
hepatocellular dysfunction without neurologic sequelae. In most pediatric series, the mean age of
presentation is around the end of the first decade. Children who have WD are usually evaluated because
of hepatomegaly (82% of patients in one recent series) or persistent transaminase elevations, although in
one series from the United Kingdom, acute liver failure was the presenting sign in almost 50% of patients.
Among older adolescents and adults, psychiatric disturbances and neurologic dysfunction become
apparent, with such signs found in 10% to 40% of reported cases by the end of the second decade. With
increasing age, the psychoneurologic picture tends to predominate.
For the patient who presents with signs of persistent hepatic dysfunction, irrespective of
associated psychiatric or neurologic signs and symptoms, further evaluation follows an established
practice guideline that is approved by the American Association for the Study of Liver Diseases. The
diagnostic algorithm is based on three initial determinations: 1) K-F rings by slit lamp examination, 2)
serum ceruloplasmin, and 3) 24-hour urine copper excretion. The diagnosis of WD is established in
patients who present with K-F rings, a serum ceruloplasmin less than 20 mg/dL (200 mg/L), and a 24-
hour urine copper excretion of greater than 40 g. For patients who do not meet the diagnostic criteria, a
WD diagnosis may be confirmed by determining hepatic copper content on a percutaneous liver biopsy
sample. A hepatic copper concentration of greater than 250 g/g dry weight is diagnostic for WD.
Once the diagnosis is confirmed, treatment with an appropriate copper-chelating agent should
begin immediately. WD represents the uncommon situation of a metabolic disease for which a specific
pharmacologic therapy, if begun early in the disease course, is associated with complete symptom
resolution, normalization of laboratory values (except for serum ceruloplasmin), and an excellent long-
term prognosis. For patients who have hepatic disease, penicillamine, the first copper chelator to be
successfully used for the treatment of WD, is an effective agent. However, penicillamine has been
supplanted by trientine as the drug of first choice. Trientine is also indicated as primary therapy for
patients who have neurologic involvement with or without WD hepatopathy. When patients have been

Copyright 2012 American Academy of Pediatrics


2012 PREP SA ON CD-ROM

stabilized, with normal serum transaminase values, zinc, which interferes with intestinal copper
absorption, may be used either alone or in combination with trientine, accompanied by a low copper-
containing diet.

American Board of Pediatrics Content Specification(s):


Recognize the signs and symptoms of Wilson disease

Suggested Reading:
Arnon R, Calderon JF, Schilsky M, Emre S, Shneider BL. Wilson disease in children: serum
aminotransferases and urinary copper on triethylene tetramine dihydrochloride (trientine) treatment. J
Pediatr Gastroenterol Nutr. 2007;44:596-602. DOI: 10.1097/MPG.0b013e3180467715. Available at:
http://journals.lww.com/jpgn/Fulltext/2007/05000/Wilson_Disease_in_Children__Serum.11.aspx

Das SK, Ray K. Wilson's disease: an update. Nat Clin Pract Neurol. 2006;2:482493. DOI:
10.1038/ncpneuro0291. Available at:
http://www.nature.com/nrneurol/journal/v2/n9/full/ncpneuro0291.html

Koppikar S, Dhawan A. Evaluation of the scoring system for the diagnosis of Wilsons disease in children.
Liver Int. 2005;25:680-681. DOI: 10.1111/j.1478-3231.2005.01072.x:

Manolaki N, Nikolopoulou G, Daikos G,L et al. Wilson disease in children: analysis of 57 cases. J Pediatr
Gastroenterol Nutr. 2009;48:72-77. DOI: 10.1097/MPG.0b013e31817d80b8. Available at:
http://journals.lww.com/jpgn/Fulltext/2009/01000/Wilson_Disease_in_Children__Analysis_of_57_Cases.1
2.aspx

Marcellini M, Di Ciommo V, Callea F, et al. Treatment of Wilsons disease with zinc from the time of
diagnosis in pediatric patients: a single-hospital, 10-year follow-up study. J Lab Clin Med. 2005;145:139-
143. Abstract available at: http://www.ncbi.nlm.nih.gov/pubmed/15871305

Roberts EA, Schilsky ML. Diagnosis and treatment of Wilson disease: an update. Hepatology.
2008;47:2089-2111. DOI: 10.1002/hep.22261. Available at:
http://onlinelibrary.wiley.com/doi/10.1002/hep.22261/full

Copyright 2012 American Academy of Pediatrics


2012 PREP SA ON CD-ROM

Critique 140

(Courtesy of T Koch)
Kayser-Fleischer ring: golden brown ring of copper deposits peripherally at the level of Descemet
membrane in the cornea.

Copyright 2011 American Academy of Pediatrics


2012 PREP SA ON CD-ROM

Critique 140

Item C140B. Clinical Manifestations of Wilson Disease


Hepatic Acute liver failure
Chronic hepatitis
Cirrhosis
Elevated serum transaminases (persistent)
Hepatosplenomegaly

Neurologic Abnormal gait


Choreoathetoid movements
Chronic headaches (migrainelike)
Dysarthria
Dystonia
Insomnia
Parkinsonianlike rigidity
Pseudobulbar palsy
Seizures
Tremors

Ophthalmologic Kayser-Fleischer rings


Sunflower cataracts

Psychiatric Behavioral/personality changes


Depression
Neuroses
Psychosis

Renal (uncommon) Aminoaciduria


Nephrolithiasis

Clinical Manifestations of Wilson Disease.

Copyright 2011 American Academy of Pediatrics


2012 PREP SA ON CD-ROM

Question 141
During a routine health supervision visit, a mother relates that her sister recently gave birth to an
infant who had hypoplastic left heart syndrome. She has just found out she is 6 weeks pregnant with her
second child and is very worried because she has learned that other members of her family have had
children with heart problems over the past three generations. She would like to know how her baby can
be assessed prenatally for heart problems.
Of the following, you are MOST likely to advise her that a fetal heart defect can best be
diagnosed by
A. amniocentesis
B. chorionic villus sampling
C. fetal magnetic resonance imaging
D. maternal serum alpha-fetoprotein concentrations
E. prenatal level 2 ultrasonography

Copyright 2012 American Academy of Pediatrics


2012 PREP SA ON CD-ROM

Critique 141 Preferred Response: E


Prenatal ultrasonography remains the primary method for fetal anatomic imaging because it
permits good visualization of structures that are filled with fluid, allowing identification of genitourinary,
gastrointestinal, central nervous system, and cardiac abnormalities in the developing fetus. Although
second trimester anatomic ultrasonography may be performed as early as 16 weeks gestation, most
scans are performed between 18 and 20 weeks gestation. Published detection rates of major anatomic
abnormalities with second-trimester ultrasonography range between 37% and 55%, with higher rates for
central nervous system and genitourinary tract abnormalities.
Fetal echocardiography should be considered when there is an increased risk of congenital heart
disease, including a family history of congenital heart defects. Fetal echocardiography is ideally
performed between 18 and 20 weeks gestation because of the progressive nature of many cardiac
lesions and the limitations of ultrasonography related to the size of fetal cardiac structures. All standard
anatomic obstetric ultrasonographic imaging should include a four-chamber view of the fetal heart
because abnormalities in this view identify about 50% of all cases of congenital heart disease.
Fetal magnetic resonance imaging (MRI) has the potential to provide images with greater clarity,
but at present it is less commonly used due to its lack of availability, cost, and remaining technical
imaging challenges. Fetal MRI can now be performed without maternal or fetal sedation and has been
demonstrated to allow better assessment of central nervous system anomalies.
Chorionic villus sampling (CVS) performed between 10 and 13 weeks gestation allows early
analysis for cytogenetic, biochemical, and molecular abnormalities. CVS is not routinely performed before
10 weeks gestation due to the risk of limb-reduction defects. Fetal cells obtained by amniocentesis may
be analyzed for cytogenetic, biochemical, and molecular abnormalities, and alpha fetoprotein (AFP)
concentration can be determined from the amniotic fluid. Amniocentesis is generally performed at 15 to
22 weeks gestation due to the increased risk of fetal loss and clubfoot when performed earlier. Maternal
serum AFP concentrations may be elevated in open neural tube defects, defects of the genitourinary and
gastrointestinal system, and Turner syndrome, and decreased concentrations of maternal serum AFP
may suggest an increased risk of trisomy 18 and 21. Maternal serum AFP testing is generally performed
around 16 weeks of gestation. CVS, maternal serum AFP testing, and amniocentesis may reveal a fetal
condition associated with congenital heart disease, but imaging by ultrasonography or fetal
echocardiography is needed to assess prenatally for heart problems.

American Board of Pediatrics Content Specification(s):


Recognize that prenatal ultrasonography can detect major fetal anomalies (eg, hydro- or
anencephalus, myelomeningocele, congenital heart defects, gastrointestinal or genitourinary
abnormalities) as early as 16 weeks' gestation

Suggested Reading:
Bianchi DW. Prenatal genetic diagnosis. In: Taeusch HW, Ballard RA, Gleason CA, eds. Averys
Diseases of the Newborn. 8th ed. Philadelphia, PA: Elsevier Saunders; 2005:186-193

Cunniff C and Committee on Genetics. Prenatal screening and diagnosis for pediatricians. Pediatrics.
2004;114:889-894. DOI: 10.1542/peds.2004-1368. Available at:
http://pediatrics.aappublications.org/cgi/content/full/114/3/889

Reddy UM, Filly RA, Copel JA. Prenatal imaging: Ultrasonography and magnetic resonance imaging.
Obstet Gynecol. 2008;112:145-157. DOI: 10.1097/01.AOG.0000318871.95090.d9. Available at:
http://www.ncbi.nlm.nih.gov/pmc/articles/PMC2788813/?tool=pubmed

Romosan G, Henriksson E, Rylander A, Valentin L. Diagnostic performance of routine ultrasound


screening for fetal abnormalities in an unselected Swedish population in 2000-2005. Ultrasound Obstet
Gynecol. 2009;34:526-533. DOI: 10.1002/uog.6446. Available at:
http://onlinelibrary.wiley.com/doi/10.1002/uog.6446/full

Copyright 2012 American Academy of Pediatrics


2012 PREP SA ON CD-ROM

Question 142
You are seeing a 17-year-old boy in your office for suture removal following a laceration to the left
index finger. He reports that he cut himself while operating a meat slicer at a local deli, where he has
worked since dropping out of school. You have not seen him in your office for 2 years. He has a past
history of attention-deficit/hyperactivity disorder, which you briefly treated with methylphenidate. On
physical examination, in addition to a healing, sutured 2-cm finger laceration, you observe tachycardia
and injected conjunctivae bilaterally. The boy appears nervous and has difficulty recalling details of the
accident at work. You obtain a psychosocial history that includes family functioning, drug history, sexual
history, peer interaction, and mood and anxiety assessment. Upon further questioning, he admits to daily
marijuana use for more than 2 years.
Of the following, the MOST appropriate next step is to
A. advise him to stop use and follow up in 1 month
B. arrange urine drug testing
C. notify his parents of his marijuana use
D. obtain a more detailed substance use history
E. resume methylphenidate therapy

Copyright 2012 American Academy of Pediatrics


2012 PREP SA ON CD-ROM

Critique 142 Preferred Response: D


Substance use disorders are common conditions in pediatric patients, and marijuana is the most
commonly abused illicit drug. In the Monitoring the Future annual survey of twelfth graders, 6.1% reported
daily marijuana use, and 34% reported use in the past year. After marijuana, prescription and
nonprescription medications are most frequently abused. For example, 5.1% of twelfth graders reported
using oxycodone for nonmedical use in the past year. All adolescent patients should be questioned about
use of tobacco, alcohol, and other drugs.
The boy described in the vignette has a significant substance use disorder whose nature and
severity should be ascertained. Whether the problem is treated by the primary care physician or is
referred for more intensive care depends upon that information. A detailed drug history includes life
history of use of each substance, the frequency of use, the context of use, and details regarding the
effects upon the patients life. The physician needs to decide whether the substance use is best
characterized as experimental, problematic, abuse, or dependence.
Experimental use is defined as occasional use without drug-seeking behavior.
Problematic drug use is defined as substance use that has resulted in social, financial,
psychological, physical, or legal problems. Any one of the following criteria is sufficient for the diagnosis
of substance abuse:
1) Recurrent failure to meet important responsibilities
2) Recurrent use in situations when such use is likely to be physically dangerous
3) Recurrent legal problems arising from drug use
4) Continued use despite knowledge of problems caused by or aggravated by use
Substance dependence is diagnosed when three or more of the following criteria are met:
1) Tolerance (needing more to become intoxicated or discovering less effect with same amount
2) Withdrawal (characteristic withdrawal associated with the type of drug)
3) Using more or for longer periods than intended
4) Desire to or unsuccessful efforts to cut down
5) Considerable time spent in obtaining the substance or using or recovering from its effects
6) Important social, work, or recreational activities given up because of use
7) Continued use despite knowledge of problems caused by or aggravated by use
This boys daily use suggests that he has cannabis dependence, a serious health problem that
requires treatment. Merely asking him to stop will not be effective, and seeing him in 1 month is too long
of a follow-up interval to help him understand the urgency of the needed treatment. Drug testing, although
occasionally used to follow a patient in abstinence drug treatment, is not indicated at this time because
drug use is evident from the history and testing might undermine rapport with the patient.
Ideally, the family should be involved to understand the nature of the disorder, learn about
recovery, and support necessary treatment. However, the parents should not be enlisted for help without
first trying to gain the patients assent. Any discussion with the parents should be in the presence of the
boy.
Although the boy has a history of ADHD, treatment at this time with stimulant medication is
unwise because the methylphenidate itself has abuse potential and because treatment of ADHD is a
lower priority than addressing and treating substance abuse.

AAP Mental Health Competency:


Recongize the symptoms of marijuana abuse in a teenager

Suggested Reading:
American Academy of Pediatrics Task Force on Mental Health. Substance Use and Abuse Cluster
Guidance. Addressing Mental Health Concerns in Primary Care: A Clinician's Toolkit [CD-ROM]. Elk
Grove Village, IL: American Academy of Pediatrics; 2010

Gruber AJ, Pope HG Jr. Marijuana use among adolescents. Pediatr Clin North Am. 2002;49:389-413.
Abstract available at: http://www.ncbi.nlm.nih.gov/pubmed/11993290

Copyright 2012 American Academy of Pediatrics


2012 PREP SA ON CD-ROM

McArdle PA. Cannabis use by children and young people. Arch Dis Child. 2006;91:692-695. DOI:
10.1136/adc.2005.071860. Available at:
http://www.ncbi.nlm.nih.gov/pmc/articles/PMC2083022/?tool=pubmed

National Institute on Drug Abuse. High school and youth trends. NIDA InfoFacts. 2011. Available at:
http://www.drugabuse.gov/pdf/infofacts/HSYouthTrends.pdf

Copyright 2012 American Academy of Pediatrics


2012 PREP SA ON CD-ROM

Question 143
A 7-year-old girl complains of 1 day of dysuria, vaginal itching, and perineal pain without fever or
vomiting. She does not have urgency, frequency, or enuresis, but she does report a light yellow
discharge. She denies any type of trauma, including sexual abuse. Her vital signs and abdominal
examination findings are normal, and she has no flank tenderness. Perineal inspection shows a minimal
amount of mucoid vaginal discharge and moderate erythema of the vestibule. There is no evidence of
trauma. A urine dipstick evaluation is negative except for trace leukocyte esterase.
Of the following, the MOST appropriate next step in the evaluation/treatment of this patient is
A. a course of oral antibiotics for urinary tract infection
B. application of bland emollients as needed for symptom relief
C. topical anticandidal cream for 7 days
D. urine testing for gonorrhea and chlamydia
E. vaginoscopy

Copyright 2012 American Academy of Pediatrics


2012 PREP SA ON CD-ROM

Critique 143 Preferred Response: B


Dysuria can be the presenting complaint for a wide variety of conditions in children and
adolescents (Item C143). Although urinary tract infection (UTI) is among the most common causes for
this symptom, it also may result from other conditions of the genitourinary tract, skin and mucous
membrane disease, trauma, and systemic inflammatory syndromes. Among afebrile children who have
dysuria, the probability of UTI is only approximately 17%. History and careful physical examination are
important in differentiating other causes. The history should review information about systemic symptoms
(fever, vomiting, rash), urinary and genital symptoms (frequency, urgency, enuresis, pruritus, vaginal
discharge or bleeding), trauma (straddle injury, foreign body), sexual activity (including sexual abuse and
consensual sexual activity in adolescents), exposure to irritants (detergents, perfumes, bubble baths),
and past history of UTI. Family history may suggest the possibility of stones or metabolic disease.
Physical examination should include careful perineal inspection for both boys and girls, particularly noting
evidence of trauma, rashes, or hypopigmentation and whether boys are circumcised.
The girl described in the vignette has symptoms and signs characteristic of vaginitis, likely
nonspecific vulvovaginitis. This self-limited condition requires no treatment other than symptom relief,
such as bland emollients, as needed. If her condition does not improve over time, vaginoscopy might be
indicated to search for a foreign body, but this generally is not needed at the initial evaluation. Similarly,
with no history of sexual abuse and characteristic physical examination findings, testing for sexually
transmitted organisms can be avoided unless the patient does not improve or new history comes to light.
In an adolescent, Candida albicans is a common cause of vaginitis, but it is rare in an otherwise healthy
school-age, prepubertal child. Antibiotics should be withheld from an afebrile child who has a normal urine
dipstick examination result and no additional symptoms of UTI unless a urine culture becomes positive.

American Board of Pediatrics Content Specification(s):


Recognize that the etiologies of dysuria may be age related and that numerous other etiologies
include vaginitis, chemical irritations, and trauma
Recognize the importance of perineal inspection in girls with dysuria

Suggested Reading:
Farhi D, Wendling J, Molinari E, et al. Non-sexually related acute genital ulcers in 13 pubertal girls: a
clinical and microbiological study. Arch Dermatol. 2009;145:38-45. Available at: http://archderm.ama-
assn.org/cgi/content/full/145/1/38

Fleisher GR. Evaluation of dysuria in children. UpToDate Online 18.3. 2010. Available for subscription at:
http://www.uptodate.com/online/content/topic.do?topicKey=ped_symp/2196

Shaikh N, Morone NE, Lopez J, et al. Does this child have a urinary tract infection? JAMA.
2007;298:2895-2904. Abstract available at: http://www.ncbi.nlm.nih.gov/pubmed/18159059

Supe-Markavina F, Kaskel FJ. Dysuria. In McInerny TK, Adam HM, Campbell DE, Kamat DM, Kelleher
KJ, Hoekelman RA, eds. American Academy of Pediatrics Textbook of Pediatric Care. Elk Grove Village,
IL: American Academy of Pediatrics; 2009:1474-1478

Copyright 2012 American Academy of Pediatrics


2012 PREP SA ON CD-ROM

Critique 143

Item C143. Causes of Dysuria


Urinary ! Urinary tract infection Genital ! Vaginitis
" Upper (pyelonephritis) " Nonspecific
" Lower (cystitis) " Nonsexually transmitted
! Urethritis: Neisseria gonorrhea, " Sexually transmitted
Chlamydia trachomatis " Candida albicans
! Nephrolithiasis ! Cervicitis (sexually
" Hypercalciuria transmitted infection)
" Hyperuricosuria ! Pelvic inflammatory disease
! Dysfunctional voiding ! Balanitis/balanoposthitis
Skin/mucous ! Varicella Inflammatory ! Behet syndrome
membrane ! Stevens-Johnson syndrome diseases ! Conjunctivitis, arthritis, and
! Herpes simplex virus infection urethritis
(sexually transmitted) ! Crohn disease
! Lichen sclerosis et atrophicus
! Acute genital ulcers (Lipschtz
ulcer)
! Labial adhesions
Other ! Trauma
! Pinworms

Copyright 2011 American Academy of Pediatrics


2012 PREP SA ON CD-ROM

Question 144
At the 2-week health supervision visit for a newborn, his mother expresses concern that the baby
has an infection in one eye. She describes yellow discharge that sometimes is crusted on the lashes. She
also has noted increased tearing of the affected eye. There have been no signs of other illness, and the
baby is exclusively breastfeeding well. Physical examination reveals a soft anterior fontanelle and no
erythema of the bulbar or palpebral conjunctivae but a small amount of crusted discharge on the
eyelashes. There is a small palpable mass just inferior to the medial canthus (Item Q144). There are no
skin lesions, and the remainder of the examination findings are normal.
Of the following, the MOST likely diagnosis is
A. bacterial conjunctivitis
B. corneal abrasion
C. dacryocystocele
D. dacryostenosis
E. viral conjunctivitis

Copyright 2012 American Academy of Pediatrics


2012 PREP SA ON CD-ROM

Question 144

(Courtesy of M Rimsza)
Mass, as described for the infant in the vignette.

Copyright 2012 American Academy of Pediatrics


2012 PREP SA ON CD-ROM

Critique 144 Preferred Response: C


Congenital obstruction of the nasolacrimal duct (dacryostenosis) is common in newborns,
occurring in as many as 6% of all infants. A thin, imperforate membrane is found at the distal end of the
duct. A common presentation is increased tearing and crusting of the lashes in the neonate. Often,
parents confuse this with conjunctivitis, but the clinician can differentiate these two phenomena by the
absence of erythema of the conjunctivae and the chronicity of the symptoms. The diagnosis may be
confirmed by extrusion of mucoid discharge with massage of the nasolacrimal duct. Dacryostenosis can
usually be managed expectantly. Parents should be instructed to massage the nasolacrimal area two or
three times a day with warm water on a soft cloth. Topical antibiotics are of little benefit unless there are
signs of conjunctivitis such as erythema of the conjunctiva or significant purulent exudate.
Rarely, nasolacrimal duct stenosis results in formation of a dacryocystocele, which is a firm,
bluish cystic mass located just inferior to the medial canthus of the eye, as described for the infant in the
vignette. This abnormality results from not just a distal obstruction but from failure of canalization,
proximal obstruction of the duct, and resultant trapping of mucus and squamous debris. When suspected,
consultation with an ophthalmologist is indicated.
The clinical picture of corneal abrasion usually is a red eye and increased tearing in an
uncomfortable-appearing infant. Viral and bacterial conjunctivitis usually present with erythema of bulbar
and palpebral conjunctivae and increased tearing. Viral conjunctivitis also usually is bilateral and may be
associated with other signs of viral infection, including fever, rhinorrhea, or cough.

American Board of Pediatrics Content Specification(s):


Recognize the typical presentation of a blocked nasolacrimal duct
Know the appropriate management of a blocked nasolacrimal duct

Suggested Reading:
Olitsky SE, Hug D, Plummer LS, Stass-Isern M. Disorders of the eye: disorders of the lacrimal system. In:
Kliegman RM, Stanton BF, St. Geme JW III, Schor NF, and Behrman RE, eds. Nelson Textbook of
Pediatrics. 19th ed. Philadelphia, PA: Saunders Elsevier; 2011:2165-2166

Takahashi Y, Kakizaki H, Chan WO, Selva D. Management of congenital nasolacrimal duct obstruction.
Acta Ophthalmol. 2010;88:506-513. DOI: 10.1111/j.1755-3768.2009.01592.x. Abstract available at:
http://www.ncbi.nlm.nih.gov/pubmed/19681790

Wong RK, VanderVeen DK. Presentation and management of congenital dacryocystocele. Pediatrics.
2008;122:e1108-e1112. DOI: 10.1542/peds.2008-0934. Available at:
http://pediatrics.aappublications.org/cgi/content/full/122/5/e1108

Copyright 2012 American Academy of Pediatrics


2012 PREP SA ON CD-ROM

Question 145
A 12-year-old girl presents to your office 2 weeks after the onset of a viral respiratory illness. She
recovered from the illness 8 days ago, but she now complains of chest pain, shortness of breath, and
fatigue. On physical examination, her heart rate is 110 beats/min, respiratory rate is 28 breaths/min, blood
pressure is 75/45 mm Hg, and oxygen saturation is 95%. The pale girl exhibits tachypnea, muffled heart
tones without an obvious murmur, a palpable liver 3 cm below the right costal margin, and poor peripheral
pulses. She has minimal palpable chest discomfort when sitting, but the pain on palpation increases when
she is supine. Electrocardiography reveals ST-segment abnormalities and diffusely reduced voltages.
Chest radiography is shown in image (Item Q145).
Of the following, the MOST likely diagnosis is
A. endocarditis
B. myocardial infarction
C. myocarditis
D. pericarditis
E. tachyarrhythmia

Copyright 2012 American Academy of Pediatrics


2012 PREP SA ON CD-ROM

Question 145

(Courtesy of M Lewin)
Chest radiograph, as described for the girl in the vignette.

Copyright 2012 American Academy of Pediatrics


2012 PREP SA ON CD-ROM

Critique 145 Preferred Response: D


The chest radiograph obtained for the girl described in the vignette documents pericarditis, an
inflammatory process that affects the pericardium. It is often associated with a pericardial effusion that
can be serous, sanguinous, purulent, or fibrinous, depending on the cause and how long the effusion has
been present. In children and adolescents, the most common cause is viral (postinfectious pericarditis)
due to the inflammatory process adversely affecting the pericardium. The next most common cause is
idiopathic. Malignant pericardial effusions are associated with the entire range of intrathoracic and
metastatic malignancies, and autoimmune disorders can also result in pericarditis and pericardial
effusions.
After cardiac surgery, approximately 10% of pediatric patients exhibit postpericardiotomy
syndrome, which is characterized by irritability, fever, elevated inflammatory indices, and a pericardial
effusion. The effusion is typically found to be a transudate containing a high proportion of inflammatory
cells. Other sources of a pericardial effusion after open heart surgery include bleeding into the pericardial
sac associated with a postcardiac bypass coagulopathy or suture line oozing and a chylous pericardial
effusion due to disruption of flow through the lymphatic channels to the thoracic duct.
The classic sign of pericarditis is a friction rub auscultated on the cardiovascular examination,
usually at the lower left sternal border. Other physical signs include positional chest pain (pain worsens in
the supine position and lessens when upright); diaphoresis; fatigue; shortness of breath; and the
possibility of pericardial tamponade causing distant and muffled heart sounds, jugular vein distention, and
pulsus paradoxus.
The treatment for viral or idiopathic pericarditis or in the case of postpericardiotomy syndrome is
anti-inflammatory agents. The most common are aspirin and nonsteroidal anti-inflammatory drugs.
Severe cases may require:
Pericardiocentesis to treat pericardial effusion/tamponade
Antibiotics if a bacterial cause is identified
Corticosteroids
Surgery: rarely, a pericardial window or stripping (removal) of the pericardium is required,
particularly in chronic cases where pericardiocentesis has proven unsuccessful
Viral endocarditis is rare in children. Instead of the muffled heart tones described in the vignette,
a murmur of valve insufficiency or stenosis resulting from the infective process affecting one of the
cardiac valves is heard. Myocardial infarction in an otherwise healthy child is also rare, and the chest pain
associated with a myocardial infarction is not positional. Although the electrocardiographic findings might
include ST-segment changes, deep Q waves without reduced overall QRS complex voltages also are
seen. Myocarditis is a relatively common finding in pediatrics, most often due to an underlying viral
infection. However, the presence of positional chest pain and muffled heart tones is not typical. A
tachyarrhythmia is unlikely because this patients heart rate is only minimally elevated and there is no
arrhythmia on electrocardiography. An intermittent arrhythmia is possible, but chest pain is not an
associated finding.

American Board of Pediatrics Content Specification(s):


Know the pathogenesis of pericarditis
Know the treatment of pericarditis, including the importance of surgical drainage

Suggested Reading:
Drossner DM, Hirsh DA, Sturm JJ, et al. Cardiac disease in pediatric patients presenting to a pediatric ED
with chest pain. Am J Emerg Med. 2011;29:632-638. DOI: 10.1016/j.ajem.2010.01.011. Abstract available
at: http://www.ncbi.nlm.nih.gov/pubmed/20627219

Durani Y, Giordano K, Goudie BW. Myocarditis and pericarditis in children. Pediatr Clin North Am.
2010;57:1281-1303. DOI: 10.1016/j.pcl.2010.09.012. Abstract available at:
http://www.ncbi.nlm.nih.gov/pubmed/21111118

Copyright 2012 American Academy of Pediatrics


2012 PREP SA ON CD-ROM

Question 146
A 12-year-old girl who has idiopathic partial epilepsy that has been well controlled with
carbamazepine for 1 year develops an ear infection. Because she is allergic to penicillin, an urgent care
physician prescribes a course of azithromycin. Three days later, she presents to the emergency
department with vomiting and dizziness. On physical examination, she has end-gaze nystagmus in both
horizontal directions and a broad-based gait.
Of the following, the MOST likely explanation for this girls clinical findings is
A. carbamazepine toxicity
B. confusional migraine
C. otitic hydrocephalus
D. postictal presentation
E. unilateral labyrinthitis

Copyright 2012 American Academy of Pediatrics


2012 PREP SA ON CD-ROM

Critique 146 Preferred Response: A


The girl described in the vignette presents with symptoms and signs of anticonvulsant toxicity,
including nausea, dizziness, ataxic gait, and end-gaze nystagmus. The most likely cause is a drug-drug
interaction between carbamazepine, whose dose has been optimized, and the recently prescribed
macrolide antibiotic. Macrolide antibiotics may interfere with carbamazepine metabolism, resulting in an
increased serum concentration and the presentation described. Because different physicians may
prescribe chronic anticonvulsants and short courses of antibiotics, it is important for physicians to check
for drug-drug interactions.
The presence of vertical and bilateral horizontal end-gaze nystagmus makes the other four
diagnoses less likely. Confusional migraine can present with the symptoms reported for this girl, and the
head pain may not be prominent in a child, but there should not be nystagmus. Otitic hydrocephalus
refers to a complication of an ear infection or mastoiditis, when an adjacent sinus venous thrombosis
occurs. The thrombosis obstructs venous outflow that, in turn, reduces normal cerebrospinal fluid
resorption through arachnoid granulations, ultimately raising intracranial pressure (ICP). Symptoms
include vomiting and severe headache that is worse when lying flat. The elevated ICP may cause ocular
malalignment, usually involving cranial nerve 6, but not nystagmus in all directions of gaze. Similarly, end-
gaze nystagmus is not a finding in a postictal state. Labyrinthitis or vestibular neuritis due to infection and
inflammation of the inner ear or vestibular nerve causes classic vertigo and unidirectional nystagmus.

American Board of Pediatrics Content Specification(s):


Know the interactions of anticonvulsants with other drugs, including other anticonvulsants

Suggested Reading:
Mikati MA. Seizures in childhood. In: Kliegman RM, Stanton BF, St. Geme JW III, Schor NF, and
Berhman RE, eds. Nelson Textbook of Pediatrics. 19th ed. Philadelphia, PA: Saunders Elsevier;
2011:2013-2039

Major P, Thiele EA. Seizures in children: laboratory diagnosis and management. Pediatr Rev.
2007;28:405-414. DOI: 10.1542/pir.28-11-405. Available at:
http://pedsinreview.aappublications.org/cgi/content/full/28/11/405

Copyright 2012 American Academy of Pediatrics


2012 PREP SA ON CD-ROM

Question 147
A 10-year-old girl and her 14-year-old brother in your practice both had phenylketonuria (PKU)
diagnosed at birth. With early identification and initiation of dietary restriction, both children have done
well academically and socially. Their mother is looking ahead to their college years and asks about
recommendations regarding ongoing dietary management.
Of the following, you are MOST likely to tell her that dietary restriction can be lifted after age 15
years
A. for both children
B. but must be reinstituted for the girl before she becomes pregnant
C. but must be reinstituted for the girl during pregnancy
D. for the boy only
E. for neither child

Copyright 2012 American Academy of Pediatrics


2012 PREP SA ON CD-ROM

Critique 147 Preferred Response: E


In the past, children who had PKU were advised that they could relax dietary restrictions beyond
childhood, but most metabolic centers in the United States today recommend lifelong dietary treatment for
all patients. This is especially critical for affected girls because fetal exposure to high concentrations of
phenylalanine circulating in the maternal bloodstream pose high risks for teratogenic effects and may
result in congenital anomalies, microcephaly, and intellectual disabilities through in utero exposure.
Accordingly, neither child in the vignette should alter his or her dietary protocols. Past studies have
demonstrated altered concentration abilities and reaction times for individuals who have PKU when they
are off dietary restriction and have elevated circulating phenylalanine concentrations. If a female has PKU
and is not compliant with her phenylalanine-restricted diet, it is essential for her to reinstitute dietary
therapy before conception, with the goal of achieving phenylalanine concentrations of less than 6 mg/dL
(363.3 mcmol/L) at least 3 months before conception. Phenylalanine concentrations should be maintained
between 2 and 6 mg/dL (121.1 and 363.3 mcmol/L) throughout the pregnancy to minimize potential
adverse fetal effects.
Although screening for PKU has been performed in the United States for more than 40 years,
children born outside of the United States may not be screened and rarely are diagnosed before 6
months of age. Rapid evaluation of a positive screening test and prompt institution of dietary protocols
once the diagnosis is confirmed results in normal physical and intellectual development. Dietary treatment
includes the use of foods and medical protein sources (such as special formulas) that are low in
phenylalanine, with the judicious addition of natural proteins to the diet to provide the necessary amount
of phenylalanine for optimal growth and development. Untreated PKU may lead to acquired microcephaly,
delayed speech acquisition, seizures, eczema, permanent intellectual disabilities, and behavioral
abnormalities. More recently, individuals who have hyperphenylalanemia and even classic PKU have
attained some biochemical benefit from the addition of oral tetrahydrobiopterin (a cofactor for the enzyme
phenylalanine hydroxylase) to standard dietary management.

American Board of Pediatrics Content Specification(s):


Be familiar with the long-term prognosis of phenylketonuria and the effects of dietary adherence to
outcome

Suggested Reading:
Committee on Genetics. Maternal phenylketonuria. Pediatrics. 2008;122:445-449. DOI:
10.1542/peds.2008-1485. Available at: http://pediatrics.aappublications.org/cgi/content/full/122/2/445

Kaye CI and the Committee on Genetics. Newborn screening fact sheets. Pediatrics. 2006;118:e934-
e963. DOI: 10.1542/peds.2006-1783. Available at:
http://pediatrics.aappublications.org/cgi/content/full/118/3/e934

Mitchell JJ, Scriver CR. Phenylalanine hydroxylase deficiency. GeneReviews. 2010. Available at:
http://www.ncbi.nlm.nih.gov/bookshelf/br.fcgi?book=gene&part=pku

Copyright 2012 American Academy of Pediatrics


2012 PREP SA ON CD-ROM

Question 148
You are seeing a 13-year-old girl, in whom you diagnosed anorexia nervosa approximately 18
months ago, for a follow-up visit. She had started to restrict her food intake about 6 months before her
first visit. There was no history of binging or purging. She had become progressively more isolated from
her friends and was very anxious and irritable. Currently, her mother states that she is doing well at
school and has one friend. She is eating everything but still in small quantities. She has not had
menarche yet but is otherwise asymptomatic. Her mother had her menarche at age 12 years. On
physical examination, the girl has normal vital signs, a body mass index of 17.4, and no focal findings.
When her father comes in after your examination, he is very upset that she is not cured after all this time
and that they still have to supervise her meals and eating habits. You discuss the usual course of this
illness and prognosis with him.
Of the following, the factor MOST likely to be associated with a poor prognosis for this girl is
A. absence of binging and purging
B. comorbid psychiatric illness
C. early onset of illness (<14 years)
D. good family support
E. short duration of illness

Copyright 2012 American Academy of Pediatrics


2012 PREP SA ON CD-ROM

Critique 148 Preferred Response: B


Although one of the diagnostic criteria for anorexia nervosa (AN) is amenorrhea, the onset of AN
may predate the onset of puberty or may occur in the early pubertal stages, interrupting further
development and resulting in primary rather than secondary amenorrhea, which is the case for the girl
described in the vignette. Prognostic factors for eating disorders have not been reliably identified. Overall,
it is estimated that about 50% of patients do well, about 30% do not do so well, and about 20% do poorly.
The mortality rate of 5% to 10% is the highest among psychiatric disorders, with death resulting from
either suicide or medical complications.
Most longitudinal studies indicate that the onset of AN before adulthood, especially before age 14
years, along with early, intensive treatment is associated with a good prognosis. Other factors that are
good prognosticators are good family support and a shorter duration of illness, suggesting that the
behaviors have not become entrenched and the parents, with guidance, should be able to aid in recovery.
Thus, early recognition and intensive treatment improve the prognosis. Factors associated with a less
positive prognosis include the presence of binging and purging, longer duration of illness before
treatment, poor family relations, and comorbid psychiatric illnesses.

American Board of Pediatrics Content Specification(s):


Know the factors affecting the prognosis for adolescents with anorexia nervosa

Suggested Reading:
Bloomfield D. In brief: secondary amenorrhea. Pediatr Rev. 2006;27:113-114. DOI: 10.1542/pir.27-3-113.
Available at: http://pedsinreview.aappublications.org/cgi/content/full/27/3/113

Fisher M. Treatment of eating disorders in children, adolescents, and young adults. Pediatr Rev.
2006;27:5-16. DOI: 10.1542/pir.27-1-5. Available at:
http://pedsinreview.aappublications.org/cgi/content/full/27/1/5

Kaplowitz PB. Delayed puberty. Pediatr Rev. 2010;31:189-195. DOI: 10.1542/pir.31-5-189. Available at:
http://pedsinreview.aappublications.org/cgi/content/full/31/5/189

Papadopoulos FC, Ekbom A, Brandt L, Ekselius L. Excess mortality, causes of death and prognostic
factors in anorexia nervosa. Br J Psychiatry. 2009;194:10-17. DOI: 10.1192/bjp.bp.108.054742. Available
at: http://bjp.rcpsych.org/cgi/content/full/194/1/10

van Son GE, van Hoeken D, van Furth EF, Donker GA, Hoek HW. Course and outcome of eating
disorders in a primary care-based cohort. Int J Eat Disord. 2010;43:130-138. DOI: 10.1002/eat.20676.
Abstract available at: http://www.ncbi.nlm.nih.gov/pubmed/19308996

Wentz E, Gillberg IC, Anckarster H, Gillberg C, Rstam M. Adolescent-onset anorexia nervosa: 18-year
outcome. Br J Psychiatry. 2009;194:168-174. DOI: 10.1192/bjp.bp.107.048686. Available at:
http://bjp.rcpsych.org/cgi/content/full/194/2/168

Copyright 2012 American Academy of Pediatrics


2012 PREP SA ON CD-ROM

Question 149
You are discussing the upcoming gastric tube placement and Nissen fundoplication surgery with
the physician parents of one of your patients, who has spinal muscular atrophy type I. The boy is 8
months old and has been receiving enteral feeding through a nasogastric tube. It has been difficult to
control his gastroesophageal reflux medically, and the nasogastric tube interferes with achieving a
consistent face mask seal when he uses noninvasive continuous positive airway pressure at night. His
parents are concerned about the anesthesia because they have heard that there is an association
between neuromuscular disorders and malignant hyperthermia.
Of the following, the information regarding malignant hyperthermia that you are MOST likely to
share with these parents is that
A. DNA testing can identify all susceptible patients
B. it occurs most commonly in adults
C. it occurs only with the use of halothane
D. it usually is inherited in an autosomal recessive pattern
E. the mortality rate is less than 5%

Copyright 2012 American Academy of Pediatrics


2012 PREP SA ON CD-ROM

Critique 149 Preferred Response: E


Malignant hyperthermia (MH), although rare, is an anesthetic emergency that must be identified
and treated quickly. MH occurs when an inhalational anesthetic (such as halothane or sevoflurane) or
depolarizing neuromuscular blocker (succinylcholine) triggers a sudden influx of calcium into the
sarcoplasm of susceptible muscles. The resultant hypermetabolic syndrome leads to profound metabolic
and respiratory acidosis, hyperthermia, tachycardia, hypertension, muscle spasms, rigidity, and
rhabdomyolysis. The incidence of MH is estimated to be 1 in 14,000 pediatric anesthesia cases, with
nearly 50% of all events occurring in children younger than 19 years of age. The mortality rate is 5% or
less, but it was estimated to be as high as 70% before the development of effective management
techniques.
Identification of patients at risk for MH involves a careful preanesthetic screening for personal or
family history of anesthetic complications. First-degree relatives of anyone who has had an MH episode
should undergo muscle biopsy, with subsequent in vitro caffeine-halothane contracture testing as well as
genetic analysis because up to 40% of patients have been identified with one of the several genes that
are associated with MH susceptibility. Although previously believed to be an autosomal dominant
disorder, MH appears to have a multifactorial inheritance pattern. Patients who have neuromuscular
conditions such as muscular dystrophy may experience muscular rigidity, myoglobinuria, or hyperkalemic
cardiac arrest after succinylcholine administration. The progression of these symptoms to fulminant MH is
uncertain because the incidence of neuromuscular conditions is approximately equal to the incidence of
MH and, therefore, distinguishing between them is difficult.
Triggering agents should be avoided for patients who have a known or suspected susceptibility to
MH and an alternative anesthetic agent (eg, propofol) should be employed. Such techniques often are
used for patients who have known neuromuscular diseases as well. Immediate treatment of patients in
whom an MH episode is suspected includes immediate cessation of triggering agents, administration of
dantrolene, and termination of the surgical procedure, with further supportive treatment provided for
individual symptoms.

American Board of Pediatrics Content Specification(s):


Recognize the conditions associated with malignant hyperthermia during general anesthesia

Suggested Reading:
Berlin RE, Samuels PJ. Malignant hyperthermia. In: Wheeler DS, Wong HR, Shanley TP, eds. Pediatric
Critical Care Medicine: Basic Science and Clinical Evidence. New York, NY: Springer-Verlag; 2007:1725-
1729

Litman RS. Malignant hyperthermia: clinical diagnosis and management of acute crisis. UpToDate Online
18.3. 2010. Available for subscription at:
http://www.uptodate.com/online/content/topic.do?topicKey=anesth/5205

Litman RS. Susceptibility to malignant hyperthermia. UpToDate Online 18.3. 2010. Available for
subscription at: http://www.uptodate.com/online/content/topic.do?topicKey=anesth/5694

Sarnat HB. Metabolic myopathies: malignant hyperthermia. In: Kliegman RM, Stanton BF, St. Geme JW
III, Schor NF, and Berhman RE, eds. Nelson Textbook of Pediatrics. 19th ed. Philadelphia, PA: Saunders
Elsevier; 2011:2130-2132

Copyright 2012 American Academy of Pediatrics


2012 PREP SA ON CD-ROM

Question 150
A 16-year-old boy presents to your office for his annual health supervision visit. His past medical
history includes chemotherapy and radiation therapy to the head and neck for Hodgkin lymphoma
diagnosed when he was 11 years old. Following his therapy, he has done extremely well, with normal
growth and development. The only medication he takes is levothyroxine. On physical examination, you
palpate a 1.25-cm nontender, mobile nodule in the lower left lobe of his thyroid.
Of the following, the next BEST step is to
A. increase his thyroid hormone replacement
B. perform an I-123 scan
C. refer him for biopsy of the nodule
D. refer him for resection of the nodule
E. refer him for thyroid ultrasonography

Copyright 2012 American Academy of Pediatrics


2012 PREP SA ON CD-ROM

Critique 150 Preferred Response: C


Thyroid nodules are an uncommon finding in pediatrics, affecting fewer than 2% of the
population. However, the likelihood of malignancy is 10% to 15% in children who have thyroid nodules,
which is considerably higher than the risk of thyroid cancer in affected adults. Therefore, it is critical for
general pediatricians to perform thorough neck examinations and refer patients who have abnormal
findings. Children who have been exposed to head, neck, or total body irradiation, such as the boy
described in the vignette, require special care with the thyroid examination because they are at even
higher risk for developing malignant thyroid nodules.
All nodules of greater than 1 cm or those of any size in a high-risk patient that have concerning
findings (hard, irregular border, nonmobile) require immediate referral for biopsy. Nodules less than 1 cm
should be evaluated by ultrasonography and referred for biopsy if ultrasonography reveals increased
blood flow via Doppler, calcifications, or irregular borders because these findings increase the risk for
malignancy. Increasing thyroid hormone replacement for this boy is not appropriate because there is no
evidence of undertreatment. Even if there were such evidence, a biopsy would still be indicated. Neither I-
123 scan nor thyroid ultrasonography is considered a necessary first-line test, although most thyroid
biopsies are now performed using ultrasonographic guidance. Resection of the nodule is a possible
solution in some cases, but having a histologic diagnosis is preferable before planning surgery because
nodules that demonstrate malignancy often require a more aggressive surgical approach than a simple
resection.

American Board of Pediatrics Content Specification(s):


Understand the importance of referral in a child with a thyroid mass/nodule
Know the significance of a previous history of irradiation to the head and neck in a patient with a
thyroid mass/nodule

Suggested Reading:
Corrias A, Mussa A, Baronio F, et al; for the Study Group for Thyroid Diseases of Italian Society for
Pediatric Endocrinology and Diabetology (SIEDP/ISPED). Diagnostic features of thyroid nodules in
pediatrics. Arch Pediatr Adolesc Med. 2010;164:714-719. Abstract available at:
http://www.ncbi.nlm.nih.gov/pubmed/20679162

Gharib H, Papini E, Paschke R. Thyroid nodules: a review of current guidelines, practices, and prospects.
Eur J Endocrinol. 2008;159:493-505. DOI: 10.1530/EJE-08-0135. Available at: http://eje-
online.org/cgi/content/full/159/5/493

Gharib H, Papini E, Paschke R, Duick DS, Valcavi R, Hegeds L, Vitti P; AACE/AME/ETA Task Force on
Thyroid Nodules. American Association of Clinical Endocrinologists, Associazione Medici Endocrinologi,
and European Thyroid Association medical guidelines for clinical practice for the diagnosis and
management of thyroid nodules: executive summary of recommendations. J Endocrinol Invest.
2010;33:287-291. Abstract available at: http://www.ncbi.nlm.nih.gov/pubmed/20479572

Copyright 2012 American Academy of Pediatrics


2012 PREP SA ON CD-ROM

Question 151
A 7-year-old child is squinting when looking at books and complaining of difficulty seeing the
blackboard in his first-grade classroom.
Of the following, the MOST appropriate test of visual acuity for this child is the
A. Allen card test
B. cross cover test
C. HOTV matching test
D. Snellen letter test
E. tumbling E test

Copyright 2012 American Academy of Pediatrics


2012 PREP SA ON CD-ROM

Critique 151 Preferred Response: D


Regular vision screening and eye examinations are necessary to identify conditions that may
affect academic functioning or lead to impaired visual function. Choosing the appropriate vision screening
test to use depends on the cognitive abilities of the child being evaluated. The available vision screening
tests, ranked in order of the level of cognitive ability needed for their use from highest to lowest are:
Snellen letters or Snellen numbers, tumbling E, HOTV matching test, and Allen card test. The test
employed should be the one at the highest level the child is able to perform.
The correct visual acuity screening test for the boy described in the vignette is the Snellen letter
test, which is designed for ages 6 years and older. The tumbling E Test, HOTV matching test, and Allen
card test are designed for ages 3 to 5 years and are indicated when the child is unable to perform vision
testing by letter and numbers. The tumbling E test consists of the letter E in spatial orientations, with the
child needing to identify what direction the legs of the E are pointing. The HOTV matching test consists
of the letters H, O, T, and V on a testing board, which the child needs to match with letters on the wall
chart. If the child is unable to perform either of these tests, the Allen card test may be used. For this test,
the child must be able to look at the pictures shown and either identify verbally or match the pictures
shown on the wall chart. The cross cover test checks for ocular alignment. Any eye movement on the test
is an indication for referral.

American Board of Pediatrics Content Specification(s):


Know the common visual acuity screening tests for preschool and school-age children and the age
range for which they are appropriate

Suggested Reading:
Committee on Practice and Ambulatory Medicine, Section on Ophthalmology. Eye examination and vision
screening in infants, children, and young adults. Pediatrics. 1996;98:153-157. Available at:
http://pediatrics.aappublications.org/cgi/reprint/98/1/153

Olitsky SE, Hug D, Plummer LS, and Stass-Isern M. Examination of the eye. In: Kleigman RM, Stanton
BF, St. Geme JW III, Schor NF, and Behrman RE, eds. Nelson Textbook of Pediatrics. 19th ed.
Philadelphia, PA: Saunders Elsevier; 2011:2148-2187

Copyright 2012 American Academy of Pediatrics


2012 PREP SA ON CD-ROM

Question 152
A 4-year-old girl who has a chronic tracheostomy due to subglottic stenosis is admitted to your
hospital with increased respiratory distress, purulent tracheal secretions, and fever. A Gram stain of
tracheal secretions reveals numerous white blood cells and gram-negative rods.
Of the following, the BEST choice for empiric antibiotic therapy for this child is
A. ampicillin-sulbactam
B. azithromycin
C. clindamycin
D. piperacillin-tazobactam
E. trimethoprim-sulfamethoxazole

Copyright 2012 American Academy of Pediatrics


2012 PREP SA ON CD-ROM

Critique 152 Preferred Response: D


Patients who have chronic tracheostomies, such as the girl described in the vignette, are at an
increased risk for colonization and subsequent localized infection with a number of organisms, including
Staphylococcus aureus, anaerobes, and enteric gram-negative rods such as Pseudomonas aeruginosa.
Antimicrobial therapy for such patients should include coverage for these organisms. Of the agents listed,
piperacillin-tazobactam is the only one that has such broad coverage, including Pseudomonas.
The broad-spectrum coverage of ampicillin-sulbactam includes many enteric gram-negative rods
but not Pseudomonas. Azithromycin, clindamycin, and trimethoprim-sulfamethoxazole do not have
coverage for Pseudomonas.
Antibiotics that have antipseudomonal activity include the antipseudomonal penicillins
(piperacillin, ticarcillin), third- and fourth-generation cephalosporins (ceftazidime, cefepime), carbapenems
(meropenem, imipenem), aminoglycosides (gentamicin, tobramycin, amikacin), aztreonam, and certain
fluoroquinolones (ciprofloxacin, levofloxacin).
Other clinical settings in which Pseudomonas infection needs be considered include fever and
neutropenia, cystic fibrosis, burn wound infections, and chronic urinary tract infections. Pseudomonas
infection is also associated with hot tub folliculitis and osteochondritis occurring after nail puncture
wound to the foot through a sneaker or crush injury to the ear pinna associated with ear piercing.
Treatment of hot tub folliculitis generally does not require antibiotics, and Pseudomonas osteochondritis
requires surgical drainage, with antibiotics as supplementary therapy.

American Board of Pediatrics Content Specification(s):


Know which antibiotics are generally effective against Pseudomonas infections

Suggested Reading:
Giamarellou H, Antoniadou A. Antipseudomonal antibiotics. Med Clin North Am. 2001;85:1942. Abstract
available at: http://www.ncbi.nlm.nih.gov/pubmed/11190351

Kanj SS, Sexton DJ. Treatment of Pseudomonas aeruginosa infections. UpToDate Online 18.3. 2010.
Available for subscription at: www.uptodate.com/online/content/topic.do?topicKey=gram_rod/5770

Copyright 2012 American Academy of Pediatrics


2012 PREP SA ON CD-ROM

Question 153
A 15-year-old girl presents to your office with her mother, who reports that the girl is not acting
like her usual self. The girls mood is euphoric, and she is talking incessantly about herself, telling
everyone that she will be graduating from high school early and attending a prestigious college. She
recently went on shopping spree with her mothers credit card. She has been distracted in school and not
able to pay attention in class. According to the mother, her daughter has only slept 3 hours the past few
nights. She has no history of illicit drug use, and results of a urine drug screen are negative. Detailed
psychosocial history reveals no apparent trigger for this episode or any hallucinatory phenomena.
Findings on physical examination are normal.
Of the following, the MOST likely diagnosis is
A. adjustment disorder of adolescence
B. attention-deficit/hyperactivity disorder
C. bipolar disorder
D. hyperthyroidism
E. schizoaffective disorder

Copyright 2012 American Academy of Pediatrics


2012 PREP SA ON CD-ROM

Critique 153 Preferred Response: C


The girl described in the vignette exhibits symptoms of elevated mood, increased self-esteem,
decreased need for sleep, increased talkativeness, and increased distractibility, which are characteristic
of bipolar disorder. In addition, her symptoms all are occurring in a discrete episode that is a significant
change from her baseline functioning. Bipolar disorder can be difficult to diagnose in children and
adolescents because of the developmental differences in symptom expression, the high rate of comorbid
disorders, and the lack of biologic tests to confirm the diagnosis. The American Psychiatric Association
Diagnostic and Statistical Manual of Mental Disorders-IV criteria for mania (originally developed for
adults) are used to diagnose bipolar disorder. These criteria include a distinct period of abnormally and
persistently elevated, expansive, or irritable mood lasting at least 1 week (or any duration if hospitalization
is necessary). During the period of the mood disturbance, at least three (or four if the mood is irritable) of
the following symptoms must be present:
Increased self-esteem or grandiosity
Decreased need for sleep
More talkative than usual or pressure to keep talking
Flight of ideas or subjective experience that thoughts are racing
Distractibility
Increase in goal-directed activity or psychomotor agitation
Excessive involvement in pleasurable activities that have potential for painful consequences
In addition, the mood disturbance should be severe enough to cause marked impairment in
functioning and not be due to the direct physiologic effects of a substance or medical condition.
Attention-deficit/hyperactivity disorder (ADHD) is a chronic condition that generally presents
before 7 years of age, but this girl has a new onset of symptoms that are also not typical of ADHD,
including euphoria and grandiosity. Although hyperthyroidism may be associated with increased activity
and lack of sleep, euphoria is absent and characteristic findings on the physical examination may be
observed (eg, hyperreflexia, tachycardia, exophthalmos). Adjustment disorder of adolescence requires a
psychosocial stressor triggering emotional-behavioral symptoms within 3 months of the stressor. Although
this girl has symptoms of disorganization, she does not have signs of psychosis, such as hallucinations or
responding to internal stimuli (eg, hearing voices and responding to them). Schizoaffective disorder
involves psychosis as a primary presenting symptom.

AAP Mental Health Competency:


Know that a history of manic episodes of multiple days duration is needed in order to diagnose bipolar
disorder in children

Suggested Reading:
American Psychiatric Association. Mood disorders. In: Diagnostic and Statistical Manual of Mental
Disorders. 4th ed. Text Revision. Washington, DC: American Psychiatric Association; 2000:345-428

Kowatch RA, DelBello MP. Pediatric bipolar disorder: emerging diagnostic and treatment approaches.
Child Adolesc Psychiatr Clin N Am. 2006;15:73-108. DOI: 10.1016/j.chc.2005.08.013. Abstract available
at: http://www.ncbi.nlm.nih.gov/pubmed/16321726

McClellan J, Kowatch R, Findling RL; Work Group on Qualtiy Issues. Practice parameter for the
assessment and treatment of children and adolescents with bipolar disorder. J Am Acad Child Adolesc
Psychiatry. 2007;46:107-125. Abstract available at: http://www.ncbi.nlm.nih.gov/pubmed/17195735

Copyright 2012 American Academy of Pediatrics


2012 PREP SA ON CD-ROM

Question 154
A 10-month-old male infant presents to the emergency department with his third buttock abscess
in the past 3 months. Each infection has occurred in a different place on his buttocks. He was treated
previously with bedside incision and drainage and oral cephalexin, and no cultures were sent. On
physical examination, he has a temperature of 38.0C, heart rate of 110 beats/min, and respiratory rate of
30 breaths/min. He does not appear toxic and has moist mucous membranes. The lateral aspect of his
left buttock, along the border of his diaper, has a 3x3-cm area of erythema, warmth, induration, and
central fluctuance. The remainder of his physical examination findings are within normal parameters.
Of the following, the MOST appropriate antimicrobial therapy for this patient is
A. amoxicillin
B. ceftriaxone
C. cephalexin
D. clindamycin
E. doxycycline

Copyright 2012 American Academy of Pediatrics


2012 PREP SA ON CD-ROM

Critique 154 Preferred Response: D


A significant proportion of community-associated (CA) Staphylococcus aureus strains are
methicillin-resistant. Therefore, empiric antimicrobial therapy for skin and soft-tissue infections (SSTI), as
described for the infant in the vignette, should include activity against methicillin-resistant S aureus
(MRSA), especially in communities that have a significant MRSA prevalence. Clindamycin has activity
against susceptible strains of MRSA and is appropriate therapy for this child. Clindamycin also can treat
SSTI caused by Streptococcus pyogenes. Doxycycline can be used to treat MRSA SSTI in children 8
years of age and older, but its activity against S pyogenes is variable. Cephalexin and ceftriaxone have
activity against S pyogenes and methicillin-susceptible S aureus (MSSA) but not against MRSA.
Amoxicillin can treat infections caused by S pyogenes but not those caused by MSSA or MRSA.
The recurrent skin abscesses, such as described for the child in the vignette, usually require
incision and drainage in addition to antimicrobial therapy. Skin abscesses are collections of purulent
material within the dermis and deeper skin tissues. Furunculosis is the development of a small abscess
associated with a hair follicle in which purulent material extends through the dermis into the subcutaneous
tissue. In otherwise healthy children, the most common pathogens causing skin abscesses and
furunculosis are S aureus and S pyogenes.
S aureus colonizes the skin (anterior nares, axillae, perineum, rectum, vagina, throat) of up to
50% of healthy children. Colonization is a risk factor for infection. Most CA-MRSA infections manifest as
SSTI. In immunocompetent patients who are afebrile and have small skin abscess or furuncles due to
MRSA, incision and drainage alone may be sufficient for resolution of disease; the procedure also
provides the opportunity to obtain a culture and susceptibility data to guide further antimicrobial
management, if necessary. In previously healthy patients who are febrile and have larger skin abscesses
or furuncles, incision and drainage in addition to antimicrobial therapy is recommended.

American Board of Pediatrics Content Specification(s):


Know that methicillin-resistant Staphylococcus aureus (MRSA) is a common cause of skin and soft
tissue infections
Know the most important component of therapy (ie, incision and drainage) for methicillin-resistant
Staphylococcus aureus furunculosis)

Suggested Reading:
American Academy of Pediatrics. Staphylococcal infections. In: Pickering LK, Baker CJ, Kimberlin DW,
Long SS, eds. Red Book: 2009 Report of the Committee on Infectious Diseases. 28th ed. Elk Grove
Village, IL: American Academy of Pediatrics; 2009:601-615

Stevens DL, Bisno AL, Chambers HF, et al. Practice guidelines for the diagnosis and management of skin
and soft-tissue infections. Clin Infect Dis. 2005;41:1373-1406. DOI: 10.1086/497143. Available at:
http://cid.oxfordjournals.org/content/41/10/1373.long

Copyright 2012 American Academy of Pediatrics


2012 PREP SA ON CD-ROM

Question 155
A 9-month-old girl is brought to the clinic for a follow-up visit after an episode of acute
pyelonephritis due to Escherichia coli 1 month ago. She was treated with intravenous antibiotics for 48
hours and discharged with a prescription for trimethoprim/sulfamethoxazole (TMP/SMX) for an additional
8 days. Upon completion of her treatment doses of antibiotics, she was started on TMP/SMX as a
prophylactic agent. Three weeks after hospital discharge, she underwent voiding cystourethrography,
which revealed grade II reflux on the left side. Review of systems today reveals good urine output and
one soft stool per day.
Of the following, the MOST appropriate next step is to
A. change TMP/SMX to amoxicillin
B. continue TMP/SMX
C. discontinue prophylactic antibiotics
D. prescribe oxybutynin chloride
E. prescribe polyethylene glycol

Copyright 2012 American Academy of Pediatrics


2012 PREP SA ON CD-ROM

Critique 155 Preferred Response: B


The present standard of care in the management of acute pyelonephritis is a 10-day treatment
course with antibiotics followed by prophylactic antibiotic administration. Children younger than 5 years of
age should undergo voiding cystourethrography (VCUG) as soon as convenient. The consensus
statement from the American Academy of Pediatrics Committee on Quality Improvement-Subcommittee
on Urinary Tract Infection recommends prescription of antimicrobials in prophylactic doses to infants and
young children (2 months to 2 years) following a urinary tract infection (UTI) until imaging studies are
completed and assessed.
The recommendation for outpatient management of UTI in children is to assess for risk factors.
Anatomic abnormalities such as hydronephrosis or renal cysts can be seen on ultrasonography, and
vesicoureteral reflux (VUR) can be seen in one third of infants and children who have febrile UTI. Other
risk factors in toddlers who are toilet-trained include constipation and dysfunctional voiding. Constipation
is strongly associated with UTI and appears to lead to bladder neck compression and resultant urinary
retention.
The young girl described in the vignette had unilateral grade II VUR identified on VCUG following
an episode of acute pyelonephritis. The recommended prophylactic antibiotic of choice for infants and
children is trimethoprim/sulfamethoxazole (TMP/SMX) due to its favorable antibiotic spectrum and low
risk of adverse effects. Accordingly, this therapy should be continued for this girl. Amoxicillin is preferred
for infants younger than 2 months of age to avoid sulfa-related complications in this age group. However,
amoxicillin should not be used as a prophylactic agent beyond 2 months of age due to increasing
resistance of Escherichia coli. Because this child is not constipated, using polyethylene glycol as a
laxative is not necessary at present. Oxybutynin chloride is an anticholinergic agent that is sometimes
prescribed for symptoms of frequency or enuresis if it is due to uninhibited bladder contractions. Recently,
conflicting reports have surfaced regarding the benefits of antibiotic prophylaxis in children who have UTI.
A large multicenter study entitled Randomized Intervention for Children With Vesicoureteral Reflux
(RIVUR) is ongoing. The goals of this study are to determine the value of prophylactic antibiotics in
reducing the likelihood of renal scarring or UTI recurrence.

American Board of Pediatrics Content Specification(s):


Know the indication for long-term antibiotic prophylaxis against urinary tract infection

Suggested Reading:
American Academy of Pediatrics; Committee on Quality Improvement; Subcommittee on Urinary Tract
Infection. Practice parameter: the diagnosis, treatment, and evaluation of the initial urinary tract infection
in febrile infants and young children. Pediatrics. 1999;103:843-852. Available at:
http://pediatrics.aappublications.org/cgi/content/full/103/4/843

Feld LG, Mattoo TK. Urinary tract infections and vesicoureteral reflux in infants and children. Pediatr Rev.
2010;31:451-463. DOI: 10.1542/pir.31-11-451. Available at:
http://pedsinreview.aappublications.org/cgi/content/full/31/11/451

Mattoo TK. Are prophylactic antibiotics indicated after a urinary tract infection? Curr Opin Pediatr.
2009;21:203206. Available at: http://www.ncbi.nlm.nih.gov/pmc/articles/PMC2725357/?tool=pubmed

Copyright 2012 American Academy of Pediatrics


2012 PREP SA ON CD-ROM

Question 156
You are evaluating a 2-week-old boy for his first health supervision visit after he was discharged
from the hospital following an uneventful birth. The parents are concerned because their son has been
having noisy breathing during both inspiration and expiration for the past week, which is increasing in
frequency. They report that he is eating well and has not had fever or nasal congestion. On physical
examination, you note that the infant has regained his birthweight but has inspiratory stridor. A chest
radiograph demonstrates normal lung fields without infiltration or consolidation. The heart size is normal,
but the aortic arch is right-sided.
Of the following, the MOST likely explanation for the neonates stridor is
A. gastroesophageal reflux
B. laryngomalacia
C. subglottic laryngeal web
D. vascular ring
E. vocal cord paralysis

Copyright 2012 American Academy of Pediatrics


2012 PREP SA ON CD-ROM

Critique 156 Preferred Response: D


The stridor and a right-sided aortic arch described for the infant in the vignette suggest a
diagnosis of vascular ring. Although most right-sided aortic arches are asymptomatic, ones that encircle
the trachea and esophagus can present with different signs and symptoms. Children who have right-sided
aortic arches should be evaluated for a chromosome 22q11 deletion, which also may be associated with
hypocalcemia and cardiac defects. The double aortic arch is the most common vascular ring and often
presents within the first few weeks after birth (Item C156A). Noisy breathing with stridor that does not
change with position or a bronchodilator is a classic symptom. Patients can also present with emesis,
choking, or an apparent life-threatening event. A vascular ring also may be associated with a congenital
laryngeal web. Isolated congenital laryngeal webs can present similarly to a vascular ring, but they are
less common.
Vocal cord paralysis may be unilateral (Item C156B) or bilateral. Stridor may be present, but more
common symptoms include a weak cry, coughing, and choking. Intubation, surgery, or traumatic forceps
delivery are common preceding events. The uneventful birth and absence of a weak cry in this infant
make vocal cord paralysis unlikely.
Laryngomalacia is the most common cause of congenital stridor and likely is due to delayed
maturation of the larynx. The typical presentation is chronic inspiratory noise rather than the biphasic
noise described for this infant. In addition, the onset of symptoms is often later, occurring at 4 to 6 weeks
and sometimes as late as 2 to 3 months of age. A classic laryngoscopic finding is an omega-shaped
epiglottis (Item C156C).
Because laryngeal tissues are sensitive to acid, gastroesophageal reflux (GER) can manifest as
stridor or hoarseness in infants. When symptomatic, however, most infants present with coughing or
emesis. Fortunately, although most babies experience GER, symptoms generally resolve without
treatment by age 6 months. The lack of coughing and emesis make GER less likely for this infant.
When evaluating abnormal breathing noises, the clinician should consider classifying the noise as
primarily inspiratory, expiratory, or biphasic. Stridor is a harsh inspiratory noise that is generally caused
by conditions above the level of the vocal cords; disorders inferior to the vocal cords result in expiratory
wheezing. Congenital stridor generally occurs within the first postnatal month. A detailed history
specifically related to noise changes associated with feeding or position is important, as is documentation
of any congenital anomalies or complications during birth. A radiologic evaluation beginning with a chest
radiograph followed by additional studies as indicated (eg, computed tomography scan, barium
esophagraphy, and/or fiberoptic laryngoscopy and bronchoscopy) usually establishes the diagnosis.

American Board of Pediatrics Content Specification(s):


Understand the appropriate approach to the evaluation of congenital stridor

Suggested Reading:
Goudy S, Bauman N, Manaligod J, Smith RJ. Congenital laryngeal webs: surgical course and outcomes.
Ann Otol Rhinol Laryngol. 2010;119:704-706. Abstract available at:
http://www.ncbi.nlm.nih.gov/pubmed/21049857

Vicencio AG, Parikh S, Adam HS. In brief: laryngomalacia and tracheomalacia: common dynamic airway
lesions. Pediatr Rev. 2006;27:e33-e35. DOI: 10.1542/pir.27-4-e33. Available at:
http://pedsinreview.aappublications.org/cgi/content/full/27/4/e33

Warren JB, Anderson JM. Newborn respiratory disorders. Pediatr Rev. 2010;31:487-496. DOI:
10.1542/pir.31-12-487. Available at: http://pedsinreview.aappublications.org/cgi/content/full/31/12/487

Copyright 2012 American Academy of Pediatrics


2012 PREP SA ON CD-ROM

Critique 156

(Courtesy of A Johnson)
In a double aortic arch, persistent paired embryonic arches encircle the trachea and esophagus.
RSA=right subclavian artery, RCCA=right common carotid artery, LCCA=left common carotid artery,
LSA=left subclavian artery.

Copyright 2011 American Academy of Pediatrics


2012 PREP SA ON CD-ROM

Question 157
A 13-year-old wrestler comes to the office with an injury to his right ear. He reports that his ear
struck the mat during practice, and it became swollen shortly thereafter. He was not wearing his
protective head gear when this occurred. On physical examination, you note that the right external ear is
markedly swollen (Item Q157) and the normal architecture of the pinna is distorted. His tympanic
membrane is intact.
Of the following, the MOST appropriate next step in the treatment of this patients injury is to
A. apply a pressure dressing over the ear
B. apply ice packs to ear
C. aspirate the blood from the hematoma
D. prescribe acetaminophen for pain
E. prescribe antibiotics

Copyright 2012 American Academy of Pediatrics


2012 PREP SA ON CD-ROM

Question 157

(Courtesy of the Media Lab at Doernbecher)


Swelling, as described for the boy in the vignette.

Copyright 2012 American Academy of Pediatrics


2012 PREP SA ON CD-ROM

Critique 157 Preferred Response: C


The boy described in the vignette has an auricular hematoma, an injury that results from shearing
forces that separate the skin and perichondrium of the pinna from the cartilage, tearing the perichondrial
blood vessels. The hematoma that forms may cause avascular necrosis of the cartilage or new cartilage
formation, with subsequent permanent deformity of the pinna (aka wrestlers or cauliflower ear).
Evacuation of the hematoma is a relatively simple procedure when performed within 7 days of injury and
prevents formation of the pinna deformity. Acetaminophen may be recommended for pain, ice packs may
be applied initially, and a pressure dressing may be applied after evacuation, but none of these
interventions or antibiotics alone are sufficient treatment.
The hematoma can be evacuated either by aspiration or incision and drainage. Some authorities
report that reaccumulation is more common after simple aspiration. Local anesthesia by direct infiltration
or auricular block should be provided. The most fluctuant area should be aspirated using an 18-gauge
needle or incised with a 15-blade scalpel, followed by suctioning of the fluid and clot. After evacuation, a
compression dressing that conforms to the normal contour of the ear should be applied. The ear should
be examined daily for reaccumulation or signs of infection. Although there are no studies demonstrating
benefit, antibiotics to prevent infection with skin flora are recommended. Recurrent auricular hematomas
and those older than 7 days should be referred to a specialist for treatment.

American Board of Pediatrics Content Specification(s):


Know the diagnosis and treatment of a hematoma of the external ear

Suggested Reading:
Giles WC, Iverson KC, King JD, Hill FC, Woody EA, Bouknight AL. Incision and drainage followed by
mattress suture repair of auricular hematoma. Laryngoscope. 2007;117:2097-2099. DOI:
10.1097/MLG.0b013e318145386c. Abstract available at: http://www.ncbi.nlm.nih.gov/pubmed/17921905

Leybell I. Drainage, auricular hematoma. eMedicine Specialties, Clinical Procedures. 2009. Available at:
http://emedicine.medscape.com/article/82793-overview

Mudry A, Pirsig W. Auricular hematoma and cauliflower deformation of the ear: from art to medicine. Otol
Neurotol. 2009;30:116-120. DOI: 10.1097/MAO.0b013e318188e905. Abstract available at:
http://www.ncbi.nlm.nih.gov/pubmed/18800018

Copyright 2012 American Academy of Pediatrics


2012 PREP SA ON CD-ROM

Question 158
You are asked to see a 16-year-old boy who has jaundice. He was well until 2 months ago, when
he developed symptoms of an upper respiratory tract infection and scleral icterus was noted. At that time,
blood tests showed:
Hemoglobin, 14.5 g/dL (145 g/L)
Albumin, 4.2 g/dL (42 g/L)
Aspartate aminotransferase, 20 units/L
Alanine aminotransferase, 22 units/L
Bilirubin (total), 4.5 mg/dL (76.9 mcmol/L)
Bilirubin (conjugated), 0.2 mg/dL (3.4 mcmol/L)
His respiratory symptoms resolved after 1 week and his jaundice cleared. At his office visit today,
he appears well and is anicteric. Physical examination demonstrates no abnormalities. Follow-up
laboratory evaluation documents:
Hemoglobin, 14.2 g/dL (142 g/L)
Albumin, 4.3 g/dL (43 g/L)
Aspartate aminotransferase, 22 units/dL
Alanine aminotransferase, 20 units/dL
Bilirubin (total), 2.4 mg/dL (41.0 mcmol/L)
Bilirubin (conjugated), 0.1 mg/dL (1.7 mcmol/L)
Of the following, the elevated bilirubin in this patient MOST likely results from
A. alpha-1-antitrypsin deficiency
B. Crigler-Najjar syndrome
C. Dubin-Johnson syndrome
D. Gilbert syndrome
E. hepatitis A infection

Copyright 2012 American Academy of Pediatrics


2012 PREP SA ON CD-ROM

Critique 158 Preferred Response: D


The mild increase in unconjugated bilirubin that becomes clinically apparent during an acute
respiratory illness and otherwise normal liver function test results described for the boy in the vignette
represent a typical presentation for Gilbert syndrome (GS), a benign disorder of bilirubin conjugation. GS
is an autosomal recessive condition resulting from a mutation in the UGT-1 gene that impairs the
functioning of the enzyme UDP-glucuronyl transferase. Clinical jaundice is noted most often in
adolescents and young adults during periods of fasting, intercurrent illness, or physical stress. Both daily
and seasonal variations are seen, although hyperbilirubinemia is mild and, by definition, less than 6
mg/dL (102 mol/L). Up to one third of patients have normal bilirubin values. Although GS has been
reported to complicate other conditions associated with hyperbilirubinemia, including neonatal physiologic
jaundice and the hemoglobinopathies, patients who have GS are otherwise healthy and exhibit no
associated hepatic or hematologic abnormalities. No clinical sequelae are associated with GS.
Accordingly, no further diagnostic testing is indicated for the boy in the vignette, and both he and his
family should be reassured about the benign nature of the condition.
Crigler-Najjar syndrome (types I and II) is an extremely rare, autosomal recessive condition
caused by absent (type I) or limited (type II) UDP-glucuronyl transferase activity. The disorder generally
presents in the first few days after birth. In type I disease, serum unconjugated bilirubin values of 25 to 35
mg/dL (425 to 600 mcmol/L) or higher are common, and infants are at high risk of developing kernicterus
unless aggressive phototherapy is initiated early. In contrast, patients who have type II disease (and
rarely may present at an older age) usually have unconjugated bilirubin concentrations of less than 20
mg/dL (350 mcmol/L) and respond dramatically to treatment with phenobarbital, an inducer of the partially
active UDP-glucuronyl transferase enzyme. Another disorder of bilirubin metabolism that presents in the
newborn period is Dubin-Johnson syndrome. This condition, along with Rotor syndrome, is a rare
disorder of hepatocellular excretion of conjugated bilirubin. Characteristic findings include elevations in
the direct-reacting (conjugated) bilirubin fraction but without other evidence of cholestasis or hepatic
dysfunction.
Alpha-1-antitrypsin (A1AT) deficiency is an autosomal recessive disorder that may present with a
cholestatic picture in newborns who express the Pi ZZ phenotype, identified by a characteristic serum
electrophoretic migration pattern. The condition is not strictly a deficiency syndrome, but rather results
from misfolding of the A1AT protein, thus preventing its hepatocellular export. Available data indicate that
only 10% of newborns who have the Pi ZZ phenotype develop symptomatic liver disease with a clinical
picture of cholestasis and elevations in serum transaminases. In these infants, A1AT deficiency must be
considered in the differential diagnosis of neonatal direct hyperbilirubinemia. Although 50% of older Pi ZZ
patients may develop transaminase elevations at some point, clinical expression of the disorder usually is
characterized by obstructive pulmonary disease. Finally, infection with the hepatotropic virus hepatitis A
may be ruled out for this boy on the basis of normal serum transaminase values.

American Board of Pediatrics Content Specification(s):


Recognize the clinical presentation of a child with Gilbert syndrome

Suggested Reading:
Bancroft JD, Kreamer B, Gourley GR. Gilbert syndrome accelerates development of neonatal jaundice. J
Pediatr. 1998;132:656-660. Abstract available at: http://www.ncbi.nlm.nih.gov/pubmed/9580766

Borlak J, Thum T, Landt O, Erb K, Hermann R. Molecular diagnosis of a familial nonhemolytic


hyperbilirubinemia (Gilbert's syndrome) in healthy subjects. Hepatology. 2000;32:792-795. DOI:
10.1053/jhep.2000.18193. Available at:
http://onlinelibrary.wiley.com/doi/10.1053/jhep.2000.18193/abstract

Burchell B, Hume R. Molecular genetic basis of Gilbert's syndrome. J Gastroenterol Hepatol.


1999;14:960-966. DOI: 10.1046/j.1440-1746.1999.01984.x. Available at:
http://onlinelibrary.wiley.com/doi/10.1046/j.1440-1746.1999.01984.x/full

Copyright 2012 American Academy of Pediatrics


2012 PREP SA ON CD-ROM

Hsieh TY, Shiu TY, Huang SM, et al. Molecular pathogenesis of Gilbert's syndrome: decreased TATA-
binding protein binding affinity of UGT1A1 gene promoter. Pharmacogenet Genomics. 2007;17:229-236.
DOI: 10.1097/FPC.0b013e328012d0da. Abstract available at:
http://www.ncbi.nlm.nih.gov/pubmed/17496722

Pashankar D, Schreiber RA. Jaundice in older children and adolescents. Pediatr Rev. 2001;7:219-226.
DOI: 10.1542/pir.22-7-219. Available at: http://pedsinreview.aappublications.org/cgi/content/full/22/7/219

Strassburg CP. Pharmacogenetics of Gilbert's syndrome. Pharmacogenomics. 2008;6:703-715. DOI:


10.2217/14622416.9.6.703. Abstract available at: http://www.ncbi.nlm.nih.gov/pubmed/18518849

Copyright 2012 American Academy of Pediatrics


2012 PREP SA ON CD-ROM

Question 159
You are attending a delivery at term for a woman who received limited prenatal care. Upon
reviewing the chart while awaiting the delivery, you find that she first presented for prenatal care at the
beginning of the second trimester. Limited screening was done, but you note that the maternal serum
alpha-fetoprotein concentration measured at 16 weeks gestation was described as low.
Ultrasonography was scheduled, but the mother did not show up for the appointment. She was lost to
follow-up after that time, and no further was performed.
Of the following, the MOST likely potential diagnosis for the infant is
A. anencephaly
B. omphalocele
C. spina bifida
D. tracheoesophageal fistula
E. trisomy 21

Copyright 2012 American Academy of Pediatrics


2012 PREP SA ON CD-ROM

Critique 159 Preferred Response: E


Maternal serum alpha-fetoprotein (msAFP) first emerged as a screening tool for open neural tube
defects in the fetus in the early 1970s. Alpha-fetoprotein (AFP) is produced by the fetal yolk sac,
gastrointestinal tract, and liver, with concentrations in the fetus and amniotic fluid significantly higher than
those in the maternal serum. AFP normally enters the amniotic fluid through fetal urine, with
transplacental and transamniotic diffusion leading to the maternal serum concentrations. In conditions
such as neural tube (eg, anencephaly, spina bifida) and ventral abdominal wall (eg, gastroschisis,
omphalocele) defects, where the AFP can enter the amniotic fluid as a transudate across the exposed
membranes and blood vessels, the increased AFP in amniotic fluid translates to elevated maternal serum
values. For unclear reason, fetuses that have tracheoesophageal fistulas may also have increased AFP
concentrations. Low msAFP levels, as reported for the woman in the vignette, may be associated with
fetuses affected by trisomy 18 and 21 and are felt to be related to decreased hepatic production.
Maternal screening in pregnancy has grown in complexity since the development of msAFP
testing. Many centers offer a first-trimester screen between the 11th and 13th weeks of pregnancy. It
includes three parts: maternal serum blood testing for pregnancy-associated plasma protein (PAPP-A)
and human chorionic gonadotropin (hCG) as well as ultrasound measurement of the fetal nuchal
translucency. This testing permits the potential identification of fetuses at risk for certain conditions
including trisomy 21 and trisomy 18, but it does not detect neural tube defects.
Although msAFP testing may be done separately, it is generally included as part of second
th th
trimester pregnancy screening. This triple screen is performed optimally between the 16 and 18 weeks
of pregnancy. It encompasses measurement of three maternal serum markers: msAFP, HCG, and estriol.
th th
In some centers, a quad screen is done between the 16 and 18 weeks. This screen is similar to the
triple screen, but it also includes inhibin-A measurement. These two screening strategies, in combination
with maternal parameters, allow the potential identification of neural tube defects, abdominal wall defects,
and certain chromosomal disorders in pregnancy.
A positive screening result in the first or second trimester should prompt further discussion,
including potential consultation with a genetic counselor. Abnormal screening tests require additional
testing to confirm a diagnosis, as false-positive results are possible.

American Board of Pediatrics Content Specification(s):


Know the significance of abnormal maternal serum alpha-fetoprotein concentrations
Know that the measurement of maternal serum alpha fetoprotein concentration is a useful screening
test for the diagnosis of open neural tube defects in a fetus

Suggested Reading:
American Academy of Pediatrics, Committee on Genetics. Prenatal screening and diagnosis for
pediatricians. Pediatrics. 2004; 114:889-894. Available at:
http://pediatrics.aappublications.org/content/114/3/889.full.pdf+html

Malone FD et al. First-trimester or second-trimester screening, or both, for Downs syndrome. NEJM.
2005; 353:2001-2011.

Philip AGS, Wald NJ. Historical perspectives: maternal serum alpha-fetoprotein and fetal abnormalities.
NeoReviews. 2004;5:e507-e510. DOI: 10.1542/neo.5-12-e507. Available at:
http://neoreviews.aappublications.org/cgi/content/full/5/12/e507

Copyright 2012 American Academy of Pediatrics


2012 PREP SA ON CD-ROM

Question 160
A 7-year-old girl is brought to your clinic with complaints of vaginal bleeding for 2 days. The child
is appropriately grown, with height and weight in the 50th percentile. She has had no chronic diseases, is
taking no medications, has no skin conditions, and is using no topical creams. She denies any trauma or
sexual abuse. She has felt well and has had no respiratory or gastrointestinal symptoms, although two
family members recently had diarrhea. The mothers menarche was at age 12 years. On physical
examination, the well-appearing girl has no breast or pubic hair development or skin lesions. Perineal
inspection reveals only a small amount of vaginal discharge that is blood-tinged, with otherwise normal
findings for a prepubertal girl.
Of the following, the MOST appropriate diagnostic test is
A. computed tomography scan of the brain
B. follicle-stimulating hormone/luteinizing hormone measurement
C. karyotype
D. pelvic examination under anesthesia
E. vaginal culture for Shigella

Copyright 2012 American Academy of Pediatrics


2012 PREP SA ON CD-ROM

Critique 160 Preferred Response: E


The assessment of abnormal vaginal bleeding in a child first requires determination of the
patients Sexual Maturity Rating (SMR). The girl described in the vignette has prepubertal secondary
sexual characteristics (SMR 1), is normally grown for age, and has normal findings on physical
examination, including perineal inspection, except for vaginal discharge. This suggests that an endocrine,
central nervous system, or chromosomal cause is unlikely. An examination under anesthesia is reserved
for patients who have active bleeding, if there are concerns for vaginal trauma, or for patients who are
unresponsive to conservative management. Shigella culture is appropriate for this girl because Shigella
vaginitis is one of the most common causes of prepubertal vaginal bleeding.
Prepubertal causes of vaginal bleeding can be categorized as traumatic, foreign body-related,
anatomic, infectious, neoplastic, dermatologic, and (rarely) genetic (Item C160). The most common cause
of benign bleeding is a foreign body, often toilet paper, in the vaginal vault. Symptoms are persistent
vaginal discharge, intermittent bleeding, and foul-smelling odor. The foreign body may be seen by careful
perineal inspection, particularly with the patient in the knee-chest position. The foreign body may be
removed with a swab or forceps or by warm vaginal wash in a cooperative child or with the use of
sedation; general anesthesia is reserved for particularly difficult cases. Infectious vaginitis can be due to a
number of respiratory and enteric pathogens. Although group A Streptococcus is the most common cause
of prepubertal bacterial vaginitis (usually without bleeding), Shigella vaginitis is a frequent cause of
vaginal bleeding. As in the case described in the vignette, either the patient or family members may have
diarrhea, but Shigella vaginitis has been found even without concomitant gastrointestinal symptoms.

American Board of Pediatrics Content Specification(s):


Know the causes of vaginal bleeding in the prepubertal girl

Suggested Reading:
Davis AJ, Katz VL. Pediatric and adolescent gynecology: gynecologic examination, infections, trauma,
pelvic mass, precocious puberty. In: Katz VL, Lentz GM, Lobo RA, Gershenson DM, eds. Comprehensive
Gynecology. 5th ed. Philadelphia, PA: Mosby Elsevier; 2007:chapter 13

Fischer G, Rogers M. Vulvar disease in children: a clinical audit of 130 cases. Pediatr Dermatol.
2000;17:1-6. DOI: 10.1046/j.1525-1470.2000.01701.x. Abstract available at:
http://www.ncbi.nlm.nih.gov/pubmed/10720979

Jasper JM. Vulvovaginitis in the prepubertal child. Clin Pediatr Emerg Med. 2009;10:10-13. DOI:
10.1016/j.cpem.2009.01.003. Abstract available at: http://www.clinpedemergencymed.com/article/S1522-
8401(09)00006-8/abstract

Kokotos F, Adam HM. In brief: vulvovaginitis. Pediatr Rev. 2006;27:116-117. DOI: 10.1542/pir.27-3-116.
Available at: http://pedsinreview.aappublications.org/cgi/content/full/27/3/116

Laufer MR, Emans SJ. Vulvovaginal complaints in the prepubertal child. UpToDate Online 18.3. 2010.
Available with subscription at:
http://www.uptodate.com/online/content/topic.do?topicKey=ped_symp/18681

Copyright 2012 American Academy of Pediatrics


2012 PREP SA ON CD-ROM

Critique 160

Item C160. Causes of Prepubertal Vaginal Bleeding


Category Causes
Anatomic Urethral prolapse
Labial adhesions
Dermatologic Lichen sclerosus et atrophicus
Friable warts
Foreign body
Infectious Respiratory pathogens
Enteric pathogens
Trauma Straddle injury
Sexual abuse
Tumor Sarcoma botryoides
Endodermal sinus tumors
Rare causes McCune-Albright syndrome (rare
presentation)

Copyright 2011 American Academy of Pediatrics


2012 PREP SA ON CD-ROM

Question 161
A young mother in your practice presents for the 6-month health supervision visit for her third
child whom she is breastfeeding. The older children are 2 and 4 years of age. The 4-year-old child
recently required extensive dental extractions and capping of the deciduous teeth. You note that the 2-
year-old is carrying a baby bottle of juice in the examination room. The infant you are examining has two
lower incisors.
Of the following, the MOST appropriate advice to give this mother about her childrens dental
health is to
A. await eruption of the upper incisors before arranging a dental appointment for the infant
B. begin brushing the babys teeth with toothpaste
C. continue breastfeeding the infant because it may prevent caries
D. offer juice only from a cup to the 2-year-old child
E. reassure her that dental caries are not hereditary

Copyright 2012 American Academy of Pediatrics


2012 PREP SA ON CD-ROM

Critique 161 Preferred Response: D


Families have become increasingly reliant on their primary care physicians for advice on oral
health because of the shortage of pediatric dentists in many communities and an increasing awareness of
the impact of dental health on the overall health of both children and adults. Dental disease has been
implicated recently in the health of pregnant women and the cardiovascular health of older adults. It has
been known for a long time that dental caries in children may affect a healthy diet, self-esteem, and oral
comfort. Further, dental caries in early childhood represents a large financial burden due to the cost of
tooth extractions, dental examinations, and surgeries under anesthesia. Suppurative complications may
include dental abscess, buccal cellulitis, preseptal and orbital cellulitis, and hematogenous seeding
resulting in osteomyelitis.
Historically, caries in young children have been attributed largely to feeding practices such as
sleeping with or propping a bottle and delayed weaning. However, there is also evidence that other
factors, including a family history of dental caries, transmission of Streptococcus mutans from mother to
infant, and parenting practices, may increase the risk for caries. Socioeconomic barriers to dental care
and ethnic and genetic factors also may increase the risk.
For the family in the vignette, the most appropriate advice is to counsel the mother about the risk
of giving her children sugary drinks such as juice in a bottle. Breastfeeding has not been shown to be
either causative or protective in development of childhood caries. It is not appropriate to advise the
mother to brush her infants teeth with toothpaste, although she should be advised on dental hygiene and
proper nutrition. For infants, simply wiping the teeth with a cloth after feeding is satisfactory. Many parents
are surprised to learn that dental care begins with the eruption of the first teeth and that pediatric dentists
are interested in providing the first dental visit as soon as the first tooth erupts. Some parents equate
dental visits with painful procedures and x-rays rather than simple oral evaluation, attention to hygiene,
and professional cleaning, which are routine parts of pediatric dental visits.
The pediatrician should make early dental referral a part of anticipatory guidance at infant health
supervision visits and should take a family dental history. The American Academy of Pediatrics
recommends the first dental visit by age 1 year. Practical guidance for weaning, introduction of the cup at
age 6 months, avoidance of sugary beverages and snacks, and cleaning of teeth nonintrusively from an
early age is crucial to parent education.
In some areas and settings, application of dental fluoride varnish by the pediatrician is available
and reimbursed by insurers.

American Board of Pediatrics Content Specification(s):


Know how to counsel parents regarding their child's dental care
Recognize the relationship of dietary intake and feeding habits to dental health

Suggested Reading:
Beil HA, Rozier RG. Primary health care providers' advice for a dental checkup and dental use in children.
Pediatrics. 2010;126:e435-e441. DOI: 10.1542/peds.2009-2311. Available at:
http://pediatrics.aappublications.org/cgi/content/full/126/2/e435

de Oliveira C, Watt R, Hamer M. Toothbrushing, inflammation, and risk of cardiovascular disease: results
from Scottish Health Survey. BMJ. 2010;340:C2451. DOI: 10.1136/bmj.c2451. Available at:
http://www.bmj.com/content/340/bmj.c2451.short

Hauser A, Fogarasi S. Periorbital and orbital cellulitis. Pediatr Rev. 2010;31:242-249. DOI:
10.1542/pir.31-6-242. Available at: http://pedsinreview.aappublications.org/cgi/content/full/31/6/242

Kramer MS, Vanilovich I, Matush L, et al. The effect of prolonged and exclusive breast-feeding on dental
caries in early school-age children. New evidence from a large randomized trial. Caries Res.
2007;41:484488. DOI: 10.1159/000108596. Abstract available at:
http://www.ncbi.nlm.nih.gov/pubmed/17878730

Copyright 2012 American Academy of Pediatrics


2012 PREP SA ON CD-ROM

Lewis C, Lynch H, Richardson L. Fluoride varnish use in primary care: what do providers think?
Pediatrics. 2005;115: e69-e76. DOI: 10.1542/peds.2004-1330. Available at:
http://pediatrics.aappublications.org/cgi/content/full/115/1/e69

Section on Pediatric Dentistry. Oral health risk assessment timing and establishment of the dental home.
Pediatrics. 2003;111:1113-1116. Available at:
http://pediatrics.aappublications.org/cgi/content/full/111/5/1113

Slayton RL. Fluoride facts: what pediatricians need to know about fluoride agents for children, including
supplementation. AAP News. 2010;31(March):30

Sriraman N. Pediatric residents can provide oral health screening and treatment. AAP Grand Rounds.
2007 Sep;18:27-28. Extract available at:
http://aapgrandrounds.aappublications.org/cgi/content/extract/18/3/27

Copyright 2012 American Academy of Pediatrics


2012 PREP SA ON CD-ROM

Question 162
A 5-year-old boy presents to the emergency department 30 minutes after he ingested some of his
mothers tricyclic antidepressant. Over the ensuing hour of observation in the emergency department, he
develops lethargy, irritability, and autonomic nervous system findings of mydriasis, dry mouth, and urinary
retention. Within 3 hours of ingestion, these symptoms have resolved.
Of the following, the MOST appropriate next step in management is
A. chest radiography and arterial blood gas
B. discharge home without further evaluation
C. electrocardiography and continuous cardiac monitoring
D. serum electrolytes assessment
E. tricyclic serum drug concentration assessment and discharge home

Copyright 2012 American Academy of Pediatrics


2012 PREP SA ON CD-ROM

Critique 162 Preferred Response: C


An overdose of any of the tricyclic antidepressant drugs may result in a fatal cardiac dysrhythmia.
Often, the effect on the heart is delayed; presentation to the emergency department with various other
signs and symptoms may precede the cardiac findings. Such earlier features may include lethargy,
irritability, anticholinergic effects upon the autonomic nervous system associated with sympathetic
nervous system dysfunction, and seizures. Anticholinergic effects of tricyclic antidepressants include dry
mouth, blurred vision, tachycardia, urinary retention, constipation, dizziness, and vomiting.
The cardiac toxicity associated with this class of agents most often manifests as a prolongation of
the QRS complex on electrocardiographic monitoring. Therefore, the boy described in the vignette should
receive electrocardiography and continuous cardiac monitoring. In addition, atrioventricular block can
occur, and prolongation of the PR interval (first-degree heart block) is common. The QRS complex
prolongation is caused by delayed conduction through a poisoned myocardium. As toxic effects on the
myocardium worsen, loss of cardiac mechanical activity and severely widened QRS complexes occur
simultaneously. Ventricular tachyarrhythmias are a common late finding. Electromechanical dissociation
(EMD) can result, leading to the need for urgent cardiopulmonary resuscitation. EMD does not respond to
electrical cardioversion, instead requiring cardiac support with extracorporeal membrane oxygenation or
some other form of ventricular assist device. If there is metabolic acidosis, infusion of sodium bicarbonate
is appropriate. Two mechanisms are postulated for its therapeutic effect. Tricyclic antidepressants are
protein-bound and become less bound in more acidic conditions. By reversing the acidosis, protein
binding increases and bioavailability decreases. An alternative explanation is that the sodium load helps
to reverse the sodium channel-blocking effects of the tricyclic antidepressants. Treatment is otherwise
supportive.
Respiratory depression can result from the central nervous system sedative properties of these
agents, and if present, warrants careful laboratory and clinical observation. However, these findings do
not appear late, and this boy has no indication of respiratory compromise. Discharging the boy after 3
hours of observation in the emergency department is not appropriate, given the potential for late cardiac
effects. Although serum electrolyte imbalance could exacerbate any ventricular dysrhythmia, tricyclic
antidepressant toxicity does not directly affect serum electrolyte concentrations or renal function. The
tricyclic antidepressants are highly metabolized by the cytochrome P450 hepatic enzymes. Measuring
serum concentrations of the tricyclic antidepressants may be useful in assessing the potential for toxicity,
but such findings alone should not lend reassurance when early noncardiac symptoms are present. In this
situation, further inpatient monitoring is always indicated.

American Board of Pediatrics Content Specification(s):


Understand that cardiac dysrhythmias may occur late after ingestion of tricyclic antidepressants

Suggested Reading:
Kerr GW, McGuffie AC, Wilkie S. Tricyclic antidepressant overdose: a review. Emerg Med J.
2001;18:236-241. DOI: 10.1136/emj.18.4.236. Available at:
http://www.ncbi.nlm.nih.gov/pmc/articles/PMC1725608/?tool=pubmed

Rosenbaum T, Kou M. Are one or two dangerous? Tricyclic antidepressant exposure in toddlers. J Emerg
Med. 2005;28:169-174. DOI: 10.1016/j.jemermed.2004.08.018. Abstract available at:
http://www.ncbi.nlm.nih.gov/pubmed/15707813

Singh N, Singh KH, Khan IA. Serial electrocardiographic changes as a predictor of cardiovascular toxicity
in acute tricyclic antidepressant overdose. Am J Ther. 2002;9:75-79. Abstract available at:
http://www.ncbi.nlm.nih.gov/pubmed/11782822

Copyright 2012 American Academy of Pediatrics


2012 PREP SA ON CD-ROM

Question 163
The mother of a 7-year-old boy brings in her son because he has been staring off into space in
his second-grade classroom. The teacher is concerned that he may be having seizures.
Of the following, the feature of staring spells that is MOST consistent with absence epilepsy is
A. body rocking
B. greater than 1 minute duration
C. interruption of play
D. limb twitching
E. preserved responsiveness

Copyright 2012 American Academy of Pediatrics


2012 PREP SA ON CD-ROM

Critique 163 Preferred Response: C


Descriptions of a child frequently staring off suggest the possibility of absence epilepsy.
Absence epilepsy is a childhood syndrome characterized by a number of key features that differentiates it
from complex partial staring seizures and other causes of staring (Item C163).
The cause of absence epilepsy is unknown, although the pathophysiology involves characteristic,
highly synchronized, diffuse bursts of brain electrical activity seen on electroencephalography that
correspond to the period of lost awareness. These events occur unpredictably, causing an abrupt
cessation of ongoing behavior, such as interruption of play. The treatment of choice for absence epilepsy
is ethosuximide, based on both efficacy and adverse effect profile. Even when absence epilepsy is
effectively treated, however, problems with inattention may persist. Valproic acid is is an alternative but is
generally reserved for atypical absence epilepsies.
Body rocking may be a stereotypy, during which a child may stare off, but there is no loss of
consciousness. This represents dissociation, not seizure. Limb twitching or hand flapping are also not
characteristic of absence seizures. Preserved responsiveness to external stimuli is characteristic of bored
staring, behavioral dissociation, or stereotypies but is not true of seizures. The duration of absence
seizures is usually brief, just a few seconds, not more than 1 minute.

American Board of Pediatrics Content Specification(s):


Understand the drugs used to treat absence epilepsy
Recognize the characteristics of absence epilepsy

Suggested Reading:
Glauser TA, Cnaan A, Shinnar S, et al; Childhood Absence Epilepsy Study Group. Ethosuximide, valproic
acid, and lamotrigine in childhood absence epilepsy. N Engl J Med. 2010;362:790-799. DOI:
10.1056/NEJMoa0902014. Available at:
http://www.ncbi.nlm.nih.gov/pmc/articles/PMC2924476/?tool=pubmed

Major P, Thiele EA. Seizures in children: laboratory diagnosis and management.Pediatr Rev.
2007;28:405-414. DOI: 10.1542/pir.28-11-405. Available at:
http://pedsinreview.aappublications.org/cgi/content/full/28/11/405

Rosenow F, Wyllie E, Kotagal P, Mascha E, Wolgamuth BR, Hamer H. Staring spells in children:
descriptive features distinguishing epileptic and nonepileptic events. J Pediatr. 1998;133:660-663.
Available at: http://www.ncbi.nlm.nih.gov/pubmed/9821425

Sadleir LG, Farrell K, Smith S, Connolly MB, Scheffer IE. Electroclinical features of absence seizures in
childhood absence epilepsy. Neurology. 2006;67:413-418. Abstract available at:
http://www.ncbi.nlm.nih.gov/pubmed/16894100

Copyright 2012 American Academy of Pediatrics


2012 PREP SA ON CD-ROM

Critique 163

Item C163. Characteristics of Conditions Involving Staring

Behavioral Staring
Partial Epilepsy With Complex
Feature Absence Epilepsy Dissociation From The
Partial Seizures
Environment

Onset Unpredictable Unpredictable Predictable eg, when child is


bored

Appearance Staring with glassy eyes Staring with glassy eyes Staring only

Interruptible? No No Yes; may require touch or


loud voice

Automatic Sometimes Often Rarely, although children


behaviors such who have repetitive
as chewing or lip behaviors such as tics or
smacking stereotypies may have
staring

Duration Seconds Seconds to minutes Variable

Consistency of Highly consistent Highly consistent Not very consistent


overall
appearance and
duration of
events

Ictus offset Abrupt; child returns to Gradual; child has postictal Immediate
normal awareness and confusion or sleepiness
activity

Frequency Many episodes daily Less frequent than Variable


absence; may be daily or
less often

Copyright 2011 American Academy of Pediatrics


2012 PREP SA ON CD-ROM

Question 164
A mother brings her 11-year-old son to your office after he refused to go to school due to
periumbilical abdominal pain for the third time this week. The abdominal pain is not associated with
diarrhea, vomiting, fever, or bloody stools. He has missed an average of 2 to 3 days of school per week
over the past few months. According to the mother, his pain is less severe on weekends, does not wake
him from sleep, and is not associated with weight loss. She states that he is an average student, and on
further questioning, he denies problems with other children in the school or bullying. On physical
examination, the boy is afebrile and his height and weight are at the 25th percentile for age. Abdominal
examination reveals no guarding, tenderness, rebound, or organomegaly. Occult stool blood testing
yields negative results. You order a complete blood count, erythrocyte sedimentation rate, and urinalysis
and tell the mother you would like to see them in 1 week to discuss test results. As you are leaving the
room, the mother requests a note from you for the school to provide home tutoring.
Of the following, the MOST appropriate next best step in management is to
A. create a rapid return-to-school plan for the child
B. recommend that the child change schools
C. refer the child to a gastroenterologist
D. start a trial of proton pump inhibitors
E. write the note for the child as requested by the mother

Copyright 2012 American Academy of Pediatrics


2012 PREP SA ON CD-ROM

Critique 164 Preferred Response: A


The boy described in the vignette is exhibiting school avoidance. Such avoidance might arise
from social stress, family dysfunction, bullying, poor school performance, or other primary psychiatric
disorder. Regardless of the cause, the most important initial treatment is to help the family implement a
rapid return to school plan. Characteristics of this plan include sending the child to school daily regardless
of discomfort (but not if vomiting or exhibiting symptoms of a new problem other than the chronic
functional abdominal pain), assigning a consistent family member to transport the child to school,
establishing communication between family and school personnel, and assuring close medical follow-up.
A gastrointestinal-focused intervention such as referral to a gastroenterologist or a trial of a
proton pump inhibitor is not indicated because of the normal physical examination findings and the
negative history reported for the boy. Changing schools might be indicated if there was a history of
bullying or there are other concerns about his safety in the current school. However, without such a
history, changing schools should be avoided because it will likely have negative effects on his educational
achievement and does not address his need to develop adaptive coping strategies.
Children who fail to attend school have been divided into two clinically significant groups: school
avoiders (who are typically absent with parental permission) and truants (who are absent without
permission). Those who avoid school for 1 month have a documented greater dropout rate and have
sought psychiatric evaluation by age 18 at a greater rate than the population that does not avoid school.
Overlaps exist between the groups, with both evidencing higher rates of childhood psychiatric disorders
than peers who do not avoid school. The incidence of conduct disorders and depression is higher among
truants, and anxiety disorders are more common in school avoiders. Individuals who have a mixture of
truancy and school avoidance have the highest rate of psychiatric disorders.

AAP Mental Health Competency:


Recognize that school refusal is considered to be a relative psychiatric emergency

Suggested reading: references


Egzer HL, Costello EJ, Angold A. School refusal and psychiatric disorders: a community study. J Am
Acad Child Adolesc Psychiatry. 2003;42:797-807. Abstract available at:
http://www.ncbi.nlm.nih.gov/pubmed/12819439

Knollmann M, Knoll S, Reissner V. Metzelaars J, Hebebrand J. School avoidance from the point of view
of child and adolescent psychiatry: symptomatology, development, course, and treatment. Dtsch Arztebl
Int. 2010;107:43-49. DOI: 10.3238/arztebl.2010.0043. Available at:
http://www.ncbi.nlm.nih.gov/pmc/articles/PMC2822958/?tool=pubmed

Copyright 2012 American Academy of Pediatrics


2012 PREP SA ON CD-ROM

Question 165
A 2-year-old boy in your practice is doing well because of immediate institution of dietary
galactose restriction following the diagnosis of classic galactosemia at 14 days of age. Although he
initially had mild jaundice, his liver function is normal, and the mild cataracts noted in infancy seem to
have resolved. His parents ask about long-term developmental and medical consequences of
galactosemia.
Of the following, you are MOST likely to tell them that
A. affected individuals are at high risk for the development of cataracts in early adulthood
B. affected men have significant risks for infertility
C. affected school-age children are at risk for learning disabilities
D. dietary nonadherence is associated with long-term neurologic deficits
E. preschool children on dietary restriction rarely have any speech delays

Copyright 2012 American Academy of Pediatrics


2012 PREP SA ON CD-ROM

Critique 165 Preferred Response: C


Although dietary control can eliminate some of the more severe or life-threatening consequences
of high concentrations of galactose in galactosemia, potential sequelae still can occur. Developmental
delays and speech delays are seen with high frequency even in children who maintain good dietary
control. Developmental challenges include attentional difficulties, visuospatial problems, and difficulties
with mathematics. In prospective studies, more than 50% of children being treated for galactosemia had
speech defects, including decreased vocabulary acquisition, articulation problems, or verbal apraxia.
Almost 50% of these children also exhibited developmental delays when followed to 6 years of age, with
formal testing showing mildly depressed developmental or intelligence quotients.
Although cataracts are identified in about 30% of affected infants, 50% of these are mild,
transient, and primarily evident in the neonatal period. The risk for premature ovarian failure in girls who
have galactosemia is high (>80%), but affected boys have normal fertility. As noted previously, good
dietary therapy does not completely prevent developmental consequences, but such delays are more
significant in infants initially treated after 2 months of age. Specific neurologic deficits, including tremors
and extrapyramidal signs, may be identified in about 18% of affected individuals older than 5 years of
age, and they are not reduced by good dietary control. Substantial growth restriction also may be seen in
affected children through adolescence, but many of these individuals eventually achieve normal adult
heights.

American Board of Pediatrics Content Specification(s):


Be familiar with the long-term prognosis and chronic management of galactosemia

Suggested Reading:
Elsas LJ II. Galactosemia. GeneReviews. 2010. Available at:
http://www.ncbi.nlm.nih.gov/bookshelf/br.fcgi?book=gene&part=galactosemia

Kaye CI and the Committee on Genetics. Newborn screening fact sheets. Pediatrics. 2006;118:e934-
e963. DOI: 10.1542/peds.2006-1783. Available at:
http://pediatrics.aappublications.org/cgi/content/full/118/3/e934

Copyright 2012 American Academy of Pediatrics


2012 PREP SA ON CD-ROM

Question 166
You are sharing the results of laboratory testing with the mother of a set of fraternal twins aged 16
years and at Sexual Maturity Rating 5. The girl has a hemoglobin (Hgb) of 12.2 g/dL (122 g/L), with a
mean corpuscular volume (MCV) of 85 fL. The boys Hgb is 13.1 g/dL (131 g/L), with an MCV of 80 fL.
They both are active adolescents and, other than occasional complaints of tiredness, are asymptomatic.
Of the following, the MOST appropriate interpretation of the evaluation is that
A. both adolescents have iron deficiency anemia
B. both adolescents need folic acid supplements
C. the boy is anemic and needs iron medication
D. the girl is anemic and needs iron medication
E. the results are normal in both adolescents

Copyright 2012 American Academy of Pediatrics


2012 PREP SA ON CD-ROM

Critique 166 Preferred Response: C


The hemoglobin (Hgb) value increases in males during pubertal progression because of
increasing androgen concentrations. In contrast, the value in females stays steady because of the
combination of lower androgen concentrations and menstrual losses. It is important to correlate laboratory
values for adolescents with their Sexual Maturity Rating (SMR). In white males, the mean Hgb is 13.2
g/dL (132 g/L) at SMR 1, rising to 15.4 g/dL (154 g/L) (range of 14.0 to 17.0 g/dL [140 to 170 g/L]) at
SMR5. The median hematocrit (Hct) at SMR1 is 41% (36% to 45%) rising to 46% (41% to 50%) at SMR
5. The mean MCV rises from 82.8 to 88.2 fL at SMR 5. The Hgb is slightly lower in African American
males who have a mean of 14.6 g/dL (146 g/L) at SMR5. In white females at SMR5, the mean Hgb is
13.4 g/dL (134 g/L) (range, 11.9 to 15.1 g/dL [119 to 151 g/L]), mean Hct is 39.6% (range, 0.36% to
0.45%), and mean MCV is 89.6 fL (range, 82 to 99 fL).
The Hgb and MCV values for the girl described in the vignette fall within the acceptable range,
but her brothers values are in the anemic range. During adolescence, the need for iron in both sexes is
increased because of rapid growth and increased blood volume and muscle mass. Further, iron
deficiency anemia is common in both sexes because of poor nutritional habits and in girls due to
menstrual losses. Active adolescents who have borderline values may benefit from iron supplements, but
this boy needs therapeutic doses of iron to treat his anemia.

American Board of Pediatrics Content Specification(s):


Know that after puberty, the normal hemoglobin concentration in males is 14 to 18 g/dL

Suggested Reading:
Carswell JM, Stafford DEJ. Normal physical growth and development. In Neinstein LS, Gordon CM,
Katzman Dk, Rosen DS, Woods ER, eds. Adolescent Health Care: A Practical Guide. 5th ed.
Philadelphia, PA: Lippincott Williams & Wilkins, a Wolters Kluwer business; 2008:3-26

Hero M, Wickman S, Hanhijrvi R, Siimes MA, Dunkel L. Pubertal upregulation of erythropoiesis in boys
is determined primarily by androgen. J Pediatr. 2005;146:245-252. Abstract available at:
http://www.ncbi.nlm.nih.gov/pubmed/15689918

Looker AC, Gunter EW, Cook JD, et al. Comparing serum ferritin values from different population
surveys. National Center for Health Statistics. Table 3. Levels of selected iron status indicators of persons
20-44 years of age, by sex and national origin: NHANES II, HHANES, and NHANES III pilot studies. Vital
Health Stat. 1991;2(111). Available at: http://www.cdc.gov/nchs/data/series/sr_02/sr02_111.pdf

Richardson M. Microcytic anemia. Pediatr Rev. 2007;28:5-14. DOI: 10.1542/pir.28-1-5. Available at:
http://pedsinreview.aappublications.org/cgi/content/full/28/1/5

Copyright 2012 American Academy of Pediatrics


2012 PREP SA ON CD-ROM

Question 167
You are caring for a 16-year-old girl who was admitted to the hospital for sepsis. She required 40
mL/kg of isotonic fluid in the emergency department for hypotension, but she says that she now feels
much better. She is laughing with her mother about getting out of a couple of days of school and
catching up on my TV shows. Her temperature is 38.2C, heart rate is 90 beats/min, respiratory rate is
35 breaths/min, blood pressure is 100/60 mm Hg, and oxygen saturation is 98% on 1 L of oxygen via
nasal cannula. An arterial blood gas shows a pH of 7.28, PaCO2 of 30 mm Hg, and PaO2 of 100 mm Hg.
Of the following, the MOST likely cause of the patients tachypnea is her
A. anxiety
B. fever
C. metabolic acidosis
D. metabolic alkalosis
E. pain

Copyright 2012 American Academy of Pediatrics


2012 PREP SA ON CD-ROM

Critique 167 Preferred Response: C


The girl described in the vignette is experiencing an elevated respiratory rate to compensate for
her underlying metabolic acidosis due to sepsis. Her arterial blood gas measurement demonstrates a
partially compensated metabolic acidosis, as evidenced by a decreased pH of 7.28 and decreased
PaCO2 of 30 mm Hg. Her current affect (indicating the absence of pain or anxiety) and her minimally
elevated temperature would not explain her tachypnea.
Respiratory rates vary across a relatively wide range in pediatric patients, depending on factors
such as age and activity status. A patients respiratory drive can be affected by variety of stimuli, including
hypoxia, lung irritation or stretch, acidosis, central nervous system disease, fever, pain, anxiety, or
medications. Breathing faster than normal (tachypnea) often is a response to an underlying disease
process and a compensatory response to maintain normal physiologic concentrations of oxygen or
carbon dioxide. Strict definitions of tachypnea are difficult to determine and always must be considered in
association with other factors such as current clinical status and individual history. Hyperventilation can
be defined as breathing faster or deeper than what is metabolically required. Hyperventilation may be
primary, as seen in central nervous system diseases such as trauma or infection or with medications such
as salicylates and caffeine, or due to stimuli such as pain or anxiety.

American Board of Pediatrics Content Specification(s):


Distinguish between tachypnea that is compensatory versus hyperventilation

Suggested Reading:
Greenbaum LA. Electrolyte and acid-base disorders: acid-base balance. In: Kliegman RM, Stanton BF,
St. Geme JW III, Schor NF, and Behrman RE, eds. Nelson Textbook of Pediatrics. 19th ed. Philadelphia,
PA: Saunders Elsevier; 2011:212-249

Rose BD. Simple and mixed acid-base disorders. UpToDate Online 18.3. 2010. Available for subscription
at: http://www.uptodate.com/online/content/topic.do?topicKey=fldlytes/30198

Copyright 2012 American Academy of Pediatrics


2012 PREP SA ON CD-ROM

Question 168
A 12-year-old girl presents with a 2-week history of nausea, headache, and increased thirst. Her
growth chart is shown in image (Item Q168). Review of systems yields no findings of note except for
recurrent yeast infections and one episode of enuresis over the past 6 months. Urine dipstick shows large
glucose and negative nitrites, leukocyte esterase, and ketones. Blood glucose measures 224 mg/dL (12.4
mmol/L).
Of the following, the MOST appropriate initial treatment for this patient is
A. acarbose
B. insulin
C. metformin
D. chlorpropamide
E. rosiglitazone

Copyright 2012 American Academy of Pediatrics


2012 PREP SA ON CD-ROM

Question 168

(Courtesy of M Haller)

Copyright 2012 American Academy of Pediatrics


2012 PREP SA ON CD-ROM

Critique 168 Preferred Response: C


The girl described in the vignette has type 2 diabetes. For all patients who have diabetes (both
pediatric and adult), the American Diabetes Association recommends that initial therapies include both
medical nutrition therapy (lifestyle changes) and pharmacologic intervention. The drug of choice for most
patients who have type 2 diabetes, including the girl described in the vignette, is metformin. Metformin
has a relatively low cost, tolerable adverse effect profile, and efficacy in improving glycemic control and
other diabetes-related comorbidities. Metformin decreases hepatic glucose production and enhances
insulin sensitivity, resulting in lowered blood glucose concentrations, an improved lipid profile, and mild
weight loss. Gastrointestinal adverse effects, such as abdominal discomfort and diarrhea, are the most
common complaints (20% to 30%) related to metformin use. However, these effects can be minimized by
using extended-release formulations of the drug and slowly titrating the dose. Metformin is
contraindicated for patients who have severe renal or hepatic disease and should be withheld before
radiographic studies requiring the administration of radiocontrast agents. In addition, young women
should be advised that the drug improves ovarian function and could increase the risk of pregnancy if
they are sexually active and are not using appropriate contraception.
Although other oral hypoglycemic agents, such as acarbose, chlorpropamide, and rosiglitazone,
may be effective in improving glycemic control, few if any, have similar long-term safety or efficacy data in
children. Insulin is the therapy of choice for patients who have new-onset diabetes and severe glucose
toxicity (ie, those presenting with diabetic ketoacidosis or hyperosmotic hyperglycemic state) but is not
typically needed to control hyperglycemia in those who have newly diagnosed type 2 diabetes without
ketonuria and with only mild hyperglycemia, such as this girl.
With the increasing number of overweight and obese youth, the presentation of type 2 diabetes in
children has increased considerably. Many more children will be at risk for developing type 2 diabetes as
adults than developing frank diabetes during childhood, but pediatricians should perform appropriate
case-finding studies in their high-risk patients and appreciate standard-of-care treatments for children
who have type 2 diabetes.

American Board of Pediatrics Content Specification(s):


Formulate the treatment approaches to type 2 diabetes

Suggested Reading:
Anand SG, Mehta SD, Adams WG. Diabetes mellitus screening in pediatric primary care. Pediatrics.
2006;118:1888-1895. DOI: 10.1542/peds.2006-0121. Available at:
http://pediatrics.aappublications.org/cgi/content/full/118/5/1888

Daniels SR, Greer FR; Committee on Nutrition. Lipid screening and cardiovascular health in childhood.
Pediatrics. 2008;122:198-208. DOI: 10.1542/peds.2008-1349. Available at:
http://pediatrics.aappublications.org/cgi/content/full/122/1/198

Nathan DM, Buse JB, Davidson MB, et al; American Diabetes Association; European Association for
Study of Diabetes. Medical management of hyperglycemia in type 2 diabetes: a consensus algorithm for
the initiation and adjustment of therapy: a consensus statement of the American Diabetes Association
and the European Association for the Study of Diabetes. Diabetes Care. 2009;32:193-203. DOI:
10.2337/dc08-9025. Available at: http://care.diabetesjournals.org/content/32/1/193.long

Copyright 2012 American Academy of Pediatrics


2012 PREP SA ON CD-ROM

Question 169
An 8-year-old boy in your practice has attention-deficit/hyperactivity disorder and learning issues.
He currently is receiving specialized educational services and methylphenidate for his attention difficulties
and hyperactivity. He does well with the structure that is in place at school but has issues with compliance
at home when completing his homework. His parents seek guidance in establishing a behavioral
modification approach for him at home.
Of the following, the BEST intervention is
A. extinction
B. habit reversal
C. spanking
D. stress anxiety reduction procedures
E. token economy

Copyright 2012 American Academy of Pediatrics


2012 PREP SA ON CD-ROM

Critique 169 Preferred Response: E


The use of rewards is more effective in promoting behavioral change than is punishment.
Therefore, the boy described in the vignette should respond best to token economy, which consists of
providing rewards or privileges for the childs positive behavior and losing these for negative behaviors.
Extinction is the denial of all attention after a child engages in negative behavior, which can be an
effective approach but often causes a transient increase in negative behavior that can be discouraging for
a parent. Although spanking may immediately decrease a childs negative behavior, it also teaches a
child that hitting is acceptable and results in other discipline approaches losing their efficacy. Habit
reversal is a behaviorally based treatment for repetitive behaviors that serve no adaptive purpose. This
treatment is not indicated for this child because he is not displaying a habit disorder (eg, pulling his hair).
Stress anxiety reduction procedures are indicated when a child is having difficulty handling a stressful
situation such as test taking.

American Board of Pediatrics Content Specification(s):


Understand the utility of behavior modification approaches in the overall management of children with
learning, developmental, and behavioral problems

Suggested Reading:
Christopherson ER. Behavioral management: theory and practice. In: Parker S, Zuckerman B, Augustyn
M, eds. Developmental and Behavioral Pediatrics: A Handbook for Primary Care. 2nd ed. Philadelphia,
PA: Lippincott Williams & Wilkins, a Wolters Kluwer business; 2005:55-60

Pipan ME, Blum NJ. Basics of child behavior and primary care management of common behavioral
problems. In: Voight RG, Macias MM, Myers SM, eds. American Academy of Pediatrics Developmental
and Behavioral Pediatrics. Elk Grove Village, IL: American Academy of Pediatrics; 2011: 37-58

Wilms Floet AM, Scheiner C. Grossman L. Attention-deficit/hyperactivity disorder. Pediatr Rev.


2010;31:56-69. DOI: 10.1542/pir.31-2-56. Available at:
http://pedsinreview.aappublications.org/cgi/content/full/31/2/56

Copyright 2012 American Academy of Pediatrics


2012 PREP SA ON CD-ROM

Question 170
A 2-year-old boy who received a diagnosis of congenital hydrocephalus and underwent
ventriculoperitoneal (VP) shunt placement at 1 month of age presents with a 2-day history of decreased
activity, fever, and vomiting. Fluid obtained from his VP shunt reveals:
3
White blood cell count, 77/mm with 90% neutrophils and 10% lymphocytes
3
Red blood cell count, 45/mm
Glucose, 32 mg/dL (1.7 mmol/L)
Protein, 185 mg/dL (1.85 g/L)
Gram stain reveals gram-positive cocci in clusters. You are consulted about the selection of
empiric antibiotic therapy pending further neurosurgical evaluation.
Of the following, the MOST appropriate agent for empiric therapy is
A. cefazolin
B. ceftriaxone
C. nafcillin
D. trimethoprim-sulfamethoxazole
E. vancomycin

Copyright 2012 American Academy of Pediatrics


2012 PREP SA ON CD-ROM

Critique 170 Preferred Response: E


Coagulase-negative staphylococci are common colonizers of the human skin and are generally
nonpathogenic. However, they are an important cause of infection in the presence of foreign bodies (eg,
intravascular catheters, ventriculoperitoneal shunts, prosthetic devices) or in immunocompromised hosts.
These organisms have the ability to produce a biofilm that helps encase the bacteria on the foreign body
and acts as a barrier against antibiotic penetration to the organism.
Vancomycin is the drug of choice for coagulase-negative staphylococcal infections. Some experts
suggest adding rifampin or gentamicin for potential synergy. Even with appropriate antibiotic therapy,
however, removal of the foreign body may be necessary to clear such infections.
Cefazolin, ceftriaxone, nafcillin, and trimethoprim-sulfamethoxazole do not have reliable activity
against coagulase-negative staphylococci. In addition, first-generation cephalosporins such as cefazolin
do not penetrate the central nervous system.
Linezolid is a potential alternative agent for treating infections caused by coagulase-negative
staphylococci (and other resistant gram-positive organisms) but should be considered a secondary
selection because of its much greater expense and the more limited clinical experience with its use. Other
alternative agents that have activity against coagulase-negative staphylococci include quinupristin-
dalfopristin, daptomycin, and tigecycline, but these agents have not been approved for use in children.

American Board of Pediatrics Content Specification(s):


Understand that coagulase-negative staphylococcal infections are usually associated with
intravascular or urinary catheters, CSF shunts, and other foreign bodies

Suggested Reading:
McGirt MJ, Zaas A, Fuchs HE, George TM, Haye K, Sexton DJ. Risk factors for pediatric
ventriculoperitoneal shunt infection and predictors of infectious pathogens. Clin Infect Dis. 2003;36:858
862. DOI: 10.1086/368191. Available at: http://cid.oxfordjournals.org/content/36/7/858.long

Rogers KL, Fey PD, Rupp ME. Coagulase-negative staphylococcal infections. Infect Dis Clin North Am.
2009;23:7398. DOI: 10.1016/j.idc.2008.10.001. Abstract available at:
http://www.ncbi.nlm.nih.gov/pubmed/19135917

Copyright 2012 American Academy of Pediatrics


2012 PREP SA ON CD-ROM

Question 171
A previously healthy 7-year-old boy presents with a 2-day history of progressive right knee pain
and swelling and increasing difficulty bearing weight. He reports falling on the playground 1 week ago but
was fine after the fall. On physical examination, his temperature is 39.0C, heart rate is 120 beats/min,
and respiratory rate is 25 breaths/min. He is lying on a stretcher, obviously uncomfortable. His right knee
is swollen and warm, but there is no significant overlying erythema or effusion. Movement of the knee
joint is moderately painful, but he can tolerate the maneuver. The lateral aspect of the distal femur is
tender to palpation. His mucous membranes are moist and lungs are clear. His abdomen is benign. There
are no other findings of note on the remainder of his examination. His white blood cell count is
3 9
25.0x10 /mcL (25.0x10 /L), with 75% polymorphonuclear leukocytes, 20% lymphocytes, and 5%
monocytes. Blood culture results are pending.
Of the following, the MOST likely pathogen causing this patients illness is
A. Haemophilus influenzae type b
B. Kingella kingae
C. Salmonella non-typhi
D. Staphylococcus aureus
E. Streptococcus agalactiae

Copyright 2012 American Academy of Pediatrics


2012 PREP SA ON CD-ROM

Critique 171 Preferred Response: D


The fever, leukocytosis, and point tenderness of the femur described for the child in the vignette
are suspicious for osteomyelitis, most likely caused by Staphylococcus aureus. Osteomyelitis in children
usually is hematogenous in origin because of the rich vascular supply to their growing bones.
Approximately one third of patients who have acute hematogenous osteomyelitis (AHO) report
antecedent minor trauma to the affected extremity.
S aureus is the most common cause of AHO in children of all ages, followed by Streptococcus
pyogenes (group A Streptococcus). Kingella kingae is a less common cause of AHO and usually occurs
in children younger than 3 years of age. S pneumoniae and Haemophilus influenzae type b (Hib) are
rarely isolated in young children who have AHO because of the widespread use of pneumococcal and Hib
conjugate vaccines. Salmonella non-typhi and other gram-negative bacilli are uncommon causes of
infection that occur most often in children who have hemoglobinopathies or immunodeficiencies. In
neonates, Enterobacteriaceae are less common pathogens isolated in AHO than is S aureus. Coagulase-
negative staphylococci occasionally can cause AHO in neonates, especially in very low-birthweight
infants who have medical devices in place. S agalactiae (group B Streptococcus) AHO occurs in
neonates and usually affects the tibia or humerus.
The isolation of bacteria from bone or contiguous structures best establishes the diagnosis of
osteomyelitis. Cultures from bone, subperiosteal abscess, or adjacent structures provide a bacteriologic
diagnosis in 66% to 75% of cases; blood cultures yield a pathogen in approximately 50% of cases.
Ancillary studies that are helpful in the evaluation of a child in whom AHO is suspected include serum C-
reactive protein and erythrocyte sedimentation rate, although elevated values of these acute-phase
reactants are not specifically diagnostic of AHO. Plain radiography of the affected area usually reveals
soft-tissue edema in the area of the metaphysis at 3 days after the onset of symptoms, muscle swelling
and obliteration of the fat planes at 3 to 7 days, and evidence of bone destruction by 10 to 21 days.
Magnetic resonance imaging is the preferred imaging modality in patients being evaluated for AHO
because it is more sensitive and provides more specific anatomic information, including delineating
subperiosteal or soft-tissue abscesses that may require surgical drainage, than other modalities.
Radionucleotide imaging can be useful in multifocal disease or when the site of infection is unclear, but it
had decreased sensitivity in neonates, older infants, and children in some reports.
The emergence of community-associated methicillin-resistant S aureus has coincided with an
increasing incidence of AHO complicated by subperiosteal, intramedullary, and soft-tissue abscesses.
Therefore, prompt evaluation of the patient who has suspected AHO is necessary to determine the need
for surgical intervention. Prompt and adequate surgical debridement coupled with appropriate
antimicrobial therapy can reduce morbidity.
In all children, the initial choice of antimicrobial therapy should include intravenous agents that
have potent activity against staphylococci and group A streptococci. The use of semisynthetic penicillins
(eg, nafcillin or oxacillin) or a first-generation cephalosporin (eg, cefazolin) is appropriate in communities
where MRSA is not prevalent. In regions where MRSA has emerged as a significant pathogen, initial
therapy with vancomycin or clindamycin is recommended. Clindamycin has excellent penetration into
bone and favorable pharmacokinetics. However, some regions have reported increasing clindamycin
resistance among S aureus isolates, which limits its use for empiric therapy. Experience with some of the
newer agents (linezolid, daptomycin) is limited. Trimethoprim-sulfamethoxazole and tetracycline have
been used in some cases, but pediatric data are limited. Empiric treatment of neonatal AHO should
include intravenous agents active against Enterobacteriaceae in addition to staphylococci and
streptococci. For treating gram-negative pathogens, beta-lactam antibiotics, such as third-generation
cephalosporins or extended-spectrum penicillins, are preferred to aminoglycosides because of more
favorable pharmacokinetics for treating skeletal infections.
Once the organism has been identified in culture and susceptibilities are known, modifying
therapy to the most effective, narrow-spectrum agent is warranted. The decision about transitioning from
parenteral to oral therapy is patient-specific and should be based on: 1) clinical and laboratory evidence
of an initial therapeutic response, 2) the severity and associated complications (eg, endocarditis, septic
pulmonary emboli) of the illness, 3) the patients ability to take oral therapy, and 4) the familys ability and
willingness to administer oral therapy. In most cases, an oral antibiotic for the treatment of AHO must be
administered in higher doses than usually given for other infections to achieve effective concentrations in

Copyright 2012 American Academy of Pediatrics


2012 PREP SA ON CD-ROM

bone. Oral therapy may not be appropriate in patients who have not met the previously noted criteria for
transitioning from intravenous therapy, those who are immunocompromised, neonates, and those in
whom gastrointestinal absorption of medication is altered. The duration of treatment for AHO ranges from
3 weeks to 6 weeks and is dependent upon the clinical and microbiologic response to therapy, the bone
involved, and associated complications of the illness.

American Board of Pediatrics Content Specification(s):


Understand the management of osteomyelitis
Know the age-related microbiology of osteomyelitis

Suggested Reading:
Conrad DA. Acute hematogenous osteomyelitis. Pediatr Rev. 2010;31:464-471. DOI: 10.1542/pir.31-11-
464. Available at: http://pedsinreview.aappublications.org/cgi/content/full/31/11/464

Gutierrez KM. Osteomyelitis. In: Long SS, Pickering LK, Prober CG, eds. Principles and Practice of
Pediatric Infectious Diseases. 3rd ed. Philadelphia, PA: Churchill Livingstone Elsevier; 2008:474-481

Krogstad P. Osteomyelitis. In: Feigin RD, Cherry JD, Demmler-Harrison GJ, Kaplan SL, eds. Feigin &
Cherrys Textbook of Pediatric Infectious Diseases. 6th ed. Philadelphia, PA: Saunders Elsevier;
2009:725-741

Copyright 2012 American Academy of Pediatrics


2012 PREP SA ON CD-ROM

Question 172
An 8-year-old boy presents with cola-colored urine without blood clots. He was well until 2 days
ago, when he developed a sore throat with upper respiratory tract infection symptoms. He denies any
dysuria, frequency, urgency, flank pain, or trauma. On physical examination, his temperature is 37.8C,
heart rate is 84 beats/min, respiratory rate is 18 breaths/min, and blood pressure is 118/78 mm Hg. He
has no costovertebral tenderness, abdominal tenderness, or edema. The urinalysis reveals:
Specific gravity, 1.025
pH, 6.0
3+ blood
3+ protein
1+ leukocyte esterase
Nitrite, negative
Microscopy shows more than 100 red blood cells/high-power field (hpf) and 5 to 10 white blood
cells/hpf. Other laboratory findings include:
Blood urea nitrogen, 24 mg/dL (8.6 mmol/L)
Creatinine, 0.9 mg/dL (79.6 mcmol/L)
Complement component 3 (C3), 140 mg/dL (normal, 80 to 200 mg/dL)
Complement component 4 (C4), 30 mg/dL (normal, 16 to 40 mg/dL)
Antinuclear antibody, negative
Of the following, the MOST likely diagnosis is
A. acute pyelonephritis
B. immunoglobulin A glomerulonephritis
C. postinfectious glomerulonephritis
D. urolithiasis
E. viral cystitis

Copyright 2012 American Academy of Pediatrics


2012 PREP SA ON CD-ROM

Critique 172 Preferred Response: B


The boy described in the vignette presents with the clinical picture of acute glomerulonephritis
(cola-colored urine, hematuria/proteinuria, and mild azotemia). Lower tract bleeding, such as from viral
hemorrhagic cystitis or stones within the urinary tract, is characterized by bright red urine, often
accompanied by clots. Cola-colored urine without clots points to the upper urinary tract, typically
glomerular hematuria. The boys history reveals a close temporal relationship between the onset of
pharyngitis and cola-colored urine. This 2-day lag period between the upper respiratory symptoms and
the nephritis is sometimes described as synpharyngitic, which is characteristic of but not specific for
immunoglobulin A (IgA) glomerulonephritis. Such a temporal relationship differs from that seen in
postinfectious or poststreptococcal glomerulonephritis, in which a 7- to 21-day lag period is seen between
the onset of pharyngitis and the development of gross hematuria.
The evaluation of a child in whom acute glomerulonephritis (AGN) is suspected includes
determining the blood pressure and the renal function. If the patient is normotensive or has only mildly
elevated blood pressure, the focus turns to the serum creatinine. Mild elevation of the creatinine rules out
severe nephritis at that moment, but the creatinine measurement may need to be repeated in a day or
two to assess whether it is rising, remaining the same, or returning toward normal. Children who have
AGN and a rising serum creatinine require urgent nephrology consultation for possible renal biopsy.
Following assessment of serum creatinine, a serologic evaluation should be undertaken, including
complement factor 3 and 4 (C3 and C4), antinuclear antibody, and anti-double-stranded DNA. The boy in
the vignette has normal C3 and C4 values. The combination of a normocomplementemic GN and a
synpharyngitic pattern to the onset of glomerulonephritis makes IgA glomerulonephritis likely. IgA
glomerulonephritis can also present with asymptomatic hematuria or hematuria/proteinuria. Patients who
have gross hematuria have a better prognosis. Definitive diagnosis requires a renal biopsy, which also
allows assessment of the severity of IgA glomerulonephritis. Treatment usually consists of
immunosuppression such as with corticosteroids and angiotensin-converting enzyme inhibitors,
depending on the renal biopsy findings and the severity of azotemia and proteinuria.
Alternative diagnostic explanations for the patient in the vignette include AGN due to Alport
nephritis and membranoproliferative glomerulonephritis (hypocomplementemic in two thirds of cases at
disease onset), both of which are less common than IgA glomerulonephritis. The patient lacks any clinical
features of acute pyelonephritis. It should be noted that the mild pyuria (5 to 10 white blood cells/hpf) is
consistent with the inflammation accompanying the glomerulonephritis. Patients who have pyelonephritis
would have more pyuria, and the white blood cell count would be greater than the red blood cell count.

American Board of Pediatrics Content Specification(s):


Recognize the signs and symptoms of IgA nephropathy

Suggested Reading:
Kapur G. Immunoglobulin A nephropathy. In: Chand DH, Valentini RP, eds. Clinicians Manual of
Pediatric Nephrology. Singapore: World Scientific Publishing; 2011

Wyatt RJ, Novak J, Gaber LW, Lau KK. Immunoglobulin A nephropathy and Henoch-Schnlein purpura
nephritis. In: Kher KK, Schnaper HW, Makker SP, eds. Clinical Pediatric Nephrology. 2nd ed. London,
England: Informa Healthcare; 2007:213-221

Copyright 2012 American Academy of Pediatrics


2012 PREP SA ON CD-ROM

Question 173
A 16-year-old boy presents to your office with a history of bilateral nasal congestion. Initially he
thought he had a viral illness and bought an over-the-counter nasal decongestant. His symptoms
improved at first, but despite using the nasal spray regularly for 2 weeks, his nasal congestion has
persisted. On physical examination, the boy appears healthy but is having obvious difficulty breathing
through his nose. A nasal examination shows beefy-red turbinates, with friability of the nasal mucosa.
Of the following, the MOST likely reason for the patients symptoms is
A. allergic rhinitis
B. nasal polyposis
C. nonallergic rhinitis with eosinophilia
D. rhinitis medicamentosa
E. vasomotor rhinitis

Copyright 2012 American Academy of Pediatrics


2012 PREP SA ON CD-ROM

Critique 173 Preferred Response: D


The combination of prolonged use of an over-the-counter nasal decongestant and beefy-red
turbinates described for the boy in the vignette is classic for rhinitis medicamentosa. This condition results
from the prolonged use of topical nasal vasoconstrictive medications. Appropriate counseling includes
limiting the use of these drugs to no longer than 3 to 4 days. If rhinitis medicamentosa is suspected,
treatment consists of cessation of the topical vasoconstrictive spray and use of a nasal corticosteroid for 1
to 2 weeks. Occasionally, a short course of oral corticosteroids is required.
Allergic and nonallergic rhinitis are common in children and adolescents. Allergic rhinitis is an
immunoglobulin (Ig)E-mediated response to airborne allergens such as trees, grass, and pet dander.
Common nasal symptoms include clear rhinorrhea, sneezing, nasal congestion, nasal pruritus, and
swollen inferior turbinates. Although pale, bluish turbinates are the classic nasal examination finding
associated with allergic rhinitis, most patients have normal, erythematous, or swollen turbinates. A beefy-
red appearance is not consistent with allergic rhinitis.
Nonallergic rhinitis is more common than allergic rhinitis in children and may have similar
symptoms. Common causes of nonallergic rhinitis include viral infections, sinusitis, gustatory rhinitis,
vasomotor rhinitis (due to irritants such as cold air, strong odors), and nonallergic rhinitis with eosinophilia
(NARES). Vasomotor rhinitis may lack specific triggers identified in the history, but the prolonged use of a
nasal decongestant is more suggestive of rhinitis medicamentosa. NARES is a unique condition in which
patients have symptoms similar to allergic rhinitis, but allergy testing results are negative. As part of the
criteria for NARES, a nasal smear demonstrates abundant eosinophils.
Allergic rhinitis and nonallergic rhinitis are often indistinguishable on physical examination,
although common signs seen in allergic rhinitis include infraorbital swelling (allergic shiners); a
transverse nasal crease (due to chronic rubbing of the nose); and pale, bluish turbinates. Nonallergic
rhinitis may have similar symptoms but frequently stems from exposure to irritants such as smoke, odors,
or dust.
Nasal polyps can occur with chronic sinusitis, allergic rhinitis, Samter triad (asthma, aspirin
sensitivity, nasal polyposis), allergic fungal sinusitis, and cystic fibrosis. They typically present with
anosmia and chronic nasal congestion. Polyps usually originate from the ethmoid sinus and present as
pale grapelike structures that do not change in appearance after use of a topical vasoconstrictive spray.

American Board of Pediatrics Content Specification(s):


Distinguish between allergic rhinitis and nonallergic rhinitis by history and physical examination

Suggested Reading:
Gargiulo KA, Spector ND. Stuffy Nose. Pediatr Rev. 2010;31:320-325. DOI: 10.1542/pir.31-8-320.
Available at: http://pedsinreview.aappublications.org/cgi/content/full/31/8/320

Quillen D, Feller D. Diagnosing rhinitis: allergic vs. nonallergic. Am Fam Physician. 2006;73:15831590.
Available at: http://www.aafp.org/afp/2006/0501/p1583.html

Copyright 2012 American Academy of Pediatrics


2012 PREP SA ON CD-ROM

Question 174
A 15-year-old baseball player presents to the office after being struck in the face by a pitch. He
reports that he had a profuse nosebleed immediately after the event, and now his nose is swollen and it is
hard to breathe through his left nostril. On physical examination, the bridge of his nose is markedly
swollen and there are ecchymoses developing under both eyes. A clot is visible in the right nostril and a
purple swelling is visible on the medial aspect of his left nostril.
Of the following, the MOST appropriate next step in the treatment of this patient is to
A. apply a warm pack to his nose
B. pack both nostrils to prevent further epistaxis
C. prescribe phenylephrine nose drops
D. prescribe prophylactic antibiotics
E. refer him immediately to an otolaryngologist

Copyright 2012 American Academy of Pediatrics


2012 PREP SA ON CD-ROM

Critique 174 Preferred Response: E


The presence of a bluish swelling on the nasal septum following facial trauma, as described for
the boy in the vignette, suggests the presence of a nasal septal hematoma. Other suggestive findings
include pain in the tip of the nose and septal swelling unresponsive to topical vasoconstrictor agents (eg,
phenylephrine). This condition, which may be present acutely or develop within 72 hours of nasal trauma,
requires prompt intervention to prevent permanent facial deformity. The boy should be referred
immediately to an otolaryngologist for evaluation and treatment.
Initial physical examination after nasal trauma should assess for the presence of a nasal septal
hematoma by visual inspection, presence of cerebrospinal fluid leak by inspection and fluid analysis for
beta-2-transferrin, and the presence of nasal bone or other facial fractures by palpation. If facial bone
fractures are suspected, computed tomography scan is indicated for further evaluation. Radiographs are
not indicated acutely for isolated nasal bone fractures, but patients should be referred to a specialist for
re-evaluation and intervention if deformity is evident after the swelling subsides in 2 to 3 days. Initial
interventions following uncomplicated nasal trauma include application of ice to decrease swelling,
acetaminophen analgesia, and a short course of phenylephrine to improve nasal obstruction, but
prophylactic packing or antibiotics are not indicated.
Nasal septal hematomas occur after trauma when the nasal mucoperichondrium is sheared from
the nasal cartilage, with resultant disruption of perichondrial blood vessels. Evacuation of the hematoma
is indicated to prevent pressure-induced avascular necrosis and infection. Typically performed by an
otolaryngologist, the procedure is accomplished as follows: the hematoma is incised, the clot and debris
are suctioned, and the mucoperichondrium is sutured to the cartilage to prevent reaccumulation.
Prophylactic antibiotics are indicated after drainage to prevent abscess formation with nasopharyngeal
organisms.

American Board of Pediatrics Content Specification(s):


Recognize the significance of a hematoma of the nasal septum
Recognize the physical findings and know the management of a hematoma of the nasal septum

Suggested Reading:
Mendez DR, Lapointe A. Nasal trauma and fractures in children. UpToDate Online 18.3. 2010. Available
for subscription at: http://www.uptodate.com/online/content/topic.do?topicKey=ped_trau/4407

Menger DJ, Tabink I, Nolst Trenit GJ. Treatment of septal hematomas and abscesses in children. Facial
Plast Surg. 2007;23:239-243. DOI: 10.1055/s-2007-995816. Abstract available at:
http://www.ncbi.nlm.nih.gov/pubmed/18085498

Ngo J, Schraga ED. Drainage, nasal septal hematoma. eMedicine Specialties, Clinical Procedures. 2009.
Available at: http://emedicine.medscape.com/article/149280-overview

Savage RR, Valvich C, Serwint JR. In brief: hematoma of the nasal septum. Pediatr Rev. 2006;27:478-
479. DOI: 10.1542/pir.27-12-478. Available at:
http://pedsinreview.aappublications.org/cgi/content/full/27/12/478

Copyright 2012 American Academy of Pediatrics


2012 PREP SA ON CD-ROM

Question 175
The mother of an 8-year-old boy calls you to seek advice after her son told her that he had been
videotaped yesterday by an older boy while changing out of his swimming suit in the locker room at the
local pool. Two other young boys were taped at the same time by the video on the older boys cell phone.
Immediately after the child told his mother, she called 911 and police were dispatched to the scene. The
police confiscated the cell phone and verified that the young boys had been videotaped. After
interviewing all parties involved, a teenage boy who did the taping was arrested and placed into juvenile
custody. The mother reports that her son had difficulty falling asleep last night, but he is acting normally
today.
Of the following, the BEST next step to take is to
A. advise the mother to maintain normal routines
B. arrange for a videotaped interview of the boy
C. prescribe clonidine to aid in sleeping
D. prescribe fluoxetine to help prevent posttraumatic stress disorder
E. refer the family to a mental health specialist for evaluation

Copyright 2012 American Academy of Pediatrics


2012 PREP SA ON CD-ROM

Critique 175 Preferred Response: A


Unfortunately, most children are exposed to some type of trauma during their childhood. The
parents and child described in the vignette should be reassured that most children experience no long-
term adverse effects from such experiences. In addition, parents should be advised to maintain normal
routines and employ natural supports (eg, spend time with friends and relatives, go to school, engage in
usual activities). An expectation that the boy should soon be able to continue with life as usual
communicates that he has not been necessarily damaged by the incident and offers the opportunity to do
something other than dwelling on the incident. The child should be specifically reassured that he has
done nothing wrong. Any questions the boy has should be answered, but parents should be discouraged
from repeatedly forcing the child to discuss the event, which could be more traumatizing. If the child has
persistent behavioral changes (eg, sleep disturbances, unusual fears, or avoidance of usual activities), he
should be seen for further evaluation.
Interviewing the boy to determine the specific details of the event is not indicated or appropriate
because it could interfere with the forensic investigation. An interviewer trained in forensic evaluations
might be called upon separately by law enforcement to conduct an unbiased videotaped interview that is
admissible in court. Referring the family to a mental health specialist is not needed because the child is
not currently symptomatic, and a referral at this time will unnecesssarily suggest to the family that the
child is likely to sustain serious mental health problems from the incident. Prescribing medications such
as fluoxetine or clonidine immediately after a traumatic event has not been shown to prevent either an
acute stress reaction or posttraumatic stress disorder.

AAP Mental Health Competency:


Identify an appropriate psychological management approach for a recently traumatized child

Suggested Readings:
Cohen JA, Mannarino AP, Iyengar S. Community treatment of posttraumatic stress disorder for children
exposed to intimate partner violence: a randomized controlled trial. Arch Pediatr Adolesc Med.
2011;165:16-21. Abstract available at: http://www.ncbi.nlm.nih.gov/pubmed/21199975

Layne CM. Developing interventions for trauma-exposed children: a comment on progress to date, and 3
recommendations for further advancing the field. Arch Pediatr Adolesc Med. 2011;165:89-90

Silverman WK, Hinshaw SP. The second special issue on evidence-based treatments for children and
adolescents: a 10-year update. J Clin Child Adolesc Psychol. 2008;37:1-7

Silverman WK, Ortiz CD, Viswesvaran C, et al. Evidence-based psychosocial treatments for children and
adolescents exposed to traumatic events. J Clin Child Adolesc Psychol. 2008;37:156-183. Abstract
available at: http://www.ncbi.nlm.nih.gov/pubmed/18444057

Copyright 2012 American Academy of Pediatrics


2012 PREP SA ON CD-ROM

Question 176
A mother brings in her 5-week-old infant girl because of feeding difficulties. The baby weighed
3,300 g when born at term, and she has breastfed exclusively. Approximately 2 weeks ago, the parents
noted that the baby became increasingly irritable, particularly during feedings, and she began spitting-up
4 to 6 times per day. Physical examination demonstrates a well-developed, alert but irritable infant whose
weight is 3.85 kg, heart rate is 180 beats/min, and respiratory rate is 70 breaths/min. Lung sounds are
clear. On physical examination, you note a hyperdynamic precordium and a grade 2/6 holosystolic
cardiac murmur. Chest auscultation yields normal results. You palpate a firm liver edge 5.0 cm below the
right costal margin. The spleen is not palpable. You also note a 2x2-cm hemangioma on the abdominal
wall. Results of laboratory tests include:
Hemoglobin, 9.8 g/dL (9.8 g/L)
3 9
White blood cell count, 4.8x10 /mcL (4.8x10 /L)
3 9
Platelet count, 80x10 /mcL (80x10 /L)
Peripheral blood smear, Burr cells and schistocytes noted
Electrolytes, normal
Bilirubin, 1.6 mg/dL (27.4 mcmol/L)
Chest radiography demonstrates mild cardiomegaly.
Of the following, the study that is MOST likely to demonstrate the cause of this infants symptoms
is
A. abdominal ultrasonography
B. acid alpha-glucosidase assay
C. bone marrow aspiration
D. Coombs test
E. echocardiography

Copyright 2012 American Academy of Pediatrics


2012 PREP SA ON CD-ROM

Critique 176 Preferred Response: A


The infant described in the vignette presents with anemia, thrombocytopenia, and clinical signs
suggesting high-output cardiac dysfunction. In addition to tachycardia and tachypnea, the most significant
physical finding is hepatomegaly, with a liver edge palpated 5 cm below the right costal margin. The
differential diagnosis of liver enlargement during infancy can include inflammatory, infiltrative, obstructive,
storage, and vascular disorders (Item C176). This infants clinical findings suggest an intrahepatic
hemangioma that necessitates abdominal ultrasonography.
The additional hemangioma-related complications (high-output cardiac failure, thrombocytopenia,
and hemolysis) exhibited by the infant point toward the development of the Kasabach-Merritt
phenomenon, a rare consequence of large vascular lesions that is characterized by hemolytic anemia,
thrombocytopenia, and coagulopathy.
Hemangiomas are the most common soft-tissue tumors of infancy, with an occurrence rate of 5%
to 10% and a female-to-male predominance of 3:1. Most hemangiomas are cutaneous lesions, and
greater than 55% may be identified at birth. They typically undergo a rapid early proliferative phase,
reaching their maximum size by 6 to 8 months, followed by a gradual spontaneous involution.
Although rare, compared with cutaneous lesions, hemangiomas of the liver are the most common
hepatic vascular lesions found in newborns that often are multiple and involve both lobes. They are
frequently, but not always, associated with cutaneous hemangiomas. Indeed, the newborn who has
multiple cutaneous lesions should undergo screening for visceral hemangiomatosis. Similar to cutaneous
lesions, hepatic hemangiomas grow rapidly during early infancy. However, they are associated with
significantly higher morbidity and mortality rates than cutaneous hemangiomas and frequently present
clinically as the consequence of secondary anemia and high-output congestive heart failure.
Ultrasonography with evaluation of blood flow by Doppler is a cost-effective, noninvasive
technique that demonstrates the high-flow pattern characteristic of the hemangioma, helping to
differentiate this lesion from solid tumors and other vascular or lymphatic abnormalities. Acid alpha-
glucosidase assay, bone marrow aspiration, Coombs test, and echocardiography may be used in the
evaluation of hepatomegaly, but none can target the likely cause of this infants illness. The acid alpha-
glucosidase assay is the diagnostic study of choice in the evaluation of Gaucher disease, a lipid storage
disorder characterized not only by hepatomegaly but also by massive splenomegaly. Bone marrow
aspiration is routinely performed to evaluate suspected blood cell dyscrasias, other nonhematopoietic
malignancies, and storage diseases but would not be helpful in this infant in whom intravascular
hemolysis (schistocytes on smear) is a possibility. In neonatal hemolytic disorders, the peripheral blood
smear Coombs test is used to evaluate blood group incompatibilities. Hemolysis, extramedullary
hematopoiesis, splenomegaly, and hyperbilirubinemia are common findings; thrombocytopenia is
unusual. Finally, echocardiography may demonstrate a high-output state in this infant, but hemolysis and
thrombocytopenia would be unusual associated findings in congenital cardiac disease.
Liver size is assessed best by physical examination using a digital percussion technique. Imaging
studies are extremely useful for identifying structures of the hepatobiliary tract and intrahepatic lesions,
but these techniques should not be employed routinely to determine liver size. In newborns, the mean
liver span, assessed by percussion in the right midclavicular line, ranges from 4.5 to 5 cm. During normal
development, liver size increases linearly with both body weight and height, and by 12 years of age, the
mean normal span is 7 to 8 cm in boys and 6 to 7 cm in girls. Because the precise position of the liver in
the right upper abdominal quadrant may vary from individual to individual and may be affected by an
associated clinical condition (eg, low-lying liver in the presence of pulmonary hyperinflation), a liver edge
that is palpated below the right costal margin does not necessarily indicate enlargement. When the liver
edge is palpable, a distance below the right costal margin of more than 3.5 cm during the neonatal period
and greater than 2 cm in older infants and children is considered abnormal, irrespective of associated
signs and symptoms.
Depending on patient age, the observation of hepatomegaly, in conjunction with a careful history
and the presence of other physical findings, should determine the course of diagnostic evaluation.
Although hepatomegaly may represent a transient observation accompanying a systemic viral illness,
persistent liver enlargement warrants further evaluation. When the liver edge is firm (as in the vignette), a
storage or infiltrative disorder must be considered.

Copyright 2012 American Academy of Pediatrics


2012 PREP SA ON CD-ROM

American Board of Pediatrics Content Specification(s):


Know the significance of hepatomegaly in a 1-month-old infant
Recognize age-related changes during physical examination of the liver

Suggested Reading:
DAgata ID, Balistreri W. Evaluation of liver disease in the pediatric patient. Pediatr Rev. 1999;20:376-
389. DOI: 10.1542/pir.20-11-376. Available at:
http://pedsinreview.aappublications.org/cgi/content/full/20/11/376

Drolet BA, Esterly NB, Frieden IJ. Hemangiomas in children. N Engl J Med. 1999;341:173-181. Extract
available at http://www.nejm.org/doi/full/10.1056/NEJM199907153410307

Lawson EE, Grand RJ, Neff RK, Cohen LF. Clinical estimation of liver span in infants and children. Am J
Dis Child. 1978;132:474-476. Abstract available at: http://www.ncbi.nlm.nih.gov/pubmed/645673

Naveh Y, Berant MJ. Assessment of liver size in normal infants and children. J Pediatr Gastroenterol
Nutr. 1984;3:346-348

Reiff MI, Osborn LM. Clinical estimation of liver size in newborn infants. Pediatrics. 1983;71:46-48.
Abstract available at: http://pediatrics.aappublications.org/cgi/content/abstract/71/1/46

Schiavetti A, De Pasquale MD, Di Salvo S, Ventriglia F, Clerico A. Recombinant interferon alfa 2a in


hepatic hemangiomatosis with congestive heart failure: a case report. Pediatr Hematol Oncol.
2003;20:161-165. Abstract available at: http://www.ncbi.nlm.nih.gov/pubmed/12554528

Wolf AD, Lavine JE. Hepatomegaly in neonates and children. Pediatr Rev. 2000;21:303-310. DOI:
10.1542/pir.21-9-303. Available at: http://pedsinreview.aappublications.org/cgi/content/full/21/9/303

Copyright 2012 American Academy of Pediatrics


2012 PREP SA ON CD-ROM

Critique 176

Item C176. Differential Diagnosis of Hepatomegaly in Infants and Children


Inflammatory Autoimmune hepatitis
Bile acid synthetic and transport defects
Idiopathic neonatal hepatitis
Infection (bacterial, parasitic, viral)
Medications
Toxins
Infiltrative Extramedullary hematopoiesis
Hemophagocytic syndromes
Leukemia, lymphoma
Metastatic neoplasms
Primary liver neoplasms (hepatoblastoma, hemangioma)
Obstructive Choledochal cyst
Cholelithiasis
Extrahepatic biliary atresia
Malignancy (primary and metastatic)
Storage Fat
! Malnutrition (Kwashiorkor)
! Obesity (nonalcoholic fatty liver disease)
! Parenteral nutrition
Glycogen
! Diabetes
! Glycogen storage diseases
! Parenteral nutrition
Lipid (metabolic disorders)
! Diabetes
! Fatty acid oxidation defects
! Gaucher disease
! Niemann-Pick disease
! Wolman disease
Other
! Hemochromatosis
! Neonatal iron storage disease
! Wilson disease
Vascular Budd-Chiari syndrome (hepatic vein thrombosis)
Congestive heart failure
Peliosis hepatis (multiple causes)
Pericardial disease (restrictive)
Suprahepatic web of the inferior vena cava
Veno-occlusive disease

Copyright 2011 American Academy of Pediatrics


2012 PREP SA ON CD-ROM

Question 177
During your evaluation at 48 hours after birth, you note that an infant consistently holds her head
tilted to the left. She was delivered at term by cesarean section due to breech presentation. No difficulties
were described with the extraction at delivery. On physical examination, the infant holds her head tilted to
the left with her chin deviated to the right (Item Q177). She has normal tone, strength, reflexes, and
movement in all four extremities. You palpate a mass within the left sternocleidomastoid muscle.
Of the following, the MOST appropriate initial management strategy is to
A. consult orthopedic surgery for a sternocleidomastoid muscle release
B. initiate therapy with onabotulinumtoxinA
C. obtain clavicular radiography
D. obtain magnetic resonance imaging of the head and neck
E. order physical therapy for stretching of the neck muscles

Copyright 2012 American Academy of Pediatrics


2012 PREP SA ON CD-ROM

Item 177

(Courtesy of S Izatt)
Head tilt, as described for the infant in the vignette.

Copyright 2012 American Academy of Pediatrics


2012 PREP SA ON CD-ROM

Critique 177 Preferred Response: E


The infant described in the vignette has the typical findings of congenital muscular torticollis
(CMT), which responds to a stretching program in 90% of cases if started within 3 months of birth. CMT is
generally discovered at or shortly after birth, with unilateral contracture of the sternocleidomastoid muscle
causing the head to tilt to the affected side and the chin to rotate to the unaffected side. Upon clinical
examination, a palpable mass is found within the affected sternocleidomastoid muscle. This mass
resolves over time, developing into fibrous tissue.
The cause of CMT is unclear, although genetic conditions, fetal positioning abnormalities, and
birth trauma have all been proposed. Compared with the general population, there is a higher incidence
of CMT in breech infants. It is important to note that up to 8% of infants who have CMT also have hip
dysplasia.
In cases of torticollis where a mass or contracture cannot be found in the sternocleidomastoid
muscle, nonmuscular causes must be considered, including congenital vertebral anomalies, atlantoaxial
rotatory displacement, neurogenic torticollis, and paroxysmal torticollis of infancy. Consultation with a
pediatric ophthalmologist or pediatric neurologist may assist in the evaluation. In contrast to CMT, which
is a benign condition, nonmuscular torticollis may be life-threatening.
The most common imaging test to support the diagnosis of CMT is ultrasonography, which
demonstrates the fibrous tissue within the sternocleidomastoid muscle. Anteroposterior and lateral
radiographs of the cervical spine may be indicated when the diagnosis of nonmuscular torticollis is being
considered to evaluate for congenital vertebral anomalies. However, clavicular radiographs do not
contribute to the diagnosis of CMT. Magnetic resonance imaging of the brain and neck is not considered
cost-effective or necessary in CMT, but it may be needed in the evaluation of nonmuscular cases of
torticollis.
As noted previously, most affected infants respond to stretching of the neck muscles when
initiated within 3 months from birth. However, the efficacy of physical therapy is limited if it is delayed
beyond the first postnatal year. OnabotulinumtoxinA may be considered for resistant cases of CMT.
Injection into the sternocleidomastoid muscle has resulted in only partial improvement in range of motion
in some patients. Consultation with orthopedic surgery for sternocleidomastoid release is reserved for
infants who have not improved after 12 months of physical therapy or who develop facial asymmetry or
plagiocephaly.

American Board of Pediatrics Content Specification(s):


Understand the etiology of congenital torticollis
Understand that physical therapy (stretching) of the neck by a pediatric physical therapist and/or a
parent may be successful treatment for torticollis

Suggested Reading:
Do TT. Congenital muscular torticollis: current concepts and review of treatment. Curr Opin Pediatr.
2006;18:26-29. DOI: 10.1097/01.mop.0000192520.48411.fa. Abstract available at:
http://www.ncbi.nlm.nih.gov/pubmed/16470158

Robin NH. Congenital muscular torticollis. Pediatr Rev. 1996; 17:374-375. Available at:
http://pedsinreview.aappublications.org/content/17/10/374.full.pdf+html

Spiegel DA, Dormans JP. The neck: torticollis. In: Kliegman RM, Stanton BF, St. Geme JW III, Schor NF,
and Behrman RE, eds. Nelson Textbook of Pediatrics. 19th ed. Philadelphia, PA: Saunders Elsevier;
2011:2377-2379

Tatli B, Aydinli N, Caliskan M, Ozmen M, Bilir F, Acar G. Congenital muscular torticollis: evaluation and
classification. Pediatr Neurology. 2006;34:41-44. DOI; 10.1016/j.pediatrneurol.2005.06.010. Abstract
available at: http://www.ncbi.nlm.nih.gov/pubmed/16376277

Copyright 2012 American Academy of Pediatrics


2012 PREP SA ON CD-ROM

Question 178
A 17-year-old boy comes to your office for an employment physical. On physical examination,
you note thoracic kyphosis. He has been healthy and has no history of trauma or surgery. He cannot
correct the kyphosis voluntarily by standing straight. There are no other findings of note. Radiographs of
his thoracic spine show wedging of three consecutive anterior vertebral bodies at the apex of the curve.
Of the following, the MOST important indication for surgical treatment of his kyphosis is
A. absence of the deformity at birth
B. history of well-controlled mild persistent asthma
C. intermittent back pain not limiting activity
D. sixty-five degrees of kyphosis using the Cobb technique
E. unacceptable aesthetic appearance

Copyright 2012 American Academy of Pediatrics


2012 PREP SA ON CD-ROM

Critique 178 Preferred Response: E


Thoracic kyphosis of 20 to 50 degrees Cobb angle is considered normal; curves greater than that
are termed hyperkyphosis. Flexible or postural kyphosis is generally asymptomatic and can be corrected
voluntarily. No vertebral abnormalities are seen on radiographs, and a supine lateral hyperextension view
shows complete correction of the curve. By contrast, structural kyphosis persists despite positioning, as
described for the boy in the vignette, and is divided into developmental/congenital, idiopathic, and
acquired forms. Congenital kyphosis results from anomalies of the vertebral bodies; developmental
causes include such entities as Marfan syndrome, achondroplasia, and mucopolysaccharidoses; and
acquired kyphosis can result from trauma, infection, neoplasia, radiotherapy, or neuromuscular disease.
Scheuermann disease, the most common type of idiopathic structural kyphosis, is characterized
by the presence of more than 5 degrees of anterior wedging involving vertebral bodies at the apex of the
deformity. The classic definition requires involvement of three or more consecutive vertebral bodies, but
some authors accept two or even one abnormal vertebral body as sufficient to make the diagnosis. The
cause is unknown but may have a genetic component. It becomes most prominent between the ages of
12 and 15 years and affects boys more commonly than girls. Some studies find that affected teens are of
greater stature and weight compared with healthy controls, raising questions as to whether increased size
causes increased mechanical forces or whether both size and curvature are related to hormonal
influences.
Treatment for kyphosis is individualized, based on the degree of deformity, number of years of
growth remaining, and severity of symptoms. For many cases, no treatment is warranted, but serial
casting and/or bracing may be considered if the curve exceeds 60 degrees and there is at least 1 year of
growth left. Curves of 70 to 90 degrees are generally considered cosmetically unacceptable and surgical
treatment is usually recommended.
Back pain is frequent at the apex of the curve but is rarely severe, often intermittent, and rarely
limiting of activity. In adulthood, affected patients have back pain with the same frequency as the general
population, although the intensity of pain may be greater. They typically have the same school
performance and employability as the general population. Unless severe, back pain does not indicate a
need for treatment beyond exercises to increase flexibility and strength; when pain is severe, spinal
fusion may be considered.
Pulmonary function is usually preserved until the curve reaches 100 degrees, at which time spinal
fusion is used to prevent further progression. Kyphosis sufficient to cause pulmonary impairment is
associated with a restrictive pattern on pulmonary function tests. Mild asthma would not be a
consequence of kyphosis and would not be an indication for surgery.
Congenital failure of anterior vertebral formation usually presents soon after birth, progresses
rapidly, and can lead to neurologic symptoms from spinal cord compression. Affected patients nearly
always need early surgical intervention due to their worse prognosis, but they do not represent the
majority of patients who have kyphosis.

American Board of Pediatrics Content Specification(s):


Know the presenting symptoms of kyphosis
Know the treatment objectives for kyphosis: pain relief, prevention of neurologic deficit, achievement
of acceptable appearance, good follow-up evaluation

Suggested Reading:
Bernstein RM, Cozen H. Evaluation of back pain in children and adolescents. Am Fam Physician.
2007;76:1669-1676. Available at: http://www.aafp.org/afp/2007/1201/p1669.html

Fotiadis E, Kenanidis E, Samloadas E, Chritodoulou A, Akritopoulos P, Arkitopoulou K. Scheuermanns


disease: focus on weight and height role. Eur Spine J. 2008;17:673-678. DOI: 10.1007/s00586-008-0641-
x. Available at: http://www.ncbi.nlm.nih.gov/pmc/articles/PMC2367416/?tool=pubmed

Spiegel DA, Dormans JP. Bone and joint disorders: the spine. In: Kliegman RM, Stanton BF, St. Geme
JW III, Schor NF, and Behrman RE, eds. Nelson Textbook of Pediatrics. 19th ed. Philadelphia, PA:
Saunders Elsevier; 2011:2365-2377

Copyright 2012 American Academy of Pediatrics


2012 PREP SA ON CD-ROM

Question 179
A couple who is new to the community comes to you for a prenatal visit. They live in a home with
a private well and have questions about the safety of providing well water to their newborn.
Of the following, you are MOST likely to advise them to use
A. boiled tap water
B. bottled distilled water
C. bottled drinking water
D. filtered tap water
E. tap water

Copyright 2012 American Academy of Pediatrics


2012 PREP SA ON CD-ROM

Critique 179 Preferred Response: A


In the United States, municipal water supplies, community wells, and private wells equipped with
modern purification systems are generally believed to safeguard the water supply for drinking. Up to one
sixth of the homes in the United States have water supplied by private wells, as do some child care
centers and schools in suburban and rural areas. The Environmental Protection Agency regulates water
safety in municipal water supplies and community wells but not in private wells, which may lead to some
risks. Risks for children from drinking contaminated water are greater than for adults, in many cases due
to the dose response and dose per body weight of both microorganisms and chemical contaminants.
The American Academy of Pediatrics has published a policy statement that may assist
practitioners in providing advice to families about proper construction, maintenance, and surveillance of
wells to provide a safe drinking water supply, including the recommended periodicity and costs of routine
testing of well water. Annual testing for fecal coliforms is recommended; for most chemical contaminants
such as hydrocarbons, testing should be performed every 3 to 5 years. More frequent testing may be
indicated if there are outbreaks of intestinal disease in the community.
For the family described in the vignette, the source of their water is important to discuss. Because
the familys water supply is from their own private well, they need to evaluate the water supply to
determine if it is safe. Meanwhile, they should use boiled water. Use of bottled water, including distilled, is
expensive and may not contain fluoride. In addition, bottled water may contain chemical contaminants
from the plastic that could confer long-term health risks for the family.
Although infections in children caused by drinking water that is contaminated by fecal coliforms or
parasites (especially Cryptosporidium) may be identified by gastrointestinal symptoms such as diarrhea
and vomiting, chronic ingestion of chemical contaminants may result in silent intoxication or more vague
symptom complexes. There is evidence that industrial hydrocarbons such as trichloroethylenes, used as
degreasers and aerosolized solvents, can seep into groundwater, reservoirs, and the water supply. These
agents have been linked to both low birthweight and birth defects.
Filtered tap water (via faucet or pitcher/carafe-mounted filters) may reduce the presence of lead,
Cryptosporidium, and Giardia lamblia, but they are meant for water that is already processed through a
municipal water supply and are not intended to detoxify heavily contaminated water.
Families who have private wells should be advised to engage the services of a professional well
monitoring technician available through local water, health, or agricultural departments or private well
installation and service firms. Those who choose to self-monitor and maintain wells may find resources
for acceptable levels of fecal coliform contaminants and instructions for decontamination (using
household bleach) from local health departments.
Infant formula should not be prepared with well water containing high concentrations of nitrates
(>10 mg/dL), and parents should be made aware of other potential ground water contaminants that may
have long-term effects on health for both children and adults, including arsenic, methyl tertiary butyl ester,
and perchlorates.

American Board of Pediatrics Content Specification(s):


Know the contaminants potentially found in drinking water (eg, E. coli, Cryptosporidium,
trichlorethylene, perchlorethylene)

Suggested Reading:
American Academy of Pediatrics. Cryptosporidiosis. In: Pickering LK, Baker CJ, Kimberlin DW, Long SS,
eds. Red Book: 2009 Report of the Committee on Infectious Diseases. 28th ed. Elk Grove Village, IL:
American Academy of Pediatrics; 2009:272-273. Available at:
http://aapredbook.aappublications.org/cgi/content/full/2009/1/3.32

Rogan WJ, Brady MT, the Committee on Environmental Health, and the Committee on Infectious
Diseases. Drinking water from private wells and risks to children. Pediatrics. 2009;123: e1123-e1137.
DOI: 10.1542/peds.2009-0752. Available at:
http://pediatrics.aappublications.org/cgi/content/full/123/6/e1123

Copyright 2012 American Academy of Pediatrics


2012 PREP SA ON CD-ROM

United States Environmental Protection Agency. Drinking Water Contaminants: National Primary Drinking
Water Regulations. 2010. Available at: http://water.epa.gov/drink/contaminants/index.cfm

Watson RE, Jacobson CF, Williams AL, Howard WB, DeSesso JM. Trichloroethylene-contaminated
drinking water and congenital heart defects: a critical analysis of the literature. Reprod Toxicol.
2006;21:117-147. DOI: 10.1016/j.reprotox.2005.07.013. Abstract available at:
http://www.ncbi.nlm.nih.gov/pubmed/16181768

Copyright 2012 American Academy of Pediatrics


2012 PREP SA ON CD-ROM

Question 180
An overweight 14-year-old girl presents to your office for the first time for a health supervision
visit. She has a strong family history of early myocardial infarctions and hypercholesterolemia. Her father
has a total cholesterol of 290 mg/dL (7.5 mmol/L), and her paternal grandfather (who died of a heart
attack at 54 years of age) had a total cholesterol of 325 mg/dL (8.4 mmol/L). You measure the girls total
serum cholesterol, which reveals a value of 220 mg/dL (5.7 mmol/L).
Of the following, the MOST appropriate step in the initial management of this childs condition is
A. assessment for evidence of diabetes mellitus
B. assessment of a fasting lipid profile
C. initiation of an exercise program and low-fat diet
D. initiation of lipid-lowering pharmacologic therapy
E. repeat measurement of total serum cholesterol in 6 months

Copyright 2012 American Academy of Pediatrics


2012 PREP SA ON CD-ROM

Critique 180 Preferred Response: B


Childhood hyperlipidemia has taken on new urgency because of the current epidemic of
childhood obesity and the subsequent increasing risk of type 2 diabetes mellitus, hypertension, and
cardiovascular disease in adolescents and adults. Screening children and adolescents via fasting lipid
profiles remains a targeted approach. Overweight children, such as the girl described in the vignette,
belong to a special risk category of individuals who need cholesterol screening regardless of family
history or other risk factors. Accordingly, this girl should undergo a fasting lipid profile assessment.
Secondarily, she and her family should be counseled about prevention of cardiovascular disease by
following the Dietary Guidelines for Americans and increasing physical activity.
Prevalence studies have shown that the concentration of serum lipids and lipoproteins increases
during early childhood and reaches values similar to those seen in young adults by approximately 2 years
of age. This knowledge is important when making recommendations for screening because readings
obtained before 2 years of age may not reflect values in subsequent years of childhood or adult values.
Population-based studies have shown that for ages 4 to 19 years, the mean total cholesterol
concentration is 165 mg/dL (4.3 mmol/L). Age-specific values for mean total cholesterol concentration
actually peak at 171 mg/dL (4.4 mmol/L) at 9 to 11 years of age. They subsequently decrease during
pubertal development and increase thereafter. This has important implications for the timing of cholesterol
screening and the cut-points used because lipid concentrations are age- and maturation-dependent (Item
C180A).
A targeted approach to cholesterol screening for children from the National Cholesterol Education
Program (NCEP) of the National Heart, Lung, and Blood Institute was published in 1992 and
subsequently adopted by the American Academy of Pediatrics. This approach recommends screening
children who have a family history of premature cardiovascular disease or high blood concentrations of
cholesterol. In addition, screening is recommended for pediatric patients for whom family history is not
known or those who have other risk factors for cardiovascular disease such as obesity, hypertension, and
diabetes mellitus. Studies have shown that although screening is useful for identifying children who have
elevated cholesterol concentrations, 30% to 60% of such children and adolescents are missed by the
targeted strategy.
In summary, this new population approach to pediatric hyperlipidemia includes the following:
A healthful diet for all children older than 2 years that includes low-fat dairy products. For children
between 12 months and 2 years of age for whom obesity is a concern or who have a family history of
obesity, dyslipidemia, or cardiovascular disease, the use of reduced-fat milk is appropriate.
For pediatric patients who are overweight or obese and have high triglyceride or low high-density
lipoprotein cholesterol concentrations, weight management is the primary treatment, which includes
improvement of diet with nutritional counseling and increased physical activity to produce improved
energy balance.
Pharmacologic intervention should be considered for patients 10 years and older who meet the
criteria in (Item C180B).
Although assessment for diabetes mellitus, initiation of dietary and exercise recommendations,
and possibly starting lipid-lowering agents are appropriate for the girl in the vignette, the first step in
determination of risk is to obtain a fasting lipid profile. This serves as the basis for the remainder of the
diagnostic evaluation and therapeutic recommendations. Repeat measurement of total serum cholesterol
should not be delayed for 6 months.

American Board of Pediatrics Content Specification(s):


Plan the initial management of a child with a positive family history of hyperlipidemia

Suggested Reading:
American Academy of Pediatrics. National Cholesterol Education Program: report of the expert panel on
blood cholesterol levels in children and adolescents. Pediatrics. 1992;89:525 584. Abstract available at:
http://pediatrics.aappublications.org/cgi/content/abstract/89/3/525

Daniels SR. Screening for familial hypercholesterolemia: what is the most effective strategy? Nat Clin
Pract Cardiovasc Med. 2008;5:130-131. DOI: 10.1038/ncpcardio1084

Copyright 2012 American Academy of Pediatrics


2012 PREP SA ON CD-ROM

Daniels SR, Greer FR; Committee on Nutrition. Lipid screening and cardiovascular health in childhood.
Pediatrics. 2008;122:198-208. DOI: 10.1542/peds.2008-1349. Available at:
http://pediatrics.aappublications.org/cgi/content/full/122/1/198

Copyright 2012 American Academy of Pediatrics


2012 PREP SA ON CD-ROM

Item 180

Item C180A. Cut-points for Total Cholesterol and Low-density Lipoprotein (LDL) Cholesterol
Concentrations in Children and Adolescents

Total Cholesterol, mg/dL


Category Percentile LDL, mg/dL (mmol/L)
(mmol/L)
Acceptable <75th <170 (4.40) <110 (2.85)
th th
Borderline 75 to 95 170 to 199 (4.40 to 5.15) 110 to 129 (2.85 to 3.34)
th
Elevated >95 > 200 (5.18) > 130 (3.36)
Adapted from National Cholesterol Education Program Guidelines for Children and Adolescents

Adapted from the National Cholesterol Education Program Guidelines for Children and Adolescents.

Copyright 2011 American Academy of Pediatrics


2012 PREP SA ON CD-ROM

Critique 180

Item C180B. Recommended Low-density Lipoprotein (LDL) Cholesterol Concentrations for


Pharmacologic Treatment of Children and Adolescents 10 Years and Older

Patient Characteristics Recommended Cut-points


No other risk factors for cardiovascular LDL cholesterol concentration is persistently
disease >190 mg/dL (4.92 mmol/L) despite diet therapy
Other risk factors present, including LDL cholesterol concentration is persistently
obesity, hypertension, or cigarette smoking >160 mg/dL (4.14 mmol/L) despite diet therapy
or positive family history of premature
cardiovascular disease
Children who have diabetes mellitus Pharmacologic treatment should be considered
when LDL cholesterol concentration is 130
mg/dL (3.36 mmol/L)
LDL=low density lipoprotein. Reprinted with permission from Daniels SR, Greer FR; Committee on
Nutrition. Lipid screening and cardiovascular health in childhood. Pediatrics. 2008;122:198-208

(Reprinted with permission from Daniels SR, Greer FR; Committee on Nutrition. Lipid screening and
cardiovascular health in childhood. Pediatrics. 2008;122:198-208)

Copyright 2011 American Academy of Pediatrics


2012 PREP SA ON CD-ROM

Question 181
A 17-year-old girl presents for evaluation after a first generalized tonic-clonic seizure, which
occurred the morning after she was up late at a party. You suspect juvenile myoclonic epilepsy, a form of
epilepsy that may present with seizures in adolescence.
Of the following, the feature, if present, that is MOST consistent with this diagnosis is
A. brief, rapid jerks noted in the mornings
B. cortical dysplasia on brain magnetic resonance imaging
C. multiple hypopigmented skin macules
D. partial discharges on electroencephalography
E. positive toxicology screen for marijuana

Copyright 2012 American Academy of Pediatrics


2012 PREP SA ON CD-ROM

Critique 181 Preferred Response: A


Juvenile myoclonic epilepsy (JME) is characterized by adolescent onset, generalized tonic-clonic
seizures, and fast involuntary muscle jerks termed myoclonus. As described in the vignette, both the
first generalized tonic-clonic seizures and the myoclonic jerking commonly occur in the morning. This is a
form of generalized epilepsy, and medications effective for generalized epilepsies such as valproic acid,
phenytoin, or lamotrigine are often prescribed. Routine or sleep-deprived electroencephalography (EEG)
may show synchronous generalized bursts of epileptic firing, that is, characteristic spike wave bursts
present diffusely in all EEG leads.
Cortical dysplasia describes ectopic gray matter, islands of neurons that failed to migrate fully in
utero to form appropriately organized neuronal layers in the surface of cerebral cortex. The electrical
activity in such cells may propagate abnormally, leading to focal seizures. Such dysplasias are often
visible on magnetic resonance imaging (MRI), particularly once myelination is fairly mature, after 24
months of age. In contrast, the brain MRI results in JME are typically normal.
Multiple hypopigmented macules suggest a neurocutaneous disease such as tuberous sclerosis
complex, which often results in epilepsy with one or more seizure types but not as described for the girl in
the vignette. Partial discharges on EEG are abnormal electrical discharges that emanate from one brain
area and may spread to cause seizures. JME is characterized by generalized, not partial, epileptiform
discharges. A positive toxicology screen for marijuana is not specific for JME.

American Board of Pediatrics Content Specification(s):


Recognize the clinical manifestations of juvenile myoclonic epilepsy

Suggested Reading:
Mikati MA. Seizures in childhood. In: Kliegman RM, Stanton BF, St. Geme JW III, Schor NF, and
Behrman RE, eds. Nelson Textbook of Pediatrics. 19th ed. Philadelphia, PA: Saunders Elsevier;
2011:2013-2039

Major P, Thiele EA. Seizures in children: laboratory diagnosis and management. Pediatr Rev.
2007;28:405-414. DOI: 10.1542/pir.28-11-405. Available at:
http://pedsinreview.aappublications.org/cgi/content/full/28/11/405

Verrotti A, Manco R, di Marco G, Chiarelli F, Franzoni E. The treatment of juvenile myoclonic epilepsy.
Expert Rev Neurother. 2006;6:847-854. DOI: 10.1586/14737175.6.6.847. Abstract available at:
http://www.ncbi.nlm.nih.gov/pubmed/16784408

Copyright 2012 American Academy of Pediatrics


2012 PREP SA ON CD-ROM

Question 182
You are treating an 18-month-old child in whom glycogen storage disease type 1 (GSD1 or von
Gierke disease) was diagnosed at 2 months of age when she presented to the emergency department
with a hypoglycemic seizure and hepatomegaly. With prompt diagnosis and intervention, her condition
improved significantly, and she is doing well on overnight nasogastric cornstarch supplements, with
decreased hepatomegaly and rare episodes of symptomatic hypoglycemia. Her parents have questions
about future medical complications of GSD1 in addition to hypoglycemia and hepatomegaly.
Of the following, the MOST common long term medical problem found in children with GSD1
despite optimal treatment is
A. chronic constipation
B. decreased growth rate
C. precocious puberty
D. splenomegaly
E. thrombophlebitis

Copyright 2012 American Academy of Pediatrics


2012 PREP SA ON CD-ROM

Critique 182 Preferred Response: B


The most likely long-term issue for the girl described in the vignette is decreased growth rate;
most children who have GSD1 experience failure to thrive, followed by suboptimal growth velocity and
short stature in adulthood. Strict dietary management and better metabolic control has resulted in
improved final adult height in recent decades, but growth issues continue to be a concern. Constipation is
not a specific problem in children who have GSD1, and delayed rather than precocious puberty is seen in
poorly controlled or untreated patients. Hepatomegaly and even renomegaly are seen in those who have
GSD1 due to accumulation of glycogen, but splenomegaly does not occur. The glycogen accumulation is
responsive to medical nutrition therapies aimed at maintaining a normal serum glucose concentration.
Thrombophilia or thrombosis is not more common in children or adults who have GSD1, although
impaired platelet function associated with GSD1 can result in bleeding tendencies and recurrent epistaxis.
Newborns who have GSD1 may present with hypoglycemic seizures, but more often they are
diagnosed between 3 and 4 months of age with evidence of hepatomegaly, lactic acidosis, elevated lipid
and uric acid values, and symptomatic hypoglycemia. Other features may include full cheeks with thin
limbs, diarrhea, and cutaneous xanthomas. Metabolically, affected infants and children can develop lactic
acidosis and hypoglycemia after even very short periods of fasting. Therefore, treatment is targeted at
preventing hypoglycemia and providing sufficient nutrients to support normal somatic growth and
development. Affected infants receive frequent smaller feedings and often require overnight continuous
nasogastric (NG) feedings. Older children (>2 years) may be managed with oral cornstarch feedings twice
a day, but occasionally they also need NG feedings during the night. Allopurinol sometimes is
administered to prevent gout if uric acid concentrations do not decrease sufficiently with dietary
management. Renal stones and nephrocalcinosis are common and serious sequelae of GSD1 that can
lead to progressive kidney dysfunction. Supplemental citrate may help to delay or prevent some of the
renal complications, and patients may benefit from ongoing surveillance by a pediatric nephrologist. Lipid-
lowering agents may be necessary to treat hyperlipidemia if this remains a concern after achieving
excellent metabolic control.
Hepatic adenomas are eventually seen in most patients who have GSD1 (usually appearing after
10 years of age) and pose a risk for intrahepatic hemorrhage and even a small risk for malignant
transformation. Osteoporosis is also a common complication, with significantly reduced bone mineral
density observed in children, even before adolescence. In middle childhood, rickets and anemia may
become clinically evident. Polycystic ovarian syndrome is seen in most girls who have GSD1, but it is
unclear whether this affects long-term fertility. Less common complications include pulmonary
hypertension and occasionally pancreatitis due to elevated triglyceride concentrations.
Genetically there are two major GSD1 subtypes: type 1a due to glucose-6-phosphatase
deficiency and type 1b due to glucose-6-phosphate translocase deficiency. Although clinically similar,
type 1a is associated with the platelet dysfunction and bleeding tendency noted previously, and type 1b is
associated with impaired neutrophil and monocyte function with a poor respiratory-burst response to
stimuli that leads to secondary neutropenia and increased susceptibility to gram-positive infections as well
as oral and gastrointestinal ulcers.

American Board of Pediatrics Content Specification(s):


Be familiar with the long-term prognosis and chronic management of glycogen storage disease

Suggested Reading:
Bali DS, Chen Y-T, Goldstein JL. Glycogen storage disease type 1. GeneReviews. 2010. Available at:
http://www.ncbi.nlm.nih.gov/bookshelf/br.fcgi?book=gene&part=gsd1

Dagli AI, Zori RT, Heese BA. Testing strategies for inborn errors of metabolism in the neonate.
NeoReviews. 2008;9:291-298. DOI: 10.1542/neo.9-7-e291. Available at:
http://neoreviews.aappublications.org/cgi/content/full/9/7/e291

Levy PA. Inborn errors of metabolism. Part 1: Overview. Pediatr Rev. 2009;30:130-138. DOI:
10.1542/pir.30-4-131. Available at: http://pedsinreview.aappublications.org/cgi/content/full/30/4/131

Copyright 2012 American Academy of Pediatrics


2012 PREP SA ON CD-ROM

Safouh H, Elsayyed R, Mansour S, Taha G, Elshabrawy M, Elkaraksy H. Renal involvement in children


with glycogen-storage disease [abstract]. Pediatrics. 2008;121:S143-S144. DOI: 10.1542/peds.2007-
2022TTTTT. Available at:
http://pediatrics.aappublications.org/cgi/content/abstract/121/Supplement_2/S143-b

Copyright 2012 American Academy of Pediatrics


2012 PREP SA ON CD-ROM

Question 183
You are seeing a 15-year-old sexually active girl who complains of vague lower abdominal pain
and a vaginal discharge. She has no systemic symptoms but has experienced intermittent dysuria over
the past week. She believes that she needs only a prescription for a yeast infection because she was
treated for this a few weeks ago but the discharge did not resolve completely.
Of the following, the MOST appropriate next step is to
A. obtain a vaginal swab for a wet mount evaluation only
B. perform a speculum and bimanual examination
C. perform an external genital inspection only
D. provide an antifungal prescription
E. send a urine specimen for culture only

Copyright 2012 American Academy of Pediatrics


2012 PREP SA ON CD-ROM

Critique 183 Preferred Response: B


Newer techniques and changes in recommendations have made screening of the asymptomatic
adolescent for sexually transmitted infections noninvasive and easy. The age for a routine first
Papanicolaou test (Pap smear for cervical cancer screening) has been raised to 21 years to prevent
unnecessary and possibly harmful evaluations and treatments to the cervix in younger patients. Nucleic
acid amplification tests (NAATs) to screen for Neisseria gonorrhoeae and Chlamydia trachomatis
infections can be completed with urine samples or vaginal swabs. Thus, given these recommendations,
there is no need to perform a routine pelvic examination in an asymptomatic sexually active female.
However, these changes should not be interpreted as a reason not to perform a pelvic examination in a
symptomatic adolescent because many conditions could be missed without such an examination.
The adolescent described in the vignette is sexually active and has dysuria, a vaginal discharge,
and abdominal pain. As a result, a speculum and bimanual examination is the most appropriate next step
in her evaluation. The presence of dysuria raises the possibility of urethritis from a sexually transmitted
infection as well as a urinary tract infection. Therefore, in addition to a urine culture, her external genitalia
should be inspected carefully to identify lesions that could result in external dysuria (pain when urine
flows over an external lesion such as an ulcer), the presence of a vaginal discharge, and discharge from
the urethra that may be visible with or without milking the urethra. A vaginal discharge may result from
either vaginitis or cervicitis and, therefore, requires more than inspection of the external genitalia and a
blind swab of the vagina for wet mount evaluation or NAAT testing. Inspection for the nature of the
discharge, the presence of a foreign body, possible cervical ulcers, discharge from the cervical os, and
friability help with immediate identification of a cause for the presenting complaint. In addition, the
presence of abdominal pain in this girl warrants a bimanual examination to rule out pelvic inflammatory
disease (PID). Other reasons to consider a complete pelvic examination include menstrual disorders such
as delayed onset of menarche, lack of or excessive bleeding, or severe menstrual cramps as well as
unexplained pelvic pain, pregnancy-related complaints, or suspected abuse. For adolescents who are not
sexually active, a rectoabdominal examination can substitute for a vaginal bimanual examination.
Prescription of medications without an evaluation generally should be avoided. A yeast infection
would not be a cause of lower abdominal pain for this girl, and possible PID would be missed without a
speculum and bimanual examination.

American Board of Pediatrics Content Specification(s):


Know the value of a pelvic examination in the diagnosis of sexually transmitted diseases
Know the indications for a pelvic examination in adolescents

Suggested Reading:
ACOG Committee on Practice BulletinsGynecology. ACOG Practice bulletin. number 109: cervical
cytology screening. Obstet Gynecol. 2009;114:1409-1420. DOI: 10.1097/AOG.0b013e3181c6f8a4

Braverman PK, Breech L; The Committee on Adolescence. Gynecologic examination for adolescents in
the pediatric office setting. Pediatrics. 2010;126:583-590. DOI: 10.1542/peds.2010-1564. Available at:
http://pediatrics.aappublications.org/cgi/content/full/126/3/583

Cavanaugh RM Jr. Screening adolescent gynecology in the pediatrician's office: have a listen, take a
look. Pediatr Rev. 2007;28:332-342. DOI: 10.1542/pir.28-9-332. Available at:
http://pedsinreview.aappublications.org/cgi/content/full/28/9/332

Emans JS. Office evaluation of the child and adolescent. In: Emans SJH, Laufer MR, Goldstein DP, eds.
Pediatric and Adolescent Gynecology. 5th ed. Philadelphia, PA: Lippincott Williams & Wilkins, a Wolters
Kluwer business; 2005:1-50

Hillard PJA. Consultation with the specialist: dysmenorrhea. Pediatr Rev. 2006;27:64-71. DOI:
10.1542/pir.27-2-64. Available at: http://pedsinreview.aappublications.org/cgi/content/full/27/2/64

Copyright 2012 American Academy of Pediatrics


2012 PREP SA ON CD-ROM

Weitzel M, Emans JS. Gynecological examination of the adolescent female. In Neinstein LS, Gordon CM,
Katzman DK, Rosen DS, Woods ER, eds. Adolescent Health Care: A Practical Guide. 5th ed.
Philadelphia, PA: Lippincott Williams & Wilkins, a Wolters Kluwer business; 2008:665-670

Copyright 2012 American Academy of Pediatrics


2012 PREP SA ON CD-ROM

Question 184
You are caring for an 8-year-old girl who was involved in a motor vehicle-pedestrian crash.
Despite maximal medical and surgical therapy, she developed intractable intracranial hypertension, and
she now has fixed and dilated pupils as well as an absence of corneal, cough, and gag reflexes. Her
temperature is 36.0C, heart rate is 90 beats/min, respiratory rate is 25 breaths/min on the mechanical
ventilator, blood pressure is 95/60 mm Hg, and oxygen saturation is 96%. She is in a cervical collar due
to a suspected cervical spine injury, and her tympanic membranes are ruptured bilaterally. Results of the
apnea test (absence of respiratory movements and a rise in PaCO2 after temporarily disconnecting the
ventilator) are consistent with brain death.
Of the following, the MOST commonly used ancillary test to support the diagnosis of brain death
in this situation is
A. cerebral blood flow studies
B. electroencephalography
C. magnetic resonance imaging of the brain
D. somatosensory evoked potentials
E. transcranial Doppler ultrasonography

Copyright 2012 American Academy of Pediatrics


2012 PREP SA ON CD-ROM

Critique 184 Preferred Response: A


Brain death can be defined as the clinical demonstration of irreversible cessation of function of
the entire brain, including the cerebral cortex and the brainstem. In 1987, a multidisciplinary task force
created guidelines for the clinical diagnosis of brain death in pediatric patients, although substantial
variation exists in the standards and their application. Despite these variations, most institutional,
professional society, or state guidelines emphasize the following:

History:
Known and irreversible cause
Absence of confounding factors such as:
Central nervous system depressant drugs
Hypothermia
Neuromuscular blockers
Severe electrolyte and metabolic disorders that significantly affect consciousness
Unresuscitated shock
Clinical Criteria:
Comatose without spontaneous movement or respiratory effort
No response to auditory or visual stimuli
Bilateral absence of motor responses, excluding spinal reflexes
Absence of brainstem reflexes:
Pupils
Absence of pupillary light reflex
Pupils midpoint or dilated
Ocular movement
Absence of oculovestibular reflex (ice-water caloric test)
This test should not be performed if tympanic membranes are not intact
Absence of oculocephalic reflex (dolls eye test)
This test should not be performed when cervical instability is proven or suspected
Facial sensation and facial motor response
Absence of corneal reflex
Pharyngeal and tracheal reflexes
Absence of cough reflex
Absence of gag reflex
Examination Interval:
Two examinations by separate clinicians are recommended, with a period of observation that
varies, depending on patient age, between examinations. The waiting period is the most variable
component of both guidelines and actual practice.

Apnea Test:
No respiratory effort in response to apnea and a rise in PaCO2, as documented by blood gas
assessment
Confounding factors may interfere with the clinical diagnosis of brain death, so that the diagnosis
cannot be made with certainty on clinical grounds alone. Ancillary testing should be used to support the
diagnosis of brain death when part of the clinical criteria cannot be reliably performed or evaluated (eg,
extensive facial trauma or clinical instability preventing completion of the apnea test) or when the validity
of the clinical examination may be compromised by sedating medications or metabolic abnormalities.
Ancillary studies that have been used in adults include cerebral angiography, electroencephalography
(EEG), radionuclide imaging studies, somatosensory evoked potentials, and transcranial Doppler
ultrasonography. Each type of study has advantages and limitations, and local expertise and availability
may help determine the most appropriate study in a specific situation.
Documentation of electrocerebral silence on an EEG was the recommended confirmatory
ancillary study in the 1987 guidelines. However, EEGs cannot detect deep cerebral or brainstem activity;

Copyright 2012 American Academy of Pediatrics


2012 PREP SA ON CD-ROM

electrical interference can occur in an intensive care unit setting; and hypothermia, sedatives, and
metabolic disturbances can affect the readings. More recent guidelines usually recommend a cerebral
flow study [angiography or nuclear medicine flow (Item C184)] as the test of choice. A review of pediatric
brain death determinations in Southern California from 2000 to 2004 demonstrated that ancillary studies
were used in 112 of 277 pediatric brain death determinations, with cerebral blood flow studies used three
times as often as EEGs. Other ancillary studies, including transcranial Doppler ultrasonography,
somatosensory evoked potentials, and computed tomography scan or magnetic resonance imaging with
angiography have not been adequately studied in pediatric patients to recommend their use.

American Board of Pediatrics Content Specification(s):


Recognize the criteria for brain death
Recognize the role of neurodiagnostic studies in the determination of brain death

Suggested Reading:
Antommaria AHM. Ethics for the pediatrician: conceptual and ethical issues in the declaration of death.
Pediatr Rev. 2010;31;427-430. DOI: 10.1542/pir.31-10-427. Available at:
http://pedsinreview.aappublications.org/cgi/content/full/31/10/427

Crain N, Boyle RJ. In brief: pediatric brain death. Pediatr Rev. 2002;23:222-223. DOI: 10.1542/pir.23-6-
222. Available at: http://pedsinreview.aappublications.org/cgi/content/full/23/6/222

Mathur M, Petersen LC, Stadtler M, et al. Variability in pediatric brain death determination and
documentation in southern California. Pediatrics. 2008;121;988-993. DOI: 10.1542/peds.2007-1871.
Available at: http://pediatrics.aappublications.org/cgi/content/full/121/5/988

Report of a Special Task Force: Guidelines for the determination of brain death in children. Pediatrics.
1987;80;298-300. Available at: http://pediatrics.aappublications.org/cgi/content/abstract/80/2/298

Shemie S, Doig C, Dickens B, et al. Severe brain injury to neurological determination of death: Canadian
forum recommendations. CMAJ. 2006;174; S1-S13. DOI: 10.1503/cmaj.045142. Available at:
http://www.ncbi.nlm.nih.gov/pmc/articles/PMC1402399/?tool=pubmed
nd
Wijdicks EFM. Brain death. 2 ed. New York, NY: Oxford University Press: 2011

Copyright 2012 American Academy of Pediatrics


2012 PREP SA ON CD-ROM

Critique 184

(Courtesy of D Krowchuk)
Brain scan in brain death: Radionuclide brain scan of a 14-month-old child who presented with diffuse
cerebral edema and transtentorial herniation demonstrates no intracerebral blood flow.

Copyright 2011 American Academy of Pediatrics


2012 PREP SA ON CD-ROM

Question 185
A 15-year-old girl presents for evaluation of irregular menses. She had menarche at 12 years of
age but has had irregular periods ever since. She has had a period every 2 to 3 months for the past year.
She has a strong family history of irregular menses and type 2 diabetes. She is 160 cm tall and weighs 85
kg. Her blood pressure is 135/85 mm Hg. Hemoglobin A1c measures 5.3% (0.053). A urine pregnancy
test is negative. No other laboratory evaluations have been obtained in the past 5 years.
Of the following, the next BEST step in managing this patient is to measure
A. estradiol
B. fasting blood glucose
C. fasting lipids
D. luteinizing and follicle-stimulating hormones
E. postprandial glucose

Copyright 2012 American Academy of Pediatrics


2012 PREP SA ON CD-ROM

Critique 185 Preferred Response: C


The girl described in the vignette has metabolic syndrome, a collection of signs, symptoms, and
laboratory findings that are associated with a markedly increased risk for diabetes and microvascular and
macrovascular disease. Although there is no unified definition in either adults or children, currently used
definitions typically involve the presence of at least three of the following abnormalities: 1) obesity, 2)
hypertension, 3) hypertriglyceridemia, 4) low high-density lipoprotein cholesterol, and 5) glucose
intolerance. At present, the metabolic syndrome is found in 5% of children and adolescents in population-
based studies and in up to 50% of severely obese youth. Because there is considerable overlap between
the metabolic syndrome and polycystic ovarian syndrome (PCOS), pediatricians must be aware of the
need to screen women who have signs and symptoms of PCOS for metabolic syndrome.
The girl described in the vignette has irregular menses, hypertension, and a high-normal
hemoglobin A1c (HbA1c) value. A fasting lipid profile would provide the most useful information in
determining her status as a patient who has metabolic syndrome and would document her potential need
for aggressive treatment of the associated comorbidities. Estradiol and gonadotropin measurements are
of limited value in evaluating women who have irregular menses, although women who have PCOS
typically have relatively higher luteinizing hormone-to-follicle-stimulating hormone ratios compared with
women who have regular menstrual cycles. Assessment of fasting or postprandial blood glucose would
be reasonable screening tests for glucose abnormalities in this patient, but the results would be unlikely to
provide additional highly useful information beyond the already provided normal HbA1c value.

American Board of Pediatrics Content Specification(s):


Plan appropriate screening tests for metabolic syndrome
Identify the risk factors that necessitate screening tests for metabolic syndrome

Suggested Reading:
Ford ES, Li C. Defining the metabolic syndrome in children and adolescents: will the real definition please
stand up? J Pediatr. 2008;152:160-164. DOI: 10.1016/j.jpeds.2007.07.056. Available at:
http://www.jpeds.com/article/S0022-3476(07)00754-8/abstract

Lee S, Bacha F, Gungor N, Arslanian S. Comparison of different definitions of pediatric metabolic


syndrome: relation to abdominal adiposity, insulin resistance, adiponectin, and inflammatory biomarkers.
J Pediatr. 2008;152:177-184. DOI: 10.1016/j.jpeds.2007.07.053. Abstract available at:
http://www.ncbi.nlm.nih.gov/pubmed/18206686

Weiss R, Dziura J, Burgert TS, et al. Obesity and the metabolic syndrome in children and adolescents. N
Engl J Med. 2004;350:2362-2374. Available at:
http://www.nejm.org/doi/full/10.1056/NEJMoa031049#t=articleBackground

Copyright 2012 American Academy of Pediatrics


2012 PREP SA ON CD-ROM

Question 186
For the past 2 years, you have been providing medications for a 9-year-old girl in whom you
diagnosed attention-deficit/hyperactivity disorder (ADHD) using parent and teacher assessments and
family history. You have sequentially prescribed methylphenidate, dextroamphetamine, and currently
atomoxetine. All have yielded the same benefits on her inattention and impulsivity but have not improved
her occasional aggressive behaviors. Today her mother brings her in, saying that she has had a
significant worsening in her aggression and is now getting into fights at school. When asked about the
most recent incident, the girl replies, Sheryl was making faces at me for days, so I hit her, and it serves
her right. The mother describes her daughter as being chronically vindictive at home, aggressively
paying back her siblings as much as several days later when she feels she has been wronged. The
mother reports no major changes in the household and no traumatic or bullying incidents for the girl. She
indicates that her husband has a history of anger management difficulties.
Of the following, the MOST appropriate next step in care is to
A. add guanfacine to the girls regimen
B. have the mother and teacher complete the Vanderbilt Diagnostic Rating Scale
C. prescribe risperidone at bedtime
D. refer the family for behavior management training
E. wean the girl off the atomoxetine and initiate a trial of lisdexamfetamine

Copyright 2012 American Academy of Pediatrics


2012 PREP SA ON CD-ROM

Critique 186 Preferred Response: D


The girl described in the vignette has responded well to standard treatment for ADHD, but her
persistent calculated aggressive behaviors suggest the need for further interventions. Children who
exhibit aggressive behavior problems are most likely to be helped by behavior management training,
which educates individuals in the world around them (parents, teachers) to reward the children for
positive social behaviors and to have consistent, appropriate consequences such as physical limits and
the withdrawal of positive attention for undesired behaviors. Many different types of behavior
management training, including Parent Management Training, Emotion Coaching, and 123 Magic, have
been shown to accomplish this goal.
The girls described aggression is considered a cold type of aggression, in that it is calm,
planned, and calculated to obtain a goal. Medications such as risperidone and guanfacine typically do not
reduce this type of aggression. Hot aggression, which is impulsive, poorly planned, and associated with
fight-or-flight arousal, is much more likely to respond to medication. The situation in which medications
reduce the frequency and severity of hot aggression typically involves a treatable comorbidity such as
ADHD that is being addressed successfully with a medication (most commonly, stimulants for ADHD).
Extreme impulsive or hot aggression that is unresponsive to behavior management might be treated
with a nonspecific impulsive aggression medication (eg, risperidone or valproic acid), but such an
approach should be undertaken cautiously because of potential medication adverse effects.
The Vanderbilt Diagnostic Rating Scale aids in determining the current presence/absence of
ADHD symptoms, but this assessment is not indicated because the diagnosis has been confirmed and
the condition has responded to appropriate therapy. In addition, there is no need to change ADHD
medication to another agent such as lisdexamfetamine (prodrug of dextroamphetamine) in a child whose
ADHD symptoms are already well-controlled and have responded equally well to different medications.

AAP Mental Health Competency:


Know the difference between "hot" or impulsive aggression, and "cold" or planned aggression

Suggested Reading:
American Academy of Pediatrics Task Force on Mental Health. Distruptive Behavior and Aggression
Cluster Guidance. Addressing Mental Health Concerns in Primary Care: A Clinician's Toolkit [CD-ROM].
Elk Grove Village, IL: American Academy of Pediatrics; 2010

Connor DF. Aggression and Antisocial Behavior in Children and Adolescents. New York, NY: The
Guilford Press; 2002

Hilt RJ, French WP. Aggression by children and adolescents. In: Cheng K, Myers KM, eds. Child and
Adolescent Psychiatry: The Essentials. 2nd ed. Philadelphia, PA: Lippincott, Williams & Wilkins, a Wolters
Kluwer business; 2011:400-415

Weisz JR, Hawley KM, Doss AJ. Empirically tested psychotherapies for youth internalizing and
externalizing problems and disorders. Child Adolesc Psychiatr Clin N Am. 2004;13:729-815. Abstract
available at: http://www.ncbi.nlm.nih.gov/pubmed/15380784

Copyright 2012 American Academy of Pediatrics


2012 PREP SA ON CD-ROM

Question 187
A 4-month-old infant comes to your office for a health supervision visit. When you pass through
the waiting room, you observe his young mother prop the infants bottle while he is in his stroller.
Of the following, the MOST appropriate action is to
A. advise the mother to prop only bottles containing water
B. discuss the advantages of holding her baby during feedings
C. explain that the child is too young to have the bottle propped
D. recommend that the mother obtain a bottle sling
E. tell the mother that a bottle should not be propped when the infant is falling asleep

Copyright 2012 American Academy of Pediatrics


2012 PREP SA ON CD-ROM

Critique 187 Preferred Response: B


Infants should be fed when hungry, warm, and dry, not just when they are fussy in an attempt to
quiet them. In addition, the bottle should be held, not propped, regardless of the infants age. Even the
use of a safe bottle holder such as a bottle sling should be avoided. Parents should be counseled that
holding their baby when feeding enhances physical closeness and a feeling of security for the baby.
Propping a bottle increases the risk of choking and the development of otitis media. Parents also should
be advised that their infant should not be put to bed with a bottle of formula or juice, a practice that could
lead to dental decay.
A parent should not force a baby to eat. If the child stops feeding, the parent should try to burp
the baby. If the infant still does not want to feed after burping, he or she has had enough. If the baby
prefers the formula to be warmed, the parent should not use the microwave, which might create hot spots
that can burn a babys mouth.

American Board of Pediatrics Content Specification(s):


Distinguish normal variations in feeding from patterns that reflect poor parenting practices (eg,
feeding to quiet the infant, propping the bottle)

Suggested Reading:
Berkowitz RJ, Den Besten PK, Karp JM. Prevention of dental caries. In: McInerney TK, Adam HM,
Campbell DE, Kamat DM, Kelleher KJ, Hoekelman RA, eds. AAP Textbook of Pediatric Care. Elk Grove
Village, IL: American Academy of Pediatrics; 2009:Chapter 33.

Stettler N, Bhatia J, Parish A, Stallings VA. The feeding of infants and children. In: Kleigman RM, Stanton
BF, St. Geme JW III, Schor NF, and Behrman RE, eds. Nelson Textbook of Pediatrics. 19th ed.
Philadelphia, PA: Saunders Elsevier; 2011:160-170

Copyright 2012 American Academy of Pediatrics


2012 PREP SA ON CD-ROM

Question 188
A 4-month-old boy presents with his second episode of oral thrush. He is growing and developing
normally and has had no other infections. He was born at term by cesarean section. The pregnancy was
complicated by maternal diabetes that was managed with diet. There were no problems in the nursery,
and he was discharged with his mother, who is a nurse, at 2 days of age. His umbilical cord fell off at 2
weeks of age. He has been exclusively breastfed. He has received his 2-month and 4-month vaccines
without significant reaction and has received no medications except for oral multivitamins. The mother
has a history of eczema, but there is no history of immune deficiency disorder on either side of the family.
After discussing the diagnosis with the mother, she asks about risk factors for recurrent episodes of
thrush.
Of the following, the factor that is MOST likely to be associated with an increased risk for
recurrent oral thrush is
A. breastfeeding
B. cesarean section
C. contaminated vitamin dropper
D. maternal eczema
E. maternal gestational diabetes

Copyright 2012 American Academy of Pediatrics


2012 PREP SA ON CD-ROM

Critique 188 Preferred Response: C


Oral mucocutaneous candidiasis or thrush is a common infection in the first 6 postnatal months,
presumably due to the infants immunologic immaturity and compromise of the epithelial barriers. Candida
albicans is a ubiquitous pathogen that frequently colonizes the skin, mucous membranes, vagina, and
gastrointestinal tract. Contaminated bottle nipples, pacifiers, or droppers can be sources of candidal
infection in infants. A contaminated vitamin dropper most likely is the source for the infant described in the
vignette. Although breastfed babies can get thrush and a mothers nipples can be infected with yeast, the
incidence of thrush is higher in formula-fed children.
Maternal vaginal colonization with Candida has been associated with acquisition, and thrush
occurs less frequently in infants delivered by cesarean section. Atopic dermatitis has not been associated
with an increased risk for thrush in infancy. Poorly controlled diabetes in adults is associated with an
increased risk of candidal infection, but maternal gestational diabetes has not been implicated with thrush
development in babies.
Recurrent or persistent candidiasis beyond 6 to 12 months of age raises concern for an
immunodeficiency, specifically a defect in T cells, including that seen with human immunodeficiency virus
infection. In addition, further extension of the infection into the esophagus or systemic invasion is
frequently seen in immunodeficient patients.
Other risk factors for persistent or recurrent oral candidiasis in otherwise healthy children include
antibiotic use, inhaled steroid use without adequate rinsing afterwards, and nutritional deficiencies.

American Board of Pediatrics Content Specification(s):


Know the various conditions that predispose to persistent or recurrent candidiasis of the oral cavity in
an infant younger than 6 months of age: maternal breast colonization, contaminated vitamin dropper,
antibiotic use, pacifier use
Know the various conditions that predispose to persistent or recurrent candidiasis of the oral cavity in
children older than 6 months of age: immune deficiency, AIDS, antibiotic use

Suggested Reading:
Akpan A, Morgan R. Oral candidiasis. Postgrad Med J. 2002;78:455459. DOI: 10.1136/pmj.78.922.455.
Available at: http://www.ncbi.nlm.nih.gov/pmc/articles/PMC1742467/?tool=pubmed

American Academy of Pediatrics. Candidiasis (moniliasis, thrush). In: Pickering LK, Baker CJ, Kimberlin
DW, Long SS, eds. Red Book: 2009 Report of the Committee on Infectious Diseases. 28th ed. Elk Grove
Village, IL: American Academy of Pediatrics; 2009:245-249

Copyright 2012 American Academy of Pediatrics


2012 PREP SA ON CD-ROM

Question 189
You are examining a 3-week-old preterm infant who was born at 26 weeks gestation and is
receiving continuous positive airway pressure (CPAP) respiratory support. He has had increasing
episodes of apnea and bradycardia over the past 12 hours. On physical examination, his temperature is
37.0C, heart rate is 150 beats/min, and respiratory rate is 50 breaths/min on CPAP pressure support of 6
mm Hg with an FiO2 0.35. The infant has a flat and soft anterior fontanelle, moist mucous membranes,
normal heart and lung sounds, and benign findings on abdominal examination. His extremities are mildly
warm, and his left knee is swollen without noticeable erythema. Range of motion of the left knee is mildly
decreased compared with the right. A clean and dry dressing covers a peripherally inserted central
3 9
venous catheter in the right arm. The white blood cell count is 9.0x10 /mcL (9.0x10 /L), with 55%
polymorphonuclear leukocytes, 10% band forms, 30% lymphocytes, and 5% monocytes. Cerebrospinal
3
fluid evaluation reveals 15 white blood cells/mm with 80% lymphocytes, 10% polymorphonuclear
leukocytes, and 10% monocytes; protein of 70 mg/dL; and glucose of 60 mg/dL. Urinalysis yields normal
results. Blood, cerebrospinal fluid, and urine specimens have been sent for cultures. A magnetic
resonance imaging study reveals increased signal in the left knee. You order broad-spectrum intravenous
antibiotics.
Of the following, the next BEST step is
A. arthrocentesis of the left knee
B. bone scan of the left knee
C. radiograph of the left knee
D. removal of the central venous catheter
E. ultrasonography of the left knee

Copyright 2012 American Academy of Pediatrics


2012 PREP SA ON CD-ROM

Critique 189 Preferred Response: A


The preterm infant described in the vignette has hematogenously acquired infective osteoarthritis,
most likely caused by Staphylococcus aureus. Arthrocentesis of the knee and cultures of fluid and bone
would assist in establishing the diagnosis. The unique neonatal anatomy allows spread of bacteria from
the metaphyseal bone through the transphyseal vessels into the epiphysis and the adjacent joint.
Therefore, unlike most infections in older children, neonatal disease involves both the bone and joint
contiguously.
The most common clinical presentation of infective osteoarthritis is an occult bacteremia that
leads to inoculation of bone, and multiple sites can be involved in up to 50% of cases. The femur and tibia
are the most commonly infected bones (80%), and adjacent arthritis is common. The clinical findings,
especially in preterm infants, can be subtle. Infants may be irritable, with mild swelling of the affected area
and decreased movement of the limb. Marked limb swelling and erythema may be absent. The infants
can be afebrile and have normal vital signs. Serum inflammatory markers (C-reactive protein, erythrocyte
sedimentation rate) may not be markedly elevated and are not specific. Less often, infants can present
with overt sepsis that involves bacteremia and focal limb abnormality that is not the most prominent
symptom. Rarely, infection can result from penetrating trauma from instrumentation or procedures (eg,
catheters, heel stick).
Plain radiography is important for evaluating for fracture, but destructive bony changes from
infection will not be apparent until at least 10 days after the onset of symptoms. The sensitivity of
magnetic resonance imaging (MRI) for detecting changes consistent with osteomyelitis approaches
100%. Bone scans in neonates can be nondiagnostic. Ultrasonography of the knee in a preterm neonate
may not reveal a significant effusion and is of little utility in the patient described in the vignette, in whom
changes already are evident on MRI. Removal of the central venous catheter may be warranted if blood
cultures reveal a pathogen but is unnecessary if they are negative.

American Board of Pediatrics Content Specification(s):


Know the clinical manifestations of pyogenic arthritis in neonates compared with those in older
children

Suggested Reading:
Berberian G, Firpo V, Soto A, et al. Osteoarthritis in the neonate: risk factors and outcome. Braz J Infect
Dis. 2010;14:413-418. DOI: 10.1590/S1413-86702010000400018. Available at:
http://www.scielo.br/scielo.php?script=sci_arttext&pid=S1413-
86702010000400018&lng=en&nrm=iso&tlng=en

Krogstad P. Osteomyelitis. In: Feigin RD, Cherry JD, Demmler-Harrison GJ, Kaplan SL, eds. Feigin &
Cherrys Textbook of Pediatric Infectious Diseases. 6th ed. Philadelphia, PA: Saunders Elsevier;
2009:725-741

Nizet V, Bradley JS. Staphylococcal infections. In: Remington JS, Klein JO, Wilson CB, Nizet,
Maldonado. Infectious Diseases of the Fetus and Newborn Infant, 7th Ed. Elsevier Saunders; 2011:504

Copyright 2012 American Academy of Pediatrics


2012 PREP SA ON CD-ROM

Question 190
You recently diagnosed Burkitt lymphoma in one of your patients. He is an 8-year-old boy, and
his pediatric oncologist is planning to treat him with CODOX-M (cyclophosphamide, doxorubicin,
vincristine, methotrexate intravenously and cytarabine and methotrexate intrathecally) for cycles 1 and 3
and IVAC (ifosfamide, etoposide, cytarabine intravenously and methotrexate intrathecally) for cycles 2
and 4. As her sons longstanding pediatrician, the boys mother asks you about the adverse effects of the
therapy. Because her cousin is undergoing dialysis, she asks you specifically which medication is most
likely to damage her sons kidneys.
Of the following, the MOST nephrotoxic medication being prescribed is
A. cyclophosphamide
B. cytarabine
C. doxorubicin
D. etoposide
E. ifosfamide

Copyright 2012 American Academy of Pediatrics


2012 PREP SA ON CD-ROM

Critique 190 Preferred Response: E


Clinicians need to be familiar with adverse effects of medications being used to treat their
patients, even if they are medications prescribed by specialists. Cyclophosphamide is an alkylating agent
that has the following adverse effect profile: bone marrow suppression (especially leukopenia),
hemorrhagic cystitis (which is potentially fatal), alopecia, infertility, nausea, and vomiting. It has been
rarely (<1% of cases) associated with hepatotoxicity, hemorrhagic ureteritis, renal tubular necrosis, and
syndrome of inappropriate antidiuretic hormone secretion. Doxorubicin is an anthracycline whose best
known adverse effect is cardiac toxicity, which can be either acute (arrhythmias) or delayed (decreased
ejection fraction and congestive heart failure). In addition, leukopenia is common. Other than discoloration
of the urine, there is no known renal toxicity associated with this agent. Cytarabine is an antimetabolite
whose adverse effect profile includes myelosuppression and hepatotoxicity; it is infrequently associated
with urinary retention and renal dysfunction. Etoposide is a podophyllotoxin-derived antineoplastic agent
that has a number of adverse effects, including marrow suppression, alopecia, ovarian failure, and
gastrointestinal (GI) symptoms of nausea/vomiting. Hepatotoxicity occurs infrequently, but nephrotoxicity
is generally not associated with this agent.
Ifosfamide is an alkylating agent whose adverse effects include central nervous system toxicity
(somnolence), alopecia, myelosuppression, and GI symptoms. Further, a large number of patients can
have renal toxicity. Based on recent review, the most common manifestation of ifosfamide-induced renal
toxicity is proximal tubular dysfunction, followed by a decrease in glomerular filtration rate (GFR).
Proximal tubulopathy occurs in 30% of treated individuals, with Fanconi syndrome occurring in 5% and
decreased GFR (defined as more than three times the upper limit of normal creatinine) occurring in 20%
to 25% of patients. Among the clinical manifestations of the tubulopathy are metabolic acidosis,
hypophosphatemia, hypokalemia, hypomagnesemia, and rickets. The risk of ifosfamide nephrotoxicity is
best correlated with the total cumulative dose. Additional risk factors include young age (3 to 5 years),
concomitant use of nephrotoxic agents such as platinum drugs, renal irradiation, and unilateral
nephrectomy.
Other chemotherapeutic agents with known nephrotoxicity include the platin drugs (cisplatin and
carboplatin) and methotrexate. Cisplatin causes a dose-dependent reduction in GFR and a magnesium-
wasting tubulopathy in nearly all treated patients, resulting in hypomagnesemia, hypocalcemia, and
hypokalemia. Carboplatin is a cisplatin analog that has less nephrotoxicity. Its major adverse effect is
myelosuppression. Surprisingly, the combination of ifosfamide with carboplatin may be more nephrotoxic
(renal insufficiency/tubulopathy) than the ifosfamide/cisplatin combination. Methotrexate (MTX), when
2
used in high doses (1,000 to 33,000 mg/m ) with leucovorin, is associated with renal dysfunction in
approximately 2% of patients. The mechanism of MTX-induced renal failure is hypothesized to be due to
tubular precipitation. Importantly, renal insufficiency due to MTX appears to be completely reversible,
usually within a few weeks.
Other nephrotoxic drugs are nonsteroidal anti-inflammatory drugs (NSAIDS), which are known
inhibitors of cyclooxygenase, thereby decreasing the biosynthesis of prostaglandins. In euvolemic
patients, prostaglandins have negligible effects, but acute renal failure can result in dehydration. In
addition to their prerenal-like effects, NSAIDS have been associated with acute tubular necrosis (ATN)
and acute interstitial nephritis (AIN). Prerenal effects also can also be observed with angiotensin-
converting enzyme inhibitors and calcineurin inhibitors (cyclosporine and tacrolimus).
Antibiotics such as aminoglycosides and vancomycin have been similarly associated with ATN.
Sulfa and penicillin antibiotics have been associated with AIN. Antifungal agents such as amphotericin B
and foscarnet are associated with ATN, while the antiviral agent acyclovir is associated with renal
dysfunction due to tubular injury. Other agents, such as radiocontrast, can induce ATN. Less well-known
drugs associated with AIN are proton pump inhibitors such as omeprazole and lansoprazole.

American Board of Pediatrics Content Specification(s):


Know the drug classes that can cause renal toxicity (eg, antibiotics, NSAIDs, chemotherapeutic
agents, cyclosporine, tacrolimus)

Suggested Reading:

Copyright 2012 American Academy of Pediatrics


2012 PREP SA ON CD-ROM

Bennett WM. Cyclosporine and tacrolimus nephrotoxicity. UpToDate Online 18.3. 2010. Available for
subscription at: http://www.uptodate.com/online/content/topic.do?topicKey=renltran/7995

Jones DP, Spunt SL, Green D, Springate JE; Children's Oncology Group. Renal late effects in patients
treated for cancer in childhood: a report from the Children's Oncology Group. Pediatr Blood Cancer.
2008;51:724-731. DOI: 10.1002/pbc.21695. Available at:
http://www.ncbi.nlm.nih.gov/pmc/articles/PMC2734519/?tool=pubmed

Patzer L. Nephrotoxicity as a cause of acute kidney injury in children. Pediatr Nephrol. 2008;23:2159-
2173. DOI: 10.1007/s00467-007-0721-x. Available at:
http://www.springerlink.com/content/vj7g27m014313q40/

Rose BD. Ifosfamide nephrotoxicity. UpToDate Online 18.3. 2010. Available for subscription at:
http://www.uptodate.com/online/content/topic.do?topicKey=fldlytes/9575

Rose BD, Post TW. NSAIDs: acute kidney injury (acute renal failure) and nephrotic syndrome. UpToDate
Online 18.3. 2010. Available for subscription at:
http://www.uptodate.com/online/content/topic.do?topicKey=renlfail/6250

Copyright 2012 American Academy of Pediatrics


2012 PREP SA ON CD-ROM

Question 191
A 12-year-old boy has a 3-year history of worsening nasal congestion and anosmia. He has a
history of allergic rhinitis, but his usual medications no longer control his symptoms. On physical
examination, you see a polyp in his right naris. Sinus computed tomography scan demonstrates
opacification of his right maxillary sinus, with hyperattenuation of the mucin.
Of the following, the MOST likely cause for this boys sinus symptoms is
A. allergic fungal sinusitis
B. allergic rhinitis
C. chronic bacterial sinusitis
D. cystic fibrosis
E. primary ciliary dyskinesia

Copyright 2012 American Academy of Pediatrics


2012 PREP SA ON CD-ROM

Critique 191 Preferred Response: A


The differential diagnosis for a nasal polyp includes cystic fibrosis, asthma, chronic allergic
rhinitis, primary ciliary dyskinesia, and chronic bacterial sinusitis. The history of allergic rhinitis coupled
with unilateral sinusitis and hyperattenuated mucin (Item C191) reported for the boy in the vignette is
pathognomonic for allergic fungal sinusitis (AFS). AFS accounts for 10% to 15% of chronic rhinosinusitis
in adolescents and young adults. The condition represents an immunoglobulin (Ig)E-mediated response
to particular fungi (eg, Bipolaris or Curvularia), resulting in inflammation, thick mucin, and nasal polyposis.
Endoscopic surgery is usually required, followed by prolonged use of oral corticosteroids.
Cystic fibrosis should always be considered in children who present with nasal polyps. However,
the boy in the vignette does not have any other clinical features consistent with that diagnosis, such as
poor weight gain, nail clubbing, recurrent pneumonia, or otitis media. When compared with allergic
rhinitis, chronic bacterial sinusitis is more likely to be complicated by nasal polyposis. Polyps most
commonly occur in the ethmoid sinuses, but mucin that appears hyperattenuated is uncommon.
Hyperattenuation on computed tomography scan is believed to result from iron deposition by fungal
elements. The boy in the vignette has a history of allergic rhinitis, but uncomplicated allergic rhinitis by
itself rarely is associated with polyps or thick mucin.
Ciliary dyskinesia is an autosomal recessive disorder that results from a defect in the radial
spokes or dynein arms of cilia. Patients can present with symptoms suggestive of cystic fibrosis, but they
have normal sweat chloride test results. Situs inversus is seen in approximately 50% of patients and is
called Kartagener syndrome. Ciliary dyskinesia is less common than AFS and requires the presence of
situs inversus, an abnormal saccharin test, low exhaled nasal nitric oxide concentrations, or an abnormal
brush biopsy of nasal cilia.

American Board of Pediatrics Content Specification(s):


Know the conditions associated with nasal polyps in children: cystic fibrosis, asthma, chronic allergic
rhinitis, chronic sinusitis

Suggested Reading:
Campbell JM, Graham M, Gray HC, Bower C, Blaiss MS, Jones SM. Allergic fungal sinusitis in children.
Ann Allergy Asthma Immunol. 2006;96:286-290. DOI: 10.1016/S1081-1206(10)61237-9. Abstract
available at: http://www.ncbi.nlm.nih.gov/pubmed/16498849

Tan R, Spector S. Pediatric sinusitis. Curr Allergy Asthma Rep. 2007;7:421-426. Abstract available at:
http://www.ncbi.nlm.nih.gov/pubmed/17986371

Taylor A, Adam HM. In brief: sinusitis. Pediatr Rev. 2006;27:395-397. DOI: 10.1542/pir.27-10-395.
Available at: http://pedsinreview.aappublications.org/cgi/content/full/27/10/395

Copyright 2012 American Academy of Pediatrics


2012 PREP SA ON CD-ROM

Critique 191

(Courtesy of K Waibel)
Computed tomography scan, as described for the boy in the vignette, showing opacification of the right
maxillary sinus. Unlike in bacterial sinusitis, the opacification is not homogeneous. Rather, there are
areas of hyperattenuation (whiter color, arrow).

Copyright 2011 American Academy of Pediatrics


2012 PREP SA ON CD-ROM

Question 192
A 7-year-old boy is brought to the emergency department after being poked in the left eye while
he and his friends were playing with pointed sticks. He reports that the injury occurred approximately 2
hours ago and that the pain is worsening. He states that the light hurts his eye and that he has been
unable to see clearly with that eye since the injury. On physical examination, the boy appears
uncomfortable and is holding his left eye closed. There is marked epiphora and conjunctival injection. His
extraocular movements are intact, but his left pupil is elliptically shaped and poorly reactive to light. He
can discern only light with his left eye. You decide to consult an ophthalmologist.
Of the following, the MOST appropriate next step, while waiting for the ophthalmologist to arrive,
is to
A. administer an oral dose of ibuprofen
B. apply topical antibiotic ointment to the eye
C. have the child lay on the bed with his head elevated 45 degrees
D. place an occlusive eye patch over the left eye
E. sedate the child with ketamine

Copyright 2012 American Academy of Pediatrics


2012 PREP SA ON CD-ROM

Critique 192 Preferred Response: C


The boy described in the vignette has sustained trauma to his left eye and requires a careful
evaluation to determine the location and extent of injury. Initial evaluation should include identification of
the timing, mechanism, and location of the injury; assessment of visual symptoms, including change in
vision and presence of flashing lights or floaters; and a physical examination that progresses externally
from the face, lids, and orbits internally to the globe. During the initial assessment, management should
be directed at preventing any increase in intraocular pressure, either by direct pressure to the eye, patient
position, or medication. This includes eye protection using an eye shield (not a pressure patch),
recumbent positioning with the head of the bed at 45 degrees, and avoidance of any medications that
might increase intraocular pressure, such as ketamine. In addition, ibuprofen should be avoided because
it may increase the likelihood of rebleeding in the case of hyphema, and ointment should be avoided
because it will make the eye examination difficult. Tetanus immunoprophylaxis should not be overlooked,
especially if globe rupture is suspected.
After the patient has been examined for life-threatening or other serious injuries, the eye
examination should begin with inspection of the face and lids for evidence of swelling, bruising, or
lacerations. Lid contusions and lacerations should raise suspicion for globe or orbital injury. The rim of the
orbit should be palpated for deformities and extraocular movements assessed. The lid should be everted
to examine for any foreign material that may be trapped underneath. Before globe examination, visual
acuity should be determined using a Snellen chart. Evaluation of the globe begins with inspection of the
conjunctivae for foreign bodies and hemorrhage. The cornea should be assessed with a penlight as well
as fluorescein stain and blue light for abrasions or evidence of penetration. Pupils should be checked for
appropriate direct and consensual responses as well as for symmetry. An abnormally shaped pupil
strongly suggests the presence of an open globe. The anterior chamber/iris/lens should be inspected with
a penlight and, if possible, slitlamp for evidence of hyphema, uveitis, or lens dislocation. Lastly, the
fundus should be examined. Based on this methodic examination (Item C192), the injury can be
characterized and definitive treatment provided.

American Board of Pediatrics Content Specification(s):


Plan the acute management of an eye injury

Suggested Reading:
Andreoli CM, Gardiner MF. Open globe injuries: emergent evaluation and initial management. UpToDate
Online 18.3. 2010. Available for subscription at:
http://www.uptodate.com/online/content/topic.do?topicKey=ped_trau/10742

Bord SP, Linden J. Trauma to the globe and orbit. Emerg Med Clin North Am. 2008;26:97123. DOI:
10.1016/j.emc.2007.11.006. Abstract available at: http://www.ncbi.nlm.nih.gov/pubmed/18249259

Golden DJ, Acerra JR. Globe rupture: treatment and medication. eMedicine Specialties, Emergency
Medicine, Ophthalmology. 2010. Available at: http://emedicine.medscape.com/article/798223-treatment

Khaw PT, Shah P, Elkington AR. Injury to the eye. BMJ. 2004;328:3638. DOI: 10.1136/bmj.328.7430.36.
Available at: http://www.ncbi.nlm.nih.gov/pmc/articles/PMC313907/?tool=pubmed

Podbielski DW, Surkont M, Tehrani N, Ratnapalan RS. Pediatric eye injuries in a Canadian emergency
department Can J Ophthalmol 2009;44:51922.

Copyright 2012 American Academy of Pediatrics


2012 PREP SA ON CD-ROM

Critique 192

Copyright 2011 American Academy of Pediatrics


2012 PREP SA ON CD-ROM

Question 193
You are evaluating a 15-year-old girl for primary amenorrhea. She was well until 7 years of age,
at which time she was diagnosed with type 1 diabetes mellitus. Her diabetic control has been good for the
past 2 years, with an acceptable hemoglobin A1C reading 6 weeks ago. Both the girl and her mother are
concerned about her delayed menarche. The girl also reports experiencing irregular bowel habits and
intermittent abdominal discomfort for the past year, with occasional loose stools. The mother has been
diagnosed with irritable bowel syndrome. Physical examination of the well-appearing, thin adolescent
shows a height of 162 cm, weight of 45 kg, and Sexual Maturity Rating of 3. There are no other findings
of note on the remainder of the physical examination.
Of the following, the test that is MOST helpful in determining the cause of this girls symptoms is
A. abdominal magnetic resonance imaging
B. human leukocyte antigen typing
C. pelvic ultrasonography
D. serum follicle-stimulating hormone and luteinizing hormone assessment
E. tissue transglutaminase antibody assessment

Copyright 2012 American Academy of Pediatrics


2012 PREP SA ON CD-ROM

Critique 193 Preferred Response: E


The adolescent girl described in the vignette, who has well-controlled diabetes mellitus, is being
evaluated for primary amenorrhea. When screened, as many as 10% of patients attending diabetes
clinics test positive for celiac disease (CD). Tissue typing data have confirmed that both type 1 diabetes
and CD exhibit a high prevalence of the human leukocyte antigen (HLA) alleles DQ2 and DQ8.
Accordingly, particularly because of her gastrointestinal symptoms and delayed puberty, CD must be
ruled out for this girl. Antiendomysial antibodies (EMA) or antibodies to tissue transglutaminase (TTG) are
currently the most sensitive and specific available serologic markers for the disorder. Abnormal results
indicate the need to proceed to endoscopic small intestinal biopsy, which remains the gold standard for
diagnosing CD.
CD was, until recently, considered a rare disorder in the United States. It is characterized by
symptoms related to macronutrient malabsorption, including chronic diarrhea, abdominal distension, and
weight loss. However, best evidence from screening studies suggests that its prevalence in this country
may be as high as 1 in 130 individuals, with even higher rates found in certain European nations (eg, 1 in
99 individuals in Finland). The presentation of CD may reflect diverse clinical manifestations (Item C193).
Individuals in whom CD should be suspected must extend well beyond patients who manifest
typical gastrointestinal symptoms and should include relatives of patients who have CD and people who
have short stature, anemia, fatigue, hypertransaminasemia, and osteoporosis. Large-scale screening
studies have demonstrated significant mines of individuals who have CD, both in at-risk groups and in
those previously believed not to be at risk. These data were based upon EMA screening, followed by a
TTG determination as well as a confirmatory DQ2/DQ8 HLA haplotype. In first-degree relatives of patients
who had CD, disease prevalence was found to be 4% to 5%, and this rate was as high in asymptomatic
first- and second-degree relatives as it was in relatives manifesting CD symptoms. Results of screening of
healthy individuals who were believed not to be at risk demonstrated an overall CD prevalence rate of
0.75%. However, this rate increased significantly in nonrelatives of CD patients who exhibited a wide
range of disorders, even in the absence of gastrointestinal symptoms. These individuals, now designated
as at high risk for CD, included people who have type 1 diabetes mellitus, anemia refractory to iron
therapy, arthritis, osteoporosis, infertility, and trisomy 21.
Available evidence has confirmed both the sensitivity and specificity of the EMA and TTG assays
for CD screening. Because CD represents an autoimmune disorder arising from the immune response to
gliadin, the ethanolic digest of gluten contained in wheat, barley, and rye, serum antigliadin
immunoglobulin G and A concentrations have long been employed as screening studies. However, these
antibody assays, although exhibiting a high sensitivity rate for CD, are far less specific than either EMA or
TTG. Accordingly, they should not be routinely used for screening suspected patients. Virtually 100% of
those who have CD demonstrate either the DQ2 or DQ8 haplotype, but the prevalence of these HLA
alleles may be as high as 30% in the general population. HLA testing, therefore, should be reserved
primarily for evaluating symptomatic patients and patients who have associated conditions in whom
antibody screening or small intestinal biopsy results are equivocal. In these cases, the absence of both
DQ2 and DQ8 haplotypes effectively rules out CD as a diagnosis. Assessment of serum follicle-
stimulating hormone and luteinizing hormone as well as abdominal magnetic resonance imaging and
pelvic ultrasonography may provide information about the status of sexual development in a patient who
has amenorrhea, but they will not provide evidence leading to the correct diagnosis.

American Board of Pediatrics Content Specification(s):


Recognize the clinical manifestations of celiac disease

Suggested Reading:
Catassi C, Kryszak D, Louis-Jacques O, et al. Detection of celiac disease in primary care: a multicenter
case-finding study in North America. Am J Gastroenterol. 2007;102:1454-1460. DOI: 10.1111/j.1572-
0241.2007.01173.x. Abstract available at: http://www.ncbi.nlm.nih.gov/pubmed/17355413

Fasano A, Berti I, Gerarduzzi T, et al. Prevalence of celiac disease in at-risk and not-at-risk groups in the
United States: a large multicenter study. Arch Intern Med. 2003;163:286-292. Available at:
http://archinte.ama-assn.org/cgi/content/full/163/3/286

Copyright 2012 American Academy of Pediatrics


2012 PREP SA ON CD-ROM

Green PH. The many faces of celiac disease: clinical presentation of celiac disease in the adult
population. Gastroenterology. 2005;128(4 suppl):S74-S78. Abstract available at:
http://www.ncbi.nlm.nih.gov/pubmed/15825130

Hill ID, Dirks MH, Liptak GS, et al. Guideline for the diagnosis and treatment of celiac disease in children:
recommendations of the North American Society for Pediatric Gastroenterology, Hepatology and
Nutrition. J Pediatr Gastroenterol Nutr. 2005;40:1-19. Available at:
http://journals.lww.com/jpgn/Fulltext/2005/01000/Guideline_for_the_Diagnosis_and_Treatment_of.1.aspx

Lurz E, Scheidegger U, Spalinger J, Schni M, Schibli S. Clinical presentation of celiac disease and the
diagnostic accuracy of serologic markers in children. Eur J Pediatr. 2009;168:839-845. DOI:
10.1007/s00431-008-0845-4. Available at: http://www.springerlink.com/content/23463731r51u0431/

Mki M, Mustalahti K, Kokkonen J, et al. Prevalence of celiac disease among children in Finland. N Engl
J Med. 2003;348:2517-2524. Available at: http://www.nejm.org/doi/full/10.1056/NEJMoa021687#t=article

Narula P, Porter L, Langton J, et al. Gastrointestinal symptoms in children with type 1 diabetes screened
for celiac disease. Pediatrics. 2009;124:e489-e495. DOI: 10.1542/peds.2008-2434. Available at:
http://pediatrics.aappublications.org/cgi/content/full/124/3/e489

Copyright 2012 American Academy of Pediatrics


2012 PREP SA ON CD-ROM

Critique 193

Item C193. Gastrointestinal and Nongastrointestinal Symptoms of Celiac Disease

! Gastrointestinal ! Nongastrointestinal
o Abdominal distension o Anemia
o Anorexia o Arthritis
o Constipation o Behavioral/neuropsychiatric changes
o Chronic diarrhea o Cerebellar ataxia
o Failure to thrive o Delayed puberty
o Irritable bowel syndrome o Dental enamel hypoplasia
o Lactose intolerance o Dermatitis herpetiformis
o Weight loss o Elevated transaminase values
o Epilepsy
o Infertility
o Osteopenia/osteoporosis
o Short stature

Copyright 2011 American Academy of Pediatrics


2012 PREP SA ON CD-ROM

Question 194
You are speaking with the mother of a newborn at 18 hours after birth. She is worried about the
pustules on her sons neck and upper chest. She has read about methicillin-resistant Staphylococcus
aureus on the Internet and is concerned that he has this infection. Upon physical examination, the alert
and active infant has multiple milky white pustules that are 2-mm in diameter on the neck and upper
chest; there is no warmth or erythema around the lesions. In addition, there are several faint brownish
macules, many of which have a rim of scale (Item Q194).
Of the following, the MOST likely diagnosis is
A. congenital candidiasis
B. erythema toxicum neonatorum
C. miliaria crystallina
D. staphylococcal falliculitis
E. transient neonatal pustular melanosis

Copyright 2012 American Academy of Pediatrics


2012 PREP SA ON CD-ROM

Question 194

(Reprinted with permission from Krowchuk DP, Mancini AJ, eds. Pediatric Dermatology. A Quick
Reference Guide. 2nd ed. Elk Grove Village, IL: American Academy of Pediatrics. 2011)
Macules, as described for the newborn in the vignette.

Copyright 2012 American Academy of Pediatrics


2012 PREP SA ON CD-ROM

Critique 194 Preferred Response: E


The infant described in the vignette has transient neonatal pustular melanosis (TNPM), which is
characterized by three types of lesions: superficial vesiculopustules, ruptured pustules with a ring of
surrounding scale, and hyperpigmented brown macules that remain at the site of the earlier pustules.
TNPM may be seen in any location at birth, although it is most commonly found on the forehead, chin,
neck, and trunk. It occurs in up to 5% of African-American infants and is uncommon in other racial
groups. Although infants may present with any combination of the lesions at birth, the pustules are the
initial phase and are easily unroofed by wiping and bathing, leaving the surrounding scale. The remaining
hyperpigmented macules may remain for 3 to 4 months.
The differential diagnosis of neonatal skin disorders characterized by vesiculopustules includes
erythema toxicum neonatorum, TNPM, miliaria crystallina, neonatal herpes simplex virus infection,
congenital candidiasis, infantile acropustulosis, and staphylococcal folliculitis (pustulosis). Erythema
toxicum neonatorum is seen in 50% of infants and generally is not present until 24 to 48 hours after birth.
The lesions are often discrete, appearing as erythematous macules with a central papule in any location
other than the palms and soles (Item C194A). The lesions of miliaria crystallina are characterized by
fragile, small vesicles filled with clear fluid that do not have surrounding erythema (Item C194B). The lack
of surrounding erythema helps to distinguish miliaria crystallina from neonatal herpes simplex virus
infection, which typically appears as clustered vesicles on an erythematous base. Congenital candidiasis
presents in the first days after birth as a diffuse erythematous papular and pustular rash that may crust
and desquamate (Item C194C). It is acquired in utero and can involve any location, including the palms
and soles. The vesiculopustular rash of infantile acropustulosis is specific to the hands and feet,
developing in the first postnatal months rather than the first postnatal days and recurring in a cyclic
pattern (Item C194D). Staphylococcal folliculitis (Item C194E) occurs most often between 7 and 12 days
of age, with a higher incidence in male infants. Pustules are defined as less than 10 mm in diameter and
may be localized or diffuse.
Analysis of the contents of the pustules can be helpful in the diagnosis and management of
vesiculopustular skin disorders (Item C194F). Methicillin-resistant Staphylococcus aureus infections are
increasing in otherwise healthy infants, with the severity of illness ranging from folliculitis to invasive
disease, including osteomyelitis, septic arthritis, bacteremia, and meningitis. Increased awareness is
necessary for early identification and treatment.

American Board of Pediatrics Content Specification(s):


Recognize the lesions of transient neonatal pustular melanosis
Know that a Gram stain will help distinguish between transient neonatal pustular melanosis and
staphylococcal pustules

Suggested Reading:
American Academy of Pediatrics. Skin disorders in neonates/infants. In: Krowchuk DP, Mancini AJ, eds.
nd
Pediatric Dermatology A Reference Guide. 2 ed. Elk Grove Village, lL: American Academy of Pediatrics;
2007:433-454

Fortunov RM, Kaplan SL. Methicillin-resistant Staphylococcus aureus in previously healthy neonates.
NeoReviews. 2008;9:e580-e584. DOI: 10.1542/neo.9-12-e580. Available at:
http://neoreviews.aappublications.org/cgi/content/full/9/12/e580

Fortunov RM, Hulten KG, Hammerman WA, Mason EO Jr, Kaplan SL. Evaluation and treatment of
community-acquired Staphylococcus aureus infections in term and late-preterm previously healthy
infants. Pediatrics. 2007;120:937-945. DOI: 10.1542/peds.2007-0956. Available at:
http://pediatrics.aappublications.org/cgi/content/full/120/5/937

Morelli JG. The skin: diseases of the neonate. In: Kliegman RM, Stanton BF, St. Geme JW III, Schor NF,
and Behrman RE, eds. Nelson Textbook of Pediatrics. 19th ed. Philadelphia, PA: Saunders Elsevier;
2011:2218-2220

Copyright 2012 American Academy of Pediatrics


2012 PREP SA ON CD-ROM

Critique 194

(Courtesy of P Sagerman)
Erythema toxicum is characterized by solitary papules or vesicles with surrounding erythema.

Copyright 2011 American Academy of Pediatrics


2012 PREP SA ON CD-ROM

Critique 194

(Reprinted with permission from Krowchuk DP, Mancini AJ, eds. Pediatric Dermatology. A Quick
Reference Guide. 2nd ed. Elk Grove Village, IL: American Academy of Pediatrics; 2011)
Fragile tiny vesicles without surrounding erythema are characteristic of miliaria crystallina.

Copyright 2011 American Academy of Pediatrics


2012 PREP SA ON CD-ROM

Critique 194

(Courtesy of D Krowchuk)
In congenital candidiasis, infants exhibit erythematous papules, pustules, and scaling.

Copyright 2011 American Academy of Pediatrics


2012 PREP SA ON CD-ROM

Critique 194

(Reprinted with permission from Krowchuk DP, Mancini AJ, eds. Pediatric Dermatology. A Quick
Reference Guide. 2nd ed. Elk Grove Village, IL: American Academy of Pediatrics; 2011)
Infantile acropustulosis is characterized by recurring crops of pruritic vesiculopustules that affect the
hands and feet.

Copyright 2011 American Academy of Pediatrics


2012 PREP SA ON CD-ROM

Critique 194

(Reprinted with permission from Krowchuk DP, Mancini AJ, eds. Pediatric Dermatology. A Quick
Reference Guide. 2nd ed. Elk Grove Village, IL: American Academy of Pediatrics; 2011)
Staphylococcal folliculitis appears as papules and pustules with surrounding erythema.

Copyright 2011 American Academy of Pediatrics


2012 PREP SA ON CD-ROM

Critique 194

Item C194F. Differentiation of Vesiculopustular Skin Disorders Via Analysis of Pustule Contents
Disorder Microscopy Culture
Erythema toxicum neonatorum Eosinophils, no organisms Negative
Transient neonatal pustular melanosis Neutrophils, no organisms Negative
Staphylococcal folliculitis Neutrophils, gram-positive cocci Staphylococcus aureus
Congenital candidiasis Potassium hydroxide preparation Candida sp
reveals pseudohyphae or spores
Neonatal herpes simplex virus Tzanck smear reveals Herpes simplex virus
infection multinucleated giant cells (may also use
polymerase chain
reaction testing)

Copyright 2011 American Academy of Pediatrics


2012 PREP SA ON CD-ROM

Question 195
A 14-year-old girl first developed redness across the bridge of her nose and on her cheeks about 3
months ago. Subsequently, the rash spread to involve more of her face as well as her hands. She now
reports increasing fatigue and some weakness; she has difficulty climbing stairs and has needed help
brushing her hair. She has had no fever or joint swelling. Physical examination of the appropriately
developed teen reveals normal vital signs; a heliotrope rash involving the eyelids, nasal bridge, and
cheeks (Item Q195A); and erythematous scaling papules on the extensor surfaces of her metacarpal,
phalangeal, and interphalangeal joints (Item Q195B). She has weakness and tenderness of the proximal
musculature and must employ the Gower maneuver to arise from sitting.
Of the following, the test that is MOST likely to contribute to the diagnosis is
A. cranial computed tomography scan

B. edrophonium (Tensilon ) test
C. electroneurography
D. magnetic resonance imaging of proximal muscles
E. skin biopsy

Copyright 2012 American Academy of Pediatrics


2012 PREP SA ON CD-ROM

Item 195

(Courtesy of D Krowchuk)
Facial eruption, as described for the girl in the vignette.

Copyright 2012 American Academy of Pediatrics


2012 PREP SA ON CD-ROM

Question 195

(Courtesy of D Krowchuk)
Hand eruption, as described for the girl in the vignette.

Copyright 2012 American Academy of Pediatrics


2012 PREP SA ON CD-ROM

Critique 195 Preferred Response: D


The girl described in the vignette has the typical facial rash of juvenile dermatomyositis (JDM): a
red-to-violet masklike rash involving the cheeks and eyelids and sometimes extending to the ears (ie,
heliotrope). She also exhibits Gottron papules, flat-topped erythematous hyperkeratotic papules usually
overlying metacarpophalangeal or interphalangeal joints (Item C195). JDM is the most common of the
inflammatory myopathies affecting children and adolescents, with an estimated incidence of
approximately 2 to 4 cases/million children per year. Diagnostic criteria developed in 1975 require the
presence of three or more criteria plus the characteristic rash for a definite diagnosis of JDM or two
criteria plus rash for a probable diagnosis. The criteria include: proximal muscle weakness (sparing facial
and eye muscles), elevated muscle enzyme values, myopathic electromyography (EMG), and
characteristic findings on muscle biopsy.
Recently, a greater understanding of the pathogenesis and the availability of newer technology
has changed the diagnostic approach, and efforts are underway to revise the diagnostic criteria. A survey
of rheumatologists throughout the world showed that only about 60% of practitioners used EMG or
muscle biopsy, opting instead for less invasive testing to confirm the diagnosis. Magnetic resonance
imaging of proximal muscles indicating inflammatory muscle changes was the next most useful diagnostic
test, according to rheumatologists. Additional tests, including searches for p155 antigen, myositis-specific
and -related antibodies, and von Willebrand antigen, may be used, but these factors are not consistently
found in each patient. Major histocompatability screening also finds the presence of HLA-DQA1*0501 in
more than 80% of affected patients, but its presence is not required for diagnosis.
The edrophonium test is used in the diagnosis of myasthenia gravis. In that condition, extraocular
muscle involvement is the most prominent finding and rash is not a typical component. Nerve conduction
studies might be useful in the evaluation of peripheral nerve disease, which manifests with greater distal
than proximal weakness. Cranial computed tomography scan might be useful in the diagnosis of centrally
caused conditions, but elevated muscle enzyme values described for this girl make myopathy a more
likely diagnosis. A skin biopsy might be performed if the rash is atypical, but rheumatologists in a recent
survey rated skin biopsy as not usually helpful or necessary for the diagnosis of JDM.

American Board of Pediatrics Content Specification(s):


Recognize the classic picture of dermatomyositis: pain, weakness, rash
Know the appropriate laboratory evaluation of dermatomyositis

Suggested Reading:
Compeyrot-Lacassagne S, Feldman B. Inflammatory myopathies in children. Rheum Dis Clin North Am.
2007;33:525-553. DOI: 10.1016/j.rdc.2007.07.002. Abstract available at:
http://www.ncbi.nlm.nih.gov/pubmed/17936176

Feldman BM, Rider LG, Reed AM, Pachman LM. Juvenile dermatomyositis and other idiopathic
inflammatory myopathies of childhood. Lancet. 2008;371:2201-2212. DOI: 10.1016/S0140-
6736(08)60955-1http://www.thelancet.com/popup?fileName=cite-using-doi. Abstract available at:
http://www.ncbi.nlm.nih.gov/pubmed/18586175

McCann LF, Juggins AD, Maillard SM, et al; Juvenile Dermatomyositis Research Group. The Juvenile
Dermatomyositis National Registry and Repository (UK and Ireland): clinical characteristics of children
recruited within the first 5 years. Rheumatology. 2006;45:1255-1260. DOI: 10.1093/rheumatology/kel099.
Available at: http://rheumatology.oxfordjournals.org/content/45/10/1255.long

Tzaribachev N, Well C, Schedel J, Horger M. Whole-body MRI: a helpful diagnostic tool for juvenile
dermatomyositis case report and review of the literature. Rheumatol Int. 2009;29:1511-1514. DOI:
10.1007/s00296-009-0890-y. Abstract available at: http://www.ncbi.nlm.nih.gov/pubmed/19301008

Copyright 2012 American Academy of Pediatrics


2012 PREP SA ON CD-ROM

Critique 195

(Courtesy of D Krowchuk)
Gottron papules: erythematous scaling papules overlying the knuckles (arrows) in a patient who has
dermatomyositis.

Copyright 2011 American Academy of Pediatrics


2012 PREP SA ON CD-ROM

Question 196
The mother of an 11-year-old girl brings in her daughter because she noted white-to-pale yellow
vaginal discharge on the girls cotton underwear when doing the laundry. The child denies discomfort or
itching. The mother notes that the girl likes to take bubble baths. Results of a careful interview raise no
concerns about sexual abuse or activity. Physical examination reveals normal vital signs with no fever
and normal head, eyes, ears, nose, and throat findings without pharyngitis or conjunctivitis. She has
Sexual Maturity Rating 3 breast development and normal genitourinary examination findings, with normal
hymenal tissue, pink vaginal mucosa, and scant white discharge. No tenderness is elicited on abdominal
examination.
Of the following, the MOST likely cause of this girls discharge is
A. chlamydial infection
B. nonspecific vulvovaginitis
C. physiologic leukorrhea
D. streptococcal vaginitis
E. vaginal candidiasis

Copyright 2012 American Academy of Pediatrics


2012 PREP SA ON CD-ROM

Critique 196 Preferred Response: C


The girl described in the vignette has a painless, scant white vaginal discharge. Because she has
a Sexual Maturity Rating of 3, it is most likely that the discharge represents the estrogenic effect on the
vaginal mucosa as a precursor of menarche, usually occurring 4 to 6 months after the onset of such
physiologic leukorrhea.
The differential diagnosis of vaginal discharge associated with complaints of discomfort or foul
smell include foreign body (as trivial as retained toilet paper or as significant as foreign objects
deliberately inserted), sexual abuse and sexually transmitted infection (eg, chlamydial infection),
pinworms, vaginal candidiasis, and group A beta-hemolytic streptococcal infection. In addition, especially
in younger girls, structural abnormalities such as ectopic ureter, urethral prolapse, rectovaginal fistula,
and neoplasms (including sarcomas) must be considered as rare causes.
The most common cause of vulvar itching or thin vaginal discharge, especially in the prepubertal
girl, is nonspecific vulvovaginitis, which is usually caused by irritation of this sensitive area by perfumes,
deodorant soaps, shampoos, or bubble baths. The condition also may be caused by inadequate genital
hygiene (eg, inadequate wiping or wiping from back to front) or overly aggressive genital hygiene after
voiding or stooling, which causes excessive friction.
Streptococcal vaginitis is usually accompanied by pain, severe erythema of the vulva and often
the perianal area, and constitutional signs and symptoms (eg, fever, nausea, and malaise). It may occur
with group A streptococcal pharyngitis and is treated with penicillins or cephalosporins.
Sexual abuse or other sexual contact must be considered, even in the absence of signs of
penetration, in the girl who presents with vulvar and vaginal complaints, especially discharge, regardless
of age. Sexually transmitted infections may be spread not only by genital-to-genital contact but also by
oral-genital contact. Gonococcal infections generally present with purulent discharge, but signs of
chlamydial infection may be minimal. Trichomonas vaginalis infection and bacterial vaginosis are
uncommon in prepubertal children.

American Board of Pediatrics Content Specification(s):


Evaluate and recognize the cause of a vaginal discharge in a preadolescent girl

Suggested Reading:
American Academy of Pediatrics. Gonococcal Infections. In: Pickering LK, ed. Red Book: 2009 Report of
the Committee on Infectious Diseases. 28th ed. Elk Grove Village, IL: American Academy of Pediatrics;
2009:305-313. Available at: http://aapredbook.aappublications.org/cgi/content/full/2009/1/3.45 Accessed
February 8, 2011

American Academy of Pediatrics. Group A Streptococcal Infections. In: Pickering LK, ed. Red Book: 2009
Report of the Committee on Infectious Diseases. 28th ed. Elk Grove Village, IL: American Academy of
Pediatrics; 2009:616-628. Available at:
http://aapredbook.aappublications.org/cgi/content/full/2009/1/3.125. Accessed February 8, 2011

Kellogg N and the Committee on Child Abuse and Neglect. The evaluation of sexual abuse in children.
Pediatrics. 2005;116:506-512. DOI: 10.1542/peds.2005-1336. Available at:
http://pediatrics.aappublications.org/content/116/2/506.full.pdf

Matytsina LA, Greydanus DE, Gurkin YA. Vaginal microbiocoenosis and cytology of prepubertal and
adolescent girls: their role in health and disease. World J Pediatr. 2010;6:32. Available at:
http://www.wjpch.com/article.asp?article_id=360&article_id1=360&type_i=2#Vaginal microbiocoenosis
and cytology of prepubertal and adolescent girls: their role in health and disease_1

Sugar NF, Graham EA. Common gynecologic problems in prepubertal girls. Pediatr Rev. 2006;27:213-
223. DOI: 10.1542/pir.27-6-213. Available at:
http://pedsinreview.aappublications.org/cgi/content/full/27/6/213

Copyright 2012 American Academy of Pediatrics


2012 PREP SA ON CD-ROM

Question 197
You are seeing a 10-year-old girl for follow-up evaluation of behavior problems and recurrent
abdominal pain. Her mother reports that she continues to be irritable most of the time, even with friends,
she has problems falling asleep, and she talks back when pressed to complete tasks such as getting
dressed in the morning. She adds that symptoms have been ongoing for a couple of months, stating last
summer she wasnt like this. The girls teacher reports that she has been quiet and withdrawn in class.
The girls abdominal pain is daily and periumbilical in location. She admits to decreased appetite, denies
nausea, and reports one soft bowel movement per day. Physical examination, including neurologic
evaluation, shows no findings of note other than periumbilical tenderness to deep palpation. The girl
exhibits no guarding, masses, or organomegaly.
Of the following, the MOST appropriate next diagnostic step is to
A. collect a stool sample for bacterial culture
B. complete a Child Depression Inventory
C. complete a Connors Parent Rating Scale short form
D. measure her ceruloplasmin
E. obtain abdominal radiography

Copyright 2012 American Academy of Pediatrics


2012 PREP SA ON CD-ROM

Critique 197 Preferred Response: B


The constellation of symptoms described for the girl in the vignette best represents the
presentation of depression in a school-age child. Unlike in adults, who predominantly present with
depressed mood, children who have depression often exhibit irritability, anxiety, and psychosomatic
symptoms. Other symptoms of depression can include weight gain or loss, fatigue, sleep disturbance,
and difficulty concentrating. The most appropriate next step is to administer a screening tool to better
characterize her symptoms. The Child Depression Inventory is one of several validated rating scales
available for assessing childhood depression.
The Connors Parent Rating Scale short form is used to aid in the diagnosis of attention-
deficit/hyperactivity disorder (ADHD). The short form of this scale focuses only on ADHD symptoms. This
girls symptoms have been present for only 2 months, and patients who have ADHD typically display
chronic symptoms starting at a younger age. Furthermore, although ADHD can present with a chief
complaint of behavior problems, it is more often characterized by impulsivity and inattention. Abdominal
radiography can help characterize constipation, which can contribute to abdominal pain, but this child
reports one soft bowel movement per day, making constipation unlikely. Bacterial culture of stool is
appropriate for the child who has acute or chronic diarrhea, which this girl does not report. Assessment of
ceruloplasmin to detect Wilson disease may be appropriate if the girl had neurologic signs,
hepatomegaly, or other stigmata of liver disease.

AAP Mental Health Competency:


Recognize that childhood depression can present with predominantly irritable mood rather than
predominantly depressed mood

Suggested Reading:
American Academy of Pediatrics Task Force on Mental Health. Depression Cluster Guidance. Addressing
Mental Health Concerns in Primary Care: A Clinician's Toolkit [CD-ROM]. Elk Grove Village, IL: American
Academy of Pediatrics; 2010

McCauley GR, Gudmundsen GR, Rockhill C, Banh M. Child and adolescent depressive disorders. In:
cheng K, Myers KM, eds. Child and Adolescent Psychiatry: The Essentials. 2nd ed. Philadelphia, PA:
Lippincott Williams & Wilkins, a Wolters Kluwer business; 2011:177-196

Prager LM. Depression and suicide in children and adolescents. Pediatr Rev. 2009;30:199-205. DOI:
10.1542/pir.30-6-199. Available at: http://pedsinreview.aappublications.org/cgi/content/full/30/6/199

Copyright 2012 American Academy of Pediatrics


2012 PREP SA ON CD-ROM

Question 198
A 5-year-old boy presents to the emergency department with a 5-day history of fever, bilateral
conjunctivitis, cracked and red lips, a strawberry tongue, and swelling of the dorsal surfaces of his hands
and feet. On physical examination, you identify unilateral, nontender cervical lymphadenopathy.
Laboratory assessment yields an elevated erythrocyte sedimentation rate and C-reactive protein value
with sterile pyuria.
Of the following, the MOST appropriate initial therapy for this child is
A. dependent upon the presence or absence of coronary artery involvement
B. high-dose aspirin alone if coronary arteritis is absent
C. immune globulin intravenous and high-dose aspirin irrespective of echocardiographic findings
D. immune globulin intravenous and low-dose aspirin if echocardiography shows the absence of
coronary arteritis
E. immune globulin intravenous if echocardiography identifies dilated coronary arteries

Copyright 2012 American Academy of Pediatrics


2012 PREP SA ON CD-ROM

Critique 198 Preferred Response: C


The boy described in the vignette has clinical manifestations and laboratory findings consistent
with Kawasaki disease (KD), also known as mucocutaneous lymph node syndrome. This autoimmune
disease manifests as a systemic, necrotizing, medium-sized vessel vasculitis that is seen primarily in
children younger than 5 years of age. It affects many organ systems, including the blood vessels, skin,
mucous membranes, and lymph nodes, but its most serious effect is on the heart, where it can cause
severe coronary artery aneurysms in untreated children. Without treatment, mortality may approach 1%,
usually within 6 weeks of onset. With treatment, the mortality rate is less than 0.01% in the United States.
Although its cause is uncertain, there is often a pre-existing viral infection, which may play a role in its
pathogenesis. The conjunctival and oral mucosae, along with the skin, become erythematous. Edema is
often seen in the hands and feet, and the cervical lymph nodes are frequently enlarged but not tender.
Also, a remittent fever of at least 40.0C is characteristic of the acute phase of the disease. In untreated
children, the febrile period lasts approximately 10 days but may range from 5 to 25 days. Specific criteria
for the diagnosis have been established (Item C198).
Immune globulin intravenous (IGIV) is the standard treatment for KD and is administered in high
doses (2 g/day via a single continuous infusion over 12 hours), with marked improvement usually noted
within 24 hours. If the fever does not respond, a second dose is administered. In rare cases, additional
anti-inflammatory agents may need to be considered. In terms of preventing coronary artery aneurysm,
IGIV monotherapy is most useful within the first 7 to 10 days of fever onset. The incidence of coronary
aneurysms in the absence of therapy is in the range of 20%. This compares with an incidence of
approximately 3% to 5% when IGIV is administered in the acute phase of the disease process.
Salicylate therapy, particularly aspirin, remains an important part of the treatment but alone is not
as effective as IGIV. Some question whether aspirin therapy confers additional benefit above that of IGIV
alone in the control of coronary aneurysms among those who respond to IGIV. Aspirin therapy is started
at high doses (typically 80 to 100 mg/kg per day) until the fever subsides and is continued as an
antiplatelet agent at a low dose (3 to 5 mg/kg per day) for 6 to 8 weeks (or longer if coronary dilation
persists). Because children who have KD will be taking aspirin for up to several months, vaccination
against varicella and influenza is required because these infections increase the risk of aspirin-induced
Reye syndrome.
Therapy for KD is not dependent upon the identification of coronary artery involvement because
coronary dilation may be seen only after the therapeutic window has passed. In addition, aspirin has
never been shown to be an effective single therapeutic agent in KD, irrespective of the presence or
absence of coronary dilation. Although some question the utility of aspirin therapy to augment the
prevention of coronary aneurysms, the current American Academy of Pediatrics as well as American
Heart Association guidelines call for high-dose aspirin before fever resolution and low-dose therapy for 6
to 8 weeks thereafter. As noted previously, the use of IGIV is indicated, irrespective of the presence or
absence of coronary dilation.

American Board of Pediatrics Content Specification(s):


Recognize the value of high-dose intravenous immune globulin and aspirin therapy in the treatment of
Kawasaki disease

Suggested Reading:
Haftel HM. Rheumatic diseases of childhood: Kawasaki disease. In: Marcdante K, Kliegman R, Behrman
RE, eds. Nelson Essentials of Pediatrics. 6th ed. Philadelphia, PA: Saunders Elsevier; 2011:343-344

Newburger JW, Sleeper LA, McCrindle BW, et al. Randomized trial of pulsed corticosteroid therapy for
primary treatment of Kawasaki disease. N Engl J Med. 2007;356:663-675. Available at:
http://www.nejm.org/doi/full/10.1056/NEJMoa061235#t=article

Newburger JW, Takahashi M, Gerber MA, et al. Diagnosis, treatment, and long-term management of
Kawasaki disease: a statement for health professionals from the Committee on Rheumatic Fever,
Endocarditis, and Kawasaki Disease, Council on Cardiovascular Disease in the Young, American Heart

Copyright 2012 American Academy of Pediatrics


2012 PREP SA ON CD-ROM

Association. Pediatrics. 2004;114:1708-1733;. DOI: 10.1542/peds.2004-2182. Available at:


http://pediatrics.aappublications.org/cgi/content/full/114/6/1708

Tse SM, Silverman ED, McCrindle BW, Yeung RS. Early treatment with intravenous immunoglobulin in
patients with Kawasaki disease. J Pediatr. 2002;140:450455. DOI: 10.1067/mpd.2002.122469. Abstract
available at: http://www.ncbi.nlm.nih.gov/pubmed/12006960

Copyright 2012 American Academy of Pediatrics


2012 PREP SA ON CD-ROM

Critique 198

!"#$%&'()*%Criteria for Diagnosis of Kawasaki Disease


+#,#-%./%!%0%12345%16-2"7.8%244.972"#1%:7";%2"%<#24"%/.6-=%./%";#%/.<<.:78>%/7,#%9;28>#4
! Bilateral nonsuppurative conjunctivitis
! One of more changes of the mucous membranes of the upper respiratory tract, including
pharyngeal injection, dry fissured lips, injected lips, and "strawberry" tongue
! One or more changes of the extremities, including peripheral erythema, peripheral edema,
periungual desquamation, and generalized desquamation
! Polymorphous rash, primarily truncal
! Cervical lymphadenopathy >1.5 cm in diameter
Disease cannot be explained by some other known disease process
* A diagnosis of Kawasaki disease can be made if fever and only three changes are present in conjunction
with coronary artery disease documented by two-dimensional echocardiography or coronary angiography.

Reprinted with permission from Haftel HM. Rheumatic diseases of childhood: Kawasaki disease. In:
Marcdante K, Kliegman R, Behrman RE, eds. Nelson Essentials of Pediatrics. 6th ed. Philadelphia, PA:
Saunders Elsevier; 2011:343-344

Copyright 2011 American Academy of Pediatrics


2012 PREP SA ON CD-ROM

Question 199
A 9-year-old boy has had 3 days of progressive difficulty walking. This morning he had difficulty
combing his hair and began complaining of back pain. He has had no urinary or fecal incontinence. In the
emergency department, he is afebrile and has normal vital signs. Physical examination yields normal
results, with no pain or swelling of his joints. His neurologic examination shows normal mentation,
normally reactive pupils with full extraocular movements, and no nystagmus. He has slight weakness of
eye closure. Upper limbs have normal bulk and tone but mild weakness, with the shoulders worse than
the elbows or wrists. He cannot arise unassisted from the floor and has weakness at the hips, knees, and
ankles. He is areflexic. Concerned about his subacute generalized weakness, you obtain a forced vital
capacity, which is greater than 2 L.
Of the following, the test that is MOST likely to clarify the diagnosis is
A. cerebrospinal fluid testing for protein concentrations

B. edrophonium (Tensilon ) testing
C. electromyography of the upper limbs
D. fecal testing for botulinum spores
E. magnetic resonance imaging of the spine

Copyright 2012 American Academy of Pediatrics


2012 PREP SA ON CD-ROM

Critique 199 Preferred Response: A


The boy described in the vignette is presenting with subacute, rapidly progressive weakness
involving his legs initially and now his arms. Because a number of causes of generalized weakness can
progress rapidly to respiratory failure and death, it is important to verify respiratory sufficiency with either
forced vital capacity or negative inspiratory force. Respiratory failure does not appear imminent for this
boy because his forced vital capacity exceeds 2 L.
Back pain is characteristic of pathology in nerve roots. Based on this boys presenting symptoms,
the anatomic source is unlikely to be the brain (bilateral, but with normal mentation) or spinal cord (normal
bowel and bladder function, no sensory level). The sign of areflexia in this setting is consistent with a
diagnosis of Guillain-Barr syndrome, an acute inflammatory demyelinating
polyradiculopathy/polyneuropathy that can cause acute respiratory failure and autonomic dysregulation,
leading to death. Accordingly, diagnosis, appropriate supportive care in the intensive care unit, and urgent
treatment are vital. Obtaining a lumbar puncture and identifying elevated cerebrospinal fluid protein with a
normal white blood cell count confirms the diagnosis. However, sometimes the characteristic protein
elevation is delayed several days.

Edrophonium (Tensilon ) is an acetylcholinesterase inhibitor that increases acetylcholine at the
neuromuscular junction. Improved strength after edrophonium is characteristic of myasthenia gravis,
which presents with more variable, fluctuating, and fatiguing weakness. Fecal testing for botulinum spores
should be obtained in cases of generalized weakness in infancy. After infancy, the mode of transmission
is ingestion of the toxin, not the spores. This boys clinical presentation does not support spinal cord
pathology and, therefore, spine magnetic resonance imaging is not indicated.
Nerve conduction studies (in the distribution of the observed weakness) and electromyography
(of any affected apparently weak muscle) may be helpful, particularly once the cerebrospinal fluid protein
findings are known. Nerve conduction studies can clarify if nerve involvement is primarily demyelinating
(slower velocities, conduction block, dispersion) or axonal (smaller amplitudes). Electromyography is the
least comfortable part of this procedure because it involves repeatedly inserting needles into muscles.
This may provide evidence of denervation and reinnervation.

American Board of Pediatrics Content Specification(s):


Know the expected results of laboratory procedures such as examination of the cerebrospinal fluid,
nerve conduction studies, and electromyography in Guillain-Barre syndrome

Suggested Reading:
Durand MC, Porcher R, Orlikowski D, et al. Clinical and electrophysiological predictors of respiratory
failure in Guillain-Barr syndrome: a prospective study. Lancet Neurol. 2006;5:1021-1028. DOI:
10.1016/S1474-4422(06)70603-2. Abstract available at: http://www.ncbi.nlm.nih.gov/pubmed/17110282
http://www.thelancet.com/popup?fileName=cite-using-doi
Hughes RA, Swan AV, Raphal JC, Annane D, van Koningsveld R, van Doorn PA. Immunotherapy for
Guillain-Barr syndrome: a systematic review. Brain. 2007;130:2245-2257. DOI: 10.1093/brain/awm004.
Abstract available at: http://brain.oxfordjournals.org/content/130/9/2245.long

Ryan MM. Guillain-Barr syndrome in childhood. J Paediatr Child Health. 2005;41:237-241. DOI:
10.1111/j.1440-1754.2005.00602.x. Abstract available at: http://www.ncbi.nlm.nih.gov/pubmed/15953319

Sarnat HB. Neuromuscular disorders: Guillain-Barr syndrome. In: Kliegman RM, Stanton BF, St. Geme
JW III, Schor NF, and Behrman RE, eds. Nelson Textbook of Pediatrics. 19th ed. Philadelphia, PA:
Saunders Elsevier; 2011:2143-2146

Tasdemir HA, Dilber C, Kanber Y, Uysal S. Intravenous immunoglobulin for Guillain-Barr syndrome: how
effective? J Child Neurol. 2006;21:972-974. Abstract available at:
http://www.ncbi.nlm.nih.gov/pubmed/17092465

Copyright 2012 American Academy of Pediatrics


2012 PREP SA ON CD-ROM

Willoughby RE Jr. Cerebellar ataxia, transverse myelitis and myelopathy, GuillainBarr syndrome,
neuritis, and neuropathy. In: Long SS, Pickering LK, Prober CG, eds. Principles and Practice of Pediatric
Infectious Diseases. 3rd ed. Philadelphia, PA: Churchill Livingstone Elsevier, 2008:318-323

Copyright 2012 American Academy of Pediatrics


2012 PREP SA ON CD-ROM

Question 200
A 2-year-old girl presents to the emergency department with what is eventually diagnosed as a
hypocalcemic seizure. Administration of oral calcitriol improves her serum calcium concentration.
Laboratory testing documents an extremely low parathyroid hormone value. The girl has a history of a
cleft palate that was repaired at 6 months of age and an incidental finding of a right-sided aortic arch
noted on routine chest radiograph. She is otherwise healthy and doing well developmentally.
Of the following, the MOST likely test to confirm her underlying diagnosis is
A. echocardiography
B. fluorescence in situ hybridization for a 22q deletion
C. magnetic resonance imaging of the brain
D. routine karyotype
E. ultrasonography of the neck

Copyright 2012 American Academy of Pediatrics


2012 PREP SA ON CD-ROM

Critique 200 Preferred Response: B


Hypoparathyroidism resulting in a hypocalcemic seizure combined with a history of a cleft palate
and a right-sided aortic arch is consistent with a 22q11.2 microdeletion (also known as DiGeorge,
Shprintzen, or velocardiofacial syndrome), which occurs in approximately 1 in 6,000 livebirths. Although a
routine karyotype sometimes may uncover a chromosome 22q deletion in this cytogenetic region, many
deletions are too small to be detected on routine karyotype analysis. Instead, a fluorescence in situ
(FISH) study using probes for this commonly deleted region is used to diagnose this condition (Item
C200). Hypoparathyroidism can occur as an isolated abnormality, but the finding of significant
hypocalcemia in any infant, child, or adolescent due to hypoparathyroidism should prompt an
investigation for this common microdeletion syndrome. Echocardiography might uncover defects of the
heart or great vessels, including tetralogy of Fallot, interrupted aortic arch, a ventricular septal defect,
aberrant origin of the subclavian artery, or truncus arteriosus, which are the most common cardiac
abnormalities identified with 22q11.2 deletion, but it would not reveal the diagnosis. A magnetic
resonance imaging (MRI) finding consistent with a central nervous system defect is found in 1 in 9
affected individuals, but MRI of the brain alone would not confirm or refute a diagnosis of a 22q11.2
deletion. Ultrasonography of the neck would not be especially helpful because even normal-sized
parathyroid glands may be difficult to identify with this imaging modality, and the diagnosis of
hypoparathyroidism can be made more reliably by measuring calcium, phosphorus, and parathyroid
hormone.
Although considered a normal anatomic variant, a right-sided aortic arch is seen in more than
30% of children who have 22q11.2 deletions; more than 50% have another identified vascular or cardiac
defect. More than 10% of affected individuals have a cleft of the posterior soft palate, and close to 15%
have velopharyngeal insufficiency that can lead to nasal regurgitation, hypernasal speech, and difficulties
with speech intelligibility. Although some investigators identified an absent or hypoplastic thymus (which
would result in immunodeficiency and possibly serious, life-threatening infections) in more than 20% of
individuals who have 22q11.2 deletions, this may represent an ascertainment bias because patients who
have normal immune function may not be diagnosed until later in life or completely escape detection.
Nervous system defects occur in more than 10% of affected patients and include spina bifida, brainstem
anomalies, communicating hydrocephalus, and heterotopias. Renal anomalies such as hydronephrosis,
renal hypoplasia, or renal cysts have been identified in more than 10% of patients.
Much less well delineated, but certainly of concern, are the cognitive findings associated with a
22q11.2 deletion. Although up to 10% of affected children inherit the deletion from a parent who may or
may not have any clinical features, many children and young adults who have this deletion have
significant learning disabilities or cognitive deficits. Autistic spectrum disorders with or without intellectual
disabilities are seen with increased frequency in this population. Psychiatric disorders, including
schizophrenia and other forms of psychosis, may be identified in late childhood or early adolescence, with
increasingly difficult behavioral problems or hallucinations as common presenting signs.

American Board of Pediatrics Content Specification(s):


Know that DiGeorge syndrome (22q-) can be a cause of hypoparathyroidism

Suggested Reading:
Bobey-Wright NAM, Tcheurekdjian H, Wara D, Lewis DB. Immunologic aspects of DiGeorge syndrome.
NeoReviews. 2005; 6: 471-478. Avaiable at: http://neoreviews.aappublications.org/cgi/reprint/6/10/e471

Botto LD, May K, Fernhoff PM, et al. A population-based study of the 22q11.2 deletion: phenotype,
incidence, and contribution to major birth defects in the population. Pediatrics. 2003;112:101-107.
Available at: http://pediatrics.aappublications.org/cgi/content/full/112/1/101

McDonald-McGinn DM, Driscoll DA, Emanuel BS, et al. Detection of a 22q11.2 deletion in cardiac
patients suggests a risk for velopharyngeal incompetence. Pediatrics. 1997;99:e9. DOI:
10.1542/peds.99.5.e9. Available at: http://pediatrics.aappublications.org/cgi/content/full/99/5/e9

Copyright 2012 American Academy of Pediatrics


2012 PREP SA ON CD-ROM

Critique 200

(Courtesy of M Pettenati)
Metaphase fluorescence in situ hybridization in DiGeorge syndrome: The green signal is the centromere
of chromosome 22 and the red signal is the DiGeorge syndrome region. The chromosome lacking the red
signal is abnormal, with a deletion of the DiGeorge region (ie, 22q-).

Copyright 2011 American Academy of Pediatrics


2012 PREP SA ON CD-ROM

Question 201
A 14-year-old girl, who has experienced irregular bleeding since menarche at age 11 years,
presents with painless menstrual bleeding of 14 days duration. She is using 8 to 10 super-pads per day.
She says she was told that her period could be irregular in the first few years, but she is feeling tired and
is upset with the number of days of bleeding. The only finding of note on physical examination is mild
pallor. Her heart rate is 82 beats/min and blood pressure is 120/80 mm Hg, with no postural changes.
Laboratory tests show a hemoglobin of 9.4 g/dL (94 g/L) with a normal platelet count, prothrombin time,
partial thromboplastin time, and von Willebrand panel.
Of the following, the MOST appropriate treatment for this girl is
A. a course of iron therapy and an iron-rich diet
B. a daily dose of oral progesterone pills
C. combined oral contraceptive pills
D. gynecologic referral for a dilatation and curettage
E. tracking with a menstrual calendar and follow-up appointment in 3 months

Copyright 2012 American Academy of Pediatrics


2012 PREP SA ON CD-ROM

Critique 201 Preferred Response: C


Painless profuse bleeding of endometrial origin from physiologic anovulation is referred to as
dysfunctional uterine bleeding (DUB). There is no systemic or local pathology, as determined by the
history and physical examination, and the condition should resolve in 2 years postmenarche, when most
menstrual cycles become ovulatory. The girl described in the vignette has concerns about heavy and
prolonged menses. Normal flow can last up to 7 days, with the use of up to seven menstrual pads in a
day. The likelihood that she has a bleeding disorder is low because she is now 3 years postmenarche
and her hemoglobin value is only mildly decreased. In addition, the findings on screening for bleeding
disorders are normal. Most bleeding disorders are characterized by heavy bleeding in the first or second
menstrual cycles, with the quantity of blood loss creating significant anemia with much lower hemoglobin
values. Because there is no ovulation or production of progesterone with DUB, the endometrial lining
builds up from unopposed action of estrogen on the tissue, and when the lining is subsequently shed, the
bleeding can be prolonged and heavier than usual.
Therapy for DUB depends on the severity of bleeding. If the hemoglobin is in the normal range,
the patient should keep a careful menstrual record, have frequent follow-up evaluations, and be advised
to eat an appropriate iron-rich diet or take iron supplements. Because the hemoglobin for the girl in the
vignette has dropped below 10 mg/dL (100 g/L), she has persistent bleeding and is mildly symptomatic,
further bleeding should be prevented, which can be accomplished with combined hormonal contraception
to regulate the cycles and make the menstrual flow briefer and lighter. Consideration should be given to
hospitalizing her to ascertain that the bleeding is controlled and the hemoglobin concentration is
stabilized. Hospitalization should be considered if the patient is anemic and unable to tolerate or comply
with outpatient therapy. Hospitalization is indicated if the patient is hemodynamically unstable, is
symptomatic (eg, syncope or lightheadedness with change in posture), or has a hemoglobin value low
enough to require a transfusion. If there is any reason to not use estrogen, progesterone-only pills may be
tried. Surgical intervention is almost never needed for DUB. In fact, if the patient has bled down to the
basal layer of the endometrium, curettage may cause permanent amenorrhea and Asherman syndrome
(scarring within the uterine cavity). If DUB does not resolve in 2 years, the patient should be evaluated for
possible other conditions, such as polycystic ovarian syndrome.

American Board of Pediatrics Content Specification(s):


Know the preferred treatment of dysfunctional uterine bleeding in adolescence and that surgical
intervention is rarely necessary

Suggested Reading:
American Academy of Pediatrics, Committee on Adolescence, American College of Obstetricians and
Gynecologists, and Committee on Adolescent Health Care. Menstruation in girls and adolescents: using
the menstrual period as a vital sign. Pediatrics. 2006;118:2245-2250. DOI: 10.1542/peds.2006-2481.
Available at: http://pediatrics.aappublications.org/cgi/content/full/118/5/2245

Emans JS. Dysfunctional uterine bleeding. In: Emans SJH, Laufer MR, Goldstein DP, eds. Pediatric and
Adolescent Gynecology. 5th ed. Philadelphia, PA: Lippincott Williams & Wilkins, a Wolters Kluwer
business; 2005:270-286

Gray SH, Emans SJ. Abnormal vaginal bleeding in adolescents. Pediatr Rev. 2007;28:175 - 182. DOI:
10.1542/pir.28-5-175. Available at: http://pedsinreview.aappublications.org/cgi/content/full/28/5/175

Gupta N, Corrado S, Goldstein M. Hormonal contraception for the adolescent. Pediatr Rev. 2008;29:386
397. DOI: 10.1542/pir.29-11-386. Available at:
http://pedsinreview.aappublications.org/cgi/content/full/29/11/386

Mitan LAP, Slap GB. Dysfunctional uterine bleeding. In: Neinstein LS, Gordon CM, Katzman DK, Ro9sen
DS, Woods ER, eds. Adolescent Health Care: A Practical Guide. 5th ed. Philadelphia, PA: Lippincott
Williams & Wilkins, a Wolters Kluwer business; 2008:687-690

Copyright 2012 American Academy of Pediatrics


2012 PREP SA ON CD-ROM

Question 202
You are called to evaluate a 2-year-old girl in the emergency department who has septic shock.
She has received 80 mL/kg of normal saline since her arrival 45 minutes ago as well as appropriate
antibiotics. On physical examination, her heart rate is 140 beats/min, respiratory rate is 30 breaths/min,
and blood pressure is 65/40 mm Hg. She appears lethargic but arouses with vigorous stimulation. Her
extremities are cool, it is difficult to feel peripheral pulses, and her capillary refill time is 5 seconds.
Of the following, the MOST appropriate next step is initiation of an infusion of
A. dobutamine
B. dopamine
C. milrinone
D. norepinephrine
E. vasopressin

Copyright 2012 American Academy of Pediatrics


2012 PREP SA ON CD-ROM

Critique 202 Preferred Response: B


Septic shock, the presence of sepsis and cardiovascular organ dysfunction, as described for the
girl in the vignette, is a medical emergency that requires prompt recognition and treatment. One of the
most important factors in lowering the mortality associated with septic shock is early and aggressive fluid
resuscitation, defined as isotonic fluid boluses of 20 mL/kg titrated according to clinical assessment of
adequacy of cardiac output, such as heart rate, urine output, and level of consciousness. Often, 60 mL/kg
is needed for the initial resuscitation of a child who has septic shock. The choice of fluid (crystalloid
versus colloid) has been the subject of much debate, with little evidence to support the superiority of one
over the other.
For those who have fluid-refractory shock (evidence of persistent shock despite appropriate fluid
resuscitation), such as this girl, the addition of vasoactive agents is required. Dopamine or epinephrine is
considered first-line therapy for cold shock (increased heart rate, normal-to-low blood pressure,
decreased pulses), and norepinephrine is first-line therapy for warm shock (increased heart rate,
decreased blood pressure, and bounding pulses). Vasopressin has been used in cases of refractory
warm shock, but a recent multicenter controlled study did not demonstrate any benefit with the addition of
this agent to standard therapy, and, in fact, a trend toward increased mortality was seen that did not
reach statistical significance. Dobutamine and milrinone are indicated for patients experiencing
cardiogenic shock.

American Board of Pediatrics Content Specification(s):


Know the guidelines for the initial therapy of hypovolemic or septic shock

Suggested Reading:
Brierly J, Carcillo JA, Choong J, et al. Clinical practice parameters for hemodynamic support of pediatric
and neonatal septic shock: 2007 update from the American College of Critical Care Medicine. Crit Care
Med. 2009;37:666-688. DOI: 10.1097/CCM.0b013e31819323c6. Abstract available at:
http://www.ncbi.nlm.nih.gov/pubmed/19325359

Choong K, Bohn D, Fraser DD, et al; Canadian Critical Care Trials Group. Vasopressin in pediatric
vasodilatory shock: a multicenter randomized controlled trial. Am J Resp Crit Care Med. 2009;80:632
639. DOI: 10.1164/rccm.200902-0221OC. Available at:
http://ajrccm.atsjournals.org/cgi/content/full/180/7/632

McKiernan MA, Lieberman SA. Circulatory shock in children: an overview. Pediatr Rev. 2005;26:451-460.
DOI: 10.1542/pir.26-12-451. Available at:
http://pedsinreview.aappublications.org/cgi/content/full/26/12/451

Copyright 2012 American Academy of Pediatrics


2012 PREP SA ON CD-ROM

Question 203
During the annual health supervision visit for an 11-year-old boy, his mother advises you that
over the last year he has had considerable pubertal development, with rapid linear growth and both
testicular and penile enlargement. On physical examination, you note that his height is above the 95th
percentile for his age and confirm that his testicular volume is 12 mL (3.5 cm in length) bilaterally. A bone
age radiograph demonstrates skeletal maturity of 14 years.
Of the following, his upper-to-lower segment ratio would be expected to be CLOSEST to
A. 0.9
B. 1.1
C. 1.3
D. 1.5
E. 1.7

Copyright 2012 American Academy of Pediatrics


2012 PREP SA ON CD-ROM

Critique 203 Preferred Response: A


The upper-to-lower (U:L) segment ratio is an important part of the physical examination that can
be used to detect subtle abnormalities of growth and development. The distance from the pubic
symphysis to the floor (in a standing patient) is measured to obtain the lower segment value; this is
subtracted from the patients height to obtain the upper segment value. The U:L ratio is then calculated
using the two values. In infants, the legs are relatively shorter than the rest of the body, resulting in a
typical U:L ratio of 1.7. As children grow, the U:L ratio decreases to approximately 1.0 at 10 years of age.
In pubertal children or those who have completed puberty, the U:L ratio is typically between 0.85 and
0.95. The boy described in the vignette has experienced mildly precocious puberty, as evidenced by his
relatively tall stature and advanced bone age. Accordingly, his U:L ratio should be reflective of a pubertal
or postpubertal male (U:L=0.9).
Children who experience early puberty often have relative tall stature during puberty. However,
they also complete their growth prematurely and have a somewhat lower peak adult height than expected
(because of the rapid fusion of their growth plates and shorter legs). Conversely, children who experience
later puberty may have relative short stature in youth but an adult height that is slightly above
expectations (because of the slow but constant prepubertal growth of long bones without rapid maturation
of the growth plate). Because the timing of puberty affects both linear growth rate and skeletal maturity,
children who have precocious puberty may have slightly higher (but still within the reference range) U:L
ratios, and children who have delayed puberty have slightly lower U:L ratios. More extreme examples of
changes in the U:L segment ratios are observed in syndromes characterized by abnormal growth
patterns. For example, Marfan syndrome is characterized by very long legs and, therefore, a very low U:L
ratio; patients who have hypochondroplasia have shorter limbs and, therefore, have elevated U:L ratios.

American Board of Pediatrics Content Specification(s):


Understand the impact of relatively early or late puberty on final adult height
Know that the upper body segment-to-lower body segment ratio is lower in teens during their growth
spurt than in adults

Suggested Reading:
Rose SR, Vogiatzi MG, Copeland KC. A general pediatric approach to evaluating a short child. Pediatr
Rev. 2005;26:410 - 420. DOI: 10.1542/pir.26-11-410. Available at:
http://pedsinreview.aappublications.org/cgi/content/full/26/11/410

Rosen D. Physiologic growth and development during adolescence. Pediatr Rev. 2004;25:194-200. DOI:
10.1542/pir.25-6-194. Available at: http://pedsinreview.aappublications.org/cgi/content/full/25/6/194

Rosenfeld RG, Cohen P. Disorders of growth hormone/insulin like growth factor secretion and action. In:
Sperling MA, ed. Pediatric Endocrinology. 3rd ed. Philadelphia, PA: Saunders; 2008:254-334

Copyright 2012 American Academy of Pediatrics


2012 PREP SA ON CD-ROM

Question 204
A 15-month-old boy bangs his head when he gets upset and does not get his way as well as
when he falls asleep in his crib. He currently says about five words and is ambulating independently. His
parents are concerned that he will seriously hurt himself and are puzzled about how to decrease this
behavior.
Of the following, the MOST appropriate next step is to
A. have the child fitted for a soft helmet
B. have the parents hold him when he begins to bang his head
C. instruct the parents to ignore the behavior
D. monitor the child for possible autism
E. refer the boy for an early intervention evaluation

Copyright 2012 American Academy of Pediatrics


2012 PREP SA ON CD-ROM

Critique 204 Preferred Response: C


The boy described in the vignette is banging his head because of frustration and possibly as a
relaxation or comfort measure at bedtime. As a result, his parents should be instructed to ignore the
behavior. They should not pick him up and hold him, which could reinforce the behavior. The use of a soft
helmet may be protective for a child who bangs his or her head continuously, but it does not diminish the
behavior. Children who have autism or other developmental delays may bang their heads or display other
self-stimulatory behaviors, but because this boy has no signs of developmental delay, there is no
indication to either refer him for an early intervention evaluation or monitor him for autism.
Head banging may occur in about 25% of infants and toddlers, tends to be more common in
boys, and peaks by about 18 to 24 months of age. Most normally developing children outgrow the habit
by age 3 years. Head banging may relax or comfort the child and, therefore, may be performed as a
toddler falls asleep or wakes during the night. Children who are teething may also bang their heads,
possibly to distract themselves from the pain. Head banging also may occur when a child is frustrated or
during a tantrum. Some children may bang their heads to gain attention, particularly if the parents have a
strong reaction to the behavior.

American Board of Pediatrics Content Specification(s):


Recognize that head banging does not indicate a sensory deficit

Suggested Reading:
Ryan CA, Gosselin GJ, DeMaso DR. Habit and tic disorders. In: Kliegman RM, Stanton BF, St. Geme JW
III, Schor NF, and Behrman RE, eds. Nelson Textbook of Pediatrics. 19th ed. Philadelphia, PA: Saunders
Elsevier; 2011:75-77

Pipan ME, Blum NJ. Basics of child behavior and primary care management of common behavioral
problems. In: Voight RG, Macias MM, Myers SM, eds. American Academy of Pediatrics Developmental
and Behavioral Pediatrics. Elk Grove Village, IL: American Academy of Pediatrics; 2011:37-58

Copyright 2012 American Academy of Pediatrics


2012 PREP SA ON CD-ROM

Question 205
A 16-year-old boy presents with bilateral cervical lymphadenitis, fatigue, and
hepatosplenomegaly. Laboratory studies show:
3 9
White blood cell count, 8.2x10 /mcL (8.2x10 /L) with 32% neutrophils, 61% lymphocytes, and 7%
atypical lymphocytes
Heterophile antibodies, negative
Epstein-Barr virus antibodies, negative
Cytomegalovirus antibodies, negative
Toxoplasma immunoglobulin M, positive
As you are reviewing the diagnosis with the patient and his family, they ask where he might have
acquired this infection.
Of the following, the MOST likely source of this patients infection is
A. contaminated cat litter
B. fecal-oral transmission from an infected food handler
C. ingestion of undercooked meat
D. respiratory droplets from an infected classmate
E. scratch from an infected kitten

Copyright 2012 American Academy of Pediatrics


2012 PREP SA ON CD-ROM

Critique 205 Preferred Response: C


The positive Toxoplasma immunoglobulin M result, mild atypical lymphocytosis, and negative
serologies for Epstein-Barr virus and cytomegalovirus described for the boy in the vignette suggest a
diagnosis of acute Toxoplasma infection. Felines are the natural host for T gondii. Cats excrete infectious
oocysts in their feces, which can be ingested by other animals and disseminated to their tissues, including
skeletal muscle and other organs. Ingestion of undercooked meat (especially ground beef or lamb) and
drinking unpasteurized goat milk are the most common mechanisms of infection in people. Gardening in
soil contaminated with cat feces also can lead to the transmission of the pathogen.
Although the oocysts are excreted in cat feces, they must mature (sporulation) over 24 to 48
hours before they become infectious. Exposure through direct contact with cat feces in household litter
boxes has not been demonstrated to be a frequent route of infection for people. Frequent changing of cat
litter can minimize this risk for transmission further. Transmission of T gondii from food handlers,
respiratory transmission, or transmission from a kitten scratch has not been documented.
Acute Toxoplasma infection in immunocompetent hosts acquired after birth is generally
asymptomatic, but when symptomatic infection occurs, it may present as a mononucleosis-like illness,
with generalized lymphadenopathy, fever, chills, malaise, headaches, myalgias, pharyngitis, or
hepatosplenomegaly. Lymphocytosis with some degree of atypia may be present.
Congenital infection with Toxoplasma may present with hepatosplenomegaly, jaundice,
generalized lymphadenopathy, thrombocytopenia, and meningoencephalitis leading to microcephaly,
chorioretinitis, hydrocephalus, seizures, or deafness. Cerebral calcifications may be seen on
ultrasonography or computed tomography scan. Early diagnosis and treatment with sulfadiazine and
pyrimethamine plus supplemental leucovorin may improve long-term outcome.
Isolated ocular toxoplasmosis also can occur in children or adults as a result of reactivation of
congenital infection or, rarely, with acquired infection.
Finally, in symptomatic human immunodeficiency virus infection or other chronic
immunodeficiency states, reactivated Toxoplasma infection can cause encephalitis, pneumonitis, or
systemic toxoplasmosis.

American Board of Pediatrics Content Specification(s):


Identify the clinical manifestations of Toxoplasma gondii infections acquired after birth
Know the epidemiology of toxoplasmosis: hosts, intermediate hosts, modes of transmission (vertical
transmission from mother to infant, ingestion of cysts from contaminated food or soil)

Suggested Reading:
American Academy of Pediatrics. Toxoplasma gondii infections (toxoplasmosis). In: Pickering LK, Baker
CJ, Kimberlin DW, Long SS, eds. Red Book: 2009 Report of the Committee on Infectious Diseases. 28th
ed. Elk Grove Village, IL: American Academy of Pediatrics; 2009:667-672

Jones JL, Dargelas V, Roberts J, Press C, Remington JS, Montoya JG. Risk factors for Toxoplasma
gondii infection in the United States. Clin Infect Dis. 2009;49:878884.DOI: 10.1086/605433. Available at:
http://cid.oxfordjournals.org/content/49/6/878.long

Remington JS, Thulliez P, Montoya JG. Recent developments for diagnosis of toxoplasmosis. J Clin
Microbiol. 2004;42:941-945. DOI: 10.1128/JCM.42.3.941-945.2004. Available at:
http://www.ncbi.nlm.nih.gov/pmc/articles/PMC356902/?tool=pubmed

Copyright 2012 American Academy of Pediatrics


2012 PREP SA ON CD-ROM

Question 206
A 10-month-old previously healthy female infant presents with a 3-day history of fever and rash
that started on her trunk and spread to her extremities. Blisters developed on her abdomen today. On
physical examination, her temperature is 38.0C, heart rate is 110 beats/min, and respiratory rate is 35
breaths/min. She has mild facial edema and perioral crusting (Item Q206A). Her mucous membranes are
mildly dry but without lesions. An erythematous, sunburn-like eruption is present on her trunk and
extremities and is accentuated in the flexor creases (Item Q206B). The rash is warm to the touch and
mildly painful. There are two flaccid bullae on the abdomen and a circular erosion on the chest at the site
of prior monitor lead placement (Item Q206C).
Of the following, the test MOST likely to establish the diagnosis is a
A. blood culture
B. creatinine phosphokinase assessment
C. hepatic transaminase assessment
D. serum antibody test
E. skin biopsy

Copyright 2012 American Academy of Pediatrics


2012 PREP SA ON CD-ROM

Question 206

(Courtesy of D Krowchuk)
Crusting, as described for the infant in the vignette.

Copyright 2012 American Academy of Pediatrics


2012 PREP SA ON CD-ROM

Question 206

(Courtesy of D Krowchuk)
Sunburnlike eruption, as described for the infant in the vignette.

Copyright 2012 American Academy of Pediatrics


2012 PREP SA ON CD-ROM

Question 206

(Courtesy of D Krowchuk)
Erosion, as described for the infant in the vignette.

Copyright 2012 American Academy of Pediatrics


2012 PREP SA ON CD-ROM

Critique 206 Preferred Response: E


The clinical findings described for the infant in the vignette are classic for staphylococcal scalded
skin syndrome (SSSS), a toxin-mediated exfoliative dermatitis caused by the hematogenous spread of
two exotoxins, epidermolytic toxin A (ET-A) and epidermolytic toxin B (ET-B), which are produced by
toxigenic strains of Staphylococcus aureus. Although a definitive diagnosis of SSSS depends on skin
biopsy and cultures, in most cases the diagnosis is made clinically. Skin biopsy of lesions reveals splitting
that occurs beneath and within the stratum granulosum layer of the epidermis. Blood cultures in children
usually are negative, but cultures of the focal skin lesions or pyogenic foci may grow S aureus.
The clinical manifestations of SSSS are age-related. Fever and a tender, scarlatiniform eruption
with localized bullous impetigo, such as occurred in this infant, are common. Sloughing of superficial skin
occurs with gentle friction as ocurred at the site at monitor lead placement for the child described in the
vignette (Nikolsky sign). Children have generalized desquamation that is especially prominent on the face
and neck. Affected neonates present with a generalized exfoliation called Ritter disease.
The management of SSSS includes careful evaluation for dehydration and superinfection.
Electrolyte concentrations and hydration status should be monitored in patients who have extensively
denuded skin. Careful attention to antisepsis is important for areas of unprotected skin. Parenteral
(followed by oral) antistaphylococcal antibiotic therapy is recommended to treat focal areas of infection
and decrease the burden of staphylococci.
A complete blood count and assessment of creatinine phosphokinase and hepatic transaminases
can be useful in establishing the diagnosis of S aureus toxic shock syndrome, but they are not helpful in
patients who have SSSS. Serum antibody tests for Rocky Mountain spotted fever, leptospirosis, or
measles may be indicated in patients who have significant exposure histories and present with fever and
rash but are not of use for the patient described in the vignette, who has a classic presentation of SSSS.

American Board of Pediatrics Content Specification(s):


Recognize the appearance of the staphylococcal scalded skin syndrome
Know that staphylococcal scalded skin syndrome is mediated by a toxin released by certain strains of
staphylococci

Suggested Reading:
American Academy of Pediatrics. Staphylococcal infections. In: Pickering LK, Baker CJ, Kimberlin DW,
Long SS, eds. Red Book: 2009 Report of the Committee on Infectious Diseases. 28th ed. Elk Grove
Village, IL: American Academy of Pediatrics; 2009:601-615

Lowell GS. Daum RS. Staphylococcus aureus. In: Long SS, Pickering LK, Prober CG, eds. Principles and
Practice of Pediatric Infectious Diseases. 3rd ed. Philadelphia, PA: Churchill Livingstone Elsevier;
2008:679-692

Copyright 2012 American Academy of Pediatrics


2012 PREP SA ON CD-ROM

Question 207
A 15-year-old girl presents with gross hematuria (bright red blood with clots) accompanied by
sharp left-sided back pain. She denies fever, dysuria, frequency, urgency, or trauma. On physical
examination, her temperature is 37.3C, heart rate is 90 beats/min, respiratory rate is 18 breaths/min, and
blood pressure is 116/72 mm Hg. You note no costovertebral angle tenderness, suprapubic tenderness,
or edema. Urinalysis reveals:
Specific gravity, 1.020
pH, 6
3+ blood
Trace protein
Leukocyte esterase, negative
Nitrite, negative
Microscopy documents 20 to 50 red blood cells/high-power field (hpf) and 5 to 10 squamous
epithelial cells/hpf. Abdominal ultrasonography shows mild dilation of the collecting system on the left,
with some debris in the bladder. You suspect urolithiasis.
Of the following, the study that is MOST likely to establish the diagnosis is
A. abdominal computed tomography scan
B. diethylene-triamine-penta-acetic acid (DTPA) furosemide renal scan
C. magnetic resonance urography
D. random urine sample for calcium and creatinine
E. 24-hour urine collection for calcium and creatinine

Copyright 2012 American Academy of Pediatrics


2012 PREP SA ON CD-ROM

Critique 207 Preferred Response: A


The girl described in the vignette presents with gross hematuria and unilateral sharp back pain.
Based on the color of the urine (bright red with clots), the possibility of lower urinary tract bleeding should
be considered. Furthermore, in the absence of trauma, the possibility of urolithiasis needs to be
entertained. Findings on both urinalysis, which documented red blood cells on microscopy, and
ultrasonography, which revealed mild dilation of the collecting system ipsilateral to the side of the clinical
symptoms, support this diagnosis. Pediatric patients who have stone disease present most commonly
with pain and hematuria; more rarely, they can present with a urinary tract infection, renal failure, or
recurrent abdominal pain (without flank pain or hematuria). When facing this clinical situation, the
practitioner must decide on the best study to establish the diagnosis of urolithiasis.
Unless the patient passes a small stone, which can be analyzed, the clinician must attempt to see
the stone with the assistance of imaging. Either ultrasonography or noncontrast spiral computed
tomography (CT) scan of the abdomen can be used (Item C207). The latter study is more sensitive but
involves radiation exposure. Accordingly, most pediatric nephrologists and urologists recommend
ultrasonography as the initial test. However, if ultrasonography is not diagnostic and the condition is still
suspected, spiral CT scan should be performed. For patients who have known stone disease and require
follow-up imaging, ultrasonography is preferred over CT scan to avoid radiation risk. DTPA furosemide
renal scan is a nuclear medicine technique best used to diagnose obstruction in a patient who has
hydronephrosis such as ureteropelvic junction obstruction. This test may show partial obstruction in the
patient in the vignette (if the stone is obstructive), but the finding would be nonspecific. Magnetic
resonance urography is less sensitive than CT scan in diagnosing stone disease; it is used in evaluation
of hematuria or in assessment of congenital urologic abnormalities. Urine collections may suggest
(random collection) or confirm (24-hour collection) the presence of hypercalciuria, but such findings in
isolation do not document or exclude the presence of urolithiasis.
Once urolithiasis is diagnosed, the next step is to determine its pathogenesis. Thorough history,
including family history of stone disease, should be pursued because it is often positive. Risk factors for
stone disease include malabsorptive diseases associated with steatorrhea (eg, cystic fibrosis and
inflammatory bowel disease), medications (vitamin D preparations, furosemide, topiramate, indinavir),
ketogenic diet, and immobilization. Laboratory evaluation consists of urinalysis with microscopy to confirm
the presence of hematuria and to check for crystals, which can be helpful in the setting of stone disease
as well as assessment of electrolytes, blood urea nitrogen, creatinine, calcium, magnesium, phosphorus,
uric acid, intact parathyroid hormone, and vitamin D (25-hydroxyvitamin D and 1,25 dihydroxyvitamin D).
Twenty-four hour urine collection can be very helpful in predicting the underlying metabolic explanation
for the presence of stone disease and should measure volume, creatinine, sodium, calcium, magnesium,
phosphorus, oxalate, citrate, uric acid, and cystine. Ideally, stone analysis is performed, but this can only
be achieved through straining of urine or surgical extraction. Depending on results of the stone analysis,
blood chemistries, and 24-hour urine collection, the patient may undergo further testing more specific to
the abnormality discovered. For example, moderate-to-severe hyperoxaluria necessitates specific testing
for primary hyperoxaluria type I and II.

American Board of Pediatrics Content Specification(s):


Plan the evaluation of a child with urinary tract stones

Suggested Reading:
Alon US, Srivastava T. Urolithiasis. In: Kher KK, Schnaper HW, Makker SP, eds. Clinical Pediatric
Nephrology. 2nd ed. London, England: Informa Healthcare; 2007:539-551

Hoppe B, Kemper MJ. Diagnostic examination of the child with urolithiasis or nephrocalcinosis. Pediatr
Nephrol. 2010;25:403-413. DOI: 10.1007/s00467-008-1073-x. Available at:
http://www.ncbi.nlm.nih.gov/pmc/articles/PMC2810372/?tool=pubmed

Polito C, La Manna A, Signoriello G, Marte A. Recurrent abdominal pain in childhood urolithiasis.


Pediatrics. 2009;124:e1088-1094. DOI: 10.1542/peds.2009-0825. Available at:
http://pediatrics.aappublications.org/cgi/content/full/124/6/e1088

Copyright 2012 American Academy of Pediatrics


2012 PREP SA ON CD-ROM

Critique 207

(Courtesy of D Mulvihill)
Noncontrast computed tomography scan of the pelvis shows a density at the ureterovesical junction,
consistent with a ureteral stone.

Copyright 2011 American Academy of Pediatrics


2012 PREP SA ON CD-ROM

Question 208
The parents of an 8-year-old patient call for an appointment because they are concerned that she
may have an anxiety problem. The father explains that other family members receive treatment for
anxiety. Your office procedure is to have parent and initial teacher Vanderbilt Diagnostic Rating Scale
forms completed before all visits for possible mental health problems. The scores obtained for this girl
indicate the presence of possible anxiety or depression. Further history obtained upon questioning
increases your suspicion of a mental health problem.
Of the following, the MOST appropriate next step is to
A. administer the National Institute of Mental Health Diagnostic Interview for Children IV (DISC
IV)
B. administer the Pediatric Symptom Checklist
C. complete syndrome-specific scales for anxiety and depression
D. recommend cognitive behavioral therapy for an anxiety disorder
E. refer the child for play therapy

Copyright 2012 American Academy of Pediatrics


2012 PREP SA ON CD-ROM

Critique 208 Preferred Response: C


A diagnosis of anxiety disorder or depression cannot and should not be based upon a general
behavior rating scale such as the Vanderbilt Diagnostic Rating Scale or the Pediatric Symptom Checklist
(PSC). These instruments do not have sufficient sensitivity and specificity; symptoms recorded on a rating
scale actually may be a reflection of an overanxious parent or an entirely different problem such as a
learning disorder causing anxiety symptoms in school. If results of a general screening instrument such
as the Vanderbilt indicate a potential problem, the clinician should follow up with a syndrome-specific
scale to assist with the diagnosis and monitoring of a specific problem such as anxiety. Syndrome-
specific scales for anxiety include the Screen for Child Anxiety Related Emotional Disorders (SCARED),
the Multidimensional Anxiety Scale for Children (MASC), and the Spence Childrens Anxiety Scale.
Syndrome-specific scales for depression include the Childhood Depression Rating Scale (CDRS), the
Beck Depression Inventory for Youth (BDI-Y), and the Center for Epidemiology Studies Depression Scale
modified for children (CES-DC). When used in combination with a good interview and history, such scales
can provide important information for both the diagnosis and monitoring of anxiety or depressive
symptoms.
The DISC IV is composed of 3,000 questions and is used primarily for research purposes.
Administering the DISC IV is not within the scope of practice of the average practitioner. The PSC is a
general-purpose mental/behavioral health screening tool for primary care that assesses the likelihood that
some type of mental/behavioral health disorder is present. Because the PSC is not more diagnostically
specific for anxiety or depressive disorders than the Vanderbilt, its use for this child would not be helpful.
No evidence-based studies support play therapy for anxiety or depression in children. Cognitive
behavioral therapy for anxiety is an evidence-based treatment, but the diagnosis should be confirmed
before initiation of any treatment.
Pediatricians can greatly assist families by properly screening and managing common mental
health problems. Although mental health specialists can perform these assessments, this approach is not
cost-effective for the family and can delay care. Further, many families may not be willing to bring their
child to a mental health specialist. Indeed, studies have shown that fewer than 50% of families follow
through on such a referral.

AAP Mental Health Competency:


Recognize the positive role of rating scales in assessing child depression and anxiety disorders in a
pediatric practice.

Suggested Readings:
American Academy of Pediatrics Task Force on Mental Health. Anxiety Cluster Guidance. Addressing
Mental Health Concerns in Primary Care: A Clinician's Toolkit [CD-ROM]. Elk Grove Village, IL: American
Academy of Pediatrics; 2010

Connolly SD, Bernstein GA: Work Group on Quality Issues. Practice parameter for the assessment and
treatment of children and adolescents with anxiety disorders. J Am Acad Child Adolesc Psychiatry.
2007;46:267-283. Abstract available at: http://www.ncbi.nlm.nih.gov/pubmed/17242630

Kataoka SH, Zhang L, Wells K. Unmet need for mental health care among U.S. children: variation by
ethnicity and insurance status. Am J Psychiatry. 2002;159:1548-1555. Available at:
http://ajp.psychiatryonline.org/cgi/content/full/159/9/1548

Silverman WK, Ollendick TH. Evidence-based assessment of anxiety and disorders in children and
adolescents. J Clin Child Adolesc Psychol. 2005;34:380-411. Abstract available at:
http://www.ncbi.nlm.nih.gov/pubmed/16026211

Stancin T, Aylward GP. Assessment of development and behavior. In: Wolraich ML, Drotar DD, Dworkin
PH, Perrin EC, eds. Developmental-Behavioral Pediatrics: Evidence and Practice. Philadelphia, PA:
Mosby Elsevier; 2008:144-176

Copyright 2012 American Academy of Pediatrics


2012 PREP SA ON CD-ROM

Stancin T, Perrin EC. Behavioral screening. In: Augustyn M, Zuckerman B, Caronna EB, eds. The
Zuckerman Parker Handbook of Developmental and Behavioral Pediatrics for Primary Care. 3rd ed.
Philadelphia, PA: Lippincott, Williams & Wilkins, a Wolters Kluwer business; 2011:44-47

United States Department of Health and Human Services. Mental Health: A Report of the Surgeon
General. Rockville, MD: United States Department of Health and Human Services, Substance Abuse and
Mental Health Services Administration, Center for Mental Health Services, National Institutes of Health,
National Institute of Mental Health; 1999. Available at:
www.surgeongeneral.gov/library/mentalhealth/home.html

Copyright 2012 American Academy of Pediatrics


2012 PREP SA ON CD-ROM

Question 209
A mother brings her 3-year-old daughter in for evaluation. She enrolled her daughter in child care
1 week ago. Over the past 3 days, the child has experienced yellowish rhinorrhea and cough. Her
appetite has remained good, and her highest temperature has been 37.3C. On physical examination, the
child is interactive and playful. Her tympanic membranes are normal bilaterally, but yellowish rhinorrhea is
visible in both nares. The remainder of the physical examination findings are unremarkable. You order a
sinus radiograph series, which shows bilateral maxillary mucosal thickening with opacification of the
ethmoid sinuses.
Of the following, the MOST appropriate next therapy is
A. amoxicillin orally
B. azithromycin orally
C. ceftriaxone intramuscularly
D. observation
E. trimethoprim-sulfamethoxazole orally

Copyright 2012 American Academy of Pediatrics


2012 PREP SA ON CD-ROM

Critique 209 Preferred Response: D


Despite the discolored rhinorrhea and abnormal sinus radiograph described for the girl in the
vignette, the combination of a new child care environment and duration of symptoms of less than 1 week
are consistent with a viral upper respiratory tract infection (URI). Patients who have viral URIs should be
managed with observation and symptomatic treatment only.
The American College of Radiology recommends that no sinus imaging studies be performed for
acute sinusitis unless a patient does not respond appropriately to antibiotics or there is concern for
complications. In general, plain film sinus radiographs are difficult to interpret and have both high false-
negative and false-positive rates. Further, the sinuses are still developing in children, which may appear
as opacification. In addition, one study demonstrated that 80% of computed tomography (CT) scan
abnormalities in patients who had acute URI symptoms resolved within 2 to 4 weeks without treatment. In
patients who have chronic sinusitis (ie, symptom duration >12 weeks), coronal CT scan is the preferred
imaging modality. Compared with radiography, the CT scan better delineates the sinus structures,
including the ostiomeatal complex.
Because the primary pathogens associated with acute bacterial sinusitis are Streptococcus
pneumoniae, nontypeable Haemophilus influenzae, and Moraxella catarrhalis, antibiotics directed against
these bacteria should be administered. If the girl in the vignette had had symptoms that persisted more
than 7 to 10 days or she appeared more ill, with persistent constitutional symptoms such as fever or
lethargy, antibiotics would be appropriate. Amoxicillin remains the first-line medication, with
cephalosporins, azithromycin, and clarithromycin preferred in penicillin-allergic patients. Because of
changes in resistance patterns, trimethoprim-sulfamethoxazole and erythromycin-sulfisoxazole are no
longer recommended as first- or second-line treatment options. The use of ceftriaxone for sinusitis is
generally limited to patients whose symptoms are refractory to other antibiotic regimens.

American Board of Pediatrics Content Specification(s):


Recognize the limitations of imaging studies in diagnosing acute sinusitis in young children

Suggested Reading:
Dykewicz MS, Hamilos DL. Rhinitis and sinusitis. J Allergy Clin Immunol. 2010;125(suppl 2):S103-S115.
DOI: 10.1016/j.jaci.2009.12.989. Available at: http://www.jacionline.org/article/S0091-6749(09)02881-
4/fulltext

Gwaltney JM Jr, Phillips CD, Miller RD, Ricker DK. Computed tomographic study of the common cold. N
Engl J Med. 1994;330:25-30. Available at:
http://www.nejm.org/doi/full/10.1056/NEJM199401063300105#t=article

Subcommittee on Management of Sinusitis and Committee on Quality Improvement. Clinical practice


guideline: management of sinusitis. Pediatrics. 2001;108:798-808. Available at:
http://pediatrics.aappublications.org/cgi/content/full/108/3/798

Triulzi F, Zirpoli S. Imaging techniques in the diagnosis and management of rhinosinusitis in children.
Pediatr Allergy Immunol. 2007;18(suppl 18):46-49. DOI: 10.1111/j.1399-3038.2007.00633.x. Abstract
available at: http://www.ncbi.nlm.nih.gov/pubmed/17767608

Copyright 2012 American Academy of Pediatrics


2012 PREP SA ON CD-ROM

Question 210
A 10-year-old girl presents with right wrist pain. According to her mother, the girl has been
complaining of this pain for 2 weeks, and the pain has persisted despite administration of daily ibuprofen.
The girl cannot recall any recent injury and has had no fever or other systemic symptoms. She is right-
handed and participates in gymnastics and soccer. Physical examination of her wrist reveals no redness
or swelling, but she has moderate tenderness diffusely over the radial aspect and pain with wrist flexion
and extension. She has no snuff-box tenderness and her hand is neurovascularly intact. You obtain
radiographs of the wrist (Item Q210).
Of the following, the MOST likely explanation for this childs wrist pain is
A. a navicular fracture
B. a sprain
C. an overuse injury
D. osteomyelitis
E. septic arthritis

Copyright 2012 American Academy of Pediatrics


2012 PREP SA ON CD-ROM

Question 210

(Coutesy of A Weiss-Kelly)
Radiographs, as described for the girl in the vignette.

Copyright 2012 American Academy of Pediatrics


2012 PREP SA ON CD-ROM

Critique 210 Preferred Response: C


Overuse injury of the distal radial physis is a well-recognized condition among young gymnasts
and the most likely diagnosis for the girl described in the vignette. It is believed that the chronic, repetitive
physical loading on immature physeal cartilage of the distal radius causes physeal ischemia that, if
unaddressed, may lead to osseous necrosis. Although most of these stress injuries resolve with rest and
do not result in permanent damage, several case reports documenting premature partial or complete
physeal closure with radial growth arrest in these young athletes have raised concern.
The prevalence of radiographically documented stress-related physeal injuries ranges from 10%
to 85% of young gymnasts studied. The findings typically include irregular widening of the physes,
thickening of the zone of provisional calcification, and subchondral sclerosis, as are seen in the
radiograph from the patient in the vignette. Radiographs in patients who have navicular fractures may be
normal initially, but after 2 weeks, they demonstrate fracture healing and callus formation. Sprains and
septic arthritis would not be expected to produce radiographic changes, and osteomyelitis is clinically
unlikely with the absence of systemic symptoms.
Clinically, athletes who have overuse injuries initially present with chronic wrist pain on weight-
bearing and progress to decreased wrist range of motion. Physical symptoms precede radiographic
changes. Experts suggest that early evaluation of pediatric athletes who develop joint pain is important in
identifying stress injuries so that appropriate interventions, including rest until symptoms resolve,
modification of training regimens, and strengthening/conditioning routines, can be implemented.

American Board of Pediatrics Content Specification(s):


Recognize that a distal radial epiphyseal injury is common in young gymnasts

Suggested Reading:
Caine D, DiFiori J, Maffulli N. Physeal injuries in childrens and youth sports: reasons for concern? Br J
Sports Med. 2006;40:749760. DOI: 10.1136/bjsm.2005.017822. Available at:
http://www.ncbi.nlm.nih.gov/pmc/articles/PMC2564388/?tool=pubmed

Dwek JR, Cardoso F, Chung CB. MR imaging of overuse injuries in the skeletally immature gymnast:
spectrum of soft-tissue and osseous lesions in the hand and wrist. Pediatr Radiol. 2009;39:13101316.
DOI: 10.1007/s00247-009-1428-x. Available at:
http://www.ncbi.nlm.nih.gov/pmc/articles/PMC2776148/?tool=pubmed

Webb BG, Rettig LA. Gymnastic wrist injuries. Curr Sports Med Rep. 2008;7:289295. Abstract available
at: http://www.ncbi.nlm.nih.gov/pubmed/18772690

Copyright 2012 American Academy of Pediatrics


2012 PREP SA ON CD-ROM

Question 211
An 8-month-old girl presents to your office because of poor growth. She was born at 36 weeks
gestation and had a birthweight of 2.5 kg. No problems were noted during the neonatal period, and early
growth and development were normal. However, at the age of about 3 months, she was hospitalized
because of purulent otitis media. Since that time, she has been treated on two more occasions for otitis
media. Over the past month, her parents have noted that the baby has two to three large, foul-smelling,
bulky stools per day, and she has failed to gain weight. Her developmental milestones are normal for age,
and there are no findings of note on the family history. Physical examination of the alert, well-hydrated
infant shows a weight of 6.4 kg, length of 66 cm, and palpable liver edge 3.0 cm below the right costal
margin. Initial laboratory study results include:
Hemoglobin, 10.8 g/dL (108 g/L)
3 9
White blood cell count, 3.0x10 /mcL (3.0x10 /L) (15% neutrophils, 75% lymphocytes, 10%
monocytes)
3 9
Platelet count, 50x10 /mcL (50x10 /L)
Alanine aminotransferase, 61 units/L (normal, up to 30 units/L)
Albumin, 3.8 g/dL (38 g/L)
Prothrombin time, 12.5 seconds
International Normalized Ratio, 1.1
Of the following, the study that is MOST likely to establish the diagnosis for this infant is
A. genetic testing
B. liver biopsy
C. serum amylase
D. sweat chloride test
E. tissue transglutaminase antibody assay

Copyright 2012 American Academy of Pediatrics


2012 PREP SA ON CD-ROM

Critique 211 Preferred Response: A


The poor weight gain; history of multiple episodes of otitis media; recent history of several large
stools per day; physical finding of extremity bruising; and laboratory findings that indicate anemia as
well as neutropenia, a platelet count in the low normal range, and a mild increase in alanine
aminotransferase described for the infant in the vignette suggest a disorder characterized by nutrient
malabsorption and abnormal hematopoiesis. The clinical condition in the pediatric age group that
manifests these findings is Shwachman-Diamond syndrome (SDS), a rare autosomal recessive
disorder that is the second most common cause of exocrine pancreatic insufficiency in childhood, after
cystic fibrosis (CF). SDS also is characterized by bone marrow dysfunction, skeletal abnormalities, and
an increased leukemia risk. Affected children typically present in early infancy with steatorrhea; growth
failure; often deficiencies of fat-soluble vitamins A, D, E, and K; and symptoms arising from bone
marrow dysfunction. However, SDS, unlike CF, is not associated with abnormalities in pancreatic
ductular morphology or function. In fact, lipase excretion increases with age, and older patients often
experience normalization of pancreatic function and fat absorption. Thus, in later childhood and
adolescence, approximately 50% of patients who have SDS eventually do not require enzyme
replacement therapy. Up to 90% of affected patients manifest specific genetic mutations at
chromosomal locus 7q11. In infancy, the diagnosis is usually made by genetic testing for the specific
mutation, although many cases may be identified by confirming extensive fatty replacement of the
pancreas on imaging studies (such as abdominal magnetic resonance imaging, ultrasound, or
computed tomography). Pancreatic insufficiency is defined as the loss of pancreatic exocrine function,
resulting in reduced digestive enzyme output and the consequent failure of adequate intraluminal
nutrient hydrolysis and absorption. Causes of pancreatic insufficiency in children are listed in (Item
C211). With the notable exception of CF, most of these conditions are rare disorders. Nevertheless, an
evaluation of exocrine pancreatic function should be considered for all patients who present with signs
and symptoms suggestive of maldigestion/malabsorption.
Initial assessment of this problem must include a sweat chloride determination. A positive test
(>60 mmol/L) strongly indicates a diagnosis of CF, although both false-positive and false-negative results
may occur. The gold standard for evaluation of exocrine pancreatic function is the determination of
enzyme output following intravenous secretin stimulation. However, this study is cumbersome, involving
endoscopic intubation of the duodenum, and it is difficult to perform accurately. Determination of serum
amylase levels may not be useful in this clinical setting because they may be low, normal, or elevated
despite pancreatic insufficiency. Furthermore, during infancy, salivary gland production of amylase may
result in normal serum levels. Accordingly, other tests are more widely employed to identify the
malabsorptive state, and several screening studies have been used to assess exocrine pancreatic
function. The serum immunoreactive trypsinogen value is low in most cases of pancreatic insufficiency,
including SDS, but is markedly elevated in most infants who have CF, in whom exocrine pancreatic
insufficiency is the consequence of pancreatic ductular obstruction, leading to reflux of enzyme into the
bloodstream. For older patients who have CF, in whom progressive fibrosis and acinar destruction of the
exocrine pancreas has occurred, serum trypsinogen concentrations are well below normal in 95% of
cases. More recently, assay of fecal elastase has been employed to evaluate pancreatic function.
Pancreatic elastase is excreted in stool, and it is not affected by bacterial degradation. Exocrine
insufficiency is suggested by a fecal elastase concentration of less than 200 g/g stool. However,
although the test is highly sensitive, it has decreased specificity when evaluating exocrine pancreatic
insufficiency in children.
Initial estimates of steatorrhea may be made by a qualitative fecal fat determination, following a
fat stain for microscopic analysis of a fecal smear. However, the fecal fat stain does not accurately assess
fat output, which can only be determined from a 72-hour stool collection. This test is reserved for infants
older than 6 months of age because of the normal state of relative pancreatic insufficiency in younger
infants. Total stool output is collected in a dry, preweighed vessel over a 72-hour period, during which
time the patient consumes a diet containing 3 g/kg fat per day to a maximum of 100 g/day. A coefficient of
fat absorption (total fat excreted total fat intake) of less than 0.93 indicates malabsorption.
A liver biopsy may show nonspecific changes, including steatosis, in patients who have
malnutrition, but they are not specific for SDS. The sweat chloride test, which should be performed, rules
out CF but does not establish a diagnosis of pancreatic insufficiency. Finally, the tissue transglutaminase

Copyright 2012 American Academy of Pediatrics


2012 PREP SA ON CD-ROM

antibody assay is both a sensitive and a specific screening study for celiac disease, but for an infant who
likely is not consuming gluten at the time of symptom onset, celiac disease is an unlikely diagnosis.

American Board of Pediatrics Content Specification(s):


Recognize and diagnose exocrine pancreatic insufficiency in infants

Suggested Reading:
Belamarich PF. Recognizing and diagnosing pancreatic insufficiency in infants. Pediatr Rev. 2002;23:69-
70. DOI: 10.1542/pir.23-2-69. Available at: http://pedsinreview.aappublications.org/cgi/content/full/23/2/69

Burroughs L, Woolfrey A, Shimamura A. Shwachman-Diamond syndrome: a review of the clinical


presentation, molecular pathogenesis, diagnosis, and treatment. Hematol Oncol Clin North Am.
2009;23:233-248. DOI: 10.1016/j.hoc.2009.01.007. Available at:
http://www.ncbi.nlm.nih.gov/pmc/articles/PMC2754297/?tool=pubmed

Ip WF, Dupuis A, Ellis L, et al. Serum pancreatic enzymes define the pancreatic phenotype in patients
with Shwachman-Diamond syndrome. J Pediatr. 2002;141:259-265. Abstract available at:
http://www.ncbi.nlm.nih.gov/pubmed/12183724

Mack DR, Forstner GG, Wilschanski M, Freedman MH, Durie PR. Shwachman syndrome: exocrine
pancreatic dysfunction and variable phenotypic expression. Gastroenterology. 1996;111:1593-1602.
Abstract available at: http://www.ncbi.nlm.nih.gov/pubmed/8942739

Shwachman H, Diamond L, Oski F, Khaw KT. The syndrome of pancreatic insufficiency and bone marrow
dysfunction. J Pediatr. 1964;65:645-663

Copyright 2012 American Academy of Pediatrics


2012 PREP SA ON CD-ROM

Critique 211

Item C211. Conditions Associated With Pancreatic Insufficiency In Children


! Cystic fibrosis
! Shwachman-Diamond syndrome
! Chronic pancreatitis
" Autosomal-dominant hereditary pancreatitis
" Anatomic causes
! Annular pancreas
! Pancreas divisum
" Drug-induced (multiple agents)
" Trauma
" Metabolic disorders
! alpha-1-antitrypsin deficiency
! Fatty acid oxidation defects
! Lipoprotein lipase gene defects
! Mitochondrial respiratory chain disorders
! Organic acidemias
! Johanson-Blizzard syndrome
! Pearson syndrome
! Congenital rubella
! Pancreatic agenesis/hypoplasia

Copyright 2011 American Academy of Pediatrics


2012 PREP SA ON CD-ROM

Question 212
You are called by a neonatologist regarding the transfer of a convalescing preterm infant from the
tertiary hospital to your care at the community hospital. The 4-week-old infant was born at 29 weeks
gestation, weighing 1,100 g. He required mechanical ventilation for 1 week for respiratory distress
syndrome and needed supplemental oxygen therapy for 1 additional week after extubation. He received
indomethacin treatment for a patent ductus arteriosus (PDA), with his last cardiac echocardiography 4
days ago showing closure of the PDA. Head ultrasonography revealed a right-sided grade I
intraventricular hemorrhage 1 week after birth. He is now breathing room air and tolerating nasogastric
feedings of fortified human milk.
Of the following, the MOST important consultant to have available at the community hospital for
this infant is a pediatric
A. audiologist
B. cardiologist
C. neurologist
D. ophthalmologist
E. pulmonologist

Copyright 2012 American Academy of Pediatrics


2012 PREP SA ON CD-ROM

Critique 212 Preferred Response: D


Preterm infants who have received oxygen therapy, such as the infant described in the vignette,
are at risk for retinopathy of prematurity (ROP) and require serial retinal examinations by a qualified
ophthalmologist until the retina matures. ROP is a disorder of retinal vascularization that was first
described in preterm infants in the 1940s and remains a leading cause of blindness in childhood in the
United States. The extent of retinal vascularization seen at birth is directly related to gestational age in the
preterm infant. Overproduction of the developing blood vessels at the transition point between the
vascularized and avascular retina as the infant matures leads to retinopathy. In severe cases, the vessels
break through the retina into the vitreous space and begin to pull the retina off the choroidal bed, leading
to retinal detachment and visual loss (Item C212).
The early cases of ROP in the 1940s were attributed to excessive oxygen exposure, with a
subsequent dramatic decline in incidence seen in the 1950s and 1960s when oxygen use was limited. As
the ability to care for more immature infants developed in the 1970s, ROP rates again increased,
suggesting that prematurity also had a role in the development of ROP. Ongoing research focuses on the
determinants of angiogenesis in the postnatal developing eye, including oxygen saturation, vascular
endothelial growth factor, and insulin-like growth factor.
Screening guidelines published by the American Academy of Pediatrics in 2006 recommend that
any infant whose birthweight is less than 1,500 g or whose gestational age is 30 weeks or less at birth
undergoes retinal screening examinations. Infants between 1,500 and 2,000 g or born at greater than 30
weeks gestation may be screened if the attending pediatrician feels that they are at risk for ROP. ROP
development correlates with the infants postmenstrual age (gestation at birth plus chronologic age), with
ROP generally not becoming visible until at least 4 weeks and rarely before 31 weeks postmenstrual age.
The timing of the first ROP examination is at 31 weeks postmenstrual age for infants born at 27 weeks or
less gestation. The first ROP examination is 4 weeks after birth for those born at 28 weeks or greater
gestation. Retinal examinations must be performed by an ophthalmologist trained to identify and classify
the retinal findings of ROP because detection of active ROP is essential for the initiation of early
treatment with peripheral retinal ablation with laser to prevent progression to detachment. ROP screening
is discontinued when retinal vascularization is completely matured, although trained ophthalmologists
also may stop screening at 45 weeks postmenstrual age if no active disease is seen. Outpatient follow-up
is recommended yearly because preterm infants are at increased risk for strabismus and myopia.
Preterm infants at risk for ROP should not be transferred without verifying that a pediatric
ophthalmologist experienced in ROP examination is available. Ophthalmologists with ongoing
involvement in examination of infants who have ROP or telemedicine with digital wide-field corneal
contact cameras may be acceptable alternatives.
Although preterm infants are at risk of cardiovascular, respiratory, and neurologic complications,
infants who are transferred to community hospitals for convalescent care rarely require an in-house
pediatric cardiologist, pulmonologist, or neurologist. The ability to perform routine newborn hearing
screening before the discharge of a preterm infant is essential, but the presence of a pediatric audiologist
is not necessary.

American Board of Pediatrics Content Specification(s):


Recognize that preterm infants who have been treated with oxygen require a first retinal examination
at 4 to 6 weeks of age to identify those who have retinopathy of prematurity
Know the groups of infants who should be screened for retinopathy of prematurity

Suggested Reading:
Section of Ophthalmology, American Academy of Pediatrics, American Academy of Ophthalmology,
American Association for Pediatric Ophthalmology and Strabismus. Screening examination of premature
infants for retinopathy of prematurity. Pediatrics. 2006;117:572-576. DOI: 10.1542/peds.2005-2749.
Available at: http://pediatrics.aappublications.org/cgi/content/full/117/2/572

Chen ML, Guo L, Smith LEH, Dammann CEL, Dammann O. High or low oxygen saturation and severe
retinopathy of prematurity: a meta-analysis. Pediatrics. 2010;125:e1483-e1492. DOI: 10.1542/peds.2009-
2218. Available at: http://pediatrics.aappublications.org/cgi/content/full/125/6/e1483

Copyright 2012 American Academy of Pediatrics


2012 PREP SA ON CD-ROM

Fleck BW, McIntosh N. Retinopathy of prematurity: recent developments. NeoReviews. 2009;10:e20-e30.


DOI: 10.1542/neo.10-1-e20. Available at: http://neoreviews.aappublications.org/cgi/content/full/10/1/e20

Phelps DL. Retinopathy of prematurity: history, classification, and pathophysiology. NeoReviews.


2001;2:e153-e166. DOI:10.1542/neo.2-7-e153. Available at:
http://neoreviews.aappublications.org/cgi/content/full/2/7/e153

Copyright 2012 American Academy of Pediatrics


2012 PREP SA ON CD-ROM

Critique 212

(Courtesy of S Freedman)
This eye has developed a partial retinal detachment (stage IVA), despite having had laser several weeks
earlier. The area of laser treatment is seen on the far left, but there is a wide pink ridge of fibrovascular
tissue (black arrows), with elevation of the retina almost to the macula (gray area whose margins are
shown by blue arrows). This eye may progress to stage IVB retinal detachment if the retinal elevation
continues to include the macula and may benefit from surgery termed lens-sparing vitrectomy and
membrane peeling.

Copyright 2011 American Academy of Pediatrics


2012 PREP SA ON CD-ROM

Question 213
During a health supervision visit, you note that an 18-month-old boy has erosions of the medial
portions of his maxillary central incisors and brown discoloration of several teeth (Item Q213). He was
born at term following an uncomplicated pregnancy and has been well, except for two episodes of otitis
media that were successfully treated with amoxicillin. His physical examination findings are otherwise
normal.
Of the following, the MOST likely factor contributing to this boys findings is
A. amoxicillin exposure
B. enamel hypoplasia
C. excessive fluoride exposure
D. exclusive breastfeeding
E. maternal oral colonization with Streptococcus mutans

Copyright 2012 American Academy of Pediatrics


2012 PREP SA ON CD-ROM

Question 213

(Reprinted with permission. Copyright 2002-2010 by the American Academy of Pediatric Dentistry. All
rights reserved.)
Findings, as described for the boy in the vignette.

Copyright 2012 American Academy of Pediatrics


2012 PREP SA ON CD-ROM

Critique 213 Preferred Response: E


Dental caries has been called the most common chronic disease and the most common
infectious disease of childhood. The term early childhood caries (ECC) has replaced older terms such as
nursing bottle caries, baby bottle tooth decay, and milk bottle caries. Although its definition varies among
authors, ECC generally refers to caries affecting the primary dentition, especially in the first 3 years after
birth. It ranges from decay involving a single tooth to widespread caries involving the entire mouth.
Interaction of the tooth, ingested carbohydrates, and oral bacteria comprise the pathophysiology
of caries. Streptococcus mutans is the predominant organism in the oral flora of children who develop
caries and colonizes in the infants mouth at an early age. Maternal oral flora is generally the source of
the infants oral flora, and dental care for the maternal-infant dyad has been suggested as a component of
primary prevention for ECC. These cariogenic bacteria use dietary sugars to form dental plaque, which
allows the bacteria to adhere to the teeth. The microbes also ferment ingested carbohydrates to form an
acidic environment that causes enamel demineralization and shifts the oral flora further toward acid-
tolerant cariogenic bacteria.
The initial appearance of caries is an opaque white area on the tooth. As demineralization
progresses, teeth lose structure, progressing to the end stage of the process, development of cavities.
Classically, baby bottle caries involved cavities of the central incisors, but all teeth are susceptible to this
process.
The incidence of ECC has increased by 15% over the past 2 decades. Between 1999 and 2002,
more than 40% of United States children developed caries in their primary teeth. Because the condition
develops so early, the first visit to a dental clinician should occur within 6 months of the eruption of the
first tooth and not later than 1 year of age. Risk factors for ECC have been recognized, although the best
predictor of primary tooth caries is previous caries. Caries are more common in children from low-income

households, including those enrolled in the Women, Infants and Children program, Head Start , and
Medicaid. Excessive consumption of sugars, especially sucrose, also increases cavity development.
Although some report that human milk may produce more caries than cow milk formula, exclusive
breastfeeding is not a risk factor for early childhood caries. Excessive fluoride exposure during the period
of enamel formation leads to discoloration of the teeth, ranging from chalky white enamel (mild) (Item
C213) to a pitted, brown appearance (severe), but it does not result in tooth erosion. Grey-brown to
yellow discoloration of teeth occurs in children exposed to multiple courses of tetracycline, but such
findings have not been associated with amoxicillin use. Enamel hypoplasia is typically found in preterm
infants; children experiencing nutritional deficiencies (eg, rickets); or those who have genetic, metabolic,
or endocrine disorders. In a well child, enamel hypoplasia would not be expected to contribute to the
formation of dental caries.

American Board of Pediatrics Content Specification(s):


Know the usual clinical picture of "nursing bottle" caries

Suggested Reading:
Berkowitz RJ, Den Besten PK, Karp JM. Prevention of dental caries. In: McInerny TK, Adam HM,
Campbell DE, Kamat DM, Kellehr KJ, Hoekelman RA, eds. American Academy of PediatricsTextbook of
Pediatric Care. Elk Grove Village, IL: American Academy of Pediatrics; 2009:293-296

Edelstein BL. Solving the problem of early childhood caries: a challenge for all. Arch Pediatr Adolesc
Med. 2009;163:667-668

Section on Pediatric Dentistry and Oral Health. Preventive oral health intervention for pediatricians.
Pediatrics. 2008;122:1387-1394. DOI: 10.1542/peds.2008-2577. Available at:
http://pediatrics.aappublications.org/cgi/content/full/122/6/1387

Twetman S. Prevention of early childhood caries (ECC) review of literature published 1998-2007. Eur
Arch Paediatr Dent. 2008;9:12-18. Abstract available at: http://www.ncbi.nlm.nih.gov/pubmed/18328233

Copyright 2012 American Academy of Pediatrics


2012 PREP SA ON CD-ROM

Critique 213

(Courtesy of M Rimsza)
Fluorosis is characterized by chalky white areas (arrows) most notable on the central incisors.

Copyright 2011 American Academy of Pediatrics


2012 PREP SA ON CD-ROM

Question 214
You are seeing a 14-month-old boy for a follow-up visit for anemia. When he was 12 months old,
his hemoglobin value was 10 g/dL (100 g/L). Further testing at 13 months revealed a mean corpuscular
volume of 62 fL on complete blood count, with the remainder of the red blood cell indices and the smear
within normal limits. Reticulocyte count was 0.5% (0.005). When you review the childs diet, his mother
reports that he drinks 40 oz of cow milk per day and his favorite food is macaroni and cheese. He eats
little meat and few vegetables.
Of the following, the BEST recommendation for management of this childs condition is to
A. encourage replacement of some cow milk with fruit juice
B. prescribe a chewable multivitamin with iron tablet once daily
C. prescribe therapeutic ferrous sulfate drops three times daily
D. resume use of infant formula until his hemoglobin increases
E. switch from cow milk to a 30 kcal/oz cow milk-based nutritional supplement

Copyright 2012 American Academy of Pediatrics


2012 PREP SA ON CD-ROM

Critique 214 Preferred Response: C


The boy described in the vignette has a low mean corpuscular volume and hemoglobin value
consistent with iron deficiency. The most common cause of iron deficiency in this age group is inadequate
dietary intake of iron due to a lack of iron-rich foods in the diet. Cow milk contains little bioavailable iron.
Furthermore, drinking large quantities of cow milk in infancy may be associated with significant intestinal
blood loss.
The most appropriate management of this boys anemia is the administration of ferrous sulfate
drops three times daily. After 2 to 4 weeks of therapy, a repeat complete blood count and reticulocyte
count should be performed to determine if he is responding to therapy. If so, the ferrous sulfate should be
continued for 3 months to replenish iron stores. If he does not respond to this therapy, further evaluation
is indicated.
A chewable multivitamin with iron is intended to provide the minimum daily requirement for dietary
intake; it contains inadequate iron to treat deficiency. Similarly, infant formula and cow milk-based
supplements do not contain sufficient iron to treat deficiency. Substituting juice for milk may reduce
gastrointestinal blood loss from cow milk protein exposure, but it does not provide adequate protein and
nutrients for a healthy diet. Indeed, excessive intake of fruit juice is associated with the development of
obesity and can cause diarrhea.
Infant formulas contain enough iron to meet the routine needs of growing infants younger than 6
months of age. In the past, gastrointestinal disturbances, especially constipation and colic, were
mistakenly attributed to the presence of iron in infant formula, and low-iron formulas were made available.
Use of such formulas may have contributed to iron deficiency in infants who required iron to recover from
physiologic anemia of infancy as well as to provide adequate iron for central nervous system health.
Infant cereals have been recommended as a source of extra iron for older infants, but recent concern
about the relative nutritional emptiness and high simple carbohydrate content of cereals makes this
recommendation somewhat obsolete. New recommendations include introduction of meat into the diet at
6 months and more attention to vegetable sources of iron.
Prior recommendations to delay the introduction of meat into the infant diet in the United States
also may have contributed to the risk for iron deficiency in older infants. Recent evidence points to
introduction of meat as early as age 4 months, a practice common in other parts of the world, as a means
of increasing iron in the diet that is associated with no additional risk for food allergy or other adverse
outcomes.
In November of 2010, the American Academy of Pediatrics released new recommendations for
iron supplementation of infants and children. A new emphasis is placed on ensuring adequate
supplementation of exclusively breastfed infants who are not receiving complementary feedings of iron-
rich foods after age 4 months. Breastfed babies should be supplemented with 1 mg/kg per day of iron
starting at age 4 months.

American Board of Pediatrics Content Specification(s):


Know that dietary deficiency is the most common cause of iron deficiency anemia in young children
Know that cow milk contains very little bio-available iron and that an infant with iron deficiency often
drinks large amounts of cow milk
Judge the nutritional adequacy of infant formulas in relation to mineral content

Suggested Reading:
American Academy of Pediatrics. Iron. In: Kleinman RE, ed. Pediatric Nutrition Handbook. 6th ed. Elk
Grove Village, IL: American Academy of Pediatrics; 2009:403-422

Baker RD, Greer FR, The Committee on Nutrition. Diagnosis and prevention of iron deficiency and iron-
deficiency anemia in infants and young children (0-3 years of age). Pediatrics. 2010;126:1040-1050. DOI:
10.1542/peds.2010-2576. Available at: http://pediatrics.aappublications.org/cgi/content/full/126/5/1040

Greer FR, Sicherer SH, Burks AW, and the Committee on Nutrition and Section on Allergy and
Immunology. Effects of early nutritional interventions on the development of atopic disease in infants and
children: the role of maternal dietary restriction, breastfeeding, timing of introduction of complementary

Copyright 2012 American Academy of Pediatrics


2012 PREP SA ON CD-ROM

foods, and hydrolyzed formulas. Pediatrics. 2008;121:183-191. DOI: 10.1542/peds.2007-3022. Available


at: http://pediatrics.aappublications.org/cgi/content/full/121/1/183

Martins S, Logan S, Gilbert RE. Iron therapy for improving psychomotor development and cognitive
function in children under the age of three with iron deficiency anaemia. Cochrane Database Syst Rev.
2001,;2:CD001444. DOI: 10.1002/14651858.CD001444. Available at:
http://onlinelibrary.wiley.com/o/cochrane/clsysrev/articles/CD001444/frame.html

Pak-Gorstein S, Haq A, Graham EA. Cultural influences on infant feeding practices. Pediatr Rev.
2009;30:e11-e21. DOI: 10.1542/pir.30-3-e11. Available at:
http://pedsinreview.aappublications.org/cgi/content/full/30/3/e11

Soliman AT, Al Dabbagh MM, Habboub AH, Adel A, Humaidy NA, Abushahin A. Linear growth in children
with iron deficiency anemia before and after treatment. J Trop Pediatr. 2009;55:324-327. DOI:
10.1093/tropej/fmp011. Available at: http://tropej.oxfordjournals.org/content/55/5/324.long

Wu AC, Lesperance L, Bernstein H. Screening for iron deficiency. Pediatr Rev. 2002;23:171-178. DOI:
10.1542/pir.23-5-171. Available at: http://pedsinreview.aappublications.org/cgi/content/full/23/5/171

Copyright 2012 American Academy of Pediatrics


2012 PREP SA ON CD-ROM

Question 215
You are called to the delivery room to assess a newborn who has a heart rate of 60 beats/min.
On physical examination, you determine that the infant is in no distress and has no other findings of note
apart from the profound bradycardia. You recommend observation in the neonatal intensive care unit and
order 12-lead electrocardiography (Item Q215). Upon questioning the mother, you learn that she has
Sjgren syndrome.
Of the following, the MOST likely nature of the rhythm disturbance experienced by the infant is
A. blocked premature atrial contractions
B. first-degree atrioventricular block
C. second-degree atrioventricular block
D. sinus bradycardia
E. third-degree atrioventricular block

Copyright 2012 American Academy of Pediatrics


2012 PREP SA ON CD-ROM

Question 215

(Courtesy of M Lewin)
Lead I from the electrocardiogram of the infant described in the vignette.

Copyright 2012 American Academy of Pediatrics


2012 PREP SA ON CD-ROM

Critique 215 Preferred Response: E


The maternal autoimmune disorders systemic lupus erythematosus and Sjgren syndrome are
associated with the development of neonatal heart block, which is caused by the inflammatory response
to circulating autoantibodies upon the fetal cardiac conduction system that results in damage to
atrioventricular conduction. Current obstetric recommendations in maternal lupus and Sjgren syndrome
call for weekly assessment of fetal cardiac conduction between 18 and 32 weeks gestation (the time of
highest risk) and administration of corticosteroids if conduction disturbances are identified. However, the
efficacy of this approach is not entirely validated.
Although a small subset of neonates who have complete (third-degree) atrioventricular block
experience symptoms when ventricular dysfunction occurs (typically absent when the newborn heart rate
is greater than 60 beats/min), most are symptom-free, such as the newborn described in the vignette.
These infants require no pharmacologic therapy. Pacemaker implantation is reserved for those who
develop signs and symptoms of a bradycardia-induced cardiomyopathy. The findings in complete heart
block that results in ventricular dysfunction are indistinguishable from those of any other cause for
cardiomyopathy. They include fatigue, poor feeding, diaphoresis, poor peripheral perfusion, irritability,
rales on physical examination of the lungs, a gallop rhythm on cardiac auscultation, and hepatomegaly.
As noted for this newborn, the electrocardiographic findings diagnostic of complete heart block
include bradycardia for age and lack of correlation between the P wave and QRS complex. Because the
impulse is blocked, an accessory junctional pacemaker typically activates the ventricles. This is known as
a junctional or ventricular escape rhythm. Because this accessory pacemaker also activates
independently of the impulse generated at the sinoatrial node, two independent rhythms can be noted on
the tracing:
The P waves with a regular P-to-P interval represent the first rhythm.
The QRS complexes with a regular R-to-R interval represent the second rhythm (Item C215A).
The PR interval varies because the hallmark of complete heart block is no apparent relationship
between P waves and QRS complexes.
Although blocked premature atrial contractions can appear similar to complete heart block on
auscultation (ie, a regular bradycardic rate), the electrocardiographic tracing appears vastly different. The
tracing of a patient who has blocked premature atrial contractions demonstrates early P waves after the
previous sinus beat, the P waves do not have an accompanying QRS complex, and the P-wave axis
differs between sinus and nonconducted beats. First-degree atrioventricular block is defined by a PR
interval greater than normal for age (normal ranges from 100 to 160 msec in the infant and 140 to 180
msec in the adolescent/adult) (Item C215B). In addition, first-degree atrioventricular block is not
associated with bradycardia. Second-degree atrioventricular block has two forms: type 1 and type 2. In
both types, a P wave is blocked from initiating a QRS complex. Type 1 second-degree atrioventricular
block is also known as Mobitz I or Wenckebach periodicity. Mobitz I heart block is characterized by
progressive prolongation of the PR interval on electrocardiography on consecutive beats followed by a
blocked P wave (ie, a dropped QRS complex) (Item C215C). After the dropped QRS complex, the PR
interval resets and the cycle repeats. This is almost always a benign condition for which no specific
treatment is needed. Type 2 second-degree atrioventricular block or Mobitz II is almost always a disease
of the distal conduction system (His-Purkinje system). Mobitz II heart block is characterized on
electrocardiography by intermittently nonconducted P waves not preceded by PR prolongation and not
followed by PR shortening (Item C215D). The medical significance of this type of atrioventricular block is
that it may progress rapidly to complete heart block. Sinus bradycardia is a defined by bradycardia for
age with a normal PR interval and an appropriate conduction pattern between each P wave and its
accompanying QRS complex.

American Board of Pediatrics Content Specification(s):


Recognize complete heart block from the findings on electrocardiography and physical examination

Suggested Reading:
Doniger SJ, Sharieff GQ. Pediatric dysrhythmias. Pediatr Clin North Am. 2006;53:85-105. DOI:
10.1016/j.pcl.2005.10.004. Abstract available at: http://www.ncbi.nlm.nih.gov/pubmed/16487786

Copyright 2012 American Academy of Pediatrics


2012 PREP SA ON CD-ROM

Mevorach D, Elchalal U, Rein AJ. Prevention of complete heart block in children of mothers with anti-
SSA/Ro and anti-SSB/La autoantibodies: detection and treatment of first-degree atrioventricular block.
Curr Opin Rheumatol. 2009;21:478-482. DOI: 10.1097/BOR.0b013e32832ed817. Available at:
http://www.ncbi.nlm.nih.gov/pubmed/19584727

Copyright 2012 American Academy of Pediatrics


2012 PREP SA ON CD-ROM

Critique 215

(Courtesy of M Lewin)
Complete heart block is characterized by regularly occurring P waves (arrows) that do not correlate with
the QRS complex. In this tracing the second and fourth P waves are buried within the QRS complex.

Copyright 2011 American Academy of Pediatrics


2012 PREP SA ON CD-ROM

Critique 215

(Courtesy of M Lewin)
Portion of an electrocardiographic tracing in first-degree atrioventricular block resulting from acute
rheumatic fever, which illustrates prolongation of the PR interval.

Copyright 2011 American Academy of Pediatrics


2012 PREP SA ON CD-ROM

Critique 215

(Courtesy of M Lewin)
Rhythm strip in second-degree heart block Mobitz type I (Wenckebach) showing progressive prolongation
of the PR interval followed by a nonconducted P wave and dropped QRS complex.

Copyright 2011 American Academy of Pediatrics


2012 PREP SA ON CD-ROM

Critique 215

(Courtesy of M Lewin)
Rhythm strip in second-degree heart block Mobitz type II showing intermittently nonconducted P waves
and dropped QRS complexes. However, unlike in Mobitz type I second-degree heart block, the PR
interval is constant (not progressively increasing).

Copyright 2011 American Academy of Pediatrics


2012 PREP SA ON CD-ROM

Question 216
A 6-month-old girl presents to the urgent care clinic with vomiting, fussiness, and head tilt. She
had a similar episode 1 month ago. She was born at term with no complications and has been otherwise
healthy. She is a good eater and has no diarrhea or other gastrointestinal problems. Developmentally she
has been on track and currently she sits independently. On physical examination, her head shape is
normal and the circumference is 43 cm. Growth parameters are otherwise normal. The pale, fussy infant
has a soft abdomen with normal bowel sounds. When placed in a sitting position, her head tilts to the left.
Sternocleidomastoid muscles feel symmetric in size. After you straighten her head and neck, she
resumes the tilted posture. Her eyes are normally aligned, visual tracking is normal and full, there is no
nystagmus, and facial movements are symmetric during crying. Tone in vertical suspension is normal, as
are reflexes. Muscle bulk is also normal, with no distal muscle wasting or fasciculations.
Of the following, the condition that BEST explains this constellation of findings is
A. brainstem glioma
B. congenital muscular torticollis
C. dystonia musculorum deformans
D. left fourth nerve palsy
E. paroxysmal torticollis of infancy

Copyright 2012 American Academy of Pediatrics


2012 PREP SA ON CD-ROM

Critique 216 Preferred Response: E


The girl described in the vignette presents with head tilt that developed in infancy. The
appearance of this phenomenon, termed torticollis, is generally both head tilting to one side and rotation
of the chin toward the other side. In this case, the episodic history (the torticollis comes and goes) and
normal comprehensive physical and neurological examination findings are reassuring. The most likely
explanation is paroxysmal torticollis of infancy, an idiopathic neurologic condition of infancy characterized
by bouts of torticollis that last for hours or days. The cause is unknown, and the prognosis is for continued
episodes, with waning after age 2 and resolution by age 3 years. Mild delays in fine and gross motor skills
are common, as is a family history of migraine. There is no accepted medical treatment or therapy.
A fixed torticollis in infancy is also generally benign. However, because a head tilt or torticollis can
result from a variety of serious and even life-threatening conditions, the evaluation needs to be
systematic. Congenital muscular torticollis presents in early infancy with a consistent, not paroxysmal
head tilt. Generally, there is an imbalance in the tone or activation of the sternocleidomastoid muscle,
leading to palpable hypertrophy or contracture of this muscle in about 50% of cases. The head tilt directs
toward and the chin rotation directs away from the affected sternocleidomastoid muscle. Sometimes such
a persistent head tilt results in some positional molding of the skull bones. Treatment involves physical
therapy. Congenital muscular torticollis is also found in a variety of congenital spinal malformations.
A tumor infiltrating the brainstem can affect multiple cranial nerves as well as motor and sensory
pathways. The paroxysmal history and absence of any abnormalities in eye movement for this girl is
reassuring and makes brainstem neoplasms unlikely. Dystonia musculorum deformans is an autosomal
dominant form of dystonia (DYT 1: MIM ID #128100) that can include cervical dystonia. When it presents
in childhood, it begins in older children, typically localizing in a limb before generalizing to other body
areas, and the time course is not paroxysmal. Lesions of the fourth cranial nerve (the trochlear nerve) can
present with head tilt. The fourth cranial nerve innervates the inferior oblique eye muscle. Activating this
nerve rotates the eye inward (intorsion). A palsy in the left fourth nerve results in external rotation of the
left eye that is not paroxysmal. In addition, to compensate for this, a head tilt to the right, not left, would
be observed.

American Board of Pediatrics Content Specification(s):


Recognize that the differential diagnosis of torticollis includes head tilt secondary to malformation of
the cervical spine, visual disturbance, posterior fossa tumor, etc.
Differentiate between congenital and paroxysmal torticollis

Suggested Reading:
Carlo WA. The newborn. In: Kliegman RM, Stanton BF, St. Geme JW III, Schor NF, and Behrman RE,
eds. Nelson Textbook of Pediatrics. 19th ed. Philadelphia, PA: Saunders Elsevier; 2011:532-540

Mikati MA, Obeid M. Conditions that mimic seizures. In: Kliegman RM, Stanton BF, St. Geme JW III,
Schor NF, and Behrman RE, eds. Nelson Textbook of Pediatrics. 19th ed. Philadelphia, PA Saunders
Elsevier; 2011:2039

Rosman NP, Douglass LM, Sharif UM, Paolini J. The neurology of benign paroxysmal torticollis of
infancy: report of 10 new cases and review of the literature. J Child Neurol. 2009;24:155-160. Abstract
available at: http://www.ncbi.nlm.nih.gov/pubmed/19182151

Spiegel DA, Dormans JP. The neck. In: Kliegman RM, Stanton BF, St. Geme JW III, Schor NF, and
Behrman RE, eds. Nelson Textbook of Pediatrics. 19th ed. Philadelphia, PA: Saunders Elsevier;
2011:2377-2379

Copyright 2012 American Academy of Pediatrics


2012 PREP SA ON CD-ROM

Question 217
You are seeing a new patient for a 2-month health supervision visit. According to medical records
that the mother brings with her, the infant was born at term, weighing 7 lb 2 oz. In the newborn period she
had no episodes of hypoglycemia and was discharged home with her mother at 2 days of age. On
physical examination, you note right-sided hemihyperplasia, with increased girth and length of her right
arm and leg as well as mild overgrowth of the right half of the thorax and abdomen. You find no evidence
of underlying subcutaneous vascular defects; there is no macroglossia, umbilical hernia, or ear lobe
creases that are common in children who have Beckwith-Wiedemann syndrome.
Of the following, the MOST important screening test for this child is
A. chest radiography
B. fasting blood glucose
C. limb radiography
D. renal ultrasonography
E. urinary catecholamine screening

Copyright 2012 American Academy of Pediatrics


2012 PREP SA ON CD-ROM

Critique 217 Preferred Response: D


The infant described in the vignette presents with isolated right-sided overgrowth or
hemihyperplasia (the preferred term over hemihypertrophy). The recommendation for screening in cases
of isolated hemihyperplasia includes renal ultrasonography every 3 months through age 8 years (some
recommend through puberty) to identify a renal tumor before it becomes clinically apparent. In the
absence of features of an underlying vascular condition such as Klippel-Trenaunay-Weber syndrome
(KTWS) or features suggestive of Beckwith-Wiedemann syndrome (BWS), the most important potential
complication is the development of Wilms tumor, which may be found in nearly 6% of cases of isolated
hemihyperplasia.
Screening for neuroblastomas with periodic chest radiographs and measurement of urinary
catecholamines has been recommended by some experts for children diagnosed with BWS but is not
indicated for children who have isolated hemihyperplasia. Fasting blood glucose screening also is not
indicated in the absence of features of BWS. Limb radiography to assess limb length discrepancies is not
indicated at this time, because orthopedic intervention is best undertaken closer to or during puberty,
when it is feasible to project anticipated bone growth.
Additional features suggestive of BWS include: polyhydramnios, somatic overgrowth at birth
(macrosomia), macroglossia, visceromegaly, umbilical hernia/omphalocele, ear lobe creases or posterior
helical pits, and neonatal hypoglycemia. The risk for Wilms tumor in infants and children who have BWS
is between 5% and 7%, with the diagnosis seen in most before the age of 4 years and 95% diagnosed by
age 8 years. In addition to Wilms tumor, children who have BWS are at increased risk for other embryonal
tumors, including hepatoblastomas and neuroblastomas. Additional screening recommendations are for
measurement of alpha-fetoprotein in the first few years after birth to detect early hepatoblastomas.
Less common overgrowth syndromes that also are associated with increased risks for tumors
include Simpson-Golabi-Behmel syndrome and Perlman syndrome. However, some overgrowth
syndromes (eg, Costello and Sotos syndromes) are not associated with a significant increased risk for
tumors. Children who have KTWS are at high risk for hemihyperplasia due to increased vascular supply
to affected body regions caused by localized capillary-lymphatic-venous malformations. This entity can be
differentiated from BWS or isolated hemihyperplasia by identifying the vascular malformations that have
underlying red/purple discoloration or irregular and palpable vascular elements, with or without superficial
varicosities. If KTWS is present in a child who has secondary hemihyperplasia, Wilms tumor screening is
not indicated because such children do not appear to be at increased risk for any embryonal tumors.

American Board of Pediatrics Content Specification(s):


Recognize that children with hemihypertrophy and somatic overgrowth syndromes should be
periodically evaluated for the development of associated embryonal tumors

Suggested Reading:
Greene AK, Kieran M, Burrows PE, Mulliken JB, Kasser J, Fishman SJ. Wilms tumor screening is
unnecessary in Klippel-Trenaunay syndrome. Pediatrics. 2004;113:e326-e329. Available at:
http://pediatrics.aappublications.org/cgi/content/full/113/4/e326

Shuman C, Beckwith JB, Smith AC, Weksberg R. Beckwith-Wiedemann syndrome. GeneReviews. 2010.
Available at: http://www.ncbi.nlm.nih.gov/bookshelf/br.fcgi?book=gene&part=bws

Copyright 2012 American Academy of Pediatrics


2012 PREP SA ON CD-ROM

Question 218
You are seeing a 16-year-old girl for the first time who complains of amenorrhea. The mothers
menarche was at age 12 years. The girl developed pubic hair at age 11 years and breast buds at age 12
years. She has no other symptoms. The mother reports that the girl eats well and has been active all her
life. Physical examination reveals a height of 57 in, weight of 89 lb, body mass index of 19.3, breast tissue
at Sexual Maturity Rating (SMR) 2, and pubic hair at SMR 4. A urine pregnancy test shows negative
results. Laboratory results include: luteinizing hormone of 10 mIU/mL (normal adult female, 2 to 95
mIU/mL), follicle-stimulating hormone of 42 mIU/mL (normal adult female, 1 to 30 mIU/mL), and prolactin
of 27 ng/mL (normal, 5 to 23 ng/mL).
Of the following, the MOST likely cause of this girls primary amenorrhea is
A. congenital adrenal hyperplasia
B. excessive exercise
C. imperforate hymen
D. prolactinoma
E. Turner syndrome

Copyright 2012 American Academy of Pediatrics


2012 PREP SA ON CD-ROM

Critique 218 Preferred Response: E


The absence of menses or amenorrhea often is broadly classified as primary (no menarche by
age 16 years) or secondary (lack of menses for 3 to 6 months, once initiated). Other than genital tract
obstruction, the causes for primary and secondary amenorrhea may overlap. For example, a girl may
begin sexual activity before the onset of her first period, and pregnancy could be the cause of primary
amenorrhea. The source of amenorrhea might be better assessed by considering causes in different
compartments: the outflow tract, the ovaries, or the hypothalamic-pituitary axis. Imperforate hymen or the
Mayer-Rokitansky-Kster-Hauser syndrome may be the cause in the genital tract outlet. In the ovaries,
the cause may be chromosomally related, as in Turner syndrome, or nonchromosomal, as with damage
to the ovaries such as after radiation or chemotherapy. These entities produce hypergonadotropic
hypogonadism in which the follicle-stimulating hormone (FSH) concentration is high. The cause of
polycystic ovarian syndrome (PCOS), the most common endocrinopathy seen in adolescent females, is
unknown but is believed to be related to insulin action. Finally, the source may lie in the hypothalamic-
pituitary area (resulting in hypogonadotropic hypogonadism with normal or low FSH), often due to stress,
weight loss, excessive exercise, or more rarely, a space-occupying or destructive pituitary lesion. If the
patient has evidence of androgen excess (eg, hirsutism, acne, clitoromegaly) and amenorrhea, adrenal
causes should be sought, in addition to evaluation for PCOS.
The short stature combined with minimal breast development and amenorrhea described for the
girl in the vignette suggests the possibility of Turner syndrome. Her FSH value is high, indicating ovarian
failure. Late-onset congenital adrenal hyperplasia (21-hydroxlase deficiency) causes signs of androgen
excess. Excessive exercise with failure to ingest sufficient calories may cause stunting of growth and
interruption of puberty, but it would not result in ovarian failure. Females who have imperforate hymens
complete pubertal growth and secondary sexual development normally. They present with monthly lower
abdominal pain and may have an enlarged uterus that is palpable above the pubic symphysis.
Examination of the external genitalia should reveal a blue bulging membrane (Item C218). Prolactinomas
are often micro- rather than macroadenomas and, therefore, produce no symptoms or signs of increased
intracranial pressure. With a tumor, the prolactin concentration is usually greater than 100 ng/mL.
Affected patients occasionally present with delayed puberty or secondary amenorrhea with or without
galactorrhea.

American Board of Pediatrics Content Specification(s):


Know that the differential diagnosis of primary amenorrhea includes anatomic variants of the genital
tract, pregnancy, ovarian pathology, adrenal disease, and hypothalamic/pituitary disorders

Suggested Reading:
American Academy of Pediatrics, Committee on Adolescence, American College of Obstetricians and
Gynecologists, and Committee on Adolescent Health Care. Menstruation in girls and adolescents: using
the menstrual period as a vital sign. Pediatrics. 2006;118:2245-2250. DOI: 10.1542/peds.2006-2481.
Available at: http://pediatrics.aappublications.org/cgi/content/full/118/5/2245

Bloomfield D. In brief: secondary amenorrhea. Pediatr Rev. 2006;27:113-114. DOI: 10.1542/pir.27-3-113.


Available at: http://pedsinreview.aappublications.org/cgi/content/full/27/3/113

Committee on Adolescence. Identifying and treating eating disorders. Pediatrics. 2003;111:204-211.


Available at: http://pediatrics.aappublications.org/cgi/content/full/111/1/204

Emans JS. Amenorrhea in the adolescent. In: Emans SJH, Laufer MR, Goldstein DP, eds. Pediatric and
Adolescent Gynecology. 5th ed. Philadelphia, PA: Lippincott Williams & Wilkins, a Wolters Kluwer
business; 2005:214-269

Fleischman A, Gordon CM, Neinstein LS. Menstrual disorders: amenorrhea and polycystic ovary
syndrome. In: Neinstein LS, Gordon CM, Katzman D, Rosen DS, Woods ER, eds. Adolescent Health
Care: A Practical Guide. 5th ed. Philadelphia, PA: Lippincott Williams & Wilkins, a Wolters Kluwer
business; 2008:691-705

Copyright 2012 American Academy of Pediatrics


2012 PREP SA ON CD-ROM

Lerand SJ, Williams JR. In brief: the female athlete triad. Pediatr Rev. 2006;27:e12-e13.
DOI:10.1542/pir.27-1-e12. Available at: http://pedsinreview.aappublications.org/cgi/content/full/27/1/e12

Marinkovic M, Capouya J, Cimino D. Index of suspicion: case 3. Pediatr Rev. 2007;28:63-68. DOI:
10.1542/pir.28-2-63. Available at: http://pedsinreview.aappublications.org/cgi/content/full/28/2/63

Copyright 2012 American Academy of Pediatrics


2012 PREP SA ON CD-ROM

Critique 218

(Reprinted with permission from Torok K, Bhende MS. Index of suspicion. Pediatr Rev. 2008;29:25-30)
Imperforate hymen: note the bulging hymenal tissue.

Copyright 2011 American Academy of Pediatrics


2012 PREP SA ON CD-ROM

Question 219
During a health supervision visit, the mother of 8-year-old girl and 10-year-old boy tells you that
the children have been receiving counseling for anxiety for 3 months. The mother has a history of anxiety,
and she has responded well to sertraline. She reports that the boy is not making much progress in
therapy and asks you if a medication trial would be appropriate for him.
Of the following, the MOST appropriate initial response is to
A. advise the mother to give the current therapist more time to see if an improvement occurs
B. ask the boy if he wants a new therapist
C. obtain consent from the mother to contact the therapist
D. prescribe sertraline for the boy
E. refer the family to another therapist

Copyright 2012 American Academy of Pediatrics


2012 PREP SA ON CD-ROM

Critique 219 Preferred Response: C


Significant advances have been made in the assessment and treatment of anxiety disorders in
children and adolescents. Evidence-based practices are now available that can help to address anxiety
disorders in a timely fashion. However, the specific intervention depends on the anxiety disorder that is
present. For example, a child who has posttraumatic stress disorder might benefit from trauma-focused
cognitive behavior therapy, but the child who has school phobia might require a different approach, such
as a graded exposure plan for reintroduction and accommodation to being in school. Characterizing the
type of anxiety might be assisted by the use of anxiety-specific rating scales, such as the Screen for Child
Anxiety Related Emotional Disorders (SCARED). For the boy described in the vignette, the nature of the
anxiety disorder has not been defined. Accordingly, the clinician should contact the therapist to clarify the
diagnosis and the type and goals of therapy that is being used for this child. Recommending that a family
continue with a therapist without first determining that they are receiving treatment that is appropriate for
their childs issues is not fair to the patient or family.
Liking a therapist can be a factor in the effectiveness of psychotherapy, but a childs feelings
regarding the therapy may simply mean that the therapist is pushing for necessary changes that are
uncomfortable. Doing exposure work for fears in anxiety treatment, for example, can be temporarily
uncomfortable and places a new demand on the familys home time.
Starting sertraline because the family perceives that the child is not making progress may not be
indicated because it is unclear if this boy has an anxiety disorder and if so, whether it is being treated
appropriately. Beginning a selective serotonin reuptake inhibitor simultaneously with starting therapy is
appropriate for children who have moderate severe anxiety disorders and for children who have not
responded to an adequate trial of appropriate counseling therapy alone.
Referring the family to another therapist before communicating with the current therapist is not
appropriate. Changing therapists may undercut the hard work of a competent therapist using appropriate
treatments. Being familiar with community mental health resources can aid the pediatrician in making
successful referrals and obtaining follow-up information on their patients.

AAP Mental Health Competency:


Know how to counsel a parent/child that are doubting the effectiveness of a current counseling
therapy

Suggested Readings:
American Academy of Pediatrics Task Force on Mental Health. Anxiety Cluster Guidance. Addressing
Mental Health Concerns in Primary Care A Clinicians Toolkit (CD-ROM). Elk Grove Village, IL: American
Academy of Pediatrics; 2010.

Chansky TE. Freeing Your Child From Anxiety: Powerful, Practical Solutions To Overcome Your Childs
Fears, Worries, and Phobias. New York, NY: Broadway Books; 2004

Connolly SD, Bernstein GA; Work Group on Quality Issues. Practice parameter for the assessment and
treatment of children and adolescents with anxiety disorders. J Am Acad Child Adolesc Psychiatry.
2007;46:267-283. Abstract available at: http://www.ncbi.nlm.nih.gov/pubmed/17242630

Copyright 2012 American Academy of Pediatrics


2012 PREP SA ON CD-ROM

Question 220
You are evaluating an 18-month-old boy who was pulled from a swimming pool (water
temperature of 15.0C) after an estimated 5 minutes of submersion. Cardiopulmonary resuscitation (CPR)
was initiated at the scene, and the boy was transported to the emergency department while undergoing
advanced cardiac life support. Spontaneous circulation was restored in the emergency department after a
total of 20 minutes of CPR. On initial physical examination, the boy was comatose, with mildly reactive
pupils and abnormal flexion with painful stimuli. He had no spontaneous eye opening. He has now been
in the pediatric intensive care unit receiving mechanical ventilation and maximal intensive care support for
12 hours. At this point, he exhibits briskly reactive pupils, spontaneous eye opening, and purposeful
withdrawal from pain.
Of the following, the prognostic factor that is BEST associated with a favorable neurologic
outcome for this boy is
A. establishment of a perfusing rhythm in the emergency department
B. improvement in neurologic examination findings within 24 hours
C. submersion time of less than 20 minutes
D. total CPR time of less than 30 minutes
E. water temperature of less than 18.0C

Copyright 2012 American Academy of Pediatrics


2012 PREP SA ON CD-ROM

Critique 220 Preferred Response: B


Unintentional injuries are the leading cause of death in children (ages 1 to 19 years) in the United
States, with drowning second only to motor vehicle-related deaths as the cause of these injuries. In 2002,
the World Congress on Drowning and the World Health Organization revised the definition of drowning as
the process of experiencing respiratory impairment from submersion/immersion in liquid. It was also
recommended that drowning outcomes be classified as death, morbidity, or no morbidity and that the
term near-drowning no longer be used. Morbidity can be further classified as moderately disabled,
severely disabled, vegetative state/coma, and brain death.
About 80% of pediatric drowning victims survive, with more than 90% experiencing no morbidity.
For those who require care in the pediatric intensive care unit, approximately 50% survive with no
morbidity, 25% survive with morbidity, and 25% die. Prognostic factors to guide both resuscitation
duration and family counseling have proven difficult. In general, outcome is related to duration of
drowning, with intact survival rarely reported after 25 minutes of submersion, except in icy water (defined
as water temperature of 0 to 5C). Factors associated with poor survival include length of submersion
greater than 10 minutes, need for ongoing CPR in the emergency department, CPR of more than 25
minutes, Glasgow Coma Scores of 3 to 5 on admission to the hospital, and the presence of
hyperglycemia. For the boy described in the vignette, the factor best associated with a favorable
neurologic outcome is the presence of spontaneous and purposeful movements within 24 hours of the
drowning episode.

American Board of Pediatrics Content Specification(s):


Understand the factors that predict the prognosis in a patient who has had a near-drowning episode

Suggested Reading:
Brenner RA, Committee on Injury, Violence, and Poison Prevention. Prevention of drowning in infants,
children, and adolescents-technical report. Pediatrics. 2003;112;440-445. Available at:
http://pediatrics.aappublications.org/cgi/content/full/112/2/440

Committee on Injury, Violence, and Poison Prevention. Prevention of drowning in infants, children, and
adolescents-policy statement. Pediatrics. 2003;112;437-439. Available at:
http://pediatrics.aappublications.org/cgi/content/full/112/2/437

Meyer RJ, Theodorou AA, Berg RA. Childhood drowning. Pediatr Rev. 2006;27;163-169. DOI:
10.1542/pir.27-5-163. Available at: http://pedsinreview.aappublications.org/cgi/content/full/27/5/163

Shephard E, Quan L. Drowning and submersion injury. In: Kliegman RM, Stanton BF, St. Geme JW III,
Schor NF, and Behrman RE, eds. Nelson Textbook of Pediatrics. 19th ed. Philadelphia, PA: Saunders
Elsevier; 2011:341-348

Copyright 2012 American Academy of Pediatrics


2012 PREP SA ON CD-ROM

Question 221
A 12-year-old boy presents for evaluation of short stature. His mother reports that he has been
growing but that he is not keeping up with the growth rates of his peers. His mother is 160 cm tall (~63 in)
and his father is 172 cm tall (~68 in). With the exception of his relative short stature, his medical history is
unremarkable, and he takes no medications. His growth curve is shown in image (Item Q221). A bone
age radiograph demonstrates a skeletal maturity of 10 years. The boy asks if he will always be short.
You advise him that the best way to estimate his potential for growth is to calculate his midparental target
height and compare it to his current height and his skeletal maturity.
Of the following, the BEST estimate of his midparental target height is
A. 162.5 cm (64 in)
B. 167.5 cm (66 in)
C. 172.5 cm (68 in)
D. 177.5 cm (70 in)
E. 182.5cm (72 in)

Copyright 2012 American Academy of Pediatrics


2012 PREP SA ON CD-ROM

Question 221

(Courtesy of M Haller)

Copyright 2012 American Academy of Pediatrics


2012 PREP SA ON CD-ROM

Critique 221 Preferred Response: C


Midparental height is an excellent tool for estimating genetic potential for linear growth if both of
the biological parents heights are known. Because parents often incorrectly report their own heights,
measuring them can be useful in ensuring an accurate estimate. The midparental height (in inches) is
calculated by adding the mothers height and fathers height; adding 5 in for a male child or subtracting 5
in for a female child; and dividing that value by 2. In centimeters, the calculation is performed by adding or
subtracting 12.5 cm before dividing by 2.
The mother of the boy described in the vignette has a height of 63 in and the fathers height is 68
in. Therefore, the midparental height for the boy is (63 + 68 + 5)/2 = 68 in. In centimeters, the calculation
is (160 + 172 +12.5)/2 = 172.5 cm.
Estimation of midparental height can be used to determine if growth is occurring outside of
genetically predicted norms. If the childs current height percentile is followed along the growth curve until
the age of 18, the height should be within 2 standard deviations of the midparental height prediction (2 in
or 5 cm).

American Board of Pediatrics Content Specification(s):


Recognize the methods for assessing the relationship of parental stature to an individual adolescent's
stature

Suggested Reading:
Kemp SF, Frindik JP. Disorders of growth. In: Sarafoglou K, Hoffmann G, Roth K, eds. Pediatric
Endocrinology and Inborn Errors of Metabolism. New York, NY: McGraw-Hill Professional; 2009:441-476

Rosenfeld RG, Cohen P. Disorders of growth hormone/insulin like growth factor secretion and action. In:
Sperling MA, ed. Pediatric Endocrinology. 3rd ed. Philadelphia, PA: Saunders; 2008:254-334

Copyright 2012 American Academy of Pediatrics


2012 PREP SA ON CD-ROM

Question 222
A 17-year-old boy comes to your office for medication management of his attention-
deficit/hyperactivity disorder (ADHD). He explains that he is considering stopping his medication before
his last year in high school. His parents are upset because they are fearful that his academic success will
diminish and that he may make poor social choices. His parents ask about the long-term outcome for
ADHD.
Of the following, the MOST appropriate response is that
A. certain features of ADHD (risk taking, fast-paced approach, outgoing style) may be
advantageous in some occupations
B. longitudinal studies have not found elevated anxiety or mood disorders among adults who have
ADHD
C. males who have ADHD have a greater ability to handle stressful situations
D. more than 75% of children who have ADHD no longer have inattention or have a need for
stimulant medications in adulthood
E. studies have not found a higher rate of divorce among adults who have ADHD

Copyright 2012 American Academy of Pediatrics


2012 PREP SA ON CD-ROM

Critique 222 Preferred Response: A


There is good evidence of the existence of attention-deficit/hyperactivity (ADHD) in adults; 75% to
85% of affected children continue to have symptoms of impulsivity and low attention into teenage years
and adulthood. Individuals who have ADHD show decreased hyperactivity as they get older, but
adolescents may have more immature interactions with their peers. Longitudinal studies have found
elevated anxiety or mood disorders among adults who have ADHD, and males have decreased ability to
handle stress. Studies have found higher rates of divorce among adults who have ADHD compared with
the general population.
However, many individuals who have ADHD can compensate for their weaknesses and attain
above-average achievement. For example, although adults report more issues with having speeding
tickets and less satisfactory job performance, the young man described in the vignette may find that his
risk taking, fast-paced approach, and outgoing style may work to his advantage in certain occupations.

American Board of Pediatrics Content Specification(s):


Know the long-term outcome for children with ADHD as adolescents and adults

Suggested Reading:
American Psychiatric Association. Diagnostic criteria for ADHD. In: Diagnostic and Statistical Manual of
Mental Disorders. 4th ed. Text revision. Washington, DC: American Psychiatric Association; 2000:85-94

Reiff MI, Stein MR. Attention-deficit/hyperactivity disorder. In: Voight RG, Macias MM, Myers SM, eds.
American Academy of Pediatrics Developmental and Behavioral Pediatrics. Elk Grove Village, IL:
American Academy of Pediatrics; 2011:327-348

Wilms Floet AM, Scheiner C, Grossman L. Attention deficit/hyperactivity disorder. Pediatr Rev.
2010;31:56 69. DOI: 10.1542/pir.31-2-56. Available at:
http://pedsinreview.aappublications.org/cgi/content/full/31/2/56

Copyright 2012 American Academy of Pediatrics


2012 PREP SA ON CD-ROM

Question 223
A 3-year-old boy presents with a temperature of 39.5C and a first-time generalized seizure. Over
the past 3 days, he has had 8 to 10 loose, liquid stools and abdominal pain. He attends child care, and
several other children in the center have been reported to have diarrhea.
Of the following, the MOST likely cause of this childs illness is infection with
A. Campylobacter jejuni
B. rotavirus
C. Salmonella enteritidis
D. Salmonella typhi
E. Shigella flexneri

Copyright 2012 American Academy of Pediatrics


2012 PREP SA ON CD-ROM

Critique 223 Preferred Response: E


Acute onset of fever, abdominal cramps, and diarrhea (often with blood and mucus) over 3 days,
as described for the boy in the vignette, are suggestive of a bacterial gastroenteritis. The associated
seizure and attendance at a child-care center support the diagnosis of Shigella infection.
Campylobacter jejuni infection can present with fever and similar gastrointestinal symptoms but
has not been associated with seizures. In addition, Campylobacter is typically transmitted from
contaminated poultry; outbreaks in child-care centers are uncommon.
Nontyphoidal Salmonella infections, such as S enteritidis, are acquired from animal reservoirs,
including poultry and livestock. Transmission from contaminated meats and eggs is the usual source of
outbreaks, although other foods, including ice cream, fruits, and cider, have been implicated. Secondary
person-to-person transmission can occur. Transmission of nontyphoidal Salmonella from contact with pet
reptiles is another important source of infections. Infection may range from asymptomatic to
gastroenteritis with diarrhea, abdominal cramps, and fever. Seizures are not commonly associated with
these infections. Salmonella bacteremia can occur in younger children (<1 year old).
S typhi is a solely human pathogen that is rare in the United States. Infection is associated with
crowding and poor hygienic conditions. Most cases in the United States are acquired during international
travel. Typhoid fever is a protracted illness characterized by fever, constitutional symptoms, abdominal
pain and tenderness, hepatosplenomegaly, rose spots on the skin, and changes in mental status.
Rotavirus infection is transmitted by the fecal-oral route and can be found on multiple surfaces in
child-care centers. Clinically, the illness is characterized by the acute onset of fever and vomiting, with
watery diarrhea developing 24 to 48 hours later. Severe infection generally occurs in children 2 years of
age and younger.

American Board of Pediatrics Content Specification(s):


Distinguish between Salmonella and Shigella with regard to pathophysiology and symptomatology

Suggested Reading:
Acheson DWK. Differential diagnosis of microbial foodborne disease. UpToDate Online 18.3. 2010.
Available for subscription at: www.uptodate.com/online/content/topic.do?topicKey=gi_infec/11820

American Academy of Pediatrics. Salmonella infections. In: Pickering LK, Baker CJ, Kimberlin DW, Long
SS, eds. Red Book: 2009 Report of the Committee on Infectious Diseases. 28th ed. Elk Grove Village, IL:
American Academy of Pediatrics; 2009:584-589

American Academy of Pediatrics. Shigella infections. In: Pickering LK, Baker CJ, Kimberlin DW, Long SS,
th
eds. Red Book: 2009 Report of the Committee on Infectious Diseases. 28 ed. Elk Grove Village, IL:
American Academy of Pediatrics; 2009:593-596

Copyright 2012 American Academy of Pediatrics


2012 PREP SA ON CD-ROM

Question 224
You are examining a 2-week-old term infant who was born at home to a mother who had no
prenatal care. On physical examination, you note moderate bilateral conjunctival injection, eye discharge,
and periorbital edema. The infant is afebrile and otherwise doing well. The remainder of the physical
examination findings are normal.
Of the following, the MOST appropriate antibiotic therapy for this infant is
A. amoxicillin
B. doxycycline
C. erythromycin
D. levofloxacin
E. sulfamethoxazole

Copyright 2012 American Academy of Pediatrics


2012 PREP SA ON CD-ROM

Critique 224 Preferred Response: C


The infant described in the vignette is exhibiting characteristics of ophthalmia neonatorum,
conjunctivitis occurring in the first month after birth and most likely caused by Chlamydia trachomatis.
Oral erythromycin therapy for 14 days is recommended; topical therapy is ineffective. Although oral
erythromycin therapy is the treatment of choice, clinicians should be aware that this drug has been
associated with the development of infantile hypertrophic pyloric stenosis. Doxycycline can be used to
treat Chlamydia infections in children older than 7 years of age and adults but is not appropriate therapy
for infants and young children.
C trachomatis is the most common cause of infective conjunctivitis in neonates because of the
high prevalence of maternal genital infections, especially in mothers who have received no prenatal care.
Evaluation of the mother for concomitant infection with Neisseria gonorrhoeae, human immunodeficiency
virus (HIV), syphilis, and hepatitis B is recommended. The transmission rate from untreated mothers who
have C trachomatis infection to infants ranges from 20% to 60%. Infants who have Chlamydia
conjunctivitis typically present with unilateral or bilateral mucopurulent conjunctivitis and erythema and
eyelid swelling in the second week after birth. Diagnosis requires a specimen that contains conjunctival
cells because the pathogen is intracellular. Nucleic acid amplification tests have become the gold
standard for diagnosis. Rapid testing by direct fluorescent stain, enzyme immunoassay, or DNA probe is
sensitive but difficult to standardize. Cultivating the organism in cell culture is tedious and slow.
N gonorrhoeae is the most serious pathogen that can cause ophthalmia neonatorum because of
its ability to damage the eye, but its incidence as a pathogen in this disease has markedly decreased with
the widespread implementation of perinatal ocular prophylaxis. When gonococcal ophthalmia neonatorum
occurs, symptoms usually appear in the first postnatal week. Unilateral or bilateral mucopurulent
conjunctivitis and eyelid erythema and edema can be accompanied by corneal involvement. Corneal
ulceration can lead to scarring and visual impairment. Diagnosis is confirmed by culture of ocular exudate
on chocolate agar. Blood cultures and cerebrospinal fluid cultures (in febrile, ill infants) should be
obtained. Evaluation for concomitant infection with C trachomatis, HIV, syphilis, and hepatitis B is
recommended. A single intramuscular injection of ceftriaxone is the preferred antimicrobial treatment.
These babies should be hospitalized for ophthalmologic management.
Herpes simplex virus (HSV) ophthalmia neonatorum usually occurs in the second postnatal week
and manifests as unilateral or bilateral eyelid edema and conjunctivitis with serous drainage. HSV can
cause severe corneal damage (keratoconjunctivitis) to the neonatal eye. Diagnosis is made by
conjunctival viral culture, and intravenous acyclovir is administered for treatment.
For mothers who do not have sexually transmitted infections, the pathogens that can cause
neonatal conjunctivitis are Staphylococcus aureus, Streptococcus pneumoniae, Haemophilus influenzae,
and viridans streptococci. In hospitalized infants, disease can be caused by gram-negative pathogens
such as Klebsiella pneumoniae, Escherichia coli, and Pseudomonas aeruginosa. Intravenous therapy
directed to the pathogen isolated in culture of the exudate is recommended.
Beyond the neonatal period, acute bacterial conjunctivitis is most often caused by nontypable H
influenzae, S pneumoniae, other streptococci (S mitis), and Moraxella catarrhalis. S aureus and S
epidermidis are more likely to colonize than infect the lids and conjunctivae. N gonorrhoeae conjunctivitis
is rare. Topical antibiotic therapy with agents such as erythromycin, bacitracin-polymyxin, polymyxin-
trimethoprim, or fluoroquinolones (eg, levofloxacin) is recommended to shorten symptoms. Topical
aminoglycosides have no activity against streptococci. Oral antibiotics, such as amoxicillin, may be useful
in the treatment of children who have otitis media together with conjunctivitis. Sulfamethoxazole has no
role in the treatment of conjunctivitis.
Viral conjunctivitis can be caused by adenovirus, picornaviruses (enterovirus and coxsackievirus),
and HSV. Supportive care is recommended for the treatment of most cases of viral conjunctivitis, except
that caused by HSV. HSV eye infections can cause severe damage, leading to vision impairment.
Treatment of HSV conjunctivitis should be undertaken in consultation with an ophthalmologist.

American Board of Pediatrics Content Specification(s):


Know the clinical manifestations of conjunctivitis (influence of age)
Know the treatment of conjunctivitis

Copyright 2012 American Academy of Pediatrics


2012 PREP SA ON CD-ROM

Suggested Reading:
American Academy of Pediatrics. Chlamydia trachomatis. In: Pickering LK, Baker CJ, Kimberlin DW,
Long SS, eds. Red Book: 2009 Report of the Committee on Infectious Diseases. 28th ed. Elk Grove
Village, IL: American Academy of Pediatrics; 2009:255-259

American Academy of Pediatrics. Gonococcal infections. In: Pickering LK, Baker CJ, Kimberlin DW, Long
SS, eds. Red Book: 2009 Report of the Committee on Infectious Diseases. 28th ed. Elk Grove Village, IL:
American Academy of Pediatrics; 2009:305-313

Richards A, Guzman-Cottrill JA. Conjunctivitis. Pediatr Rev. 2010;31:196-208. DOI: 10.1542/pir.31-5-196.


Available at: http://pedsinreview.aappublications.org/cgi/content/full/31/5/196

Weiss AH. Conjunctivitis beyond the neonatal period. In: Long SS, Pickering LK, Prober CG, eds.
Principles and Practice of Pediatric Infectious Diseases. 3rd ed. Philadelphia, PA: Churchill Livingstone
Elsevier, 2008:495-499

Weiss AH. Conjunctivitis in the neonatal period (ophthalmia neonatorum). In: Long SS, Pickering LK,
Prober CG, eds. Principles and Practice of Pediatric Infectious Diseases. 3rd ed. Philadelphia, PA:
Churchill Livingstone Elsevier, 2008:492-494

Copyright 2012 American Academy of Pediatrics


2012 PREP SA ON CD-ROM

Question 225
A 3-week-old term infant presents with vomiting, increased fussiness, and poor weight gain. In
answer to your questioning, his mother states he has a vigorous suck, decreased tearing, and several wet
diapers per day. The infant is formula-fed. On physical examination, he is fussy and difficult to console.
His weight is 2.9 kg (birthweight was 3.2 kg), temperature is 37.3C, heart rate is 180 beats/min,
respiratory rate is 40 breaths/min, and blood pressure is 76/36 mm Hg. His anterior fontanelle is flat and
mucous membranes are tacky. His chest is clear to auscultation, he has tachycardia without murmurs,
and he has no abdominal masses. Skin evaluation reveals reduced turgor, and capillary refill time is
approximately 3 to 4 seconds. Laboratory results include: sodium of 168 mEq/L (168 mmol/L), potassium
of 3.5 mEq/L (3.5 mmol/L), chloride of 130 mEq/L (130 mmol/L), and bicarbonate of 20 mEq/L (20
mmol/L). Urinalysis shows a specific gravity of 1.002, pH of 5.5, and otherwise negative results.
Of the following, the MOST likely diagnosis for this patient is
A. Bartter syndrome
B. diabetes insipidus
C. Fanconi syndrome
D. Gitelman syndrome
E. improper formula preparation

Copyright 2012 American Academy of Pediatrics


2012 PREP SA ON CD-ROM

Critique 225 Preferred Response: B


The infant described in the vignette presents with severe dehydration (300 g below birthweight at
3 weeks of age), good urine output, and hypernatremia with a dilute urine on urinalysis, all of which are
suggestive of decreased urinary concentrating ability, as can occur with a tubulopathy or diabetes
insipidus (DI). Tubulopathies that occur in infancy include Bartter syndrome, which is a genetic disorder
typically involving the Na/K/2Cl- channel in the thick ascending limb of the loop of Henle. Due to the
electrolyte losses, patients develop hypokalemia, hyponatremia, and contraction alkalosis with severe
dehydration. The electrolyte values reported for the infant in the vignette are inconsistent with Bartter
syndrome. Similarly, Fanconi syndrome, which is a proximal tubulopathy related to a number of
underlying causes (nephropathic cystinosis is most common), can present in the first postnatal year with
hypokalemia, hypophosphatemia, metabolic acidosis, glycosuria, and failure to thrive. Again, this infants
electrolyte values are inconsistent with Fanconi syndrome. Gitelman syndrome is a defect of the Na/Cl
transporter in the distal convoluted tubule that has a later age of onset and is characterized by
hypokalemia, hypomagnesemia, and hypocalciuria. The infants hypernatremia could be a result of
improper formula preparation, which could occur if the caregiver did not prepare it with sufficient water.
However, in that situation, the infant would be expected to have a concentrated urine in an effort to
compensate for the insufficient free water intake, and this infants urine specific gravity of 1.002 excludes
this possibility.
DI results in polyuria and polydipsia in an individual who is at risk for hypernatremic dehydration
due either to a defect in release of antidiuretic hormone (ADH) from the posterior pituitary (central DI) or
resistance to the action of ADH in the cortical collecting duct in the nephron (nephrogenic DI [NDI]).
Normally, vasopressin (ADH) binds to the vasopressin receptor (V2R) in the cortical collecting duct,
leading to phosphorylation of aquaporin 2, a protein that forms water channels, which translocates to the
luminal surface, allowing for effective water absorption. Central DI can be either inherited (due to defects
in the AVP gene encoding for ADH) or acquired. Acquired causes of central DI include brain tumors in the
hypophyseal region (craniopharyngioma or glioma) or traumatic injury to the pituitary region, as can occur
with injury or neurosurgery.
NDI can either be genetic or acquired. Mutations in the V2R gene, which is located on the X
chromosome and encodes for the vasopressin receptor, account for most cases (90%) of the inherited
forms of NDI. The remaining 10% are autosomal recessively inherited (so females may be affected) due
to a mutation of the AQP2 gene that results in a defect in the aquaporin 2 water channel. Acquired forms
of NDI can result from electrolyte disturbances (hypokalemia, hypercalcemia), medications (lithium,
foscarnet, amphotericin B, demeclocycline), or structural abnormalities (obstructive uropathy, renal
dysplasia).
Clinically, most children who have NDI present in the first year after birth, usually after the first
postnatal week, when the glomerular filtration rate begins to increase. Early diagnosis is essential due to
rapid fluid shifts, with reports of 10% weight loss overnight in affected infants. Infants who are severely
affected are at risk for developmental delay.
The diagnosis of NDI is suspected based on dilute urine (urine osmolality of 50 to 100 mOsm/L)
in the setting of hypernatremia. The diagnosis should not be made using a water deprivation test because
of the risks of severe dehydration in infants. Instead, intravenous desmopressin (DDAVP) is administered
at a dose of 0.3 mcg/kg to assess tissue response. Lack of response to DDAVP supports NDI and rules
out central DI. Treatment of central DI consists of DDAVP; children who have NDI are treated with low-
dose thiazide diuretics with or without prostaglandin inhibitors such as indomethacin.

American Board of Pediatrics Content Specification(s):


Recognize the signs and symptoms of diabetes insipidus in children
Recognize the association between cranial injury/surgery and diabetes insipidus, and that it is an
inherited disorder

Suggested Reading:
Goodyer P. Disorders of tubular transport. In: Kher KK, Schnaper HW, Makker SP, eds. Clinical Pediatric
Nephrology. 2nd ed. London, England: Informa Healthcare; 2007:317-336

Copyright 2012 American Academy of Pediatrics


2012 PREP SA ON CD-ROM

Quan A, Quigley R, Satlin LM, Baum M. Water and electrolyte handling by the kidney. In: Kher KK,
Schnaper HW, Makker SP, eds. Clinical Pediatric Nephrology. 2nd ed. London, England: Informa Healthcare;
2007:15-35

Saborio P, Tipton GA, Chan JCM. Diabetes insipidus. Pediatr Rev. 2000;21:122-129. DOI:
10.1542/pir.21-4-122. Available at: http://pedsinreview.aappublications.org/cgi/content/full/21/4/122

Copyright 2012 American Academy of Pediatrics


2012 PREP SA ON CD-ROM

Question 226
A 6-year-old boy who has moderate persistent asthma has experienced more frequent asthma
symptoms as well as nasal congestion and headaches for the past 4 weeks. Recently, he went to the
dentist because of upper tooth pain, but the dentist stated his examination findings were normal, and
there was no evidence of dental caries.
Of the following, the MOST likely cause for the boys symptoms is
A. allergic rhinitis
B. bacterial sinusitis
C. migraine headache
D. nonallergic rhinitis
E. viral upper respiratory tract infection

Copyright 2012 American Academy of Pediatrics


2012 PREP SA ON CD-ROM

Critique 226 Preferred Response: B


The combination of nasal congestion, headaches, worsening asthma control, and upper tooth
pain described for the boy in the vignette is concerning for acute bacterial sinusitis. General expert
consensus recommends that symptoms should persist longer than 7 to 10 days to make this diagnosis.
Other signs and symptoms of sinusitis include purulent nasal discharge, fever, facial pressure or
congestion, anosmia, halitosis, cough, otalgia, and fatigue.
Allergic rhinitis can be a risk factor for an asthma exacerbation or acute sinusitis, but discolored
rhinorrhea and tooth pain are not consistent with uncomplicated allergic rhinitis. Primary headache
disorders such as migraines often result in head and sinus pain. If initial antibiotic treatment does not
improve suspected sinus symptoms in a patient whose complaint is headache or pressure, consideration
should be given to sinus imaging such as computed tomography scan. Nonallergic rhinitis is more
common than allergic rhinitis in children and may result in symptoms that are similar to allergic rhinitis.
Common causes of nonallergic rhinitis include gustatory rhinitis, vasomotor rhinitis (due to irritants such
as cold air and strong odors), and nonallergic rhinitis with eosinophilia. The lack of a specific irritant
associated with nonallergic rhinitis makes this unlikely for this boy. Viral upper respiratory tract infections
can result in the symptoms described in this vignette, but symptoms persisting past 1 to 2 weeks are
unlikely in these infections.

American Board of Pediatrics Content Specification(s):


Know that sinusitis should be included in the differential diagnosis of toothache, sore throat,
persistent cough, and poorly controlled asthma

Suggested Reading:
Dykewicz MS, Hamilos DL. Rhinitis and sinusitis. J Allergy Clin Immunol. 2010;125(suppl 2):S103-S115.
DOI: 10.1016/j.jaci.2009.12.989. Available at: http://www.jacionline.org/article/S0091-6749(09)02881-
4/fulltext

Slavin RG, Spector RL, Bernstein IL, et al; American Academy of Allergy, Asthma and Immunology;
American College of Allergy, Asthma and Immunology; Joint Council of Allergy, Asthma and Immunology.
The diagnosis and management of sinusitis: a practice parameter update. J Allergy Clin Immunol.
2005;116(suppl):S13-S47

Copyright 2012 American Academy of Pediatrics


2012 PREP SA ON CD-ROM

Question 227
A 14-year-old girl presents to the office with right wrist pain after falling on her outstretched hand
while roller skating 2 hours ago. On physical examination, you note minimal swelling of her right wrist and
point tenderness at the base of the first metacarpal. Her hand is neurovascularly intact. You obtain
radiographs of her right wrist and forearm, which do not reveal any fracture.
Of the following, the MOST appropriate next step in this patients management is
A. ice and acetaminophen for pain
B. magnetic resonance imaging of the wrist and forearm
C. no further intervention
D. orthopedic follow-up evaluation if pain persists for more than 1 week
E. a thumb spica splint

Copyright 2012 American Academy of Pediatrics


2012 PREP SA ON CD-ROM

Critique 227 Preferred Response: E


Tenderness in the anatomic snuff box (base of the first metacarpal) following a fall onto an
outstretched hand, as described for the girl in the vignette, is highly suggestive of a scaphoid (or
navicular) fracture. Based on these clinical findings, she should be placed in a thumb spica splint, despite
the absence of the abnormal findings on her radiograph. Follow-up orthopedic and radiographic
evaluation should occur 7 to 10 days later, whether or not initial radiographs reveal a fracture. Ice and
analgesia are adjunctive to immobilization, and the cost-benefits of magnetic resonance imaging in the
acute setting have not been determined. Failure to treat these fractures appropriately with early
immobilization may lead to malunion, nonunion, or avascular necrosis. In a study of adult patients, 41% of
medical liability cases following wrist injuries were related to inadequate initial treatment of scaphoid
fractures.
The anatomy of the scaphoid bone makes it vulnerable to poor healing and avascular necrosis.
The arterial blood supply enters the bone at the distal pole and traverses the length of the bone to exit at
the proximal pole. Eighty percent of scaphoid fractures occur through the middle of the bone (the waist)
or proximal end, locations where a fracture is most likely to interrupt the blood supply and lead to
ischemia with resultant avascular necrosis. Besides location, fracture instability and delay in care are
factors strongly associated with nonunion.
Seventy percent of all wrist bone fractures occur in the scaphoid (Item C227) and are typically
seen in individuals 15 to 60 years old. A hallmark of these injuries in children is the relative frequency of
normal radiographs on initial examination. In a recent study of 104 scaphoid fractures in children, 30%
were detected only on follow-up radiographs. In addition to mechanism of injury, the physical examination
findings that suggested the diagnosis in these patients included pain on the volar aspect of the distal first
metacarpal with active wrist movement and with radial deviation.
When patients who have suspected scaphoid fractures are appropriately immobilized in a short-
arm thumb spica splint or cast until radiographic union is documented (6 to 20 weeks, depending on
fracture location), 90% to 95% of fractures heal satisfactorily.

American Board of Pediatrics Content Specification(s):


Recognize that a scaphoid fracture is associated with a poor prognosis

Suggested Reading:
Boles CA. Wrist, scaphoid fractures and complications. eMedicine Specialties, Radiology,
Musculoskeletal. 2010. Available at: http://emedicine.medscape.com/article/397230-overview

Burroughs KE. Scaphoid fractures. UpToDate Online 18.3. 2010. Available for subscription at:
http://www.uptodate.com/online/content/topic.do?topicKey=ad_orth/13882

Evenski AJ, Adamczyk MJ, Steiner RP, Morscher MA, Riley PM. Clinically suspected scaphoid fractures
in children. J Pediatr Orthop. 2009;29:352-355. DOI: 10.1097/BPO.0b013e3181a5a667. Abstract
available at: http://www.ncbi.nlm.nih.gov/pubmed/19461376

Wheeless CR III. Scaphoid/scaphoid fracture. In: Wheeless Textbook of Orthopaedics. 2010. Available
at: http://www.wheelessonline.com/ortho/scaphoid_scaphoid_fracture

Copyright 2012 American Academy of Pediatrics


2012 PREP SA ON CD-ROM

Critique 227

(Courtesy of M Wright)
Scaphoid fracture (arrow).

Copyright 2011 American Academy of Pediatrics


2012 PREP SA ON CD-ROM

Question 228
You are seeing a 18-month-old boy because of poor growth and inadequate weight gain. He
weighed 3,100 g at birth, following a term, uncomplicated pregnancy and delivery. The baby was
breastfed exclusively until 10 months of age, when he was weaned to whole cow milk and a mixed solid
food diet. At 13 months, he developed several loose bowel movements per day, and his diet was adjusted
to include lactase-treated milk (1% milk fat). Since that time, although he has gained weight slowly, he
continues to have two to three loose bowel movements per day. Over the past several weeks, he has
developed eczema, with sharply demarcated, dry, scaly patches appearing prominently in the
periorificial and acral areas (Item Q228). His mother reports that the child has also experienced recent
hair loss. His height and weight have crossed from the 25th to the 10th percentiles.
Of the following, this childs symptoms are MOST likely the consequence of
A. cow milk protein allergy
B. essential fatty acid deficiency
C. gluten-sensitive enteropathy
D. Hartnup disease
E. zinc deficiency

Copyright 2012 American Academy of Pediatrics


2012 PREP SA ON CD-ROM

Question 228

(Courtesy of D Krowchuk)
Perioral rash as exhibited by the boy in the vignette.

Copyright 2012 American Academy of Pediatrics


2012 PREP SA ON CD-ROM

Critique 228 Preferred Response: E


The diarrhea, poor weight gain, dermatitis, and hair loss described for the toddler in the vignette
strongly indicates a nutritional disorder related to zinc deficiency. This problem may occur as a
consequence of altered mineral metabolism, dietary deficiency, or a malabsorptive state. The syndrome
most closely linked to this clinical presentation is acrodermatitis enteropathica (AE), a rare inborn error of
zinc metabolism.
Zinc is an essential trace mineral that is found in virtually all body tissues and is required for
normal cell replication and functional development. Zinc also serves as a critical cofactor for numerous
enzyme systems. In the deficiency state, organ systems known to be affected clinically include the
epidermal, gastrointestinal, immune, skeletal, reproductive, and central nervous systems. Because the
mineral is a ubiquitous component in the cells of multiple tissues and organs, zinc deficiency leads to a
wide range of clinical abnormalities. Zinc is also known to play a central role in maintenance of normal
immune system function, and zinc-deficient persons experience increased susceptibility to a variety of
pathogens.
In the developed world, dietary or acquired zinc deficiency is uncommon. Cases have been
reported in the setting of long-term parenteral nutrition without zinc supplementation as well as in clinical
disorders associated with a malabsorptive state, including severe protein-losing enteropathy, Crohn
disease, cystic fibrosis, and celiac disease. In the absence of an at-risk condition, the recommended daily
intake for zinc (Item C228A) is age- and sex-dependent.
AE is inherited in an autosomal recessive pattern, and it is the consequence of mutations in the
SLC39A4 gene on chromosome locus 8q24.3. This locus encodes a specific intestinal transporter that is
required for zinc uptake. The genetic abnormality results in a marked decrease in zinc absorption, and
even though some intestinal zinc uptake occurs in AE, the amount absorbed is insufficient to prevent the
overt deficiency state in formula-fed infants. Because human milk often contains sufficient zinc to prevent
the onset of symptoms, breastfed infants who have the AE mutation may not present clinically until some
time after weaning. Symptoms of AE typically include periorificial and acral dermatitis (Item C228B),
diarrhea, failure to thrive, and alopecia. Neurobehavioral changes are common, particularly in older
infants and children; affected infants frequently are described as fussy and irritable, often with a hoarse
cry. In older children, dystrophic nail changes and recurrent infections are common signs. If untreated, the
mortality rate is high in early childhood.
Other clinical conditions, including severe cow milk protein allergy and celiac disease, may
predispose to zinc deficiency. However, isolated periorificial and acral dermatitis are not typical in these
disorders. Essential fatty acid (EFA) deficiency is rare, occurring in infants receiving a diet that lacks even
a small amount of EFA (linoleic and linolenic acids). The disorder was initially described in neonates
receiving long-term parenteral nutrition without lipid supplementation. Symptoms include a scaly
dermatitis that may mimic ichthyosis, alopecia, thrombocytopenia, and growth retardation. Finally,
Hartnup disease is another rare autosomal recessive disorder that involves impaired neutral amino acid
transport in the small intestine and the proximal tubule of the kidney. Patients exhibit a wide range of
clinical presentations, and many remain asymptomatic. Problems associated with Hartnup disease
include pellagra-like skin eruptions, cerebellar ataxia, and aminoaciduria.

American Board of Pediatrics Content Specification(s):


Recognize the distribution of the rash of acrodermatitis enteropathica
Know that acrodermatitis enteropathica is a manifestation of zinc deficiency

Suggested Reading:
Andrews GK. Regulation and function of Zip4, the acrodermatitis enteropathica gene. Biochem Soc
Trans. 2008;36:1242-1246. DOI: 10.1042/BST0361242. Available at:
http://www.ncbi.nlm.nih.gov/pmc/articles/PMC2634863/?tool=pubmed

Hambidge M. Human zinc deficiency. J Nutr. 2000;130(5S suppl):1344S1349S. Available at:


http://jn.nutrition.org/content/130/5/1344S.long

Copyright 2012 American Academy of Pediatrics


2012 PREP SA ON CD-ROM

Leonard D, Koca R, Acun C, et al. Visual diagnosis: three infants who have perioral and acral skin
lesions. Pediatr Rev. 2007;28:312-318. DOI: 10.1542/pir.28-8-312. Available at:
http://pedsinreview.aappublications.org/cgi/content/full/28/8/312

Maverakis E, Fung MA, Lynch PJ, et al. Acrodermatitis enteropathica and an overview of zinc
metabolism. J Am Acad Dermatol. 2007;56:116-124. Abstract available at:
http://www.ncbi.nlm.nih.gov/pubmed/17190629

Prasad AS. Impact of the discovery of human zinc deficiency on health. J Am Coll Nutr. 2009;28:257-265.
Abstract available at: http://www.ncbi.nlm.nih.gov/pubmed/20150599

Prasad AS. Zinc in human health: an update. J Trace Elem Exp Med. 1998;11:6387. DOI:
10.1002/(SICI)1520-670X(1998)11:2/3<63:AID-JTRA2>3.0.CO;2-5. Abstract available at:
http://onlinelibrary.wiley.com/doi/10.1002/(SICI)1520-670X(1998)11:2/3%3C63:AID-JTRA2%3E3.0.CO;2-
5/abstract

Schmitt S, Kry S, Giraud M, Drno B, Kharfi M, Bzieau S. An update on mutations of the SLC39A4
gene in acrodermatitis enteropathica. Hum Mutat. 2009;30:926-933. DOI: 10.1002/humu.20988. Abstract
available at: http://www.ncbi.nlm.nih.gov/pubmed/19370757

Copyright 2012 American Academy of Pediatrics


2012 PREP SA ON CD-ROM

Critique 228

Item C228A. Recommended Dietary Allowances for Zinc


0 to 6 7 to 12 1 to 3 4 to 8 9 to 13 14 to 18 >19
Age
months months years years years years years
Male 2 mg 3 mg 3 mg 5 mg 8 mg 11 mg 11 mg
Female 2mg 3 mg 3 mg 5 mg 8 mg 9 mg 8 mg
From Institute of Medicine, Food and Nutrition Board. Dietary Reference Intakes for Vitamin A, Vitamin K,
Arsenic, Boron, Chromium, Copper, Iodine, Iron, Manganese, Molybdenum, Nickel, Silicon, Vanadium, and
Zinc. Washington, DC: National Academy Press; 2001

Copyright 2011 American Academy of Pediatrics


2012 PREP SA ON CD-ROM

Critique 228

(Reprinted with permission from Leonard D, et al. Visual diagnosis. Three infants who have perioral and
acral skin lesions. Pediatr Rev. 2007;28:312-318)
The lesions of acrodermatitis enteropathica are well-defined scaling patches that appear as if they have
been pasted on the skin. Lesions typically are located periorificially and in the diaper area.

Copyright 2011 American Academy of Pediatrics


2012 PREP SA ON CD-ROM

Question 229
You are evaluating a 28 weeks gestational age infant who was recently discharged from the
neonatal intensive care unit after a 4-month hospital stay. She was delivered prematurely due to severe
pregnancy-induced hypertension and intrauterine growth restriction, with a birthweight of 700 g (3rd
percentile). She had a complicated hospital course that included severe respiratory distress syndrome
and necrotizing enterocolitis, with prolonged periods of parenteral nutrition and poor weight gain. She now
is breathing room air and receiving oral feeding of 150 mL/kg per day of human milk fortified to 24 kcal/oz.
Her weight in your office is 2,500 g (<3rd percentile). Her parents have many concerns about her small
size and neurodevelopmental outcome.
Of the following, the MOST appropriate counseling for this family is that
A. early growth deficits are not associated with poor neurodevelopmental outcomes
B. feeding strategies that provide 90 kcal/kg per day maintain optimal growth
C. higher protein intake after birth improves neurodevelopmental indices at 18 months
D. increased amounts of lipid after hospital discharge support brain development
E. nutrition in the second year after birth is critical to neurodevelopmental outcome

Copyright 2012 American Academy of Pediatrics


2012 PREP SA ON CD-ROM

Critique 229 Preferred Response: C


Preterm infants are at increased risk of poor growth and decreased neurodevelopmental indices
due to inadequate nutrition after birth, but higher protein intakes are believed to improve these outcome
measures. Studies have demonstrated that higher protein intakes in parenteral and enteral nutrition are
associated with increased protein accretion and weight gain as well as improved neurodevelopmental
outcomes at both 18 months and 7 to 8 years of age. Early initiation of parenteral amino acids at 3 g/kg
per day or more by the fifth postnatal day in very low-birthweight infants has been associated with better
growth at 36 weeks postmenstrual age and less suboptimal head growth at 18 months corrected
gestational age. Current guidelines for very low-birthweight infants recommend a protein content of 3.6
g/kg per day at an average energy intake of 120 kcal/kg per day. Evidence of inadequate protein intake,
such as a low serum urea nitrogen concentration (<4 mg/dL[1.4 mmol/L]), may suggest the need to
increase protein supplementation.
Poor growth in the preterm infant is also linked to daily energy intake. Improved
neurodevelopment and growth are associated with weight gain of more than 18 g/kg per day and head
circumference growth of more than 0.9 cm/wk during hospitalization after birth. The estimated daily
energy requirement of a preterm infant for maintenance and growth is 120 kcal/kg per day. The actual
energy requirement varies from infant to infant, with conditions such as intrauterine growth restriction,
chronic lung disease, congestive heart failure, short bowel syndrome, and infection often increasing the
energy needs. Increases in feeding volumes or caloric content may be needed to sustain a weight gain of
more than 18 g/kg per day.
The first year after birth is critical for nutritional compensation to allow catch-up in both somatic
and brain growth; the impact is limited after this time period. Continued attention to energy and protein
content in the diet of preterm infants after discharge from the hospital is essential. No data support
increasing the amount of lipid after discharge for enhanced brain development.

American Board of Pediatrics Content Specification(s):


Know the various circumstances in which the caloric requirements for premature infants vary

Suggested Reading:
Adamkin DH. Nutrition management of the very low-birthweight infant. II. Optimizing enteral nutrition and
postdischarge nutrition. NeoReviews. 2006;7:e608-e613. DOI: 10.1542/neo.7-12-e608. Available at:
http://neoreviews.aappublications.org/cgi/content/full/7/12/e608

American Academy of Pediatrics Committee on Nutrition. Nutritional needs of the preterm infant. In:
Kleinman RE, ed. Pediatric Nutrition Handbook. 6th ed. Elk Grove Village, IL: American Academy of
Pediatrics; 2009:79-112

Franz AR, Pohlandt F, Bode H, et al. Intrauterine, early neonatal, and postdischarge growth and
neurodevelopmental outcome at 5.4 years in extremely preterm infants after intensive neonatal nutritional
support. Pediatrics. 2009;123:e101-e109. DOI: 10.1542/peds.2008-1352. Available at:
http://pediatrics.aappublications.org/cgi/content/full/123/1/e101

Stephens BE, Walden RV, Gargus RA, et al. First-week protein and energy intakes are associated with
18-month developmental outcomes in extremely low birth weight infants. Pediatrics. 2009;123:1337-1343.
DOI: 10.1542/peds.2008-0211. Available at:
http://pediatrics.aappublications.org/cgi/content/full/123/5/1337

Copyright 2012 American Academy of Pediatrics


2012 PREP SA ON CD-ROM

Question 230
You are seeing a 9-year-old prepubertal boy for the first time for a health supervision visit. The
father mentions that the boy is playing competitive tennis, and his coach would like him to start a strength
training program to improve his sports performance. The child has a past history of a murmur, but the
father does not know any further details. The boy is currently asymptomatic and his examination findings
are normal.
Of the following, the MOST appropriate advice to provide the father about strength training is that
A. evidence shows that strength training affects growth plates adversely and impairs linear growth
B. specific strength-training exercises should be learned initially with no or low resistance
C. the most commonly reported adverse affects of strength training are injuries to joint structures
such as ligaments
D. the risk of injury to children in supervised strength training programs exceeds the risk of injury in
team sports
E. to be effective, the athlete must participate in strength training four or more times per week

Copyright 2012 American Academy of Pediatrics


2012 PREP SA ON CD-ROM

Critique 230 Preferred Response: B


Even as obesity and sedentary lifestyles become common, participation in competitive sports is
increasing among youth. Coaches or athletes may see strength training as a method to enhance
performance and prevent or rehabilitate injuries. Pediatricians have been concerned that resistance
training might prove harmful, particularly to prepubertal children. However, an increasing body of literature
demonstrates that resistance training, when pursued with proper technique and appropriate supervision,
can be used successfully and safely by both adolescents and preadolescents to increase muscle
strength. Evidence that this type of training is effective in preventing or lessening the severity of injury is
limited, and no evidence indicates that it prevents catastrophic sports injury.
Strength training can begin once the child has developed appropriate balance and postural
support, generally at age 7 or 8. All such training should be performed with close, appropriately trained,
and (optimally) certified supervision. Initially, training should begin with no or low resistance until
technique is perfected. Weight load can be added and subsequently increased in 10% increments.
Sessions should last 20 to 30 minutes and occur two to three times per week; four or more sessions per
week provide no additional benefit and may be associated with overuse injuries. Strength training should
be coupled with aerobic conditioning, and appropriate warm up and cool down is encouraged. Of course,
performance-enhancing drugs must not be used. With appropriate supervision, the risk of injury is low for
strength training, accounting for less than 1% of sports-related injuries among school-age children in one
study. By contrast, football contributed 19%, basketball 5%, and soccer 2% of injuries. Most strength
training-related injuries were muscle strains rather than joint injuries, with low back pain being the most
common complaint among high-school students in several studies.
Although the risk from appropriately performed resistance training is limited, certain children
require more in-depth medical evaluation before they embark on such a program. Among those who
require clearance before participation are: 1) children/teens who have uncontrolled hypertension; 2)
patients who have undergone chemotherapy with anthracyclines (because of the drugs cardiotoxicity); 3)
patients who have cardiomyopathies, particularly hypertrophic cardiomyopathy; 4) youth who have
moderate-to-severe pulmonary hypertension; 5) those who have Marfan syndrome with a dilated aortic
root; and 6) children and teens who have seizure disorders that are not well controlled. Being skeletally
immature is not a contraindication because there is no evidence that resistance training affects growth
plates or linear growth.

American Board of Pediatrics Content Specification(s):


Understand the importance of skeletal maturity in dictating the appropriate type of physical training

Suggested Reading:
AAP Council on Sports Medicine and Fitness. Strength training by children and adolescents. Pediatrics.
2008;121:835-840. DOI: 10.1542/peds.2007-3790. Available at:
http://pediatrics.aappublications.org/cgi/content/full/121/4/835

Faigenbaum AD, Kraemer WJ, Blimkie CJ, et al. Youth resistance training: updated position statement
paper from the National Strength and Conditioning Association. J Strength Cond Res. 2009;23(suppl
5):S60-S79. DOI: 10.1519/JSC.0b013e31819df407. Abstract available at:
http://www.ncbi.nlm.nih.gov/pubmed/19620931

Logsdon VK. Training the prepubertal and pubertal athlete. Curr Sports Med Rep. 2007;6:183-189.
Abstract available at: http://www.ncbi.nlm.nih.gov/pubmed/19202665

Copyright 2012 American Academy of Pediatrics


2012 PREP SA ON CD-ROM

Question 231
A 2-year-old boy who has trisomy 21 has been plagued by middle ear infections for several
months. You last saw him days ago and prescribed high-dose amoxicillin at 80 mg/3kg per day for
recurrent otitis media. Today he has a new onset of drainage from the ear and continued fussiness and
nocturnal awakening. Although his tympanic membranes are always difficult to see through his tiny
canals, today purulent drainage occludes the membrane completely. You decide to discontinue the
amoxicillin therapy.
Of the following, the BEST course of action for this patient is to
A. administer intramuscular ceftriaxone
B. administer one dose of intramuscular ampicillin
C. begin topical fluoroquinolone otic drops
D. begin trimethoprim-sulfamethoxazole
E. refer him for urgent placement of tympanostomy tubes

Copyright 2012 American Academy of Pediatrics


2012 PREP SA ON CD-ROM

Critique 231 Preferred Response: A


The boy described in the vignette is at higher-than-usual risk for persistent and recurrent otitis
media because he has trisomy 21. He continues to have symptoms despite 3 days of antibiotic therapy
and now has ear drainage. These findings are an indication that his antibiotic therapy should be changed.
Other indications for a change in antibiotic therapy during the treatment of acute otitis media include
persistent or recurrent fever after 2 to 3 days of therapy or suppurative complications.
For treatment of clinical failure 3 days into antibiotic therapy, as described for this boy, the
American Academy of Pediatrics recommends high-dose amoxicillin-clavulanate (90 mg/kg per day
amoxicillin and 6.4 mg/kg per day clavulanate) or intramuscular ceftriaxone for 1 to 3 days. A 3-day
course of ceftriaxone is more effective than a single dose in achieving a bacteriologic cure. If a beta-
lactamase-producing organism (such as nontypeable Haemophilus influenzae or Moraxella catarrhalis) is
suspected, other agents may be indicated, including cefpodoxime, cefdinir, or cefuroxime. Although
macrolides may be used as first-line agents in children who are allergic to penicillin, their use is
controversial due to high rates of pneumococcal resistance to these agents. Finally, trimethoprim-
sulfamethoxazole or sulfasoxazole is not adequate for treating Streptococcus pneumoniae infection.
Topical fluoroquinolones have no proven efficacy in acute otitis media in the presence of an intact
tympanic membrane. They may be useful in this child, who is presumed to have had a ruptured tympanic
membrane, but they should not supplant systemic antibiotic use.
Although otolaryngology evaluation that includes tympanocentesis may be needed for the child
who fails to respond to a change in antibiotics, urgent referral to an ear-nose-throat specialist for
tympanostomy tube placement is not indicated. Intramuscular ampicillin is unlikely to be effective for the
child who has been receiving amoxicillin and would not be as effective as ceftriaxone because it is a
short-acting antibiotic.

American Board of Pediatrics Content Specification(s):


Know the common indications for changing antibiotic therapy during acute otitis media: persistent or
recurrent ear pain or fever or both after two to three days of therapy; the development of a
suppurative complication

Suggested Reading:
Gould JM, Matz PS. Otitis media. Pediatr Rev. 2010;31:102-116. DOI: 10.1542/pir.31-3-102. Available at:
http://pedsinreview.aappublications.org/cgi/content/full/31/3/102

Kozyrskyj AL, Klassen TP, Moffatt M, Harvey K. Short-course antibiotics for acute otitis media. Cochrane
Database Syst Rev. 2010;9:CD001095. DOI: 10.1002/14651858.CD001095.pub2. Available at:
http://onlinelibrary.wiley.com/o/cochrane/clsysrev/articles/CD001095/frame.html

Paradise JL, Bluestone CD. Consultation with the specialist: tympanostomy tubes: a contemporary guide
to judicious use. Pediatr Rev. 2005;26:61-66. DOI: 10.1542/pir.26-2-61. Available at:
http://pedsinreview.aappublications.org/cgi/content/full/26/2/61

Subcommittee on Management of Acute Otitis Media. Diagnosis and management of acute otitis media.
Pediatrics. 2004;113:1451-1465. Available at:
http://pediatrics.aappublications.org/cgi/content/full/113/5/1451

Copyright 2012 American Academy of Pediatrics


2012 PREP SA ON CD-ROM

Question 232
A previously healthy 15-year-old boy presents to the emergency department with a 60-minute
history of tachycardia. He describes a rapid heart rate without irregularity, slight light-headedness,
anxiety, and mild shortness of breath. He can suppress the tachycardia transiently by performing a
Valsalva maneuver. On physical examination, the only finding of note is his heart rate of 230 beats/min.
Twelve-lead electrocardiography demonstrates tachycardia with a ventricular rate of 220 beats/min (Item
Q232). You elect to initiate therapy in an effort to convert the tachycardia to sinus rhythm.
Of the following, the BEST choice for initial pharmacologic therapy is
A. adenosine continuous drip infusion
B. adenosine intravenous bolus
C. amiodarone intravenous infusion
D. digoxin orally
E. propranolol orally

Copyright 2012 American Academy of Pediatrics


2012 PREP SA ON CD-ROM

Question 232

(Courtesy of M Lewin)
Electrocardiographic tracing, as described for the boy in the vignette.

Copyright 2012 American Academy of Pediatrics


2012 PREP SA ON CD-ROM

Critique 232 Preferred Response: B


Supraventricular tachycardia (SVT), synonymous with paroxysmal atrial tachycardia, is the result
of re-entry of electrical activity passing from the atria to the ventricles over the atrioventricular (AV) node
and back again to the atria via a retrograde conducting pathway. In infants, this retrograde conducting
pathway is usually an accessory pathway via the AV node. In older children, this type of accessory
pathway is common, but the retrograde tract by which the electrical wave re-enters the atria is a so-called
retrograde AV nodal pathway. In this situation, when there are dual AV nodal pathways, the technical
name for the SVT is AV node re-entry tachycardia. During AV re-entry tachycardia, careful inspection of
the electrocardiography (ECG) tracing often reveals the presence of retrograde P waves after the normal
QRS complex. The QRS complex is of normal morphology (unchanged from baseline sinus rhythm).
Resting ECG may reveal the pattern of Wolff-Parkinson-White syndrome, with a short PR interval and a
slurred upstroke of the QRS complex termed a delta wave (pre-excitation) (Item C232).
SVT is diagnosed from either capture of the acute event by 12-lead ECG or additional testing
after suspicious symptoms are reported (tachycardia, palpitations, and lightheadedness), as for the boy in
the vignette. Patients often describe a rapid onset and rapid cessation of the tachycardia, as opposed to
the gradual acceleration and deceleration of heart rate more typical of sinus tachycardia. Diagnostic
testing can include 24-hour ambulatory (Holter) monitoring, although this is only occasionally effective
because episodes are usually sporadic. More efficacious is longer-term ambulatory (event) monitoring in
which the patient wears or carries a monitor that can be used to capture events and transmit data via a
transtelephonic device.
The goal of initial pharmacologic therapy is to interrupt the re-entry tachycardia loop. Vagal
maneuvers are effective because the AV node is highly innervated by vagal parasympathetic efferents.
The AV node is also sensitive to intravenous administration of adenosine, which causes transient
complete AV block by blocking AV node conduction and terminating the re-entry SVT. Direct current
cardioversion is also effective in converting SVT to sinus rhythm.
After the acute episode of SVT is terminated, appropriate chronic pharmacologic management
must be determined. The most common chronic treatment of SVT in the pediatric population is -blocker
therapy (propranolol or atenolol). Propranolol and atenolol are nonselective b-blockers, blocking the
action of epinephrine and norepinephrine on both 1- and 2-adrenergic receptors. -adrenergic
antagonists should be used with caution in patients who have reactive airway disease because these
agents can exacerbate a bronchospastic event. Digoxin is also employed in the chronic treatment of SVT,
although the serum concentrations required to obtain efficacious therapy are closer to the toxic range
and, thus, most clinicians have moved to safer medications. A variety of other medications can be used,
including flecainide, sotalol, and amidarone. Flecainide should not be used in children who have SVT and
congenital heart disease due to reports of sudden death in this population. For children who are either
intolerant of medications or in whom medication is ineffective, catheter-based intervention can be used.
Radiofrequency catheter ablation allows energy to be applied to the site of retrograde conduction, either
burning or freezing the tissue, resulting in cell necrosis and interruption of the re-entrant loop. This highly
effective strategy is associated with greater than 90% success in routine re-entry SVT.
Continuous adenosine infusion is ineffective because of the agents extremely short half-life; only
a rapid adenosine bolus reaches the heart quickly enough to achieve its desired effect. Amiodarone
infusion can be used to control SVT, but this is not considered first-line therapy due to the potential for
significant adverse effects. Oral digoxin is ineffective for the acute management of SVT because
therapeutic serum concentrations are not achieved for several days. Short-acting intravenous beta-
blocker therapy can be effective, but propranolol should never be administered intravenously because of
its long half-life. Oral propranolol is not typically used for the acute management of SVT.

American Board of Pediatrics Content Specification(s):


Plan the initial therapy for paroxysmal atrial tachycardia

Suggested Reading:
Salerno JC, Seslar SP. Supraventricular tachycardia. Arch Pediatr Adolesc Med. 2009;163:268-274.
Available at: http://archpedi.ama-assn.org/cgi/content/full/163/3/268

Copyright 2012 American Academy of Pediatrics


2012 PREP SA ON CD-ROM

Tingelstad J. Consultation with the specialist: cardiac dysrhythmias. Pediatr Rev. 2001;22:91-94. DOI:
10.1542/pir.22-3-91. Available at: http://pedsinreview.aappublications.org/cgi/content/full/22/3/91

Copyright 2012 American Academy of Pediatrics


2012 PREP SA ON CD-ROM

Critique 232

(Courtesy of A Friedman)
In Wolff-Parkinson-White syndrome, delta waves (arrows) are present that represent pre-excitation (early
depolarization of the QRS complex).

Copyright 2011 American Academy of Pediatrics


2012 PREP SA ON CD-ROM

Question 233
In late summer, a 14-year-old previously healthy boy presents to the emergency department in
convulsive status epilepticus. Vital signs show low-grade fever. The seizures persist despite
administration of two doses of lorazepam and 20 mg/kg fosphenytoin intravenously. The child is given 20
mg/kg phenobarbital, intubated endotracheally, and brought to the intensive care unit for further
management. Computed tomography scan yields normal results. Lumbar puncture shows 15 white blood
cells, 3 red blood cells, and normal glucose and protein concentrations. Stat electroencephalography
shows continuing electrographic seizures. A midazolam infusion is started, and 20 mg/kg acyclovir is
administered.
Of the following, the MOST likely diagnosis is
A. acetaminophen ingestion
B. aneurysmal hemorrhage
C. encephalitis
D. idiopathic status epilepticus
E. mitochondrial disease

Copyright 2012 American Academy of Pediatrics


2012 PREP SA ON CD-ROM

Critique 233 Preferred Response: C


The boy described in the vignette has developed refractory status epilepticus. In a previously
healthy child, the differential diagnosis is broad, involving various pathologies in the cerebral cortex. For
this boy, the fever, absence of any intracranial pathology on head computed tomography (CT) scan, and
elevated cerebrospinal fluid (CSF) white blood cell count support encephalitis as the most likely cause.
Encephalitis, or inflammation of the brain, is a nonspecific term. Many infections, such as those caused
by arboviruses, enteroviruses, and herpesviruses, can be causative. Herpes encephalitis is a devastating
illness that is treatable with acyclovir, and it is critical to initiate treatment early. Classic findings of
radiologic changes in the temporal lobe and elevated CSF red blood cell count are not always present
initially. CSF should be sent for diagnostic polymerase chain reaction studies. Autoimmune diseases
involving neurologic antibodies (eg, anti-NMDA receptor) can also present with status epilepticus. In
some cases, the cause of the encephalitis may not be identified, and treatment is supportive.
Acetaminophen intoxication presents with anorexia, nausea, vomiting, malaise, and diaphoresis
followed by right upper quadrant pain and clinical laboratory findings consistent with hepatitis. It does not
present with the clinical findings, such as seizure, and laboratory findings reported in the vignette.
Blood can function as an irritant to the surface of the cerebrum, causing seizures. Thus, after
trauma or aneurysmal rupture, subarachnoid hemorrhage can cause seizures. Both intracranial infections
and subarachnoid hemorrhage can present with headache and nuchal rigidity, but the boy in the vignette
has no history of trauma and no prominent headache. Subarachnoid hemorrhage is generally detected on
head CT scan, and in cases where it is missed, the red blood cell count in the CSF is elevated.
Idiopathic status epilepticus indicates status epilepticus that is intrinsic to the brain but not due to
any identified cause. The presence of fever and elevated CSF white blood cell count exclude that
diagnosis for this boy. Mitochondrial diseases can present suddenly or chronically with a large variety of
neurologic symptoms. Sudden presentations can include confusion, strokelike symptoms, status
epilepticus, or respiratory insufficiency. Febrile illnesses can trigger metabolic decompensation as well.
Although much rarer than infectious encephalitis, the presence of a mitochondrial disease should be
considered in children who present with acute confusional states or seizures. The presence of a CSF
pleocytosis and age at presentation makes this diagnosis less likely than encephalitis.

American Board of Pediatrics Content Specification(s):


Know the clinical symptoms of encephalitis

Suggested Reading:
American Academy of Neurology and the Practice Committee of the Child Neurology Society. Neurology.
2006;67:1542-1550. DOI: 10.1212/01.wnl.0000243197.05519.3d. Available at:
http://www.neurology.org/content/67/9/1542.long

Chin RF, Neville BG, Scott RC. A systematic review of the epidemiology of status epilepticus. Eur J
Neurol. 2004;11:800-810. DOI: 10.1111/j.1468-1331.2004.00943.x. Abstract available at:
http://www.ncbi.nlm.nih.gov/pubmed/15667410

Foerster BR, Thurnher MM, Malani PN, Petrou M, Carets-Zumelzu F, Sundgren PC. Intracranial
infections: clinical and imaging characteristics. Acta Radiol. 2007;48:875-893. DOI:
10.1080/02841850701477728. Abstract available at: http://www.ncbi.nlm.nih.gov/pubmed/17924219

Lewis P, Glaser CA. Encephalitis. Pediatr Rev. 2005;26:353-363. DOI: 10.1542/pir.26-10-353. Available
at: http://pedsinreview.aappublications.org/cgi/content/full/26/10/353

Mikati MA. Seizures in childhood. In: Kliegman RM, Stanton BF, St. Geme JW III, Schor NF, and
Behrman RE, eds. Nelson Textbook of Pediatrics. 19th ed. Philadelphia, PA: Saunders Elsevier;
2011:2013-2039

Parikh S. The neurologic manifestations of mitochondrial disease. Dev Disabil Res Rev. 2010;16:120-
128. DOI: 10.1002/ddrr.110. Abstract available at: http://www.ncbi.nlm.nih.gov/pubmed/20818726

Copyright 2012 American Academy of Pediatrics


2012 PREP SA ON CD-ROM

Riviello JJ Jr, Ashwal S, Hirtz D, et al; American Academy of Neurology Subcommittee; Practice
Committee of the Child Neurology Society. Practice parameter: diagnostic assessment of the child with
status epilepticus (an evidence-based review): report of the Quality Standards Subcommittee

Copyright 2012 American Academy of Pediatrics


2012 PREP SA ON CD-ROM

Question 234
You are called to attend the delivery of a newborn male who had been monitored carefully
prenatally for moderate oligohydramnios. Prenatal ultrasonography suggested bilateral cystic dysplastic
kidneys, with no other congenital anomalies identified.
Of the following, the MOST likely immediate life-threatening problem in this infant is
A. bladder outlet obstruction
B. cardiac arrhythmias
C. liver failure
D. renal failure
E. respiratory failure

Copyright 2012 American Academy of Pediatrics


2012 PREP SA ON CD-ROM

Critique 234 Preferred Response: E


For the newborn described in the vignette, the most immediate medical concern is the possibility
of respiratory failure from pulmonary hypoplasia. This respiratory failure is felt to be due to decreased
amniotic fluid volume, which may lead to reduced fetal respiratory excursion which impedes pulmonary
development. In such cases, despite optimal respiratory support, neonates who have Potter sequence
(oligohydramnios with pulmonary hypoplasia, clubfeet, and flattened facies) may die shortly after birth. On
the other hand, infants who have less significant pulmonary hypoplasia and more pulmonary reserves
may achieve long-term survival.
Renal failure and bladder outlet obstruction can be addressed after the initial stabilization of the
infant. There are many potential causes for renal insufficiency in utero, including renal aplasia, obstructive
uropathies (posterior urethral valves, ureterovesical and ureteropelvic obstruction), and bilateral cystic
dysplastic kidneys (sometimes referred to as multicystic dysplastic kidneys). Cystic dysplastic kidneys
may be seen in the absence of obstruction or hydronephrosis and are believed to be due to a vascular
disruption or field defect occurring during early fetal kidney development. Infants who have Potter
sequence do not appear to be at increased risk for cardiac arrhythmias in the absence of electrolyte
imbalances. Hepatic cysts are frequently associated with polycystic kidneys, but they do not typically
pose any serious health concerns.
Routine antenatal ultrasonography has led to incidental identification of renal malformations,
hydronephrosis, and other obstructive uropathies in many fetuses. Under certain circumstances,
consideration may be given to fetal surgery in an attempt to relieve bladder outlet obstruction and
preserve fetal kidney function, when risks for survival are significantly improved with prompt prenatal
intervention. However, the limited number of centers that currently offer this fetal surgical procedure as
well as the significant maternal and fetal risks must be taken into consideration when counseling a couple
whose fetus has this type of urologic defect.

American Board of Pediatrics Content Specification(s):


Recognize the association of bilateral renal aplasia or severe dysplasia with pulmonary hypoplasia
(Potter syndrome)

Suggested Reading:
Carey WA, Talley LI, Sehring SA, Jaskula JM, Mathias RS. Outcomes of dialysis initiated during the
neonatal period for treatment of end-stage renal disease: a North American Pediatric Renal Trials and
Collaborative Studies special analysis. Pediatrics. 2007;119:e468-e473. DOI: 10.1542/peds.2006-1754.
Available at: http://pediatrics.aappublications.org/cgi/content/full/119/2/e468

Holmes N, Harrison MR, Baskin LS. Fetal surgery for posterior urethral valves: long-term postnatal
outcomes. Pediatrics. 2001;108:e7. Available at:
http://pediatrics.aappublications.org/cgi/content/full/108/1/e7

Kennedy WA II. Assessment and management of fetal hydronephrosis. NeoReviews. 2002;3:e214-e219.


DOI: 10.1542/neo.3-10-e214. Available at:
http://neoreviews.aappublications.org/cgi/content/full/3/10/e214

Copyright 2012 American Academy of Pediatrics


2012 PREP SA ON CD-ROM

Question 235
A 17-year-old girl is brought to the emergency department by her parents because of vomiting.
She has no fever, headache, abdominal pain, or diarrhea. She says that over the past 3 years she has
periods of time when she vomits and then she is fine for a while. She denies inducing the vomiting. Her
periods are regular, and her last one was 2 weeks ago. On physical examination, you note normal vital
signs, a body mass index of 28.5, a small subconjunctival hemorrhage on the right eye, and slight
enlargement of her parotid glands bilaterally. Laboratory results reveal a normal complete blood count
and erythrocyte sedimentation rate, amylase of 75 U/L, and lipase of 1 U/L. Her pregnancy test is
negative, and a urinalysis has a specific gravity of 1.030 with trace protein and ketones.
Of the following, the MOST likely explanation for these findings is
A. acute pancreatitis
B. bulimia nervosa
C. cyclic vomiting
D. diabetic ketoacidosis
E. ectopic pregnancy

Copyright 2012 American Academy of Pediatrics


2012 PREP SA ON CD-ROM

Critique 235 Preferred Response: B


The girl described in the vignette has a history of recurrent vomiting, subconjunctival hemorrhage,
and parotid gland enlargement, which strongly suggests the diagnosis of bulimia nervosa. Affected
patients often have a net caloric gain despite vomiting because of the amount of calories consumed
during their recurrent binge episodes. Therefore, they may be overweight and have normal menses.
Patients who have been inducing vomiting for a long period of time may be able to have emesis with
limited, if any, stimulation, even occasionally by gently jumping up and down.
The criteria for diagnosis of bulimia nervosa are:
Recurrent episodes of binge eating that are characterized by eating an amount of food in a period of
time that is clearly larger than most people would eat during a similar time period and in similar
circumstances coupled with a sense of lack of control over eating during the episode.
Use of inappropriate behaviors to prevent weight gain, such as self-induced vomiting; misuse of
laxatives, diuretics, enemas, or other medications; fasting; or excessive exercise.
The binge eating and inappropriate behaviors occur twice or more each week for a period of 3
months.
Self-image is inappropriately influenced by body shape and weight.
Two types of bulimia nervosa exist. In the purging type, the person regularly engages in self-
induced vomiting or misuse of laxatives, diuretics, or enemas. In the nonpurging type, the person uses
other inappropriate compensatory behaviors, such as fasting or excessive exercise, but does not
regularly engage in self-induced vomiting or the misuse of laxatives, diuretics, or enemas.
The lack of abdominal pain with normal amylase and lipase values rules out pancreatitis for this
girl. Cyclic vomiting is a syndrome of recurrent stereotypic spells of vomiting in between which the patient
is completely well. Frequently associated symptoms include fatigue, pallor, anorexia, and nausea. Cyclic
vomiting is more common in younger children, who tend to outgrow it in their preteen or early teen years,
but they often subsequently develop migraine headaches. There is a strong family history of migraine.
The occasional adolescent who has this syndrome experiences intense episodes of vomiting that may
occur with menses; parotid enlargement is not a typical feature. The lack of glucose on the urinalysis for
this girl rules out diabetic ketoacidosis, and the lack of abdominal pain and menstrual bleeding along with
a negative pregnancy test makes ectopic pregnancy an unlikely cause of the vomiting.

American Board of Pediatrics Content Specification(s):


Know the characteristics of bulimia and the criteria for diagnosis

Suggested Reading:
American Psychiatric Association. 307.51. Eating disorders. In: Diagnostic and Statistical Manual of
Mental Disorders, Fourth Edition, Text Revision. Washington DC: American Psychiatric Association;
2000:589-595

Buckelew S, Slivka M. Index of suspicion: case 1. Pediatr Rev. 2010;31:341-346. DOI: 10.1542/pir.31-8-
341. Available at: http://pedsinreview.aappublications.org/cgi/content/full/31/8/341

Chandran L, Chitkara M. Vomiting in children: reassurance, red flag, or referral? Pediatr Rev.
2008;29:183-192. DOI: 10.1542/pir.29-6-183. Available at:
http://pedsinreview.aappublications.org/cgi/content/full/29/6/183

Cuvellier JC, Lpine A. Childhood periodic syndromes. Pediatr Neurol. 2010;42:1-11. DOI:
10.1016/j.pediatrneurol.2009.07.001 Abstract available at: http://www.ncbi.nlm.nih.gov/pubmed/20004856

Fisher M. Treatment of eating disorders in children, adolescents, and young adults. Pediatr Rev.
2006;27:5-16. DOI: 10.1542/pir.27-1-5. Available at:
http://pedsinreview.aappublications.org/cgi/content/full/27/1/5

North American Society for Pediatric Gastroenterology, Hepatology, and Nutrition consensus statement
on the diagnosis and management of cyclic vomiting syndrome. Li BU, Lefevre F, Chelimsky GG, et al;

Copyright 2012 American Academy of Pediatrics


2012 PREP SA ON CD-ROM

North American Society for Pediatric Gastroenterology, Hepatology, and Nutrition. J Pediatr Gastroenterol
Nutr. 2008;47:379-393. DOI: 10.1097/MPG.0b013e318173ed39. Available at:
http://journals.lww.com/jpgn/Fulltext/2008/09000/North_American_Society_for_Pediatric.22.aspx

Copyright 2012 American Academy of Pediatrics


2012 PREP SA ON CD-ROM

Question 236
You are called to evaluate an 8-year-old boy who fell while climbing a tree and landed on the left
side of his abdomen on a fence. He did not lose consciousness, was transported to the emergency
department by emergency medical services, and complains of left-sided shoulder pain and tenderness
over the left side of his abdomen. On physical examination, he has a heart rate of 125 beats/min,
respiratory rate of 27 breaths/min, and blood pressure of 105/70 mm Hg. His oxygen saturation by pulse
oximetry is 85% while receiving 2 L/min oxygen via a nasal cannula. His heart rhythm is normal with no
murmurs, he has clear and equal breath sounds, and his abdomen is mildly distended, with abrasions,
ecchymosis, and tenderness in the left upper quadrant.
Results of laboratory studies from 2 hours ago include:
Serum sodium, 140 mEg/L (140 mmol/L)
Serum potassium, 4 mEq/L (4 mmol/L)
Serum chloride, 105 mEq/L (105 mmol/L)
Hematocrit, 30% (0.3)
Blood urea nitrogen, 12.0 mg/dL (4.3 mmol/L)
Serum creatinine, 0.6 mg/dL (53.0 mcmol/L)
Aspartate aminotransferase, 75 units/L
Alanine aminotransferase, 45 units/L
Amylase, 40 units/L
Lipase, 35 units/L
Of the following, the BEST test to evaluate for abdominal injury in this boy is
A. abdominal radiograph
B. computed tomography scan of the abdomen with contrast
C. computed tomography scan of the abdomen without contrast
D. diagnostic peritoneal lavage
E. focused abdominal sonography for trauma (FAST)

Copyright 2012 American Academy of Pediatrics


2012 PREP SA ON CD-ROM

Critique 236 Preferred Response: B


Blunt trauma accounts for more than 90% of trauma-associated pediatric injuries, with the liver
and spleen injured most commonly. Most often, the spleen is injured by a direct blow to the left upper
quadrant. Initial evaluation of pediatric patients following blunt abdominal trauma should focus on
mechanism of injury, past medical history, and physical findings. Mechanisms of injury that should raise
the suspicion for significant intra-abdominal injury include: motor vehicle crashes where the patient is
unrestrained or restrained only by a lap seatbelt, motor vehicle-pedestrian incidents, all-terrain vehicle
crashes, falls, and direct blows to the abdomen from a variety of mechanisms, including sports and
nonaccidental trauma. The history of landing on his abdomen raises suspicion for such an injury for the
boy described in the vignette. Past medical history should focus on inherited or acquired coagulation
disorders, medications that interfere with the coagulation process, or underlying medical conditions or
viral infections that may cause splenomegaly and subsequent increased risk of splenic rupture. Physical
examination findings that are consistent with splenic injury include tenderness, abrasions, and
ecchymosis, as described for this boy. Referral of pain to the left shoulder (Kehr sign) due to
diaphragmatic irritation by blood is also common.
Laboratory studies that should be obtained when intra-abdominal injury is suspected include a
complete blood count or hematocrit, liver function studies, amylase or lipase assessment, and urinalysis.
Patients who have abnormal screening laboratory results or findings on a physical examination that are
difficult to interpret due to developmental status or altered mental status should undergo abdominal
computed tomography (CT) scan with contrast to define the location and extent of injuries (Item C236).
The use of intravenous and oral contrast is recommended to help determine organ perfusion, the
presence of free intraperitoneal fluid, and bowel integrity. Focused abdominal sonography for trauma
(FAST) is just beginning to be evaluated in children and cannot be recommended as a replacement for
CT scan at this time. Abdominal radiographs are not useful in the evaluation of abdominal trauma
because of their low sensitivity in diagnosing solid-organ injury. The use of diagnostic peritoneal lavage is
generally limited to emergent situations when the patient is being taken to the operating room for other
life-threatening injuries and rapid evaluation is needed to determine the need for an exploratory
laparotomy.

American Board of Pediatrics Content Specification(s):


Plan the initial evaluation of a patient with probable splenic rupture

Suggested Reading:
Brandow AM, Camitta BM. The spleen: hyposplenism, splenic truama, and splenectomy. In: Kliegman
RM, Stanton BF, St. Geme JW III, Schor NF, and Behrman RE, eds. Nelson Textbook of Pediatrics. 19th
ed. Philadelphia, PA: Saunders Elsevier; 2011:1723

Davis DH, Localio AR, Stafford PW, Helfaer MA, Durbin DR. Trends in operative management of pediatric
splenic injury in a regional trauma system. Pediatrics. 2005;115:89-94. DOI: 10.1542/peds.2004-0508.
Available at: http://pediatrics.aappublications.org/cgi/content/full/115/1/89

Guralnick S, Serwint JR. In brief: blunt abdominal trauma. Pediatr Rev. 2008;29:294-295. DOI:
10.1542/pir.29-8-294. Available at: http://pedsinreview.aappublications.org/cgi/content/full/29/8/294

Copyright 2012 American Academy of Pediatrics


2012 PREP SA ON CD-ROM

Critique 236

(Courtesy of B Poss)
Computed tomography scan of the patient described in the vignette demonstrating a large splenic
laceration (arrow).

Copyright 2011 American Academy of Pediatrics


2012 PREP SA ON CD-ROM

Question 237
A 14-year-old girl presents for her annual health supervision visit. She has no complaints, and her
review of systems reveals no findings of note. However, her mother is concerned that her daughter has
not yet had menarche. The girls height is at the 5th percentile and her weight is at the 25th percentile. On
physical examination, she has Sexual Maturity Rating (SMR) 1 breast development and SMR 3 pubic
hair. Examination of the external genitalia reveals a patent vagina and pink mucosae.
Of the following, the BEST next step in evaluation of this patient is to
A. measure serum estradiol
B. obtain a karyotype
C. perform a bone age radiograph
D. reassure the family and see them again in 1 year
E. re-examine her in 6 months

Copyright 2012 American Academy of Pediatrics


2012 PREP SA ON CD-ROM

Critique 237 Preferred Response: B


In general, any girl who fails to demonstrate evidence of puberty by the age of 14 should be
considered for additional investigation. In young women, the first physical sign of puberty is the
development of breast buds, allowing physicians to identify girls who need further evaluation through
physical examination. However, with a growing number of overweight and obese young women, it is
important for physicians to palpate breast tissue to confirm findings noted on visual inspection. True
breast buds are firm and centered directly under the areola; adipose tissue is soft and can appear
anywhere on the anterior chest. Pubic hair is not a true sign of puberty because most pubic hair
development is caused by adrenal androgens, not ovarian androgens.
The girl described in the vignette has no breast buds (SMR 1) and pink vaginal mucosa, both of
which indicate no estrogenization. In addition, she is relatively short, with a height at the 5th percentile.
These findings should raise suspicion for Turner syndrome, which requires karyotyping for diagnosis.
Measuring serum estradiol is unnecessary because her physical examination findings indicate that the
serum estradiol value will be prepubertal. A bone age radiograph would be useful in documenting her
skeletal maturity, but it is not the most important next step in evaluating this girl. The lack of evidence of
puberty at 14 years of age argues against reassuring the family and re-examining the girl in 6 months or 1
year, which could delay the diagnosis and appropriate therapy.

American Board of Pediatrics Content Specification(s):


Know that the absence of any signs of puberty after the age of about 13 years in a girl or about 14
years in a boy merits investigation

Suggested Reading:
Kaplowitz PB. Delayed puberty. Pediatr Rev. 2010;31:189-195. DOI: 10.1542/pir.31-5-189. Available at:
http://pedsinreview.aappublications.org/cgi/content/full/31/5/189

Postellon DC, Daniel MA. Turner syndrome. eMedicine Specialties, Pediatrics: Genetics and Metabolic
Disease, Medical Topics. 2010. Available at: http://emedicine.medscape.com/article/949681-overview

Pralong FP, Crowley WF Jr. Diagnosis and treatment of delayed puberty. UpToDate 18.3. 2010. Available
at: http://www.uptodate.com/patients/content/topic.do?topicKey=~YOPqPJdSo3CS39c

Copyright 2012 American Academy of Pediatrics


2012 PREP SA ON CD-ROM

Question 238
An 8-year-old boy is having attention difficulties in his third-grade classroom. He has undergone
psychoeducational testing and has not had a learning disability identified. His parents and teachers have
completed Vanderbilt rating forms, and the results are significant for inattention and impulsivity. You are
considering starting the child on medication to treat his attention-deficit/hyperactivity disorder.
Of the following, the MOST significant historical information that would affect your decision to
start treatment with a stimulant medication is
A. absence epilepsy in his 6-year-old sister
B. bipolar disorder in his paternal uncle
C. mild motor tic in the child
D. myocardial infarction in the paternal grandfather at the age of 65 years
E. sudden death of his 15-year-old brother while playing basketball

Copyright 2012 American Academy of Pediatrics


2012 PREP SA ON CD-ROM

Critique 238 Preferred Response: E


The most significant historical information affecting the decision to start treatment with a stimulant
medication for the boy described in the vignette would be his brothers sudden unexpected death
because such a death may have been due to a hereditary cardiac disorder such as a cardiomyopathy or
long QT syndrome. Recently, concern has been raised about the risk of sudden cardiac death in pediatric
patients treated with stimulant drugs. The American Academy of Pediatrics policy is that a targeted
cardiac history and physical examination be performed in candidates for stimulant drugs. Routine
electrocardiography is not indicated before starting a stimulant medication. Consultation with a pediatric
cardiologist should be sought if the patient has:
Known cardiac disease, palpitation, syncope, or seizures
Family history of sudden death in child or young adults, hypertrophic cardiomyopathy, or long QT
syndrome
Abnormalities on the cardiac examination
The presence of a minor motor tic is not a contraindication for starting medication, but it may
require additional monitoring to determine if the frequency of tics increases. Neither the presence of
bipolar disorder in the uncle nor absence seizures in the sister is a contraindication. Clinicians should be
vigilant about any symptoms of staring spells in the child or exacerbation of mood issues during use of
the medication. A family history of a cardiac event such as a myocardial infarction would be relevant if it
occurred at a younger age than 65 years.
The first-line agents for treatment of attention-deficit/hyperactivity disorder (ADHD) are stimulant
medications. More than 80% of children respond to this class of medications with improvements in their
symptoms. The generic forms are methylphenidate and amphetamines. Other medications that are used
include nonstimulants such as atomoxetine and guanfacine. Atomoxetine is considered a second-line
drug because it is less effective in treating ADHD than stimulants.
Contraindications to specific medications used for the treatment of ADHD are listed in (Item
C238A) and adverse effects are listed in (Item C238B).

American Board of Pediatrics Content Specification(s):


Know the medications used in treating ADHD
Know the side effects of medications used to treat ADHD, the contraindications to their use, and the
potential for their abuse

Suggested Reading:
American Academy of Pediatrics Committee on Quality Improvement, Subcommittee on Attention-
Deficit/Hyperactivity Disorder. Clinical practice guideline: diagnosis and evaluation of the child with
attention- deficit/hyperactivity disorder. Pediatrics. 2000;105:1158-1170. Available at:
http://pediatrics.aappublications.org/cgi/content/full/105/5/1158

Hammerness P, Traum AZ, Becker J, Deshpande A. Cardiovascular risk in ADHD pharmacotherapy.


Contemp Pediatr. 2009(Nov);26:34-46. Available at:
http://www.modernmedicine.com/modernmedicine/article/articleDetail.jsp?id=644218&pageID=1&sk=&dat
e=

Perrin JM, Friedman RA, Knilans TK, the Black Box Working Group, the Section on Cardiology and
Cardiac Surgery. Cardiovascular monitoring and stimulant drugs for attention-deficit/hyperactivity
disorder. Pediatrics. 2008;122:451-453. DOI: 10.1542/peds.2008-1573. Available at:
http://pediatrics.aappublications.org/cgi/content/full/122/2/451

PubMed Health. Atomoxetine. 2008. Available at:


http://www.ncbi.nlm.nih.gov/pubmedhealth/PMH0000222#a603013-sideEffects

PubMed Health. Dextroamphetamine and Amphetamine. 2010. Available at:


http://www.ncbi.nlm.nih.gov/pubmedhealth/PMH0000166#a601234-sideEffects

Copyright 2012 American Academy of Pediatrics


2012 PREP SA ON CD-ROM

PubMed Health. Methylphenidate. 2011. Available at:


http://www.ncbi.nlm.nih.gov/pubmedhealth/PMH0000606#a682188-sideEffects

Wilms Floet AM. Scheiner C, Grossman L. Attention-deficit/hyperactivity disorder. Pediatr Rev.


2010;31;56-69. DOI: 10.1542/pir.31-2-56. Available at:
http://pedsinreview.aappublications.org/cgi/content/full/31/2/56

Copyright 2012 American Academy of Pediatrics


2012 PREP SA ON CD-ROM

Critique 238

Item C238A. Contraindications to Medications Used to Treat ADHD

Active Ingredients Contraindications


Amphetamines Monoamine oxidase (MAO) inhibitors within 14 days, glaucoma, symptomatic
cardiovascular disease, hyperthyroidism, moderate-to-severe hypertension
Methylphenidate MAO inhibitors within 14 days, glaucoma, symptomatic cardiovascular
disease, hyperthyroidism, moderate-to-severe hypertension, pre-existing
severe gastrointestinal narrowing; exercise caution when used with
anticoagulants, anticonvulsants, phenylbutazone, and tricyclic antidepressants
Atomoxetine MAO inhibitors within 14 days, glaucoma; may interfere with selective
serotonin reuptake inhibitor metabolism (uses CYP2D6 system); drug
interaction with albuterol; jaundice or laboratory evidence of liver injury
Guanfacine Coadministration of guanfacine and valproic acid can result in increased
concentrations of valproic acid; use caution with patients taking ketoconazole
and other strong CYP3A4/5 inhibitors because elevation of plasma guanfacine
concentration increases the risk of hypotension, bradycardia, and sedation

Copyright 2011 American Academy of Pediatrics


2012 PREP SA ON CD-ROM

Critique 238

Item C238B. Adverse Effects of Medications Used to Treat ADHD

Active Ingredients Adverse Effects


Amphetamines Nervousness, decreased appetite, abdominal pain, headache, and sleep
issues
Methylphenidate Decreased appetite, abdominal pain, headache, and sleep issues
Atomoxetine Increased risk for suicidal thoughts (black box warning);
decreased appetite, abdominal pain, nausea and sleepiness, rare cases
of hepatitis (reversible)
Guanfacine Sleepiness and fatigue; less common side effects (occur in approximately
1%): hypotension, headache, and dizziness.

Copyright 2011 American Academy of Pediatrics


2012 PREP SA ON CD-ROM

Question 239
A 3-year-old child who lives in Philadelphia presents to the local emergency department in August
with a 2-day history of fever, decreased oral intake, and sore throat, followed by the onset of a rash today.
He attends a day camp, where he swims daily, but he has no history of travel. Physical examination
reveals a temperature of 39.5C, heart rate of 120 beats/min, respiratory rate of 22 breaths/min, and
oxygen saturation of 100% in room air. He is alert and verbal. His pharynx is erythematous without
exudate and neck is supple without significant adenopathy. His lungs are clear. He has a regular heart
rhythm without murmur, rub, or gallop; his pulses are full and equal; and his capillary refill is less than 2
seconds. Abdominal and neurologic examinations reveal no findings of note. He has scattered petechiae
on his abdomen and extremities. Laboratory results include:
3 9
White blood cell count, 12.4x10 /mcL (12.4x10 /L) with 48% neutrophils, 39% lymphocytes, 11%
monocytes, and 2% eosinophils
Hemoglobin, 13.6 g/dL (136 g/L)
Hematocrit, 39% (0.39)
3 9
Platelet count, 229x10 /mcL (229x10 /L)
Of the following, the BEST test to diagnose this childs condition is
A. blood culture
B. bone marrow aspiration
C. serology for Rocky Mountain spotted fever
D. throat and rectal viral cultures
E. throat culture for group A Streptococcus

Copyright 2012 American Academy of Pediatrics


2012 PREP SA ON CD-ROM

Critique 239 Preferred Response: D


Although the acute onset of fever and petechiae raises concern for invasive bacterial disease in
the otherwise healthy boy described in the vignette, who has a normal complete blood count (CBC), viral
infection is a more likely diagnosis. The summer season and swimming history increase the likelihood of
an enteroviral infection. Throat and rectal viral cultures are appropriate for isolating these agents.
A blood culture likely would be sent in this setting, but invasive bacterial infection (eg, Neisseria
meningitidis, Streptococcus pneumoniae, Staphylococcus aureus, group A Streptococcus, and enteric
gram-negative rods) is highly unlikely in a well-appearing child who has a normal CBC result, and the
culture probably would be negative. With normal cell lines detected on CBC, including platelets, and the
acute onset of illness with fever, this is unlikely a hematologic problem or malignancy, making bone
marrow aspiration unnecessary. The well appearance of the child, lack of prodromal symptoms, and lack
of potential dog tick exposure argue against a diagnosis of Rocky Mountain spotted fever. Although the
boy had a sore throat, scattered petechiae are not the type of rash typically associated with group A
streptococcal infection, obviating the need for a throat culture.

American Board of Pediatrics Content Specification(s):


Formulate the differential diagnosis of a patient with a purpuric rash

Suggested Reading:
Leung AKC, Chen KW. Evaluating the child with purpura. Am Fam Physician. 2001;64:419-428. Available
at: http://www.aafp.org/afp/2001/0801/p419.html

Raffini L. Evaluation of purpura in children. UpToDate Online 18.3. 2010. Available for subscription at:
www.uptodate.com/online/content/topic.do?topicKey=ped_symp/15155

Copyright 2012 American Academy of Pediatrics


2012 PREP SA ON CD-ROM

Question 240
A 4-year-old boy presents with a 4-day history of worsening right eyelid swelling and redness
after a mosquito bite. On physical examination, his temperature is 38.0C, heart rate is 100 beats/min,
and respiratory rate is 25 breaths/min. His right eyelid is markedly swollen, red, and tender, and he is
unable to open it fully. His conjunctivae are clear, and extraocular movements are not limited. There is no
proptosis. Visual acuity is difficult to assess fully but appears normal. There are no other physical findings
3 9
of note. The white blood cell count is 19.0x10 /mcL (19.0x10 /L), with 55% polymorphonuclear
leukocytes, 20% band forms, 20% lymphocytes, and 5% monocytes.
Of the following, the MOST appropriate antibiotic for this patient is
A. ampicillin-sulbactam
B. cefazolin
C. clindamycin
D. doxycycline
E. trimethoprim-sulfamethoxazole

Copyright 2012 American Academy of Pediatrics


2012 PREP SA ON CD-ROM

Critique 240 Preferred Response: C


The boy described in the vignette has periorbital (preseptal) cellulitis characterized by eyelid
edema and erythema, normal extraocular movements, apparent normal visual acuity, and absence of
proptosis and chemosis. Staphylococcus aureus and Streptococcus pyogenes (group A Streptococcus)
(GAS) are the most common causes of periorbital cellulitis in cases in which cultures are obtained. In
addition, community-associated methicillin-resistant S aureus (CA-MRSA) has emerged as a significant
pathogen. Clindamycin can treat GAS and susceptible strains of CA-MRSA and is appropriate therapy for
this boy. The use of a semisynthetic penicillin (eg, dicloxacillin, nafcillin, oxacillin) or a first-generation
cephalosporin (eg, cephalexin, cefazolin) is appropriate in communities where MRSA is not prevalent.
Doxycycline can be used to treat uncomplicated soft-tissue infections caused by susceptible strains of S
aureus in children older than 7 years of age, but it has variable activity against GAS. Trimethoprim-
sulfamethoxazole is effective against susceptible strains of S aureus but is not useful for treating GAS.
Vancomycin should be used in patients who have severe infection and in communities where clindamycin
resistance is prevalent.
Periorbital cellulitis occurs most commonly in children younger than 5 years of age. It can be due
to localized infection or inflammation of the eyelid, conjunctivae, or adjacent structures (hordeolum,
dacrocystitis, dacroadenitis, trauma), such as occurred in the patient in the vignette, who developed
infection following an insect bite. In such cases, GAS and S aureus are the most common causes of
infection, but S pneumoniae and nontypable Haemophilus influenzae also can cause infection in patients
who have dacryoadenitis and dacrocystitis. Antimicrobial therapy can be guided based on Gram stain of
an aspirate from the infected area.
Periorbital cellulitis also can be caused by inflammatory edema of rhinosinusitis, most often
involving the ethmoids. In such cases, S pneumoniae, nontypable H influenzae, and Moraxella catarrhalis
are the usual pathogens. Appropriate antibiotic therapy in such cases can include amoxicillin-clavulanate,
a second-generation cephalosporin (cefuroxime), or a third-generation cephalosporin (cefdinir,
cefpodoxime, cefotaxime, ceftriaxone).
Occasionally, periorbital cellulitis is caused by hematogenous seeding in children younger than
18 months of age, with an antecedent viral upper respiratory tract infection. S pneumoniae, nontypable H
influenzae, GAS, and S aureus are likely pathogens. Cerebrospinal fluid analysis should be performed in
young infants or children who are severely ill. Parenteral therapy with a third-generation cephalosporin
and clindamycin or vancomycin is appropriate.
Finally, periorbital cellulitis caused by a mixed aerobic and anaerobic infection should be
considered in patients who have dental abscesses. Ampicillin-sulbactam or clindamycin can be used in
those who have suspected anaerobic infections and require intravenous antibiotics. Appropriate oral
therapy includes amoxicillin-clavulanate or clindamycin.
A complete history and physical examination are necessary for all children who have suspected
periorbital cellulitis. A history of sinusitis, dental problems, and trauma, including eye surgery, should be
elicited. Physical examination focusing on the presence or absence of eyelid fluctuance, proptosis,
chemosis, and normal extraocular movements and visual acuity is important. Patients who have severe
eyelid swelling that prevents eye examination or those in whom orbital abscess (eg, orbital cellulitis) is
suspected should undergo computed tomography scan with contrast of the sinuses and orbits. Eyelid (or
other) abscesses should be drained and purulent material sent for culture.
The decision to treat a patient with intravenous versus oral therapy should be based on the
severity of disease and clinical response. A 7- to 10-day course of treatment may be appropriate for those
who have simple periorbital cellulitis. Duration of therapy is guided by clinical response. Children who
have bacteremia should receive 10 days of parenteral therapy. Patients who have complicated disease
may require longer courses of antibiotics.

American Board of Pediatrics Content Specification(s):


Know the diagnostic approach for periorbital (preseptal) cellulitis
Know the treatment of periorbital (preseptal) cellulitis

Suggested Reading:

Copyright 2012 American Academy of Pediatrics


2012 PREP SA ON CD-ROM

Hauser A, Fogarasi S. Periorbital and orbital cellulitis.Pediatr Rev. 2010;31:242-249. DOI: 10.1542/pir.31-
6-242. Available at: http://pedsinreview.aappublications.org/cgi/content/full/31/6/242

Wald E. Periorbital and orbital infections. In: Long SS, Pickering LK, Prober CG, eds. Principles and
Practice of Pediatric Infectious Diseases. 3rd ed. Philadelphia, PA: Churchill Livingstone Elsevier,
2008:511-515

Copyright 2012 American Academy of Pediatrics


2012 PREP SA ON CD-ROM

Question 241
You are seeing a 10-year-old girl who has systemic lupus erythematosus (SLE) for a health
supervision visit. You diagnosed SLE when she was 8 years old, based on an initial presentation of
idiopathic thrombocytopenic purpura and a positive antinuclear antibody test and the subsequent
development of arthritis and a positive anti-double-stranded DNA test. She is currently doing well in
school and has no concerns about her memory or problem-solving skills. The only finding of note on her
physical examination is a slight erythematous rash in a malar distribution.
Of the following, the MOST useful screening test for other organ involvement in this girl is
A. Coombs test
B. erythrocyte sedimentation rate
C. magnetic resonance imaging of the brain
D. urinalysis
E. Venereal Disease Research Laboratory (VDRL) testing

Copyright 2012 American Academy of Pediatrics


2012 PREP SA ON CD-ROM

Critique 241 Preferred Response: D


The girl described in the vignette presents with a malar rash, which is the dermatologic
manifestation of systemic lupus erythematosus (SLE). It is important to screen the girl for the
development of renal disease, which can be subclinical initially, and urinalysis is appropriate to screen for
occult renal disease. Because the greatest morbidity of SLE results from renal or central nervous system
(CNS) disease, clinicians need to maintain a low threshold for screening for both. Disease involving the
CNS may manifest with seizures, psychosis, headaches, difficulty with concentration, or memory
problems. Magnetic resonance imaging of the brain is not warranted in this girl in the absence of such
symptoms. Screening for other organ involvement such as Coombs-positive hemolytic anemia is
appropriate for a child who is pale or in whom suspicion of anemia is high, but delayed diagnosis of
hemolytic anemia is less concerning than missing occult renal disease. The erythrocyte sedimentation
rate is likely to be elevated in the presence of an apparent flare of her inflammatory disease, but this test
result is nonspecific. A false-positive VDRL test is common in lupus, but this finding is nonspecific and
would add little to the understanding of the clinical status of a young girl experiencing an apparent lupus
flare.
Renal disease occurs in 75% to 80% of children who have SLE, usually within 2 years of
diagnosis. Renal manifestations are not typically apparent at diagnosis; most patients present with fever,
rash, arthritis, and mucositis. The combination of high frequency, high morbidity, and clinically silent
nature of renal disease makes screening paramount. Typical screening includes urinalysis with
microscopy, urine protein-to-creatinine ratio, and measurement of serum creatinine and albumin. Renal
manifestations can range from asymptomatic hematuria and proteinuria to overt nephritis (gross
hematuria, azotemia, and hypertension), nephrotic syndrome, or hypertension. Patients in whom
proteinuria (>1+ on a dipstick or urine protein-to-creatinine ratio >0.2 [mg/mg]) is discovered warrant
expeditious nephrology consultation for assessment and possible renal biopsy.

American Board of Pediatrics Content Specification(s):


Understand that renal disease is a common complication of systemic lupus erythematosus

Suggested Reading:
Gottlieb BS, Ilowite NT. Systemic lupus erythematosus in children and adolescents. Pediatr Rev.
2006;27:323-330. DOI: 10.1542/pir.27-9-323. Available at:
http://pedsinreview.aappublications.org/cgi/content/full/27/9/323

Nester CM, Thomas DB, Gipson DS. Kidney in systemic lupus erythematosus and vasculitis. In: Kher KK,
Schnaper HW, Makker SP, eds. Clinical Pediatric Nephrology. 2nd ed. London, England: Informa Healthcare;
2007:245-260

Copyright 2012 American Academy of Pediatrics


2012 PREP SA ON CD-ROM

Question 242
A 12-year-old girl is admitted to the intensive care unit with anaphylactic shock after being stung
by a wasp. She was playing outside when she was stung twice on her arm. Within 10 minutes, she
developed abdominal pain and dyspnea. Her parents attempted to administer an oral antihistamine, but
the girl vomited and then collapsed. Emergency medical services arrived in 10 minutes to find an
unresponsive child whose initial systolic blood pressure was 60 mm Hg. They administered intravenous
fluids and two doses of epinephrine, which improved the girls level of consciousness and blood pressure.
Her parents, who are physicians, have many questions about wasp stings and what can be done to
prevent reactions in the future.
Of the following, you are MOST likely to advise the parents that
A. Hymenoptera immunotherapy decreases the risk of anaphylaxis with future stings to
approximately 30%
B. Hymenoptera immunotherapy is contraindicated due to the severity of the girls sting reaction
C. initial allergy testing should be performed within 2 weeks after the sting reaction
D. serum tryptase should be measured 1 to 2 weeks after the reaction
E. the girls future risk for anaphylaxis to a wasp sting is less than 10% per sting

Copyright 2012 American Academy of Pediatrics


2012 PREP SA ON CD-ROM

Critique 242 Preferred Response: D


The girl in the vignette developed anaphylactic shock after a wasp sting and should be referred to
an allergist for further evaluation. Approximately 1% to 10% of patients who experience severe
anaphylaxis after an insect sting have underlying mastocytosis or a clonal mast cell disorder. For patients
who experience a severe anaphylaxis episode to a flying insect sting, a baseline serum tryptase should
be obtained 1 to 2 weeks after the reaction. Those who have serum tryptase measurements of 11.4
ng/mL or greater should be referred to an immunologist or hematologist for consideration of a bone
marrow biopsy, with specific attention to analysis for c-kit mutations, CD25, and mast cell staining.
Patients who develop severe immunoglobulin (Ig)E-mediated symptoms are at increased risk for
similar symptoms with future stings, with a 20% to 60% incidence per sting. Diagnosis of stinging insect
allergy involves a detailed history, followed by either venom skin testing, serum venom-specific IgE
testing, or both. Due to the risk of anaphylaxis during skin testing, some allergists start with serum IgE
testing in patients who have experienced life-threatening venom anaphylaxis. However, serum testing
results may be negative in 10% to 20% of patients. For those who have negative results to serum IgE
testing, skin testing is the next step. Serum testing may be performed at any time, but initial venom skin
testing should be performed 4 to 6 weeks after the reaction. Although skin testing at 2 weeks postreaction
has been shown to identify 70% to 80% of patients, waiting until 4 to 6 weeks increases identification to
90%. Further, patients who have negative skin testing results despite a strong clinical history should have
repeat skin testing 1 to 3 months later.
Because of the severity of this childs reaction, Hymenoptera immunotherapy should be strongly
considered because without immunotherapy, children who have had anaphylaxis have a 20% to 60% risk
per sting of another episode. This risk is reduced to 10% for honey bees and 5% for other flying insects
with Hymenoptera immunotherapy. The immunotherapy is continued for 3 to 5 years before considering
discontinuation, although patients who experience a near-fatal reaction may require lifelong
immunotherapy.

American Board of Pediatrics Content Specification(s):


Recognize life-threatening reactions to Hymenoptera stings (hypotension, wheezing, laryngeal
edema)

Suggested Reading:
Bonadonna P, Perbellini O, Passalacqua G, et al. Clonal mast cell disorders in patients with systemic
reactions to Hymenoptera stings and increased serum tryptase levels. J Allergy Clin Immunol.
2009;123:680-686. DOI: 10.1016/j.jaci.2008.11.018. Available at: http://www.jacionline.org/article/S0091-
6749(08)02211-2/fulltext

Bonadonna P, Zanotti R, Mller U. Mastocytosis and insect venom allergy. Curr Opin Allergy Clin
Immunol. 2010;10:347-353. DOI: 10.1097/ACI.0b013e32833b280c. Abstract available at:
http://www.ncbi.nlm.nih.gov/pubmed/20485157

Booker GM, Adam HM. In brief: insect stings. Pediatr Rev. 2005;26:388-389. DOI: 10.1542/pir.26-10-388.
Available at: http://pedsinreview.aappublications.org/cgi/content/full/26/10/388

Copyright 2012 American Academy of Pediatrics


2012 PREP SA ON CD-ROM

Question 243
Emergency medical services personnel bring a 3-month-old infant to the emergency department
in status epilepticus. The mother reports that the infant was well when she put her to sleep approximately
4 hours ago and that she found her seizing when she checked on her 20 minutes ago. On initial physical
examination, the infants temperature is 35.6C, heart rate is 175 beats/min, respiratory rate is 45
breaths/min, blood pressure is 90/60 mm Hg, and pupils are equally round and reactive. She is having
equally symmetric, generalized tonic-clonic movements of her extremities. Her bedside glucose
determination is 145 mg/dL (8.0 mmol/L). After two doses of midazolam, the seizure activity stops. A
head computed tomography scan is read as normal, and further testing is initiated.
Of the following, the assessment that is MOST likely to yield abnormal results for this infant is
A. cerebrospinal fluid
B. serum ammonia
C. serum calcium
D. serum sodium
E. skeletal survey

Copyright 2012 American Academy of Pediatrics


2012 PREP SA ON CD-ROM

Critique 243 Preferred Response: D


Status epilepticus in an infant is a life-threatening emergency. The most common causes are
metabolic derangements, structural brain abnormalities, brain injury, and infection. The sudden onset of
seizures in a previously well infant, the lack of fever, and the normal head computed tomography scan
(CT) findings for the infant in the vignette are highly suggestive of a metabolic cause. Elevated serum
ammonia concentrations, as might be seen in a patient who has liver failure or a urea cycle defect, are
more likely to cause lethargy and coma than seizures, and although low serum calcium can cause status
epilepticus, the hypothermia noted in this infant is a well-recognized sign of hyponatremia. A skeletal
survey to rule out occult fractures would have been indicated if the head CT scan had revealed
hemorrhage or other signs of trauma that might have been due to child abuse. Finally, the sudden onset
of seizures in the absence of fever is an uncommon presentation of meningitis.
Hyponatremia in an infant caused by water intoxication is a well-recognized clinical entity, most
often resulting from inappropriate feeding practices. Whether because of confusion about formula
reconstitution, purposeful overdilution of formula, or overfeeding with water, affected infants receive
excess free water, leading to euvolemic hyponatremia. Lethargy, irritability, and seizures are the most
common presenting signs. The diagnosis is most commonly made on the basis of laboratory findings,
with subsequent corroboration, often many hours later, by patient history.
In the largest study of symptomatic infantile hyponatremia, the clinical features that distinguished
infants who had hyponatremia and presented to the emergency department with seizures from those who
had normal sodium values were hypothermia (temperature <36.5C) and increased seizure severity, as
indicated by greater length of seizures, development of status epilepticus, and increased need for
anticonvulsant therapy. The seizures were more difficult to stop and the infants required emergency
intubation more frequently. Early recognition and subsequent treatment of severe hyponatremia in these
patients to a serum sodium concentration of more than 125 mEq/L (125 mmol/L) with 3% saline solution
is an important adjunct to seizure control and overall outcome.

American Board of Pediatrics Content Specification(s):


Recognize water intoxication in an infant

Suggested Reading:
Farrar HC, Chande VT, Fitzpatrick DF, Shema SJ. Hyponatremia as the cause of seizures in infants: a
retrospective analysis of incidence, severity, and clinical predictors. Ann Emerg Med. 1995;26:42-48.
Abstract available at: http://www.ncbi.nlm.nih.gov/pubmed/7793719

Greenbaum LA. The pathophysiology of body fluids and fluid therapy: electrolyte and acid-base disorders:
sodium. In: Kliegman RM, Stanton BF, St. Geme JW III, Schor NF, and Behrman RE, eds. Nelson
Textbook of Pediatrics. 19th ed. Philadelphia, PA: Saunders Elsevier; 2011:212-242

Martindale JL, Goldstein JN, Pallin DJ. Emergency department seizure epidemiology. Emerg Med Clin
North Am. 2011;29:1527. DOI: 10.1016/j.emc.2010.08.002 Available at:
http://www.emed.theclinics.com/article/PIIS0733862710000751/fulltext

Scott RC, Kirkham FJ. Clinical update: childhood convulsive status epilepticus. Lancet. 2007;370:724-
726. DOI: 10.1016/S0140-6736(07)61357-9

Copyright 2012 American Academy of Pediatrics


2012 PREP SA ON CD-ROM

Question 244
You are asked to see a 6-week-old infant who has just been hospitalized because of an apparent
life-threatening event. The baby was delivered via cesarean section at term following an uncomplicated
pregnancy because of failure of labor progression. He weighed 3,200 g. He was started on cow milk
protein-based formula at birth. At about 2 weeks of age, he developed postprandial emesis, and these
episodes have increased so that the baby spits up once or twice after each feeding. He currently
consumes 6 oz of formula every 3 to 4 hours. Several hours ago, immediately after feeding, the baby
appeared to be choking, stopped breathing, and developed perioral cyanosis. He expelled formula from
his nose and mouth. The parents called 911, and the infant was brought to the emergency department,
where he appeared active and alert and had a weight of 4,400 g. The infant was admitted for evaluation
and observation.
Of the following, the MOST appropriate next step is
A. barium esophagography
B. intraesophageal pH monitoring
C. lansoprazole administration
D. ranitidine administration
E. reduced feeding volumes

Copyright 2012 American Academy of Pediatrics


2012 PREP SA ON CD-ROM

Critique 244 Preferred Response: E


An apparent life-threatening event (ALTE) is an observer-dependent phenomenon that is
characterized by a combination of apnea, abnormal muscle tone (limpness, rigidity), choking, and color
change (pallor, cyanosis, plethora). ALTEs typically are reported in infants at 1 to 2 months of age and
are rarely described after age 8 months. Although specific causes are poorly understood,
gastroesophageal reflux (GER) has been implicated. However, supportive data are conflicting, and the
relationship between apnea and GER remains controversial. Accordingly, for the infant described in the
vignette, whose clinical history otherwise suggests a diagnosis of uncomplicated GER that may be
exacerbated by overfeeding, current best evidence suggests that the most appropriate approach to
management is to reduce feeding volumes and increase feeding frequency.
GER is defined by the passage of gastric contents into the esophagus, with or without
regurgitation or vomiting. It is a normal physiologic process occurring several times per day in healthy
infants, children, and adults. Most episodes are transient, last fewer than 3 minutes, occur in the initial 2
hours postprandially, and are followed by rapid esophageal clearance of the refluxate. Such brief,
recurring GER events are generally unassociated with clinical signs or symptoms. During the first 3
postnatal months, regurgitation (defined as effortless passage of gastric contents into the mouth) or
vomiting events (which may be projectile) are noted daily in 50% of infants. These GER episodes arise
from several potential anatomic and physiologic mechanisms, the most prominent being repeated
transient relaxations of the lower esophageal sphincter (LES). As in the vignette, a diagnosis of GER in
infants is based solely upon a history of spitting-up. Formal intraesophageal pH monitoring is of no value
in routine evaluation. This study should be reserved for assessing equivocal reflux cases or in an attempt
to correlate symptoms with reflux episodes. Barium esophagraphy cannot quantify the extent or severity
of GER and is used solely to document anatomic integrity.
Symptoms and signs that have been associated with prolonged or increased GER include both
respiratory and nonrespiratory events (Item C244). Although regurgitation or vomiting are not consistent
findings in older patients who have GER-related complications, spitting-up, with or without expulsion of
gastric contents from the mouth, is considered a necessary condition for reaching a clinical diagnosis of
reflux during infancy.
Early studies suggested that a mechanism for reflux-induced apnea involved acid stimulation of
pharyngeal and esophageal chemoreceptors, leading to laryngospasm. However, more recent large case
series have failed to demonstrate a consistent GER-apnea link. Where respiratory status was monitored
along with both esophageal pH and bioelectrical impedance in infants presenting with a history of apnea,
only 15% of apneic episodes were correlated with GER. Furthermore, these episodes were as likely to
occur with non-acid as with acid GER. Additional data have failed to demonstrate any clinical efficacy of
acid reduction therapy in preventing or ameliorating apnea events. Thus, acid blockade with either a
histamine-2-receptor antagonist (ranitidine) or a proton pump inhibitor (lansoprazole) is not indicated in
this clinical setting.
GER has also been implicated as a causative or exacerbating factor for other respiratory
disorders. Where studied using esophageal pH monitoring, 60% to 70% of children who have reactive
airway disease demonstrate pathologic reflux. However, whether this degree of GER is a primary problem
or a secondary phenomenon caused by lung hyperinflation and downward movement of the diaphragm,
leading to displacement of the LES into the chest accompanied by a reduction in LES pressure, is
unclear. Several studies have attempted to assess the role of acid reduction therapy in ameliorating
asthma symptoms in both children and adults, and most have failed to demonstrate consistent treatment
efficacy. However, a few reports have shown improvement in asthma symptoms following acid blockade
for patients who have poorly controlled asthma, especially those who have predominantly nocturnal
symptoms.
GER has been implicated in causing recurrent pneumonia and interstitial lung disease, especially
in children who have significant neurologic impairment. In these cases, lung disease presumably results
from failure of normal airway mechanisms to protect the lungs from aspirated gastric contents. Several
case series have presented conflicting results regarding the efficacy of either medical or surgical GER
therapy in improving lung function and reducing the risk of pneumonia in affected patients. Because
gastroesophageal and respiratory function are closely linked, the finding of pathologic GER in patients
who have chronic pulmonary disease is not surprising. In one report, 27% of patients who had cystic

Copyright 2012 American Academy of Pediatrics


2012 PREP SA ON CD-ROM

fibrosis reported symptoms of heartburn. However, where studied by pHometry, the prevalence of
pathologic GER in cystic fibrosis was even higher. Accordingly, GER may be considered an exacerbating
factor for children and adults who have a wide range of chronic respiratory disorders, but as previously
discussed, the effectiveness of acid blockade on respiratory symptoms in these patients has not been
clearly demonstrated.

American Board of Pediatrics Content Specification(s):


Recognize the association between gastroesophageal reflux and respiratory symptoms

Suggested Reading:
Akinola E, Rosenkrantz TS, Pappagallo M, McKay K, Hussain N. Gastroesophageal reflux in infants <32
weeks gestational age at birth: lack of relationship to chronic lung disease. Am J Perinatol. 2004;21:57
62. DOI: 10.1055/s-2004-820512. Abstract available at: http://www.ncbi.nlm.nih.gov/pubmed/15017467

Condino AA, Sondheimer J, Pan Z, Gralla J, Perry D, OConnor JA. Evaluation of gastroesophageal
reflux in pediatric patients with asthma using impedance-pH monitoring. J Pediatr. 2006;149:216219.
Abstract available at: http://www.ncbi.nlm.nih.gov/pubmed/16887437

Mousa H, Woodley FW, Metheney M, Hayes J. Testing the association between gastroesophageal reflux
and apnea in infants. J Pediatr Gastroenterol Nutr. 2005;41:169177. Available at:
http://journals.lww.com/jpgn/Fulltext/2005/08000/Testing_the_Association_Between_Gastroesophageal.4
.aspx

Strdal K, Johannesdottir GB, Bentsen BS, et al. Acid suppression does not change respiratory
symptoms in children with asthma and gastro-oesophageal reflux disease. Arch Dis Child. 2005;90:956
960. DOI: 10.1136/adc.2004.068890. Available at:
http://www.ncbi.nlm.nih.gov/pmc/articles/PMC1720585/?tool=pubmed

Vandenplas Y, Hauser B. Gastroesophageal reflux, sleep pattern, apparent life threatening event and
sudden infant death. The point of view of a gastroenterologist. Eur J Pediatr. 2000;159:726729. DOI:
10.1007/s004310000544. Abstract available at: http://www.ncbi.nlm.nih.gov/pubmed/11039125

Vandenplas Y, Rudolph CD, Di Lorenzo C, et al. Pediatric gastroesophageal reflux clinical practice
guidelines: joint recommendations of the North American Society for Pediatric Gastroenterology,
Hepatology, and Nutrition (NASPGHAN) and the European Society for Pediatric Gastroenterology,
Hepatology, and Nutrition (ESPGHAN). J Pediatr Gastroenterol Nutr. 2009;49:498-547. DOI:
10.1097/MPG.0b013e3181b7f563. Available at:
http://journals.lww.com/jpgn/Fulltext/2009/10000/Pediatric_Gastroesophageal_Reflux_Clinical.22.aspx

Copyright 2012 American Academy of Pediatrics


2012 PREP SA ON CD-ROM

Critique 244

Item C244. Symptoms and Signs Possibly Associated With Gastroesophageal Reflux
Respiratory Nonrespiratory
! Symptoms: ! Symptoms:
o Cough o Chest pain
o Hoarseness o Dysphagia, odynophagia
o Stridor o Hematemesis
o Wheezing o Irritability
o Poor weight gain/weight loss
o Recurrent regurgitation/vomiting
o Rumination
! Signs: ! Signs:
o Apparent life-threatening events o Anemia
o Laryngeal/pharyngeal inflammation o Barrett esophagus
o Recurrent pneumonia o Dental erosion
o Dystonic neck posturing (Sandifer
syndrome)
o Esophageal stricture
o Esophagitis
o Feeding refusal

Adapted from Vandeplas Y, Rudolph CD, Di Lorenzo C, et al. Pediatric gastroesophageal reflux clinical
practice guidelines: joint recommendations of the North American Society for Pediatric Gastroenterology,
Hepatology, and Nutrition (NASPGHAN) and the European Society for Pediatric Gastroenterology,
Hepatology, and Nutrition (ESPGHAN). J Pediatr Gastroenterol Nutr. 2009;49:498-547

Copyright 2011 American Academy of Pediatrics


2012 PREP SA ON CD-ROM

Question 245
You are called by an obstetric colleague about the antibiotic of choice for intrapartum prophylaxis
for group B Streptococcus (GBS) infection. A 22-year-old primigravida has presented in labor at 39
weeks gestation. She is currently afebrile with intact membranes. Her prenatal records demonstrate a
positive GBS screening result at 36 weeks gestation. Upon questioning, the woman states that she has a
penicillin allergy that manifests with a faint rash but no anaphylaxis, angioedema, or urticaria.
Of the following, the MOST appropriate antibiotic for intrapartum prophylaxis for this patient is
A. azithromycin
B. cefazolin
C. clindamycin
D. erythromycin
E. vancomycin

Copyright 2012 American Academy of Pediatrics


2012 PREP SA ON CD-ROM

Critique 245 Preferred Response: B


Cefazolin is the most appropriate antibiotic for intrapartum group B Streptococcus (GBS)
prophylaxis for the woman described in the vignette, who reports being allergic to penicillin, although she
has no history of anaphylaxis, angioedema, respiratory distress, or urticaria with penicillin or
cephalosporin administration. In spite of the use of antibiotics for intrapartum prophylaxis since the early
1990s, GBS continues to be sensitive to ampicillin, penicillin, and first-generation cephalosporins. The
penicillin-allergic woman at high risk for anaphylaxis who is colonized with GBS should have antimicrobial
susceptibility testing ordered on the antenatal GBS culture. Because resistance rates ranging from 25%
to 50% are seen with erythromycin, it is no longer an acceptable alternative. Clindamycin, with resistance
rates ranging from 13% to 20%, is used in these women if inducible clindamycin resistance is negative.
Vancomycin is reserved for those penicillin-allergic women at high risk for anaphylaxis with a GBS isolate
that is resistant to clindamycin. Azithromycin is not used for intrapartum GBS prophylaxis, although it may
be used to treat Chlamydia infection.
GBS has been the leading cause of early-onset neonatal sepsis since the 1970s, with vertical
transmission of the organism from the vagina of a colonized woman to the amniotic fluid after the onset of
labor or rupture of membranes in most cases. The incidence of early-onset neonatal GBS sepsis has
declined 80% since the initiation of screening, with 1,200 cases estimated annually using surveillance
studies. Of these cases, 70% have occurred in term infants, and the mortality rate in this subset is 2% to
3%.
Due to the increased risk of early-onset GBS sepsis in women who have a history of GBS
isolated from the urine at any time in the current pregnancy or an infant who was previously affected with
invasive GBS disease should receive intrapartum antibiotic prophylaxis and do not require screening. In
all other situations, screening for GBS colonization status is performed in the late third trimester because
screening cultures obtained 5 or fewer weeks before delivery have a 95% to 98% negative predictive rate.
Upon the onset of labor or with rupture of membranes, all women colonized with GBS receive intrapartum
antibiotic prophylaxis, based upon maternal history of penicillin allergy and GBS isolate sensitivities, if
indicated. Antibiotic prophylaxis is not routinely recommended for women who have intact membranes
when the cesarean section is performed before the onset of labor. Treatment strategies have also been
developed for women presenting in labor whose GBS status is unknown.

American Board of Pediatrics Content Specification(s):


Understand the importance of maternal screening for group B Streptococcus and the appropriate
treatment of women with positive results

Suggested Reading:
American Academy of Pediatrics. Group B streptococcal infections. In: Pickering LK, Baker CJ, Kimberlin
DW, Long SS, eds. Red Book: 2009 Report of the Committee on Infectious Diseases. 28th ed. Elk Grove
Village, IL: American Academy of Pediatrics; 2009:628-634

Centers for Disease Control and Prevention. Prevention of perinatal group B streptococcal disease.
Revised guidelines from CDC, 2010. MMWR Recomm Rep. 2010;59(RR10):1-32. Available at:
http://www.cdc.gov/mmwr/preview/mmwrhtml/rr5910a1.htm?s_cid=rr5910a1_w

Centers for Disease Control and Prevention. Trends in perinatal group B streptococcal disease-United
States, 2000-2006.MMWR Morb Mortal Wkly Rep. 2009;58:109-112. Available at:
http://www.cdc.gov/mmwr/preview/mmwrhtml/mm5805a2.htm

Copyright 2012 American Academy of Pediatrics


2012 PREP SA ON CD-ROM

Question 246
A 16-month-old girl is brought to the emergency department because of persistent crying for
several hours. She has had clear rhinorrhea, cough, and an undocumented fever for 3 days. She has had
adequate oral intake, has vomited twice, and has had no diarrhea. She has no underlying medical
conditions and no exposures. Current medications include acetaminophen and an over-the-counter
cough and cold medicine. Physical examination reveals an agitated, crying child, who is inconsolable. Her
temperature is 37.8C, heart rate is 192 beats/min, respiratory rate is 36 breaths/min, blood pressure is
122/78 mm Hg, and oxygen saturation is 98%. The remainder of physical examination findings are
normal. Complete blood count, electrolytes, urinalysis, and cerebrospinal fluid studies yield normal
results. Electrocardiography shows sinus tachycardia.
Of the following, the MOST likely cause of this childs symptoms is
A. acetaminophen overdose
B. early meningitis
C. intussusception
D. myocarditis
E. reaction to cold and cough medicine

Copyright 2012 American Academy of Pediatrics


2012 PREP SA ON CD-ROM

Critique 246 Preferred Response: E


The differential diagnosis for an irritable and crying child, such as the girl described in the
vignette, is broad, ranging from trauma to infection to metabolic conditions to toxicologic issues. This girls
evaluation has ruled out many of the serious entities in the differential diagnosis, such as meningitis. In
fact, her history and abnormal vital signs suggest a likely cause: toxic effects from cough and cold
medication.
Over-the-counter cough and cold medications may contain one or more components, usually with
decongestant, antihistamine, expectorant, or cough suppressant effects. Multiple studies have shown that
these medications have little or no efficacy in the treatment of upper respiratory tract infections in children
compared with placebo. For this reason, the United States Food and Drug Administration (FDA) was
petitioned and convened a meeting in 2007 to review the safety of these products. Before the meeting,
however, major manufacturers pulled from production products marketed for children younger than 2
years of age and added a label warning against using the medications for sedative purposes. Although
experts further recommended that the products be eliminated for children younger than 6 years of age, a
pharmaceutical trade group subsequently voluntarily changed labeling to warn about their use in children
younger than 4 years.
The most common decongestant in cold and cough preparations is pseudoephedrine, which may
have sympathomimetic effects, such as tachycardia, arrhythmia, hypertension, central nervous system
stimulation, and rarely seizures. Antihistamines such as diphenhydramine, brompheniramine,
chlorpheniramine, and carbinoxamine can lead to central nervous system agitation or depression,
dysrhythmias, hypertension, seizures, and in extreme cases, respiratory depression. Guaifenesin, the
most commonly used expectorant, is generally well tolerated but may cause mild gastrointestinal
discomfort. Dextromethorphan and codeine are the most widely used cough suppressants and appear to
have efficacy greater than placebo in adults, but such efficacy is not noted for children.
Dextromethorphan is derived from opiates and acts at the central nervous system level to inhibit cough.
Although it does not have the addictive or analgesic properties of other opiates, it can cause euphoria,
hallucinations, lethargy, coma, nausea, dizziness, drowsiness, ataxia, nystagmus, and urinary retention. It
does have abuse potential because of the euphoric effects it may produce. In lower doses, codeine can
lead to somnolence, ataxia, gastrointestinal distress, and pruritus; in higher doses, it leads to depressed
mental status and respiratory depression. Since 1997, the American Academy of Pediatrics has
recommended against the use of codeine- and dextromethorphan-containing cough remedies in children.
Recently, the use of cold and cough preparations has been implicated in the deaths of a number of
infants and children, further strengthening the argument against their use.
The absence of abnormalities in the cerebrospinal fluid of this girl makes meningitis very unlikely,
even at an early stage. Intussusception causes episodes of crying in children, but they are typically
intermittent rather than continuous. Alternatively, intussusception may present with lethargy; continuous
agitation is not typical. Myocarditis may result in tachycardia out of proportion to fever, but usual
electrocardiographic findings, including decreased precordial voltages, are absent in this patient. Because
some cold and cough preparations may contain analgesics, it is prudent to measure the serum
concentration of acetaminophen for this patient, but agitation, tachycardia, and hypertension are not
characteristic of acetaminophen overdose.

American Board of Pediatrics Content Specification(s):


Recognize that over-the-counter cough and cold preparations have not been adequately studied in
children younger than 6 years of age and that they are not recommended for use for the common
cold
Know the active ingredients and the common adverse effects and toxicities of over-the-counter cough
and cold preparations (eg, dextromethorphan, antihistamines, pseudoephedrine/phenylephrine,
guaifenesin)

Suggested Reading:
Carr BC. Efficacy, abuse, and toxicity of over-the-counter cough and cold medicines in the pediatric
population. Curr Opin Pediatr. 2006;18:184-188. DOI: 10.1097/01.mop.0000193274.54742.a1. Abstract
available at: http://www.ncbi.nlm.nih.gov/pubmed/16601501

Copyright 2012 American Academy of Pediatrics


2012 PREP SA ON CD-ROM

Committee on Drugs. American Academy of Pediatrics. Use of codeine- and dextromethorphan-


containing cough remedies in children. Pediatrics. 1997;99:918-920. DOI: 10.1542/peds.99.6.918.
Available at: http://pediatrics.aappublications.org/cgi/content/full/99/6/918

Gunn VL, Taha SH, Liebelt EL, Serwint JR. Toxicity of over-the-counter cough and cold medications.
Pediatrics. 2001;108:e52. Available at: http://pediatrics.aappublications.org/cgi/content/full/108/3/e52

Kuehn BM. Debate continues over the safety of cold and cough medicines for children. JAMA.
2008;300:2354-2356

Rimsza ME, Newberry S. Unexpected infant deaths associated with use of cough and cold medications.
Pediatrics. 2008;122:e318-e322. DOI: 10.1542/peds.2007-3813. Available at:
http://pediatrics.aappublications.org/cgi/content/full/122/2/e318

Sharfstein JM, North M, Serwint JR. Over the counter but no longer under the radar pediatric cough and
cold medications. N Engl J Med. 2007;357:2321-2324. Available at:
http://www.nejm.org/doi/full/10.1056/NEJMp0707400

Copyright 2012 American Academy of Pediatrics


2012 PREP SA ON CD-ROM

Question 247
A 7-year-old boy comes to the emergency department complaining of back pain for the past 3
weeks after falling on the playground. A few days after the onset of pain, he was evaluated at an urgent
care center, where the examination and plain radiographs were reported to be normal. After initially
improving, the pain has become progressively worse, and recently he developed fever. On physical
examination, his vital signs are normal except for a temperature of 38.3C. He seems mildly
uncomfortable and has difficulty climbing onto the examination table because of the pain. Examination of
the back reveals tenderness over the midline lumbar region and spasm of the paraspinal muscles.
Findings on neurologic examination are normal. A complete blood count reveals a white blood cell count
3 9
of 15.0x10 /mcL (15.0x10 /L) with 76% polymorphonuclear leukocytes, 21% lymphocytes, and 3% bands.
The erythrocyte sedimentation rate is 45 mm/hr.
Of the following, the BEST test to establish the diagnosis for this child is
A. abdominal ultrasonography
B. blood culture
C. bone scan
D. computed tomography scan of the spine
E. magnetic resonance imaging of the spine

Copyright 2012 American Academy of Pediatrics


2012 PREP SA ON CD-ROM

Critique 247 Preferred Response: E


The boy described in the vignette has back pain, fever, tenderness to palpation, and paraspinal
muscle spasm, with elevation of his white blood cell count and erythrocyte sedimentation rate. Although
the differential diagnosis of back pain is broad, this constellation of symptoms suggests an infectious
process such as discitis, vertebral osteomyelitis, epidural abscess, or paraspinal abscess. The test that is
most likely to reveal the cause of this childs back pain is magnetic resonance imaging (MRI) of the spine.
For patients who have discitis, the MRI can show loss of disc space height and an abnormal signal in the
disk space. If discitis is complicated by osteomyelitis, there can also be an abnormal signal of the
adjacent vertebral bodies (Item C247A). On MRI, an epidural abscess may be demonstrated by a fluid
collection displacing the spinal cord and peripheral enhancement along the margins of the abscess (Item
C247B). Paraspinal abscess appears on MRI as a mass with uptake in the paraspinal structures outside
the vertebral canal.
Abdominal ultrasonography may be helpful in the evaluation of children who have abdominal pain
(which also may occur in some children who have discitis) but will not reveal the cause of this boys pain.
MRI is considered more sensitive than bone scan or computed tomography scan, although the latter may
be used to guide needle biopsy of the disk space if such testing is required. Although blood culture should
be obtained for the patient described in the vignette, results will not localize the cause of his back pain.
The differential diagnosis of back pain is extensive. Other causes include trauma,
musculoskeletal disorders (eg, nonspecific musculoskeletal back pain, spondylolysis, spondylolisthesis,
Scheuermann disease), neoplasms (eg, osteoid osteoma, leukemia, osteosarcoma), inflammatory
processes (eg, ankylosing spondylitis), and other conditions (eg, pneumonia, retrocecal appendicitis,
pyelonephritis, sickle cell pain crisis).

American Board of Pediatrics Content Specification(s):


Formulate a differential diagnosis for back pain in children and adolescents

Suggested Reading:
American College of Radiography. ACR Appropriateness Criteria. Available at:
http://www.acr.org/secondarymainmenucategories/quality_safety/app_criteria.aspx

Conrad DA. Acute hematogenous osteomyelitis. Pediatr Rev. 2010;31:464-471. DOI: 10.1542/pir.31-11-
464. Available at: http://pedsinreview.aappublications.org/cgi/content/full/31/11/464

Davis PJ, Williams HJ. Best practice: The investigation and management of back pain in children. Arch
Dis Child Educ Pract Ed. 2008;93:73-83 DOI: 10.1136/adc.2006.115535. Abstract available at:
http://www.ncbi.nlm.nih.gov/pubmed/18495896

Diepenmaat ACM, van der Wal MF, de Vet HCW, Hirasing RA. Neck/shoulder, low back, and arm pain in
relation to computer use, physical activity, stress, and depression among Dutch adolescents. Pediatrics.
2006;117:412-416. DOI: 10.1542/peds.2004-2766. Available at:
http://pediatrics.aappublications.org/cgi/content/full/117/2/412

Janner D, Barron SA. Index of suspicion: case 1. Pediatr Rev. 2007;28:27-32. DOI: 10.1542/pir.28-1-27.
Available at: http://pedsinreview.aappublications.org/cgi/content/full/28/1/27

Pellis F, Balagu F, Rajmil L, et al. Prevalence of low back pain and its effect on health-related quality of
life in adolescents. Arch Pediatr Adolesc Med. 2009;163:65-71. Available at: http://archpedi.ama-
assn.org/cgi/content/full/163/1/65

Rodriguez DP, Poussaint TY. Imaging of back pain in children. AJNR Am J Neuroradiol. 2010;1:787-802.
DOI: 10.3174/ajnr.A1832. Available at: http://www.ajnr.org/cgi/content/full/31/5/787

Copyright 2012 American Academy of Pediatrics


2012 PREP SA ON CD-ROM

Critique 247

(Courtesy of L Lowe)
Discitis in a 3-year-old boy who has back pain. Sagittal fat-suppressed, contrast-enhanced T1-weighted
magnetic resonance imaging shows loss of disc space height at L3-4 level (arrow), consistent with
discitis. Abnormal signal is also seen throughout the L3 and L4 vertebral bodies, which raises concern for
secondary osteomyelitis.

Copyright 2011 American Academy of Pediatrics


2012 PREP SA ON CD-ROM

Critique 247

(Courtesy of L Lowe)
Epidural abscess in a 16-year-old boy who has back pain. Sagittal fat-suppressed, contrast-enhanced
T1-weighted magnetic resonance imaging demonstrates a well-defined posterior fluid collection
displacing the spinal cord anteriorly at the L3-L5 level (arrows). Peripheral enhancement is visible along
the margins of the epidural abscess in addition to diffuse enhancement of distal thecal sac.

Copyright 2011 American Academy of Pediatrics


2012 PREP SA ON CD-ROM

Question 248
A 10-year-old girl presents to your office with fever, shortness of breath, and a recent history of
pharyngitis. Physical examination reveals tenderness of the left ankle without swelling or redness,
nontender nodules on the elbows (Item Q248) and knees, rales across both lung bases, and a II/VI high-
pitched blowing systolic ejection murmur at the apex. An antistreptolysin O titer is elevated, and 12-lead
electrocardiography identifies first-degree atrioventricular block. Echocardiography shows thickened
mitral valve leaflets with severe mitral regurgitation.
Of the following, the MOST appropriate treatment for this patient is
A. erythromycin and aspirin
B. erythromycin and corticosteroids
C. penicillin, furosemide, and aspirin
D. penicillin, furosemide, and corticosteroids
E. penicillin, furosemide, aspirin, and corticosteroids

Copyright 2012 American Academy of Pediatrics


2012 PREP SA ON CD-ROM

Question 248

(Courtesy of G Nankervis and the Red Book Online)


Small nodules (arrows) on the extensor surface of the elbow, as described for the girl in the vignette.

Copyright 2012 American Academy of Pediatrics


2012 PREP SA ON CD-ROM

Critique 248 Preferred Response: D


The girl described in the vignette has acute rheumatic fever based on the Jones criteria, including
evidence of obvious carditis. Oral prednisone is considered standard of care in treating severe acute
rheumatic carditis, particularly when accompanied by ventricular dysfunction leading to congestive heart
failure. Whether corticosteroid therapy changes the risk for long-term valve pathology is unclear.
Antibiotic therapy is used during the acute illness to control the immediate risk of persistent infection.
Furosemide is indicated for the girl in the vignette because she exhibits severe mitral regurgitation.
Although chronic mitral regurgitation does not typically require diuretic therapy, acute regurgitation is less
well tolerated. Acute regurgitation can lead to left atrial dilation, pulmonary congestion, left ventricular
dilation, and the eventual development of a dilated cardiomyopathy. Diuretic therapy may prove useful in
controlling the volume load on the left heart and potentially avert the development of left ventricular
dysfunction.
Penicillin (administered via either the intravenous or oral route) is considered first-line antibiotic
therapy. For those who have a penicillin allergy, an alternate antibiotic such as erythromycin can be used.
High-dose salicylate therapy can shorten the duration of acute symptoms, but there are no data that such
treatment reduces the risk of serious lifelong rheumatic heart disease. Combined anti-inflammatory
therapy with aspirin as well as corticosteroids is not typically beneficial other than for pain control due to
joint inflammation afforded by aspirin.

American Board of Pediatrics Content Specification(s):


Plan the initial management of acute rheumatic fever

Suggested Reading:
Cilliers AM. Rheumatic fever and its management. BMJ. 2006;333:1153-1156. DOI:
10.1136/bmj.39031.420637.BE. Available at:
http://www.ncbi.nlm.nih.gov/pmc/articles/PMC1676147/?tool=pubmed

Dajani A, Taubert K, Ferrieri P, Peter G, Shulman S; Committee on Rheumatic Fever, Endocarditis, and
Kawasaki Disease of the Council on Cardiovascular Disease in the Young, the American Heart
Association. Treatment of acute streptococcal pharyngitis and prevention of rheumatic fever: a statement
for health professionals. Pediatrics. 1995;96:758-764. Available at:
http://pediatrics.aappublications.org/cgi/content/abstract/96/4/758

Steer AC, Carapetis JR. Acute rheumatic fever and rheumatic heart disease in indigenous populations.
Pediatr Clin North Am. 2009;56:1401-1419. DOI: 10.1016/j.pcl.2009.09.011. Abstract available at:
http://www.ncbi.nlm.nih.gov/pubmed/19962028

Copyright 2012 American Academy of Pediatrics


2012 PREP SA ON CD-ROM

Question 249
An otherwise healthy 2-year-old child has mild unilateral ptosis on the right. On questioning, the
mother believes this eyelid always has been slightly droopy. A picture of the child from infancy confirms
this. On physical examination, the child has equal pupil sizes in light and dark and normal pupil reactions.
Extraocular movements are full and conjugate. Physical examination findings are otherwise normal.
Of the following, the MOST likely diagnosis is
A. congenital ptosis
B. Horner syndrome
C. Kearns-Sayre syndrome
D. myasthenia gravis
E. orbital tumor

Copyright 2012 American Academy of Pediatrics


2012 PREP SA ON CD-ROM

Critique 249 Preferred Response: A


The droopy eyelid since birth with no accompanying problems with pupils or eye movements
described for the child in the vignette is most consistent with congenital ptosis. Congenital ptosis involves
an isolated abnormality of the levator muscle of one or both upper eyelids. The eyelids upper crease is
often indistinct. The child may compensate by lifting the chin or eyebrow to improve vision under the
drooping lid. Treatment is surgical, usually during the late preschool years. Strabismus (ocular
malalignment) or anisometropia (differing refractive power of the eyes) often accompanies congenital
ptosis, so ophthalmologic evaluation is important.
Horner syndrome is a symptom complex related to sympathetic denervation and consisting of
partial ptosis, miosis (constricted pupil that does not dilate normally in darkness), and sometimes
anhidrosis (lack of sweating). The ptosis results from loss of sympathetic input to the tarsal muscle. This
ptosis is only partial because levator muscle function remains intact. The differential diagnosis is large
and includes congenital forms, trauma, and neoplasms. Congenital cases may have heterochromia.
Kearns-Sayre syndrome is a mitochondrial myopathy that causes chronic progressive external
ophthalmoplegia (CPEO), bilateral pigmentary retinopathy, and cardiac conduction abnormalities. The
CPEO includes paralysis of both muscles controlling the lids and eye movement. Onset is usually before
age 20 years.
Myasthenia gravis is a neuromuscular junction disorder, usually caused by autoantibodies that
bind to and partially block the acetylcholine receptor at the neuromuscular junction, resulting in fluctuating
weakness that increases with use. Eyelids are commonly involved, leading to unilateral or bilateral ptosis,
and involvement of other eye movement muscles can lead to diplopia. Characteristically, the strength of
affected muscles is better in the morning, and the weakness can be demonstrated by activating
(fatiguing) the muscles for a few minutes. For example, the examiner asks the patient to look up at a
target. Initially, the lids rise as the patient follows the instruction. Within a short period of time (<3
minutes), the levator muscles affected by myasthenia weaken and the lids droop.
Orbital tumors can cause mechanical constriction of lid and eye movement as well as a bulging
forward of the lid.

American Board of Pediatrics Content Specification(s):


Know the importance of differentiating congenital from acquired ptosis

Suggested Reading:
Guercio JR, Martyn LJ. Congenital malformations of the eye and orbit. Otolaryngol Clin North Am.
2007;40:113-140. DOI: 10.1016/j.otc.2006.11.013. Abstract available at:
http://www.ncbi.nlm.nih.gov/pubmed/17346564

Howard GR. Eyelid retraction. In: Yanoff M, Duker JS, eds. Ophthalmology. 3rd. Ed. St. Louis, MO:
Mosby Elsevier, 2009:1393-1396

Olitsky SE, Hug D, Plummer LS, Stass-Isern M. Abnormalities of the lids. In: Kliegman RM, Stanton BF,
St. Geme JW III, Schor NF, and Behrman RE, eds. Nelson Textbook of Pediatrics. 19th ed. Philadelphia,
PA: Saunders Elsevier; 2011:2163-2165

Copyright 2012 American Academy of Pediatrics


2012 PREP SA ON CD-ROM

Question 250
You are examining a 6-year-old girl who is new to your practice during a health supervision visit.
You note that she has a low posterior hairline and a rather short neck with decreased range of motion,
and her mother explains that she has a condition known as Klippel-Feil syndrome. Her father also has
this condition, and the mother explains that it runs on my husbands side of the family. A radiography
report provided by the mother confirms cervical vertebral fusion abnormalities consistent with Klippel-Feil
syndrome.
Of the following, the MOST likely additional clinical feature to expect in this patient is
A. conductive hearing loss
B. kyphosis
C. Madelung deformity
D. microtia
E. Sprengel anomaly

Copyright 2012 American Academy of Pediatrics


2012 PREP SA ON CD-ROM

Critique 250 Preferred Response: E


The typical clinical features of Klippel-Feil syndrome include a short neck, low posterior hairline,
and limited neck range of motion in the context of cervical vertebral segmentation defects (most often
fusion of C2-C3). Sprengel anomaly (upward displacement of the scapula) is seen in as many as 50% of
affected patients (Item C250). Although Klippel-Feil syndrome most often is identified in individuals who
have no family history of similar defects (ie, isolated cases), both autosomal dominant and autosomal
recessive pedigrees have been described in the literature. Hearing problems are seen in nearly 80% of
affected individuals, but sensorineural and mixed hearing loss significantly outnumber cases of pure
conductive hearing loss. Scoliosis and microtia are less common associated defects; kyphosis is not a
recognized associated finding. Madelung wrist deformity results from asymmetric growth of the radius,
causing volar displacement of the hand and wrist and dorsal protrusion of the ulna. This is not seen as
part of Klippel-Feil syndrome, but may occur following trauma or seen as part of certain skeletal
dysplasias or metabolic conditions affecting bone growth.
Much less common associated findings include facial asymmetry; laryngeal cartilage
malformations; cleft palate; and restricted movement of elbows, wrists, ankles, or knees with radiographic
evidence of carpal and tarsal bony fusion. Embryologists have postulated that the features of Klippel-Feil
syndrome are etiologically consistent with disruption in the vascular supply to specific regions of the body
(specifically, the subclavian arteries, vertebral arteries, or their branches), much like other multiple
congenital anomaly field-defect syndromes such as Poland anomaly and Moebius sequence.

American Board of Pediatrics Content Specification(s):


Know the clinical and radiologic features of Klippel-Feil syndrome

Suggested Reading:
Hoyme HE, Jones KL, Dixon SD, et al. Prenatal cocaine exposure and fetal vascular disruption.
Pediatrics. 1990;85:743-747. Abstract available at:
http://pediatrics.aappublications.org/cgi/content/abstract/85/5/743

Joint Committee on Infant Hearing. Year 2007 position statement: principles and guidelines for early
hearing detection and intervention programs. Pediatrics. 2007;120:898-921. DOI: 10.1542/peds.2007-
2333. Available at: http://pediatrics.aappublications.org/cgi/content/full/120/4/898

Copyright 2012 American Academy of Pediatrics


2012 PREP SA ON CD-ROM

Critique 250

(Courtesy of E Anthony)
Anterior-posterior radiograph of the cervical spine shows multilevel segmentation anomalies (ie,
congenital fusion) in the mid-to-lower cervical spine. This patient also has a common association,
Sprengel deformity (elevation and medial rotation of the left scapula [arrow]).

Copyright 2011 American Academy of Pediatrics


2012 PREP SA ON CD-ROM

Question 251
The parents of a 15-year-old boy would like your help in sending their son to a drug rehabilitation
facility. He disagrees and feels he can stop use of illicit drugs on his own. He reportedly started smoking
and drinking at age 12 years. His mother recently found drug paraphernalia in his room, prompting the
visit. He tells her that his friend asked him to hold these items for him but they are not his. In deciding on
a plan, you review risk factors.
Of the following, this boys prognosis is MOST influenced by
A. age at which substance use began
B. duration of substance use
C. his current age
D. peer substance use
E. types of substances used

Copyright 2012 American Academy of Pediatrics


2012 PREP SA ON CD-ROM

Critique 251 Preferred Response: A


Adolescents who choose to experiment with illicit drugs usually start with substances that are
legal for use in adults and, thus, more easily accessible, such as tobacco and alcohol, often followed by
marijuana. These three substances frequently are referred to as gateway drugs, whose use precedes
progression to harder drugs. Some younger children may start with inhalants available in the household
and progress to tobacco and alcohol when they become available. Trends in use of these substances can
be obtained from surveys, including the Monitoring the Future Survey (MTF), sponsored by the National
Institute of Drug Abuse; the Youth Risk Behavior Surveillance Survey (YRBSS) from the Centers for
Disease Control and Prevention; and the National Survey on Drug Use and Health (NSDUH), conducted
by the Substance Abuse and Mental Health Services Administration (SAMHSA). Recent results from such
surveys have noted that alcohol and tobacco use are on the decline, while marijuana use is on the rise
after years of decline.
Trying substances a few times is a relatively common behavior among adolescents, but further
experimentation, especially at a young age (before age 14 years) has been linked to future abuse and
dependence. The younger the age of onset, the more likely is the presence of other risk factors, such as
family use of substances, family disorganization, lack of supervision, personality issues (eg, impulsivity),
and school problems. In this context, continued use of substances becomes a coping mechanism, and
the adolescent does not learn other healthier ways to deal with issues.
The young age at which the boy in the vignette began using drugs coupled with his probable use
alone in his room, especially if there are other behavioral (eg, school absenteeism, falling grades, risky
sexual behavior) and psychiatric problems, supports the need for intensive intervention. Adolescents who
begin use later and use substances for a shorter time are more likely to be able to stop. Even those who
use multiple substances may be able to quit all use if they have few other risk factors. Peer substance
use may normalize the use of substances and make them more easily available, but without other risk
factors, most adolescents can resist peer pressure. The route of administration rather than the substance
itself is often more of a concern. The rapidity of onset of action, as with inhalation or injection, makes the
substance more addicting and, thus, harder to give up.

American Board of Pediatrics Content Specification(s):


Understand the general trends in substance use and abuse for alcohol, marijuana, and tobacco in the
US among children and adolescents

Suggested Reading:
Eaton DK, Kann L, Kinchen S, et al. Youth Risk Behavior Surveillance --United States, 2007. MMWR
Surv Summ. 2008;57(SS4):1-131. Available at:
http://www.cdc.gov/mmwr/preview/mmwrhtml/ss5704a1.htm

Johnston LD, O'Malley PM, Bachman HG, Schulenberg JE. Monitoring the Future National Results on
Adolescent Drug Use: Overview of Key Findings, 2009. Bethesda, MD: National Institute on Drug Abuse;
2010. Available at: http://www.monitoringthefuture.org/pubs/monographs/overview2009.pdf

Kandel DB, Yamaguchi K, Chen K. Stages of progression in drug involvement from adolescence to
adulthood: further evidence for the gateway theory. J Stud Alcohol. 1992;53:447-457. Abstract available
at: http://www.ncbi.nlm.nih.gov/pubmed/1405637

Lynskey MT, Heath AC, Bucholz KK, et al. Escalation of drug use in early-onset cannabis users vs co-
twin controls JAMA. 2003;289:427-433. Available at: http://jama.ama-assn.org/content/289/4/427.long

Substance Abuse and Mental Health Services Administration. Results from the 2007 National Survey on
Drug Use and Health. 2008. Available at: http://store.samhsa.gov/product/SMA08-4343

Warner LA, White HR. Longitudinal effects of age at onset and first drinking situations on problem
drinking. Subst Use Misuse. 2003;38:1983-2016. Abstract available at:
http://www.ncbi.nlm.nih.gov/pubmed/14677779

Copyright 2012 American Academy of Pediatrics


2012 PREP SA ON CD-ROM

Question 252
You are seeing a 4-month-old infant who has a 4-hour history of inconsolable crying and frequent
bile-colored emesis. His mother reports that he has had no stools today. He was born at term via vaginal
delivery with no prenatal or neonatal complications. Results of his 2-month health supervision visit were
within normal parameters, and he received the appropriate immunizations at that time. Physical
examination today reveals a well-nourished child in distress whose temperature is 37.0C, heart rate is
160 beats/min, respiratory rate is 45 breaths/min, blood pressure is 78/50 mm Hg, and oxygen saturation
is 95% in room air. His lungs are clear to auscultation. His perfusion is decreased peripherally. Abdominal
examination documents a markedly distended abdomen, minimal bowel sounds, and diffuse pain. You
order abdominal radiography (Item Q252).
Of the following, the MOST likely cause of this childs symptoms is
A. appendicitis
B. gastroenteritis
C. Mallory-Weiss tear
D. necrotizing enterocolitis
E. volvulus

Copyright 2012 American Academy of Pediatrics


2012 PREP SA ON CD-ROM

Question 252

(Courtesy of B Poss)
Abdominal radiograph, as described for the child in the vignette.

Copyright 2012 American Academy of Pediatrics


2012 PREP SA ON CD-ROM

Critique 252 Preferred Response: E


The infant described in the vignette has a history and physical examination findings that are
consistent with an acute obstructive process that requires immediate surgical referral. His symptoms
began acutely 4 hours ago and are significant for bile-covered emesis, which is concerning for an
obstructive process. On physical examination, he has tachycardia, decreased perfusion, and marked
abdominal distension that appears to be painful. Abdominal radiography (Item C252) demonstrates
dilated loops of bowel, pneumatosis intestinalis (air in the bowel wall), and a paucity of intraluminal
colonic bowel gas, all of which are consistent with an acute small bowel obstruction that is confirmed by
subsequent ultrasonography. Volvulus associated with malrotation is the most common cause of small
bowel obstruction in patients younger than 1 year of age who have not had previous bowel surgery. This
is a life-threatening diagnosis, and any patient who has symptoms of acute volvulus must receive
radiographic imaging (ultrasonography, upper gastrointestinal radiographic series) to rule out the
diagnosis. Prompt surgical intervention is needed to prevent intestinal ischemia, and surgical fixation of
the bowel (Ladd procedure) is performed to prevent subsequent episodes. In addition, an incidental
appendectomy is performed because the appendix will lie in an abnormal position after repair, which
would make diagnosis of appendicitis difficult.
Acute abdominal pain in childhood can be a diagnostic dilemma because of the wide range of
possible causes as well as the inherent difficulties in examining a preverbal or developmentally delayed
child. Careful history taking and physical examination in conjunction with appropriate laboratory and
radiologic testing can help guide the pediatrician in determining the likely cause, initiating appropriate
management, and ensuring that a pathologic condition requiring surgical intervention, an acute surgical
abdomen, is not missed. History and physical examination findings consistent with a potential need for
surgical consultation and intervention include a history of abdominal trauma, pain that is made worse with
movement, involuntary guarding, rebound tenderness, and tenderness with percussion. Laboratory
studies such as white blood cell count, erythrocyte sedimentation rate, lipase and amylase measurement,
and urinalysis can help confirm diagnostic suspicions but are not diagnostic alone. Radiologic imaging
(plain film radiography, computed tomography scan, ultrasonography, or contrast enema) may be
indicated to assist with diagnosis or guide subsequent treatment.
Acute appendicitis in the young pediatric patient can be challenging to recognize, but the bile-
covered emesis and abdominal radiography findings for this infant are not consistent with this diagnosis.
Gastroenteritis is very unlikely due to the lack of stools, physical examination findings, and radiography
results. Necrotizing enterocolitis also is unlikely based on the infants age and baseline health. In addition,
although pneumatosis intestinalis is classically seen in necrotizing enterocolitis, abdominal distension
tends to be diffuse rather than localized. Mallory-Weiss tears (tears in the esophageal mucosa, often due
to forceful vomiting or portal hypertension) typically present with hematemesis or melena in addition to
abdominal pain.

American Board of Pediatrics Content Specification(s):


Recognize an acute "surgical abdomen"

Suggested Reading:
Bales W, Liacouras CA. Intestinal atresia, stenosis, and malrotation. In: Kliegman RM, Stanton BF, St.
Geme JW III, Schor NF, and Behrman RE, eds. Nelson Textbook of Pediatrics. 19th ed. Philadelphia, PA:
Saunders Elsevier; 2011:1277-1281

Ross A, LeLeiko NS. Acute abdominal pain. Pediatr Rev. 2010;31:135-144. DOI: 10.1542/pir.31-4-135.
Available at: http://pedsinreview.aappublications.org/cgi/content/full/31/4/135

Copyright 2012 American Academy of Pediatrics


2012 PREP SA ON CD-ROM

Critique 252

(Courtesy of B Poss)
Abdominal radiograph in volvulus showing dilated loops of bowel, pneumatosis intestinalis (arrow), and a
paucity of intraluminal colonic bowel gas.

Copyright 2011 American Academy of Pediatrics


2012 PREP SA ON CD-ROM

Question 253
The mother of an 8-year-old boy in whom you diagnosed type 1 diabetes 2 months ago calls your
office for advice. There has been a gastroenteritis outbreak in the boys school, and 2 days ago he
developed fever, vomiting, and diarrhea. He no longer is interested in eating, although he is able to drink.
She reports his blood glucose measurement as 205 mg/dL (11.4 mmol/L). His insulin regimen includes 15
units of glargine insulin administered daily at bedtime and aspart insulin administered before meals (1 unit
per 15 g carbohydrate) and for correction of high blood glucose (1 unit for every 25 mg/dL [1.39 mmol/L]
above 125 mg/dL [6.9 mmol/L]).
Of the following, the MOST appropriate next step is to
A. administer 0.1 units/kg of regular insulin subcutaneously
B. administer 0.1 units/kg per hour of regular insulin intravenously
C. administer 4 units of subcutaneous aspart insulin based on his blood glucose
D. check for urine ketones
E. discontinue glargine until he is able to eat normally

Copyright 2012 American Academy of Pediatrics


2012 PREP SA ON CD-ROM

Critique 253 Preferred Response: D


The primary defect in type 1 diabetes is the complete destruction of insulin-producing beta cells.
Because affected patients manufacture no endogenous insulin, they require exogenous insulin to prevent
fatty acid metabolism (and the resulting ketoacidosis), regardless of their blood glucose value.
Accordingly, patients who have type 1 diabetes must always take insulin, even if they are fasting.
Furthermore, whenever an affected child has a vomiting illness, such as the boy described in the vignette,
urine must be assessed for ketones to be sure that ketosis is not occurring. This boy has mild
hyperglycemia (blood glucose of 205 mg/dL [11.4 mmol/L]) and is using a glargine-based regimen.
Because glargine is a 24-hour-acting insulin with a minimal peak effect, his risk of hypoglycemia is
minimal. The boy should continue on his usual glargine dose and check his urine for ketones. Regardless
of blood glucose values, the presence of more than trace ketones in the urine always indicates a
physiologic need for more insulin.
Discontinuing the boys glargine insulin could result in severe ketoacidosis once his previous
glargine dose is metabolized. He does not require additional regular or rapid-acting (aspart) insulin at this
point because his blood glucose is only mildly elevated and he is not eating. If he does have evidence of
ketones, 15% to 25% of his total daily dose of insulin should be administered as an additional
subcutaneous injection of regular or rapid-acting insulin to stop further ketosis. Children who have ketosis
and are unable to consume carbohydrates may need intravenous fluids and dextrose to maintain normal
blood glucose concentrations while they receive additional insulin to resolve the ketosis.

American Board of Pediatrics Content Specification(s):


Counsel patients on the self-management of type 1 diabetes (hyperglycemia, sick days)

Suggested Reading:
Haller MJ, Atkinson MA, Schatz D. Type 1 diabetes mellitus: etiology, presentation, and management.
Pediatr Clin North Am. 2005;52:1553-1578. DOI: 10.1016/j.pcl.2005.07.006. Abstract available at:
http://www.ncbi.nlm.nih.gov/pubmed/16301083

Kaufman FR. Type 1 diabetes mellitus. Pediatr Rev. 2003;24:291-300. DOI:10.1542/pir.24-9-291.


Available at: http://pedsinreview.aappublications.org/cgi/content/full/24/9/291

Copyright 2012 American Academy of Pediatrics


2012 PREP SA ON CD-ROM

Question 254
The parents of a 24-month-old boy express concern about three recent episodes that occurred 2
hours after the boy fell asleep. He sat up straight and let out a blood-curdling scream. The boy recently
started a child care program twice a week, and his parents are concerned that he may be getting sick
from the other toddlers in his program.
Of the following, the BEST response is to
A. obtain a throat culture
B. reassure the parents that the events are benign
C. send the child for behavioral therapy
D. send the child for electroencephalography
E. send the child for polysomnography

Copyright 2012 American Academy of Pediatrics


2012 PREP SA ON CD-ROM

Critique 254 Preferred Response: B


The child described in the vignette is having night terrors, which are benign events. The parents
need reassurance that nothing is seriously wrong with the child. Electroencephalography,
polysomnography, and throat culture are not indicated because the child is not showing evidence of
seizure disorder, sleep apnea, or throat infection. Because this behavior is not within the childs control,
behavior therapy will not be helpful.
Night terrors is a disorder of arousal that happens during the rapid transition between stage 4
non-rapid eye movement (REM) sleep and REM sleep. It is difficult to comfort a child during the event
and the child wishes to return to sleep. In addition, the child does not recall the event the next morning.
Nightmares are upsetting dreams that occur during REM sleep. In contrast to night terrors, it is more
difficult for the child to fall back to sleep after a nightmare because he or she is very frightened. The child
recalls the event the following morning.
Parents of a child who is having night terrors need to understand the physiologic cause for the
event and not be alarmed when it occurs. If the behavior occurs frequently, parents may wish to wake the
child before the time the event typically occurs.

American Board of Pediatrics Content Specification(s):


Distinguish between nightmares and night terrors
Develop a management plan for a child who has night terrors

Suggested Reading:
Pagel JF. Nightmares and disorders of dreaming. Am Fam Physician. 2000;61:2037-2042, 2044. Abstract
available at: http://www.ncbi.nlm.nih.gov/pubmed/10779247

Zuckerman B. Nightmares and night terrors. In: Parker S, Zuckerman B, Augustyn M, eds. Developmental
and Behavioral Pediatrics: A Handbook for Primary Care. 2nd ed. Philadelphia, PA: Lippincott Williams &
Wilkins, a Wolters Kluwer business; 2005:251-252

Copyright 2012 American Academy of Pediatrics


2012 PREP SA ON CD-ROM

Question 255
A 15-year-old girl presents to your clinic on the weekend with a complaint of left lower jaw pain
that developed over the past 24 hours. She also describes increased sensitivity to hot and cold on that
side. On physical examination, you note tenderness localized to a lower left molar and a tender 1.5-cm
submandibular lymph node on that side. She has otherwise been well and has no known drug allergies.
You advise her to see her dentist the next day.
Of the following, the MOST appropriate antibiotic for treating this infection pending dental
evaluation is
A. azithromycin
B. cefdinir
C. doxycycline
D. penicillin VK
E. trimethoprim-sulfamethoxazole

Copyright 2012 American Academy of Pediatrics


2012 PREP SA ON CD-ROM

Critique 255 Preferred Response: D


The acute onset of dental pain with hot and cold sensitivity described for the girl in the vignette
raises concern for a dental abscess arising from bacterial invasion of the dental pulp complicating caries.
With such infection, regional lymphadenitis may occur. Trismus and dysphagia can be seen with more
severe infection.
Odontogenic infections are polymicrobial, comprising a mixture of oral anaerobic gram-negative
rods and gram-positive organisms, including streptococcal species. The involved oral anaerobes are
Bacteroides, Fusobacterium, Peptococcus, Peptostreptococcus, and Prevotella. Pending dentist
evaluation and a possible drainage procedure, penicillin is the drug of choice for such an infection.
Emergence of beta-lactamase-producing bacteria may lead to decreased penicillin efficacy, but this has
not been clearly demonstrated to date.
Azithromycin may be an acceptable alternative for the penicillin-allergic patient, but it is not the
drug of choice. Clindamycin is a better alternative for covering these organisms in the penicillin-allergic
patient. Doxycycline, trimethoprim-sulfamethoxazole, and cefdinir all have poor activity against these
anaerobic organisms.
Complications of odontogenic infections may occur with extension of infection to adjacent spaces
in the head and neck. The clinical presentation of such rare complications depends on the location of the
teeth involved. Infection of a mandibular incisor can extend to the submental space below the mandible;
abscess in a maxillary molar or bicuspid tooth can lead to buccal extension with periorbital swelling.

American Board of Pediatrics Content Specification(s):


Recognize the varied clinical manifestations of dental infections: swelling below the jaw with a
mandibular dental abscess, periorbital swelling with a maxillary dental abscess
Know the association between an anaerobic infection and dental and periodontal disease

Suggested Reading:
Brook I. Microbiology of and principles of antimicrobial therapy for head and neck infections. Infect Dis
Clin North Am. 2007;21:355-391. DOI: 10.1016/j.idc.2007.03.014. Abstract available at:
http://www.ncbi.nlm.nih.gov/pubmed/17561074

Gould JM, Cies JJ. Dental abscess. eMedicine Specialties, Pediatrics: General Medicine, Infectious
Disease. 2010. Available at: http://emedicine.medscape.com/article/909373-overview

Copyright 2012 American Academy of Pediatrics


2012 PREP SA ON CD-ROM

Question 256
A 10-year-old girl presents to the emergency department with a 1-week history of sore throat,
fever, chest pain, and progressive left-sided neck pain and swelling. On physical examination, her
temperature is 40.0C, heart rate is 130 beats/min, respiratory rate is 34 breaths/min, blood pressure is
90/60 mm Hg, and oxygen saturation is 70% in room air. You undertake emergent endotracheal
intubation and fluid resuscitation. The left side of her neck is notably swollen, and auscultation of the
chest reveals rhonchi throughout, with decreased breath sounds in the bases bilaterally. A white blood
3 9
cell count is 30.0x10 /mcL (30.0x10 /L), with 80% polymorphonuclear leukocytes, 15% lymphocytes, and
5% monocytes. Plain radiography of the chest reveals bilateral air space disease and effusions.
Computed tomography scan of the neck documents a retropharyngeal abscess extending toward the
mediastinum.
Of the following, the MOST appropriate antimicrobial to include in this girls therapy is
A. ampicillin
B. ampicillin-sulbactam
C. cefepime
D. doxycycline
E. gentamicin

Copyright 2012 American Academy of Pediatrics


2012 PREP SA ON CD-ROM

Critique 256 Preferred Response: B


The girl described in the vignette has a retropharyngeal abscess that requires urgent medical and
surgical intervention. Retropharyngeal infections often are polymicrobial. Therefore, empiric antibiotic
therapy should include broad-spectrum agents that have activity against aerobic bacterial pathogens such
as group A and group C streptococci, Staphylococcus aureus (including methicillin-resistant S aureus),
and occasionally, Haemophilus in addition to anaerobic bacterial pathogens such as Prevotella,
Peptostreptococcus, Bacteroides, Veillonella, and Fusobacterium.
Ampicillin-sulbactam is a beta-lactamase-resistant semisynthetic penicillin that has activity
against anaerobes as well as susceptible aerobic gram-positive organisms and respiratory tract gram-
negative pathogens and is an appropriate drug to initiate for the patient in the vignette. Ampicillin without
sulbactam is inadequate because it lacks beta-lactamase activity and activity against anaerobes. Until
culture and susceptibility data are available, vancomycin should be included in the patients antimicrobial
regimen to treat methicillin-resistant S aureus. Doxycycline is not appropriate therapy for life-threatening,
invasive staphylococcal infection. Neither cefepime nor gentamicin therapy is useful because aerobic
enteric gram-negative bacilli do not play a prominent role in retropharyngeal infections.
Retropharyngeal abscesses occur in infants and young prepubertal children following suppurative
adenitis of the retropharyngeal lymph nodes. The retropharyngeal space is a potential space between the
posterior pharyngeal wall and the prevertebral (cervical) fascia that extends from the base of the skull to
the posterior mediastinum. It contains two paramedial chains of lymph nodes that drain the middle ear,
eustachian tube, adenoids, nasopharynx, and posterior paranasal sinuses. Before puberty, these chains
of lymph nodes atrophy. Therefore, older children are not likely to develop retropharyngeal infections.
Retropharyngeal abscesses can rupture and spread to the lateral pharyngeal space or the posterior
mediastinum, as occurred in the patient described in the vignette. All mature retropharyngeal abscesses
should be drained and purulent material sent for culture and susceptibilities.

American Board of Pediatrics Content Specification(s):


Know how to treat retropharyngeal abscess: drug(s) of choice, alternative drugs, ineffective drugs,
surgical drainage
Know the microbiology of retropharyngeal abscess in children

Suggested Reading:
Goldstein NA, Hammerschlag MR. Peritonsillar, retropharyngeal, and parapharyngeal abscesses. In:
Feigin RD, Cherry JD, Demmler-Harrison GJ, Kaplan SL, eds. Feigin & Cherrys Textbook of Pediatric
Infectious Diseases. 6th ed. Philadelphia, PA: Saunders Elsevier; 2009:177-184

Schwartz RH. Infections related to the upper and middle airways. In: Long SS, Pickering LK, Prober CG,
eds. Principles and Practice of Pediatric Infectious Diseases. 3rd ed. Philadelphia, PA: Churchill
Livingstone Elsevier; 2008:213-220

Copyright 2012 American Academy of Pediatrics


2012 PREP SA ON CD-ROM

Question 257
You are evaluating a 7-year-old girl who had end-stage renal disease caused by renal
hypoplasia/dysplasia and underwent renal transplantation 18 months ago. She had been receiving
peritoneal dialysis from birth until the time of transplantation. At present, she is maintained on a
combination of prednisone, mycophenolate mofetil, and tacrolimus. She has had two previous episodes
of rejection that required medical treatment with pulse methylprednisolone. Because of the two rejection
episodes, her maintenance steroid dose is approximately 50% higher than usual. During her health
supervision visit today, the girls mother raises concern that her daughter is the smallest child in the
second grade. In reviewing her growth charts, you realize that while receiving dialysis, the girl was at the
5th percentile for linear growth, but now she is at less than the 3rd percentile.
Of the following, the MOST likely cause for this childs growth impairment is
A. corticosteroid exposure
B. end-stage renal disease of 7 years duration
C. malnutrition
D. metabolic acidosis due to tacrolimus
E. vitamin D deficiency

Copyright 2012 American Academy of Pediatrics


2012 PREP SA ON CD-ROM

Critique 257 Preferred Response: A


Growth impairment is common after solid organ transplant and in children experiencing renal
failure. Causes of growth impairment in renal failure include malnutrition, metabolic acidosis, renal
osteodystrophy, anemia, abnormal growth hormone axis, and salt wasting in select patients who have
renal dysplasia. These causes of growth failure seen in dialysis patients are removed in the setting of a
functional renal transplant because they are all linked to uremia and the complications associated with
minimal renal function.
The girl described in the vignette has a functioning transplant, but her rejection episodes have
required her to receive pulse methylprednisolone and higher-than-usual doses of maintenance
corticosteroids. Exposure to such doses can affect her growth. Renal transplant should have eliminated
the causes of growth failure seen in end-stage renal disease (ESRD) patients undergoing dialysis. A
history of ESRD in the posttransplant patient is an insufficient reason for her growth regression because
children should have improved growth following transplantation. Vitamin D deficiency can occur in ESRD
patients before and after transplant, which can be treated with supplemental ergocalciferol, and calcitriol
deficiency occurs only in dialysis patients. In either case, vitamin D deficiency posttransplant is usually
subclinical and would not result in regression of linear growth. Metabolic acidosis can occur following
transplantation and is usually attributed to calcineurin inhibitors (tacrolimus or cyclosporine), but because
laboratory tests are obtained frequently in this patient population, acidosis is usually easily identified and
treated with sodium bicarbonate, sodium citrate, or calcineurin inhibitor dose reduction. It is, therefore,
highly unlikely that metabolic acidosis due to tacrolimus (the most frequently used calcineurin inhibitor
posttransplantation) could lead to linear growth failure in this patient population.
Corticosteroids have been used as part of the transplant armamentarium since the advent of solid
organ transplantation. Adverse effects include weight gain, hypertension, striae, cataracts, hirsutism,
avascular necrosis, increased risk of infection, and linear growth failure. There has been an increased
movement in solid organ transplantation toward steroid avoidance or steroid withdrawal
immunosuppressive regimens. This strategy began with liver and heart transplantation initially and has
begun to be adopted by some renal transplantation programs. A dose-dependent effect of corticosteroids
on growth has been documented, with better growth rates seen in children receiving alternate-day
regimens compared with daily regimens.
Transplantation has been shown to improve achievement of age-appropriate developmental
milestones. In one study, children 30 months of age and younger who underwent renal transplantation
had improved scores on age-appropriate developmental tests.
It is clear that renal transplantation is the ideal form of renal replacement therapy for children who
have ESRD, although they can be treated with dialysis while attempts are made to optimize the
psychosocial situation of the family. Patients experiencing organ failure for other systems, such as liver,
heart, or lung, do not have the luxury of selecting the optimal time for transplantation in the event of a
poor psychosocial situation because failure to perform a lifesaving solid organ transplant could lead to
their death. The psychosocial stresses of having a child who has a chronic illness coupled with the need
to wait for a transplant creates substantial anxiety for families. They struggle with not having control over
the future of transplantation (if waiting for an organ) and the guilt of realizing that someone else has to die
for their child to receive a lifesaving organ transplant. Other stressors include financial burden of time lost
from work and fear of organ rejection, organ loss, malignancy, and death. Support groups have been
used at some centers to allow families awaiting transplantation to speak to others who have received
transplants to share coping skills. The importance of a supportive, reliable family for the future solid organ
transplant recipient cannot be overemphasized. Adherence with clinic follow-up and medication regimens
is often used to predict posttransplantation adherence. Efforts to improve the support network of the
family are necessary pretransplantation to optimize the chances of achieving a successful outcome.

American Board of Pediatrics Content Specification(s):


Monitor growth and development in patients following all types of transplantation
Recognize psychosocial and family issues associated with all types of transplantation

Suggested Reading:

Copyright 2012 American Academy of Pediatrics


2012 PREP SA ON CD-ROM

Barbour KA, Blumenthal JA, Palmer SM. Psychosocial issues in the assessment and management of
patients undergoing lung transplantation. Chest. 2006;129:1367-1374. DOI: 10.1378/chest.129.5.1367.
Available at: http://chestjournal.chestpubs.org/content/129/5/1367.long

Davis ID, Chang P-N, Nevins TE. Successful renal transplantation accelerates development in young
uremic children. Pediatrics. 1990;86:594-600. Abstract available at:
http://pediatrics.aappublications.org/cgi/content/abstract/86/4/594

Douglas JE, Hulson B, Trompeter RS. Psycho-social outcome of parents and young children after renal
transplantation. Child Care Health Dev. 1998;24:73-83. Abstract available at:
http://www.ncbi.nlm.nih.gov/pubmed/9468781

Fine RN, Martz K, Stablein D. What have 20 years of data from the North American Pediatric Renal
Transplant Cooperative Study taught us about growth following renal transplantation in infants, children,
and adolescents with end-stage renal disease? Pediatr Nephrol. 2010;25:739-746. DOI: 10.1007/s00467-
009-1387-3. Abstract available at: http://www.ncbi.nlm.nih.gov/pubmed/20013293

Gold LM, Kirkpatrick BS, Fricker FJ, Zitelli BJ. Psychosocial issues in pediatric organ transplantation: the
parents perspective. Pediatrics. 1986;77:738-744. Abstract available at:
http://pediatrics.aappublications.org/cgi/content/abstract/77/5/738

Jabs K, Sullivan EK, Avner ED, Harmon WE. Alternate-day steroid dosing improves growth without
adversely affecting graft survival or long-term graft function. A report of the North American Pediatric
Renal Transplant Cooperative Study. Transplantation. 1996;61:31-36. Abstract available at:
http://www.ncbi.nlm.nih.gov/pubmed/8560569

Reding R, Webber SA, Fine R. Getting rid of steroids in pediatric solid-organ transplantation? Pediatr
Transplant. 2004;8:526530. DOI: 10.1111/j.1399-3046.2004.00226.x. Abstract available at:
http://onlinelibrary.wiley.com/doi/10.1111/j.1399-3046.2004.00226.x/abstract

Copyright 2012 American Academy of Pediatrics


2012 PREP SA ON CD-ROM

Question 258
A 15-year-old boy was stung by a hornet and experienced anaphylaxis with urticaria, dyspnea,
and throat swelling. He states that he has been stung a number of times in the past but only experienced
local pain and erythema at the sting site. His parents are worried because he spends a lot of time hiking
and camping with friends.
Of the following, the MOST accurate statement regarding Hymenoptera allergy is that
A. additional Hymenoptera testing is not required if initial blood test results are negative
B. Hymenoptera allergy testing should be performed only to the insect identified by the patient
C. Hymenoptera immunotherapy is generally prescribed for 1 to 2 years before discontinuation
D. immunotherapy is not recommended for children younger than 16 years of age who experience
only cutaneous symptoms
E. this boys risk for anaphylaxis to a future Hymenoptera sting is approximately 5% to 10%

Copyright 2012 American Academy of Pediatrics


2012 PREP SA ON CD-ROM

Critique 258 Preferred Response: D


The teenager described in the vignette has had an anaphylactic reaction, and is at an increased
risk for anaphylaxis with future stings, despite his history of only experiencing normal reactions to flying
insect stings in the past. After initial management, he should be given an autoinjector containing
epinephrine and a referral to an allergist. Because certain sting reactions warrant skin testing and
possible venom immunotherapy (VIT), a careful history of all sting reactions is important (Item C258A).
Interestingly, VIT is not recommended in children younger than 16 years of age who experience reactions
limited to urticaria or angioedema; the risk for future reactions in these patients is 5% to 10%, which is
similar to individuals experiencing large local reactions.
Because the reaction described for this boy is consistent with anaphylaxis, his future risk of a
reaction is 20% to 60% without VIT and, therefore, further testing should be considered. Skin testing is
recommended for individuals older than 16 years who have any type of systemic reaction, even
cutaneous symptoms. Skin testing should also be considered for younger patients who have experienced
more than cutaneous symptoms. Those who have negative skin testing results should also undergo
venom-specific immunoglobulin (Ig)E blood testing because skin testing alone may miss the presence of
IgE in 10% to 20% of patients. If serum testing is performed first and results are negative, additional
venom skin testing should be performed for the same reasons.
Because insect identification can be difficult, serum or skin testing should be performed to all five
flying insects: wasp, yellow jacket, honeybee, yellow hornet, white faced hornet. Rarely, bumblebee may
need to be added if there is a strong history for a large, fuzzy bee and results of initial honeybee testing,
which is highly cross-reactive to bumblebee, are negative. Finally, a serum tryptase should be ordered in
patients experiencing anaphylaxis because 1% to 10% may have underlying mastocytosis or a clonal
mast cell disease that was provoked by the insect sting.
Allergen immunotherapy is beneficial in high-risk patients because it lowers the risk of future
reaction to 10% for honeybees and less than 5% for the other flying insects. Current guidelines
recommend continuing VIT for 3 to 5 years. However, studies have shown that even after 5 years of VIT,
the risk for reaction after stopping VIT increases to 15% per sting, although the risk for a severe reaction
remains low (<3%). For some patients, VIT may be recommended indefinitely (Item C258B).

American Board of Pediatrics Content Specification(s):


Know that immunotherapy with insect venom is 98% effective in preventing subsequent reactions

Suggested Reading:
Bil MB, Bonifazi F. The natural history and epidemiology of insect venom allergy: clinical implications.
Clin Exp Allergy. 2009;39:1467-1476. DOI: 10.1111/j.1365-2222.2009.03324.x. Abstract available at:
http://www.ncbi.nlm.nih.gov/pubmed/19622088

Hamilton RG. Diagnosis and treatment of allergy to Hymenoptera venoms. Curr Opin Allergy Clin
Immunol. 2010;10:323-329. DOI: 10.1097/ACI.0b013e32833bcf91. Abstract available at:
http://www.ncbi.nlm.nih.gov/pubmed/20543672

Copyright 2012 American Academy of Pediatrics


2012 PREP SA ON CD-ROM

Critique 258

Item C258A. Reactions to Flying Insects in Children and Testing Recommendations

Immunoglobulin E Self-injectable
Risk for Future Immunotherapy
Type of Reaction Testing Epinephrine
Anaphylaxis Recommended
Recommended Recommended
Normal 1% to 3% No No No
Large local 5% to 10% No No No*
Cutaneous only** 5% to 10% No Yes No*
(<16 years old)
Cutaneous only 10% to 20% Yes Yes No
(>16 years old)
Systemic 20% to 60% Yes Yes Yes
anaphylaxis
*Immunotherapy has been demonstrated to decrease the incidence of large local and mild cutaneous-
only reactions.

**Cutaneous-only symptoms may include flushing, urticaria, or angioedema of the face, lip, or tongue.

Copyright 2011 American Academy of Pediatrics


2012 PREP SA ON CD-ROM

Critique 258

Item C258B. Reasons to Consider Venom Immunotherapy Longer Than 5 Years


! Severe or near-fatal reaction caused by insect sting
! Systemic reaction to field sting despite immunotherapy
! Underlying medical conditions such as cardiovascular disease
! Limited access to emergency medical care (eg, rural location)

Copyright 2011 American Academy of Pediatrics


2012 PREP SA ON CD-ROM

Question 259
A 5-year-old boy is brought to the emergency department after being struck by a car while
running across the street. The paramedics report that he ran out from between two parked cars, and
witnesses estimate the car was traveling at 40 mph. He was thrown approximately 30 feet and was
unconscious immediately after the impact. The paramedics immobilized his cervical spine, administered
oxygen using a nonrebreather mask, placed a 20-gauge intravenous line in his right antecubital fossa,
and transported him to the emergency department. Physical examination upon arrival reveals moderate
respiratory distress with a respiratory rate of 58 breaths/min and diffuse retractions, a heart rate of 140
beats/min, blood pressure of 72/38 mm Hg, and oxygen saturation of 90% on 100% oxygen. Auscultation
of the lungs documents absent breath sounds diffusely over the right hemithorax. His abdomen is
moderately distended and tender in the right upper quadrant.
Of the following, the MOST appropriate next step is to
A. begin a dopamine infusion at 15 mcg/kg per minute
B. infuse 20 mL/kg 0.9% saline rapidly
C. obtain a chest radiograph
D. obtain an abdominal computed tomography scan
E. perform needle decompression of the right chest

Copyright 2012 American Academy of Pediatrics


2012 PREP SA ON CD-ROM

Critique 259 Preferred Response: E


The initial evaluation and management of the traumatized patient should be performed
methodically and incorporate the components of the primary survey (Item C259) and the secondary
survey. The primary survey includes simultaneous assessment of vital functions and management of life-
threatening injuries (assess and address). The secondary survey is a head-to-toe, front-to-back,
complete physical examination that is performed once the patient is stabilized.
The outcome of a severely injured child is maximized by rapidly identifying life-threatening injuries
and re-establishing and maintaining adequate oxygenation, ventilation, and systemic perfusion. The boy
described in the vignette demonstrates cardiorespiratory instability, with respiratory distress, tachypnea,
tachycardia, and hypotension. The absence of breath sounds over the right hemithorax is highly
suggestive of a tension pneumothorax. As part of the primary survey, needle decompression of the right
chest should be performed. If the patients vital signs do not stabilize after this intervention, further
maneuvers should include endotracheal intubation with mechanical ventilation and fluid resuscitation with
two boluses of 20 mL/kg 0.9% saline followed by 10 mL/kg blood, if needed. Hypotension and tachycardia
that are unresponsive to fluid and blood administration strongly suggest ongoing hemorrhage and should
prompt immediate surgical consultation. With the exception of neurologic shock in the setting of spinal
cord injury, vasopressor agents are not indicated in the treatment of hypotension in the trauma patient.
Once vital functions are restored, further evaluation may proceed with the secondary survey,
radiographs (initial radiographs include lateral cervical spine, chest, and pelvis in an unresponsive patient
who has severe injuries), and laboratory tests that include hemoglobin, hematocrit, and urinalysis.
Tetanus prophylaxis should be provided to any traumatized patient who may not be up-to-date on
immunizations once the primary survey has been completed. Further evaluation, such as computed
tomography scan of the head or abdomen, should be based on the information gathered from the primary
and secondary surveys and in consultation with a surgeon.

American Board of Pediatrics Content Specification(s):


Plan the initial evaluation of an accident victim

Suggested Reading:
American College of Surgeons. Advanced Trauma Life Support. 8th ed. Chicago, IL: American College of
Surgeons; 2008

Avarello JT, Cantor RM. Pediatric major trauma: an approach to evaluation and management. Emerg
Med Clin North Am. 2007;25:803836. DOI: 10.1016/j.emc.2007.06.013. Abstract available at:
http://www.ncbi.nlm.nih.gov/pubmed/17826219

Copyright 2012 American Academy of Pediatrics


2012 PREP SA ON CD-ROM

Critique 259

Item C259. Primary Survey


Assessment Management
A. Airway/Cervical Spine: ! Open and secure airway
Assess airway patency while ! Maintain cervical spine immobilization
immobilizing the cervical spine
B. Breathing: ! Provide 100% oxygen
Assess adequacy of oxygenation via ! Assist ventilation as needed
pulse oximetry and ventilation by ! Treat life-threatening chest injuries, including:
observing respiratory rate and tidal " Tension pneumothorax
volume (chest rise) " Open chest wound
" Flail chest
" Cardiac tamponade
C. Circulation: ! Re-establish perfusion with fluid resuscitation (20-
Assess adequacy of circulation and mL/kg 0.9% saline fluid boluses or 10-mL/kg doses
perfusion of packed red blood cells)
Measure heart rate, blood pressure, ! Treat significant hemorrhage
capillary refill time
D. Disability: ! Maximize oxygenation and perfusion, normalize
Assess neurologic status by examining ventilation (do not hyperventilate)
pupil equality/reactivity and level of ! Consider adjunctive therapies (oncotic agents,
consciousness (alert, responsive to diuretics)
voice, responsive to pain, unresponsive)
E. Exposure: ! Remove all clothes
Examine for other life-threatening injuries

Copyright 2011 American Academy of Pediatrics


2012 PREP SA ON CD-ROM

Question 260
A 10-month-old girl presents to your office because of diarrhea, irritability, and a fall-off in her
weight gain. She was born at term to a 28-year-old gravida 1, para 1, woman following an uncomplicated
pregnancy and delivery and had a birthweight of 2,900 g. She was exclusively breastfed for the first 2
months after birth. At 2 months of age, human milk was supplemented with cow milk protein-based
formula. Because of increasing problems with postprandial emesis and colicky symptoms, human milk
feeding was discontinued and several formula changes were attempted. At 6 months of age, she was
started on a soy protein formula thickened with rice cereal because of symptoms suggesting
gastroesophageal reflux. Since that time, her mother has noted an increased frequency and reduced
consistency of the babys stools. The infant now passes six to eight loose-to-watery bowel movements
per day. Physical examination of the alert, well-developed, somewhat thin-appearing infant shows a
length of 72 cm and a weight of 8.0 kg. The only additional finding of note is an erythematous and eroded
diaper dermatitis.
Of the following, the MOST appropriate diagnostic test for this infant is a
A. breath hydrogen test
B. d-xylose absorption study
C. lactose tolerance test
D. tissue transglutaminase antibody
E. zinc measurement

Copyright 2012 American Academy of Pediatrics


2012 PREP SA ON CD-ROM

Critique 260 Preferred Response: A


The infant described in the vignette has experienced a fall-off in weight gain associated with the
passage of multiple loose, watery stools per day. Coincidentally, diarrhea began soon after the babys
diet was changed to a soy formula thickened with rice cereal. The erythematous, eroded diaper dermatitis
is consistent with the passage of watery, acidic stools, reflecting an osmotic diarrhea accompanied by
bacterial fermentation of unabsorbed carbohydrate. Because proprietary soy formulas contain sucrose or
glucose polymers as their constituent carbohydrates and rice cereal is composed largely of complex
glucose polymers as starch molecules, the infants dietary history and clinical findings strongly suggest a
diagnosis of sucrase-isomaltase (SI) deficiency, the most common congenital disaccharidase deficiency.
The most appropriate and noninvasive study to confirm this diagnosis is a breath hydrogen test following
oral administration of sucrose as the carbohydrate challenge. A definitive diagnosis of SI deficiency
requires upper gastrointestinal tract endoscopy and duodenal biopsy for direct measurement of SI
activity.
Intestinal malabsorption is usually suspected on the basis of clinical findings that include bulky,
malodorous stools; chronic diarrhea; and growth and weight gain disturbances. In many patients, even
those who do not have a prominent history of diarrhea, other signs of malnutrition suggest a problem in
nutrient absorption or utilization. Disturbed digestive-absorptive function may result in anemia, bruising,
edema, osteopenia, and reduced muscle mass. The differential diagnosis is a long one and includes both
common and rare disorders, several of which may present during early infancy. The major causes of
malabsorption during infancy and childhood are shown in (Item C260A), and these represent digestive
abnormalities, alterations in the intraluminal milieu, infectious processes, mucosal changes leading to a
reduced villus absorptive surface area, shortened bowel length, and nutrient transport defects.
Specific screening tests (Item C260B) are used to document the malabsorptive state. Reaching a
definitive diagnosis often requires referral to a pediatric gastroenterologist for the performance of more
invasive testing, including upper endoscopy and small bowel biopsy. Initial screening should include an
examination of the stool. An acidic fecal pH indicates bacterial fermentation, and a stool reducing
substances test can identify any unfermented reducing sugars. The fecal fat stain shows the presence of
excreted lipid and should be a precursor to a formal 72-hour quantitative fecal fat determination. Fat
malabsorption often is manifested by the passage of bulky, malodorous stools, but it may also be
accompanied by a complaint of constipation. In any case, when fat malabsorption is suspected, either
because of stool characteristics or signs of malnutrition, pancreatic insufficiency must be considered. For
the general pediatrician, this problem should be evaluated initially with a sweat chloride determination to
rule out cystic fibrosis, by far the most common cause of pancreatic insufficiency in childhood. Other
useful screening tests in the evaluation of pancreatic excretory function include fecal elastase and trypsin
assessment. Formal pancreatic function testing involves endoscopy and duodenal intubation and should
be carried out only in a specialized referral center.
An important screening study for any patient presenting with a suspicion of malabsorption, signs
of malnutrition, or undiagnosed gastrointestinal symptoms is the tissue transglutaminase antibody (TTG)
assessment, a highly sensitive and specific screening test for celiac disease. This test is useful as a
diagnostic tool only if the patient is consuming a source of gluten. Because TTG is an immunoglobulin A
(IgA) antibody, the test should always be accompanied by assessment of total IgA to identify patients who
may be IgA-deficient. Other studies may be used to assess overall small bowel absorptive function as a
measure of villus surface area and integrity in patients who have suspected diffuse microvillus
abnormalities. These tests, however, do not provide specific diagnostic information about the primary
disease. The d-xylose absorption study takes advantage of this five-carbon sugars property to be
excreted in the urine following proximal jejunal absorption without undergoing intraluminal hydrolysis or
endogenous metabolism.
Vitamin and trace minerals may be measured as an indicator of fat absorption (vitamins A, D, E,
K) as well as ileal (vitamin B12) and proximal small bowel absorptive function (iron, zinc, folate). Although
serum zinc concentrations are not accurate predictors of total body zinc nutriture, low values have been
reported in malnourished states associated with underlying bowel disorders, including Crohn disease and
celiac disease. The lactose tolerance test has largely been supplanted by the lactose breath hydrogen
test to identify patients who have lactose intolerance, a common problem in those who have bowel
disease, malnutrition, and chronic diarrhea. Inflammatory bowel (IBD) serologies may also be obtained,

Copyright 2012 American Academy of Pediatrics


2012 PREP SA ON CD-ROM

particularly when there is a family history of IBD, or in the presence of other signs and symptoms of
Crohn disease. However, negative studies do not rule out IBD, especially in pediatric patients.

American Board of Pediatrics Content Specification(s):


Know the appropriate laboratory tests for malabsorption

Suggested Reading:
Goulet O, Ruemmele F. Causes and management of intestinal failure in children. Gastroenterology.
2006;130 (2 suppl 1):S16-S28. Abstract available at: http://www.ncbi.nlm.nih.gov/pubmed/16473066

Green PH, Jabri B. Celiac disease. Annu Rev Med. 2006;57:207-221. Abstract available at:
http://www.ncbi.nlm.nih.gov/pubmed/16409146

Jacob R, Zimmer K-P, Schmitz J, Naim HY. Congenital sucrase-isomaltase deficiency arising from
cleavage and secretion of a mutant form of the enzyme. J Clin Invest. 2000;106:281287. DOI:
10.1172/JCI9677. Available at: http://www.ncbi.nlm.nih.gov/pmc/articles/PMC314311/?tool=pubmed

Montalto M, Santoro L, D'Onofrio F, et al. Classification of malabsorption syndromes. Dig Dis.


2008;26:104-111. DOI: 10.1159/000116767. Abstract available at:
http://www.ncbi.nlm.nih.gov/pubmed/18431059

Pietzak MM, Thomas DW. Childhood malabsorption. Pediatr Rev. 2003;24:195-206. DOI: 10.1542/pir.24-
6-195. Available at: http://pedsinreview.aappublications.org/cgi/content/full/24/6/195

Robayo-Torres CC, Quezada-Calvillo R, Nichols BL. Disaccharide digestion: clinical and molecular
aspects. Clin Gastroenterol Hepatol. 2006;4:276-287. Abstract available at:
http://www.ncbi.nlm.nih.gov/pubmed/16527688

Copyright 2012 American Academy of Pediatrics


2012 PREP SA ON CD-ROM

Critique 260

Item C260A. Major Causes of Malabsorption in Infancy and Childhood


Mechanism Disorder
Maldigestion
! Insufficient intraluminal bile salts Biliary obstruction
Chronic liver failure
Chronic pancreatitis
! Inadequate pancreatic enzymes
Cystic fibrosis (CF)
Pancreatic insufficiency (non-CF)
Enterokinase deficiency
! Deficiency of intestinal microvillus Lactase deficiency
membrane enzymes Sucrase-isomaltase deficiency
Abnormal intraluminal environment Bacterial overgrowth
Blind loop syndrome
Chronic infection Giardiasis
Mucosal abnormality Autoimmune enteropathy
Celiac disease
Crohn disease
Congenital microvillus atrophy
Intestinal epithelial dysplasia (tufting enteropathy)
Microvillus membrane inclusion disease
Short bowel Intestinal resection (Crohn disease, volvulus,
intussusception, necrotizing enterocolitis)
Impaired transport Abetalipoproteinemia
Intestinal lymphangiectasia

Copyright 2011 American Academy of Pediatrics


2012 PREP SA ON CD-ROM

Critique 260

Item C260B. Screening Tests in the Evaluation of Suspected Malabsorption

Test Abnormal Result Suggests:


Sweat chloride Cystic fibrosis

Tissue transglutaminase antibody, IgA Celiac disease

Inflammatory bowel disease serologies* Crohn disease

Micronutrient concentrations
! Iron, B12, folate, zinc Celiac disease, Crohn disease, short bowel,
bacterial overgrowth

! Fat-soluble vitamins (A, E, 25-OH D), Pancreatic insufficiency, liver disease, celiac
prothrombin time disease, bacterial overgrowth, intestinal
lymphangiectasia

Fecal enzyme/protein concentrations


! alpha-1-antitrypsin Protein-losing enteropathy
! Elastase, trypsin Pancreatic insufficiency

Fecal fat stain " 72-hour fecal fat Intraluminal maldigestion (pancreatic
insufficiency, hepatobiliary disorder) versus
mucosal abnormality
Stool pH, reducing substances Disaccharidase deficiency, mucosal injury,
bacterial overgrowth, short bowel, celiac
disease
Hydrogen breath testing
Disaccharidase deficiency (lactase, sucrase-
isomaltase), celiac disease, bacterial
overgrowth

*Anti-Saccharomyces cerevisiae antibody-positive is consistent with Crohn disease

Copyright 2011 American Academy of Pediatrics


2012 PREP SA ON CD-ROM

Question 261
You are examining a 4-kg, 37-week gestational age infant 46 hours after birth. She was born to a
30-year-old woman who has type 1 diabetes. The infants sibling had jaundice but did not require
phototherapy. The infant has been bottle feeding well, with six wet diapers and two stools over the past
day. Her current weight is 3.8 kg. On physical examination, she has a large right cephalohematoma and
jaundice that extends from her face caudally to her trunk. Her total serum bilirubin measures 11.0 mcg/dL
(188.1 mcmol/L) and the conjugated serum bilirubin is 0.2 mg/dL (34.2 mcmol/L).
Of the following, the MOST significant risk factor for the development of severe
hyperbilirubinemia in this patient is
A. bottle feeding
B. cephalohematoma
C. current serum bilirubin concentration
D. macrosomia due to maternal diabetes
E. sibling who had jaundice

Copyright 2012 American Academy of Pediatrics


2012 PREP SA ON CD-ROM

Critique 261 Preferred Response: B


The infant described in the vignette has a large right cephalohematoma, which is a significant risk
factor for the development of severe hyperbilirubinemia. Nearly all term infants have hyperbilirubinemia in
the first week after birth, defined by a serum bilirubin value of 2.0 mg/dL (34.2 mcmol/L) or higher, and up
to 60% are visibly jaundiced. Physiologic jaundice in the newborn is attributed to the breakdown of red
blood cells, decreased uptake and excretion of bilirubin from liver cells, and increased enterohepatic
circulation of bilirubin. Nonphysiologic jaundice is often described as an exaggeration of one or more of
the mechanisms of physiologic jaundice, with a resultant increase in serum bilirubin concentration. Due to
the risk of bilirubin toxicity to the basal ganglia and brainstem nuclei, infants must be carefully assessed
before discharge to identify those at risk for severe hyperbilirubinemia. The American Academy of
Pediatrics published clinical practice guidelines in 2004 for the management of hyperbilirubinemia in
newborns of 35 weeks gestation or more, including a nomogram of the risk of developing severe
hyperbilirubinemia according to serum bilirubin value and postnatal age in hours (Item C261).
Interpretation of serum bilirubin values also requires consideration of the risk factors present for
the development of severe hyperbilirubinemia before discharge. Major risk factors for the development of
severe hyperbilirubinemia in infants of 35 weeks or more gestation include a predischarge bilirubin value
in the high-risk zone in the bilirubin nomogram, jaundice seen in the first 24 hours after birth, hemolytic
disease, gestational age of 35 to 36 weeks, previous sibling(s) requiring phototherapy, cephalohematoma
or significant bruising, exclusive breastfeeding associated with poor feeding and excessive weight loss,
and East Asian race (eg, Chinese, Japanese, Korean). Among the minor risk factors are a predischarge
bilirubin value in the high-intermediate risk zone in the bilirubin nomogram, gestational age of 37 to 38
weeks, jaundice observed before discharge, previous sibling(s) who had jaundice, macrosomic infant of a
diabetic mother, maternal age of 25 years or more, and male sex. Exclusive bottle feeding and African
American race have been associated with a decreased risk of severe hyperbilirubinemia.
Management of infants who have hyperbilirubinemia includes careful examination, close
monitoring, and initiation of therapy if indicated. Guidelines for both the initiation of phototherapy and
consideration of exchange transfusion have been developed based on serum bilirubin value, postnatal
age in hours, and risk category. Infants being discharged to home require thorough parental teaching and
appropriate follow-up. If concerns exist regarding follow-up, discharge should be delayed until the risk for
developing severe hyperbilirubinemia has passed.

American Board of Pediatrics Content Specification(s):


Plan the management of a patient with hyperbilirubinemia [non-physiologic jaundice]

Suggested Reading:
Ambalavanan N, Carlo WA. Digestive system disorders: jaundice and hyperbilirubinemia in the newborn.
In: Kliegman RM, Stanton BF, St. Geme JW III, Schor NF, and Behrman RE, eds. Nelson Textbook of
Pediatrics. 19th ed. Philadelphia, PA: Saunders Elsevier; 2011:603-608

Hansen TWR. Core concepts: bilirubin metabolism. NeoReviews. 2010;11:e316-e322. DOI:


10.1542/neo.11-6-e316. Available at: http://neoreviews.aappublications.org/cgi/content/full/11/6/e316

Keren R, Bhutani VK. Predischarge risk assessment for severe neonatal hyperbilirubinemia.
NeoReviews. 2007;8:e68-e76. DOI: 10.1542/neo.8-2-e68. Available at:
http://neoreviews.aappublications.org/cgi/content/full/8/2/e68

Subcommittee on Hyperbilirubinemia. Clinical practice guideline: management of hyperbilirubinemia in


the newborn infant 35 or more weeks of gestation. Pediatrics. 2004;114 297-316. Available at:
http://pediatrics.aappublications.org/cgi/content/full/114/1/297

Copyright 2012 American Academy of Pediatrics


2012 PREP SA ON CD-ROM

Critique 261

Reprinted with permission from Bhutani VK, Johnson L, Sivieri EM. Predictive ability of a predischarge hour-specific serum bilirubin for
subsequent significant hyperbilirubinemia in healthy term and near-term newborns. Pediatrics 1999;103:6-14

Item C261: Nomogram for Designation of Risk of Hyperbilirubinemia. Risk designation of term and near-term
well newborns based on their hour-specific serum bilirubin values. The high-risk zone is designated by the 95th
percentile track. The intermediate-risk zone is subdivided to upper- and lower-risk zones by the 75th percentile
track. The low-risk zone has been electively and statistically defined by the 40th percentile track.

Copyright 2011 American Academy of Pediatrics


2012 PREP SA ON CD-ROM

Question 262
You are seeing a 16-year-old boy in your office for a sports preparticipation examination. He has
no complaints, but his parent asks about a rash that has been present for several months. On physical
examination, you note a rash on his neck and upper back composed of hypopigmented macules (Item
Q262).
Of the following, the MOST likely cause of the rash is
A. Borrelia burgdorferi
B. Malassezia
C. nickel dermatitis
D. parvovirus B19
E. Streptococcus pyogenes

Copyright 2012 American Academy of Pediatrics


2012 PREP SA ON CD-ROM

Question 262

(Reprinted with permission from Krowchuk DP, Mancini AJ, eds. Pediatric Dermatology. A Quick
Reference Guide. Elk Grove Village, Il: American Academy of Pediatrics; 2nd ed. 2011)
Rash, as described for the boy in the vignette.

Copyright 2012 American Academy of Pediatrics


2012 PREP SA ON CD-ROM

Critique 262 Preferred Response: B


The rash described for the boy in the vignette is tinea (pityriasis) versicolor, a common, superficial
fungal skin disorder caused by any one of a number of species of Malassezia. It is characterized by
hypopigmented (Item C262A) to pink-to-brown (Item C262B) round-to-confluent macules with fine scale.
It typically involves the neck, upper back, chest, and shoulders; occasionally the face is affected. It is
more common in hot, humid climates but does occur at a lower frequency in temperate climates. In the
United States, it most commonly affects adolescents and young adults and is rare before puberty.
Although tinea versicolor occasionally causes some pruritus, most patients are asymptomatic. Diagnosis
is made by the characteristic appearance, but it can be confirmed by potassium hydroxide examination of
scrapings from the rash showing the classic spaghetti and meatballs (hyphae and blastospore)
appearance (Item C262C). The condition is easily treated with topical therapy, but the recurrence rate is
high. The usual treatment of choice is topical selenium sulfide, but topical ketoconazole is also effective.
For patients who do not tolerate or do not respond to topical treatment, oral azoles (ketoconazole,
itraconazole, and fluconazole) may be used. These same topical and oral medications are used to
prevent the frequent recurrences.
The most typical skin eruption associated with Borrelia burgdorferi, the spirochete that causes
Lyme disease, is erythema migrans. This rash is characterized by a large, red, circular patch with central
clearing (Item C262D) and is often accompanied by systemic symptoms. Parvovirus B19 is the causative
organism of erythema infectiosum (fifth disease), with its slapped-cheek appearance and erythematous
reticular rash involving the arms. Nickel dermatitis is characterized by lichenification and scaling (Item
C262E). Streptococcus pyogenes has a wide range of skin manifestations from impetigo to scarlet fever;
these are easily differentiated from tinea versicolor by clinical appearance.

American Board of Pediatrics Content Specification(s):


Recognize the clinical appearance of tinea versicolor

Suggested Reading:

Browning JC. An update on pityriasis rosea and other similar childhood exanthems. Curr Opin Pediatr.
2009;21:481-485. DOI: 10.1097/MOP.0b013e32832db96e. Abstract available at:
http://www.ncbi.nlm.nih.gov/pubmed/19502983

Crespo-Erchiga V, Florencio VD. Malassezia yeasts and pityriasis versicolor. Curr Opin Infect Dis.
2006;19:139-147. DOI: 10.1097/01.qco.0000216624.21069.61. Abstract available at:
http://www.ncbi.nlm.nih.gov/pubmed/16514338

Janik MP, Heffernan MP. Yeast infections: candidiasis and tinea (pityriasis) versicolor. In; Wolf K,
Goldsmith LA, Katz SI, Gilchrest BA, Paller AS, Leffell DJ, eds. Fitzpatricks Dermatology in General
Medicine. 7th ed. New York, NY: The McGraw-Hill Companies, Inc; 2008:chapter 189

Krowchuk DP, Mancini AJ. Tinea versicolor. In: Pediatric Dermatology A Quick Reference Guide. 2nd ed.
Elk Grove Village, IL: American Academy of Pediatrics; 2011

Copyright 2012 American Academy of Pediatrics


2012 PREP SA ON CD-ROM

Critique 262

(Reprinted with permission from Krowchuk DP, Mancini AJ, eds. Pediatric Dermatology. A Quick
Reference Guide. 2nd ed. Elk Grove Village, IL: American Academy of Pediatrics; 2011)
Tinea versicolor may produce hypopigmented macules.

Copyright 2011 American Academy of Pediatrics


2012 PREP SA ON CD-ROM

Critique 262

(Reprinted with permission from Krowchuk DP, Mancini AJ, eds. Pediatric Dermatology. A Quick
Reference Guide. 2nd ed. Elk Grove Village, IL: American Academy of Pediatrics; 2011)
Hyperpigmented scaling macules on the chest in tinea versicolor.

Copyright 2011 American Academy of Pediatrics


2012 PREP SA ON CD-ROM

Critique 262

(Reprinted with permission from the PediaLink Essentials course, Pediatric Dermatology Skin Infections)
In tinea versicolor, a potassium hydroxide preparation performed on scale reveals short hyphae (red
arrows) and spores (yellow arrows), the so-called spaghetti and meatballs appearance.

Copyright 2011 American Academy of Pediatrics


2012 PREP SA ON CD-ROM

Critique 262

(Reprinted with permission from Krowchuk DP, Mancini AJ, eds. Pediatric Dermatology. A Quick
Reference Guide. 2nd ed. Elk Grove Village, IL: American Academy of Pediatrics; 2011)
Erythma migrans: an expanding ring of erythema.

Copyright 2011 American Academy of Pediatrics


2012 PREP SA ON CD-ROM

Critique 262

(Courtesy of D Krowchuk)
A lichenified patch from contact dermatitis due to nickel in a necklace.

Copyright 2011 American Academy of Pediatrics


2012 PREP SA ON CD-ROM

Question 263
A 14-year-old boy presents for a preparticipation sports evaluation for baseball. He plays
shortstop. His mother is very concerned about his playing because of the injuries she has heard about in
professional and collegiate athletes. You explain to her that appropriate equipment, including a batting
helmet, is needed to provide protection for her son.
Of the following, the LARGEST percentage of baseball injuries can be prevented by also using
A. a mouth guard
B. a protective cup
C. elbow pads
D. knee pads
E. polycarbonate goggles

Copyright 2012 American Academy of Pediatrics


2012 PREP SA ON CD-ROM

Critique 263 Preferred Response: A


More than 50% of all high school students participate in athletics, and injury prevention should be
a mainstay of sports participation for youth. Both parents and coaches should provide and insist on the
use of equipment and safety rules to prevent injury in young athletes. Mouth injuries, along with other
head injuries, account for the majority of injuries (48%) sustained by youth in baseball. Injuries generally
are caused by contact with sports equipment (eg, the bat, the ball, and the base). Most serious injuries
result from being struck by a batted ball and are more common among infield players. Other injuries
include those to the leg and groin as well as the chest. Although these are uncommon, use of a protective
cup in all sports is recommended to prevent testicular injury.
Dental and facial injuries may be prevented best by using a mouth guard both in the field and at
the plate in baseball to avoid injury by pitched and batted balls. Plastic and metal helmets with face
protection have been available for batters for several years but are used uncommonly in high school
athletics. Clearly, use of a helmet with face protection is important to prevent cranial injuries for the
catcher. Even with the use of a helmet, mouth guards protect further against injuries of teeth and from
teeth to the oral mucosa. The American Association of Orthodontists recommends that mouth guards be
used for the following sports: baseball, football, soccer, basketball, wrestling, softball, ice and field hockey,
volleyball, and lacrosse.
Elbow and knee pads may be helpful in prevention of abrasions and other minor injuries, but they
are unlikely to prevent serious injury. Polycarbonate goggles are recommended for batting, but evidence
for their routine use in fielding is lacking. Eye protection is afforded by most helmets with face protection.

American Board of Pediatrics Content Specification(s):


Know the indications for the use of mouth guards in athletics

Suggested Reading:
Collins CL, Comstock RD. Epidemiological features of high school baseball injuries in the United States,
20052007. Pediatrics. 2008;121:1181-1187. DOI: 10.1542/peds.2007-2572. Available at:
http://pediatrics.aappublications.org/cgi/content/full/121/6/1181

Committee on Sports Medicine and Fitness. Risk of injury from baseball and softball in children.
Pediatrics. 2001;107:782-784. Available at: http://pediatrics.aappublications.org/cgi/content/full/107/4/782

Lawson BR, Comstock RD, Smith GA. Baseball-related injuries to children treated in hospital emergency
departments in the United States, 19942006. Pediatrics. 2009;123:e1028-e1034. DOI:
10.1542/peds.2007-3796. Available at: http://pediatrics.aappublications.org/cgi/content/full/123/6/e1028

Section on Pediatric Dentistry and Oral Health. Preventive oral health intervention for pediatricians.
Pediatrics. 2008;122:1387-1394. DOI: 10.1542/peds.2008-2577. Available at:
http://pediatrics.aappublications.org/cgi/content/full/122/6/1387

Turpin DL., Keels M. Prevention efforts can take bite out of dental trauma. AAP News. 2010 March;31:13.
Available at: http://aapnews.aappublications.org/content/31/3/13.full.pdf+html?sid=677d68c8-59c6-4ae8-
9517-82333d51cd1b

Copyright 2012 American Academy of Pediatrics


2012 PREP SA ON CD-ROM

Question 264
You are called to examine an otherwise well-appearing term 2-hour-old infant whom the nurse
has noted to be dusky. On physical examination, he has cyanosis of the distal extremities of the hands
and feet (Item Q264), his mucous membranes and his trunk are pink, his lungs are clear, and he has no
organomegaly. His cardiac examination reveals no murmurs, rubs, or clicks, and his pulses are
symmetric.
Of the following, the MOST likely pulse oximetry reading obtained on the foot of this infant is
A. 54% to 63%
B. 64% to 73%
C. 74% to 83%
D. 84% to 93%
E. 94% to 100%

Copyright 2012 American Academy of Pediatrics


2012 PREP SA ON CD-ROM

Question 264

(Courtesy of the Media Lab at Doernbecher)


Cyanosis, as described for the infant in the vignette.

Copyright 2012 American Academy of Pediatrics


2012 PREP SA ON CD-ROM

Critique 264 Preferred Response: E


The well-appearing neonate described in the vignette has findings consistent with the diagnosis
of acrocyanosis, a benign condition that must be distinguished from central cyanosis. Acrocyanosis is the
transient blue discoloration of the hands and feet (Item C264) in response to vasomotor instability or a
cool environment. The perioral region (sparing the lips and mucous membranes) can also be involved.
Extremities may be cool to the touch. Acrocyanosis is believed to be caused by vasoconstriction of the
small arterioles. It typically resolves in the first few weeks or months after birth. Because the remainder of
this newborns physical examination results are normal and he has no additional clinical symptoms, no
further laboratory or radiographic testing is indicated. Oxygen saturation by pulse oximetry should be in
the normal neonatal range (>94%).
Central cyanosis is the result of deoxygenated blood or a diminished serum hemoglobin
concentration within the capillary bed. The affected newborn has reduced oxygen saturation by pulse
oximetry. Although most infants who have oxygen saturations below 80% are noted to be cyanotic by
inspection alone, cyanosis can be difficult to discern in infants who have dark skin pigmentation. Affected
infants have bluish discoloration of the lips, tongue, mucosa, and nail beds. Neonatal cyanosis is due to
either a right-to-left intracardiac shunt as a result of congenital heart disease or to respiratory compromise
resulting in a ventilation-perfusion mismatch. Other causes include hypoventilation due to neurologic
disorders or pharmacologic agents or methemoglobinemia.
Due to the uncertainty associated with visual inspection for cyanosis, in some communities an
oximetry reading is obtained as a screen in every newborn before discharge from the well baby nursery.
The infant whose oxygen saturation is less than 94% warrants further investigation. This may entail only a
thorough physical examination by an experienced primary care clinician or may require consultation with
a pediatric intensivist, cardiologist, or pulmonologist and possible additional diagnostic testing.

American Board of Pediatrics Content Specification(s):


Distinguish between central cyanosis and acrocyanosis

Suggested Reading:
Allen HD, Phillips JR, Chan DP. History and physical examination. In: Allen HD, Driscoll DJ, Shaddy RE,
Feltes TF, eds. Moss and Adams' Heart Disease in Infants, Children, and Adolescents Including the Fetus
and Young Adult. 7th ed. Philadelphia, PA: Lippincott Williams & Wilkins, a Wolters Kluwer business;
2008:58-66

Bernstein D. The cardiovascular system: Evaluation of the cardiovascular system: History and physical
examination. In: Kliegman RM, Stanton BF, St. Geme JW III, Schor NF, and Behrman RE, eds. Nelson
Textbook of Pediatrics. 19th ed. Philadelphia, PA; Saunders Elsevier; 2011:1529-1536

Copyright 2012 American Academy of Pediatrics


2012 PREP SA ON CD-ROM

Critique 264

(Courtesy of the Media Lab at Doernbecher)


Cyanosis, as described for the infant in the vignette.

Copyright 2011 American Academy of Pediatrics


2012 PREP SA ON CD-ROM

Question 265
A 3,100-g term infant, who was born via an uncomplicated, spontaneous vaginal delivery, has two
episodes of nonsuppressible, pedaling movements of limbs with eye deviation and apnea during the first
postnatal day. On physical examination, the infant has normal vital signs, appears well, and has a normal
head circumference. After administration of a loading dose of 20 mg/kg of phenobarbital, no further
episodes occur. Head computed tomography scan shows blood along the posterior rim of both occipital
lobes but is otherwise normal. A lumbar puncture shows 340 red blood cells.
Of the following, the MOST likely cause of the infants seizure is
A. neonatal stroke
B. prenatal trauma
C. sinus venous thrombosis
D. subarachnoid hemorrhage
E. subdural hemorrhage

Copyright 2012 American Academy of Pediatrics


2012 PREP SA ON CD-ROM

Critique 265 Preferred Response: D


The neonate described in the vignette has developed paroxysmal motor events in the first 24
hours after birth. Vigilance for seizures is important because these can signal the presence of serious
problems affecting the brain that require urgent diagnosis and treatment. If the events are seizures,
treatment should occur concurrently with systematic evaluation for the cause. A loading dose of
intravenous phenobarbital at 20 mg/kg is generally effective. The most common cause of seizures in the
first 24 hours after birth is hypoxic-ischemic brain injury. Other causes include hemorrhage, metabolic
derangements, and infection.
The presence of hemorrhage along the dependent brain surface described for this infant
generally indicates a subarachnoid hemorrhage. In a term baby who is otherwise healthy and is
experiencing no apparent asphyxia, this is the most likely cause. Intraventricular hemorrhage is a
possibility in preterm babies.
Subdural hemorrhage, stroke, trauma, and venous sinus thrombosis are less common causes of
seizures in neonates. In addition, they are not supported by the clinical and radiologic details described
for this infant.

American Board of Pediatrics Content Specification(s):


Know the differential diagnosis of neonatal seizures

Suggested Reading:
Hill A. Neurological problems of the newborn. In: Bradley WG, Daroff RB, Fenichel GM, Jankovic J, eds.
Neurology in Clinical Practice. 5th ed. Philadelphia, PA: Butterworth Heinemann Elsevier; 2008:chapter
84

Rennie J, Boylan G. Treatment of neonatal seizures. Arch Dis Child Fetal Neonatal Ed. 2007;92:F148-
F150. DOI: 10.1136/adc.2004.068551. Available at:
http://www.ncbi.nlm.nih.gov/pmc/articles/PMC2675465/?tool=pubmed

Silverstein FS, Jensen FE. Neonatal seizures. Ann Neurol. 2007;62:112-120. DOI: 10.1002/ana.21167.
Abstract available at: http://www.ncbi.nlm.nih.gov/pubmed/17683087

Zupanc ML. Neonatal seizures. Pediatr Clin North Am. 2004;51:961-978. DOI: 10.1016/j.pcl.2004.03.002.
Abstract available at: http://www.ncbi.nlm.nih.gov/pubmed/15275983

Copyright 2012 American Academy of Pediatrics


2012 PREP SA ON CD-ROM

Question 266
A 6-year-old child presents for a health supervision visit. His growth parameters show that his
height and weight are both between the 50th and 75th percentiles for age. You note that his head
appears rather large, and when you plot his head circumference on a special curve for older children, it is
well above the 98th percentile for age and at the 50th percentile for a 15-year-old boy. When you review
th
his growth chart, you note that his head circumference has been consistently above the 98 percentile.
He has some frontal bossing and a slightly scooped nasal bridge, but otherwise completely normal
physical exam. He has been quite healthy, with normal developmental milestones, and there are no
developmental concerns at this time. When you measure the parents heads, you note that the mothers
head circumference is normal, but the head circumference of the father is greater than the 98th percentile
for an adult male.
Of the following, the MOST likely cause for the boys macrocephaly is
A. basal cell nevus syndrome
B. benign familial macrocephaly
C. Cowden syndrome
D. neurofibromatosis type 1
E. tuberous sclerosis

Copyright 2012 American Academy of Pediatrics


2012 PREP SA ON CD-ROM

Critique 266 Preferred Response: B


th
The 6-year-old boy in the vignette has a head circumference greater than the 98 percentile for
age. Although head circumference is not usually measured after 36 months of age, special growth charts
are available for older children and adults. These charts can be used to determine if the head size falls
within the normal range for age. The boy in the vignette most likely has benign familial macrocephaly
th
because his head size has been consistently above the 98 percentile for age, his other growth
parameters are normal, there are no developmental concerns, and his father also has macrocephaly.
Benign familial macrocephaly is often transmitted as an autosomal dominant trait.
Basal cell nevus syndrome (BCNS) (also known as Gorlin syndrome) is associated with
macrocephaly, odontogenic keratocysts, coarse facial features, multiple basal cell nevi beginning to
appear in the second decade of life, and skeletal anomalies that include bifid ribs and wedge-shaped
vertebral bodies.
Cowden syndrome (CS) is part of the PTEN hamartoma tumor syndrome spectrum, which
includes Bannayan-Riley-Ruvalcaba syndrome (BRRS), Proteus syndrome, and Proteus-like syndrome,
all of which are associated with the frequent finding of macrocephaly and other somatic or developmental
findings. BRRS is identified more often in infants and younger children who have intestinal
hamartomatous polyps, multiple lipomas, and hyperpigmented macules noted on the glans penis in boys.
More recently, autism and autism spectrum disorders in a child who has true macrocephaly have been
found to have a high incidence of PTEN tumor suppressor gene mutations. Therefore, a child who has
autism and macrocephaly may benefit from PTEN mutation analysis, in addition to more routine studies
such as chromosome analysis, molecular testing for fragile X syndrome, and CGH microarray studies.
Individuals who have neurofibromatosis type 1 (NF1) frequently exhibit benign macrocephaly in
infancy and early childhood. However, a 6-year-old child who has NF1 likely would have multiple caf-au-
lait macules and axillary or inguinal freckles (the most common early cutaneous findings), and if his father
also had NF1, he also would be expected to have at least a few cutaneous or subcutaneous
neurofibromas.
Tuberous sclerosis (TS) is not typically associated with macrocephaly but is associated with
hypomelanotic macules (so-called ash leaf spots), facial angiofibromas, and ungual fibromas as well as
central nervous system calcifications/tubers/subependymal nodules that can result in seizures or
intellectual disabilities.

American Board of Pediatrics Content Specification(s):


Know the differential diagnosis of macrocephaly

Suggested Reading:
Eng C. PTEN hamartoma tumor syndrome (PHTS). GeneReviews. 2009. Available at:
http://www.ncbi.nlm.nih.gov/bookshelf/br.fcgi?book=gene&part=phts

Evans DG, Farndon PA. Nevoid basal cell carcinoma syndrome. GeneReviews. 2010. Available at:
http://www.ncbi.nlm.nih.gov/books/NBK1151/

Johnson CP, Myers SM and the Council on Children with Disabilities. Identification and evaluation of
children with autism spectrum disorders. Pediatrics. 2007;120:1183-1215. DOI: 10.1542/peds.2007-2361.
Available at: http://pediatrics.aappublications.org/cgi/content/full/120/5/1183

Moeschler JB, Shevell M and the Committee on Genetics. Clinical genetic evaluation of the child with
mental retardation or developmental delays. Pediatrics. 2006;117(6): 2304-2316. DOI:
10.1542/peds.2006-1006. Available at: http://pediatrics.aappublications.org/cgi/content/full/117/6/2304

Northrup H, Au KS. Tuberous sclerosis complex. GeneReviews. 2009. Available at:


http://www.ncbi.nlm.nih.gov/books/NBK1220/

Copyright 2012 American Academy of Pediatrics


2012 PREP SA ON CD-ROM

Question 267
You are seeing a 17-year-old boy whose mother is concerned that he is not eating as well as
before and appears to be losing weight. He is upset at being brought in and states that he is just stressed
in preparing for his upcoming examinations and not sleeping as well as usual. On physical examination,
the boys blood pressure is 130/95 mm Hg, heart rate is 95 beats/min, and body mass index is 19.7. He
appears restless and has cold and sweaty palms, mildly dilated pupils, very active bowel sounds, and
hyperactive reflexes.
Of the following, the MOST likely cause for this boys weight loss is
A. anxiety disorder
B. eating disorder
C. hyperthyroidism
D. sleep disorder
E. stimulant abuse

Copyright 2012 American Academy of Pediatrics


2012 PREP SA ON CD-ROM

Critique 267 Preferred Response: E


The young man described in the vignette is exhibiting symptoms consistent with an adrenergic
toxidrome: decreased sleep and appetite, restlessness, increased blood pressure and heart rate, dilated
pupils, increased reflexes, and bowel sounds. Other presenting symptoms may include headaches,
dizziness, dysphoria, tremors, gastrointestinal symptoms, and loss of libido. Over a period of time,
stimulant use may lead to malnutrition and eventually to feelings of hostility and paranoia. Large doses
can result in a stroke, serious cardiovascular consequences, vomiting, and seizures.
Amphetamines and cocaine are two commonly abused stimulants. Amphetamines are available
in prescription form for attention-deficit/hyperactivity disorder and narcolepsy or may be illicitly
manufactured, primarily in the form of methamphetamines. Prescription medications are being
increasingly abused by those for whom they are prescribed or by peers to whom they may be diverted for
use as study aids or intoxicants (to experience feelings of euphoria and increased self-confidence,
sociability, and libido). They also may be used to suppress appetite and facilitate weight loss. As study
aids, the effects sought are increased alertness and concentration during examination times. As with the
use of other substances, the effects depend on the circumstances/setting in which the drug is used, the
individuals previous experience, the amount used, the route of administration, and the concomitant use
of other drugs. Amphetamines are usually ingested as pills but can be crushed and injected intravenously
or snorted. Repeated use may lead to addiction, especially when used intravenously or by snorting
because these forms of administration cause a rapid increase in brain dopamine concentrations.
Eventually, obsessive behaviors, irritability, aggression, and paranoia develop. Chronic or high doses are
associated with amphetamine psychosis. Withdrawal symptoms can occur with chronic use and include
disturbed sleep patterns, agitation, increased appetite, fatigue, and depression.
An eating disorder is a possibility for this boy but would involve signs of malnutrition with
constipation, bradycardia, and hypotension. Sleep disorders should not cause tachycardia and
hypertension. Patients who have hyperthyroidism experience weight loss despite increased food intake,
tremor, and warm and sweaty palms. Underlying psychiatric conditions, such as anxiety disorder, can be
difficult to discover, especially with concurrent substance use. Previous history, family history, and follow-
up after substance use is stopped are very important elements of the history to pursue.

American Board of Pediatrics Content Specification(s):


Know the major physiologic consequences (somatic consequences) attributable to amphetamines or
the method of amphetamine administration (oral, intravenous, smoking)
Know the major behavioral consequences of amphetamine use/abuse, including whether there is a
known potential for physiologic addiction

Suggested Reading:
Greenhill LL. The science of stimulant abuse. Pediatr Ann. 2006;35:552-556. Abstract available at:
http://www.ncbi.nlm.nih.gov/pubmed/16986449

Levy S, Woolf AD. Psychoactive substances of abuse used by adolescents. In: Neinstein LS, Gordon CM,
Katzman DK, Rosen DS, Woods ER, eds. Adolescent Health Care: A Practical Guide. 5th ed.
Philadelphia PA: Lippincott Williams & Wilkins, a Wolters Kluwer business; 2008:908-940

National Institute on Drug Abuse. Drugs of Abuse: Methamphetamine. Available at:


http://www.nida.nih.gov/DrugPages/Methamphetamine.html

Sanchez-Samper X, Knight JR. Drug abuse by adolescents: general considerations. Pediatr Rev.
2009;30:83-93. DOI: 10.1542/pir.30-3-83. Available at:
http://pedsinreview.aappublications.org/cgi/content/full/30/3/83

Wilens TE, Adler LA, Adams J, et al. Misuse and diversion of stimulants prescribed for ADHD: a
systematic review of the literature. J Am Acad Child Adolesc Psychiatry. 2008;47:21-31. DOI:
10.1097/chi.0b013e31815a56f1. Abstract available at: http://www.ncbi.nlm.nih.gov/pubmed/18174822

Copyright 2012 American Academy of Pediatrics


2012 PREP SA ON CD-ROM

Wilms Floet AM, Scheiner C, Grossman L. Attention-deficit/hyperactivity disorder. Pediatr Rev.


2010;31:56-69. DOI: 10.1542/pir.31-2-56. Available at:
http://pedsinreview.aappublications.org/content/31/2/56.full.pdf+html

Copyright 2012 American Academy of Pediatrics


2012 PREP SA ON CD-ROM

Question 268
You have administered a benzodiazepine and ketamine combination to a 2-year-old boy in the
emergency department to allow placement of a thoracostomy tube by another physician for a suspected
empyema. On physical examination, the boys eyes are closed and do not open to verbal commands or
stimulation. His temperature is 37.0C, heart rate is 110 beats/min, spontaneous respiratory rate is 10
breaths/min, blood pressure is 80/50 mm Hg, and while assisting him with bag-valve-mask ventilation and
head positioning, oxygen saturation is 95% by pulse oximetry. The child does not respond to verbal
commands but does reach for his side when lidocaine is injected at the planned thoracostomy tube site.
Of the following, the level of sedation achieved for this boy is BEST defined as
A. conscious sedation
B. deep sedation
C. general anesthesia
D. minimal sedation
E. moderate sedation

Copyright 2012 American Academy of Pediatrics


2012 PREP SA ON CD-ROM

Critique 268 Preferred Response: B


The goals of procedural sedation are to provide anxiolysis and analgesia and to minimize patient
movement. Sedation should be viewed as a continuum. The American Academy of Pediatrics has defined
four levels of sedation:
Minimal sedation (formerly anxiolysis) is defined as a drug-induced state in which the patient retains
the ability to respond normally to verbal commands and cardiovascular and respiratory functions are
unaffected.
Moderate sedation (formerly conscious sedation) is a drug-induced depression of consciousness in
which patients should still respond to verbal and physical stimulation. Cardiovascular status is
maintained, but the clinician must be able to recognize and respond to potential airway compromise.
Deep sedation is defined as further depression of the level of consciousness, with partial or complete
loss of protective airway reflexes and the need for assistance with airway maintenance.
General anesthesia is the deepest level of sedation and is characterized by loss of consciousness
and airway protective reflexes. Impairment of respiratory and cardiovascular function is common.
The child described in the vignette has reached a level of deep sedation, as evidenced by his
need for assistance in maintaining his airway and ventilation. He is still able to respond to painful stimuli,
despite his depressed level of consciousness.
Safe and effective procedural sedation consists of several key processes that include evaluation,
sedation, and recovery of the patient. The American Academy of Pediatrics and the American Academy
of Pediatric Dentistry guidelines for pediatric sedation detail the appropriate levels of support needed for
each level of sedation as well as unique circumstances such as sedation during magnetic resonance
imaging. Critical components for all sedations include:
Careful assessment of the patient for underlying medical or surgical conditions that would place him
or her at increased risk during the sedation.
A focused physical examination that includes assessment of the airway for abnormalities that
increase the risk of airway obstruction. Significant concerns raised during the history and physical
examination should prompt referral to clinical settings that can support a higher level of sedation and
management of potential complications.
Appropriate fasting for elective procedures. Emergent procedures require careful assessment of the
balance between aspiration risk and the risk of delaying a procedure.
Clinicians possessing a clear pharmacologic understanding of the sedative medications to be used.
Appropriate equipment and adequate personnel to perform the procedure and monitor the patient.
Appropriate airway management, vascular access equipment, medications and reversal agents, and
personnel to allow rescue of the patient.
A properly equipped and staffed recovery area to allow return to the presedation level of
consciousness before discharge from medical supervision.
Appropriate discharge instructions and follow-up evaluation.
American Board of Pediatrics Content Specification(s):
Understand the definition of procedural sedation as opposed to deep sedation and general
anesthesia
Understand what level of observation and monitoring is recommended for a patient undergoing
procedural sedation

Suggested Reading:
American Academy of Pediatrics, American Academy of Pediatric Dentistry, Cot CJ, Wilson S, the Work
Group on Sedation. Guidelines for monitoring and management of pediatric patients during and after
sedation for diagnostic and therapeutic procedures: an update. Pediatrics. 2006;118:2587-2602. DOI:
10.1542/peds.2006-2780. Available at: http://pediatrics.aappublications.org/cgi/content/full/118/6/2587

Wetzel R. Anesthesia and perioperative care. In: Kliegman RM, Stanton BF, St. Geme JW III, Schor NF,
and Behrman RE, eds. Nelson Textbook of Pediatrics. 19th ed. Philadelphia, Pa: Saunders Elsevier;
2011:359-360

Copyright 2012 American Academy of Pediatrics


2012 PREP SA ON CD-ROM

Question 269
During a health supervision visit, the mother of a 24-month-old boy reports that her son uses a
lot of words and has a great memory. However, when you call his name, he does not respond, and when
his mother asks, You want milk? he repeats, You want milk. He then starts to recite the script from a
movie he watched. In his chart, you note that results of a previous audiology evaluation were normal. On
a general developmental screening questionnaire that his mother completed, he was below the cut-off
for both typical communication and personal social development. He also scored in the risk range on the
parent-completed autism-specific screening questionnaire. You refer him to a developmental-behavioral
specialist, whose next available appointment is in 3 months.
Of the following, the MOST appropriate next step for this boy is to
A. order baseline electroencephalography
B. provide information to his mother about language stimulation activities
C. refer him for a central auditory processing evaluation
D. refer him for early intervention services
E. refer him for evaluation for an augmented communication device

Copyright 2012 American Academy of Pediatrics


2012 PREP SA ON CD-ROM

Critique 269 Preferred Response: D


Developmental screening involves the use of a brief standardized tool to identify children at risk of
developmental delay. A formal developmental screening tool should be administered at the 9-, 18-, and
either 24- or 30-month health supervision visits as well as when there are concerns regarding
development. In addition, an autism-specific screen is recommended when the child is 18 and either 24 or
30 months old. If development screening tool results raise concerns, additional evaluation is needed to
establish a diagnosis and direct treatment.
General developmental screening tools, such as the Ages and Stages Questionnaire, are
effective for identifying children at risk for language and other developmental delays. An autism-specific
screening tool, such as the Modified Checklist for Autism in Toddlers (M-CHAT), can identify a child at
risk for autism. Other questionnaires, such as the Child Behavior Checklist, may identify the presence of
attentional issues, hyperactivity, or oppositional behaviors.
The boy described in the vignette is exhibiting developmental problems suggestive of autism
spectrum disorder (ASD). He should be referred for early intervention services while waiting for the results
of a consultation with a developmental behavioral specialist because such services can begin to address
the childs deficits. If ASD is confirmed by the specialist, the interventions may be altered to be more
specific for ASD, if needed. Although offering the parent information about language stimulation can be
helpful, it will not provide the ongoing direct intervention of therapy. None of the clinical findings raise
suspicion for seizures and, therefore, there is no support for electroencephalography. A central auditory
processing evaluation is performed in children of at least 7 years of age to ascertain how they process
auditory information; it is not appropriate for a toddler. Augmented devices are indicated for those who
are unable to use expressive language, which is not a problem for this young child.

American Board of Pediatrics Content Specification(s):


Understand the role of rating scales and questionnaires for assessment of language disorders

Suggested Reading:

Ages & Stages Questionnaires . 3rd ed. Baltimore, MD: Paul H Brookes Publishing Co, Inc; 2011.

Council on Children With Disabilities, Section on Developmental Behavioral Pediatrics, Bright Futures
Steering Committee, Medical Home Initiatives for Children With Special Needs. Policy statement.
Identifying infants and young children with developmental disorders in the medical home: an algorithm for
developmental surveillance screening. Pediatrics. 2006;118:405-420. DOI: 10.1542/peds.2006-1231.
Available at: http://pediatrics.aappublications.org/cgi/content/full/118/1/405

Macias MM, Twyman, KA. Speech and language development and disorders. In: Voight RG, Macias MM,
Myers SM, eds. American Academy of Pediatrics Developmental and Behavioral Pediatrics. Elk Grove
Village, IL: American Academy of Pediatrics; 2011:201-219

National Institute on Deafness and Other Communication Disorders. Auditory Processing Disorder in
Children. Bethesda, MD: National Institutes of Health; 2004. Available at:
http://www.nidcd.nih.gov/Pages/default.aspx

Robins D, Fein D, Barton M. Instruction and Permissions for Use of the M-CHAT. 1999. Available at:
http://www2.gsu.edu/~psydlr/Diana_L._Robins,_Ph.D._files/M-CHAT_new.pdf

Copyright 2012 American Academy of Pediatrics


2012 PREP SA ON CD-ROM

Question 270
A 17-year-old boy is brought to the emergency department with a 1-day history of a temperature
of 39.6C, progressive lethargy, and a petechial rash. On physical examination, the arousable but sleepy
boy has a temperature of 40.4C, heart rate of 180 beats/min, respiratory rate of 26 breaths/min, and
blood pressure of 90/62 mm Hg. Scattered petechiae are visible, with confluent areas on the lower
extremities (Item Q270).
Of the following, the BEST choice for initial antibiotic therapy is
A. ampicillin-sulbactam
B. clindamycin and trimethoprim-sulfamethoxazole
C. doxycycline and ceftriaxone
D. vancomycin and ceftriaxone
E. vancomycin and nafcillin

Copyright 2012 American Academy of Pediatrics


2012 PREP SA ON CD-ROM

Question 270

(Courtesy of G Schutze)
Rash, as described for the boy in the vignette.

Copyright 2012 American Academy of Pediatrics


2012 PREP SA ON CD-ROM

Critique 270 Preferred Response: D


Children who appear ill and have the acute onset of fever and petechiae (or purpura), such as the
boy described in the vignette, need to be evaluated urgently and treated for severe bacterial infection.
Although classically associated with Neisseria meningitidis infection, other invasive pathogens, such as
Streptococcus pneumoniae, Staphylococcus aureus, group A Streptococcus, and enteric gram-negative
rods, need to be considered pending culture confirmation.
Of the listed combinations, vancomycin and ceftriaxone provide the most complete coverage of
the implicated organisms, including methicillin-resistant S aureus (MRSA) and highly resistant S
pneumoniae. Ampicillin-sulbactam does not afford MRSA coverage and may not cover highly resistant
pneumococci. Clindamycin and trimethoprim-sulfamethoxazole do not provide adequate meningococcal
coverage. Doxycycline adds coverage for Rocky Mountain spotted fever, but the acute history and
progression of illness are not suggestive of this diagnosis, and vancomycin is preferable for potential
severe MRSA infections.

American Board of Pediatrics Content Specification(s):


Plan initial antibiotic therapy in a child with purpura and possible sepsis

Suggested Reading:
American Academy of Pediatrics. Meningococcal infections. In: Pickering LK, Baker CJ, Kimberlin DW,
Long SS, eds. Red Book: 2009 Report of the Committee on Infectious Diseases. 28th ed. Elk Grove
Village, IL: American Academy of Pediatrics; 2009:455-463

Mandl KD, Stack AM, Fleisher GR. Incidence of bacteremia in infants and children with fever and
petechiae. J Pediatr. 1997;131:398404. Abstract available at:
http://www.ncbi.nlm.nih.gov/pubmed/9329416

Wynn J, Cornell TT, Wong HR, Shanley TP, Wheeler DS. The host response to sepsis and
developmental impact. Pediatrics. 2010;125:10311041. DOI: 10.1542/peds.2009-3301. Available at:
http://pediatrics.aappublications.org/cgi/content/full/125/5/1031

Copyright 2012 American Academy of Pediatrics


2012 PREP SA ON CD-ROM

Question 271
A 10-year-old girl presents with a history of chronic sinusitis unresponsive to appropriate nasal
hygiene and multiple courses of appropriate antibiotic therapy. She complains of ongoing nasal
congestion, purulent nasal discharge, headache, and facial pain. Evaluations for allergy and
immunodeficiency have yielded negative results. On physical examination, she has a temperature of
38.0C, heart rate of 90 beats/min, and respiratory rate of 22 breaths/min. Her weight is 33 kg (50th
percentile) and height is 138 cm (50th percentile). You note purulent nasal discharge and tenderness to
palpation of the maxillary sinuses. Oropharyngeal examination reveals moderately large tonsils and foul-
smelling breath. Flexible endoscopy of the nasopharynx documents enlarged nasal turbinates without
polyps and bilateral adenoidal hypertrophy.
Of the following, the next BEST step to consider in this patients management, before performing
endoscopic sinus surgery, is
A. adenoidectomy
B. decongestants
C. intranasal steroids
D. repeat immune evaluation
E. tonsillectomy

Copyright 2012 American Academy of Pediatrics


2012 PREP SA ON CD-ROM

Critique 271 Preferred Response: A


The patient described in the vignette has chronic refractory sinusitis and adenoidal hypertrophy
and may benefit from adenoidectomy. The efficacy of adenoidectomy in children who have chronic
sinusitis that is refractory to medical management is controversial, but some studies have shown
favorable results in children in whom evaluations for underlying diseases (eg, immune, allergic, ciliary)
have yielded negative results. Some experts recommend adenoidectomy before considering endoscopic
sinus surgery. Extreme obstruction of the naso/oropharyngeal airways by the adenoids (obstructive sleep
apnea) is an absolute indication for adenoidectomy in children. Other conditional indications for
adenoidectomy include moderate, symptomatic nasal obstruction and recurrent acute otitis media or
chronic otitis media with effusion in children in whom tympanostomy tubes have failed. Complications of
adenoidectomy include hemorrhage, pain, emotional upset, adverse reactions to anesthesia,
bronchopulmonary infection, upper airway obstruction, and velopharyngeal insufficiency. Postoperative
bleeding within 24 hours following the procedure or at 7 to 10 days after surgery is the most common
complication. Serious hemorrhage requiring transfusion or further surgery is uncommon, occurring in 2%
to 4% of patients.
Neither intranasal steroids nor decongestants have been proven beneficial in the treatment of
chronic sinusitis, although they continue to be prescribed frequently. A repeat evaluation of the patients
immune system would be costly and unnecessary because test results would not change over time.
Tonsillectomy is indicated for patients who have extreme obstruction of the nasopharynx or oropharynx
(sleep apnea), obstruction that interferes with swallowing, tonsillar tumor, and uncontrolled tonsillar
hemorrhage. Among the conditional indications are recurrent throat infections (including tonsillitis),
chronic tonsillitis despite antimicrobial therapy, two or more episodes of peritonsillar abscess, chronic
group A streptococcal throat carriage with a contact who had rheumatic fever, severe (refractory)
halitosis, and PFAPA (periodic fever, aphthous stomatitis, pharyngitis, and adenitis) syndrome.
Complications of tonsillectomy are similar to those of adenoidectomy. Throat pain and otalgia
following tonsillectomy is more severe and longer lasting than that which occurs following adenoidectomy.
Rare complications include nasopharyngeal stenosis, atlantoaxial subluxation, persistent torticollis,
subcutaneous emphysema, and carotid artery dissection.

American Board of Pediatrics Content Specification(s):


Know the indications for a tonsillectomy
Know the complications of a tonsillectomy and adenoidectomy
Know the indications for an adenoidectomy

Suggested Reading:
Baum ED. Tonsillectomy and adenoidectomy and myringotomy with tube insertion. Pediatr Rev.
2010;31:417-426. DOI: 10.1542/pir.31-10-417. Available at:
http://pedsinreview.aappublications.org/cgi/content/full/31/10/417

Taylor A, Adam HM. In brief: sinusitis. Pediatr Rev. 2006;27:395-397. DOI: 10.1542/pir.27-10-395.
Available at: http://pedsinreview.aappublications.org/cgi/content/full/27/10/395

Copyright 2012 American Academy of Pediatrics


2012 PREP SA ON CD-ROM

Question 272
A 6-year-old girl presents for a health supervision visit that was scheduled as a follow-up
appointment after she had an elevated blood pressure at an urgent care facility during an evaluation for
abdominal pain. Her abdominal pain has resolved. Her mother recalls the blood pressure in the urgent
care center as 135/90 mm Hg. The girl has had two urinary tract infections with fever in the past, and her
father had hypertension diagnosed at age 45 years. On physical examination, the girls temperature is
37.3C, heart rate is 90 beats/min, respiratory rate is 20 breaths/min, and blood pressure is 146/86 mm
Hg. A repeat blood pressure reading is 142/88 mm Hg. The four limb blood pressures are: 142/88 mm Hg
in the right arm, 144/84 mm Hg in the left arm, 156/100 mm Hg in the right leg, and 160/96 mm Hg in the
left leg. You find no cardiac murmurs, abdominal bruits, or edema. Femoral pulses are 2+ and
symmetrical bilaterally. Renal ultrasonography shows the left kidney to be 8.5 cm with normal
corticomedullary differentiation and the right kidney to be 5.5 cm with increased echogenicity.
Of the following, the MOST likely cause for this patients elevated blood pressure is
A. coarctation of the aorta
B. essential hypertension
C. renal artery stenosis
D. renal hypoplasia/dysplasia
E. renal scarring from prior pyelonephritis

Copyright 2012 American Academy of Pediatrics


2012 PREP SA ON CD-ROM

Critique 272 Preferred Response: E


The 6-year-old girl described in the vignette presents with hypertension that was initially observed
in an urgent care facility and confirmed on two measurements in the clinic. Her past history is notable for
two febrile urinary tract infections (UTIs), and her physical examination findings are essentially
unremarkable. Of note, femoral pulses are present and blood pressures are higher in the legs than in the
arms, observations that make coarctation of the aorta highly unlikely. Essential hypertension is a
diagnosis of exclusion, often presenting with mild hypertension in adolescent patients who are overweight
and have a positive family history of hypertension. The family history of hypertension in this patients
father is likely insignificant, based on his age of onset. Renal artery stenosis is a rare cause of
hypertension in the pediatric patient and may have associated abdominal bruits on examination and a
small kidney on ultrasonography. Although this girl does have a small kidney on ultrasonography, the
increased echogenicity suggests renal parenchymal injury, which would not be expected in renal artery
stenosis. Renal hypoplasia/dysplasia can be associated with an isolated small kidney, but hypertension is
uncommon in this setting. The most likely cause of the hypertension in this child is renal scarring from
prior pyelonephritis.
Vesicoureteral reflux (VUR) is discovered in 30% to 50% of children evaluated for febrile UTI.
VUR can cause renal parenchymal damage in the form of renal scarring, which is known as reflux
nephropathy. A recent study revealed renal scarring in infants who had VUR that increased with
increasing severity of reflux: 7%, 21%, and 43% for grades III, IV, and V reflux, respectively. Hypertension
can occur in up to 20% of patients who have reflux nephropathy and may take up to 8 years to develop.
Conversely, 30% to 40% of pediatric patients who have secondary hypertension have renal scars from
VUR. This is one of the reasons why febrile UTIs are evaluated aggressively with renal imaging in the
hopes of uncovering VUR at an early age. In addition to increased risk for hypertension, reflux
nephropathy also can cause end-stage renal disease later in life. Prospective studies are ongoing to
evaluate the effectiveness of prophylactic antibiotics in the prevention of renal scars. The current
recommendations are to use prophylactic antibiotics in children who have a febrile UTI and VUR. Patients
usually respond well to angiotensin-converting enzyme inhibitors for the treatment of hypertension
associated with renal scarring. Such therapy should be used with caution in female patients of
childbearing age due to risks of teratogenicity.

American Board of Pediatrics Content Specification(s):


Recognize the association between vesicoureteral reflux and hypertension

Suggested Reading:
Farnham SB, Adams MC, Brock JW 3rd, Pope JC 4th. Pediatric urological causes of hypertension. J Urol.
2005;173:697-704. DOI: 10.1097/01.ju.0000153713.46735.98. Abstract available at:
http://www.ncbi.nlm.nih.gov/pubmed/15711246

Feld LG, Corey H. Hypertension in childhood. Pediatr Rev. 2007;28:283-298. DOI: 10.1542/pir.28-8-283.
Available at: http://pedsinreview.aappublications.org/cgi/content/full/28/8/283

Feld LG, Mattoo TK. Urinary tract infections and vesicoureteral reflux in infants and children. Pediatr Rev.
2010;31:451-463. DOI: 10.1542/pir.31-11-451. Available at:
http://pedsinreview.aappublications.org/cgi/content/full/31/11/451

McNiece KL, Portman RJ. Hypertension: epidemiology and evaluation. In: Kher KK, Schnaper HW, Makker
SP, eds. Clinical Pediatric Nephrology. 2nd ed. London, England: Informa Healthcare; 2007:461-480

National High Blood Pressure Education Program Working Group on High Blood Pressure in Children
and Adolescents. The fourth report on the diagnosis, evaluation, and treatment of high blood pressure in
children and adolescents. Pediatrics. 2004;114(2 suppl):555-576. Available at:
http://pediatrics.aappublications.org/content/114/Supplement_2/555.full.pdf+html

Copyright 2012 American Academy of Pediatrics


2012 PREP SA ON CD-ROM

Simoes e Silva AC, Silva JM, Diniz JS, et al. Risk of hypertension in primary vesicoureteral reflux. Pediatr
Nephrol. 2007;22:459-462. DOI: 10.1007/s00467-006-0349-2. Abstract available at:
http://www.ncbi.nlm.nih.gov/pubmed/17143629

Copyright 2012 American Academy of Pediatrics


2012 PREP SA ON CD-ROM

Question 273
A 14-year-old girl was brought to the emergency department after she complained of difficulty
breathing. She had been helping her family pack their household items for an upcoming move. Every time
she was near a cardboard box she noticed a funny smell and experienced difficulty breathing. Her vital
signs, including room air pulse oximetry and respiratory rate, are normal, but she points to her neck and
states that it is hard to get air in. Her oropharynx is without edema, and her lungs are clear to
auscultation without crackles or wheezing. After observation for 15 minutes, her symptoms resolve and
the girl is discharged.
Of the following, the MOST likely cause for this girls symptoms is
A. asthma exacerbation
B. gastroesophageal reflux
C. hereditary angioedema
D. nonallergic rhinitis
E. vocal cord dysfunction

Copyright 2012 American Academy of Pediatrics


2012 PREP SA ON CD-ROM

Critique 273 Preferred Response: E


The teenager described in the vignette has signs and symptoms suggestive of vocal cord
dysfunction (VCD), a condition that can both mimic and coexist with asthma. In contrast with an asthma
exacerbation, the key features that make her symptoms consistent with VCD include her normal room air
pulse oximetry reading, clear lungs, difficulty with inspiration instead of expiration, and rapid improvement
without specific intervention. The pathognomonic sign for VCD on examination is paradoxic adduction of
the vocal cords during inspiration.
Nonallergic rhinitis is common in children and adults, with symptoms occurring after exposure to
dust, dirt, or fumes. However, nonallergic rhinitis results in ocular and nasal symptoms rather than the
respiratory symptoms described in the vignette. Gastroesophageal reflux (GER) has been shown to be a
potential trigger for an asthma exacerbation, chronic cough, and postnasal drainage. However, the most
common presentation of GER in adolescents and adults is dysphagia, heartburn, or emesis. Hereditary
angioedema (HAE) is an autosomal dominant disorder characterized by severe angioedema and
abdominal pain. Symptoms often present in adolescence or early adulthood, with unexplained extremity
swelling after minor trauma. Angioedema symptoms can be life-threatening and may require hours to
days to resolve. The lack of observed angioedema and prompt resolution of symptoms in this girl make
this diagnosis unlikely.
The approach to evaluating dyspnea requires a detailed history, with specific attention to triggers,
frequency, and response to interventions. Almost all patients who have underlying asthma experience
increased symptoms with vigorous exercise. In addition, VCD may mimic or coexist with asthma. Testing
procedures such as video strobe laryngoscopy, methacholine challenge, and full lung volumes that
include diffusing capacity of the lung for carbon monoxide may help differentiate various respiratory
disorders such as interstitial lung disease, asthma, and VCD that present with dyspnea or exercise
intolerance.

American Board of Pediatrics Content Specification(s):


Know that exercise intolerance may be a presenting symptom of chronic lung diseases (eg, asthma,
interstitial lung disease) or vocal cord dysfunction

Suggested Reading:
de Groot EP. Breathing abnormalities in children with breathlessness. Paediatr Respir Rev. 2011;12:83-
87. DOI: 10.1016/j.prrv.2010.09.003. Abstract available at:
http://www.ncbi.nlm.nih.gov/pubmed/21172680

Morris MJ, Christopher KL. Diagnostic criteria for the classification of vocal cord dysfunction. Chest.
2010;138:1213-1223. Abstract available at: http://www.ncbi.nlm.nih.gov/pubmed/21051397

Copyright 2012 American Academy of Pediatrics


2012 PREP SA ON CD-ROM

Question 274
A 15-year-old boy is brought to the emergency department by emergency medical services
personnel after crashing his all-terrain vehicle into a tree. The paramedics found him under the vehicle,
responding appropriately to commands. Following extrication, they immobilized his cervical spine,
administered 100% oxygen by a nonrebreather mask, and inserted two intravenous catheters. On arrival
at the emergency department, his temperature is 36.5C, heart rate is 140 beats/min, respiratory rate is
42 breaths/min, blood pressure is 80/60 mm Hg, and oxygen saturation is 86%. He has a Glasgow Coma
Scale score of 14, bilateral femur deformities, marked right-sided chest wall tenderness, and crepitus. In
addition, when he inhales, you note that his chest wall on the right appears to sink in.
Of the following, the MOST appropriate next step is to
A. administer morphine for analgesia
B. obtain a chest radiograph with dedicated views of the ribs
C. obtain computed tomography scan of the chest
D. perform intubation and mechanical ventilation
E. perform needle decompression of the right chest

Copyright 2012 American Academy of Pediatrics


2012 PREP SA ON CD-ROM

Critique 274 Preferred Response: D


The boy described in the vignette has flail chest, a potentially life-threatening chest injury that
requires endotracheal intubation and mechanical ventilation to relieve his respiratory distress and re-
establish adequate ventilation and oxygenation. This intervention should be performed as part of the
primary survey so that his condition does not deteriorate further. Analgesia is indicated, as is a chest
computed tomography scan to assess for lung injury, but only after the patients vital functions are
stabilized. Chest radiography with dedicated views of the ribs is not helpful, and because the patient does
not have clinical evidence of a tension pneumothorax, needle decompression is not indicated.
Flail chest occurs when multiple ribs are fractured, leading to instability of the chest wall. It is an
uncommon injury in children, with only 1% to 2% of pediatric chest injuries in two large studies resulting in
flail chest, largely because of the relative flexibility of the pediatric ribs. This injury leads to respiratory
compromise through a number of mechanisms. The paradoxic movement of the flail segment (on
inspiration the chest wall retracts) impairs effective respiratory mechanics, and the pain associated with
the fractured ribs can cause the patient to decrease inspiratory volume to splint the chest, which leads
to atelectasis (a video of flail chest may be viewed by following this link:
http://www.nejm.org/doi/full/10.1056/NEJMicm0904437). Further, flail chest often is associated with
pulmonary contusions, lung injury that impairs gas exchange.
The treatment of flail chest is supportive and includes adequate analgesia and assisted
ventilation. Rib splinting or application of a binder around the chest is not helpful and can lead to further
respiratory compromise.

American Board of Pediatrics Content Specification(s):


Identify a flail chest

Suggested Reading:
Alexander P. Images in clinical medicine: flail chest. N Engl J Med. 2010;363:e35. Available at:
http://www.nejm.org/doi/full/10.1056/NEJMicm0904437

Avarello JT, Cantor RM. Pediatric major trauma: an approach to evaluation and management. Emerg
Med Clin North Am. 2007;25:803836. DOI: 10.1016/j.emc.2007.06.013. Abstract available at:
http://www.ncbi.nlm.nih.gov/pubmed/17826219

Kadish H. Chest wall injuries in children. UpToDate Online 18.3. 2010. Available for subscription at:
http://www.uptodate.com/online/content/topic.do?topicKey=ped_trau/2856

Mendez DR. Initial evaluation and stabilization of children with thoracic trauma. UpToDate Online 18.3.
2010. Available for subscription at:
http://www.uptodate.com/online/content/topic.do?topicKey=ped_trau/7695

Copyright 2012 American Academy of Pediatrics

Potrebbero piacerti anche